Вы находитесь на странице: 1из 502

Contents

Preface
ix
Acknowledgments
xi
xiii
To the Reader

I.
II.
III.
IV.
V.
VI.
VII.
VIII.
IX.
X.
XI.
XII.
XIII.

Cellular homeostasis
Resting and action potentials
7
18
Synaptic transmission
Muscle contraction
28
35
General sensory mechanisms
Cutaneous sensation
40
Audition
44
Vision
51
61
Taste
Olfaction
62
64
Spinal reflexes
The motor control system
69
78
Sleep and consciousness
Study questions
82
88
Answers and explanations

93

Cardiovascular Physiology
I.
II.
III.
IV.
V.
VI.
VII.
VIII.
IX.
X.
XI.

Neurophysiology

Vessels
93
Heart 99
107
Electrocardiography
Cardiac rate, rhythm, and conduction disturbances
118
The heart as a pump
Cardiodynamics
121
Hemodynamics
124
127
Cardiac output and venous return
131
Control of the cardiovascular system
Special circulatory beds
133
Responses to stress
135
140
Study questions
144
Answers and explanations

113

Respiratory Physiology
I.
II.
III.
IV.

Introduction
149
Respiratory mechanics
149
Gas exchange between the atmosphere and the body
Gas transport
166

149
163

V.
VI.
VII.
VIII.
IX.

220

257

Acid production and elimination


257
The hydrogen ion and pH
258
Body buffer systems
263
Generation and elimination of H+
267
Respiratory regulation of acid-base balance
269
Renal regulation of acid-base balance
270
Primary and secondary acid-base abnormalities
276
Compensatory mechanisms for primary acid-base abnormalities
Study questions
289
Answers and explanations
295

Digestive system: overview


303
The oral cavity, pharynx, and esophagus
The stomach
307
The small intestine
315
326
The colon
Study questions
328
Answers and explanations
331

303

Endocrine Physiology
I.
II.
III.
IV.
V.
VI.
VII.
VIII.

285

303

Gastrointestinal Physiology
I.
II.
III.
IV.
V.

Introduction
199
Body fluid compartments
202
Overview of renal tubular function
209
Renal clearance
21 2
Glomerular filtration and GFR
217
Renal tubular transport: reabsorption and secretion
Renal concentration and dilution of urine
227
Antidiuretic hormone (ADH)
234
Aldosterone
235
Atrial natriuretic factor (ANF)
242
Study questions
243
Answers and explanations
249

Acid-Base Physiology
I.
II.
III.
IV.
V.
VI.
VII.
VIII.

199

Renal Physiology
I.
II.
III.
IV.
V.
VI.
VII.
VIII.
IX.
X.

Pulmonary circulation
170
The distribution of ventilation
172
Ventilation:perfusion ratio
174
Respiratory control
176
Hypoxia
181
Study questions
186
Answers and explanations
192

335

General features of hormones


335
Types of hormone action
340
Basic concepts of endocrine control
343
Endocrine cells of the gastroenteropancreatic (GEP) system
Endogenous opioid peptides
348
Pituitary gland
352
Adrenal medulla
357
Adrenal cortex
363

348

IX.
X.
XI.
XII.
XIII.
XIV.

Testis
370
Ovary
377
Endocrine placenta
386
Endocrine pancreas
391
Thyroid gland
396
Parathyroid hormone, calcitonin, and vitamin D
Study questions
407
Answers and explanations
41 3

401

Comprehensive Examination

421

Index

475

Preface

Since its publication in 1984, NMS Physiology has garnered considerable response from
medical students around the world. Based on these written and oral commentaries, the authors
have revised the text to conform better to the needs of students preparing for course examinations
and standardized medical or other health science examinations. Accordingly, this new edition
of NMS Physiology contains a well-defined fund of knowledge the authors consider essential for
medical practitioners. Although this book does not focus specifically on pathology, it does provide
insight into the pathophysiologic aspects of disease by comparing normal processes in the body
with the abnormal.
The outline format has allowed the authors to emphasize the relative importance of the facts
presented and to provide students with a source of fundamental information about physiology.
Such a core guide to the discipline should prove especially valuable now that medical students
are expected to assume more and more responsibility for their own learning.
John Bullock
Joseph Boyle, III
Michael B. Wang

Acknowledgments

We collectively wish to express our thanks to the professional staff at the Harwal Publishing
Company, particularly to Matthew Harris (Medical Editor), Debra Dreger (Managing Editor), and
Martha Gay (Project Editor) for their personal support, perseverance, understanding, and expertise. We also recognize the special contribution of Weislawa Langenfeld as medical illustrator.
Our greatest measure of gratitude goes to Debra Dreger, who has earned our utmost respect for
her energy and commitment to the NMS series.
The authors
To Barbara, who, through her love and willing sacrifice, encouraged me to complete two
graduate degrees in physiology and biochemistry. Also to Laura, John, and Katherine-all important well-springs of inspiration. Not to be forgotten are the many medical and dental students
together with the foreign medical graduates who have become friends and colleagues over the
years.
John Bullock
To Patti, for her love and support over many years. To many colleagues who have made the
study of physiology an exciting and stimulating career. And, to the many students who have
asked the questions which have enlightened me.
Joseph Boyle, III
My thanks to family, friends, colleagues, and students for Jheir encouragement and support
over many years.
Michael B. Wang

To the Reader

Since 1984, the National Medical Series for Independent Study has been helping medical
students meet the challenge of education and clinical training. In this climate of burgeoning
knowledge and complex clinical issues, a medical career is more demanding than ever. Increasingly, medical training must prepare physicians to seek and synthesize necessary information and
to apply that information successfully.
The National Medical Series is designed to provide a logical framework for organizing, learning,
reviewing, and applying the conceptual and factual information covered in basic and clinical
studies. Each book includes a concise but comprehensive outline of the essential content of a
discipline, with up to 50 study questions. The combination of distilled, outlined text and tools for
self-evaluation allows easy retrieval and enhanced comprehension of salient information. Each
question is accompanied by the correct answer, a paragraph-length explanation, and specific
reference to the text where the topic is discussed. Study questions that follow each chapter use
current National Board formats to reinforce the chapter content. Study questions appearing at
the end of the text in the Comprehensive Exam vary in format depending on the book; the
unifying goal of this exam, however, is to challenge the student to synthesize and expand on
information presented throughout the book. Wherever possible, Comprehensive Exam questions
are presented in the context of a clinical case or scenario intended to simulate real-life application
of medical knowledge.
Each book in the National Medical Series is constantly being updated and revised to remain
current with the discipline and with subtle changes in educational philosophy. The authors and
editors devote considerable time and effort to ensuring that the information required by all
medical school curricula is included and presented in the most logical, comprehensible manner.
Strict editorial attention to accuracy, organization, and consistency also is maintained. Further
shaping of the series occurs in response to biannual discussions held with a panel of medical
student advisors drawn from schools throughout the United States. At these meetings, the editorial
staff considers the complicated needs of medical students to learn how the National Medical
Series can better serve them. In this regard, the staff at Harwal Publishing welcomes all comments
and suggestions. Let us hear from you.

1
Neurophysiology
Michael B. Wang

I. CELLULAR HOMEOSTASIS
A. Homeostasis is the process by which an organism maintains the composition of the extracellular
fluid (ECF) and intracellular fluid (lCF) in a steady-state condition. The composition of the ECF
and the ICF differ from each other (Table 1-1; see also Table 4-3A).

1. Extracellular fluid consists of the blood plasma and interstitial fluid. The composition of the
ECF is maintained by the cardiovascular, pulmonary, renal, gastrointestinal, endocrine, and
nervous systems acting in a coordinated fashion.

2. Intracellular fluid. The composition of the ICF is maintained by the cell membrane, which
mediates the transport of material between the ICF and ECF by diffusion, osmosis, and active
transport.

B. Cell membrane. All animal cells are enveloped by a cell membrane that is composed of a variety
of lipids and proteins.

1. Lipids. The lipids (i.e., primarily phospholipids, cholesterol, and glycolipids) are formed into a
bilayer (Figure 1-1) and are amphipathic; that is, they have a hydrophilic polar region at one
end of the molecule and a hydrophobic hydrocarbon tail at the other.
a. The hydrophilic ends of lipid molecules, which contain the phosphate group, line up facing
the ICF and ECF.
b. The hydrophobic ends of the molecules face each other in the interior of the bilayer.

2. Proteins. According to the fluid mosaic model of membrane structure, membrane proteins
insert into (and can float within) the bilayer and are anchored by covalent bonds. Some proteins
span the entire bilayer; others are contained within the intracellular or extracellular half of the
bilayer. Membrane proteins serve several functions (see Figure 1-1).

Table 1-1. Major Ionic Components of the Intracellular and Extracellular Fluids
Intracellular Concentration
Ion
Na+
K+
Ca2+
Mg2+

CIHC0 3 HPO/50/Proteins-, amino


acids -, urea, etc.

(mOsm/L)

(mEq/l)

Extracellular Concentration
(mOsm/L)

(mEq/L)
140
4
4
2
120
24
4
1

15
135
10- 4
20
4
10
10
2

15
135
2 x 10- 4
40
4
10
20
4

140
4
2
1
120
24
2
0.5

99

152

0.5

Note-The osmolarity of the intracellular fluid (lCF) is the same as that of the extracellular fluid (ECF). Also, the
milliequivalents of cations and anions are equal in the ICF as well as in the ECF.

Receptor
site

Figure 1-1. Diagram indicating some of the functions performed by proteins within the lipid bilayer of biologic
membranes.

a. Some proteins (transmembrane proteins) have active sites on both sides of the bilayer.
These proteins form:
(1) Channels, through which small, water-soluble substances can diffuse
(2) Carriers, which are used to transport material across the bilayer
b. Other proteins have an active site on only one side of the membrane.
(1) Receptor sites for antibodies, hormones, neurotransmitters, and pharmacologic agents
usually are contained within proteins located on the outer surface of cell membranes.
(2) Enzymes used to activate or inactivate various metabolic intermediates usually are
contained within proteins found on the inner surface of cell membranes.

C. Translocation across the cell membrane. Ions, nutrients, and waste products of metabolism
are transferred across the cell membrane by simple diffusion and a variety of carrier-mediated
processes.
1. Passive transport processes do not require energy. They are downhill processes.
a. Simple diffusion occurs because all particles in solution are in constant motion.
(1) Although a single particle moves in an unpredictable (random) direction, it is more likely
that the particle will move from an area where it is highly concentrated to an area where
its concentration is low than it is for the particle to move in the opposite direction. Thus,
a particle moves down its concentration gradient by diffusion.
(2) Net movement ceases when the concentration of the particle is equal everywhere within
the solution (diffusional equilibrium). Although random movement of the particles does
not cease, the concentration remains the same.
(3) Fick's law of diffusion describes the rate of diffusion through a membrane, which increases as the concentration gradient increases.
flux

= -

DxA
--d- (C;n - C out )

where flux = the amount of material (mmol) moved per unit of time; D = the diffusion
coefficient (cm 2 /sec), which is characteristic of the material being moved and the
membrane through which it is being moved; A = the area (cm 2 ) of the membrane; d =
the diffusion distance, or thickness of the membrane (cm); and C;n and C out = the
concentrations of the material (mmol/L or mmol/1 000 cm 3 ) on the inside and outside
of the membrane, respectively. The negative sign indicates that the material is moving
down its concentration gradient.
(4) Permeability. Fick's law can be simplified when biologic membranes are considered,
because the thickness of the membrane always is about 10- 6 cm. Dividing D by 10- 6
yields the permeability coefficient (P) of the membrane, and Fick's law becomes
flux

= -

P x A (C;n - C out )

Note that P = D/d and that the units of permeability are cm/sec.
(5) Diffusion of lipid-soluble materials occurs through the lipid bilayer of the cell membrane. The diffusion coefficient of lipid-soluble materials is proportional to their lipid
solubility, which often is expressed as the oil-water partition coefficient.
(6) Diffusion of water-soluble materials occurs through the aqueous channels formed
by transmembrane proteins. The diffusion coefficient of water-soluble materials is

proportional to their molecular size. Particles too large to fit through the aqueous
channels (particles with diameters greater than 0.8 nm) cannot cross the cell membrane
by simple diffusion.
b. Facilitated diffusion requires a carrier. Some particles (e.g., glucose) are too large to diffuse
through membrane channels. However, they can be moved across the membrane by a
carrier-mediated process that does not require energy (Figure 1-2A).
(1) The particle binds to a carrier on one side of the membrane and, while bound to the
carrier, is transported through the membrane. The particle dissociates from the carrier
when it reaches the other side of the membrane. Because the particle binds to the
carrier only if it is highly concentrated, the particle can only move down its concentration gradient by this process.
(2) The rate of diffusion increases as the concentration gradient increases until all of the
carrier sites are filled. At this point, the rate of diffusion can no longer increase with
increasing particle concentration. This is called saturation, or Michaelis-Menten, kinetics (see Figure 1-2B).
2. Active transport processes require energy. They are uphill processes.
a. Primary active transport. Active transport processes using adenosine triphosphate (ATP)
are called direct energy utilizing, or primary active transport, processes.
(1) The most common of these active transport systems is the sodium-potassium (Na+ -K+)
pump, or Na+ -K+ -ATPase, which uses the membrane-bound ATPase enzyme as a
carrier molecule. The Na+ -K+ pump is responsible for maintaining the high K+ and low
Na+ concentrations in the ICF (Figure 1-3). The operation of the pump can be divided
into three steps:
(a) Three ions of Na+ bind to the carrier on the inside of the cell
(b) Two ions of K+ bind to the carrier on the outside of the cell
(c) The carrier uses the energy obtained from one molecule of ATP to move two ions
of K+ into thp. cell and three ions of Na+ out of the cell
(d) Inhibition of the carrier will occur if the intracellular ATP or Na + concentration
or the extracellular K+ concentration is too low. The carrier also can be inhibited
by several therapeutic agents, such as digitalis.
(2) Other carriers with a direct energy source are available to transport a variety of ions,
such as chloride (CI-), calcium (Ca2+), and hydrogen (H+).
b. Secondary active transport. Carrier-mediated active transport systems that use the energy
stored in the Na+ concentration gradient are called indirect energy utilizing, or secondary
active transport, processes. The system can operate only if the extracellular Na + concentration is higher than the intracellular Na+ concentration. Energy is required to establish the

Glucose
Binding site

mmQOm

B
Simple diffusion
X
::J

u:::

Facilitated
diffusion

Concentration gradient

Figure 1-2. (A) The transmembrane protein responsible


for the facilitated transport of glucose undergoes a conformational change when glucose binds to the transporter on
one side of the membrane. The conformational change
allows glucose to diffuse across the membrane down its
concentration gradient. (8) The rate of transport reaches
a maximum when all binding sites on the transporter are
filled.

\.0

Intracellular
space

III

In

111

o~

K+

Extracellular
space

mDUl
00

K+

Na+ 0

00

Na+
Figure 1-3. (A) The energy contained in the high-energy phosphate bond is used to transport three ions of Na+ out
of the cell. (8) A second conformational change occurs after two K+ ions bind to the carrier, transporting K+ into the
cell and (0 causing the phosphate to dissociate from the carrier.

Na+ concentration gradient, and it is this energy that is used indirectly in the active transport
process.
(1) Transport of glucose and amino acids. Glucose and amino acids are transported
against their concentration gradients by a secondary active transport process (Figure

1-4l.
(a) Na+ binds to the carrier on the outside of the cell where the Na+ concentration is

high. After Na+ binds to the carrier, the carrier's affinity for glucose increases,
enabling it to bind glucose.
(b) When Na+ and glucose are bound to the carrier, the carrier undergoes a conformational change, during which both glucose and Na+ are exposed to the inside of the
cell.
(c) Because of the low intracellular Na + concentration, Na + dissociates from the carrier,
reducing the carrier's affinity for glucose. This allows glucose to dissociate from the
carrier despite the relatively high intracellular glucose concentration.
(d) Na + -dependent secondary active transport of glucose and amino acids is used
primarily for the transport of these nutrients by epithelial cells of the nephron and
the i ntesti ne.
(2) Transport of Ca H Calcium is transported out of the cell against its concentration
gradient by a secondary active transport process.
(a) Three Na+ ions are transported into the cell for each Ca2+ ion transported out of
the cell.
A

I :1 ,/

Na~!OFl
EC/"
Glucose

ICF

E""-alE:'~

ICF

ECF

Na+
ICF
Glucose

I~:;==--I ~

Figure 1-4. Glucose is transported through the membrane by active transport that uses an indirect energy source.
arrows, the affinity of the carrier for glucose is low in the absence of Na+. (8)
However, after Na+ binds to the carrier, the affinity for glucose is increased. Glucose and Na+ diffuse through the
membrane together. (0 Once inside the cell, Na+ is removed from the carrier, the affinity for glucose. is reduced,
and glucose dissociates from the carrier.

(A) As indica,ted by the length of the

(b) The Na+ -Ca2+ exchanger is very important for the regulation of intracellular Ca2+
and, thus, contractile force in cardiac cells (see Ch 2 II D 2 a).
(3) Other carrier-mediated secondary active transport processes, such as those that
pump synaptic transmitter substances into nerve terminals (see III C 1) or that secrete H +
ions into the proximal tubule (see Ch 5 VI B 1), are vital to normal cellular homeostasis.
(4) Carrier types
(a) Uniporters are carriers that transport a single particle in one direction, such as the
facilitated diffusion of glucose.
(b) Symporters transport two particles in the same direction, such as the secondary
active transport of glucose.
(c) Antiporters transport molecules in opposite directions, such as the Na+ -Ca2+ and
Na + -H + exchangers.
3. Filtration and osmosis. In filtration and osmosis, water is forced to flow across a biologic
membrane by hydrostatic pressure. In the kidney, for example, the capillary blood pressure
created by the heart causes plasma water to be filtered out of the glomerular capillaries and
into the nephron. In the peritubular capillaries, the osmotic pressure created by the high
concentration of plasma proteins causes water to flow from the interstitial space surrounding
the nephron into the capillaries.
a. Osmotic pressure is produced when a semipermeable membrane (i.e., a membrane
permeable only to water) separates two solutions that have different concentrations of
particles.
b. Although it is not known how the particle concentration gradient creates an osmotic pressure, the pressure across the membrane can be calculated using the van't Hoff equation
TT = ~c x R x T
where TT = the osmotic pressure (mm Hg), ~c = the difference in the concentration of
particles between the two solutions (mOsm/L), R = the natural gas constant (62 mm Hg x
L/mmol x 0 K), and T = absolute temperature (0 K)
c. Osmotic pressure is a colligative property of solutions; that is, it is related to the number
of particles dissolved in the solution.
(1) Each mmol/ L of a particle produces an osmotic pressure of about 19 mm Hg.
(a) For example, a 1 mmol/L glucose solution produces an osmotic pressure of 19 mm
Hg, and a 1 mmol/L sodium chloride (NaCi) solution produces an osmotic pressure
of 38 mm Hg (because two particles of solute exist for each molecule of NaCi).
(b) Therefore, the osmotic concentration of a substance in solution reflects the number
of particles dissolved and not the number of molecules. Thus, a 1 mmol/L glucose
solution has an osmotic concentration of 1 mOsm/L, and a 1 mmol/L NaCI solution
has an osmotic concentration of 2 mOsm/L.
(2) Osmolarity and osmolality. As a unit of concentration, osmolarity is a measure of the
moles of solute/L of solution, whereas osmolality is a measure of moles of solute/kg
of water. If the volume of solute is small (as it is for most biologic solutions), the
osmolarity and osmolality of the solution will be approximately equal and, thus, the two
terms can be used interchangeably.
d. Change in cell volume due to osmotic pressure differences. Changes in plasma osmolarity cause cells to shrink or swell, as water flows out of or into the cells due to the osmotic
pressure difference between the inside and outside of the cell.
(1) Calculations of cell volume change. The final volume of the cell subjected to a change
in extracellular osmolarity can be calculated by the expression

TTi x Vi = TTf X V f
where i and f refer to the initial and final osmolarities and volumes of the cell.
(a) For example, if a red blood cell with an initial volume of 100 /-L3 and a normal
osmolality of 285 mOsm/L is placed in a solution with an osmolality of 325 mOsm/L,
its final volume will be
285 x 100 = 325 x V f

Vf

88 /-L3

(b) The volume of water leaving the red cell is so small compared to the volume of the
extracellular solution that the osmolarity of the extracellular solution is assumed to
remain constant.
(2) Osmotic pressure differences can only cause a steady state change in cellular volume
if the particles creating the osmotic pressure difference are impermeable to the

membrane. For example, because urea easily flows through cell membranes, a change
in the extracellular concentration of urea does not cause a steady state change in cell
volume.
(3) The ability of a particle to cause a steady state change in cell volume is referred to as
its tonicity. An extracellular solution that causes water to flow into the cell, thus making
it swell, is called hypotonic. An extracellular solution that causes water to flow out of
the cell is called hypertonic. A solution causing no change in intracellular volume is
called isotonic.
e. Flow of water due to osmotic pressure differences. When a membrane separates two
solutions that have different concentrations of nonpermeable particles, the osmotic pressure
difference will cause water to flow across the membrane.
(1) The water will flow from the solution with the lower concentration of particles to the
solution with the higher concentration of particles (Figure l-SA).
(a) The flow of water can be calculated using the osmotic flow equation
flow

L x A

(11, - 11 2 )

where L = the hydraulic conductivity (,....Usec/cm 2 /mm Hg) of the membrane,


A = the area (cm 2) of the membrane, and 11, and 112 = the osmotic pressures on
either side of the membrane.
(b) This equation is used to calculate the glomerular filtration rate (see Ch 4 V A 1 c).
(2) The osmotic pressure produced by a concentration difference can be determined experimentally by measuring the amount of hydrostatic pressure that must be applied to the
solution with the higher particle concentration (higher osmotic pressure) to prevent
water from entering it from the solution with the lower particle concentration (lower
osmotic pressure; see Figure l-SB).
f. Reflection coefficient. If the membrane separating two solutions is permeable to the
particles in solution, the osmotic pressure created by the concentration difference will be
reduced. The relative permeability of the membrane to the solute is expressed by the
reflection coefficient (0-).
(1) A reflection coefficient of 1 indicates that the membrane is not at all permeable to the
particle. That is, the particle is totally reflected by the membrane.
(2) A reflection coefficient of 0 indicates that the membrane is as permeable to the particle
as it is to water.

Figure 1-5. (A) When a semipermeable membrane separates two solutions of different osmolalities, water flows from
the solution with a lower osmotic pressure (concentration) to the solution with a higher osmotic pressure (concentration). Water will flow into the chamber until the hydrostatic pressure created by the increased height of the fluid
column equals the osmotic pressure difference between the two chambers. (8) The flow of water can also be prevented
by applying pressure to the chamber containing the higher solute concentration. The amount of pressure that must
be applied to prevent the flow of water is a measure of the osmotic pressure difference between the two chambers.

(3) The reflection coefficient can be calculated using the expression


CT= 1

_ Psolute
Pwater

(4) The osmotic flow equation can be used to calculate the osmotic flow of water if it is
modified to include the reflection coefficient
flow =

CT

x L x A X ('IT 1 -

'lT 2 )

g. Almost all particles dissolved in blood plasma, except proteins, easily cross the capillary
and, thus, do not cause water to flow between the capillary and the interstitial fluid. Plasma
proteins have an osmolar concentration of about 1.2 mOsm/L and, thus, create an osmotic
pressure of about 23 mm Hg.
(1) The osmotic pressure produced by the plasma proteins, called the colloid oncotic
pressure, draws water into the capillaries from the interstitial fluid and is counteracted
by the hydrostatic pressure of the blood produced by the heart. The movement of water
through the capillaries due to hydrostatic or osmotic pressure differences is called bulk

flow.
(2) Whether water flows into or out of the capillaries depends on whether the colloid

osmotic pressure is greater or less than the hydrostatic pressure of the blood. When
water flows through the capillaries, it carries dissolved particles with it. This is called

solvent drag.
D. Donnan equilibrium
1. Definition. When two solutions containing charged particles (ions) are separated by a membrane that is permeable to some of the ions and not to others, a Donnan equilibrium will be
established. A Donnan equilibrium is an example of an electrochemical equilibrium, because
the electrical and chemical energies on either side of the membrane are equal and opposite to
each other.

2. Characteristics of a Donnan equilibrium (Figure 1-6)


a. The concentrations of ions on one side of the membrane do not equal those on the other
side of the membrane. The tendency for the ions to diffuse down their concentration gradient
is balanced by an electrical potential that develops across the membrane.
b. Within each solution, the concentrations of anions and cations are equal to each other. That
is, electroneutrality is preserved.
c. The concentration of diffusible cations is greater in the solution that contains the nonpermeable, negatively charged particles. The concentration of diffusible anions is greater in the
solution without the non permeable particles.
(1) The concentrations of the diffusible ions on either side of the membrane are described
by the expression
[Na+]l x [CI-]1

[Na+]2 x [CI-]2

where [Na+] and [CI-] represent the concentrations of diffusible ions in solutions 1 and
2 of the example illustrated in Figure 1-6. The expression is valid for any group of
diffusible ions.
(2) The side of the membrane facing the solution with the nondiffusible anions is negatively
charged compared to the other side of the membrane.
d. The solutions are not in osmotic equilibrium. The total number of particles in the solution
with the nonpermeable anions is greater than the total number of particles in the other
solution. This creates an osmotic pressure gradient, causing water to flow into the cell. The
Na+ -K+ pump keeps water from flowing into the cells by preventing the establishment of a
Donnan equilibrium despite the presence of nondiffusible proteins inside the cells.
(1) The Na+-K+ pump keeps the Na+ concentration inside the cell low enough to maintain
osmotic equilibrium between the inside and outside of the cell.
(2) In brain ischemia, for example, decreased activity of the Na+-K+ pump allows the Na+
concentration to rise. The increase in intracellular osmolarity causes water to flow into
the cell, producing neuronal damage.
II. RESTING AND ACTION POTENTIALS. An electrical potential (voltage) difference exists between the inside and outside of all cells. This is called the resting membrane potential. The
resting potential is approximately -80 mV in excitable cells (e.g., nerve cells, muscle cells) and
approximately -20 to -40 mV in nonexcitable cells (e.g., red blood cells, epithelial cells). When

Millivoltmeter

-2 0

-3"'K........~

'I

'l

~
~
~
~
~
~

156.5 mEq/L
Na+

-~+
\:
-~+
\:

-~+

143.5 mEq/L
Na+

\:

131.5 mEq/L
CI-

1'5

mEqlLA

-~+

\:
-?::+
-?::+
\:
-?::+
\:
-?::+
?::

143.5 mEq/L
CI-

-~

Figure 1-6. Characteristics of an established Donnan


equilibrium. Compared to chamber 2 (2), chamber 1 (1)
is negatively charged and has a larger solute concentration, a greater cation concentration, a smaller concentration of permeable anions (A -).

excitable cells are stimulated, an action potential is generated, during which the membrane potential
changes from -80 mV to about +45 mV and then returns to the resting potential.

A. Recording resting and action potentials


1. Intracellular recordings of membrane potentials are made with glass microelectrodes that
have tip diameters of less than 0.5 I-L, allowing them to be inserted through the membrane
without damaging the cell.
a. Figure 1-7 shows the recording made as a microelectrode is inserted into a cell at rest. The
electrical potential is 0 mV when the microelectrode is outside the cell and drops to -80
mV as soon as the microelectrode passes through the membrane and enters the ICF.
b. When the cell is stimulated, the microelectrode records the changes in membrane potential
(Le., the action potential; see 1\ C 1).

2. Extracellular recordings usually are made with metal electrodes that are placed on or near
the nerve or muscle.

a. Because these electrodes are outside the cell, they can record only changes in membrane
potential (Le., action potentials but not resting potentials). Also, they cannot record the
exact magnitude or time-course of the action potentials.
+4SmV
Microelectrode
inserted
Overshoot
intocell~

OmV

-80mV

Threshold

Undershoot

o
Time (msec)

Figure 1-7. When a microelectrode


enters a nerve cell, a resting membrane potential is recorded. Stimulation produces an action potential,
which also is recorded. The various
components of the action potential
are indicated on the diagram.

b. Extracellular recordings are useful in clinical situations when the electrical activity of excitable tissues must be monitored. For example, electroencephalograms (EEGs) are used to
aid in the diagnosis of brain disease, electrocardiograms (EKGs) are used to detect damage
to the heart, and electromyograms (recordings from skeletal muscle) are used to aid in the
diagnosis of neuropathies and myopathies.
3. Patch electrodes are used to record the changes in membrane potential that occur in an
isolated patch of membrane. These electrodes can be used to monitor the opening and closing
of single channels.

B. The resting membrane potential is the electrical potential difference between the inside and
outside of the cell.

1. Factors that determine the resting potential


a. The resting potential is produced by the movement of ions across the plasma membrane.
The magnitude of the resting potential depends on:
(1) The concentration gradient for each ion to which the membrane is permeable
(2) The relative permeability (or conductance) of the membrane for each ion
b. The resting potential of nerve and muscle is determined by the concentration gradients
and membrane permeabilities (conductances) for Na+ and K+ . The concentration gradients
for these ions are established and maintained by the Na+ -K+ pump.
(1) K+ is much more abundant in the ICF than in the ECF, whereas Na+ is more abundant
in the ECF than in the ICF (see Table 1-1).
(2) To preserve electroneutrality, the intracellular and extracellular cations must be balanced
with anions.
(a) The primary anions of the ECF are HC03 - and CI-.
(b) The primary anions of the ICF are proteins and organic and inorganic acids (e.g.,
phosphates).
c. In the resting state, the membrane conductance for K+ is about 10 times as great as it is for
Na+.
d. Changing the membrane conductance for an ion will change the membrane potential.
Conversely, changing the membrane potential will change the membrane conductance for
an ion. This relationship between membrane potential and conductance enables excitable
tissues to produce action potentials.

2. Nernst potential (also called the diffusion, or equilibrium, potential) is used to express the
concentration gradient in electrical terms.
a. The Nernst potential is a measure of the amount of work that can be done by an ion diffusing
down its concentration gradient. It is the electrical equivalent of the energy (in mY) in the
concentration gradient. The Nernst potential is expressed as

RT
C;n
E = --InzF
Cout
where E = electrical potential (mV), R = natural gas constant, T = absolute temperature
(0 K), z = volume of the ion, F = Faraday's constant (96,500 coulombs/moll, and C;n and
Cout = the concentration of the ion (mmol/U inside and outside the cell.
b. The Nernst equation can be simplified by substituting for the constants (R, T, and F) and
converting to common logarithms, yielding
[ionl;n
61 log - . - [Ionl out
where E;on = the equilibrium potential (mV) for a particular ion, and [ion];n and [ion lout = the
intracellular and extracellular concentrations of that ion. The valence is omitted because it
is +1 for both Na+ and K+.
c. Table 1-2 gives Nernst potentials for some important electrolytes. (Note that -61 is divided
by -1 to calculate the Nernst potential for CI- and HC0 3 - and by + 2 to calculate the
Nernst potential for Ca H .)
E;on

= -

3. The resting membrane is in a steady state.


a. Ionic gradients and membrane potentials are constant in both the steady state and equilibrium conditions. However, in the steady state, free energy is used to maintain the constant
membrane potentials and ionic gradients, whereas no free energy is used in equilibrium
conditions, such as the Donnan equilibrium (see I D).

Table 1-2. Nernst Potential for Ions Commonly Found in Nerve and Muscle
Cells
Concentration (mmol/L)
Ion

Extracellular

Intracellular

Na+
K+
Ca2+
H+

140

15
135
10- 4
10- 4

CIHC0 3 -

2
10- 6
120

10

24

40

Nernst
Potential (mV)

+58
-92
+129
-24

-89
-23

b. The steady state is illustrated in Figure 1-8A, which shows a typical nerve axon that is
permeable to Na+ and K+.
(1) Na+ leaks into the cell down its concentration gradient; K+ leaks out of the cell down
its concentration gradient.
(2) The Na+ -K+ pump keeps the concentration gradients for Na+ and K+ from changing
by pumping Na+ out of and K+ into the cell.

4. Extracellular K+ concentration affects the resting membrane potential. Since the permeability of the resting membrane to K+ is so much higher than it is to Na+, the value of the
resting membrane potential can be approximated by the equilibrium potential for K+.
a. Increases in extracellular K+, which make the equilibrium potential for K+ more positive,
cause the resting membrane potential to become more positive (depolari7.e).
b. Decreases in extracellular K+, which make the equilibrium potential for K + more negative,
cause the resting membrane potential to become more negative (hyperpolarize).

C. The action potential results from a sequential change in membrane conductance for Na+ and
K+.
1. Phases and shape of the action potential (see Figure 1-7)
a. Threshold. The action potential begins when the membrane is depolarized (made less
negative) from its resting potential to its threshold potential by a stimulus.

A
K+

Na+

1(1
Na+

K+

B
IK

INa

= G K (Em

= G Na (Em

- E K)

- ENa )

Figure 1-8. (A) Although Na+ and K+ continuously leak down their electrochemical gradients, their concentrations
inside the cell are maintained at a constant value by the Na+ -K+ pump. (B) The electrical analog of the cell membrane
is used to analyze how changes in ionic concentrations and conductances affect the membrane potential (see II D).

b. Upstroke. The rapid depolarization of the membrane after threshold is reached in the
depolarization phase, or upstroke, of the action potential. The upstroke is produced by
the flow of Na+ into the cell.
c. Overshoot. The portion of the action potential during which the membrane is positive is
the overshoot. The peak of the action potential is the overshoot potential.
d. Downstroke. The rapid return of the membrane towards its resting potential is the repolarization phase, or downstroke, of the action potential. The downstroke is produced by
the flow of K+ out of the cell.
e. Undershoot. The membrane potential becomes more negative than its resting value at the
end of the action potential. This is the hyperpolarization phase, or undershoot, of the
action potential.
2. All-or-none response. The action potential is an all-or-none response to a stimulus. That is, if
the stimulus is strong enough to reach threshold, the changes in membrane potential that
characterize the action potential are always the same. This feature of the action potential is
based on the regulatory gates that cover the Na+ and K+ channels (Figure 1-9).
a. Regulation of the Na+ channel
(1) The m gate covers the extracellular side of the Na+ channel and the h gate covers the
intracellular side of the Na+ channel.
(2) Both the m and the h gates must be open for Na+ to flow through the Na+ channel.
(a) When the m gate is open, the channel is said to be activated.
(b) When the h gate is closed, the channel is said to be inactivated.
b. Regulation of the K+ channel
(1) The K+ channel is regulated by a single gate-the n gate-which is located on the
extracellular side of the channel.
(2) The n gate must be open for K+ to flow through the K+ channel.
(a) When the n gate is open, the K+ channel is activated.
(b) The K+ channel does not have an inactivation gate.
3. Mechanism of gating action. The m, n, and h gates have two qualities that permit an action
potential to be generated when the cell membrane is depolarized by a stimulus: voltage dependence and time dependence.
a. Voltage dependence
(1) The m and n gates close when the membrane potential is polarized (-80 mY) and open
when the membrane potential is depolarized (made more positive).
(2) In contrast, the h gates open when the membrane potential is polarized and close when
it is depolarized.
(3) Thus, when the membrane is at its resting potential, almost all of the Na+ and K+
channels are closed by the m and n gates, respectively (see Figure 1-9).
(a) Although only a few channels actually are open when the cell is at its resting
potential, the number of open K+ channels far exceeds the number of open Na+
channels.
(b) Although the Na+ channels are closed when the membrane is at its resting potential,
the channels are not inactivated because the h gates are open.

Extracellular space

Figure 1-9. A diagrammatic representation of the gates


Intracellular space

covering the Na+ and K+ channels. The negative resting


potential tends to keep the m and n gates closed and the
h gate opened.

b. Time dependence
(1) Time is required for the gates to respond to a change in membrane potential. For
example, when the membrane potential is depolarized:
(a) The m gates open first, activating the Na+ channels
(b) The h gates then dose, inactivating the Na+ channels
(c) The n gates then open, activating the K+ channels
(2) Time dependence of the gates is essential for the production of the action potential.
(a) If the h gates closed as rapidly as the m gates opened (i.e., if the Na+ channel was
inactivated and activated at the same time), Na+ could not flow into the cell.
(b) Similarly, if the n gates opened as fast as the m gates opened (i.e., if Na+ and K+
activation occurred at the same time), the upstroke could not occur.
c. The gating mechanism produces the phases of the action potential (Figures 1-10 and
1-11).
(1) Upstroke. A positive feedback, or regenerative, process is responsible for the flow
of Na + into the cell during the upstroke (see Figure 1-10).
(a) When a stimulus depolarizes an excitable membrane, m gates on some of the Na+
channels open, allowing Na + to enter the cell (see Figure 1-11 A).
(b) The flow of Na+ into the cell causes the membrane to depolarize further, which
causes more m gates to open (the Na+ conductance increases) and allows more
Na+ to enter the cell.
(c) Thus, the response of the membrane to a stimulus (opening of the m gates) causes
an effect (membrane depolarization) that produces an even greater response.
(2) Downstroke. Inactivation of the Na+ channels and activation of the K+ channels occur
when the membrane potential is depolarized, causing the downstroke (see Figure 1-11 B).
(a) The Na+ channels are inactivated by the closing of the h gates. Inactivation stops
the flow of Na+ into the cell. If inactivation did not occur, repolarization would be
slowed because Na+ would flow into the cell during the downstroke.
(b) The K+ channels are activated by the opening of the n gates. Activation allows K+
to leave the cell, which produces the downstroke of the action potential.
(c) The inactivation of the Na+ channels must be removed before another action
potential can occur. The h gates open during the downstroke, removing the inactivation from the Na + channels (see Figure 1-11 C).
(3) Undershoot is caused by the slow closing of the K+ channels.
(a) The n gates close slowly when the membrane is repolarized during the downstroke.
(b) Consequently, the K+ conductance is higher at the end of the action potential
than during the resting state. The high K+ conductance causes the membrane to
hyperpolarize (become more negative than the resting membrane potentia\).
(c) Eventually, the n gates close and the membrane potential returns to its resting level.
4. Refractory period refers to an interval during which it is more difficult to elicit an action
potential. There are two refractory periods (see Figure 1-11).
a. Absolute refractory period. During this interval, another action potential cannot be elicited,
regardless of the strength of the stimulus.
(1) The absolute refractory period begins at the start of the upstroke and extends into the
downstroke.
(a) During the upstroke, a second action potential cannot occur because the m gates
are opening as fast as possible (see Figure 1-11 A).
(b) During the early portion of the downstroke, an action potential cannot occur because the Na+ channels are inactivated by the h gates (see Figure 1-11 B).
Opening of

mgate,

Increase in G Na

Depolarization

Na'
enters cell

Figure 1-10. When the cell is depolarized to threshold,


Na+ channels open, causing an increase in the conductance for Na+ (G N .). This allows Na+ to enter the cell,
causing further depolarization. The positive feedback system represented by this cycle is responsible for the upstroke of the action potential.

(2) The absolute refractory period ends when the number of inactivated Na+ channels is
few enough to allow another action potential to occur. Inactivation of the Na + channels
is removed when the h gates open during the downstroke.
b. Relative refractory period. During this interval, a second action potential can be elicited
if the stimulus is sufficient. The stimulus must be greater than normal, because some Na+
channels are still inactivated and more K+ channels than normal are still open (see Figure
l-l1C).
(1) The relative refractory period begins when the absolute refractory period ends.
(2) The action potential elicited during this interval has a lower upstroke velocity and a
lower overshoot potential than the normal action potential.
(a) These changes result from the increased number of inactivated Na + channels and
activated K+ channels that exist during the relative refractory period as compared
to the resti ng state.
(b) The changes do not violate the all-or-none principle of the action potential but
demand that the principle be revised to state: If a stimulus is sufficient to bring the
membrane to threshold, the strength of the stimulus will not affect the magnitude
and time-course of the action potential.
5. Propagation of the action potential. Once generated, the action potential must be propagated
(conducted) along the axon. Propagation occurs because the action potential generated at one
location on the axon acts as a stimulus for the production of an action potential on the adjacent
region of the axon.

Relative refractory period

Absolute :
refractory I
period :

Figure 1-11. Diagram illustrating how the position of the Na+ and K+ gates change during the action potential. (A)
Both the m and h gates are opened during the upstroke of the action potential, allowing Na + into the cell. (8) The
closing of the h gates (inactivation), which stops the flow of Na+ into the cell, and the opening of the n gates, which
allows K+ to flow out of the cell, are responsible for the downstroke. (C) During the undershoot, the n gates are open,
the m gates are closed, and the h gates are open. The high K+ conductance causes the cell to hyperpolarize.

a. Propagation in unmyelinated axons (Figure 1-12)


(1) The process. During the overshoot of the action potential in an unmyelinated axon, the
membrane potential becomes about +40 mY, creating an electrical potential difference
between the area of membrane on which the action potential is generated and the
adjacent, polarized area (see Figure 1-12A).
(a) Because of this potential difference, current flows passively between the two areas,
causing the adjacent region to become depolarized. If the adjacent area is depolarized to threshold, an action potential is generated (see Figure 1-12B).
(b) The depolarization of the membrane produced by the new action potential spreads
passively, and the entire process is repeated.
(c) Thus, propagation of the action potential involves the generation of action potentials
on contiguous patches of membrane along the axon.
(2) The magnitude of the action potential does not change as it is conducted along the
axon, because new action potentials are being generated constantly. This is different
from the spread of an electrotonic potential (see II F), which diminishes in size along
the axon.
(3) The speed of propagation is proportional to the square root of the fiber diameter. To
increase the speed of propagation, fiber diameter must be increased. However, there
is a practical limit to how large axons can become.
(a) For example, a typical unmyelinated fiber is 1 IJ- in diameter and propagates at about
1 m/sec.
(b) For such a fiber to propagate at 50 m/sec (the typical velocity of a myelinated
neuron), it would need to be 2.5 mm in diameter. Imagine how large a typical motor
nerve (containing 1000 axons) would be if each axon were 2.5 mm in diameter.

b. Propagation in myelinated axons


(1) The process
(a) In principle, propagation in myelinated axons is the same as in unmyelinated axons.

However, the cell membrane in myelinated axons is exposed to the ECF only at the
nodes of Ranvier. Nodes occur every 100-500 IJ-. In general, as the diameter of the
axon increases, the internodal distance increases.
(b) The membrane area between the nodes is covered by an insulating sheath of myelin
formed from Schwann cell membranes.
(c) The action potentials are generated only at the nodes. That is, an action potential
generated at one node becomes the stimulus for the generation of an action potential
at the adjacent node. The propagation of the action potential from node to node is
called saltatory conduction.
(2) The speed of propagation is proportional to the diameter of the axon and the internodal
distance. Because the action potential can spread from node to node, instead of having
to produce an action potential at each contiguous patch of membrane, propagation of
the action potential is faster than in unmyelinated nerves.
(a) Myelinated fibers are used by the nervous system when high speeds of conduction
are necessary. For example, the axons of the major ascending and descending tracts
of the spinal cord, the sensory axons used for fine tactile discrimination, and the
motor axons all are myelinated.

++++

+40mV~-60
Current flow

-80

-80

-60

-80

B
+++

A++H
+~

Figure 1-12. (A) The positive potential (+40 mY) produced during the
overshoot of the action potential
causes current to flow toward the
negative, resting portion of the axon.
The flow of current acts as a stimulus
depolarizing the axon toward threshold. (8) When threshold is achieved,
an action potential is elicited and the
entire process is repeated, causing the
action potential to be propagated
along the axon.

(b) Myelinated fibers are 1-20,." in diameter. The largest fibers conduct action potentials
at speeds of up to 120 m/sec.
D. Electrical analog of membrane potentials. An electrical analog of the cell membrane, such as
the one shown in Figure 1-88, can be used to analyze and calculate the membrane potential
during the resting state or the action potential.

1. The electrochemical gradient pushing an ion through the membrane is the difference between
the membrane potential (Em) and the equilibrium potential for the ion (E ion), which is expressed
as
driving force

Em - E;on

where the driving force is the electrical potential energy (mV).

2. The ionic current can be calculated for each ion using Ohm's law for solutions
Gion(E m - Eion)
current (milliamperes), and G ion = ionic conductance (mhos or Siemans).
lion

where lion

3. The net current across the membrane in the steady state is zero. Thus, the current flowing
into the cell must be equal and opposite to the current flowing out of the cell.
a. The inward current is due to the flow of Na+ into the cell and is given by
I;on = - G ion (Em - Eion )
b. The outward current is due to the flow of K+ out of the cell and is given by
I;on

+ G;on (Em - Eion )

4. The transference equation can be derived by setting the inward and outward currents equal
to each other and solving for Em
GN') + ( EK X --'-'---GK )
G N + G K
G N + G K
where G N and G K = the conductance for Na+ and K+, and the ratios GN./(G N + G K) and G K/
(G N + G K) = the transferences for Na+ and K+ (TN. and T K)' Thus, the transference equation
can be simplified to
Em = ( EN.

5. The electrical analog of the cell can be used to illustrate how transference affects the membrane
potential (see Figure 1-88).
a. The arrows drawn through the conductors, G K and GN., represent the transferences of K+
and Na+.
(1) If the only open channels in the membrane are K+ channels (i.e., if TK = 1 and TN. =
0), then Em will equal the value of the K+ battery (E K, or -92 mV).
(2) If the only open channels in the membrane are Na + channels (i.e., if TK = 0 and TN. =
1), then Em will equal the value of the Na+ battery (EN., or +60 mV).
(3) If 90% of the open channels in the membrane are K+ channels (i.e., if T K = 0.9 and
TN. = 0.1), then Em will be much closer to the K+ battery (E K ) than to the Na+ battery
(EN.)'
b. These examples illustrate that Em is determined primarily by the equilibrium (Nernst)
potential of the ion with the highest transference (conductance).
E. Calculation of resting membrane potential
1. Transference equation
a. Using the transference equation, the resting membrane potential is calculated as follows:
(1) Since G K is 10 times G N

10
10
T = - - = - = 0.91
K
10+1
11
(2) Similarly

T
N.

= - - = - = 0.09
10 + 1
11

(3) Substituting these values and the values for EK and ENa (from Table 1-1), the transference
equation yields a membrane potential of -79 mV

(0.91 x -93) + (0.09 x +60)

-79

b. The electrogenic nature of the Na+-K+ pump makes the actual membrane potential a few
millivolts more negative than the value calculated by the transference equation.
(1) Since three Na+ ions are pumped out of the cell for every two K+ ions that are pumped
in, the activity of the pump produces a small negative potential. The Na+-K+ pump is
labeled "pump" in the electrical analog of the membrane (see Figure 1-8B).
(2) Thus, the transference equation should be written
Em

(T Na

EN.! + (TK x EK) + Epump

(3) Because the pump potential usually is small, it generally is ignored.


2. Goldman-Hodgkin-Katz (GHK) equation. The GHK equation, like the transference equation,
is derived for a steady-state condition in which the inward and outward currents are equal and
opposite. However, the GHK equation expresses ionic currents according to the laws of
electrodiffusion, rather than Ohm's law.
a. The GHK equation is stated as
E =
m

RT
- -

PK[K];n + PNa[Na]in
log - - - - - - PdKlout + PNa[Na]out

where PK and PNa represent the permeabilities of K+ and Na+.


b. Note that when the membrane is permeable to only one ion (j.e., the permeability of the
membrane to the other ion is zero), the GHK equation reduces to the Nernst equation (see
II B 2 a).
3. Both the GHK and transference equations, thus, predict that Em will be determined primarily
by the equilibrium potential of the ion that is able to cross the membrane most easily (i.e., the
ion with the highest membrane permeability, or conductance).

F. Electrotonic potentials are changes in membrane potential that do not propagate.

1. Types of electrotonic potentials


a. A local (subthreshold) response is produced when a stimulus does not open enough
m gates to elicit an action potential. Because of the small number of open m gates, the
amount of Na+ entering the cell is insufficient to initiate a positive feedback cycle.
(1) The local response is graded; the magnitude and duration of the response vary with
the size and strength of the stimulus.
(2) The local response is nonpropagated; that is, its magnitude is insufficient to generate
another local response.
b. Other graded, non propagated responses are produced on nerve and muscle membranes.
(1) For example, the generator and receptor potentials produced by sensory stimuli on
receptors (see V B 1) and the excitatory and inhibitory potentials produced by neurotransmitters on synaptic membranes [see III A 3 b (2)] are graded and nonpropagated.
(2) The receptor and synaptic potentials differ from the local response in that the membrane
channels producing them are not voltage- or time-dependent.
2. Cable properties of the membrane. The membrane changes produced by graded, nonpropagated responses spread passively, or electrotonically, along the membrane. Cable properties of
the membrane determine the time-course and voltage changes of an electrotonic potential. The
electrical equivalent of the cable properties of an axon is illustrated in Figure 1-1 3A.
a. Time constant. When a current is applied to a membrane by a stimulus, a charge is added
to the capacitor, causing a potential difference to develop across the membrane.
(1) The potential difference, V" develops at a rate that is determined using the equation
Vt

V max

(1 - e- t/T )

where V t = the voltage at time t; V max = the voltage of the applied stimulus; T = the
time constant (seconds), or rmCm' where rm = membrane resistance (ohms) and Cm =
membrane capacitance (farads).

>

ECD

Cl

!!!

o
>

_----v
163 % V max
I
1
1

1
1

r-1

Time (t)

c
max

>

ECD

Cl

~
o

>

I
I
I

I
I
I
I

f-- A----j
Distance (x)

Figure 1-13. (A) Equivalent circuit of an axon. Each patch of axon contains a resistor representing the conductive
pathways through the membrane (rm), a capacitor representing the lipid bilayer of the membrane (em), and a resistor
representing the intracellular pathway for the flow of ions along the axon (r;l. (8) When a stimulus is applied to a
nerve membrane, the membrane depolarizes exponentially according to the equation V t = V max X (1 - e -tiT). When
t = T (the time constant), V T = 0.63 V max' (0 The magnitude of the depolarization decreases as the distance from
the stimulus increases according to the equation Vx = V max X e- x/X ). When the distance from the stimulus equals A
(the space constant), V x = 0.37 V max'

(2) The time constant, T, of the membrane is equal to the time required for the membrane
voltage to reach about 63% (1 - 1 Ie) of V max (see Figure 1-13B).
b. Space constant. The potential produced by the stimulus is spread passively along the
membrane.
(1) At any point along the membrane, the potential, V" is given by the equation

where x = distance (mm), and A = the space constant.


(2) The space constant, A, is the distance from the stimulus to the point at which the applied
voltage falls to about 37% (1 Ie) of V max (see Figure 1-13Cl.
(3) In an axon, the space constant is equal to rm/r; and, thus, increases if either of the
following occurs:
(a) The membrane resistance increases
(b) The axoplasmic resistance decreases
c. The cable properties of a neuron play an important role in determining the ability of a
stimulus to elicit an action potential and the propagation velocity of action potentials.
Graded, nonpropagating responses (e.g., synaptic and receptor potentials) must spread
passively from the patch of membrane where they are produced to a patch of membrane
that is able to produce an action potential.
(1) If the graded response is produced too far from the action potential-producing portion
of the membrane (e.g., at the end of a long dendrite), it will decay too much to be able
to depolarize the action potential-generating portion of the membrane to threshold.
(2) If the membrane resistance is reduced (e.g., by an inhibitory synaptic transmitter), the
space constant will be reduced, and the ability of an excitatory response to spread
passively to the action potential-generating region of the membrane will be reduced.

(3) If the time constant is increased, it will take longer for the action potential produced at
one point along the axon to depolarize its adjacent region to threshold, and propagation
velocity will slow.
(a) In demyelinating diseases, such as multiple sclerosis, the loss of myelin increases
the membrane capacitance, which increases the time constant.
(b) The increase in the time constant causes action potential propagation to fail, producing the sensory and motor deficits characteristic of multiple sclerosis.

III.

SYNAPTIC TRANSMISSION

is the process by which nerve cells communicate among themselves


and with muscles and glands. The synapse is the anatomic site where this communication occurs.
Most synaptic transmission is carried out by a chemical called a neurotransmitter. The neurotransmitter is released from a neuron (the presynaptic cell) and diffuses to its target (the postsynaptic cell),
where it produces a postsynaptic response. This response is an action potential (if the target cell is
another neuron), contraction (if the target cell is a muscle), or secretion (if the target cell is a
gland). The space between the presynaptic and postsynaptic cells is called the synaptic cleft. The
neurotransmitter may be inhibitory, in which case it decreases the activity of the postsynaptic cell.
In some instances, synaptic transmission may be electrical and occur through gap junctions.

A. Neuromuscular transmission refers to synaptic communication between an alpha motoneuron


and a skeletal muscle fiber. Neuromuscular transmission also occurs between autonomic efferent
fibers and both smooth and cardiac muscle cells (see III B).

1. Physiologic anatomy (Figure 1-14)


a. Light microscopic appearance. Figure 1-14A is a drawing of the neuromuscular synapse
as viewed with a light microscope.
(1) The alpha motoneuron branches as it approaches the muscle, sending axon terminals
to several skeletal muscle fibers. Each skeletal muscle fiber receives only one axon
terminal. The number of skeletal muscle fibers innervated by an alpha motoneuron
depends on the type of muscle fiber involved.
(a) Alpha motoneurons innervating large muscles used primarily for strength or postural
control innervate hundreds to thousands of skeletal muscle fibers.
(b) Alpha motoneurons innervating muscles used for precision movements innervate
just a few skeletal muscle fibers.
(2) The axon terminal lies in a groove called the synaptic trough, which is formed by an
invagination of the skeletal muscle fiber.
(3) Synaptic transmission occurs at the end-plate region of the skeletal muscle fiber.
b. Electron microscopic appearance. The details of the presynaptic and postsynaptic (endplate) membranes are visible in the electron microscopic view of the synaptic junction in
Figure 1-14B.
(1) Synaptic vesicles (about 50 nm in diameter) containing the neurotransmitter acetylcholine (ACh) are found in the presynaptic nerve terminal concentrated around specialized
presynaptic membrane structures called dense bars. The postsynaptic membrane contains receptor sites to which ACh binds.
(2) The synaptic cleft (about 60 nm wide) is filled with an amorphous network of connective
tissue called the basal lamina, in which the enzyme acetylcholinesterase (AChEase)
is bound. AChEase is responsible for degrading ACh after it has produced its effect on
the end-plate membrane of the skeletal muscle fiber.
(3) The postsynaptic membrane contains numerous junctional folds, which are membrane
invaginations located opposite the dense bars. The receptor sites for ACh are found on
the membranes of the junctional folds.

2. Synthesis, storage, and release of ACh


a. ACh is synthesized in the nerve terminal from choline and acetyl coenzyme A (acetylCoAl by the enzyme choline acetyltransferase (CAT).
b. Newly synthesized ACh is stored within the synaptic vesicles. Approximately 5000-10,000
molecules of ACh are stored within each vesicle.
c. Spontaneous release of ACh occurs by exocytosis (Figure 1-15) whenever a vesicle binds
to an attachment site on one of the dense bars.
(1) In exocytosis, the vesicle fuses with the presynaptic membrane, exposing its content to
the ECF.

Alpha moto'O"""

t;:;

Synaptic trough

Skeletal
muscle
fiber---=:!:,-,,"

. ~.

..; ..

.
. .'.. :.:."."
'

.~

Presynaptic membrane

Synaptic vesicles

og1o
OC

~-:--""""'~---Dense

bar

Junctional fold

Figure 1-14. (A) Light microscopic view of a neuromuscular junction. The alpha motoneuron branches as it reaches
the muscle, and each branch forms a synapse with a single muscle fiber at the end-plate region of the muscle. The
axon terminal lies in the synaptic trough. Junctional folds increase the surface area of the postsynaptic membrane.
(B) Electron microscopic view, showing the synaptic vesicles within the presynaptic terminal, the acetylcholinesterase
(AChEase) within the synaptic cleft, and the postsynaptic receptor sites. ACh = acetylcholine.

(2) ACh diffuses out of the vesicle into the synaptic cleft and the vesicle merges with the
presynaptic membrane.
(3) Later, new vesicles are formed from the presynaptic membrane by endocytosis.
(a) The presynaptic membrane forms invaginations that eventually bud off to form new
vesicles.
(b) The new vesicles are refilled with ACh and, once again, are available to release
their contents into the ECF.

3. Events in synaptic transmission


a. Release of neurotransmitter.

Synaptic transmission begins when an action potential is


propagated into the nerve terminal.

Reforming
of vesicles by
membrane

1"),,'"

;0

Figure 1-15. Diagram illustrating the life cycle of a synaptic vesicle. After binding to its attachment site, the
vesicle releases its transmitter, acetylcholine (ACh), by
exocytosis and then merges with the membrane. Later, a
new vesicle is formed from invaginations of the synaptic
membrane. These vesicles are filled with transmitter and
can be used again.

(1) Depolarization of the nerve terminal by the action potential causes Ca2+ channels
(which are located next to the dense bars) to open, allowing Ca2+ to enter the cell down
its electrochemical gradient.
(2) The increase in intracellular Ca2+ concentration causes about 200-300 vesicles to bind
to attachment sites and release their contents into the synaptic cleft.
b. Postsynaptic response. The ACh released into the synaptic cleft binds to ACh receptors
on the end-plate membrane, where it causes a postsynaptic response called the end-plate
potential (EPP).
(1) The ACh receptor (Figure 1-16) is a transmembrane protein consisting of five subunits
that form an aqueous channel within the lipid bilayer.
(a) Two of the subunits, called a subunits, contain binding sites for ACh.
(b) When the two a subunits are occupied by ACh, the proteins undergo a conformational change that opens a channel within the receptor.
(c) The channel is equally permeable to Na+ and K+.
(2) The channel associated with the ACh receptor is a chemically activated channel.
(a) Unlike the Na+ and K+ channels found on electrically excitable membranes (which
are activated by changes in membrane voltage), the ACh-activated channel is
opened by the binding of the neurotransmitter to the receptor and not by membrane
depolarization. These channels are sometimes called receptor-activated, or
receptor-occupied, channels.
(b) The magnitude of the EPP is proportional to the number of channels opened by
ACh (i.e., the EPP is a graded response and it is not propagated>.
(3) Opening the channel causes the cell to depolarize.
(a) When the channel is opened, Na+ enters the cell down its electrochemical gradient,
while K+ leaves the cell down its electrochemical gradient.

Figure 1-16. A diagrammatic view of the acetylcholine


(ACh) receptor. The receptor contains five subunits, two
of which (the a subunits) contain binding sites for ACh.
When both subunits are occupied, the channel gate
opens, allowing Na+ and K+ to pass through the membrane. (Adapted from Anholdt R, Lindstrom J, Montal M:
In Enzymes of Biological Membranes. Edited by Martonosi A. New York, Plenum Press, pp 335-401, 1985.)

(b) The Na+ current (IN) and the K+ current (lK) can be calculated using Ohm's law
(see II D 2) as follows
IN. = G N x (Em - ENol and IK = G K x (Em - EK)
(c) Since the electrochemical gradient for Na+ is greater than that for K+ (and the
conductances for Na+ and K+ are equal), the amount of Na+ entering the cell
exceeds the amount of K+ leaving the cell, and the cell depolarizes.
(4) The reversal potential is the potential at which no net current flows through the channel
(i.e., when the Na+ and K+ currents are equal and opposite to each other).
(a) These currents become equal and opposite to each other when the membrane
potential becomes -16 mY. (This can be verified by substituting the appropriate
values of EN. = +60 and EK = -92 into the above equations and solving for Em when
IN. = IK. Since G N = G K, the conductance terms cancel out.)
(b) If the only open channels in the membrane were the ACh synaptic channels, the
membrane potential would be equal to the reversal potential.
(i) This is analogous to what happens when the K+ channel is the only open channel
[see II D 5 a (1 )j. Under these conditions, the membrane potential is equal to
the equilibrium potential for K+ (E K).
(ii) The analogy is illustrated by the equivalent circuit diagram (see Figure 1-8B), in
which the synaptic battery (representing the reversal potential for ACh) is given
the value -16 mY.
(c) However, as the equivalent circuit diagram illustrates, the ACh channel is not the
only open channel in the membrane.
(i) If a single ACh channel is opened (as occurs when two ACh molecules bind to
it--one to each a unit), the membrane will depolarize by only a few microvolts
(,...,V).
(ii) If a single vesicle releases its contents of 5000-10,000 ACh molecules, the
membrane will depolarize by about 1 mY. The small (1 mY) depolarization
caused by the spontaneous release of one vesicle is called a miniature end-plate
potential (MEPP). Spontaneous release of vesicles occurs at a rate of about
1 /sec. Thus, MEPPs occur every second or so. The MEPPs may be important in
maintaining the integrity of the muscle fiber, because denervation of skeletal
muscle fibers leads to muscle atrophy.
(iii) The 200-300 vesicles that release their contents when an action potential invades the presynaptic nerve terminal produce a depolarization of about 50 mY.
This depolarization is the EPP.
c. The EPP depolarizes the muscle fiber membrane to threshold.
(1) The EPP is a graded, non propagated response that acts as a stimulus for the production
of an action potential on the muscle membrane contiguous to the end-plate membrane.
(2) The ACh molecules contained within the several hundred vesicles released during
neuromuscular transmission depolarize the membrane from its resting potential of -90
mV to about -40 mY.
(a) The depolarization of the end-plate membrane causes the contiguous membrane to
be depolarized [see II C 5 aj.
(b) When the membrane next to the end-plate membrane reaches threshold (about
-60 mY), an action potential is generated.
(c) The action potential is propagated along the muscle membrane and is responsible
for initiating a muscle contraction (see IV A 2).
(3) An action potential cannot be generated on the end-plate because there are no electrically excitable K+ and Na + channels present on the end-plate membrane. ACh receptors
are so densely packed into the end-plate that there is no room for any other membrane
proteins.
(4) Each time an alpha motoneuron fires an action potential, it causes all of the muscle
fibers that it innervates to contract.
d. ACh is degraded rapidly.
(1) After binding to the ACh receptor, the ACh dissociates from the receptor and is hydrolyzed by the AChEase in the synaptic cleft.
(2) Degradation of ACh is necessary to prevent it from causing multiple muscle contractions.
(3) Enzymatic destruction is a unique method for inactivating the transmitter and occurs
only at ACh synapses. Inactivation of the transmitter at all other synapses occurs when

the transmitter diffuses out of the synaptic region or is actively transported back into
the nerve terminal.
B. Autonomic synaptic transmission. The autonomic nervous system is divided into the parasympathetic and sympathetic systems, each of which has a preganglionic neuron in the central
nervous system (CNS) and a postganglionic neuron in the peripheral nervous system (PNS).
In both divisions of the autonomic nervous system, ACh is the transmitter used to communicate
between the pre- and postganglionic fibers. ACh also is used by the parasympathetic postganglionic
fibers, whereas norepinephrine is the transmitter used by the sympathetic postganglionic fibers.
Many postganglionic fibers, particularly those within the ganglia of the gastrointestinal tract (the
enteric nervous system), do not use ACh or norepinephrine as the transmitter substance. Because
the identity of the transmitter is not clear, it often is referred to as a noncholinergic, nonadrenergic

transmitter.
1. Ganglionic transmission within the sympathetic and parasympathetic divisions of the autonomic nervous system is essentially the same as that at the neuromuscular junction. ACh is
released from the preganglionic (presynaptic) fiber, diffuses across the synaptic cleft, and binds
to receptors on the postganglionic fiber, causing it to depolarize.
a. In almost all respects, this process is identical to that of skeletal muscle neurotransmission
(see III A 3). However, a single preganglionic fiber does not release enough neurotransmitter
to depolarize the postganglionic fiber to threshold. The postganglionic fiber can discharge
an action potential only when there is a summation of the postsynaptic responses (see III

C 2).

b. The ACh receptors on the postganglionic fibers are called nicotinic receptors because they
are activated by nicotine. The ACh receptors on the skeletal muscle end-plate also are
nicotinic receptors.
(1) The nicotinic receptors on the autonomic ganglion postganglionic membranes are not
identical to those on the end-plates of skeletal muscle because the pharmacologic agents
needed to block the two receptors are different.
(a) Hexamethonium is used to block ganglionic transmission.
(b) Curare is used to block neuromuscular transmission.
(2) Nicotine is extremely toxic: it causes vomiting, diarrhea, sweating, and high blood
pressure, primarily due to its action on the autonomic nervous system.
(3) Similar to the receptors on skeletal muscle, the ACh receptors on postganglionic fibers
cause the membrane to depolarize by opening channels that are equally permeable to
K+ and Na+.
2. Parasympathetic postganglionic fibers can have either an excitatory or an inhibitory effect,
depending on the postsynaptic receptor that is activated.
a. Excitatory effects are produced on a variety of smooth muscles (e.g., those within the
stomach, intestine, bladder, and bronchi) and on glands. ACh can produce its excitatory
effect by a variety of mechanisms.
(1) ACh can bind to receptors that cause the membrane to depolarize in a mechanism
similar to that occurring on the skeletal muscle end-plate.
(2) ACh can bind to receptors that increase Ca2+ conductance. The increase in Ca2+
conductance does not produce a major effect on membrane potential. However, the
Ca2+ entering the cell through the channels opened by ACh can be used to initiate
contraction in smooth muscles (see IV C 2).
(3) ACh can bind to receptors that activate the membrane-bound protein guanosine triphosphate (GTP) binding protein, or G protein (Figure 1-17 A). When the G protein
is activated (see Figure 1-17B), it initiates a series of membrane and intracellular events
leading to muscle contraction.
(a) The activated G protein (see Figure 1-17C) activates a membrane-bound lipase
called phospholipase C.
(b) Phospholipase C causes phosphatidylinositol diphosphate (PIP2 ) to break down
into two other lipids, diacylglycerol (DAG) and inositol triphosphate (IP 3 ).
(i) DAG activates protein kinase C, a cytoplasmic enzyme that activates a series
of cytoplasmic proteins by phosphorylating them. These proteins then cause a
physiologic effect such as opening an ionic channel.
(ii) IP 3 diffuses through the cytoplasm and binds to the sarcoplasmic reticulum,
causing Ca2+ to be released into the cytoplasm. The increase in intracellular
Ca2+ then leads to muscle contraction.

Agonist

Receptor
(a, or muscarinic)
A

nu
I111

G protein
(3
'Y

r~r!
1111

Phospholipase C

PIP,

trrl

ltl!
il~l

1111

~!n.uu
1 rl

~~IJ

PIP,

l111R

111r

nIl
1~1

pH~11

QJ
GOP

rrrr

n
... -,
... _'

ii~l

{IP, I
Protein kinase C

Figure 1-17. (A) When a neurotransmitter or other agonist binds to a muscarinic or a, receptor, (B) the agonistreceptor complex induces the G protein to exchange guanosine diphosphate (COP) for guanosine triphosphate (CTP).
(C) The presence of GTP causes the a subunit to separate from the G protein. The a subunit migrates within the
membrane, eventually forming a complex with and activating phospholipase C. The phospholipase C, in turn, causes
the breakdown of phosphatidylinositol diphosphate (PIP,) to inositol triphosphate UP3 ) and diacylglycerol (DAC). IP 3
releases Ca2+ from the sarcoplasmic reticulum; DAG activates protein kinase C. The a subunit contains a GTPase
that catalyzes the breakdown of GTP to GOP, inactivating the G protein.

b. Inhibitory effects of parasympathetic fibers are produced on the heart, primarily on the
pacemaker regions of the sinoatrial (SA) node (decreasing the heart rate) and atrioventricular
(AV) node (slowing conduction of the action potential from the atria to the ventricles) [see
Ch 2 II B 3 c (5) (b), C).
(1) When ACh binds to a receptor on the heart, the receptor causes a K+ channel to open.
(2) K+ flows out of the cell down its electrochemical gradient, causing the cell to hyperpolarize.
(3) The hyperpolarization acts to decrease the rate of pacemaker activity and to slow the
conduction of the action potential through the AV node.
c. The postganglionic receptors are called muscarinic because they respond to the drug
muscarine and not to nicotine. Muscarinic receptors are blocked by the drug atropine.

3. Sympathetic postganglionic fibers that release norepinephrine can have an excitatory or


inhibitory effect, depending on the type and location of the receptor activated.
a. Norepinephrine receptors are divided into beta (f3) and alpha (a) receptors.
(1) When norepinephrine binds to the f3 receptors, it activates a G protein similar to that
activated by ACh (Figure 1-18).
(a) However, in this case, the activated G protein activates a different membrane-bound
protein: adenylate cyclase.
(b) Adenylate cyclase stimulates the formation of cyclic adenosine 3',5'-monophosphate (cAMP) from ATP. The G protein that stimulates the formation of cAMP
is called the G. protein.

G protein
{3

Adenylate

1rrl T" nn
111

~il~1

~
ATP
cAMP

Figure 1-18. (A) When an agonist binds to the f3 receptor, (8) the agonist-receptor complex activates a G protein,
which, in turn, (0 activates adenylate cyclase. The adenylate cyclase catalyzes the conversion of ATP to cyclic
adenosine 3',S'-monophosphate (cAMP). cAMP affects intracellular proteins by phosphorylating them. As described
in Figure 1-17, the G protein is inactivated when guanosine triphosphate (CTP) is converted to guanosine diphosphate
(COP).

(c) cAMP then activates the cytoplasmic enzyme protein kinase A (or cAMP-

dependent protein kinase), which, in turn, produces a variety of physiologic


responses by phosphorylating an assortment of intracellular proteins.
(i) In heart cells, protein kinase A phosphorylates Ca2+ channels, which increases
the amount of Ca2+ entering the cell with each action potential and, thus,
increases the force of contraction.
(ii) In bronchiole smooth muscle cells, protein kinase A phosphorylates the Ca 2 +
pump on the sarcoplasmic reticulum, enhancing the pump's activity. The increased activity of the sarcoplasmic reticular Ca2+ pump removes Ca2+ from the
cytoplasm and, thus, decreases the force of contraction.
(2) When norepinephrine binds to the a 2 receptor (one subtype of a-adrenergic receptors),
it activates a G protein that inhibits adenylate cyclase, thus reducing the amount of
cAMP in the cell. The G protein that inhibits the formation of cAMP is the G; protein.
(3) When norepinephrine binds to the a 1 receptor (another subtype of the a-adrenergic
receptors), it activates a G protein that, similar to its action at the muscarinic receptor,
results in the formation of IP3 and DAG.
(4) Adrenergic receptors can be distinguished by the types of drugs that activate and
inhibit them.
(a) a-Adrenergic receptors are activated preferentially by epinephrine and are blocked
by phenoxybenzamine.
(b) .B-Adrenergic receptors are activated preferentially by isoproterenol and are
blocked by propranolol.
b. Norepinephrine metabolism
(1) Biosynthesis of norepinephrine occurs in the sympathetic nerve terminals. Norepinephrine is released from synaptic vesicles by exocytosis in a manner identical to that by
which ACh is released.
(2) Inactivation of norepinephrine occurs by active transport into the nerve terminal and
diffusion out of the synaptic cleft. In contrast to ACh, enzymatic degradation is not an
important mechanism for inactivating norepinephrine.
4. The enteric nervous system is an independent component of the autonomic nervous system.
It is composed of the ganglia found within the wall of the gastrointestinal tract and is responsible
for coordinating the activity of gastrointestinal smooth muscle and gastric secretions.
a. The enteric nervous system receives synaptic input from the sympathetic and parasympathetic postganglionic fibers.
b. Many of the neurotransmitters used by the enteric nervous system have not been identified.
Among those known to be active are serotonin, the enkephalins and endorphins, somatostatin, vasoactive intestinal peptides (VI P), and the purines ATP and adenosine.

C. The central nervous system


1. Synaptic mechanisms. An enormous variety of synaptic connections occur within the CNS,
and, with few exceptions, these synapses use chemical transmitter mechanisms. In a few
locations (e.g., within the retina and olfactory bulb), synaptic transmission is accomplished by
passive electrotonic spread of current between two cells.
a. Electrical synaptic transmission between two cells occurs at specialized junctions called
gap junctions (Figure 1-19).
(1) Only 2 nm separate the pre- and postsynaptic membranes at the site of gap junctions.
(2) Gap junctions are formed by membrane bridges containing aqueous channels through
which small molecules and ions can pass from one cell to another, establishing cytoplasmic continuity.
(a) When an action potential propagating along the membrane in one cell reaches the
gap junction, an electrical current flows passively through the gap from one cell to
another.
(b) Electrical current can pass through the gap in both directions, allowing either cell
to serve as the pre- or postsynaptic cell.
(3) Although gap junctions are not common in CNS synaptic transmission, they play an
important role in coordinating muscle contraction in the heart and viscera. Gap junctions
rapidly transmit an action potential that is generated in one cell to all of the other cells
within the organ. This permits the entire tissue to act as a syncytium and contract in a
coordinated fashion.

Intracellular space

Protein bridge

Figure 1-19. A gap junction forms a

low-resistance electrical pathway be


tween two cells. Although the two
cells remain separate, they maintain
cytoplasmic continuity via a channel
within a protein bridge.

Intracellular space

b. Chemical synaptic transmission within the CNS occurs by a mechanism similar to that at
the neuromuscular junction.
(1) The synaptic transmitter is synthesized in the nerve terminal, stored in vesicles, and
released by exocytosis when an action potential invades the nerve terminal.
(2) After being released from the presynaptic terminal, the transmitter diffuses across the
synaptic cleft, binds to a postsynaptic receptor, and causes the opening of channels
through which ions can flow. Both excitatory and inhibitory receptors exist on the
postsynaptic cell.
(3) The transmitter is inactivated in one of three ways.
(a) It diffuses out of the synaptic cleft.
(b) It is actively transported into the presynaptic terminal.
(c) It is enzymatically degraded (if the transmitter is ACh).

2. Summation. The postsynaptic cell integrates the information it receives from thousands of
presynaptic terminals. This is illustrated in Figure 1-20, using the alpha motoneuron found in
the ventral horn of the spinal cord as an example. (Although synaptic junctions cover most of
the cell body and proximal dendrites, only a few of these are shown in Figure 1-20.)
Inhibitory
presynaptic
fibers --------~
Axon
-f---hillock
DENDRITE ---"k--1fExcitatory
presynaptic
fibers --.---PF""'---"l,,--"

Depolarization
of axon hillock

~
Summated
IPSP

Summated
EPSP

Figure 1-20. Inhibitory and excitatory synapses are formed on an alpha motoneuron. When the amplitude of the
summated excitatory postsynaptic potentials (EPSF') exceeds the amplitude of the summated inhibitory postsynaptic
potentials uPSF'), the axon hillock is depolarized to threshold, and an action potential is generated.

a. Postsynaptic response. Because the presynaptic membrane is only about 1-2 JL in diameter,
only a few synaptic vesicles can attach to the dense bars within the presynaptic terminal
and release their contents into the synaptic cleft when an action potential invades the nerve
terminal. As a result, the postsynaptic response is only a few millivolts in amplitude. The
postsynaptic response can be either inhibitory or excitatory.
(1) The excitatory postsynaptic potential (EPSP) is the depolarization produced by an
excitatory neurotransmitter. In order for an EPSP to depolarize an alpha motoneuron to
threshold, the EPSPs must summate.
(a) Although the synaptic channel is open for only a few milliseconds, the membrane
capacitance causes the EPSP to decay more slowly. As a result, the duration of the
EPSP is about 15 msec.
(b) Temporal summation occurs if another action potential invades the nerve terminal
before the first EPSP has disappeared. The second EPSP adds to the first, producing
a larger response. If the presynaptic nerve terminal fires frequently enough, the
EPSPs can summate sufficiently to depolarize the alpha motoneuron to threshold.
(c) Spatial summation occurs if several nerve terminals fire synchronously.
(2) Inhibitory postsynaptic response (IPSP). The IPSPs produced by an inhibitory neurotransmitter, like the EPSPs, can summate to produce a larger negative response.
b. Axon hillock. The potentials produced by the excitatory and inhibitory neurotransmitters
spread passively to the axon hillock, where the action potential is generated.
(1) The threshold for producing an action potential at the axon hillock is about -65 mY.
(2) The channel opened by the excitatory neurotransmitter allows Na+ and K+ to flow
through it and, like the end-plate potential (EPP), has a reversal potential of about
-16mV.
(3) The channel opened by the inhibitory neurotransmitter allows CI- to flow through it
and has a reversal potential of about -82 mY.
(4) If the summated EPSPs are large enough to depolarize the axon hillock to threshold, an
action potential will be generated.
(5) The summated IPSPs can prevent the axon hillock from being depolarized to threshold
by hyperpolarizing the cell.
3. Presynaptic inhibition is produced by an axo-axonic synapse (Figure 1-21).
a. The number of vesicles binding to the dense bar and releasing their contents into the synaptic
cleft depends on the amount of CaH entering the presynaptic terminal, which, in turn,
depends on the magnitude of the action potential invading the nerve terminal.
(1) The presynaptic nerve terminal of the axo-axonic synapse (neuron 1) releases a transmitter that opens CI- channels on the postsynaptic membrane of the axo-axonic synapse
(neuron 2).

~""

Gyr..... :.'.

~1?/

->-\~' .'. .

>--..- . - ' " ; , . . - - . . . ,

Figure 1-21. Diagram illustrating the


axo-axonic synapse responsible for
presynaptic inhibition. Neuron 1 releases CABA, which opens CI- channels on neuron 2. The increased CIconductance reduces the amplitude
of the action potential as it approaches the nerve terminal. Because
the action potential is smaller, less
Ca2+ enters the nerve terminal, less
transmitter is released from the nerve
terminal, and the magnitude of the
EPSP produced on the postsynaptic
membrane of neuron 3 is reduced.

(2) When an action potential invades the postsynaptic neuron of the axo-axonic synapse
(neuron 2), CI- flows into the neuron through the opened CI- channels. The flow of a
negative ion into the neuron reduces the magnitude of the overshoot.
(3) Because the overshoot is smaller, less Ca2+ enters the nerve terminal, and the amount
of neurotransmitter released by neuron 2 is diminished.
(4) The reduction in transmitter release reduces the size of the EPSP produced on the alpha
motoneuron (neuron 3).
b. In presynaptic inhibition, the excitability of the postsynaptic cell is not diminished, whereas
an IPSP reduces the effectiveness of all excitatory input to a cell. By using presynaptic
inhibition, a particular excitatory input can be inhibited without affecting the ability of other
excitatory synapses to fire the cell.

IV. MUSCLE CONTRACTION. Muscle fibers are divided into two types based on their appearance
in light micrographs. Striated muscle, which includes skeletal and cardiac muscle, is characterized
by alternating light and dark bands. Smooth muscle has no distinguishing surface features. Although
all muscle types function in a similar way, several important differences exist. In the following
discussion, the structural and contractile properties of skeletal muscle are noted first, with the major
differences in cardiac and smooth muscle noted afterward.
A. Skeletal muscle
1. Structure (Figure 1-22). Skeletal muscle fibers vary in diameter from about lOlL to 100 IL and
can be several centimeters in length.
a. Fascicles. Skeletal muscle fibers are grouped into fascicles of about 20 fibers by the
perimysium, a connective tissue sheath that is continuous with the connective tissue surrounding the entire muscle.
(1) The perimysium is continuous with the endomysium surrounding each muscle fiber.
(2) The endomysium is continuous with the sarcolemma, a glycoprotein-containing sheath
that closely envelops the true cell membrane of the muscle fiber.
(3) The tight connection between the cell membranes and the surrounding connective
tissue structures makes it possible for the force developed by the muscle fibers to be
transmitted effectively to the tendons.
b. Myofibrils. Each muscle fiber is divided into myofibrils by a tubular network called the
sarcoplasmic reticulum (see Figure 1-22 B).
(1) The myofibrils are about 1 IL in diameter and extend from one end of the muscle fiber
to the other.
(2) The myofibrils are divided into functional units, or sarcomeres, by a transverse sheet
of protein called the Z disk.
(3) The Z disks of neighboring myofibrils are lined up with each other so that in histologic
slides a Z line spans the entire width of the fiber.
c. Filaments. The myofibrils contain thick and thin filaments composed of contractile proteins.
(1) Thin filaments contain the proteins actin, tropomyosin, and troponin and are about
50 A in diameter and 1 IL in length.
(a) One end of the thin filament is attached to the Z disk, so that filaments from opposing
Z disks extend longitudinally into the center of the sarcomere toward each other.
(b) The thin filaments appear to be held in place by a thin protein that connects their
free ends.
(2) Thick filaments contain the protein myosin and are about 110 A in diameter and 1.6
IL in length.
(a) The thick filaments are interspersed between the thin filaments at the center of the
sarcomere. They are not attached to the Z disk.
(b) Projections from the thick filaments called cross-bridges extend toward the thin
filaments. Cross-bridges playa fundamental role in muscle contraction.
(3) Banding pattern. The interdigitating thick and thin filaments create the pattern of light
and dark bands that characterizes light microscopic views of skeletal muscle (see Figure
1-22Al.
(a) The dark areas in the center of the sarcomere are called A bands. These contain
the thick filaments.
(b) The light areas on either side of the Z disk are called I bands. They contain the thin
filaments.

Sarcomere

H band

M line

Myofibril

~------------SARCOMERE------------~

- - - - - 1 - - - - - A band------r----

Z line
T tubule
Terminal cisterna
Figure 1-22. High- and low-power light microscopic view of human skeletal muscle. (A) Under low power, the
alternating A and I bands can be seen. The functional unit of the muscle fiber, the sarcomere, is bounded by Z lines.
(8) Under high power, the T tubules can be seen at the junction of the A band and I band. The terminal cisternae of
the sarcoplasmic reticulum form specialized junctions with the T tubules called foot processes, or junctional feet.
(c) The thick and thin filaments overlap to some extent in the A band. The area of the

A band without any thin filaments is called the H band. At the center of the H band
is the M line. This is the region of the thick filaments that does not contain any
cross-bridges.
(d) The amount of overlap between thick and thin filaments varies with sarcomere
length and determines how much force skeletal muscle will develop when it is
stimulated. When the muscle is stretched or shortened, the thick and thin filaments
slide past each other, and the I band increases or decreases in size.
d. Tubules. Two tubular networks are present in skeletal muscle fibers (see Figure 1-22B).
(1) The transverse (T) tubule is formed as an invagination of the surface of the muscle
membrane.
(a) In mammalian skeletal muscle, the T tubules are located at the junction of the A
and I bands. (In frog muscle, the T tubules are at the Z disk.>
(b) An action potential spreading over the surface of the muscle membrane is propagated into the network of T tubules.
(2) The T tubule forms specialized contacts with the sarcoplasmic reticulum (SR), the
internal tubular structure that runs between the myofibrils.
(a) The SR has a high concentration of Ca2+, which is used to initiate muscle contraction
when the muscle is stimulated.
(b) The ends of the SR expand to form terminal cisternae (TC), which make contact
with the T tubule. Small projections, or foot processes, span the 200 Aseparating
the two tubular membranes. These foot processes contain Ca2+ channels.

2. Excitation-contraction (EC) coupling is the process by which an action potential initiates the
contractile process. EC coupling involves four steps: the propagation of the action potential
into the T tubule and release of Ca2+ from the TC, the activation of the muscle proteins by
Ca2+, the generation of tension by the muscle proteins, and the relaxation of the muscle.
a. Release of Ca 2
Depolarization of the T tubule by the action potential causes the Ca2+
channels on the foot processes to open. Ca2+ flows out of the TC and into the cytoplasm.
b. Activation of muscle proteins. For a muscle to contract, the thick and thin filaments must
interact. When the cell is at rest, this interaction is inhibited. Ca2+ removes this inhibition
by binding to troponin on the thin filament.
(1) Thin filament proteins (Figure 1-23A)
(a) The backbone of the thin filament is formed by two chains of actin molecules,
which wind around each other. Each actin molecule has a myosin binding site.
(b) When not stimulated, the myosin binding site is covered by another thin filament
protein: tropomyosin. This long molecule coils around the actin chain.
(c) The position of tropomyosin on the thin filament is controlled by another protein:
troponin. The troponin molecule contains a binding site for Ca2+. When the binding
site is occupied, troponin undergoes a conformational change that causes tropomyosin to move away from its position covering the myosin binding site on actin.
(d) Once uncovered, the binding site on actin combines with the cross-bridges from
the thick filament, and contraction begins.
(2) Thick filament protein. About 300 myosin molecules coalesce to form the thick
filament.
(a) Myosin is a complex protein consisting of a globular head, the cross-bridge, and a
long tail.
(b) Rotation can occur at two points in the myosin molecule. One, located in the tail
region, is used to rotate the molecule outward when the spacing between the thick

+.

~
Tropomyosin

Troponin

Figure 1-23. (A) The thin filament is composed of three proteins. Actin forms the backbone of the thin filament. At
rest, the tropomyosin is covering the myosin binding site on actin. When Ca2+ binds to troponin, the troponin
undergoes a conformational change, pulling tropomyosin away from the myosin binding site and allowing the
cross-bridge cycle to begin. (B) In the first step of the cycle, myosin binds to actin. (0 In the second step, the
cross-bridge bends and the thin filament slides over the thick filament. During this step, the products of hydrolysis,
adenosine diphosphate (AOP) and inorganic phosphate (Pi)' are released from the cross-bridge. (0) In the third step,
a new molecule of ATP binds to the cross-bridge and the cross-bridge detaches from the thin filament. The cross-bridge
then stands up, and a new cycle begins. Cycling continues for as long as intracellular Ca2+ concentration remains
high.

and thin filaments changes. The other, located at the junction of the head and tail,
is the site where the cross-bridge bends when generating tension.
(c) Located on the cross-bridge is the enzyme myosin-ATPase, which is used to hydrolyze ATP and, thus, provide the energy for muscle contraction.
c. Generation of tension. Tension is generated by the cycling of the cross-bridges, which
occurs after they bind to the thin filament.
(1) The first step in the cross-bridge cycle is the binding of actin and myosin (see Figure
1-23B). This occurs spontaneously, after Ca H binds to troponin, and tropomyosin
moves away from its position blocking the myosin binding site on actin.
(a) At rest, the intracellular Ca 2 + concentration is less than 10- 7 moll L.
(b) When stimulated, enough Ca H is released from the TC to raise the intracellular
Ca H concentration to 10- 5 mol I L. At this concentration, all of the muscle protein
is activated.
(2) The second step is the bending of the cross-bridge and the sliding of the thin filament
across the thick filament (see Figure 1-230.
(a) The energy used to bend the cross-bridge and generate tension is obtained from
ATP.
(b) Both the ATP molecule and its hydrolyzing enzyme ATPase are attached to the
cross-bridge.
(3) The third step is the detachment of the cross-bridge from the thin filament. This occurs
after the cross-bridge has bent (see Figure 1-23D).
(a) For detachment to occur, the products of ATP hydrolysis [i.e., adenosine diphosphate (ADP) and inorganic phosphate (Pi)] must be removed from the cross-bridge
and a new molecule of A TP put in their place.
(b) If no ATP is available, the thick and thin filaments cannot be separated. (This is the
cause of rigor mortis.)
(c) As soon as myosin separates (or, perhaps, while it is separating) from actin, the
initial steps of ATP hydrolysis occur. The product of these metabolic reactions is a
high-energy ATP intermediate: myosin. ADP . Pi'
(d) Complete dissociation of the phosphate from the ADP does not occur until after
myosin has bound to actin and completed its bending cycle.
(4) In the final step, the cross-bridge returns to its original upright position. Once there, it
can participate in another cycle. Cycling continues as long as Ca H is attached to
troponin.
d. Relaxation occurs when the Ca H is removed from the cytoplasm by Ca H pumps (Ca H ATPase) located on the SR membrane. When the intracellular Ca H concentration falls below
10- 7 mollL, troponin returns to its original conformational state, tropomyosin moves back
to cover the myosin binding site on actin, and cross-bridge cycling stops.
3. Mechanical properties. The force developed by the bending of the cross-bridge is transmitted
through the thin filament to the Z disk and then through the sarcolemma and tendinous
insertions of the muscle to the bones. The contractile properties of the muscle can be studied
in two types of mechanical conditions, isometric and isotonic contractions.
a. In isometric contractions, the muscle length remains constant during the contractile event
and the force developed during the contraction is measured.
(1) Muscle twitch. Figure 1-24 illustrates the intracellular Ca H concentration and the force
developed by a muscle during a single twitch.
(a) A single electrical stimulus is used to depolarize the muscle membrane to threshold
and produce an action potential, which causes the release of Ca H from the TC.
,10- 3

1.0

Active
state

E
~

Cl

~0.5
Ql

l:
~

~,,

,(' \\

,
,,

I
o

~10-5
I

~10-7

,--

50
Time (msec)

100

(5

S
+

C\I

ctl

Figure 1-24. When intracellular


Ca2+ concentration (right ordinate)
rises above 10- 7 mmol/L, crossbridge cycling causes an increase in
muscle force (left ordinate). The duration of cross-bridge cycling is the active state, and the resulting force development is the muscle twitch.

(b) Ca2+ initiates the cross-bridge cycle, which lasts until the Ca2+ is resequestered by
the Ca2+ pumps on the SR membrane.
(c) The period during which the intracellular Ca 2 + concentration is above resting values
and the cross-bridges are cycling is called the active state.
(2) An increased strength of electrical stimulation causes an increase in the isometric
force developed by the muscle.
(a) The increased force is the result of more muscle fibers being activated. Although
this is a predictable result, it is important to note because it is one of the two major
methods used by the motor; control system to increase the force of skeletal muscle
contraction.
(b) If the initial conditions are the same, then the force developed by each muscle fiber
during a twitch remains constant. This is because the amount of Ca2+ released from
the TC activates all of the muscle protein, and the time for the SR Ca2+ pump to
resequester the Ca2+ remains the same. However, the force developed by the
muscle can be changed by varying the length of the sarcomeres and the frequency
of stimulation.
(3) Figure 1-25 illustrates the relationship between muscle length and force development
during a muscle twitch. This is called the length-tension relationship. The figure shows
both the actual muscle length and the length of the individual sarcomeres. The sarcomere
length plays an important role in the length-tension relationship.
(a) The initial length of the muscle, called the preload, is set by a load (or force) applied
to the muscle before it is stimulated.
(i) Maximal force is obtained when the preload is set at a sarcomere length of
2.2 /.t. At this length, the overlap between thick and thin filaments is optimal,
since every cross-bridge from the thick filament is opposite an actin molecule
on the thin filament. When contraction is initiated, each cross-bridge binds to a
thin filament and contributes to the contractile force.
(ii) Increasing the preload causes a decrease in the force developed by the muscle
fiber. At preloads greater than 2.2 /.t, the overlap between thick and thin filaments
is decreased. Thus, some cross-bridges do not have actin filaments with which
to combine, and the force developed by the muscle is diminished.
(iii) It is not so obvious why decreasing the preload below 2.2 /.t causes a decrease
in force development, since every cross-bridge has an actin site with which to
combine. However, at short sarcomere lengths the thin filaments bump into each
other, making it more difficult for the muscle to develop force. Also, it is possible
that the amount of Ca2+ released by the TC is reduced as a function of sarcomere
length.
(b) Varying the preload is not an important method of varying the contractile force of
skeletal muscle. Often, the muscle length is determined by the particular motor task
being performed. However, if muscle length is not constrained by the motor activity,
more force can be obtained by holding the muscle at its optimal length (i.e., where
the sarcomere length is 2.2 /.t).

1.6~~
2.2~~~

Q)

LL

3.0~8-a

-~'-'I-'-'I-'-'I-'-'I'-~I"-'I-'-'-'I
1.4

1.8

2.2
2

2.6

3.0

3.4
3

Sarcomere length
Fiber length (cm)

~)

Figure 1-25. The length-tension relationship results from the overlap between thick and thin filaments. At a sarcomere
length of 2.2 /-L, overlap is optimal and force development is maximal. At lengths greater than 2.2 /-L, force decreases
because cross-bridge overlap is lessened. At lengths less than 2.2 /-L, force is less because the thin filaments meet at
the center of the sarcomere, causing an increased resistance to shortening.

;-10- 3
1
,

~"""-.-"-

o
I

100

200
Time (msec)

~10-5 :::
+

Active
'" state

''''Cit,

Q)
()

~
0

300

,I

'"ctl

()

'----'-10 - 7 -

400

Figure 1-26. When the duration of the active state is increased by repetitive firing of the muscle, force development
increases because there is sufficient time for the series elastic component to be stretched completely.

(4) An increased frequency of electrical stimulation also increases the force of muscle
contraction.
(a) Varying the frequency of muscle stimulation is the other major method used by the
motor control system to vary the force of muscle contraction. This is illustrated in
Figure 1-26. Note that the Ca 2 + concentration does not increase above 10- 5
mol/L but that the duration of the active state is increased due to the repetitive
release of Ca2+ from the TC.
(b) The increase in muscle force due to repetitive stimulation is called summation.
When the frequency of stimulation is rapid enough to allow the force to rise smoothly
to a maximum, the response is called a tetanus. Figure 1-27 is a simple mechanical
analog of the contractile properties of the muscle, which can be used to explain the
phenomena of summation and tetanus. In this figure, the thick and thin filaments
are represented by the contractile component (CC), and the tendons and other
compliant structures of the muscle are represented by a series elastic component
(SEC).

(i) In order for the force developed by the cross-bridges to be transmitted to the
bones, the SEC must be stretched. The more the SEC is stretched, the more force
is transmitted.
(ii) Since each cross-bridge is only 100 Along, its rotation will move the thin filament
about 75 A. At least 30 cross-bridge cycles are required to stretch the SEC
sufficiently to transmit all the force of the attached cross-bridges to the bone.
(iii) Since each cycle lasts about 1 msec in fast-twitch fibers [see XII B 1 c (1), the
cross-bridges must cycle for at least 30 msec. This cannot happen in a single
twitch because the Ca2+ is resequestered too rapidly. By repeatedly stimulating
the muscle, however, the Ca2+ remains in the cytoplasm for a longer time and
allows the SEC to be stretched sufficiently.
b. In isotonic contractions, the skeletal muscle shortens.
(1) Figure 1-28 illustrates the development of force and the change in muscle length that
occurs during an isotonic contraction.
(a) In order to shorten, the muscle must lift a weight, called the afterload, which is
applied after the muscle begins to contract.
(b) When the muscle is stimulated, the cross-bridges begin to cycle, the SEC is stretched,
and force is applied to the afterload.
(i) During the initial part of the contraction, the force is less than the afterload
and shortening does not occur. This is the isometric portion of the isotonic
contraction.
(ii) After the SEC has been sufficiently stretched, the force equals the afterload and
the muscle begins to shorten.
(iii) While the muscle is shortening, the length of the SEC remains constant and the
force remains equal to the afterload, thus the term, "isotonic" contraction.
Contractile

Series
elastic
component

Figure 1-27. The contractile properties of a muscle can


be explained using a mechanical analog consisting of a
force-generating contractile component in series with an
elastic element.

..r:

0,
c

~~

31

Q)

.....J

Q)
()

0
u..

2.5

':]
I

L
Time

100 9

50 9

100 9

50 9

Figure 1-28. During an isotonic contraction, sufficient force must be generated in order for the muscle to
shorten against the afterload on the
muscle. Once sufficient force is developed, the muscle shortens at a
constant velocity. Increasing the
afterload increases the amount of
force that must be developed before
shortening can begin and decreases
the velocity and extent of shortening. Heavy lines represent an afterload of 100 g; light lines represent an
afterload of 50 g.

(iv) The velocity of shortening also remains constant during the contraction.
(2) As shown in Figure 1-28, the magnitude of the afterload affects the characteristics of
the contraction.
(a) As the afterload increases, the duration of the isometric portion of the contraction
increases. This occurs because the SEC must stretch more to transmit the force
required to lift the greater afterload.
(b) The amount of shortening that can occur decreases as the afterload increases due
to the length-tension relationship. As the sarcomere length is reduced below 2.2 IL,
the force that can be developed decreases. At some length, the maximal force that
can be developed by the muscle becomes less than the afterload, and further
shortening is impossible. The greater the afterload, the sooner this length is reached.
(c) Finally, the velocity of shortening decreases with an increase in the afterload.
This load-velocity relationship is an important characteristic of muscle because it
indicates that the greatest velocity of shortening is generated when the afterload on
the muscle is zero. The peak velocity of shortening is an indication of the cross-bridge
cycling speed.
B. Cardiac muscle, like skeletal muscle, is striated. Although it is similar to skeletal muscle in structure
and behavior, cardiac muscle displays some important differences.
1. Structure
a. Cardiac muscles are smaller than skeletal muscles. Each fiber is about 15-20 IL wide, about
100 IL long, and only about 5 IL thick, making its shape more ribbon-like than cylindrical.
b. The striation pattern of a cardiac muscle fiber is similar to that described for a skeletal muscle
fiber.
c. Cardiac muscle also has both T tubular and SR systems. The T tubule is larger than in skeletal
muscle and is located at the Z disk rather than at the junction of the A and I bands. The SR
makes contact with the T tubule and with the cell membrane.
2. EC coupling of cardiac muscle also differs in significant ways from that of skeletal muscle.
Most importantly, the amount of Ca2+ in the cytoplasm is never enough to activate all of the
muscle protein. As a result, the force of contraction can vary with the amount of Ca2+ entering
the cell. Several mechanisms may be involved in regulating Ca2+ entry during the action
potential.
a. As in skeletal muscle, Ca2+ is released from the TC by the action potential. However, the
Ca2+ is not released directly by the depolarization accompanying the action potential.
Instead, the Ca2+ entering the cell during the cardiac action potential triggers the release of
Ca2+ from the Te. This is called Ca H -induced Ca H release.
b. The Na+-Ca2+ exchange mechanism is an important regulator of intracellular Ca2+ concentration. Under resting conditions, this exchange mechanism removes Ca2+ from the cell.
However, when the cell depolarizes, the amount of Ca2+ removed is reduced, or the
exchange mechanism may even reverse direction and actually add Ca2+ to the cytoplasm.

3. Mechanical properties of cardiac muscle differ from those of skeletal muscle due to the
duration of the action potential and the ability of cardiac muscle to regulate the amount of
Ca 2 + entering the cell.
a. The action potential and the period of time during which Ca2+ remains in the cytoplasm
(the active state) are nearly equal in duration; thus, summation and tetanus are not possible.
This is not a physiologic disadvantage for the heart, which must relax after each beat so
blood can enter it.
b. The sarcomere length in a cardiac muscle prior to contraction (the preload) depends on
how much blood has entered the heart. Because this amount is under physiologic control,
it is an important regulator of the force of cardiac muscle contraction.
c. The force of contraction can vary at a given sarcomere length if the amount of Ca2+ entering
the cell is changed. This also is under physiologic control and, thus, is an important regulator
of cardiac muscle contractile force.

C. Smooth muscle differs greatly from striated muscle in appearance and in its mechanism of EC
coupling.
1. Structure
a. Smooth muscles are thinner than cardiac muscles. They are about 5-10 J.t wide and 10500 J.t long.
b. Smooth muscles are not divided into sarcomeres with interdigitating thick and thin filaments,
so they lack striations.
(1) Smooth muscle proteins are contained in thick and thin filaments, but these filaments
are not organized into sarcomeres. Instead they are dispersed throughout the cell.
(2) The thin filaments are attached to dense bodies. Some of these bodies are anchored
to the cell membrane, but most are floating within the cytoplasm.
(3) Cross-bridges extend from the thick filaments, and the cross-bridge cycle appears similar
to that of striated muscle.
c. T tubules are absent and unnecessary in smooth muscle, because the cell is small enough for
a stimulus present on the cell surface to be effective in activating the contractile machinery.
2. EC coupling. Smooth muscle differs most from striated muscle in its mechanism of activation. In
striated muscle, the cross-bridge is prevented from binding to actin by the troponin-tropomyosin
regulatory complex. In smooth muscle, there is no troponin-tropomyosin complex inhibiting
cross-bridge cycling. However, the myosin cross-bridges cannot bind to actin until they are
activated, and this activation requires that one of the light-chain proteins on the myosin head
be phosphorylated.
a. Myosin phosphorylation is catalyzed by myosin light-chain kinase (MLCK), which is
inactive at rest. MLCK is activated by combining with another enzyme, calmodulin, which
cannot occur until calmodulin has combined with Ca2+. Thus, the role of Ca2+ in smooth
muscle is to initiate a sequence of reactions ending in the phosphorylation of myosin.
b. Another enzyme, myosin phosphatase, removes the phosphate from the myosin. The
amount of myosin active at any time depends on the relative activity of the MLCK and the
phosphatase. Relaxation occurs when the intracellular Ca2+ concentration falls and the
M LCK becomes inactive. At this point, the phosphatase removes phosphate from the myosin
light-chain protein, and cross-bridge cycling ceases.
3. Mechanical properties of smooth muscle differ from those of striated muscle.
a. The speed of shortening, or the rate of cross-bridge cycling, is dependent on the phosphorylation of the myosin light chain. When the light chains are dephosphorylated by the
phosphatase enzyme, the speed of shortening decreases.
b. However, the dephosphorylated cross-bridges remain attached to actin and are able to
sustain the tone developed by the muscle early in its contraction. These attached, but
dephosphorylated, cross-bridges are called latch-bridges. Latch-bridges provide smooth
muscle with the ability to maintain tone with little energy consumption. Since the latchbridges do not cycle, or cycle very slowly, they do not use much ATP.

v.

GENERAL SENSORY MECHANISMS


A. Classification of receptors. The function of receptors is to transform (transduce) stimulus energy
into electrical energy and to transmit information about the nature of the stimulus to the CNS.
Receptors can be classified on the basis of the following criteria.

1. Source of stimulus
a. Exteroceptors, such as the eye, ear, and taste receptors, receive stimuli from outside the
body.
b. Enteroceptors, such as the chemoreceptors for measuring blood gases and the baroreceptors for measuring blood pressure, receive stimuli from within the body.
2. Type of stimulus energy
a. Mechanical receptors detect skin deformation and sounds.
b. Thermal receptors detect environmental temperatures.
c. Photoreceptors detect light.
d. Chemoreceptors detect substances that produce the sensations of smell and taste.
3. Type of sensation. Receptors are classified as touch, heat, cold, pain, light, sound, taste, and
olfactory receptors.
4. Rate of adaptation
a. Slowly adapting receptors (tonic or static receptors) fire action potentials continuously
during stimulus application.
b. Rapidly adapting receptors (phasic or dynamic receptors) fire action potentials at a decreasing rate during stimulus application.

s.

Neuron type
a. Classification. Neurons are classified on the basis of size and propagation velocity.
(1) Size. Neurons fall into one of four groups (I-IV) depending on size.
(a) Group I fibers are 12-20 /-L in diameter, group II fibers are 6-12 /-L, group III fibers
are 1-6 /-L, and group IV fibers are less than 1 /-L in diameter. All but group IV fibers
are myelinated.
(b) Figure 1-29A is a histogram showing the size distribution of the neurons within a
nerve trunk. The peaks in the distribution correspond to each of the three groups,
indicating that they are not arbitrary divisions.
(2) Propagation velocity. Figure 1-29B illustrates the recording of a compound action
potential from a nerve trunk. The compound action potential is the summed activity

A
C/l

iii 15

.D

:;:::

"0

a>

(;j

~ 10
>-

'0

C
a>
u

iii
Q.

I.

II

III

I-

11 13 15 17 19 21
II

II

Fiber diameter

v.)

:;:-

.fA alpha

.Sa>

"0

:ec.
E

Time (msec)

Figure 1-29. (A) Bar graph illustrating the size distribution of myelinated fibers within a nerve. The three peaks
in the distribution correspond to the three functional
groups of fibers, I, II, and III. (8) The compound action
potential recorded from a nerve bundle has three peaks,
which correspond to the functional grouping of myelinated nerve fibers according to conduction velocity (i.e.,
A alpha, A gamma, and A delta fibers).

of all the fibers within the nerve bundle. The action potentials are generated at one end
of the nerve trunk and propagated into the other end, where the recording electrodes
are placed.
(a) The fibers with the fastest conduction velocities reach the recording electrodes first
and are responsible for the first wave of the compound action potential. Subsequent
peaks occur when the fibers with slower conduction velocities reach the recording
electrodes.
(b) The fibers responsible for generating the first peak are called A alpha fibers, those
producing the second peak are called A gamma fibers, and those producing the
final peak are called A delta fibers. All are myelinated fibers. The slowest conducting
fibers, called C fibers, are unmyelinated (not shown in Figure 1-29).
b. Functional distribution. Certain important sensory and motor fibers are identified with
particular neuron types.
(1) Muscle spindle afferent neurons belong to the group I and group II fibers.
(2) Pain and temperature afferent neurons belong to the A delta fibers, if myelinated, and
to the C fibers, if unmyelinated.
(3) Alpha motoneurons are so-called because they are part of the A alpha group of fibers.
(4) Similarly, gamma motoneurons are part of the A gamma fiber group.
B. Transduction of stimulus energy by receptors. Each receptor is specialized to receive a particular
type of stimulus called an adequate, or appropriate, stimulus. The receptor is exquisitely sensitive
to its adequate stimulus. For example, the retina can detect the presence of a single photon of
light. However, the receptor can respond to other forms 6f stimulus energy if the stimulus is strong
enough. For example, a mechanical stimulus such as rubbing the eyes can produce a sensation of
light. The functional components of transduction in a simple mechanical receptor are illustrated
in Figure 1-30 and described here.

1. The transducer region of this receptor is at the end of the nerve terminal. In contrast, in the
eye, ear, and tongue, the transducer region is on a receptor cell that is separate from the afferent
neuron, and the two cells communicate by synaptic transmission.
a. The transducer region of the receptor is responsible for converting the stimulus energy into
an electrical signal called the receptor, or generator, potential.
b. Each receptor has a particular mechanism for producing a receptor potential. In the mechanical receptor, the stimulus produces a receptor potential by deforming the nerve terminal.
(1) The deformation opens channels that are permeable to Na+ and K+ and causes the
membrane to depolarize.
(2) As illustrated in Figure 1-30, the receptor potential follows the time-course of the
mechanical stimulus.
TRANSDUCER
SPIKE GENERATOR

STIMULUS

Amplitude

,k;,

10

Stimulus

d'~mat;o, 51 ~
-SOJ
Receptor potential

Membrane
potential
(mV)

Membrane
potential
(mV)

-80

+3g~
- 80

IITiT] IIII

J
I

100

300
500Time (msec)

Figure 1-30. When a stimulus is applied to a sensory nerve ending, a


generator potential results. The magnitude of the generator potential and
the frequency of action potential discharge are proportional to the magnitude of the stimulus.

2. The spike generator region of the receptor is responsible for converting the receptor potential
into a train of action potentials.
a. Figure 1-30 illustrates the repetitive discharge of action potentials by the receptor axon in
response to a receptor potential.
b. The receptor potential spreads passively from the transducer region to the spike generator
region. If the spike generator is depolarized to threshold, an action potential is generated.
c. At the end of the action potential, the receptor potential again causes the spike generator
membrane to depolarize toward threshold. If threshold is reached again, another action
potential is generated.

C. Adaptation. The most obvious function of adaptation is to decrease the amount of sensory
information reaching the brain. However, the brain stem mechanisms responsible for consciousness and attention are capable of keeping the information flow to the brain within tolerable limits
so that sensory adaptation is not required for this purpose. The major role of sensory adaptation
is to permit the receptor to encode information about the rate of stimulus application.

1. Adaptation occurs by two major mechanisms.


a. In one, the transducer mechanism fails to maintain a receptor potential despite continued
stimulus application.
b. In the other, the spike generator fails to sustain a train of action potentials despite the
presence of a receptor potential. This occurs because the excitability of the spike generator
membrane decreases. Although the mechanism of this decreased excitability is not known,
it may be due to an increase in:
(1) The membrane conductance to K+
(2) The activity of an electrogenic Na+ -K+ pump
(3) The inactivation of Na+ channels
2. The discharge rate in a phasic receptor increases as the rate of stimulus application increases.
a. When a stimulus is applied too slowly, adaptation of the spike generator occurs before an
action potential can be generated.
b. When a stimulus is applied rapidly enough, excitation exceeds adaptation, and a steady
discharge of action potentials occurs.
D. Sensory coding. The intensity, location, and quality of a stimulus are encoded by the sensory
system.

1. Intensity is encoded by the firing frequency of a sensory neuron.


a. As the intensity of a stimulus is increased, the magnitude of the receptor potential is increased. This relationship is expressed by the Steven's power law function

V = k x I"
where V = the magnitude of the receptor potential; k = a constant; I = the intensity of the
stimulus; and n = a constant. Figure 1-31 is a plot of this relationship in which the maximal
value of the receptor potential is about 50 mY.
(1) Assuming that the sensory system requires a receptor potential of at least 1 mV to detect
the presence of a stimulus, the maximal stimulus that can be detected is 50 times as
great as the minimally detectable stimulus.
(2) The power law relationship between stimulus intensity and receptor potential magnitude
enables this relatively small range of receptor potential amplitudes (50 to 1) to encode
a range of stimulus intensities that is greater than 1 million to 1.
b. The graph in Figure 1-31 also illustrates that sensory perception and the firing frequency of
a sensory neuron are related to stimulus intensity by the Steven's power law. This substantiates the concept that stimulus intensity is encoded by the rate of sensory nerve discharge.
2. Stimulus location is encoded primarily by the location of the sensory projection to the cerebral
cortex. This mechanism of encoding, called topographic representation, is used by the visual
and somatosensory systems to localize the point of stimulus application. The auditory system
uses a different mechanism (see VII D 2).
a. Each sensory neuron receives information from a particular sensory area called its receptive
field. The smaller the receptive field, the more precise the encoding of stimulus localization.
(1) For example, the receptive fields of the fovea are the smallest within the eye.
(2) Similarly, the receptive fields within the fingertips are much smaller than those on the
hands and back, where it is difficult to localize precisely the point of a mechanical
stimulus.

100

100

50

X
ctS

15

0>

~
~

.s35

>.

"0

.2

S.

0>

>.

15..
~

0>

c.

15
O>
"0

'c

01

ctS

50

:;-

~0>

c 50
0>

:J

c-

01

'':::

u:::

:2

0>

(5

C.

Receptor potential = 0.5 x 10.5

0>

~15

a:

11
5

0 ----0
I

1 1

1000

1 500
100

5000

10,000

Stimulus intensity

Figure 1-31. The proportionality between generator potential and stimulus can be expressed as a power function.
This same relationship can be used to describe the proportionality between stimulus magnitude and firing frequency
and between the magnitude of the stimulus and its perceived intensity. I = intensity.

b. Stimulus localization can be made more precise by lateral inhibition. This is demonstrated
in Figure 1-32, which shows three overlapping receptive fields in the skin.
(1) Two stimuli are applied to the skin, and the sensory task is to indicate whether one or
two stimuli are present. The closest that two stimuli can be to each other and still be
distinguished as separate stimuli is called the two-point threshold_
(2) Without lateral inhibition (see Figure 1-32A), the two stimuli are recognized as separate
only if they are placed in receptive fields that are separated from each other by a
nonstimulated receptive field.
(3) Lateral inhibition (see Figure 1-32B) can decrease the two-point threshold by reducing
the discharge of the neuron innervating the receptive field in the center and, thus,
making it apparent to the CNS that two stimuli are present.
3. Stimulus quality is encoded by a variety of mechanisms.
a. The simplest mechanism uses a labeled line, in which the stimulus is encoded by the
particular neural pathway that is stimulated.
(1) The basic sensory modalities are encoded in this way. Thus, the sensation of touch is
elicited whether the receptors on the skin are excited by mechanical deformation or by
electrical stimulation. Similarly, light always is evoked no matter how the retina is
activated, and sound always results from stimulation of the cochlea.
(2) The same type of sensation results no matter where along the sensory pathway the
stimulus is applied. For example, stimulating electrodes placed on the visual cortex evoke
the sensation of light, while seizures within the olfactory cortex produce sensations of
smell.
b. A more complex mechanism of coding uses the pattern of activity within the neural
pathway that is carrying information to the brain.
(1) In temporal pattern coding, the same neuron can carry two different types of sensory
information depending on its pattern of activity. For example, cutaneous cold receptors
indicate temperatures below 30 C by firing in bursts and temperatures above 30 C by
firing without bursts.
(2) In spatial pattern coding, the activity of several neurons is required to elicit a sensation.
For example, three neurons may be required to encode different taste sensations. A
sour taste may result if all three neurons are activated, while a salty taste may result if
only two neurons fire.
c. The most sophisticated mechanism of sensory coding uses feature detectors. These are
neurons within the brain that integrate information from a variety of sensory fibers and fire
to indicate the presence of a complex stimulus.

Receptive fields

Stimulus
probes

.1 1 1 I 1 I I I

.1 I 1 I 1 I I I

.1 1 I 1 1 1 I I

Figure 1-32. In order for two distinct


stimuli to be perceived, they must
stimulate receptive fields separated
by an unstimulated receptive field. (A)
Without lateral inhibition, the stimuli
produce equal amounts of discharge
in all three neurons. (8) In the presence of lateral inhibition, however,
the neuron with the receptive field in
the center is presynaptically inhibited
by collaterals from the neurons with
receptive fields located laterally. As a
result, the receptive field in the center
does not fire, and two stimuli are perceived.

(1) For example, the location of an object in space can be encoded by cortical cells
receiving information from a single eye. However, special feature detectors receiving
information from both eyes are required to specify the depth of an object in space.
(2) Similarly, the location of a sound in space requires integration of information from both
ears by feature detectors within the brain stem.

VI. CUTANEOUS SENSATION. About 1 million sensory nerve fibers innervate the skin. Most of
these are unmyelinated nerve fibers that are responsible for crude somatosensory mechanical sensation. Although far fewer in number, the large myelinated (group II) sensory fibers encode the important
sensory qualities of touch, vibration, and pressure. The sensations of temperature and pain are
encoded by small myelinated (group III, A delta) fibers and unmyelinated (group IV, C) fibers.
A. Mechanical sensation. The sensation produced by mechanical stimulation of the skin can be
divided into three submodalities: vibration, touch, and pressure. Each of these sensations is encoded by a different set of mechanical receptors.
1. Vibration is encoded by the pacinian corpuscle (Figure 1-33A).
a. The pacinian corpuscle is a rapidly adapting encapsulated receptor.
(1) Its capsule is an onion-like lamellar structure about 1 mm in diameter.
(2) A single, large (group II) sensory axon enters the capsule.
b. When a mechanical stimulus deforms the outer lamellae of the capsule, the deformation is
transmitted through the capsule to the nerve terminal.
(1) The deformation of the nerve terminal increases the membrane permeability to Na+
and K+, producing a depolarizing receptor potential.

A. Pacinian corpuscle

B. Ruffini's corpuscle

C. Meissner's corpuscle

D. Merkel's disk

Figure 1-33. The encapsulated organs of the skin all are innervated by a single, large afferent fiber. (A) The pacinian
corpuscle is a rapidly adapting receptor that encodes vibration. (B) The Ruffini's corpuscle is a slowly adapting receptor
that encodes pressure. (0 The Meissner's corpuscle is a rapidly adapting receptor responsible for detecting the speed
of stimulus application. (0) The Merkel's disk is a slowly adapting receptor capable of encoding the location of a
stimulus. Its receptor cells communicate with the primary afferent neurons by synaptic transmission.
(2) The size of the receptor potential increases in proportion to the magnitude of the
deformation. However, because the pacinian corpuscle is a very rapidly adapting receptor, only a few action potentials will be generated, regardless of stimulus intensity.
c. Adaptation occurs because the deformation of the nerve terminal is not maintained during
continuous stimulus application.
(1) Only rapid deformations are transmitted effectively to the core of the capsule.
(2) If the stimulus is applied slowly or is left in place, the inner lamellae become rearranged
so that they no longer deform the nerve terminal. This is an example of adaptation
caused by a failure to maintain the receptor potential.
d. A steady discharge of the pacinian corpuscle can be generated by a vibratory stimulus.
(1) Each time the stimulus is removed and reapplied, the pacinian corpuscle discharges
another action potential.
(2) The frequency of discharge by the pacinian corpuscle equals the frequency of a vibratory
stimulus in the range of 50-500 Hz (cycles/sec).
(a) This is about the same range of frequencies identifiable by humans, indicating that
the frequency of firing is encoding the frequency, not the intensity, of vibration.
(b) Intensity is encoded by the number of action potentials generated by each deformation (limited to two or three) and the number of pacinian corpuscles responding to
the vibration.
2. Pressure is encoded by the Ruffini's corpuscle (see Figure 1-336).
a. The Ruffini's corpuscle also is encapsulated. Its capsule is a liquid-filled collagen structure,
in which a single group II sensory axon terminates. Collagen strands within the capsule make
contact with the nerve fiber and the overlying skin. Thus, any deformation or stretch of the
skin causes the nerve terminal to depolarize and generate action potentials.
b. The Ruffini's corpuscle is a slowly adapting receptor. The train of action potentials continues
for as long as the stimulus is present.
c. The magnitude of the stimulus is encoded by the firing frequency of the Ruffini's corpuscle.
3. Touch. Both the Meissner's corpuscle and the Merkel's disk contribute to the encoding of
touch.

a. The Meissner's corpuscle is a rapidly adapting encapsulated receptor that receives a single
group II afferent nerve fiber (see Figure 1-330.
(1) Its frequency of firing is proportional to the rate of stimulus application; thus, the
Meissner's corpuscle encodes the velocity of the stimulus application.
(2) Rapid deformation occurs when the skin is jabbed quickly with a probe and when the
fingers move over a rough object. This use of velocity information (i.e., identification of
the contour of objects palpated by the fingers and hands) is especially important to blind
individuals reading braille.
b. The Merkel's disk is a unique type of cutaneous receptor because the transducer is not on
the nerve terminal but on the epithelial cells that make up the disk (see Figure 1-330).
(1) The epithelial sensory cells form synaptic connections with branches of a single
group II afferent fiber.
(2) The disk is about 0.25 mm in diameter and can be stimulated only if the stimulus is
applied directly to the disk. The small receptive field of the Merkel's disk makes it an
ideal receptor to encode information about the location of the stimulus.
B. Temperature sensation is encoded by thermoreceptors located on the free endings of small
myelinated (A delta) and unmyelinated (0 fibers. Separate receptors with discrete receptive fields
exist for encoding warm and cold sensations. Warm fibers respond only when a warm object is
applied to them, and cold fibers respond only to cool objects. Warm fibers are not as numerous
as cold fibers and, almost exclusively, are unmyelinated.

1. Warm fibers
a. Figure 1-34 shows the static firing rate of a typical warm fiber as a function of skin temperature. (The firing rates are measured after the skin has adapted to the ambient temperature.)
Warm fibers begin to respond at a skin temperature of 30 C, reach a maximal response at
about 45 C, and cease activity at about 47" C.
b. A warm fiber always increases its firing rate phasically when the skin is warmed and
phasically decreases it when the skin is cooled. Soon after the initial response to the change
in temperature, the fiber adapts its firing rate to the steady level indicated on the graph in
Figure 1-34.
2. Cold fibers. The response of cold fibers is analogous to that of the warm fibers (see Figure
1-34).
a. Cold fibers begin to display a static discharge at skin temperatures of about 15 C, reach a
maximal response at about 25 C, and cease firing at about 42 C.
b. If the skin temperature is raised above 45 C, the cold fibers begin to fire again. This is the
threshold for activating pain fibers. Thus, if the skin is heated to a temperature above 45
C, the sensation of pain is accompanied by a feeling of coolness. This is referred to as
paradoxical cold.
A

B
Skin temperature

30

Ol

25

-----------"
Cold fiber

~u

'0 3120

~~

~ g, 15
5-~
Q)
10

111111 I I 111111111 \11111 \ I

U:

111111

Warm fiber
5
I
10

20

30

40

50

111111111

111111111111111111111 III
Time

Static skin temperature (0 C)


Figure 1-34. (A) Graph illustrating the tonic level of firing in both warm and cold fibers as a function of temperature.
Spike trains illustrating the dynamic responses of warm and cold fibers to a change in temperature. When the
temperature decreases, cold fibers increase their rate of firing and then adapt to the firing rate indicated by the graph.
Similarly, when the temperature increases, warm fibers phasically increase their firing rate before adapting to the rate
indicated by the graph.
(8)

c. Cold fibers always increase their rate of firing transiently when the skin temperature is cooled
and decrease it when the skin temperature is warmed. Also, like warm fibers, cold fibers
quickly adapt their firing rate to the static levels indicated in Figure 1-34.
3. Sensory coding by thermoreceptors is complicated by the fact that the hypothalamic thermoreceptors that monitor the body's core temperature also contribute to the perception of the
thermal environment.
a. At skin temperatures within the neutral or comfort zone (31-36 C), complete perceptual
adaptation occurs (i.e., awareness of temperature disappears). Persistent warmth and coolness are felt at temperatures above and below this zone. Temperatures below 25 Care
described as unpleasantly cold, while those below 15 C and above 45 C are painful. This
is due to stimulation of pain receptors and not to the stimulation of thermoreceptors.
b. The skin is most sensitive to temperature changes when the skin temperature is 31-36 C
(the neutral zone). At these temperatures, changes of less than 0.5 C are detected easily.
(1) Transient temperature changes can be encoded by labeled-line mechanisms because
cold fibers always increase their firing rate when the skin is cooled, and warm fibers
always increase their firing rate when the skin is warmed.
(2) Coding is more difficult under static conditions, however, because cold fibers have the
same firing rate at temperatures above and below 30 C, and warm fibers have the same
firing rate at temperatures above and below 37 C. Temperatures that produce the same
firing rates can be discriminated by two coding mechanisms.
(a) Spatial coding can be used since at temperatures between 30 and 42 C both cold
and warm fibers are firing, at temperatures below 30 C only cold fibers are firing,
and at temperatures above 42 C only warm fibers are active.
(b) Temporal coding can be used by cold fibers because action potentials fire in bursts
at temperatures below 30 C but not at temperatures above 30 C.

C. Pain sensation is different from other sensations because its purpose is not to inform the brain
about the quality of a stimulus but rather to indicate that the stimulus is physically damaging.
Although unpleasant, pain is a useful sensation if it leads to removal of the damaging stimulus.
However, the sensation of pain can endure long after the stimulus is removed and the injury is
healed. This sort of chronic pain is an extremely debilitating condition that is very difficult to treat.
Much more information is required about the physiologic and psychological mechanisms of pain
before its elimination becomes a routine part of medical practice.
1. The peripheral mechanisms of pain sensation are reasonably well understood.
a. The receptors for pain, which are called nociceptors to indicate that they respond to
noxious stimuli, are on the free nerve endings of small myelinated (A delta) and unmyelinated
(C) fibers.
(1) Nociceptors are specific for painful stimuli, responding to damaging or potentially damaging mechanical, chemical, and thermal stimuli. Cutaneous receptors that respond to
non painful levels of these stimuli do not elicit pain sensations no matter how intense
the stimulus.
(2) Although the adequate stimulus for nociceptors is not known, it is assumed that a
chemical such as histamine or bradykinin is released from cells damaged by the pain
stimulus and that the chemical substance activates the nociceptors.
b. Two types of pain sensation result from the application of a strong, noxious stimulus to the
skin.
(1) Fast, or initial, pain is a discrete, well-localized, pin-prick sensation that results from
activating the nociceptors on the A delta fibers.
(2) Slow, or delayed, pain is a poorly localized, dull, burning sensation that results from
activating the nociceptors on the C fibers.
c. The two types of pain sensation use different pathways to reach the centers of consciousness
in the brain.
(1) Action potentials that are propagated by the fast pain fibers travel faster and, thus, reach
the brain before those conducted by the slow pain fibers. The sensory fibers for fast
pain have small receptive fields, travel to the cortex through the spinothalamic tract,
and are topographically represented on the cortex-all factors that account for the
ability of these fibers to encode the location of the stimulus producing the fast pain.
(2) Slow pain, which has a more diffuse pathway, travels to the brain through the
spinoreticulo-thalamic system. Collaterals of this system pass through the reticular formation to activate fiber tracts that produce the emotional perceptions accompanying

pain sensations. These pathways account for the intense unpleasantness associated with
slow pain.
d. The two types of pain sensation elicit different reflexes.
(1) Fast pain evokes a withdrawal reflex (see XI A) and a sympathetic response including
an increase in blood pressure and a mobilization of body energy supplies.
(2) Slow pain produces nausea, profuse sweating, a lowering of blood pressure, and a
generalized reduction in skeletal muscle tone. (Pain sensations originating in the muscles,
blood vessels, and viscera produce similar reflexes.)
2. The central mechanisms of pain sensation are not well known.
a. Generally it is assumed that pain sensation is,conveyed to the CNS through the anterolateral
quadrant; however, surgical section of the cord through this area is not very successful in
relieving chronic pain.
(1) Failure to relieve pain with this procedure may result from the existence of parallel pain
pathways outside the anterolateral tracts.
(2) Alternatively, chronic pain may result from the spontaneous activity of pain centers
within the CNS.
(a) Reverberating circuits that develop because of continuous pain input may fail to
stop firing when the input is removed.
(b) Denervation supersensitivity may develop in pain centers subsequent to the removal
of pain fiber input. (Denervation supersensitivity is an increased sensitivity to circulating neurotransmitters, which results when the normal synaptic input is removed
from a neuron.)
b. Treatment of chronic pain is based on attempts to remove the pain area within the brain
by surgery or to reduce the activity of the pain pathways by activating inhibitory pathways
projecting to the pain areas.
3. Referred pain
a. Pain originating in visceral organs is referred to sites on the skin. These sites are innervated
by nerves arising from the same spinal segment as the nerves innervating the visceral organs.
Because of this anatomic relationship, diagnosis of visceral disease can be made based on
the location of the referred pain. For example, ischemic heart pain is referred to the chest
and the inside of the arm.
b. Most likely, referred pain occurs because the visceral and somatic pain fibers share a
common pathway to the brain. Since the skin is topographically mapped and the viscera
are not, the pain is identified as originating on the skin and not within the viscera.
4. Projected pain. Referred pain should not be confused with projected pain, which occurs as a
result of directly stimulating fibers within a pain pathway.
a. Because a labeled-line mechanism is used to encode the location of the pain, stimulation
anywhere along the pathway will result in the same perception. For example, striking the
elbow causes pain to be projected to the hand.
b. Amputees often have sensations that appear to come from the severed limb. This is known
as phantom limb sensation and presumably results from activation of the sensory pathway
either at the site of amputation or within the CNS. Occasionally, the phantom sensation is
one of pain, but with no obvious source of stimulation, it is difficult to eliminate the pain.

VII. AUDITION
A. Physics of sound
1. Sound waves are produced by vibrating objects that cause alternating phases of compression
and rarefaction in the medium through which the sound is transmitted.
a. In air, sound travels at about 330 m/sec (1100 ft/sec or 700 mi/hr).
b. In water, sound travels much faster, at about 1500 m/sec.
2. Characteristics of sound waves
a. The frequency of sound is measured in hertz (Hz).
(1) The range for human hearing is about 20-20,000 Hz.
(2) The human voice produces sounds of about 1000-3000 Hz when speaking.
b. The amplitude or intensity of sound is measured using a logarithmic scale. The unit of
intensity is the decibel (db).

(1) Decibels are measured using the formula


db

20 log Psound
PSPL

where db = the number of decibels; Psound = the pressure of the sound stimulus; and
PSPL = the sound pressure level (SPL) at the threshold for human hearing.
(a) Thus, sound intensities are measured on a ratio scale using a subjective intensity
(threshold) as a base.
(b) The actual SPL intensity is 0.0002 dynes/ cm 2
(2) According to the formula for calculating sound pressures in decibels, a stimulus that has
a pressure 10 times as great as another will be 20 db greater in intensity than the other
stimulus.
(a) The sound pressures used during normal conversation are about 1000 times as great
as threshold, or 60 db. (This value can be confirmed using'the above equation.)
(b) An airplane produces a sound pressure of about 100,000 times that of threshold,
or 100 db.
(c) Sound pressures above 140 db (10 7 times threshold) are painful and damaging to
the auditory receptors.

B. Structure of the ear. The ear consists of three divisions: the external ear, the middle ear, and the
inner ear (Figure 1-35).

1. The external ear consists of two parts that function to direct sound waves into the auditory
apparatus.

a. In lower animals, the cartilaginous pinna (auricle) can be moved by muscular action in the
direction of a sound source to collect sound waves for the receptor organ. In humans, these
muscles have little action. By transforming the sound field, however, the convoluted pinna
does help identify sound sources. For example, the pinna aids in distinguishing a sound
source placed in front of the head from one placed behind the head.
b. The external auditory canal extends from the pinna to the tympanic membrane of the
middle ear. Its outer portion is cartilaginous and its inner portion is osseous.

2. The middle ear (tympanic cavity) is an air-filled cavity within the temporal bone. It contains
the tympanic membrane (eardrum) and the auditory ossicles (ossicular chain). Sound stimuli
pass through the pinna and strike the tympanic membrane, causing it to vibrate. The major
function of the ossicular chain is to convey the sound stimulus from the tympanic membrane
to the inner ear. Its presence is essential for normal hearing.

Oval window

Figure 1-35. The major anatomic components of the human ear. The bony labyrinth consists of the sac-like otolith
organs (i.e., the saccule and utricle) and three semicircular canals: the horizontal (H), anterior (A), and posterior (P)
canals.

a. Components of the ossicular chain


(1) Malleus. This small bone resembles a mallet. The manubrium (handle) of the malleus
is connected to the inner surface of the tympanic membrane.
(2) Incus. This bone, which resembles an anvil, articulates with the head of the malleus.
(3) Stapes. This ossicle resembles a stirrup. The head of the stapes articulates with the
incus, and the oval footplate contacts the membrane of the oval window of the
cochlea.
b. Impedance matching. To reach the auditory receptors, the sound must be transferred from
the air-filled middle ear to the fluid-filled inner ear.
(1) Effective transfer of sound energy from air to water is difficult because most of the sound
is reflected. This is due to the different mechanical (i.e., elastic, resistive, and inertial)
properties of the two media and is described as an impedance mismatching.
(2) The middle ear functions as an impedance matching device primarily by amplifying the
sound pressure.
(a) Most of this amplification occurs because the area of the tympanic membrane
(55 mm 2 ) is about 17 times as great as the stapes-oval window surface area. This
concentrates the same force over a smaller area and, thus, amplifies the pressure.
(b) A small additional amount of amplification is obtained by the mechanical advantage
that is due to the leverage of the middle ear bones.
(c) Together these effects increase the sound pressure by 22 times (27 db).
c. Minimum audibility curve (Figure 1-36). The middle ear does not amplify all sounds
equally well.
(1) Amplification is greatest for sounds between 2000 and 5000 Hz. These are the frequencies used for speech.
(2) Sounds below 20 Hz or above 20,000 Hz are not amplified at all.
(3) The ability of the middle ear to amplify some sounds better than others accounts for
the shape of the minimum audibility curve.
d. Middle ear muscles
(1) Structure and innervation
(a) Tensor tympani. This elongated muscle inserts on the manubrium of the malleus
and is innervated by the trigeminal nerve (cranial nerve V).
(b) Stapedius. This small muscle inserts on the neck of the stapes and is innervated by
the facial nerve (cranial nerve VII).
(2) Action. These muscles contract reflexly in response to intense sounds. They act on the
mobility and transmission properties of the ossicular chain to reduce the pressure of
sounds reaching the inner ear.
(3) Functional importance. Although the middle ear muscles may act to prevent receptors
from being damaged by high-intensity sounds, contraction occurs too long after the
stimulus to provide much protection. Muscle contraction also occurs just prior to
vocalization and chewing, which suggests that the middle ear muscles may act to reduce
the intensity of the sounds produced by these activities.

100

50

Figure 1-36. The minimum audibil-

o
.02

.05

.1

.2

.5

Frequency of sound (kHz)

10

20

ity curve traces the threshold for


hearing at different frequencies in the
human hearing range. Maximum sensitivity occurs at about 4 kHz. The
actual threshold at this frequency is
0.0002 dynes/cm 2 or 0 decibels.

3. The inner ear consists of two components. The bony labyrinth is a network of cavities within
the petrous part of the temporal bone. The bony labyrinth consists of three divisions: the
semicircular canals, the cochlea, and the vestibule. Within the bony labyrinth is a series
of interconnected membranous ducts called the membranous labyrinth. The membranous
labyrinth is surrounded by a fluid called perilymph, which is rich in Na+. The fluid within the
membranous ducts is endolymph, which has a high concentration of K+ .
a. The cochlea is a spirally arranged tube (two and one-half turns in humans) that functions
as the auditory analyzer of the ear. Figure 1-37 depicts the cochlea as an uncoiled, linear
structure. The cochlea contains the following components.
(1) The cochlear duct is an elongated endolymph-filled inner tube-like structure that lies
within the coiled length of the cochlea. The open space within this membranous duct
is the scala media. Four membranes enclose the scala media.
(a) The spiral ligament is a thickening of the endosteum of the cochlea, which forms
the peripheral edge of the cochlear duct.
(b) Reissner's membrane is an extremely delicate membrane that forms the roof of
the cochlear duct. It separates the scala media from the scala vestibuli.
(c) The limbus forms the central, narrow edge of the cochlear duct.
(d) The basilar membrane represents the floor of the cochlear duct and separates the
scala media from the scala tympani. It also is the seat of the organ of Corti. The
basilar membrane is broader at the apex (near the helicotrema) than it is at the
base (near the oval window) and is 100 times more stiff at the base than it is at the
apex.
(2) The scala vestibuli is the space above the Reissner's membrane.
(3) The scala tympani, the space below the basilar membrane, is continuous with the scala
vestibuli at the very apex of the cochlear duct. This junction is termed the helicotrema.
The scala vestibuli and scala tympani contain perilymph.
Stapes

Scala vestibuli

Helicotrema

Reissner's membrane
Basilar membrane

Round window

Figure 1-37. The components of the cochlea, shown as it would appear if uncoiled (upper diagram) and shown in
cross section (lower diagram).

(4) The oval window is the membrane-covered opening of the scala vestibuli, which joins
the footplate of the stapes.
(5) The round window is the membrane-covered opening of the scala tympani.
b. The organ of Corti is the specialized region that contains the sensory receptor cells of the
ear. It lies on the basilar membrane, adjacent to the limbus, and contains the following
structures (Figure 1-38).
(1) Hair cells that lie within the organ of Corti are the sensory receptors for the auditory
system. The essential excitatory event in auditory transduction is the deflection of the
cilia protruding from the surface of hair cells. When the cilia are deflected, transmitter
substance is released from the cells.
(2) Tectorial membrane. This flap of tissue rests on the tips of the cilia of the hair cells
and is attached to the limbus at one end. Any movement of the basilar membrane
causes the free end of the tectorial membrane to drag or push the cilia of the hair cells.
(3) Nerve fibers. The peripheral endings of the afferent fibers form synapses with the hair
cells, and the cell bodies form the spiral ganglion. Axons of these neurons join the
vestibulocochlear nerve (cranial nerve VIII).

C. Auditory transduction is the process by which an auditory receptor potential is generated by


sound waves entering the ear.

1. Sound wave propagation. Sound waves are propagated through the cochlea.
a. Sound waves entering the inner ear from the oval window spread along the scala vestibuli
as a traveling wave. Most of the sound energy is transferred directly from the scala vestibuli
to the scala tympani. Very little of the sound wave ever reaches the helicotrema at the apex
of the cochlea.
b. When the pressure of a sound wave causes the oval window to bulge inward, the pressure
(after passing through the incompressible fluids of the scala vestibuli, scala media, and scala
tympani) forces the round window to bulge outward.
2. Movement of the organ of Corti
a. The sound waves cause the organ of Corti to vibrate up and down, which, in turn, causes
the stereocilia to bend back and forth.
(1) When the sound wave pushes the organ of Corti downward, the stereocilia bend toward
the limbus. When the sound wave pulls the organ of Corti upward, the stereocilia bend
away from the limbus (Figure 1-39).
(2) The attachment of the stereocilia to the tectorial membrane makes it possible for the
up-and-down movement of the organ of Corti to be translated into the back-and-forth
movement of the stereocilia.
(a) The bottoms of the hair cells are anchored to the basiliar membrane, while the
stereocilia are connected to the overlying tectorial membrane.
(b) Because the tectorial and basilar membranes are attached at different points on the
limbus (see Figure 1-39A), they slide past each other as they vibrate up and down,
causing the stereocilia on the hair cells to bend back and forth.

Outer hair cells

Figure 1-38. The organ of Corti, showing the connection between the tectorial membrane and the cilia of the hair
cells.

'"'"

Basilar membrane

c
Figure 1-39. Diagram showing how the up-and-down
movement of the basilar and tectorial membrane causes
the stereocilia extending from the hair cells to bend back
and forth. (A) The tectorial and basilar membranes are
attached to the limbus at different points. (8) When the
membranes rotate upward, the tectorial membrane slides
forward relative to the basilar membrane bending the
stereocilia away from the limbus; (C) when the membranes rotate downward, the stereocilia bend toward the
limbus.

(i) When the organ of Corti moves up, the stereocilia bend away from the limbus
(see Figure 1-39B).
(ii) When the organ of Corti moves down, the stereocilia bend toward the limbus
(see Figure 1-39Cl.
b. The stereocilia are polarized.
(1) Bending the stereocilia away from the limbus causes them to depolarize.
(2) Bending the stereocilia toward the limbus causes them to hyperpolarize.
(3) Thus, when the organ of Corti is pushed downward, the hair cells hyperpolarize, and
when it is pulled upward, the hair cells depolarize.

3. Mechanism of depolarization
a. Movement of K+ into the cell. The electrochemical gradient for K+ favors the movement
of K+ into the cell.
(1) The endolymph contains a high concentration of K+ (135 mEq/U and is electrically
positive in comparison to the perilymph.
(2) Hair cells, like all other cells, contain a high concentration of K+ and are electrically
negative when compared to the perilymph.
(3) Because the endolymph is electrically positive and the hair cell is electrically negative,
a very large potential difference (in excess of -100 mY) exists across the hair cell
membrane.
(4) The large negative potential and lack of a K+ concentration difference between the
inside and outside of the hair cell create a driving force, pushing K+ into the cell.
b. K+ channels. Bending the hair cells causes K+ channels to open or close.
(1) When the stereocilia bend away from the limbus, they cause K+ channels to open. K+
then flows into the cell and the hair cell depolarizes.

(2) When the stereocilia bend toward the limbus, they cause K+ channels to close and the
hair cell hyperpolarizes.
D. Encoding of auditory information
1. Release of synaptic transmitter
a. When the hair cell depolarizes, a Ca2+ channel opens, allowing Ca 2 + to enter the cell.
b. Ca2+ initiates the release of a synaptic transmitter.
c. An auditory nerve fiber is stimulated by the synaptic transmitter.
2. Place principle of frequency discrimination. The frequency of sound that activates a particular hair cell depends on the location of the hair cell along the basilar membrane. The basilar
membrane is narrowest and stiffest at the base of the cochlea (near the oval and round windows)
and widest and most compliant at the apex of the cochlea (near the helicotrema).
a. The energy contained in high-frequency sounds passes through the organ of Corti near the
base of the cochlea, because the stiff portion of the basilar membrane resonates with high
frequencies.
b. In contrast, the energy in low-frequency sounds passes through the organ of Corti near the
apex of the cochlea, because the compliant portion of the basilar membrane resonates with
low frequencies.
3. Encoding of frequency. The auditory nerve fiber activated by a particular sound frequency is
similarly dependent on the location of the hair cell it innervates.
a. There are about 30,000 nerve fibers in each auditory nerve.
(1) Over 90% of these fibers innervate inner hair cells. Each inner hair cell receives innervation from about 10 auditory nerve fibers; each auditory nerve fiber innervates only one
hair cell.
(2) The outer hair cells, which are far more numerous, are innervated by the remaining
1 0% of the sensory fibers.
b. For low-frequency sounds, the auditory nerve fibers can fire at the same frequency as the
sound wave. This mechanism of sensory encoding is called the volley principle of frequency discrimination.
c. The sound frequency producing the greatest response in an auditory nerve fiber is called
the characteristic frequency of that nerve fiber.
4. Encoding of intensity. The frequency of firing in an auditory nerve fiber increases as the
intensity of the sound wave increases. In addition, a larger portion of the basilar membrane is
vibrated as the sound intensity increases so that more auditory nerve fibers are activated. Thus,
sound intensity is encoded by the frequency of auditory nerve discharge and by the number
of auditory nerve fibers that are active.
5. Inhibitory innervation. The hair cells receive a very prominent efferent innervation from the
superior olivary nucleus via the olivocochlear bundle. Although such a large pathway probably
plays an important role in auditory transduction, its purpose has not been deciphered.
E. Physiology of central auditory neurons
1. Tonotopic organization. Generally, any neuron of the auditory pathway can be tested with
tones of different frequencies to determine a best frequency for that cell. Neurons responding
best to low-frequency tones will be located at one end of a nucleus, while neurons responding
best to high-frequency tones will be represented at the opposite end of the nucleus. This orderly
arrangement of frequency sensitivity, termed tonotopic organization, resembles the retinotopic
organization of the visual system and the somatotopic organization of the somatosensory
system. The tonotopic map reflects the methodical arrangement of frequency sensitivity along
the length of the basilar membrane from base to apex. Tonotopic organization is prominent in
the cochlear nuclei but becomes less precise in more rostral structures of the auditory pathway.
2. Feature detection. Higher auditory centers respond to particular features of sound stimuli. For
example, cortical neurons may respond specifically to a shift from high- to low-frequenty notes,
which is why lesions of the auditory cortex may not impair the ability to discriminate frequency.
Instead, lesions of the auditory cortex cause a loss of ability to recognize a patterned sequence
of sounds. In addition, the ability to identify the position of a sound source is impaired.

3. localization of sound in space. Detection of the position of a sound source depends on the
ability of the CNS to compare intensity differences and phase differences. A sound source
located behind the head, closer to one ear than the other, produces a slightly more intense
sound in the near ear than in the remote ear. Sound absorption by the tissues of the head
attenuates sound intensity in the remote ear. In addition, at low frequencies there may be
phase differences in the sound waves striking the two ears. These minute phase and intensity
differences between the two ears are detected and discriminated by the auditory system. The
extensive decussation and complex circuitry of the auditory pathway provide the CNS with the
information necessary to identify a sound source.
F. Hearing impairments

1. Tinnitus is a ringing sensation in the ears caused by irritative stimulation of either the inner ear
or the vestibulocochlear nerve.
2. Deafness. Two classes of deafness are distinguished based on the location of the abnormality
or lesion.
a. Conduction deafness is caused by interference with the transmission of sound to the
sensory mechanism of the inner ear. Conduction deafness represents a defect of the external
or middle ear.
b. Nerve deafness results from defects of either the inner ear or the vestibulocochlear nerve.
Deafness due to a lesion in the CNS structures generally is not found clinically because of
the redundancy and bilateral nature of the central auditory pathways.

VIII. VISION
A. Physiologic optics. In order for an object to be perceived by the visual system, a clear image of
the object must be formed on the retina.

1. Principles of image formation. Several physical principles govern the process of image formation by a lens system.
a. light rays emanating from a luminous object travel in all directions (Figure 1-40A).
However, only those rays entering the lens are used to form an image.
b. Refraction is the process of bending rays of light.
(1) A converging (positive) lens causes rays of light to be bent toward the center of the
lens (see Figure 1-408). A converging lens can form an image.
(a) The object distance is the distance from the object to the lens.
(b) The image distance is the distance from the lens to the image.
(2) A diverging (negative) lens causes rays of light to be bent away from the center of the
lens (see Figure 1-408). A diverging lens cannot form an image.
c. Focal plane
(1) If the rays of light entering a converging lens are parallel, they will form an image on
the focal plane (see Figure 1-400.
(a) The focal distance is the distance from the lens to the focal plane.
(b) The power of the lens is defined as
1

p=f

where P = the power of the lens [diopters (D)] and f = the focal length (m).
(2) If the rays of light entering a converging lens are diverging from each other when they
enter the lens, they will form an image behind the focal plane.
(3) The lens formula defines the relationship between the image, object, and focal distances
(Figure 40D)
1

-+-=-=P
where 0, i, and f = object, image, and focal distances (m), respectively, and
power (D).
(4) The image and object size have the following relationship
image size

object distance

object size

image distance

P= lens

Diverging lens

Converging lens

III

c:
.9.!

Cl

c:
.6>

:E

Q;

>
c:
0
0

Q)

c:

III

Ci.

iii

u.

~j

Figure 1-40. Diagram illustrating some fundamental optical principles. (A) Although light rays travel in all directions
from a source of light, only those passing through the lens are used to form an image. (8) Converging lenses focus all
rays of light coming from a point on an object onto a single image point. Diverging lenses cannot form an image.
(C) If the rays of light entering a converging lens are parallel, their image will be formed on the focal plane. (0) If the
rays of light are not parallel, their image will be formed behind the focal plane. The relationship between the object
distance, focal distance, and image distance is given by the lens formula [see VIII A 1 c (3)]. 0 = object distance;
i = image difference; f = focal distance.

2. Refractive media of the eye. A horizontal section through the human eyeball is shown in
Figure 1-41. To reach the retina, light must pass through four refractive interfaces: the anterior
surface of the cornea, the posterior surface of the cornea, the anterior surface of the lens, and
the posterior surface of the lens.
a. Cornea
(1) Transparency
(a) The absence of blood vessels in the cornea allows light to pass through unhindered.

Cornea

Figure 1-41. Horizontal section of the eyeball.

The cornea, consequently, must receive oxygen and nutrients via diffusion through
the aqueous humor.
(b) Elevated intraocular pressure, a condition termed glaucoma, can compromise the
transparency of the cornea. Intraocular pressure is measured clinically by a tonometer, which records the resistance to a calibrated indentation of the cornea.
(2) Refractive power. The greatest amount of refraction of the eye occurs at the anterior
surface of the cornea, because the difference between the indices of refraction of air
and of the cornea is larger than the difference between the indices of refraction of the
other ocular media. The cornea contributes about two-thirds, or + 40 D, of the eye's
refractive power. The amount of refraction contributed by the cornea is fixed.
b. Lens
(1) Transparency
(a) The lens, like the cornea, is avascular. It consists of a capsule, a layer of epithelial
cells, and a system of transparent fibers. New fibers are continually laid down.
(b) As an individual ages, the lens becomes harder and less malleable. Cataracts are
opacities of the lens that reduce transparency. One cause of cataracts is an elevated
glucose level in the aqueous humor, as occurs in uncontrolled diabetes.
(2) Refractive power. The refractive power of the lens, unlike that of the cornea, is
adjustable. The unaccommodated lens contributes about one-third of the eye's refractive
power. The lens is suspended by the tough zonular fibers, which stretch the lens and
flatten it. The tension on the zonular fibers is released by contraction of the ciliary
muscle. Contraction results in the constriction of this ring of muscle fibers, which lies
anterior to the attachment of the zonular fibers. The release of tension on the zonular
fibers results in an increase in the curvature of the anterior surface of the lens. The
increased convexity of the lens increases its refractive power. The lens can add, at most,
+ 12 D of power to the ocular apparatus.
3. The "reduced eye" model. The optical properties of the eye are very complicated. To simplify
the application of image formation and lens equations to the eye, a model of the eye, called
the reduced eye (Figure 1-42), is used.

Figure 1-42. Image formation in the

t----17 mm--....,j

reduced eye.

a. How the model works


(1) In the reduced eye, the refractive surfaces of the cornea and lens are combined into a
single surface.
(a) The axial length of the reduced eye (the distance from the lens surface to the retina)
is considered to be 17 mm. In reality, the axial length of an adult human eye is
typically about 25 mm.
(b) The refractive power of the reduced eye is approximately 58.8 D.
(2) According to the lens formula, the image of an object placed 6 m or more from the eye
will be formed on the retina since
1

0.017 m

58.8 D or

= -

17 mm

(3) If the object is closer than 6 m, the image will be formed behind the retina.
b. The near point and far point define the range of distances over which a clear image can
be formed by the eye.
(1) The near point is the nearest point at which an object can be seen clearly. A typical
young adult can increase his or her refractive power by 12 D. At a maximum lens power
of 70.8 D, the near point is about 8.3 cm from the eye.
(2) The far point is the distance from the eye that an object must be placed so that it can
be seen clearly without accommodation. For a normal person, this is 6 m.
4. Accommodation. A clear image of the object can be formed if the eye accommodates for
near vision (objects closer than 6 m). Accommodation occurs by a reflex that has three
components.
a. Bulging of the lens. Contraction of the ciliary muscle releases tension on the zonular fibers
(see Figure 1-41). The elastic capsule surrounding the lens retracts, increasing the convexity
(and thus the power) of the lens.
b. Pupillary constriction. By reducing the area through which light can enter the eye, spherical
aberration is reduced. Reducing spherical aberration increases the depth of focus (the degree
to which the image can form in front of or behind the retina and still appear to be in focus).
Nearsighted persons typically squint to increase their depth of focus.
c. Convergence. The gaze of the two eyes shifts toward the center of the head in order to
keep both eyes focused on the object.
5. Refractive errors, in which distant objects cannot be seen clearly without accommodation,
are due to variations in the axial length or the refractive power of the eye.
a. Myopia (nearsightedness) results from an axial length that is too long for the refractive
power of the eye. In this case, the focal point is in front of the retina; thus, distant objects
cannot be focused on the retina.
(1) The object can be seen clearly if it is moved closer to the e~e so that the image forms
on the retina (but behind the focal plane). In severe myopia, the far point may be only
10-15 cm from the eye.
(2) As the object is moved closer to the eye, the lens accommodates for near vision.
Myopes have near points very close to the eye. Thus, they are able to do fine work
without magnifying glasses.
(3) A diverging lens can be placed in front of a myopic eye, reducing its refractive power.
When wearing such a lens, the myope can see distant objects clearly.
b. Hyperopia (farsightedness) results from an axial length that is too short for the refractive
power of the eye. In this case, distant objects cannot be focused clearly because the focal
point is in back of the retina.
(1) A distant object can be seen clearly if the hyperope increases his or her refractive power
by accommodating. If the degree of hyperopia is not very great, the person will usually
not be aware of the refractive error. However, if a large amount of accommodation is
required to focus distant objects, the constant contraction of the ciliary muscle will
cause eye strain.
(2) Because some amount of accommodation is used to see distant objects, there is not so
much available for near vision. Thus, the near point in hyperopes is further from the
retina than normal.
(3) A converging lens can be placed in front of a hyperopic eye to increase its refractive
power. When wearing a diverging lens, a hyperope can see distant objects clearly
without accommodation.

c. Presbyopia is a loss of accommodative power associated with aging. As an individual ages,


the elastic recoil of the lens decreases, and it is unable to bulge as much when the ciliary
muscles contract.
(1) Because the accommodative power is lost in presbyopia, the near point moves farther
from the eye.
(2) A converging lens is used to substitute for the lost accommodative power. The use of
these lenses in reading glasses makes near vision possible.
d. Astigmatism is due to an uneven cornea. Normally, the cornea has a spherical surface; in
astigmatism, it has more of an egg-shaped surface. As a result, the power of the lens is
different in different axes.
(1) Images formed with an astigmatic lens are distorted. For example, a point is seen as a
line, and a line appears to have a halo on either side of it.
(2) Cylindrical lenses can be worn by an astigmatic individual. These lenses add refractive
power in only one axis and thus allow the lens system to behave as a spherical surface.
B. Retina. The retina is the receptor mechanism of the eye and the ultimate tool of vision. Light and
color-sensitive elements of the retina communicate their messages, via neural pathways, to the
visual cortex of the brain.

1. Morphology and function of retinal components


a. Layers, fovea, and optic disk. The retina is divided into 10 histologically defined layers
(Figure 1-43). In the center of the retina is a yellowish region termed the macula lutea. The
fovea is a central depression in the macula lutea, about 0.3 mm in diameter, where some
of the retinal layers are displaced and the retina, consequently, is thinned. The fovea is the
visual center of the eye and the area of highest resolution. About 3 mm to the nasal side of
the fovea is the optic disk, where the ganglion cell axons turn and pierce the retina to form
the optic nerve. Receptor elements are absent at the optic disk, which creates a blind spot
on the retinal field.
b. Cells. The retina is composed of the following cells.

Pigment cell layer


Rod and cone layer

_R

R=rod
C=cone

Outer nuclear layer


l-

Outer plexiform layer

<!l
::J

H = horizontal cell
Inner nuclear layer

B = bipolar cell
A= amacrine cell

u.

o
z
o

()

UJ

a:

Inner plexiform layer

Ganglion cell layer

G = ganglion cell

Optic nerve layer


Internal limiting membrane-----------------------------Figure 1-43. Cytoarchitecture of the retina. The three-neuron circuit of the rod system differs from that of the cone
system. Convergence of receptors onto bipolar cells is seen only in the rod system. Horizontal cells and amacrine
cells link adjacent retinal circuits.

(1) Pigment cells contain melanin and are arranged in a single lamina. Their major functions
include:
(a) Absorption of stray light
(b) Phagocytosis of disks that have been sloughed from the outer segments of receptor
cells
(2) Receptor cells divide into two distinct types: rods and cones (Figure 1-44).
(a) The outer segment of rods and cones invaginates to form membrane disks.
(i) In rods, but not in cones, the disks pinch off and become independent of the
cell membrane. In rods, the disks undergo constant renewal. Disks are formed
initially at the basal pole of the outer segment and migrate through the outer
segment until they reach the apical pole. Here the disintegrating disks are phagocytosed by the pigment cells.
(ii) Cones show a more diffuse pattern of disintegration.
(iii) The photopigment molecules are embedded in the disk membranes.
(b) The inner segment contains numerous mitochondria as well as the nucleus of the
rod or cone.
(c) The receptor terminal contains synaptic vesicles.
(d) Functions of rods and cones
(i) Color vision. Only cones are involved with color vision.
(ii) Sensitivity to light. Cones function at high light intensities and, therefore, are
responsible for day (photopic) vision. Cones are not as sensitive as rods to low
levels of illumination. Rods function at low light intensities and are responsible
for night (scotopic) vision.
(iii) Visual acuity. Cones have a high level of visual acuity and are present in the
fovea. Rods have a much lower level of acuity; they are lacking in the fovea and
are confined to the peripheral retina.
(iv) Dark adaptation. When an individual moves into a darkened environment, the
eyes adapt slowly to lower levels of illumination. The threshold for sensation of
low-intensity light stimuli falls progressively, as shown in Figure 1-45. The dark
adaptation curve has two limbs, representing adaptation of cones followed by
ROD

Disks-~E-+

CONE

Outer
segment

[
] Con!1.ecting
CIlium

Inner
segment

Nucleus

Figure 1-44. Morphology of rod and


cone receptor cells. Cones, which are
responsible for color perception and
high visual acuity, are found in the
fovea. Rods, which are responsible
for night vision, are located in the peripheral retina.

10
15
Time in dark (min)

20

Figure 1-45. Dark adaptation. The eye becomes more


sensitive to low-intensity light stimuli with increasing time
in the dark. The ordinate indicates threshold light intensities.

that of rods. Rod function is suppressed during day vision and comes to the fore
only when background illumination is low.
(3) Muller cells are glial elements that have cell bodies in the inner nuclear layer. Their
processes extend to form the internal limiting membrane as well as the external
limiting membrane. The processes also fill the spaces between receptor cells and serve
as a connecting and supporting tissue.

(4) Bipolar cells and ganglion cells


(a) Bipolar cells synapse with the receptor terminals of rods or cones in the outer
plexiform layer. Bipolar cells in turn terminate on ganglion cells in synapses located
in the inner plexiform layer.
(b) Ganglion cells. The dendrites of these neurons receive their input from the bipolar
cells. Their axons course along the retina to the optic disk and form the optic nerve.
(c) Function in light sensitivity and visual acuity. The low sensitivity but high acuity
of the cone system results from the connection of single cones to individual bipolar
cells and subsequently to single ganglion cells projecting to the brain (see Figure
1-43). The rod system has a higher sensitivity because a large number of these
receptors converge onto an individual bipolar cell, and a photon striking anyone
of these rods is effective in activating the bipolar cell. The bipolar cell of the cone
i~ less sensitive, since a photon must strike a particular receptor location to be
effective. Because stimuli at several adjacent rod locations all can result in an
identical response in the second-order neuron, acuity is sacrificed in the rod system.

(5) Local circuit neurons


(a) Horizontal cells. The processes of these cells are confined to the outer plexiform
layer. They form a triad synapse at the junction of the receptor terminals and the
bipolar cells.

(b) Amacrine cells form complex synapses in the inner plexiform layer at the junction
of the bipolar cells and the dendrites of the ganglion cells.

2. Photochemistry of receptor cells


a. Bleaching of rhodopsin. When light strikes the eye, it is absorbed by the photopigments
of the retina, causing a photochemical transformation to occur (Figure 1-46). During this

AIItrans retinal

ll-cis retinal

;i;

~
Opsin

-----._
L--_ _ _ _ _- '

Figure 1-46. Action of light on the rhodopsin molecule. Initially, the chromophore moiety, 11cis retinal, fits closely
into the opsin moiety. Light causes the conformation of the 11-cis retinal to change to the elongated alltrans retinal,
causing the opsin configuration to change.

process, the color of the photopigment changes from purple to pink; consequently, this
process is called bleaching.
(1) Light causes a photoisomerization of 11-cis retinal into an intermediate form that, after
going through a series of spontaneous changes, finally is converted into all-trans retinal.
(2) The all-trans retinal dissociates from the opsin and then is converted into all-trans retinol
(vitamin A).
b. Regeneration of rhodopsin. All-trans retinol is enzymatically isomerized to 11-cis retinol
and then enzymatically oxidized to 11-cis retinal. The 11-cis retinal then spontaneously
combines with opsin, reforming rhodopsin.
(1) The conversion of the retinol to retinal is dependent on the pigment epithelium.
(2) Deficiency of vitamin A prevents the regeneration of rhodopsin. For this reason, lack of
vitamin A leads to night blindness.
c. Absorption spectrum of visual pigments
(1) Although rhodopsin absorbs light over most of the visible spectrum, it displays a maximal
absorption at about 500 nm (the wavelength of green light).
(2) There are three types of cones, each of which contains a photopigment that absorbs
light best at a particular wavelength (Figure 1-47).
(a) Red cones have a maximal absorption at a wavelength of 570 nm.
(b) Green cones have a maximal absorption at a wavelength of 535 nm.
(c) Blue cones have a maximal absorption at a wavelength of 445 nm.
(3) Color blindness is the inability to produce one or more of the cone photopigments.
(a) Monochromats have no cone photopigments and must rely entirely on rods for
their vision. They are unable to see any colors.
(b) Protonopes lack red pigment and, thus, make color confusions in the red-green
portion of the spectrum.
(c) Deuteranopes lack green pigment and, like the protonopes, are unable to distinguish colors in the red-green portion of the spectrum.
(d) Tritanopes lack blue pigment and, thus, have difficulty distinguishing colors in the
blue portion of the spectrum.
(e) Protonopes and deuteranopes are called red-green color-blind because they cannot
see colors in this region of the spectrum normally. However, they are not truly blind
in that they are able to see colors. Similarly, loss of the blue pigment does not
prevent the seeing of blue light, because light in the blue end of the spectrum is
absorbed to some extent by the red and green pigments.
(4) Color coding. Over most of the visible spectrum, light is absorbed by all three cone
pigments (see Figure 1-47). Thus, the encoding of color depends on the relative amount
of activity in each of the three cones. For example, if the red cones are stimulated most
strongly, the CNS interprets this activity as red light. If the red and green cones are
stimulated equally, then a sensation of yellow is generated by the CNS.

3. Electrophysiology of the retina


a. Receptor cell potential
(1) Receptor cells have a resting membrane potential of about -40 mV due to a high Na+
conductance.

......

"" ""

.-

....

.'o5.

,,

,,

,,

,,

,,

.c

,,

,
"

400

500

600

Wavelength (nm)

700

Figure 1-47. Spectral sensitivity of


the three types of cones. The photopigment that is present in the outer
segment of each cone type is responsible for the variance in the maximal
light absorption and sensitivity in
these cells. The dotted line indicates
cones with blue-sensitive pigment;
the solid line indicates cones with
green-sensitive pigment; and the
dashed line indicates cones with
orange-sensitive pigment.

(2) The high Na+ conductance is maintained by the presence of cyclic guanosine monophosphate (cGMP).
(3) When light strikes the eye, photoisomerization of 11-cis retinal is accompanied by the
activation of a G protein [similar to the G protein involved in synaptic transmission; see
III B 2 a (3)] called transducin, which causes cGMP levels to decrease.
(a) The G protein activates a phosphodiesterase that breaks down cGMP to GMP.
(b) The reduction in cGMP levels causes the Na+ channels to close.
(4) The closing of the Na+ channel causes the receptor to hyperpolarize. This is different
from other receptors, which are depolarized when activated by a stimulus.
(5) Increasing the light intensity increases the receptor hyperpolarization (Figure 1-48). The
amplitude of the hyperpolarization varies with the logarithm of the light intensity.
b. Organization of ganglion cell receptive fields (Figure 1-49)
(1) Central circuit. In the peripheral retina, each ganglion cell receives synaptic input from
many bipolar cells, which, in turn, receive input from a large number of receptor cells.
All of the receptor cells that connect directly to a ganglion cell through bipolar cells
form the receptive field center for that ganglion cell.
(a) In the dark, the receptor cells are depolarized and, thus, release synaptic transmitter.
Because the transmitter is inhibitory, it keeps the bipolar cell hyperpolarized. As a
result, the ganglion cell is not stimulated.
(b) When light strikes the receptor cells, it causes them to hyperpolarize. As a result,
receptor cell transmitter release is decreased, the bipolar cell depolarizes, and the
ganglion cell is stimulated.
(c) Thus, when any of the cells in the receptive field center of a ganglion cell are
stimulated by light, it causes the ganglion cell to fire.
(2) Surrounding circuit. The ganglion cell also is influenced by receptors that connect
to the bipolar cells through horizontal cells. Receptor cells lying within an annulus
surrounding the receptive field of the ganglion cell form the receptive field surround.
When light strikes any of the receptors in the receptive field surround, the ganglion cell
is inhibited through the horizontal-to-bipolar-to-ganglion-cell pathway.
(3) This organization of the ganglion cell receptive field is called an on-center, off-surround
organization to indicate that light hitting the center of the field causes ganglion cell
excitation, while light hitting the surround of the field causes ganglion cell inhibition.
There is a complementary type of organization that has off-center, on-surround characteristics. Ganglion cells with this type of receptive field are inhibited by light hitting
the center of the field and are excited when light shines on the cells within the area
surrounding the center.
(a) Because of the center-surround organization of the receptive fields, the retina transmits information only about changes in levels of illumination within the visual field.
(b) For example, a bright spot surrounded by darkness will cause a large increase in the
firing of an on-center ganglion cell, whereas a dark area surrounded by light will
cause the ganglion cell to stop firing. If both the center and the surround are bathed
by light or placed in darkness, then the ganglion cell firing rate will not change.

::;-

.s -20
iii
~

~-"'"

(J)

&. -30
(J)

c:

co

..c

~ -40

0.2

0.4

Time (sec)

0.6

Figure 1-48. Response of a receptor cell to light. Responses to three brief light flashes of increasing intensity
are shown. The receptor response is a graded hyperpolarization that increases with increasing light intensity. The
heaviest line indicates the light flash of highest intensity.

CENTER

Light

SURROUNDING
RETINA

Receptor
cells

Horizontal
cell

Bipolar
cells

Ganglion
cells

Figure 1-49. Organization of retinal circuits. Light striking one spot on the retina (center circuit) causes hyperpolarizing responses in the receptor cells, depolarization
of the bipolar cells, and spikes in the ganglion cells of this
circuit. By the action of horizontal cells, bipolar cells in
the surrounding retinal circuits are depolarized, resulting
in the hyperpolarization of the ganglion cells of these
circuits. Note that of the cell types shown, true action
potentials are seen in only the ganglion cells.

C. Neural pathways of vision


1. Optic tract. The axons of the ganglion cells are rearranged at the optic chiasm. Axons from
the right half of each retina project to the right lateral geniculate, and axons from the left half
of each retina pass to the left lateral geniculate. This means that fibers of the temporal retina
on each side pass through the chiasm without crossing, while fibers of the nasal half of each
retina decussate at the chiasm.
2. Lateral geniculate nucleus. The inputs from each eye remain segregated in the lateral geniculate; uncrossed fibers go to layers 2, 3, and 5, and crossed fibers go to layers 1, 4, and 6.
Neurons in the lateral geniculate also retain the on-center, off-surround (or the off-center,
on-surround) organization of the ganglion cells.
D. Primary visual cortex. Neurons of the cortex are responsible for visual perception. This is accomplished by cells whose input configuration allows for feature detection (Le., sensitivity to stimuli
of specific shape and orientation).
1. Feature detectors are divided into three types: simple cells, complex cells, and hypercomplex cells.
a. Simple cells
(1) Response characteristics. Like ganglion and lateral geniculate cells, simple cortical
cells have antagonistic center-surround organizations. However, rather than circular
receptive fields, simple cells have elongated receptive fields. The best simple cell response is obtained with light or dark bars. The long axis of the bar must fall on a specific
place on the retina and be oriented at a particular angle.
(2) Synaptic input. The response characteristics of simple cells arise from the input they
receive. Several adjacent ganglion cells, arrayed in a straight line at a particular orientation on the retina, project (via the lateral geniculate) onto a single simple cell of the
visual cortex. When the ganglion cell array is activated with a bar of light, the cortical
neuron is activated simultaneously. The simple cortical cell receiving input from this

ganglion cell array can be activated only by (i.e., recognize) a bar of light that is at the
proper retinal position and orientation.
b. Complex cells
(1) Response characteristics. Like simple cells, complex cortical cells respond best to light
lines or dark bars of a particular orientation. The complex cell, however, responds even
if the oriented stimulus does not fall on a particular retinal position. The stimulus can
be delivered to a larger area of the visual field and still evoke a response.
(2) Synaptic input. The response of a complex cell can be interpreted as a response to a
series of simple cells, representing adjacent portions of the retina. The antagonistic
center-surround organization is less prominent in complex cells.
.
c. Hypercomplex cells. These units respond only to light lines of particular length or to edges.
They receive and integrate input from a variety of complex cells. Hypercomplex cells
are more prominent in the prestriate cortex (areas 18 and 19) than in the primary visual
cortex (area 17).
2. Cortical columns. Cortical cells respond to lines, bars, and edges. The orientation of the linear
stimulus is critical in determining whether a cell in the visual cortex will respond. Cells that
respond to stimuli of a particular orientation are grouped together in an orientation column,
which is perpendicular to the cortical surface.

IX. TASTE, or gustation, is the sensory modality mediated by the chemoreceptors of the tongue, mouth,
and pharynx. Taste should be distinguished from flavor, which includes the olfactory, tactile, and
thermal attributes of food in addition to taste. The taste chemoreceptors are sensitive to dissolved
chemicals, both organic and inorganic.
A. Psychophysics of taste
1. Taste qualities. There are four primary taste qualities; all taste sensations are assumed to result
from various combinations of these four primaries.
a. Sweet sensation is produced by various classes of organic molecules including sugars,
glycols, and aldehydes. The tip of the tongue is the area most sensitive to sweet stimuli.
b. Bitter sensation is produced by alkaloids, such as quinine and caffeine. The back of the
tongue is the area most sensitive to these stimuli.
c. Salt sensation is produced by the anions of ionizable salts. The greatest sensitivity to salty
tastes occurs in the front half of each side of the tongue.
d. Sour sensation is produced by acids; this sensation relates, to some degree, to the pH of
applied stimulus solutions. The greatest sensitivity to these stimuli occurs in the posterior
half of each side of the tongue.
2. Threshold. The lowest concentration of a sapid substance that can be discriminated is the
threshold concentration of that substance. Increasing the area of the tongue exposed to the
solution reduces the threshold concentration. Individual differences in sensitivity are genetically
determined. For example, some individuals cannot detect phenylthiourea (a compound used
in genetics to test ability to taste) unless it exists in very high concentrations.
3. Adaptation. The intensity of a sensation resulting from continuous application of a taste stimulus
declines with time until the solution becomes tasteless. The firing rate of afferent nerve fibers
from the tongue similarly declines.
B. Peripheral taste receptors
1. Lingual papillae. These specialized protuberances on the surface of the tongue are of four
morphologic types.
a. Filiform papillae are mechanical, nongustatory structures.
b. Fungiform papillae have 8-10 taste buds on each papilla.
c. Circumvallate papillae are arranged in a V-shaped row of 7-12 on the posterior part of
the tongue. Each papilla has approximately 200 taste buds. The taste buds are located on
the sides of these large structures.
d. Foliate papillae, located on the lateral border of the tongue anterior to the circumvallate
papillae, have numerous taste buds.

2. Taste buds. Each taste bud represents a cluster of 40-60 taste cells (Figure 1-50) as well as
supporting cells. Taste buds are located on the tongue papillae, hard palate, soft palate, epiglottis, and in the pharynx.

3. Taste cells
a. Structure. These chemosensitive receptor cells are elongated cells with microvilli at the
apical surface. Dissolved substances enter the taste pore of the taste bud to contact the
microvilli. The taste cells are innervated by afferent neurons.
b. Turnover. Each taste cell has a life cycle of only a few days. The degenerating taste cell is
replaced by a cell that arises from the supporting epithelial cells. Contact with the afferent
neuron converts an epithelial cell into a taste cell. Conversely, nerve transection causes the
disappearance of taste cells.

C. Innervation of taste cells is by branches of the facial, glossopharyngeal, and vagus nerves (cranial
nerves VII, IX, and X, respectively). Nerve fibers terminate on receptor cells to form synapses.
[The tactile and temperature receptors of the mouth, tongue, and pharynx are innervated by the
trigeminal nerve (cranial nerve V).]

1. The taste buds in the anterior two-thirds of the tongue are innervated by lingual branches
of the facial nerve; the lingual nerve, which branches from the chorda tympani, is part of the
facial nerve. The cell bodies are located in the geniculate ganglion, and the nerve terminals
end in the nucleus solitarius of the medulla.

2. The taste buds in the posterior third of the tongue are innervated by the glossopharyngeal
nerve. The cell bodies lie in the superior and inferior ganglia of this nerve. The fibers relating
to taste sensation terminate in the nucleus solitarius.

3. Taste receptors in the pharyngeal aspect of the tongue and on the hard palate, soft palate,
and epiglottis are innervated by fibers of the vagus nerve. The cell bodies are located in the
superior and inferior ganglia of the vagus nerve and terminate in the nucleus solitarius.

D. Taste coding. Each nerve fiber in the gustatory nerves responds to more than one taste stimulus.
However, each fiber responds best to one of the four primary taste qualities. Thus, the coding of
a gustatory sensation is not a simple, labeled-line, chemical sensory system. Taste sensation
depends on the pattern of nerve fibers activated by a particular stimulus.

x.

OLFACTION, like taste, is a chemical sense involving receptors that are sensitive to chemicals in
solution. For olfaction, the chemicals initially are airborne; however, they must dissolve inthe mucous
layer lining the nose before they can come in contact with olfactory receptors.

Epithelium

Synaptic Afferent
vesicles neurons

Figure 1-50. Structure of a taste bud. Taste receptor


cells contain synaptic vesicles and receive afferent nerve
terminals. Supporting cells (5) are not innervated.

A. Psychophysics of olfaction
1. Characteristics of odorants. To be an effective odorant, a substance must be:
a. Volatile, since the olfactory receptors respond to chemicals transported by air into the nose
b. Water soluble (to some degree) in order to penetrate the watery mucous layer lining the
nasal epithelium to reach the receptor cell membrane

c. Lipid soluble (to some degree) in order to penetrate the cell membranes of the olfactory
receptor cells to stimulate those cells
2. Threshold. The olfactory receptors have varying sensitivity to substances. For many substances,
the sensitivity is so high that a few molecules interacting with a receptor are sufficient to
produce excitation.
3. Odor discrimination. The olfactory system has the capacity to discriminate various chemical
compounds. This ability depends on the receptor sites on the membranes of receptor cells,
which vary in their capability to accept odorant molecules of complementary conformation.
In some instances, the olfactory system can discriminate dextro- and levorotatory forms as well
as cis- and trans- conformations of a molecule. In contrast to gustatory sensation, no primary
olfactory qualities can be identified.
4. Dynamic range. The olfactory system has limited ability to discriminate differences in odorant
concentration in ambient air. Perceived odor intensity conforms to a power function with an
exponent of less than 1 (see V 0 1).
5. Adaptation. Olfactory sensation decreases very rapidly with continued exposure to an odorant.

B. Olfactory mucosa. The olfactory mucosa refers to the specialized part of the nasal mucosa that
contains the olfactory receptor cells. It is distinguished from the surrounding respiratory mucosa
by the presence of tubular Bowman's glands, the absence of the rhythmic ciliary beating that
characterizes the respiratory mucosa, and its distinctive yellow-brown pigment. A mucous layer
covers the entire epithelium.
1. Location. The olfactory mucosa is located on the superior nasal concha, adjacent to the nasal
septum. Because of its superior position in the nasal cavity, the olfactory mucosa is not directly
exposed to the flow of inspired air entering the nose. Odorant molecules come in contact with
the olfactory mucosa by sniffing (i.e., by short, forceful inspirations). Sniffing produces turbulence in the air flow and thereby transports molecules to the receptor cells.
2. Innervation of the olfactory mucosa is by the olfactory nerve (cranial nerve I) and some
branches of the trigeminal nerve (cranial nerve V). The irritative character of some odorants is
due to stimulation of the free nerve endings of the trigeminal nerve.
3. Cells. The olfactory mucosa is composed of three cellular elements (Figure 1-51).
a. Receptor cells are bipolar neurons with a single dendrite that extends to the surface and
terminates in a knob. Projecting from the knob of each sensory neuron are cilia that have

Figure 1-51. The olfactory epithelium. Olfactory receptor cells are situated among supporting cells. The cilia lie
within the mucous layer covering the epithelium. New
receptors are generated from basal cells. Bowman's
glands, which contribute to the mucus secretion, are not
shown.

the familiar microtubule configuration of two central microtubules surrounded by a ring of


nine microtubules. The cilia reach the outer surface of the mucous layer. The axon that
leaves the basal end of the receptor cell is unmyelinated. Receptor cells degenerate and
have a life span of about 60 days.
b. Supporting cells have a columnar shape. Microvilli extend from the surface of these cells
into the mucous layer covering the nasal mucosa.
c. Basal cells are stem cells from which new receptor cells are formed. There is a continuous
replacement of receptor cells by mitosis of basal cells.

C. Receptor cell electrophysiology


1. Receptor potential. The adsorption of odorant molecules to the plasma membrane of the cilia
of the receptor cells generates a receptor potential in the receptor cell.
2. Electro-olfactogram (EOG) is a tracing of the slow monophasic negative potential detected
by an electrode placed on the olfactory mucosa. It reflects the local, graded receptor potentials
evoked in the population of olfactory receptors by an odorous stimulus. The amplitude of the
EOG increases with increases in the intensity of the stimulus.
3. Impulse initiation. Neural impulses are generated by the receptor potential. Spikes originate
at the point in the olfactory receptor where the cell narrows to form the olfactory fiber or

axon.
4. Spike activity. The resting activity in olfactory fibers is extremely low. Odorous stimulation
causes excitation of most fibers, but a reduction in firing rate, or inhibition, is noted in some
cases. Conduction velocity is slow because the fibers are unmyelinated and small in diameter.
S. Sensory coding. A specific olfactory receptor does not respond to a particular compound or
category of compounds; instead, an individual receptor responds to many odors. Furthermore,
no two receptor cells have identical responses to a series of stimuli. Sensory perception,
therefore, is based on the pattern of receptors activated by the stimulus.

XI. SPINAL REFLEXES. A reflex is an automatic response to a stimulus carried out by a relatively simple
neuronal network consisting of a receptor, an afferent pathway, and an effector organ. Spinal cord
reflexes are classified as originating from cutaneous receptors or from muscle receptors.
A. Cutaneous reflexes. The most important of the cutaneous reflexes is the withdrawal, flexor, or
pain reflex, by which a body part is withdrawn from the site of a painful stimulus. Several reflex
pathways within the spinal cord act together to coordinate the activity of all of the muscles
necessary to produce a smooth movement.
1. Receptors for the withdrawal reflex are the nociceptors located on the free nerve endings
of A delta and C fibers. Upon entering the spinal cord, the pain fibers synapse on many
interneurons. Some of these convey information to the CNS. Others form several reflex pathways that coordinate the withdrawal of the limb.
a. The major afferent pathway travels through several interneurons before synapsing on the
alpha motoneurons innervating the muscles used to withdraw the limb from the painful
stimulus. Because several interneurons are interposed between the afferent and efferent
neurons, the reflex is described as multisynaptic or polysynaptic. The interneurons form
several pathways of different lengths, through which the pain stimulus can reach the alpha
motoneuron.
(1) Reverberating circuits are a consistent feature of the withdrawal reflex pathways. In
these circuits, a branch from the axon of an interneuron in the reflex pathway feeds back
onto previously excited interneurons, causing them to become reexcited. Reverberating
circuits cause activity in the reflex pathway to continue even after the sensory receptor
has stopped firing. This is called afterdischarge.
(2) Under most circumstances, only the minimal number of muscles necessary to withdraw
the limb are activated. This is called local sign. For example, if the hand accidentally
touches a hot stove, just the hand may move away.
(3) When the stimulus is very strong, however, the withdrawal reflex may require the
involvement of more muscle groups. This is called irradiation. Thus, if the hand picks
up a hot coal, not only will the fingers open to drop it, but the entire arm will withdraw
and the individual may leap away from the site of painful stimulation.

b. Another important pathway formed by the interneurons within the spinal cord terminates
on the antagonistic motoneurons. This is an inhibitory pathway.
(1) By inhibiting the motoneurons that innervate muscles antagonistic to those withdrawing
the limb, this pathway ensures that the flexion movement is not impeded by contraction
of the extensors.
(2) This type of neuronal organization, in which the reflex pathway activating one group of
alpha motoneurons also inhibits its antagonistic motoneuron, is quite common within
the spinal cord and is called reciprocal innervation.
c. Finally, the interneurons form pathways that cross the spinal cord to innervate the
extensor motoneurons on the contralateral side. This is called a crossed extensor reflex.

2. Integration of the withdrawal reflex occurs on the alpha motoneuron, the final common
pathway through which all the afferent fibers act. If the excitatory pathways dominate, the
alpha motoneuron discharges a train of action potentials; if the inhibitory pathways dominate,
the neuron does not fire.

3. Effector organs of the withdrawal reflex are the skeletal muscles that cause withdrawal of
the limb. Although they are called flexors, these muscles are flexors in the physiologic, not the
anatomic, sense. For example, the muscles that cause the fingers to open in order to drop a
hot coal, although anatomically referred to as extensors, are considered to be flexors because
they are involved in the withdrawal reflex.

4. Behavioral responses. The anatomic organization of the withdrawal reflex produces the following characteristic behavior.

a. The withdrawal reflex has a relatively long latency because the afferent pathway uses small,
slowly conducting fibers and involves many synapses.

b. The afterdischarge that results from the parallel pathways and reverberating circuits causes
the response to outlast the stimulus. This keeps the affected limb away from the painful
stimulus while the brain determines where to put it.
c. The crossed extensor reflex produces a patterned response in which the affected limb flexes
while the contralateral limb extends. In the lower limbs, this allows the contralateral limb
to support the body while the other limb is raised off the ground.

B. Muscle reflexes. Two important reflexes originate in the muscles: the stretch reflex and the
lengthening reaction. The stretch reflex causes the reflex contraction of a muscle that is stretched.
For example, when the patellar tendon is tapped by a reflex hammer, it causes the quadriceps
muscle to be stretched. The stretched muscle contracts reflexly, causing the leg to be elevated
(called the knee jerk reflex). The lengthening reaction causes inhibition of alpha motoneurons
innervating muscles that are under tension, allowing them to lengthen.

1. The stretch reflex


a. The receptor for the stretch reflex is the muscle spindle. This complex, spindle-shaped,
encapsulated receptor contains muscle fibers that have both sensory and motor innervation
(Figure 1-52).

Dynamic gamma fiber


Group II
afferent fiber
la afferent fiber
Figure 1-52. Diagram of an intrafusal muscle fiber, showing its nuclear bag and nuclear chain fibers. The afferent
innervation (la and II fibers) and efferent innervation (gamma dynamic and gamma static fibers) of the intrafusal
muscle fiber also are illustrated.

(1) The number of spindles in each muscle depends on the task performed by the muscle.
Muscles involved in precision movements contain many more spindles than muscles
used to maintain posture. For example, hand muscles have about 80 spindles, which is
about 20% of the number of spindles contained in back muscles weighing 100 times
as much.
(2) The muscle fibers within the spindle are called intrafusal muscle fibers in contrast to
the extrafusal muscle fibers responsible for generating tension. There are two types of
intrafusal muscle fibers.
(a) The nuclear bag fibers are 30 J.t in diameter and 7 mm in length and are so-called
because their nuclei appear to be bunched up in the center of the cell as if in a bag.
About 2-5 nuclear bag fibers exist in a typical spindle.
(b) The nuclear chain fibers are 15 J.t in diameter and 4 mm in length and are so-called
because their nuclei are lined up in a single file in the center of the fiber. About
6-10 nuclear chain fibers exist in each spindle.
(c) The connective tissue surrounding the intrafusal fibers is continuous with that of the
extrafusal fibers. As a result, when the extrafusal fiber contracts the intrafusal fiber
is shortened, and when the extrafusal fiber stretches the intrafusal fiber is lengthened.
The muscle spindle, thus, is in parallel with the extrafusal muscle fibers.
(3) Two sensory neurons emerge from the muscle spindle.
(a) A single large fiber, called a group la fiber or primary ending, sends branches to
every intrafusal fiber within the muscle spindle.
(b) Several smaller neurons, called group II fibers or secondary endings, innervate
the nuclear chain fibers.
(c) The primary endings surround the center of the intrafusal muscle fiber; the secondary
endings terminate on either side of the primary endings.
(4) The efferent fibers to the muscle spindle are called gamma fibers because their axons
belong to the A gamma group of fibers. There are two types of gamma fibers.
(a) Static gamma fibers primarily innervate nuclear chain intrafusal muscle fibers and
produce tonic activity in the la afferent fibers.
(b) Dynamic gamma fibers primarily innervate nuclear bag intrafusal muscle fibers
and generate phasic activity in the la afferent fibers.
b. The la fiber enters the spinal cord through the dorsal root and sends branches to every alpha
motoneuron that goes to the muscle from which the la originated.
(1) In contrast to the withdrawal reflex, the stretch reflex has a rapidly conducting afferent
fiber, is monosynaptic, has a short latency, and does not exhibit afterdischarge and
irradiation.
(2) In the stretch reflex, as in the withdrawal reflex, the alpha motoneuron is the final
common pathway, serving as both an integrating center and efferent pathway.
(3) Also, like the withdrawal reflex, the stretch reflex is characterized by reciprocal innervation. Thus, when a stretch reflex is elicited, the muscle antagonistic to the stretched
muscle is inhibited, allowing the agonistic muscle to contract without interference.
c. Both extensor and flexor muscles exhibit stretch reflexes. These are elicited routinely during
a neurologic examination to test for damage to either the spinal cord or the sensory or motor
neurons. The gamma efferent fibers control the sensitivity of the receptors to stretch. Their
role is discussed in XI C 2.

2. The lengthening reaction


a. The receptors for the lengthening reaction are the Golgi tendon organs.
(1) These small (0.5 mm long) encapsulated receptors are located in the tendons, between
the muscles and tendon insertions.
(2) The Golgi tendon organs are stretched whenever the muscle contracts and, thus, in
contrast to the muscle spindle, are in series with the extrafusal muscle fibers.
(3) The Golgi tendon organs have neither muscle fibers nor an efferent innervation.
b. The afferent fiber innervating the Golgi tendon organ is called a group Ib fiber. It enters
the dorsal root and forms a disynaptic pathway, which ends on the alpha motoneurons that
send axons to the muscle from which the Ib fiber originated.
(1) The disynaptic pathway of the Golgi tendon organ is inhibitory to the alpha motoneuron.
(2) The Ib fiber, like all sensory fibers, releases an excitatory transmitter. To produce
inhibition, an inhibitory interneuron must be activated.
c. Like the stretch reflex, the lengthening reaction occurs in both flexor and extensor muscles,
lacks afterdischarge and irradiation, and displays reciprocal innervation.

d. Historically, the lengthening reaction has been described as a protective reflex in which a
strong and potentially damaging muscle force reflexly causes the muscle to be inhibited,
and, as a result, the muscle lengthens instead of trying to maintain the force and risking
damage. Although this is true, it is now clear that the reflex plays a more important role in
regulating tension during normal muscle activity. The lengthening reaction is described as
autogenic inhibition, which indicates that the force generated when the muscle contracts
is the stimulus for its own relaxation.

C. Role of the stretch reflex in the control of movement. The stretch reflex is used by the motor
control system to aid in the performance of a movement. During activity generated by the motor
command system, the la fibers from the muscle spindle inform the motor control system about
the changes in muscle length and provide the alpha motoneuron with a source of excitatory input
in addition to that coming from higher centers.
1. Figure 1-53 illustrates the effect of muscle stretch and contraction on la discharge when the
gamma motoneurons are not activated.
a. Stretching the extrafusal muscle fiber causes the intrafusal muscle fiber to be stretched.
Most of the stretch occurs in the central, most compliant region of the intrafusal fiber, which
lacks sarcomeres.
(1) When the intrafusal muscle fiber is stretched, the la fiber terminal is deformed. The
deformation causes a receptor potential, which generates a train of action potentials
(see Figure 1-538).
(2) The action potentials initially discharge at a frequency proportional to the velocity of
stretch and then adapt to fire tonically at a frequency proportional to the amount of
muscle stretch.
b. Shortening the extrafusal fiber causes the central region of the intrafusal muscle fiber to be
compressed.
(1) This reduces the deformation of the la fiber terminal, causing the la fiber to reduce its
firing rate (see Figure 1-530.

Intrafusal
muscle fiber
(nuclear bag)

Extrafusal
muscle fiber

A. Control

B. Stretch

C. Contract

Figure 1-53. (A) la afferent fiber. (8) The firing rate of the la afferent fiber increases when the extrafusal muscle fiber
is stretched because the intrafusal muscle fiber also is stretched. Most of the stretch occurs at the center region of
the intrafusal muscle fiber. (C) When the extrafusal muscle fiber contracts, the la afferent discharge decreases because
the intrafusal muscle fiber slackens (i.e., the intrafusal muscle fiber is unloaded>.

(2) The reduction of firing that occurs during muscle contraction, called unloading, is
functionally disadvantageous because the eNS stops receiving information about the
rate and extent of muscle shortening.
2. Unloading can be prevented by the activity of the gamma efferent motoneurons, as shown in
Figure 1-54. The gamma motoneurons cause the sarcomeres of intrafusal muscle fibers to
shorten as the extrafusal muscle fiber shortens. As a result, the central region of the intrafusal
muscle fiber remains stretched during muscle contraction, and unloading does not occur.
a. When the dynamic gamma motoneurons are fired, only the nuclear bag intrafusal fibers
shorten. Since the nuclear bag fibers are responsible for producing the phasic (i.e., velocitysensitive) portion of the la response to stretch, stimulation of the dynamic gamma fibers
causes an increase in phasic activity without affecting the tonic activity.
b. When the static gamma motoneurons are fired, only the nuclear chain fibers shorten. Since
nuclear chain fibers are responsible for the tonic component of the la response, stimulation
of the static gamma fibers causes an increase in the tonic (length-sensitive) level of la firing
without affecting the phasic level.
3. During a normal movement, the motor command system coactivates both the alpha and
gamma motoneurons. This diminishes the amount of unloading that occurs during muscle
contraction, allowing the eNS to determine if its motor commands are being carried out.
a. For example, when the motor control system issues a command to lift a weight, the alpha
and gamma motoneurons are coactivated.
(1) As the extrafusal muscle fiber shortens, the intrafusal muscle fiber sarcomeres also
shorten.
(2) If the two muscle fibers shorten at the same rate, then the central region of the intrafusal
fiber is neither compressed nor lengthened, thus keeping la activity at a constant level.
(3) The constant level of la input to the eNS during a movement indicates that the motor
command is being carried out.
b. If the weight to be lifted is underestimated by the eNS, the motor command system does
not activate a sufficient number of alpha motoneurons to lift the weight. Thus, the extrafusal
muscle fibers do not shorten.
(1) However, the intrafusal muscle fibers do shorten. But, because the tendon ends of the
muscle cannot move, the central portion of the intrafusal fiber lengthens.
(2) Stretching the central region of the fiber causes la activity to increase, indicating that
the motor command is not being carried out. The eNS uses this information to readjust
its command to the spinal cord.
(3) Even before the eNS responds to the information provided by the la fibers, the la activity
is used at the spinal cord level to adjust the alpha motoneuron activity to meet the
unexpectedly high load.
(a) Since the la fiber synapses on the alpha motoneuron, its activity increases the
excitability of the alpha motoneuron.
(b) The increase in excitability leads to an increase in the frequency of action potential
generation and an increase in muscle force development.

Gamma
motoneuron

1111111

Gamma
motoneuron
I 1111 II

Figure 1-54. The firing rate of the la afferent fiber does


not decrease if the gamma motoneuron discharge accompanies contraction of the extrafusal muscle fiber. Unloading is prevented because the intrafusal muscle fiber
also shortens. Since the center region of the extrafusal
muscle fiber does not contain sarcomeres, it does not
shorten and, in fact, may lengthen if the gamma motoneuron causes sufficient shortening of the intrafusal muscle
fiber sarcomeres.

4. Gamma loop. The eNS is theoretically capable of initiating movements directly by stimulating
just the gamma motoneurons, using a pathway called the gamma loop (Figure 1-55).
a. Increasing gamma motoneuron activity causes the intrafusal muscle fiber sarcomeres to
shorten, which, in turn, leads to stretching of the central portion of the intrafusal fiber and
activation of the la fiber. Firing the la fibers causes alpha motoneuron activity to increase,
which results in an increased amount and force of skeletal muscle activity.
b. Although the gamma loop can elicit movement on its own, it normally does not do so.
However, because of coactivation, the gamma loop is activated during all movements and
thus contributes to the excitability and firing rate of the alpha motoneurons.

XII. THE MOTOR CONTROL SYSTEM


A. Movement. The major components of the motor control system are shown in Figure 1-56. This
system is a complex and highly integrated network in which all parts work together to produce a
movement. The specific tasks assigned to each component can be analyzed as a basis for understanding how movements are coordinated.

1. The idea for a movement is generated within the cortical association areas of the parietal
lobe.
2. Then, the idea is transferred to the motor areas of the frontal lobe, where it is organized into
a motor command.
3. The command is sent to the spinal cord for execution of the movement. The basal ganglia,
cerebellum, brain stem, and spinal cord all participate in the production of a coordinated
movement by modifying the motor command.
a. The basal ganglia provide the motor patterns necessary to maintain the postural support
required for motor commands to be carried out properly.
b. The cerebellum receives information from the motor cortex about the nature of the intended
movement and from the spinal cord about how well it is being performed. This information
is used to adjust the motor command so that the intended movement is executed smoothly.
c. The brain stem is the major relay station for all motor commands except those requiring the
greatest precision, which are transferred directly to the spinal cord. The brain stem also is
responsible for maintaining normal body posture during motor activities.
d. The spinal cord contains the final common pathways through which a movement is executed. By selecting the proper motoneurons for a particular task and by reflexly adjusting the
amount of motoneuron activity, the spinal cord plays an important role in the coordination of
motor activity.

Spinal cord

Intrafusal
muscle fiber

Alpha motoneuron
Extrafusal
muscle fiber
Figure 1-55. The gamma loop increases the firing of the alpha motoneuron during muscle contraction. The loop
begins with the gamma motoneuron, which discharges to cause intrafusal muscle fiber contraction. This leads to an
increase in la afferent fiber activity, which, in turn, causes increased alpha motoneuron discharge via a monosynaptic
reflex.

Association cortex

Motor cortex

Spinal
cord

I
Skeletal muscle

Figure 1-56. Diagram illustrating the


extensive interconnections between
the components of the motor control
system. Note that all of the descending pathways except for the pyramidal tract (thick arrow) communicate
with the spinal cord through the brain
stem.

8. The spinal cord


1. Physiologic anatomy
a. Motoneuron pool. The alpha and gamma motoneurons within the spinal cord that innervate
a particular muscle are collected together into a motoneuron pool.
(1) The alpha and gamma motoneurons are randomly distributed within the pool and
overlap to some extent with cells from other motoneuron pools.
(2) Usually, the motoneurons leave the spinal cord in several contiguous ventral roots and
then combine into a single motor nerve containing the alpha and gamma motoneurons
as well as the la, Ib, and II afferent fibers from the muscle.
b. Motor unit. Each motoneuron and all the muscle fibers it innervates form a motor unit. The
motor units within a muscle vary in size from a few muscle fibers to several thousand
muscle fibers. Muscles that perform precise movements have smaller motor units than those
responsible for large body movements and for maintaining posture.
(1) Whenever an alpha motoneuron fires, all of the muscle fibers in its motor unit are
activated.
(2) The muscle fibers belonging to a single motor unit are dispersed throughout the muscle
so that the force they produce is distributed evenly.
c. Muscle fibers. All of the muscle fibers in a motor unit are of the same physiologic type and
are categorized according to their histochemical and contractile characteristics.
(1) Fast-twitch fatigable (FF) fibers contract quickly, fatigue easily, and have the following
characteristics.
(a) Rapid contractile speeds. This results from the high myosin-ATPase activity of
their cross-bridges and the rapid sequestering of Ca2+ by their SR.
(b) Rapid fatigue. This occurs because FF fibers have few mitochondria and, thus,
cannot make use of oxidative metabolism. FF fibers rely on glycolysis for their ATP
supply and fatigue when their glucose stores are depleted.
(c) Sparse capillary supply. Because FF fibers do not make use of oxidative metabolism, the growth of surrounding capillaries is limited.
(d) Large size. Although FF fibers cannot sustain activity for long periods of time, they
can generate large contractile forces. Their large size is not a disadvantage from a

diffusional point of view because they do not make use of oxidative metabolism.
(2) Slow-twitch (S) fibers contract slowly, are virtually untiring, and have the following
characteristics.

(a) Slow contractile speeds. This is due to the low myosin-ATPase activity of their
cross-bridges and the slow sequestering of Ca2+ by their SR. These characteristics
reduce the amount of ATP used by S fibers and, thus, contribute to their resistance
to fatigue. Their long contraction times make summation and tetanus possible at
low frequencies of stimulation.
(b) Great resistance to fatigue. This is a consequence of the ability of S fibers to use
oxidative metabolism as a primary source of ATP.
(c) A rich capillary supply provides the oxygen needed by S fibers.
(d) Small size. Although S fibers cannot produce a large amount of force, they can
sustain force for a long time. Their small size is necessary for oxygen to diffuse into
the center of the fiber and for waste products to diffuse out of the fiber.
(3) Fast fatigue-resistant (FR) fibers have characteristics that are intermediate to those of
the FF and S muscle fibers.

2. Task performance. During the performance of a motor task, the small motor units, because
they are more excitable, are recruited before the large ones. This size principle of motor unit
selection has significant physiologic advantages.

a. To perform a precision movement requiring small amounts of force, it is advantageous


to use small motor units. When more force is required, larger motor units must be activated.
(1) The proper motor units are selected automatically because the motoneuron pool is
organized according to the size principle.
(a) When a small amount of force must be applied, the motor cortex provides a minimal
amount of input to the motoneuron pool, activating only the smallest motor units.
(b) If more force is required, the motor cortex increases its input to the motoneuron
pool, and larger motor units are recruited.
(2) Because of the size principle, the motor cortex does not need to specify the particular
motoneuron to be recruited during a movement. This simplifies the organization of the
motor command structure by reducing the number of cortical neurons that are involved
in generating a movement.
b. To perform a task requiring endurance, the fatigue-resistant motor units must be recruited.
When power is required, the larger motor units must be recruited.
(1) Again, motor unit selection is accomplished automatically because of the size principle.
(a) When an endurance movement must be performed, the motor cortex provides a
minimal input to the spinal cord, and the smallest, most fatigue-resistant motor units
are recruited.
(b) When the amount of force being generated by the muscle is not sufficient to execute
the movement, the motor cortex increases its input and recruits more motor units.
In all cases, the most fatigue-resistant fibers are recruited first without requiring that
the motor cortex determine which motoneuron to activate.
(2) The relation between fatigue resistance and motoneuron size also is a consequence of
the size principle. A muscle fiber's fatigability can be altered by its activity.
(a) When a muscle fiber is contracted repeatedly, its oxidative capacity and blood
supply increase, making the fiber more resistant to fatigue. Conversely, when a
muscle fiber is required to contract against large afterloads, the fiber increases in
size.
(b) Small motor units are recruited first. Thus, they are involved in all movements and,
consequently, develop fatigue resistance.

C. The brain stem contains the medulla, pons, midbrain, and parts of the diencephalon. Neuronal
circuits within these areas control many physiologic functions including blood pressure, respiration,
body temperature, sleep, and wakefulness. In addition, the reticular formation and vestibular
nuclei are important components of the motor control system.

1. The reticular formation is the relay station for all descending motor commands except those
traveling directly to the spinal cord through the medullary pyramids.

a. Normal function. The reticular formation receives and modifies motor commands to the
proximal and axial muscles of the body and is involved in the performance of all motor
activities except those fine movements performed by the distal muscles of the fingers and
hands. The reticular formation also is responsible for maintaining normal postural tone.
(1) Neurons within the pontine reticular formation send axons to the spinal cord in the
medial reticulospinal tract and are excitatory to the alpha and gamma motoneurons
innervating the extensor antigravity muscles.

(2) These neurons are prevented from firing too rapidly by inhibitory input derived from
the cerebral and cerebellar components of the motor control system. The amount of
inhibition is increased to reduce postural tone and is decreased to enhance postural
tone. The withdrawal of inhibition, called release of inhibition, frequently is used to
increase activity within the CNS.
b. Lesions
(1) If the brain stem is severed from the spinal cord, the syndrome of spinal shock results.
(a) The initial result of removing the spinal cord from the control of the brain stem is
the complete loss of reflex activity, which can last for days in cats and for months
in humans.
(b) When reflexes return, they no longer are under the influence of the brain stem and,
therefore, do not follow their normal patterns. For example, the local sign that
characterizes the withdrawal reflex disappears. Instead, even a light touch on the
foot can cause activation of all the flexor muscles in the body.
(c) The excitability of the motoneurons ultimately becomes greater than normal, causing
particular groups of muscles to contract continuously.
(2) If the cerebral and cerebellar inhibitory inputs to the reticular formation are removed
by severing the brain stem above the pontine reticular formation, the condition of
decerebrate rigidity results.
(a) Without the inhibitory input from these higher centers, the pontine reticular formation fires uncontrollably, subjecting both the alpha and gamma antigravity motoneurons to intense excitation.
(i) The excessive firing of the alpha motoneurons causes the antigravity muscles
(i.e., the leg extensors and the arm flexors) to contract continuously.
(ii) The firing of the gamma motoneurons activates the gamma loop, causing increased discharge of the la afferent fibers. These reflexly add to the alpha motoneuron excitation produced by the reticulospinal tract.
(b) Decerebrate rigidity is analogous to the condition of spasticity.
(i) Spasticity normally is caused by lesions of the descending pathways from the
motor cortex to the reticular formation.
(ii) Spasticity is characterized by an increased amount of antigravity muscle activity.
This activity is increased further when the affected muscle is stretched, due to
the increased activity of the gamma motoneurons. Because these fibers are firing
at higher than normal rates, the muscle spindles become more sensitive to stretch.
(iii) Cutting the dorsal roots reduces the amount of la input to the spinal cord and
reduces the amount of alpha motoneuron activity. This reduces the spasticity.
2. Vestibular system. Vestibular system reflexes are responsible for maintaining tone in antigravity
muscles and for coordinating the adjustments made by the limbs and eyes in response to
changes in body position.
a. Vestibular receptors. The receptors that initiate the vestibular reflexes are located within
the labyrinth (see Figure 1-35)-a system of fluid-filled, membrane-bound canals and sacs
that are continuous with the cochlea. The labyrinth consists of the sac-like otolith organs
(i.e., the saccule and utricle) and three semicircular canals. The saccule communicates
directly with the cochlea, which is beneath it, and the utricle, which is above it. Both ends
of all three semicircular canals emerge from the utricle. Each canal has an expanded end,
called the ampulla, which contains the receptor cells.
(1) Orientation
(a) The semicircular canals are responsible for detecting angular accelerations of the
head. The horizontal canal lies in a plane that is approximately parallel to the earth
when the head is held in a normal upright position. The vertical canals lie in vertical
planes. The plane of the anterior vertical canal is oriented along a line from the
center of the head toward the eye, while the plane of the posterior vertical canal
is oriented along a line from the center of the head toward the ear. The orientation
of the posterior canal on one side of the head is roughly parallel to the orientation
of the anterior canal on the other side of the head.
(b) The otolith organs are responsible for detecting linear acceleration and the static
position of the head. The receptors within the utricle are oriented in a horizontal
plane, while those in the saccule are oriented in a vertical plane.
(2) Receptor cells. The receptor cells of the vestibular system are similar to those of the
cochlea. However, unlike the auditory receptor cells, the cilia of the vestibular hair cells

are polarized (Figure 1-57). A large cilium, called the kinocilium, is located at one end
of the cell. All of the others are called stereocilia. Whenever the stereocilia are bent
toward the kinocilium, the cell is depolarized. When the stereocilia are bent away from
the kinocilium, the cell is hyperpolarized.
(a) The hair cells of the semicircular canals are located on a mass of tissue (the crista)
within the ampulla. The cilia are embedded in a gelatinous structure called the
cupula, which completely fills the ampullar space.
(i) When the head begins to move, the fluid within the semicircular canals lags
behind and pushes the cupula backward. This causes the cilia of the hair cells
to bend, producing either a depolarization or hyperpolarization depending on
whether the stereocilia are pushed toward or away from the kinocilium.
(ii) After 15-20 seconds of continuous movement at a constant velocity (e.g., the
movement of a twirling dancer), the velocity of fluid movement catches up to
that of the head, and the cupula returns to its resting position. This causes the
cilia to return to their upright position and the hair cell to return to its resting
membrane potential. Thus, the semicircular canals signal changes in motion
(acceleration) and are insensitive to movements at a constant angular velocity.
(iii) When the head stops moving, the fluid within the semicircular canals continues
to move, pushing the cupula forward. This causes the cilia to bend in the opposite
direction. Thus, if the original movement caused the hair cell to depolarize, the
hair cell hyperpolarizes when the movement ceases.
(iv) The hair cells of the horizontal canals are oriented with the kinocilium located
closest to the utricle. Head movements that bend the stereocilia toward the
utricle (utriculopedal movements) cause the hair cells to depolarize, while movements that bend the stereocilia away from the utricle (utriculofugal movements)
cause hyperpolarization. For example, when the head is rotated toward the left,
the endolymph in the left horizontal semicircular canal pushes the cupula and
stereocilia toward the utricle (Le., the fluid lags behind the head movement and
causes the cupula to move toward the right), causing the hair cells to depolarize
(Figure 1-58). At the same time, the hair cells within the right horizontal canal
are pushed away from the utricle and hyperpolarize. When the head stops
rotating, the fluid continues to move, pushing the cupula in the opposite direction.
This causes the hair cells in the left horizontal canal to hyperpolarize and those
in the right semicircular canal to depolarize.
(v) The vertical canals also work in pairs: when the anterior vertical canal on one
side is stimulated, the posterior vertical canal on the other side is inhibited. In
the vertical canals, the kinocilium is located on the side of the hair cells away
from the utricle.
(vi) Because each of the three canals is oriented in a different plane, movement of
the head in any direction causes a unique pattern of activity to be generated by
the semicircular canals. This information is used by the eNS to interpret the

Kinocilium
Stereocilia

II

~\

Hair cell

\
Vestibular nerve
fiber

Figure 1-57. Diagram illustrating the polarized hair cells


of the vestibular system. The large cilium is called the
kinocilium. The smaller cilia are called stereocilia. When
the stereocilia are bent toward the kinocilium, the cell
depolarizes; when the stereocilia are bent away, the cell
hyperpolarizes.

Rotation of head
~

Cupula

Left

(j

Relative
direction of
fluid

Right

m'~me",

11111111
Excitation

Inhibition

Figure 1-58. Diagram illustrating the movement of the


endolymph within the semicircular canal when the head
rotates to the left. The fluid lags behind the head and
pushes the cupula toward the right. As indicated, the
stereocilia are bent toward the kinocilium on the left and
away from the kinocilium on the right. Thus, the hair cells
within the left horizontal canal are stimulated and those
on the right are inhibited.

speed and direction of head movement and to make the appropriate adjustments
in posture and eye position.
(b) The hair cells of the utricle and saccule are located on a mass of tissue called the
macula.
(i) The cilia are enmeshed in a gelatinous structure filled with small calcium carbonate crystals called otoconia.
(ii) Because the otoconia are heavier than the fluid of the otolith organs, they tend
to sink and, thus, can bend the cilia of the hair cells.
(iii) When the head deviates from the horizontal position, the hair cells bend. The
kinocilium of each hair cell is oriented in a different plane so that, regardless of
the direction in which the head is tilted, some of the hair cells are maximally
stimulated while others are maximally inhibited.
(iv) Linear acceleration, which occurs when jumping down stairs or accelerating
forward in a car, also causes the otoconia to be displaced and to stimulate the
otolith organs.
b. Neuronal pathways. The hair cells of the labyrinth synapse on the vestibulocochlear nerve
(cranial nerve VII!), which projects to the vestibular nuclei within the brain stem. In addition,
the vestibular nuclei receive inhibitory input from the cerebrum and cerebellum.
(1) The vestibular nuclei send their axons into the spinal cord through a number of vestibulospinal tracts. This input is excitatory to antigravity alpha motoneurons.
(2) If the inhibitory input from the cerebrum and cerebellum is removed, the vestibular
nuclei greatly increase their firing rate and contribute to the syndrome of decerebrate
rigidity.
(a) The rigidity produced by the vestibular nuclei differs from that produced by the
reticular formation in that the vestibular nuclei primarily affect the alpha motoneurons, rather than both alpha and gamma motoneurons.
(b) Spasticity produced by the vestibular nuclei, therefore, is not reduced greatly by
cutting the dorsal roots and eliminating the la input. This type of spasticity is called
alpha rigidity to distinguish it from the type of spasticity caused by the reticular
formation, which is called gamma rigidity.
(c) When discussing the effects of the brain stem on antigravity muscles, the terms
spasticity and rigidity are used interchangeably. Clinically, however, spasticity and
rigidity are not alike. Spasticity refers to the condition in which the stretch reflexes
of the antigravity muscles are increased due to increased activity of the alpha or
gamma motoneurons. Rigidity refers to the condition seen in Parkinson's disease
(see XII 0 2 d), in which there is increased activity of all of the muscles at a joint.
c. Vestibular reflexes. The vestibular system participates in a number of reflexes.
(1) The most important of these reflexes maintains visual fixation during movement of the
head.

(a) For example, if the head is rotated to the left, the eyes

move

toward the right in

order to prevent an image from moving off the fovea. When the eyes have rotated
as far as they can, they are rapidly returned to the center of the socket. If rotation

of the head continues, the eyes once again move in the direction opposite the head
rotation. These movements of the eyes are called nystagmus.
(b) The slow movement of the eyes to maintain visual fixation is initiated by the activity
within the semicircular canals. When the head rotates to the left, the activity of
receptors in the left horizontal canal causes the eyes to move toward the right.
(c) After the body has been rotated and the movement ceases, the receptors within the
right horizontal canal are stimulated, and these cause postrotary eye movements
to occur in the opposite direction. That is, the eyes slowly move to the left until
they reach the end of the socket, at which point they return quickly to the center.
These movements continue until the cupula returns to its resting position.
(d) Lesions within the vestibular pathways can cause nystagmus to occur spontaneously,
while damage to the vestibular receptors can prevent nystagmus from occurring
during rotation of the head.
(2) Another important reflex is initiated by receptors within the otolith organs and occurs
when an individual walks down stairs or jumps from a platform.
(a) When making such a descent, the muscles in the leg begin to contract before the
feet reach the ground in order to cushion the force of impact. The otolith receptors
responsible for this reflex are stimulated by the linear acceleration of the head that
occurs during the descent.
(b) Individuals lacking otolith reflexes are prone to leg injuries because of the large
contact forces that occur when descending from a height (e.g., stepping off a bus).
D. The basal ganglia are interconnected nuclei within the cerebrum and do not make direct connections with the spinal cord. It appears that the major function of the basal ganglia is to aid the motor
cortex in generating commands concerned with controlling proximal muscle groups during a
movement. For example, when the hand is used to write on a blackboard, the large muscles of
the arm and shoulders are used to hold the hand in its proper position for writing. The coordination
of these muscle groups is believed to be under the control of the basal ganglia.
1. Physiologic anatomy. The basal ganglia consist of the striatum (which is composed of the
caudate nucleus and putamen), the globus pallidus (also called the pallidum), the subthalamic nucleus, and the substantia nigra.
a. These nuclei have complex interconnections.
(1) The globus pallidus is in the center of three major feedback loops.
(a) One pathway leaves the globus pallidus and travels through the thalamus, cortex,
and striatum before returning to the globus pallidus.
(b) Another pathway follows the same route from the globus pallidus to the thalamus,
but travels to the striatum and back to the globus pallidus without going through the
cortex.
(c) The third pathway connects the globus pallidus with the subthalamic nucleus.
(2) Another important feedback loop is established between the striatum and the substantia
nigra.
b. These pathways between the nuclei are described in a highly diagrammatic way because
not enough is known about how the basal ganglia function to allow a more realistic and
more detailed description.
2. Lesions in the basal ganglia produce characteristic deficits in motor behavior.
a. Lesions in the globus pallidus result in an inability to maintain postural support of the
trunk muscles. The head bends foward so that the chin touches the chest, and the body
bends at the waist.
(1) The motor deficits are not due to muscular weakness or failure of voluntary control
because individuals with these lesions can stand upright when requested to do so.
(2) Since the globus pallidus is the major outflow tract of the basal ganglia, it is possible
that the motor deficits occur because the cortex is deprived of information it needs to
automatically control the trunk muscles.
b. lesions in the subthalamic nucleus cause spontaneous, wild, flinging, ballistic movements
of the limbs. This syndrome is called hemiballismus.
(1) The movements, which are caused by a release of inhibition, appear on the side opposite
the lesion.
(2) Because the movements appear to be like those performed when an individual is thrown
off balance, the subthalamic nucleus is believed to be involved with controlling the
centers that issue the motor commands for balance.

(a) Normally, the subthalamic nucleus responds to the need for initiating the balancing
movement by momentarily withdrawing its inhibition from these centers.
(b) When there is a lesion in the subthalamic nucleus, these centers no longer are under
inhibitory control, and the movements are generated spontaneously.
c. Lesions within the striatum produce a variety of motor syndromes related to a release of
inhibition.
(1) Huntington's chorea is characterized by continuous, uncontrollable, quick movements
of the limbs.
(2) In athetosis the limbs, fingers, and hands perform continuous, slow, irregular, twisting
motions.
(3) Dystonia is typified by twisting, tonic-type movements of the head and trunk.
d. Lesions within the substantia nigra produce Parkinson's disease.
(1) This disease is characterized by rigidity, hypokinesia (reduction in voluntary movement), and tremor.
(a) The rigidity in Parkinson's disease involves all of the muscles at a joint and, thus, is
different from spasticity that is associated with cortical lesions. It has been described
as lead-pipe rigidity because the rigid limb, when moved, remains where it is
placed. The rigidity seen in Parkinson's disease also has been described as cogwheel
rigidity. When an examiner tries to move the limb, the limb periodically gives way
and then reestablishes its resistance to movement like cogs on a wheel.
(b) The hypokinesia is not due to a loss of muscle strength or power because normal
movements occur under certain conditions. It also is not caused by rigidity because
some cases of hypokinesia occur in the absence of any rigidity. The hypokinesia
reduces the movement patterns normally associated with motor activity. For example, the arms do not swing during walking, and facial expressions do not change
during conversation.
(c) The tremor associated with Parkinson's disease occurs at a frequency of about
4-7 cycles/sec. This is slower than the normal physiologic tremor, which has a
frequency of about 10 cycles/sec. The tremor in Parkinson's disease occurs at rest
and usually disappears during voluntary activity.
(2) The lesion of Parkinson's disease involves a pathway that uses dopamine as its neurotransmitter. Some success has been achieved in treating Parkinson's disease with
L-dopa, a precursor of dopamine that can cross the blood-brain barrier. Dopamine
inhibits the striatum sufficiently to reduce some of the clinical signs of the disease.

E. The cerebellum is intimately associated with all aspects of motor control. Its removal produces
no deficits in emotional or intellectual function but causes profound disturbances in the ability to
produce smooth, coordinated movements. The functions of the cerebellum appear to be related
to the control of the timing, duration, and strength of a movement.
1. Anatomy
a. Two transverse fissures divide the cerebellum into three lobes: the anterior, posterior, and
flocculonodular lobes. These lobes have developed at different times during evolution.
(1) The oldest lobe, the flocculonodular lobe, is called the archicerebellum.
(2) Next to evolve was the anterior lobe, which is called the paleocerebellum.
(3) The last lobe to evolve was the posterior lobe, which is called the neocerebellum.
b. Another description of the cerebellum is based on the connections it makes with other
components of the motor control system.
(1) The entire anterior lobe, and those parts of the posterior lobe that receive information
from the spinal cord, are called the spinocerebellum.
(2) The remainder of the posterior lobe receives its input from the cerebral cortex and,
thus, is called the cerebrocerebellum.
(3) The flocculonodular lobe is functionally related to the vestibular apparatus and so is
also called the vestibulocerebellum.
2. Lesions in the various divisions of the cerebellum produce characteristic motor deficits.
a. Lesions in the vestibulocerebellum cause deficits related to the loss of vestibular function,
principally, a loss of equilibrium and ataxia. Individuals with this type of lesion are unable
to maintain their balance and, so, tend to fall over when standing. When walking, these
individuals keep their legs far apart and stagger from place to place.

b. Lesions in the spinocerebellum have no obvious effects in humans, probably because


spinocerebellar functions can be assumed by the neocerebellum. In cats, lesions in the
anterior lobe cause an increase in the tone of the antigravity muscles.
c. Lesions in the cerebrocerebellum cause small motor deficits unless a wide area of the
cerebellar cortex is affected. If the outflow pathways are damaged, however, the ability to
produce smooth, coordinated movements is lost.
(1) A major sign of neocerebellar disease is decomposition of movement. Instead of all
the muscles acting in a coordinated way to produce a movement, the muscles are used
one at a time. For example, when reaching for an object, the arm first extends at the
shoulder, followed by extension at the elbow, and finally by the movement of the hand.
(2) Dysmetria (the inability to stop a movement at the appropriate time or to direct it in
the appropriate direction) is another sign of neocerebellar disease. These effects result
from the loss of the neuronal circuitry required to control the duration and strength of
a movement.
(3) The intention tremor that results from neocerebellar lesions also is related to the
inability to time and sequence movements properly. The intention tremor is different
from the resting tremor (spontaneous tremor) of Parkinson's disease and appears to
occur because an entire movement cannot be directed by a single motor command.
Instead, the movement is directed partway to the target and then halted. Several other
motor commands, each directing the movement closer to the target, are required before
the movement is completed.
(4) Adiadochokinesia is the inability to make rapidly alternating movements such as turning
the hands back and forth. This, too, appears related to the inability to time the duration
of a movement, which occurs as a consequence of lesions within the neocerebellum.
F. The cerebral cortex contains the neuronal circuits responsible for the conception and generation
of motor commands. Two parallel systems of descending pathways originate in the cerebral cortex:
the pyramidal and extrapyramidal systems. Clinically, these systems are considered together
because lesions within the cortex almost always involve both of them. These systems are functionally different, however, and should be considered separately.
1. Physiologic anatomy
a. Several areas within the cortex are responsible for generating motor commands.
(1) The primary motor area occupies the frontal cortex, just in front of the central sulcus.
It is topographically organized, with the cortical areas that control the muscles of the
hands and face occupying more space than the cortical areas that control the muscles
of the limbs and trunk.
(2) The supplementary motor area is located on the medial surface of the cortex, and it,
too, has a complete representation of the entire body.
(3) The secondary motor area occupies part of the parietal lobe, just across the central
sulcus from the region of the primary motor area that controls the facial muscles. The
neurons in this area affect movements on both sides of the body.
b. All of these cortical areas send projections to both of the descending motor systems.
(1) The axons of the pyramidal system travel directly from their origin in the cortex to their
destination in the spinal cord.
(a) However, they also send collateral fibers to all areas of the motor control system
and, thus, communicate their motor command to the basal ganglia, cerebellum, and
brain stem.
(b) The cortical cells that synapse directly on cranial nerves controlling facial muscles
perform the same function as pyramidal tract neurons and, thus, are considered
part of the pyramidal system.
(c) The pyramidal tract system is responsible for controlling muscles that make precision
movements. These include the muscles that move the fingers and hands and the
muscles that produce speech.
(2) The axons of the extrapyramidal system end on relay neurons within the brain stem.
These neurons influence the spinal cord through the reticulospinal tracts.
2. Lesions
a. Lesions in the pyramidal system cause relatively small motor deficits considering the

importance of this system and the large number of neurons involved (over 1 million in each
pyramid). The major deficit resulting from pyramidal tract lesions is weakness and a loss of
precision in the muscles controlling fine movements of the fingers.

b. Lesions within the extrapyramidal system produce more profound conditions. These
lesions cause the release of cerebral inhibition of the reticular formation, which leads to the
production of spasticity [see XII C 1 b (2)].

XIII. SLEEP AND CONSCIOUSNESS. Many of the body's regulatory mechanisms vary in their activity
during the day. For example, body temperature is about 1 C higher during the early evening than it
is at dawn, and adrenocortical hormones are secreted at levels that are higher in the morning than
they are at night. The cycle of these diurnal, or circadian, rhythms is roughly 24 hours. However,
if an individual is isolated from the environmental stimuli that indicate the normal day-night periods,
the cycle time lengthens, demonstrating that the circadian rhythms are not rigidly linked to the rotation
of the earth but can be driven by an individual's internal biological clock. It is necessary to understand
these normal variations in physiologic activities when evaluating pathologic functions. For example,
it would be misleading to compare fevers or blood cortisol levels obtained at different times of the
day. The most obvious, and probably most important, diurnal rhythm is the sleep-wake cycle.
A. Assessment of sleep states. When awake, an individual is able to perform all activities that are
required for individual and species survival (e.g., eating, drinking, learning, procreating). When
asleep, an individual is not aware of the environment and is unable to perform activities that
require consciousness. The presence of sleep can be assessed by behavioral analysis. An individual
who does not move and does not respond when spoken to or touched probably is asleep.
However, a more accurate assessment of sleep can be obtained from an EEG.

1. Obtaining the EEG


a. The EEG is obtained by placing electrodes on the scalp. The location of the electrodes
and the amplification and paper speed of the polygraph used for recording the EEG are
standardized. (The same is true for the EKG, which measures cardiac electrical activity; see
Ch 2 III C.)
b. The brain waves recorded by an EEG represent the summated activity of millions of cortical
neurons. The IPSPs, EPSPs, and the passive spread of electrical activity into the dendrites
of these neurons, rather than their action potentials, form the basis for the EEG.
2. Variations in the EEG during sleep and wakefulness. The two states of sleep-slow-wave
and fast-wave sleep-have characteristic EEG patterns.
a. When an individual is awake, the electrical activity recorded from the brain is asynchronous
and of low amplitude (Figure 1-59). This type of brain electrical recording is called a beta
wave.
b. If an individual sits quietly for a while, the brain waves gradually become larger and highly
synchronized. The typical resting EEG pattern has a frequency of 8-13 cycles/sec and is
called an alpha wave. When the eyes open or when conscious mental activity is initiated,
the EEG shifts from an alpha to a beta pattern. This is called alpha blocking.
c. As consciousness is reduced still further, an individual enters a state of sleep called slowwave sleep (SWS). SWS progresses in an orderly way from light to deep sleep (see Figure
1-59).

Beta wave

Alert

Alpha wave

Resting

Alpha wave

light sleep

,
, l'l

'i

!
(

Theta wave

..
'J

Moderate sleep

Delta wave

Deep sleep

I
Time (sec)
Figure 1-59. As an individual passes from wakefulness to deep sleep, the EEG wave increases in amplitude and
decreases in frequency. Sleep spindles indicate the presence of light sleep.

(1) Light sleep is characterized by an EEG that shows high-amplitude waves of about 1215 cycles/sec, called sleep spindles, which periodically interrupt the alpha rhythm.
(2) During moderate sleep, the EEG displays slower and larger waves called theta waves.
(3) The deepest stage of SWS produces an EEG pattern with very slow (4-7 cycles/sec)
and large waves called delta waves.
(4) Behaviorally, SWS is characterized by a progressive reduction in consciousness and an
increasing resistance to being awakened. Muscle tone is reduced, the heart and respiratory rates are decreased, and, in general, body metabolism is slowed.
d. The other type of sleep, called fast-wave or desynchronized sleep, is characterized by
the same high-frequency and low-amplitude EEG pattern that is seen in the waking state. In
this case, however, the individual clearly is unresponsive to environmental stimuli and, thus,
is asleep. For this reason, fast-wave sleep also is called paradoxical sleep.
(1) Because this state of sleep is characterized by the presence of rapid eye movements, it
also is called rapid eye movement (REM) sleep.
(2) Because dreaming occurs during REM sleep, it is also called dream sleep.
(3) Behaviorally, REM sleep is quite different from SWS.
(a) It is as difficult to arouse an individual from REM sleep as it is from deep sleep.
However, when awakened from REM sleep, the individual is immediately alert and
aware of the environment.
(b) The eyes are not the only organ that is active during REM sleep. The middle ear
muscles are active, penile erection occurs, heart rate and respiration become irregular, and there are occasional twitches of the limb musculature. Fortunately, muscle
tone also is reduced tremendously during REM sleep so the frequency and intensity
of muscle twitching do not produce injuries or awaken the individual.
B. The sleep cycle. There is an orderly progression of sleep stages and states during a typical sleep
period (Figure 1-60).

1. When an individual falls asleep, the light stage of SWS is entered first. During the next hour or
so, the individual passes into progressively deeper stages of sleep until deep sleep is reached.
After about 15 minutes of deep sleep, the depth of sleep starts to decrease and continues to
do so until the individual reenters the light stage of sleep (about 90 minutes after the start of
the first sleep cycle). At this point, the individual passes from SWS to REM sleep.
2. This cycle repeats itself about five times during the night. However, as can be observed (see
Figure 1-60), after the second cycle, the ~lLQg!\Y.~e.Q_ peri99~5?.!!=_M sleee b~come
ili9r~.r and the duration of ea.~.b .Re,do.d....oLREM.~Jeepb~C:.9.D1~Ll9Jlg.eJ,:., Afso, as morning
approaches, an indiviauarspends less time in the deeper stages of SWS and periodically
awakens.
----------3. The sleep cycle indicated (see Figure 1-60) is typical of an adult. The cycle varies greatly with
age.
a. During infancy, about 16 hours of every day are spent asleep. This drops to 10 hours during
childhood and to 7 hours during adulthood. Elderly individuals spend less than 6 hours of
each day sleeping.
Awake

Light sleep

Moderate
sleep

Deep sleep

I
2

I
I
3
4
Time (hours)

I
5

I
7

I
8

Figure 1-60. Diagram indicating the pattern of sleep during one sleep cycle, As the evening progresses, the depth
of slow-wave sleep decreases and the duration and frequency of rapid eye movement (REM) sleep episodes increase.
Note that occasional periods of wakefulness occur during the night

b. It is interesting to note that prematurely born infants spend about 80% of their sleep time
in REM sleep, whereas full-term infants spend about 50% (8 hours) in REM sleep. The total
time spent in REM sleep is reduced to about 1.5-2 hours by puberty and remains unchanged
thereafter.
c. During infancy and childhood, therefore, the reduction in~l~ep time from 16 to 10 hours
occurs<!tI'fLQ~ ~riJir~tyby.areduction of the amount of time spent in REM sleep. In adulthood,
tne:reductionjn sleep time is caused by a reduction in the time spent in" the deep stages of
SWS.

C. Physiologic basis for sleep. Areas throughout the entire brain participate in the sleep-wake cycle.
1. The waking state is maintained by the ascending reticular activating system, which is a
diffuse collection of neurons within the medulla, pons, midbrain, and diencephalon. Electrical
stimulation anywhere within this area causes the EEG pattern to change abruptly from that of
the sleep state to that of the waking state. This is called a cortical alerting or arousal response.
2. The sleep state does not result from the passive withdrawal of arousal. Two sleep centers exist
in the brain stem; one is responsible for producing SWS, and the other produces REM sleep.
a. The SWS center is located in a midline area of the medulla containing the raphe nuclei.
(1) The neurons within these nuclei use serotonin (5-hydroxytryptamine) as a neurotransmitter.
(2) Administration of serotonin directly into the cerebral ventricles of experimental animals
induces a state of SWS, whereas lesions in this region induce a permanent state of
insomnia.
b. The REM sleep center is located in specific nuclei of the pontine reticular formation, including
the locus ceruleus, which uses norepinephrine as a neurotransmitter. lesions within this
area eliminate the electrophysiologic and behavioral signs of REM sleep.
3. Sleep disorders. As noted previously, there is a cycling between SWS and REM sleep during
a normal sleep period.
a. In narcolepsy, REMsJeep.is...el}!ered dire.f!lyJrom the waking state.
(1) Individuals suffering from narcolepsy often report an intense feeling of sleepiness just
prior to an attack, although sleep sometimes occurs without warning.
(2) In some narcoleptics, the profoundreduction'in-i11liscTetonecharacterTstic of REM sleep
can occur w.itb.9ulJoss of consciOusnes!COurTng'sucli-analtack, called cataplexy, the
individual suddenly'6ec()mes-parafyzea,falls to the ground, and is unable to move.
(3) Another symptom associated with narcolepsy is the presence of a dream-like state
during wakefulness, which narcoleptics describe as a hallucination.
b. Most of the other symptoms of sleep disorders are associated--wlil1-sws. These include
sleepwalking (somnambulism), bed-wetting (nocturnal enuresis), and nightmares (pavor
nocturnus), all of which occur during51ages of SWS_
(1) During a nigbtmare that occurs in SWS. sleep, the individual wakes up screaming and
appears terrified. However, no reason for the acute anxiety is recalled.
(2) By contrast, terrifying dreams that occur during REM sleep are graphically remembered.
4. Disturbances of consciousness
a. A lesiollblockin_g~~e connecti~IlJ:~~~~~n the ascending reticular activating system and the
thal~~1J2 producesa'perman'ent state of sleep, or coma. In this situation, stimulation of
sensory pathways can ca!,Js.e. a momentary desyndlronization of the EEG but does not
produce any behavioral signs of.arousal.
"
(1) C6mais not simply a deep sleep state. It is characterized by a loss of consciousness
from which arousal cannot be elicited.
(2) Oxygen conwmption by.the brain is reduced durinB.<;:gr:D.'!, Jhis is in marked contrast
to normal sleep, in which there is no change in brain oxygen con~l,l!!lption from the
--waking state".
b. A transient pathologic loss of consciousness is called syncope (fainting). More persistent
losses of consciousness are called stupor, from which arousal can be obtained.
c. Brain death occurs when the brain no longer can achieve consciousness. Because of the
desire to obtain organs for transplant operations and the desire to remove heroic life
support systems, an objective standard for determining the presence of brain death has been
developed.

(1) Brain death is said to occur when a loss of consciousness is accompanied by a flat EEG
(i.e., an EEG with no brain waves) and a loss of all brain stem regulatory systems (e.g.,
those systems that control respiration and blood pressure).
(2) Moreover, these clinical signs must be due to traumatic or ischemic anoxia and not to
hypothermia or metabolic poisons, from which later recovery is possible.
(3) Finally, the criteria for brain death must be present for 6-12 hours.

STUDY QUESTIONS
Directions: Each of the numbered items or incomplete statements in this section is followed by answers
or by completions of the statement. Select the one lettered answer or completion that is best in each
case.

1. If the flux of glucose across a cell membrane is


directly proportional to the concentration gradient
of glucose, the transport process is
(A) simple diffusion
(B) facilitated diffusion
(C) Na+-coupled cotransport
(D) active transport

2. The diffusion of lipid-soluble particles across a


membrane would increase with an increase in any
of the following factors EXCEPT
(A) the temperature of the solution
(B) the area of the membrane
(C) the solubility of the particles in the membrane
(D) the size of the particles
(E) the concentration gradient of the particles

3. ATP is used directly in which of the following


transport processes?
(A) Transport of glucose into a red blood cell
(B) Transport of Ca2+ out of a cell by Na+-Ca2+
exchange
(C) Transport of K+ out of a cell during an action
potential
(D) Removal of norepinephrine from the synaptic
cleft by the presynaptic nerve terminal
( E) Removal of Ca2+ from the cytoplasm by the
sarcoplasmic reticulum

4. A red blood cell is placed in a saline (NaCl)


solution, and the cell's volume increases to 1.5
times its original volume. This finding indicates that
the Na+ concentration in the saline solution is approximately
(A) 50 mEq/L
(B) 75 mEq/L
(C) 100 mEq/L
(D) 200 mEq/L
(E) 600 mEq/L

5. A decrease in the extracellular K+ concentration will


(A) increase the transport of Na+ out of a cell by
the Na + -K + -ATPase active transport system
(B) increase the flow of Na+ into a cell down its
electrochemical gradient
(C) both
(D) neither

6. Plasma proteins, which have a concentration of


1.2 mmol/L and an oncotic pressure of 24 mm Hg,
normally can prevent the net flow of fluid out of
the capillaries. What concentration of glucose
would be required to prevent the flow of water out
of the capillaries, if the reflection coefficient of
glucose is O.B?
(A)
(B)
(C)
(D)
(E)

0.96 mmol/L
1.20 mmol/L
1.50 mmol/L
19.60 mmol/L
30.00 mmol/L

Questions 7-8
Use the following equilibrium potentials (E K and
EN.) and conductances (G K and G N.) to answer the
next two questions.
EK = -90 mV
EN. = +60 mV

G K = B conductance units
G N = 2 conductance units

7. Based on these values, the resting membrane


potential is approximately
(A) -60 mV
(B) -70 mV
(C) -75 mV
(D) -BO mV
(E) -90 mV

B. If the given G K were decreased by half while


the G N remained the same, what effect would this
have on the membrane potential?

(A) It would depolarize by 20 mV


(B) It would depolarize by 10 mV
(C) It would hyperpolarize by 10 mV

(0) It would hyperpolarize by 20 mV


(E) None

9. A cell at the threshold for firing an action potential has Na+ channels with
(A) most of the m gates open
(8) most of the h gates open
(C) both
(D) neither
10. Which phase of the action potential is caused
by inactivation of Na+ channels in a nerve axon?
(A)
(B)
(e)
(D)
( E)

Upstroke
Absolute refractory period
Downstroke
Undershoot phase
Relative refractory period

11. The maximum firing rate of an axon is determined primarily by the


(A) number of Na+ channels in the axon
(B) rate at which Na+ channel m gates open during the upstroke
(e) rate at which Na+ channel h gates open during
the downstroke
(D) number of K+ channels in the axon
(E) rate at which K+ channel n gates open during
the upstroke
12. What would be the result of removing the inactivation gate from the Na+ channels of a nerve
axon?
(A) Action potentials could not be elicited
(B) The rate of repolarization during the action potential would decrease
(e) Na+ conductance would decrease
(D) The membrane would hyperpolarize
13. The propagation velocity of an action potential
would be increased by
(A) an increase in the rate of depolarization during
the upstroke of the action potential
(B) an increase in the diameter of the axon
(e) both
(D) neither

14. Which statement below correctly describes


the channels located on the end-plate membranes
of skeletal muscle?
(A) The channels are activated by Aeh
(B) The channels are primarily permeable to Na+
(e) When all of the postsynaptic channels are
open, the membrane potential becomes positive
(D) The number of open channels increases when
the membrane depolarizes
15. Which ionic channel would produce the greatest amount of membrane depolarization when it is
opened by a neurotransmitter?
(A) A channel
(B) A channel
(C) A channel
(D) A channel
ea2+

specifically permeable to
equally permeable to K+
specifically permeable to
equally permeable to K+,

ea2+
and Na+
K+
Na+, and

(E) A channel specifically permeable to Na+


16. Which type of receptor is activated by Aeh
and blocked by atropine?
(A) Nicotinic
(B) Muscarinic
(C) Beta
(D) Alpha
(E) Noncholinergic, nonadrenergic
17. ea2+ is released from the sarcoplasmic reticulum by
(A) diacylglycerol
(B) G protein
(C) phospholipase e
(D) inositol triphosphate
( E) adenylate cyclase
18. When activated by ~-adrenergic receptors,
the G protein
(A)
(B)
(e)
(D)
(E)

activates phospholipase e
activates adenylate cyclase
activates protein kinase e
converts GOP to GTP
stimulates the release of ea2+ from the sarcoplasmic reticulum

19. Action potentials are initiated on the axon


hillock of eNS neurons because

23. Voltage-sensitive channels are most important


for the

(A) a large number of excitatory nerve terminals


synapse on the axon hillock
(B) inhibitory nerve terminals are located far from
the axon hillock
(e) both
(0) neither

(A) release of transmitter from nerve terminals


during synaptic transmission
(B) postsynaptic depolarization of the end-plate on
skeletal muscle
(e) release of ea2+ from the sarcoplasmic reticulum of cardiac muscle cells
(0) transmission of information within the retina
( E) response of cutaneous mechanoreceptors to
deformation of the skin

20. Which of the following substances is important for skeletal muscle contraction but not for
smooth muscle contraction?
(A) Actin
(B) Myosin-ATPase
(C) Troponin
(0) Myosin
(E) ea 2 +-ATPase
21. The twitch-to-tetanus ratio (i.e., the ratio of
a force developed during a twitch to the force
developed during a tetanus) is greater in a slowtwitch fiber than in a fast-twitch fiber because the
(A) activity of the sarcoplasmic reticular ea 2 +
pump is less in slow-twitch fibers than in fasttwitch fibers
(B) amount of ea2+ released from the sarcoplasmic reticulum is greater in slow-twitch fibers
than in fast-twitch fibers
(e) amount of ATP used during a contraction is less
in slow-twitch fibers than in fast-twitch fibers
(0) myosin-ATPase activity is greater in slowtwitch fibers than in fast-twitch fibers
(E) density of thick and thin filaments is greater in
slow-twitch fibers than in fast-twitch fibers
22. The bones of the middle ear are primarily responsible for
(A)
(B)
(e)
(0)

amplifying the sound waves reaching the ear


detecting the presence of a sound stimulus
locating the source of a sound
discriminating among different frequencies of
sound
(E) adapting to a prolonged monotonous sound

24. Which of the following is a feature of both the


stretch reflex and the withdrawal reflex?
(A)
(B)
(e)
(0)
(E)

Irradiation
Afterdischarge
Small afferent fibers
Multiple spinal synapses
Reciprocal innervation

25. In comparison to motor units that fire later


during a movement, motor units that fire at the
beginning of a movement
(A) can be tetanized at a lower frequency of stimulation
(B) generate a greater amount of force
(e) have a greater amount of glycogen stored
within them
(0) fatigue more rapidly
(E) are innervated by larger alpha motoneurons
26. In which of the following sensory systems is
the transducer and spike generator located on the
same cell?
(A)
(B)
(e)
(0)

Taste
Vision
Olfaction
Hearing

27. Auditory hair cells and taste receptor cells are


alike in which of the following ways?
(A) Both release neurotransmitter when stimulated
(B) Both can regenerate when damaged
(e) Both respond to a stimulus by opening K+selective channels
(0) Both have very high resting membrane potentials

28. Reciprocal innervation is most accurately described as

30. Which of the following conditions is most


closely related to slow-wave sleep?

(A) inhibition of flexor muscles during an extension


(B) activation of contralateral extensors during a
flexion
(e) reduction of la fiber activity during a contraction
(D) simultaneous stimulation of alpha and gamma
motoneurons
(E) inhibition of alpha motoneurons during a contraction

(A) Dreaming
(B) Atonia
(e) Bed wetting
(D) High-frequency EEG waves
(E) Irregular heart rates

29. True statements about the initial stages of a


normal sleep period include which of the following?
(A) Sleep is initiated by a decrease in the activity
of the ascending reticular activating system
(B) The first EEG sign of sleep is an increase in the
amplitude of the EEG waves
(C) Both
(D) Neither

31. A person with normal vision has a total refractive power (without accommodation) of 60 diopters (D). In order to focus an object placed 25 cm
from the eye, the refractive power of the lens must
increase by approximately
(A) 1 D
(B) 2 D
(e)

4 D

(D) 5 D
(E) 10 D
32. The variation in auditory threshold as a function of frequency (the minimum audibility curve) is
related most to the properties of the
(A) outer ear
(B) auditory canal
(e) middle ear
(D) tympanic membrane
(E) basilar membrane

Directions: Each question below contains four suggested answers of which one or more is correct.
Choose the answer.

A if 1, 2, and 3 are correct


B if 1 and 3 are correct
C if 2 and 4 are correct
D if 4 is correct
E if 1, 2, 3, and 4 are correct

33. The osmotic flow of water across a membrane


will be increased by increasing
(1) the
(2) the
the
(3) the
(4) the

reflection coefficient of the membrane


concentration of particles on one side of
membrane
temperature of the solution
permeability of the membrane

34. A Donnan equilibrium between the plasma


and interstitium is characterized by
(1) greater osmotic pressure in the plasma than in

the interstitium
(2) more negative electrical potential in the plasma
than in the interstitium
(3) greater concentration of diffusible cations in
the plasma than in the interstitium
(4) greater concentration of diffusible anions in the
plasma than in the interstitium

36. An increase in the afterload in an isotonically


contracting muscle would produce which of the
following effects?
(1) A decrease in the rate of cross-bridge cycling
(2) A decrease in the amount of stretch on the
series elastic component
(3) An increase in the force of contraction
(4) An increase in the amount of shortening

37. Compared to cones, rods can detect light at


lower intensities because
(1) rods are larger than cones
(2) rods can increase their sensitivity to light more
quickly than cones
(3) rods have larger receptive fields than cones
(4) rods are more concentrated than cones in the
fovea

35. An action potential elicited during the relative


refractory period will

38. When light strikes a visual receptor cell (a rod


or cone), it causes which of the following reactions?

(1) require a greater stimulus than normal to reach

(1) An increase in the intracellular cGMP concen-

threshold
(2) have a smaller than normal overshoot potential
(3) depolarize more slowly than normal
(4) repolarize more slowly than normal

tration
(2) A decrease in the conductance of the cell
(3) Release of an inhibitory neurotransmitter from
the cell
(4) Hyperpolarization of the cell

39. Cerebellar lesions produce which of the following motor signs?


(1) Adiadochokinesia

40. Movement disorders related to the removal of


inhibition are produced by which of the following
components of the motor control system?

(2) Dysmetria

(1) Striatum

(3) Ataxia
(4) Spontaneous tremor

(3) Substantia nigra

(2) Internal capsule

(4) Medullary pyramids

Directions: Each group of items in this section consists of lettered options followed by a set of numbered
items. For each item, select the one lettered option that is most closely associated with it. Each lettered
option may be selected once, more than once, or not at all.
Questions 41-44

Question 45-49

The diagram below illustrates the anatomic arrangement for the presynaptic inhibition of an excitatory neuron. Match each of the following descriptions with the appropriate lettered area of the
diagram.

Match each of the following descriptions with the


appropriate lettered point on the action potential
shown below.

41. A decrease in the entry of Ca2+


42. The release of GABA by exocytosis
43. A decrease in the size of the excitatory postsynaptic potential
44. An increase in the C/- conductance

45. The point at which K+ conductance is highest


46. The point contained within the absolute refractory period
47. The point at which Na+ conductance is
highest
48. The point at which the net flow across the
membrane is zero
49. The point at which the driving force for K+ is
greatest

ANSWERS AND EXPLANATIONS


1. The answer is A. [/ C 1 a] In simple diffusion, the flux is directly proportional to the concentration gradient. Carrier-mediated transport differs from simple diffusion in that it displays saturation kinetics.
Facilitated diffusion, active transport, and Na + -coupled transport all are examples of carrier-mediated
transport.

2. The answer is D. [/ C 1 a (3), (5)] The diffusion of lipid-soluble materials obeys Fick's law of diffusion,
which states that diffusion is directly proportional to the diffusion coefficient, the area of the membrane,
and the concentration gradient. In general, the diffusion coefficient increases as the solubility of the
material in the membrane increases and as the temperature increases, and it decreases as the size of the
particle increases. Thus, diffusion decreases as the size of the particle increases.
3. The answer is E. [IV A 2 d] Ca2+ is transported into the sarcoplasmic reticulum by the Ca2+-ATPase
(calcium-adenosine triphosphatase) pump. The removal of norepinephrine from the synaptic cleft and
the transport of Ca2+ are Na+-dependent, indirect-energy-utilizing, active transport systems. K+ flows
down its electrochemical gradient during an action potential. Glucose is transported into red blood cells
by facilitated diffusion.
4. The answer is C. [/ C 3 c, d] The steady state volume of a cell placed in a solution containing an
osmotic concentration different from the cell's can be calculated using the equation: 17, V, = 172 V 2 .
The osmolality of normal body fluids is approximately 285 mOsm. In this problem, a red blood cell is
placed in a solution of unknown osmolality and swells to 1.5 times its original volume. Solving the
equation for the osmolality of the extracellular fluid (17 2 ) yields 285/1.5, or 190 mOsm. A saline solution
of this osmolality will have a Na+ concentration of 95 mEq/L.
5. The answer is B. [/ C 2 a; /I 0 1-4] A decrease in extracellular K+ concentration will hyperpolarize
a cell, increasing the driving force for Na+. According to Ohm's law for solutions: INo = G No (Em - ENo),
increasing the driving force (Em - EN.l will increase the flow of Na+ current into the cell. The rate of
pump activity will be lowered if K+ drops below 1 mmol/L; thus, lowering the extracellular K+ concentration will, if anything, lower the activity of the Na+-K+ pump.

6. The answer is C. [I C 3 e, f] According to the osmotic flow equation, the flow of water is proportional
to the osmotic pressure difference across the membrane. Since glucose has a reflection coefficient of
0.8, a concentration difference of 1.5 mmol/L (1.2/0.8) would be required to equal the osmotic pressure
of 1.2 mmol/L of plasma protein, which has a reflection coefficient of 1.
7-8. The answers are: 7-A, 8-A. [/I 0 4] The membrane potential can be calculated using the transference equation, if equilibrium potentials and transferences for the permeable ions are known. In this
problem, the equilibrium potentials (E K and ENo ) are known, and the transferences (TK and TNo) can be
calculated from the conductances (G K and GN.l, since T;on = G;on/Gtotol' Thus, the transference for K+ is
8/10 (0.8), and the transference for Na+ is 2/10 (0.2). Applying the transference equation, the resting
membrane potential is calculated as
Em= EK TK + ENo TN.
= (-90 x 0.8) + (+60 x 0.2)
= -60 mV
If G K were decreased by half while G N remained the same, then TK would become 4/6 (0.67) and TNo
would become 2/6 (0.33). Using these values and applying the transference equation, the membrane
potential is calculated as
Em = EK TK + EN. TNo
= (-90 x 0.67) + (+60 x 0.33)
= -40 mV
Thus, the membrane potential would depolarize by 20 mV (i.e., from -60 mV to -40 mY).

9. The answer is B. [/I C 2 a, 3 c] When a cell is at rest, the m gates are closed and the h gates are
open. At threshold, a few (not most) of the m gates begin to open, allowing some Na+ to enter the cell.
The flow of Na+ into the cell is responsible for the upstroke of the action potential. The majority of m
gates open during the upstroke. The h gates remain open at threshold and during most of the upstroke.

10. The answer is B. [/I C 4 a, b] As long as the Na+ channels are inactivated by the h gates, another
action potential cannot be generated and the axon will remain in the absolute refractory period. The h
gates begin to open during the downstroke, and most of them are open during the relative refractory
period.
11. The answer is C. [/I C 4 a (2)] The maximum firing rate is directly proportional to the duration of
the absolute refractory period, which is related to the rate at which Na+ channels recover from inactivation (i.e., the rate at which the h gates reopen) at the end of the action potential.
12. The answer is B. [/I C 3 c (2)] The closing of the inactivation (h) gate is, in part, responsible for the
repolarization phase of the action potential. If the h gate were removed, the rate of repolarization would
be slowed. Although some of the Na+ channels are inactivated at rest, removing them would have little
or no effect on the Na+ conductance. However, if the Na+ conductance did increase slightly, the
membrane would depolarize, and there might be a negligible increase in K+ conductance. Action
potentials could still be elicited, because the activation gates on the Na+ channels would be intact.
13. The answer is C. [/I C 5] The speed of action potential propagation is determined by the rate at
which contiguous areas of the membrane are depolarized to threshold. An increase in the rate of
depolarization during the upstroke is accompanied by an increase in inward current and a more rapid
depolarization of contiguous areas. Similarly, increasing the diameter of the axon will increase the flow
of current along the axon, which, in turn, will increase the rate of depolarization.
14. The answer is A. [11/ A 3 b] The end-plate channel on skeletal muscle is activated by acetylcholine
(ACh). The channel is equally permeable to Na+ and K+ and is not opened when the membrane
depolarizes. If all of the ACh channels were to open, the membrane would depolarize to -16 mY, which
is the reversal potential for the channel.

15. The answer is A. [11/ A 3 b (4); Table 1-2] The reversal potential for a synaptic channel can be
calculated using the transference equation. When a synaptic channel is opened by a neurotransmitter,
the membrane potential moves toward the reversal potential for that channel. The reversal potentials for
the channels given in the question are as follows: Ca2+ = + 130 mY; Na+ and K+ = -16 mY; K+ =
-92 mY; K+, Na+, and Ca2+ = 33 mY; and Na+ = 60 mY. Since the Ca2+ channel has the most positive
reversal potential, it would produce the greatest amount of depolarization.
16. The answer is B. [1/182 c] Although both muscarinic and nicotinic receptors are activated by ACh,
only muscarinic receptors are blocked by atropine. Nicotinic receptors are blocked by curare. Alpha
and beta receptors are adrenergic receptors; they are activated by norepinephrine and epinephrine, not
by ACh.
17. The answer is D. [11/ 8 2 a 0)] Inositol triphosphate OP 3 ) causes Ca2+ to be released from the
sarcoplasmic reticulum. IP 3 and diacylglycerol are formed from phosphoinositol diphosphate by phospholipase C, which is activated by the G proteins that are activated by muscarinic and a 1-adrenergic
receptors.
18. The answer is B. [11/ 8 3 a] When the G protein is activated by J3-adrenergic receptors, the result
is the activation of adenylate cyclase, which, in turn, converts adenosine triphosphate (ATP) to cyclic
adenosine 3',S'-monophosphate (cAMP). cAMP increases the activity of the sarcoplasmic reticular Ca2+
pump. The G protein has intrinsic guanosine triphosphatase (GTPase) activity and, thus, is able to
hydrolyze guanosine triphosphate (GTP).
19. The answer is D. [11/ C 2 b] Action potentials are initiated on the axon hillock because it has a high
concentration of the electrically excitable Na+ and K+ channels responsible for producing an action
potential. There are very few, if any, excitatory synaptic channels on the axon hillock. Inhibitory synaptic
channels tend to be located on the cell body, close to the axon hillock, where they can most easily
inhibit the generation of an action potential.
20. The answer is C. [IV A 2 b, d, C 2] In skeletal muscle, contraction is initiated when Ca2+ binds to
troponin. Smooth muscle contraction is initiated by the phosphorylation of the myosin light-chain
proteins.

21. The answer is A. [IV A 3 a (4); XII B 1 c (1), (2)] The force developed during a twitch is related to
the duration of the active state (i.e., the time during which Ca2+ concentrations are elevated). If Ca2+
remains in the cytoplasm longer, cross-bridge cycling continues for a longer time, the series elastic
component is stretched further, and more force develops. More force is developed during a tetanic
contraction because the active state is prolonged by repetitive stimulation. However, because the
sarcoplasmic reticulum of slow-twitch fibers removes Ca2+ from the cytoplasm so slowly (i.e., its active
state lasts for a long time), the force produced during a twitch is almost as great as the force produced
during a tetanic contraction.
22. The answer is A. [VII B 2 b] When sound waves pass from air to water, most of the sound energy
is lost. By amplifying the sound, the middle ear bones make it possible for the cochlea to detect sounds,
even though the auditory sensory apparatus is within a fluid medium.

23. The answer is A. [III A 3 a] The release of synaptic transmitter occurs when voltage-sensitive Ca 2 +
channels open in response to the depolarization of the presynaptic nerve terminal by an action potential.
24. The answer is E. [XI Alb, BIb] The motor control system is organized so that neurons causing
excitation of one muscle group will inhibit the alpha motoneurons innervating the antagonist muscle
groups. This type of reciprocal innervation does not prevent agonist and antagonist muscle fibers from
being coactivated. However, in such cases, the reciprocal inhibition must be overcome. Both the stretch
reflex and the withdrawal reflex are characterized by reciprocal innervation.
25. The answer is A. [XII B 2 b (2) (b)] Smaller motoneurons are generally activated before larger
motoneurons. These neurons have slower speeds of contraction and longer active states. Thus, they can
be tetanized at lower frequencies of stimulation.

26. The answer is C. [V B 1-2; X C 1] The receptors for olfaction and the spike generator both are
found on the end of the bipolar olfactory nerve (cranial nerve I) fibers. In the other sensory cells, the
receptors are located on modified epithelial cells, and the spike generator is located on a cranial nerve
fiber. The sensory receptor cell communicates with the cranial nerve fiber by synaptic transmission.
27. The answer is A. [V B 1-2; VII C 3; IX B 3] Both auditory hair cells and taste receptor cells are
modified epithelial cells that communicate with their cranial nerve fibers by synaptic transmission. Hair
cells (but not taste cells) have a very high resting potential because they are surrounded by an extracellular
fluid (endolymph), which contains a high concentration of K+. When hair cells are stimulated, K+ -selective channels are opened, allowing K+ to enter the cell. Taste cells are able to regenerate when damaged
but auditory hair cells cannot.

28. The answer is A. [XI Alb (2)] Reciprocal innervation is most accurately described as inhibition of
the antagonist muscle when the agonist muscle is activated. For example, flexor muscles are inhibited
during an extension. Reciprocal innervation allows extensor contraction to occur without interference
from the flexor muscles that are being stretched during the movement. Under normal circumstances,
stretching the flexors will elicit a stretch reflex leading to contraction of the flexor muscles. Inhibiting the
alpha motoneurons that innervate the flexor muscles prevents the stretch reflex from interfering with the
extension. Reciprocal innervation characterizes all movements, not just extension.
29. The answer is B. [XIII A 2, C " 2] Sleep is initiated by the activity of sleep centers within the
brain stem. Although the ascending reticular activating system (ARAS) is inhibited during this time (and
wakefulness cannot occur if the ARAS is damaged), inhibition of the ARAS is not the cause of sleep
onset. Under normal circumstances, an individual falling asleep enters slow-wave sleep (SWS), which is
characterized by low-frequency, high-amplitude EEG waves.

30. The answer is C. [XIII C 3 b] Bed wetting is a sleep disorder associated with SWS. Rapid eye
movement (REM) sleep is characterized by high-frequency, low-amplitude EEG waves and is associated
with dreaming, atonia, and irregular heart rate and breathing patterns.

31. The answer is C. [VIII Ale (3)] The total refractive power (P) required to focus an object can be
calculated using the lens formula, which defines the relationship between object (0), image (i), and focal
(f) distances as
1
1
1
-+-=-=P
o
i
f
In this problem, the object distance is 0.25 m, the image distance equals the focal length of the relaxed
eye (i.e., 1/60, or 0.0167 m), and the total power of the lens required to focus the object clearly is equal
to 1 If after accommodation for near vision has occurred. Solving the equation for P gives

1
1
1
---+--=-=P
0.0167
0.25
f
Thus, the lens must increase its refractive power by about 4 D (i.e., from 60 D to 63.8 D).
32. The answer is C. [VII B 2 c] The middle ear transfers sounds from the tympanic membrane to the
cochlea. However, it does not transfer all sounds equally well. For example, sounds in the frequency
range used for speech are transferred more efficiently than those at the low and high ends of the hearing
range.
33. The answer is A (1, 2, 3). [/ C 3 e, f] The osmotic flow of water equals 0" x L x A X (7T 1 - 7T 2).
The osmotic pressure (7T) is dependent on the number of particles in solution and the temperature of the
solution, so that increasing either of these increases the osmotic pressure gradient and, thus, the osmotic
flow of water. Increasing the reflection coefficient (0") also increases the osmotic flow of water. However,
increasing the permeability of the membrane to the particles in solution, decreases 0" and, thus, decreases
the flow of water.
34. The answer is A (1, 2, 3). [/ 0] A Donnan equilibrium is established between the plasma and
interstitium because the plasma contains negatively charged, nondiffusible proteins. When equilibrium
is established, the plasma has a higher osmolarity and is negatively charged compared to the interstitium.
The negative charge increases the concentration of cations and decreases the concentration of anions
in the plasma relative to the interstitium.
35. The answer is E (all). [lie 4 b] During the relative refractory period, K+ conductance is higher than
normal so a greater stimulus is required to depolarize the nerve membrane to threshold. Also, because
of the higher than normal K+ conductance, the overshoot potential and rate of depolarization during
the upstroke are smaller. Because the overshoot potential is smaller, less n gates open, and, consequently,
repolarization occurs more slowly.
36. The answer is B (1, 3). [IV A 3 b]ln an isotonically contracting muscle, the force of contraction is
equal to the afterload. Thus, increasing the afterload increases the force of contraction. Also, when the
load on the muscle increases, the rate of cross-bridge cycling decreases. The stretch of the series elastic
component is proportional to the force developed by the muscle, so increasing the afterload increases
the stretch of the series elastic component.
37. The answer is B (1, 3). [VIII B 1 b (4) (c)] The higher sensitivity of rods in low light occurs because
rods absorb more light than cones (because rods are larger than cones) and because their receptive fields
are larger (more rods than cones converge on each ganglion cell). Although rods can increase their
sensitivity to a much greater extent than cones, they adapt more slowly when placed in the dark. There
are no rods in the fovea.
38. The answer is C (2, 4). [VIII B 3 a] When light strikes a photoreceptor cell, it initiates a series of
reactions that cause the conductance of the cell to decrease and the membrane potential to hyperpolarize.
Hyperpolarization reduces the amount of transmitter released by the cell. The chemical reactions initiated
by the absorption of light results in the conversion of cyclic guanosine monophosphate (cGMP) to GMP.
The decrease in intracellular cGMP concentration is responsible for the closing of Na+ channels and the
decrease in membrane conductance.
39. The answer is A (1, 2, 3). [XII E 2] Cerebellar lesions cause adiadochokinesia (the inability to make
rapidly alternating movements), dysmetria (the inability to stop a movement at the appropriate time or
to direct it in the appropriate direction), ataxia (failure of muscular coordination), and intention tremor
(tremor that occurs when the limb is moved). Spontaneous tremor (tremor that occurs when the patient
is at rest) is not observed.

40. The answer is A (1, 2, 3). [XII 02, F 2] The striatum and substantia nigra are part of the basal
ganglia. Lesions to these nuclei produce spontaneous movements due to the removal of their normal
inhibitory effects. Lesions in the internal capsule damage fibers from the motor cortex that normally
inhibit postural tone. Removal of this inhibitory effect results in spasticity. The medullary pyramids
contain axons that originate from neurons in the motor cortex and terminate in spinal motoneuron pools.
These neurons are normally excitatory, and, thus, when they are damaged, motor tone is reduced.
41-44. The answers are: 41-D, 42-A, 43-E, 44-C. [/II C 3; Figure 1-21] In presynaptic inhibition, an
inhibitory neuron forms an axo-axonic synapse with the neuron that is excitatory to the alpha motoneuron. The inhibitory neuron releases 'Y-aminobutyric acid [CABA] (A), which opens CI- channels on the
postsynaptic membrane (C). The action potential invading the nerve terminal is reduced in size. As a
result, less Ca2+ enters the nerve terminal (0), less transmitter is released, and the magnitude of the
excitatory postsynaptic potential on the motoneuron is reduced (E). (B) is the portion of the axon that is
not affected by presynaptic inhibition.

45-49. The answers are: 45-D, 46-C, 47-C, 48-A, 49-C. [II C 1-4; Figure 1-7] When the membrane
potential is maintained at a constant value (e.g., the resting state, point A), the net current across the
membrane is zero. The action potential begins when the membrane is depolarized from its resting
potential to its threshold potential (point B) by a stimulus.
The upstroke of the action potential is caused by the inward flow of Na+ following a large increase
in Na+ conductance. Na+ is highest at point C. The driving force for K+ (i.e., Em - EK) also is greatest at
point C because Em is furthest from EK at this point.
Point C also is contained within the absolute refractory period, which begins when threshold is reached
and lasts until enough Na+ channels have recovered from their inactivation state to allow another action
potential to be elicited.
Points 0 and E are in the relative refractory period. K+ conductance is highest at point O. The
downstroke of the action potential is caused by the outward flow of K+ following an increase in K+
conductance. Point E has a lower K+ conductance than point 0 because the n gates close as the
membrane repolarizes.

2
Cardiovascular Physiology
Joseph Boyle, III

I. VESSELS. The major functions of the cardiovascular system are to distribute metabolites and
oxygen (0 2 ) to all body cells and to collect waste products and carbon dioxide (C0 2 ) for excretion.
The cardiovascular system also is involved in thermoregulation, since heat is carried by the blood
from active metabolic sites, where it is generated to the body surface, where it is dissipated. Blood
flow to the skin and extremities is varied to enhance or retard heat loss to the environment. The
circulation also functions to distribute hormones to distant sites. The heart provides the driving
force for this system; the arteries serve as distribution channels to the organs; the veins serve as
blood reservoirs and collect the blood to return it to the heart; and the microcirculation and
capillaries serve as the exchange vessels.

A. Elastic arteries
1. Structure. The aorta and its large branches (carotid, iliac, and axillary arteries) are composed
of a thick medial layer containing large amounts of elastin and some smooth muscle cells.
The structure of these vessels makes them distensible so that they take up the volume of blood
ejected from the heart with only moderate increases in pressure.

2. Function
a. Compliance (distensibility). The elastic arteries serve as a reservoir; that is, they accommodate the stroke volume of the heart because they are distensible (see Ch 3 II D and E for
further discussion of compliance and elastance). Figure 2-1 shows the pressure-volume
relationships for the aorta in different age-groups.
(1) Note that these curves are relatively linear in the middle, which is the normal working
range. Note also that the distensibility is reduced with age (the vessels become stiffer).
(2) This means that for any volume increase there is an increase in the pressure fluctuation
as on!7ai,~. Clinically, this results in an increased pulse pressure in the older individual.
b. Elastic t;:c~il. In addition to the vessels being distensible, the elastic recoil during diastole
provides potential energy (pressure) to maintain blood flow during the diastolic phase of the
cardiac cycle.
B. Muscular arteries comprise most of the named arteries in the body.
1. Structure. As the distance from the heart increases, the amount of smooth muscle in the medial
layer in these vessels increases.
300

75 years:

40 years:

"
"

/'

~/

" 30 years

/',,"

,.."

,,/ ",'" 20 years

,,'
,,/

",

"

, " ., ,"',' /
... ," "-

,,;

Figure 2-1. Average pressure-volume relationships of arterial systems from 20-, 30-, 40-, and 75-year-old individ-

uals. Note the increasing elastance (decreased compli-

1.0
!:J.VNo

ance) that occurs with advancing age. !:J. V = volume


change; Vo = unstressed volume.

2. Function. These vessels serve as the distributing channels to the body organs and have a
relatively large lumen-to-wall-thickness ratio. The large lumen minimizes the pressure drop due
to resistance losses (Figure 2-2l.

3. Cross-sectional area and velocity of flow. Figure 2-2 shows that the total cross-sectional

;~tJ

area of the arterial tree increases markedly as it proceeds toward the periphery. Since the same
volume of blood traverses each class of vessels per unit time, the velocity of flow is an inverse
function of the cross-sectional area.

l (f it
~)

C. Arterioles, the stopcocks or valves of the circulation, control the volume of blood flowing to the
various vessel beds. The site of the major pressure drop in the cardiovascular system occurs in
the arterioles (see Figure 2-2).

1. Structure. The arterioles contain a thick layer of smooth muscle within the medial layer and
have a relatively narrow lumen. The smooth muscle allows these vessels to alter the lumen
diameter so that resistance and flow are readily controlled.

2. Function
a. Control of blood flow. The arterioles can markedly alter their diameter by contraction or
relaxation of the smooth muscle in their walls. The variation in the lumen diameter allows
a fine control over the distribution of the cardiac output to different organs and tissues. The
smooth muscle tone varies depending on the activity of the sympathetic nerves, the arterial
pressure, the local concentration of metabolites (caused by changes in metabolic rate),
various hormones, and many other mediators such as prostaglandins, thromboxanes, and
histamine. Increases in metabolic rate, such as occur in skeletal muscle during exercise,
cause vasodilation, which increases blood flow. This phenomenon is termed functional

hyperemia.
(1) Autoregulation is the mechanism whereby many organs and tissues-both of which
have a constant metabolic rate--adjust their vascular resistance to maintain a constant
blood flow in the presence of changes in arterial pressure. This phenomenon is depicted

I-pressure

Area

---- Velocity

Arteries
Vena

Left ventricle
and Aorta
100

Right ventricle and


Pulmonary artery

5000
60

80

--,

4000

50

Ci

::c

40

E 60

3000~

<D

III
III

<D

40

2000

30

co

:;

<

Z-

u
0

a..

0<D
.!!!.
E

20

Qi

>

~I

20

1000
I
I

---- ...

,.'

20

'I
,, .\

10

40
60
Cumulative blood volume (%)

80

100

Figure 2-2. Relationships between velocity, area, volume, and pressure in various segments of the cardiovascular
system. The same volume of blood must pass through each segment of the system per unit time; therefore, the velocity
and area are inversely related. The resistance of the large vessels is minimal, so pressure loss also is minimal until the
small arterioles are reached.

schematically in Figure 2-3, and several theories have been proposed to explain its
mechanism.
(a) Metabolic theory. An increase in blood pressure would initially increase blood
flow to a tissue or organ. This increased blood flow would wash out any vasodilator
substances that were present, vascular resistance would increase, and blood flow
would return to the normal levels. Increased CO 2 , H+, adenosine, prostaglandins,
K+, or PO/- or hypoxia can cause vasodilation. Autoregulation cannot be explained
in all instances by changes in the concentration of any of these substances.
(b) Myogenic theory. Vascular smooth muscle constricts when the pressure inside the
vessel is increased. Vascular smooth muscle responds to wall tension, which is a
function of pressure and wall radius according to the law of Laplace

Px r

where T = wall tension, P = pressure, and r = radius. Thus, an increase in arterial


pressure raises wall tension; blood flow increases because of the higher driving
pressure. Contraction of vascular smooth muscle returns wall tension to control
levels by reducing the vascular radius. The narrowed lumen compensates for the
.
increased arterial pressure so that blood flow returns to control levels.
(2) Reactive hyperemia is a phenomenon that occurs after occlusion of the arteries to an
organ or tissue. When the occlusion is released, the blood flow exceeds the control
level with a magnitude and duration that depends on the duration of the occlusion.
Reactive hyperemia seems to be caused by a metabolic mechanism that controls blood
flow to these tissues.
b. Dampening of pulsations. Arterioles convert the pulsatile flow in the arteries to a steady
flow in the capillaries.
D. The microcirculation consists of those vessels smaller than 100 IL in diameter; it includes metarterioles, arterioles, capillaries, and postcapillary venules. Exchange of substances between
extracellular fluid and the vascular system occurs primarily in the capillaries and postcapillary
venules. In some tissues, especially the skin, there are short, low-resistance connections between
the arterioles and the veins, which are termed arteriovenous (A-V) shunts. In the skin, the A-V
shunts are involved in thermoregulation.

1. Organization. The microcirculation is a meshwork of fine vessels that distribute blood to every
cell in the body.
a. Capillaries arise directly from arterioles or metarterioles. The metarterioles are relatively
high-resistance conduits between arterioles and veins. A cuff of smooth muscle cells surrounds the origin of many capillaries and is termed the precapillary sphincter.
b. Vasomotion. Not all of the capillaries in a tissue are functional at one time, so blood flow
alternates among different capillaries from moment to moment. Under basal conditions,
1%-1 0% of the capillaries function at any moment. During high metabolic activities, many
more capillaries carry blood simultaneously, which enhances the delivery of 02 and metabolites to the tissues. The opening and closing of these vessels and the resultant changes in
blood flow are termed vasomotion.

Autoregulation_

60
120
Perfusion pressure (mm Hg)

180

Figure 2-3. Autoregulation of blood


flow. Note that blood flow remains
relatively constant over a wide range
of pressures. This can only be accomplished by changing the resistance to
flow through the particular vascular
bed in proportion to the change in
pressure.

2. Function. The microcirculation is the area where exchange occurs between the blood and the
interstitial fluids. The interstitium, in turn, equilibrates with the body cells. Many capillaries arise
from each arteriole and metarteriole, which provides a total cross-sectional area of 0.4-0.5 m 2
for the microcirculation. This large area results in an average blood flow velocity of 0.3-0.4
mm/sec in the capillaries, but the velocity varies widely, from 0 to 1.0 mm/sec within short
periods of time within the same capillary due to vasomotion.
3. Endothelial structure varies in different organs depending on function.
a. Fenestrations
(1) At one extreme, in the renal glomeruli, gastrointestinal tract, and some glands, large
(200-1000 A in diameter) transcellular fenestrations are present through the endothelial
cells. These fenestrations are coupled with an incomplete basement membrane. A thin
membrane mayor may not cover the fenestrations.
(2) At the other extreme, in the brain, the microcirculatory endothelium has no fenestrations, and a complete basement membrane is present. These structures are morphologic
evidence of the blood-brain barrier, which retards or prevents the transfer of many
substances between the blood and the brain.
b. Gap junctions. Large gaps occur between the endothelial cells in the bone marrow, liver,
and spleen. The postcapillary venules (20-60 j.L in diameter) represent the most permeable
portion of the microcirculation. This region represents leakage sites for large molecules after
the administration of certain substances (e.g., histamine, bradykinin).
c. Tight junctions. In most body tissues the clefts between endothelial cells appear fused, but
anatomic and physiologic evidence indicates that pores (about 40 A n diameter) exist
between endothelial capillary cells. These pores are absent in cerebral capillaries, which is
further evidence of a blood-brain barrier.
4. Mechanisms of exchange. The microcirculation provides a total surface area of about 700 m 2
for exchange between the circulatory system and the interstitial compartment.
a. Diffusion is the principal mechanism of microvascular exchange. Passive diffusion results
from the random movement of molecules from an area of high concentration to an area of
low concentration and does not involve any carrier molecules or demonstrate saturation
kinetics properties.
(1) The rate of diffusion depends on the solubility of the substance in the tissues, the
temperature, and the surface area available, and it is inversely related to molecular
size and the distance over which diffusion must occur.
(a) O 2 , CO 2 , water, and glucose rapidly equilibrate across the microvascular endothelium.
(b) Larger molecules (e.g., albumin) cross the endothelial barrier very slowly or are
impeded completely because the average pore size is smaller than albumin.
(2) Diffusion path. The route of transport of most substances is still controversial. It is
unclear whether transport occurs through gap junctions, fenestrae, intracellular vesicles
(cytopempsis), or across the cells themselves.
b. Bulk flow. A hydrostatic pressure difference across the endothelium results in the filtration
of water and solutes from the capillaries into the tissues. The presence of large molecules
(proteins, especially albumin) in the blood exerts a force (oncotic pressure) that counteracts
filtration. The factors determining the balance between filtration and reabsorption were
originally formulated by Ernest Starling, and the concept is termed the Starling hypothesis
(Figure 2-4). The balance between the filtration and reabsorption of water depends on the
difference in hydrostatic and oncotic pressures between the blood and the tissues and on
how leaky the vessels are. The bulk flow of water (Qw) is expressed as
Qw = K [(Pc - Pi) - (Pc - Pi)]

where K = the filtration coefficient, which varies with the permeability of the vessels and
the available exchange surface area; Pc = hydrostatic capillary pressure; Pi = hydrostatic
pressure in the interstitium; Pc = oncotic pressure of the blood; and Pi = interstitial oncotic
pressure.
(1) Factors intrinsic to the blood vessel. Increased blood pressure within the microvessels
(Pc) causes more fluid to be filtered from the capillaries; the accumulation of excess
fluid in the tissues is termed edema. A loss of plasma proteins (decreased Pc), as in
starvation, also can cause edema formation due to decreased oncotic pressure, which
normally retards filtration. Injury to vessels or infection causes the vessels to become
leakier (an increase in the filtration coefficient K), which leads to the accumulation of

A. Normal: VF = 0.10 mll100g/min

B. Arteriolar dilation: VF = 0.22 m1/100g/min


L:r='

Capillary pressure

Capillary pressure
0)

::I:

E
E

Equilibrium point

Capillary length

Capillary length

C. Increased venous pressure:


VF = 0.14 mll100g/min
60

D. Decreased oncotic pressure:


VF = 0.25 m1/100g/min

Capillary pressure

Capillary pressure

0)

::I:

E
E

Equilibrium point

Capillary length

Filtration forces

Capillary length

Reabsorption forces

Figure 2-4. Schematic drawing of the intravascular pressures involved in determining the filtration of fluid through
the microcirculation. Equilibrium point represents the pressure needed to counteract the oncotic pressures in the
blood and interstitial fluid. Flow is in l/min. The effects of various interventions are shown in panels A through D.
VF = volume filtered.

tissue fluids. Conversely, decreased pressure within the capillaries, as in shock, tends
to draw fluid out of the tissues into the vascular system.
(a) The filtration coefficient (K) varies primarily due to changes in the number of
microvessels having blood flow and also due to changes in vascular permeability.
An increased permeability occurs when the vessels are injured by hypoxia and
inflammatory products such as histamine. Dilation of arterioles, and especially precapillary sphincters, increases the number of functional capillaries (recruitment),
since dilation transmits a greater fraction of the arterial pressure into the capillary.
The greater capillary pressure dilates vessels so that perfusion increases. Hypercapnia, hypoxia, increased H+ and K+ concentrations, histamine, bradykinin, adenosine, and many other substances all can result in vasodilation and an increase in the
number of perfused capillaries.
(b) Capillary hydrostatic pressure (Pc) represents a force tending to push fluid out
into the interstitium. Pc depends especially on the venous pressure but also on the
arterial blood pressure, the arteriolar resistance, and the state of the precapillary
sphincter.
(i) Capillary pressure varies widely but ranges between 30 and 45 mm Hg in most
tissues. Whenever blood flow is present, there must be a pressure gradient so
that the capillary pressure must always exceed the venous pressure.
(ii) A low arteriolar resistance allows a greater proportion of the arterial pressure to
be transmitted to the capillaries. For example, in the kidneys the glomerular

capillary pressure is about 50-60 mm Hg, which provides a large force to


produce glomerular filtration.
(iii) Contraction of the precapillary sphincter can occlude the vascular lumen so that
Pc equilibrates with the local venous pressure; under these conditions fluid may
be reabsorbed along the entire length of the capillary.
(c) The oncotic pressure of blood (Pc) is due to the osmotic pressure of blood proteins,
which is only a small fraction (0.5%) of the total osmotic pressure of blood. The
normal Pc is 25-27 mm Hg, which provides a force tending to reabsorb fluid into
the capillaries.
(2) Factors extrinsic to the blood vessel
(a) The interstitial pressure (Pi) magnitude is controversial since different experimental
techniques have shown it to be either slightly positive or slightly negative. Pi becomes greater if the interstitial fluid volume increases (edema), which hinders further
fluid filtration from the capillary.
(b) Oncotic pressure of the interstitium (Pi). Significant amounts of plasma proteins
cross the endothelial barrier through either intercellular pores or fenestrae, depending on the tissue. Protein in the interstitial space produces a force directed
toward outward filtration of fluid from the vascular system. The effective Pi in the
interstitium is estimated as 5-10 mm Hg in most tissues.
c. Cytopempsis. Some substances are transported across the endothelium by incorporation
into cytoplasmic vesicles, which are then discharged into the interstitium. Transport of
large, lipid-insoluble molecules has been proposed by this route, but the transport of water
and small solutes would be negligible by this mechanism.
E. Lymphatic vessels start as closed endothelial tubes that are permeable to fluid and highmolecular-weight compounds.
1. Function. The major circulatory function of the lymphatic system is to remove albumin and
other macromolecules that have escaped from the microcirculation. In addition, fluid is removed from the interstitium to maintain a gel state.
2. Mechanism. Lymphatic fluid is pumped out of the tissues by the contraction of the large
lymph vessels and contiguous skeletal muscles. The lymph vessels have an extensive system of
one-way valves to maintain the flow of lymph toward the heart.
3. The rate of lymph flow varies in different organs and depends on tissue activity and capillary
permeability. Normal lymph flow is about 2 L/day for the entire body, and this volume contains
approximately 200 g of protein lost from the microcirculation. Most lymph comes from the
gastrointestinal tract and the liver.
4. Importance. The lymphatic system represents the only mechanism for returning interstitial
proteins back to the circulatory system. Due to the concentration of albumin in the blood,
back-diffusion of albumin into the capillaries is impossible. The lymph collected from the
peripheral tissues is returned to the cardiovascular system via the thoracic duct, which empties
into the left subclavian vein near its junction with the left internal jugular vein.
F. Veins
1. Structure. The veins are relatively thin-walled structures whose walls contain elastic tissue,
collagen, and smooth muscle. The venous system is a low-resistance, low-pressure, very
distensible part of the vascular system. The veins provide a larger cross-sectional area than the
arteries at equivalent distances from the heart; therefore, the resistance to flow and the velocity
of flow are less in the veins than in the arteries.
2. Function
a. Conduits. The systemic veins carry deoxygenated blood from the tissues to the right atrium;
the pulmonary veins collect the oxygenated blood from the lungs and return it to the left
atrium.
b. Reservoir. The veins serve as a fluid reservoir; 65%-75% of the circulating blood volume
is in the veins at anyone time. The veins normally are not fully distended, having an oval
cross-sectional shape; thus, small changes in pressure can cause large changes in volume
as they assume a rounded configuration. At normal pressures the veins are about 20 times
more distensible (compliant) than the arteries.

c. Venoconstriction is extremely important for increasing cardiac output. The smooth muscle
of veins responds to sympathetic stimulation by constriction, which displaces blood toward the heart and raises the ventricular filling pressure. This results in an increase in cardiac
output by increasing ventricular end-diastolic volume and ventricular stroke volume.
d. Effects of gravity. Due to the high distensibility of the veins, small pressure changes can
produce large changes in the volume of blood in the venous system.
(1) Peripheral pooling of blood results whenever a person changes from the supine to the
erect position. Standing causes a shift of about 500 ml of blood from the pulmonary
circulation to the dependent veins of the legs, which is due to the hydrostatic pressure
generated by the erect position causing a dilation of the leg veins and a peripheral
pooling of blood. This peripheral pooling decreases venous return and consequently
lowers cardiac output, which tends to decrease arterial blood pressure. These changes
normally are compensated for by a reflex increase of sympathetic tone to the veins and
the heart.
(2) Hydrostatic effects. The arterial pressure in the head is reduced by about 30 mm Hg
compared to the pressure at heart level due to the gravitational effects on the column
of blood from the heart to the brain. These changes are compensated for by a reflex
increase in the smooth muscle tone in the veins as well as increases in heart rate and
peripheral vascular resistance, which maintain an adequate arterial pressure.
e. Venous valves
(1) The veins of the dependent parts of the body are equipped with a system of valves that
prevent the backflow of venous blood. Venous valves also support the column of blood
to minimize the increase in capillary pressure in the dependent parts of the body.
(2) Insufficiency (backward leaking) of venous valves allows the full hydrostatic pressure
to be exerted by the column of blood on the capillaries, leading to the formation of
edema in dependent parts. This loss of fluid from the vascular compartment reduces
venous return and tends to lower arterial pressure. This is the mechanism that causes
soldiers who stand at attention for long periods of time to faint due to decreased cardiac
output. Varicose veins represent a marked dilation and consequent valvular insufficiency
of the dependent veins due to a familial or occupational predisposition.
f. Skeletal muscle pump. Contraction of skeletal muscle aids in venous return by compressing
the veins between the contracting skeletal muscle. This effect is aided by competent venous
valves that prevent backflow during relaxation of the skeletal muscle.
g. Respiration. The normal negative intrathoracic pressure tends to enhance venous return by
increasing the pressure gradient between the heart and the peripheral vessels. Inspiration
lowers the intrathoracic (interpleural) pressure, which increases venous return by enhancing
the pressure gradient even further.
h. Positive end expiratory pressure (PEEP), or a Val salva maneuver (forced expiration
against closed airways, commonly performed when lifting heavy objects or when moving
one's bowels), impedes venous return and produces a complex set of direct and reflex
changes in the cardiovascular system. PEEP is used to treat respiratory distress syndrome
[see Ch 3 " E 2 c (3) (a)l; the positive alveolar pressure stabilizes alveoli and minimizes or
eliminates atelectasis, which enhances gas exchange.
II.

HEART. The heart has an intrinsic contraction rate because it contains its own pacemaker. The rate of
the pacemaker is influenced by the noradrenergic neurons, which increase rate, and the cholinergic
neurons, which decrease rate. The contractile phase of the heart is termed systole, and the relaxation
or filling phase of the heart is called diastole.
A. Myocardial syncytium. The heart muscle (myocardium) is composed of separate cardiac muscle
cells that are electrically connected to one another by tight junctions. These connections are
low-resistance pathways called intercalated disks.
1. Intercalated disks occur at the Z lines of the sarcomere (Figure 2-5). Depolarization of one
cardiac cell is transmitted to adjacent cells; thus, the myocardium is a functional syncytium
(Le., a mass of cytoplasm with numerous nuclei). Actually, the heart functions as two syncytia,
one being the atria and the other the ventricles.
2. The atrioventricular (A V) valve ring separates these two masses of muscle and insulates the
electrical events of the atria from those of the ventricles. Normally, there is only one functional
electrical connection between the atria and the ventricles; this is the A V node and, its extension,
the bundle of His.

Ttubule
Sarcoplasmic reticulum
Plasma membrane

Acti

H~

))

HE
HE

U(

))

)H

HE

>H

>n
I band

A band

Figure 2-5. Schematic drawing of cardiac muscle showing sarcomeres with and without sarcoplasmic reticulum.
The overlap of the actin and myosin varies as a function of the initial length and contraction of the muscle.
B. Electrical activity
1. Resting membrane potential (RMP). The myocardial cells maintain a voltage difference of
60-90 mV across their cell membranes; the inside of the cell is electrically negative compared
to the outside. The RMP is generated because of differences in membrane permeability for
different ions and a difference in ionic concentrations between the inside and the outside of
the cell. The RMP can be predicted by means of the transference equation (see Ch 1 II D 4).
a. The resting cell can be considered to be a K+ battery, since the K+ concentration is much
higher on the inside than the outside, and the cell membrane is very permeable to K+. At
the same time, the resting membrane is rather impermeable to Na+ [see Ch 1 II D 5 a (1 )).
b. The different ionic concentrations are maintained by membrane-bound ionic pumps. These
ionic pumps obtain energy for their function by breaking down ATP to ADP. This energy is
largely derived from the metabolism of foodstuffs within the mitochondria of the cells.
2. Threshold potential. Excitable cells, such as muscle or nerve cells, undergo rapid depolarization if the membrane potential is reduced to a critical level, which is called the threshold
potential. Once the threshold potential is reached, the remainder of the depolarization is
spontaneous. This property is termed the all-or-none law.

1. Cardiac action potential (AP) refers to the change in membrane potential that occurs after
the cell receives an adequate stimulus. The myocardial cells are subdivided into slow and fast
fibers depending on the shape and the velocity of conduction of the AP (Figure 2-6).
a. Slow fibers are normally present only in the sinoatrial (SA) and AV nodes. The membrane
potential of slow fibers usually is in the range of 50 to 70 mV. Fast fibers can be converted
into slow fibers by the effects of ischemia or hypoxia (decrease in O 2 levels) or by certain
drugs (e.g., tetrodotoxin, which blocks Na + channels). The AP in slow fibers is due to the
inward movement of positive ions (Ca 2 +), and this movement neutralizes the normally
negative intracellular charge. During the AP the membrane potential approaches zero, and
the cell is said to be depolarized.
(1) Slow channels. Slow fibers apparently possess only "slow channels," through which
ions enter the cells. These slow channels limit the rate at which ions enter the cells and
produce a slow rate of cell depolarization. Thus, the upstroke of the AP requires about
100 msec in slow fibers compared to 1 msec in fast fibers.
(2) The conduction velocity of the AP along the cells is directly related to the RMP and
the rate of depolarization. Because the RMP is low, and the rate of depolarization is
slow, the AP of slow fibers has a conduction velocity of 0.02-0.1 m/sec.

o
>

50

100 ' - - - - - - - - - - - - - - -

Time (msec)

Figure 2-6. Schematic plot of the action potential from


a slow and a fast fiber, with the different phases (0 through
4) of the action potential represented. Note the diastolic
depolarization and the slow phase 0 in the slow fiber.

(3) The absolute refractory period (ARP) lasts throughout the duration of the AP. The
ARP is an interval in which no stimulus, no matter how strong, can cause another AP.
(4) The relative refractory period (RRP) may last for several seconds in slow fibers. A
stimulus applied during the RRP must be stronger than normal to elicit an AP. This AP
has an even slower conduction velocity and lower amplitude than normal. This effect
frequently blocks the conduction of impulses within the AV node so that all of the atrial
depolarizations do not reach the ventricles; this is referred to as an A V nodal block
(see IV C 2).
b. Fast fibers. The normal atrial and ventricular myocardial cells as well as the specialized
conducting tissues in the heart are fast fibers. The RMP in fast fibers is 80-90 mY; the inside
of the cell is negatively charged with respect to the outside of the cell.
(1) Na+ channels. The AP of fast fibers exhibits an extremely rapid rate of rise, because
once the threshold potential is reached, the cell membrane becomes extremely permeable to Na+.
(a) Since the Na+ concentration is much higher outside the cell and because of the
negative charge inside the cell, there is a very large electrochemical gradient for
Na +. Therefore, Na + rushes into the cell through channels that open once the
threshold potential is reached. The cell essentially becomes a Na+ battery during
the AP.
(b) The duration of the AP varies in different fast fibers and is longest in the Purkinje
and bundle of His fibers. This property provides the heart some protection against
certain arrhythmias (e.g., ventricular tachycardia).
(2) Conduction velocities vary from 0.3-1.0 m/sec in myocardial cells to about 4 m/sec
in Purkinje fibers. The fast conduction velocity insures that the entire myocardium is
depolarized almost instantaneously, which improves the effectiveness of myocardial
contraction.
(3) The ARP in fast fibers lasts until the repolarization has reached a membrane potential
of 50-60 mY.
(4) The RRP ends when the RMP of 80-90 mV is reestablished.
c. Phases of the AP. The AP can be divided arbitrarily into five phases (see Figure 2-6).
(1) Phase O. The resting membrane is relatively impermeable to Na+, but once the threshold
potential is reached the Na+ channels open and allow Na+ to rush into the cell along
the Na+ electrochemical gradient. This process is self-regenerative in that inward movement of Na+ reduces the membrane potential, which opens more Na+ channels, so
that more Na+ enters the cell. The movement of Na+ is a current-carrying process and
results in the depolarization of the cell membrane, which is termed phase O. This process
is extremely rapid in fast fibers. In slow fibers phase 0 coincides with an increase in the
Ca2+ conductance so that Ca2+ is the current-carrying ion in these fibers.
(2) Phase 1. During phase 1 the Na + conductance is reduced rapidly due to a closing of
the Na+ channels, while the membrane conductance for Ca2+ increases. The overall
effect is a small change in the membrane potential toward repolarization, which prevents
the membrane potential from reaching the Na+ equilibrium potential of +40 to +60
mY.
(3) Phase 2. The plateau of the AP in fast fibers coincides with an increased membrane
conductance for Ca2+. The inward movement of Ca2+ and the decreased efflux of K+

maintain the membrane potential near zero during this phase of the AP. In cardiac
muscle, the influx of Ca 2 + is important in the excitation-contraction (EC) process.
(4) Phase 3 is a rapid repolarization due to a reduction in the inward Na+ and Ca2+ currents
and a large increase in the outward K+ current.
(5) Phase 4
(a) Nonpacemaker cells (atrial and ventricular myocardium) exhibit a constant membrane potential during phase 4.
(b) In pacemaker tissues, such as SA and AV nodes and Purkinje fibers, there is a slow
diastolic depolarization during phase 4, which indicates the presence of automaticity. The diastolic depolarization, also termed a pacemaker potential or the slope
of phase 4, brings the membrane potential toward threshold.
(i) The cell whose membrane potential first reaches threshold is the pacemaker of
the heart, and the depolarization then spreads to the remainder of the heart. The
normal pacemaker of the heart is the SA node since its cells usually have the
highest automaticity (steepest slope of phase 4).
(ii) Elimination of SA node activity or an increased slope of phase 4 in other areas
of the heart causes a pacemaker outside the SA node to initiate the depolarization
process. This new pacemaker is termed an ectopic pacemaker since it is outside
the normal place.

C. Conduction pathways
1. The SA node is located near the junction of the superior vena cava and the right atrium (Figure
2-7). The SA node normally is the pacemaker of the heart since it has the highest automaticity.
a. Intrinsic heart rate. The isolated SA node has a firing rate of 90-120 beats/ min; this rate
is higher in young individuals and declines with advancing age. The firing rate is increased
by a rise in temperature, thyroid hormone, epinephrine, or norepinephrine.
b. Effect of vagus nerve. The right vagus nerve densely innervates the SA node and liberates
acetylcholine (ACh) from its nerve endings when stimulated. Normally, vagal activity
(vagal tone) hyperpolarizes SA node fibers by increasing K+ conductance. The hyperpolarization slows the firing rate of the SA node from its automatic rate of 90-120 beats/ min to the
actual heart rate of about 70 beats/min. Strong vagal stimulation can eliminate completely SA

Superior vena cava

Bachman's bundle

I nternodal tracts
anterior
middle
posterior

Left bundle branch


Right bundle branch
Purkinje fibers
Inferior vena cava

Figure 2-7. Schematic drawing of the specialized conducting tissues of the heart. SA = sinoatrial; AV = atrioventricular.

node impulses (sinus arrest), which causes asystole (cardiac standstill) until an ectopic
pacemaker begins to function.
c. Effect of sympathetic nerves. Stimulation of cardiac sympathetic nerves or the injection
of sympathetic-like drugs (e.g., epinephrine) increases heart rate and the automaticity of
ectopic sites. Stimulation of the cardiac sympathetic nerves or administration of sympathomimetic drugs markedly increases the force of contraction (positive inotropic effect), increases the rate of pressure developed by the ventricle, and shortens the duration of systole.
d. Ions. The major effects of ions on automaticity are caused by changes in the concentrations
of K+ and Ca2+.
(1) K+
(a) Hyperkalemia reduces the RMP by decreasing the ratio of K+;n/K+ au" Hyperkalemia in excess of 8 mEq/L depolarizes the cell and inactivates the membrane Na+
channels, which decreases the AP rate of rise and slows the conduction rate. These
changes lead to conduction disturbances (e.g., atrial standstill, AV nodal block,
reentry phenomena, increased ectopic activity), which can lead to ventricular fibrillation.
(b) Hypokalemia causes a decreased membrane conductance for K+, which reduces
RMP; this alters automaticity in the same manner as in cases of hyperkalemia.
(2) CaH
(a) Hypercalcemia slows heart rate and decreases the excitability of cardiac tissues
while it increases the tension developed by cardiac muscle. Marked hypercalcemia
results in cardiac arrest during systole.
(b) Hypocalcemia increases the slope of phase 4 of the AP and reduces RMP; both
effects increase heart rate.
2. The interatrial tract (Bachman's bundle) is a band of specialized muscle fibers that run from
the SA node to the left atrium. The interatrial tract causes almost simultaneous depolarization
of both atria since the conduction velocity through this tissue is faster than through the regular
atrial muscle fibers.
3. Internodal tracts. Three bundles of specialized cells (the anterior, middle, and posterior
internodal tracts) connect the SA and AV nodes. The internodal tracts are more resistant to
the blockade of impulses than the atrial muscle, which makes it more likely that SA node
impulses reach the AV node to initiate ventricular depolarization.
4. The AV node is located just beneath the endocardium on the right side of the interatrial septum
near the tricuspid valve (see Figure 2-7). The AV node is normally the only path by which the
ventricles are activated. It is richly supplied by fibers from both the sympathetic and vagus
nerves, which can affect the conduction of impulses. Occasionally an alternative pathway
develops between the atria and ventricles during the formation of the heart. Its presence results
in abnormal excitation of the ventricles, which can be detected by an EKG tracing; the condition
is called the Wolff-Parkinson-White syndrome (see IV C 3).
a. AV nodal delay. The conduction velocity through the AV node is extremely slow (0.020.05 m/sec), which delays ventricular depolarization for 100-150 msec after atrial depolarization. This delay provides time for atrial contraction to occur, which enhances ventricular
filling, especially at fast heart rates.
b. AV nodal block. The A V node cells have long refractory periods and a very slow conduction
velocity. This limits the number of impulses that can be conducted through the AV node to
about 180 beats/min. The ventricles are protected from being driven at excessive rates,
which can reduce the pumping effectiveness of the heart. Atrial impulses can be completely
blocked from passing through the AV node by various diseases; an ectopic ventricular
pacemaker usually functions to drive the ventricles but at a very slow rate. Under these
conditions it may be necessary to implant an electronic device (pacemaker) in order to
stimulate the ventricles to contract at an adequate rate.

c. Autonomic effects
(1) Sympathetic stimulation increases the conduction rate through the AV node and
enhances the transmission of impulses.
(2) Parasympathetic (vagal) stimulation to the AV node slows the conduction velocity
and prolongs the refractory periods. These effects make it more likely that AV block
will occur and are the result of the reduced slow inward Ca2+ current caused by ACh.

5. Ventricular conduction. Impulses conducted through the AV node are distributed to the
ventricles over specialized fibers.
a. The bundle of His is the continuation of the AV node and is located beneath the endocardium on the right side of the interventricular septum. The bundle of His divides into the
right and left bundle branches.
b. The right and left bundle branches proceed on each side of the interventricular septum
to their respective ventricles. A block of the depolarization impulse can occur in either of
the bundle branches, which delays the depolarization of the ventricle on that side. The
affected ventricle is eventually depolarized by the spread of impulses through the regular
ventricular muscle fibers. A right or left bundle branch block produces characteristic EKG
changes.
c. Purkinje fibers arise from both bundle branches and branch out extensively just beneath
the endocardium of both ventricles (see Figure 2-7). These cells have the largest diameter
in the heart (70-80 !J-) and possess the highest conduction velocity of the AP (1-4 m/
sec). This insures that both ventricles contract almost simultaneously, which increases the
effectiveness of contraction.
d. Ventricular muscle depolarization occurs from the endocardial surface to the epicardium.
Depolarization initially reaches the surface of the heart at the apex and then spreads
throughout the myocardium. The last area of the ventricles to be depolarized is the base of
the left ventricle. The conduction velocity of the AP through the ventricular myocardium is
0.3-0.4 m/sec.
D. Excitation-contraction (EC) coupling is the term used to define the events that connect the
depolarization of the cell membrane to the contraction of the muscle fibers.

1. Structure of myocardial cells. Each of the cardiac cells contains many bundles of protein
strands called myofibrils, which, in cardiac muscle, are surrounded by an extensive network
of tubules known as the sarcoplasmic reticulum (see Figure 2-5).
a. The sarcoplasmic reticulum in cardiac muscle is much less developed than in skeletal
muscle. It contains dilated terminals (cysternae), which are located next to the external
cell membrane and T tubules. The sarcoplasmic reticulum and the cysternaecontain high
concentrations of ionized Ca2+.
b. T tubules are continuations of the cell membrane, and, therefore, the T tubules conduct
the AP into the interior of the cell. T tubules are continuous with the sarcolemma and
invaginate into the interior of the cell at the Z line of the sarcomere in mammalian cardiac
cells. T tubule depolarization causes the release of Ca2+ from the sarcoplasmic reticulum
and cysternae.
c. Myofibrils are composed of thick and thin filaments called myosin and actin, respectively.
The thin filaments also contain two other proteins, troponin and tropomyosin.
d. Sarcomeres are the contractile unit of myofibrils (see Figure 2-5).
2. Contraction mechanism
a. Effects of CaH (Figure 2-8)
(1) The concentration of Ca2+ within the cell is very low between contractions. Each muscle
contraction is preceded by an AP, which depolarizes the cell membrane and T tubules.
(2) The AP causes Ca2+ release from the sarcoplasmic reticulum, and additional Ca2+
diffuses into the cell across the T tubules (slow inward Ca2+ current). The AP raises the
sarcoplasmic Ca2+ concentration about 10-fold compared to the resting conditions.
(3) At high Ca2+ concentrations, troponin binds four Ca 2 + ions per molecule. When troponin is saturated with Ca2+, a shape change occurs in the troponin, which causes the
tropomyosin molecule to uncover the cross-bridge sites. This allows cross-bridges to be
formed between actin and myosin.
(4) Under normal circumstances, all of the troponin is not saturated with Ca2+ so that
factors raising intracellular Ca2+ concentration can provide more sites for cross-bridge
formation.
(5) An increase in the number of cross-bridges formed causes an increase in the force of
the contraction.
b. Sympathetic effects. Sympathetic nerve stimulation or the administration of drugs (e.g.,
epinephrine or norepinephrine) increases the concentration of Ca2+ within the cell and
results in a more forceful contraction. Norepinephrine also causes a more rapid uptake of
Ca2+ by the sarcoplasmic reticulum, which shortens the duration of both the AP and the
contraction.

A. Resting muscle: low Ca 2+


Troponin

Ql

c:

B. Activated muscle: high Ca 2+ + ATP

Ql

c:

Crossbridge

C. Shortening: Ca 2++ ATP

D. Dissociation: Ca 2+ + ATP

.,

Ql

c:
N

Figure 2-8. Proposed sequence of


changes during the contractile process. (A) In resting muscle the Ca 2 +
concentration is low, and the troponin molecules shield the cross-bridge
sites from the myosin. (B) During depolarization of the myocardial cells,
Ca2+ concentration rises, which results in a configurational change in the
troponin so that cross-bridge sites are
uncovered. In the.presence of AlP,
cross-bridges are formed between the
actin and myosin heads, and AlP is
hydrolyzed. (C) Energy from AlP
hydrolysis causes configurational
change in the myosin heads so that
actin is ratcheted about 10 nm per
cycle in relationship to the myosin
molecule. Note the movement of the
Z line in the figure. (0) AlP is again
required to allow dissociation of the
cross-bridges to allow the muscle to
relax. Each contraction involves numerous cycles through steps C and D
as described in this figure.

c. Digitalis is given to patients who are in heart failure, because it also increases the force of
cardiac contraction by raising the intracellular Ca2+ concentration. Digitalis has been used
for several hundred years to treat heart failure and was initially prescribed as a tea prepared
from the leaves of a flowering plant (Digitalis purpurea, purple foxglove).
E. Frank-Starling effect tarlin 's aw of the heart One of the major factors that controls the
force of cardiac contraction is t e initia engt, or preload, of the muscle fibers (Figure 2-9).
Changes in preload alter the number of available cross-bridges by changing the overlap of the
!bif!<...-and thin filaments. In the intact heart the preload depends on the volume of blood in the
ventricles just before contraction begins (end-diastolic volume).
r.
(
- J ) i ' o r i r~,6o j".6-1
1. An increase in end-diastolic volume lengthens the cardiac fibers, which, UP to an oetimurfl,
exposes more of the myosin heads (cross-bridge sites) to the actin. The adci'iffonai crO'ss-Sndges
allow the heart to generate more force with each contraction. Thus, an increase in preload
produces a stron er contrac i
reater volume of blood is um
e heart. These
effects resu t in a igher pressure being generated by the myocardium.
.
"
n
-'.'.~ c (
d.,.c{~J.,,(
.;r
't ' '. f r ::

<

-"

'~ ~10
2-

~
,I

\J

~1

~
"-;',

-~

2-

'E

S
Co
S

~
~ 'E'"
~j 0'"
0

(.)

2
0
-4

12

16

20

Venous pressure (mm Hg)

Figure 2-9. A series of Frank-Starling


curves. Changes in cardiac output
can be accomplished by alterations in
the preload (venous pressure or ventricular end-diastolic volume) causing
movement along one of the curves
(Frank-Starling mechanism) or by
shifting from one curve to another.
The latter mechanism indicates
changes in ventricular contractility
(inotropism).

1 ~(//
..,'
,
c (-"~ro (/ y _
2. The Frank-Starling mechanism is important in balancing the output of the two sides of the heart
over extended periods of time. In the intact heart, changes in stroke volume also depend oJ).
the contractility of the ventricles.

F. Contractility
1. Increased contractility (positive inotropism] is defined as an increased force of contraction
at a constant ereload. Positive inotropism can also be defined as an Increase in the maximal
verocity of shortening (V max).
a. This is in contrast to the Frank-Starling relationship, where an increase in the force of
contraction is produced by an increase in preload. The difference between an increased
contractility and an increase in preload is shown in Figures 2-9 and 2-10. Positive inotropic
~ffects are roduced b
m athetic stimulation and thetffect of sympathomimetic drugs
and digitalis ~ ycosides.
b. Figure 2-9 sows a family of ventricular function (Frank-Starling) curves and depicts the
ventricular performance as a function of the ventricular end-diastolic volume or preload.
Positive inotropic effects.produce a shift toward the left; that is, the same cardiac work can
be performed at a smaller end-diastolic volume.
~

'
*
by,hpoxi,a,~,

2. Decreased contractility e
ive inotro is is caused
and';;xocardial
ischemia (lack of blood flow) or nfarcts eath of tissue). Negative inotropism causes a shift
toward the right in the ventricular function curves, which results in cardiac failure; less work is
performed by the heart from any given end-diastolic volume or pressure. The intact heart shifts

/ V max
Epinephrine
...."'..."''''.''''''
-

10
Afterload (g)

Increased preload
Control

15

Figure 2-10. The velocity of shortening as a function of afterload.


Changes in preload (venous pressure,
ventricular end-diastolic pressure, or
volume) do not cause an increase in
the maximal velocity of shortening
(V~a)' Sympathomimetic substances,
such as epinephrine, increase contractility and result in an increased
V max' La = initial length of muscle.

from one curve to another (alters contractility) in response to various physiologic or pathologic
events.

III. ELECTROCARDIOGRAPHY. The science of electrocardiography is about 100 years old. The
techniques of recording electrical activity from the heart originally were developed by Dutch physiologist, Willem Einthoven, who referred to the recording of this electrical activity as an "Elektro Kardio
Gramm" (thus the abbreviation EKG). The EKG provides a method of evaluating excitation events,
arrhythmias, tissue damage, the presence of ischemia or necrosis, and hypertrophy of the heart. The
EKG does not provide any information on the mechanical performance of the heart, and at times
even abnormal electrical activity may not be recordable.

A. Volume conduction

1. The body tissues function as an electrical conductor because they contain electrolytes. The
heart is assumed to lie centrally within the thorax, and its electrical activity is conducted to the
body surface through the body fluids.
a. Attaching electrodes to the body surface allows voltage changes to be recorded within the
body after adequate amplification of the signal. A galvanometer within the EKG machine
is used as a recording device. Galvanometers record potential differences (voltages) between
two electrodes.
b. EKG recordings (electrocardiographs) are merely the differences in voltage between two
electrodes located on the body surface.
2. Zero potentials. During diastole the cardiac cell membranes are polarized-positively charged
on the outside and negatively charged on the inside. Electrodes on the skin do not detect any
voltage differences since all parts of the heart are equally polarized. Thus, the recording shows
no deflection from the zero potential line (Figure 2-11 A).

l. Action potentials
a. Depolarization. Excitation of a portion of the heart causes the myocardial cells to depolarize, which produces a reversal of membrane potential in this area. The outside of these cells
is now negatively charged with respect to ground. Thus, a potential difference exists between
the depolarized cells and the nonexcited cells (see Figure 2-11 B). This potential difference
can be recorded from surface electrodes, and the direction of its deflection will depend on
the polarity of the electrodes. When the entire heart has been depolarized, all of the cells
are now negatively charged outside, and both electrodes again "see" the same potential.
The galvanometer again will read zero (see Figure 2-11 C).
b. Repolarization. Assuming that repolarization proceeds in the same direction as depolarization, the galvanometer will be deflected in the opposite direction (see Figure 2-11 D and E)
during the repolarization process. The resulting record is termed a biphasic action potential
because there are two waves in opposite directions.
4. Equivalent dipole. The voltage differences between resting, depolarized, or repolarizing cells
function as a battery. The various charges that are present can be represented as a point source
of charge and are termed an equivalent dipole.
a. The total charge that is present depends on the mass of tissue involved as well as the
magnitude of the membrane potentials.
b. The cardiac dipole is a vector quantity because it has both magnitude and direction. Vectors
are represented as arrows with the arrowhead indicating the direction and the length of the
arrow indicating the magnitude.
S. Surface potential, or the magnitude of the voltage recorded at the body surface, is a function
of electrode position and the orientation and magnitude of the dipole. By convention, a wave
of depolarization approaching the positive electrode results in an upward (positive) deflection
of the EKG tracing. A wave of depolarization proceeding parallel to an electrode axis (the
line connecting two electrodes) produces the maximal deflection for that dipole, whereas a
depolarization wave perpendicular to the electrode axis produces no net deflection of the
tracing (i.e., positive and negative waves are equal). Figure 2-12 depicts these relationships,
which are essential to understand when analyzing EKG recordings.
B. EKG leads. The standard EKG consists of recordings from 12 different leads. A lead consists of
the recording from electrodes placed at specific sites on the body, Each lead shows the same
cardiac events as other leads but from a different view. Additional leads are used in special
circumstances.

++++++++
- -

A. Resting

B. Depolarizing

c.
+

+++

Depolarized

-1L

D. Repolarizing

++++

E. Repolarized

Figure 2-11. The effects of depolarization and repolarization of isolated, excitable tissue on a galvanometer's
deflection, using standardized connections. Note that
when repolarization and depolarization are in the same
direction, the galvanometer deflections are oppositely directed. In the limb leads of an electrocardiogram, the
QRS complex and the T wave normally are in the same
direction.

1. Einthoven's triangle. The torso is considered to be an equilateral triangle with the right and
left shoulders and left leg as the three apices. The right leg serves as a ground connector; all
four electrodes must always be attached to the extremities, which merely serve as conductors
from the torso.
a. Zero potential lines. If lines are drawn perpendicularly from the center of each side of an
equilateral triangle, they will meet at the center of the triangle. These lines represent the
zero potential lines for the three sides of the triangle (Figure 2-13A).
b. Bipolar limb leads. Three leads are formed by measuring the potential differences between
any two of the limb electrodes. These leads are selected by a switch on all standard EKG
machines.
(1) Lead I measures the difference in voltage (potential) between the left-arm (LA) electrode
and the right-arm (RA) electrode (LA - RA).
(2) Lead II is the potential at the left leg (LL) minus the potential at RA (LL - RA).
(3) Lead III is LL - LA.
2. Unipolar (V) leads. If the three limb leads are connected to a common terminal, the combined
voltage from the three leads will be zero, theoretically. This common terminal can be attached
to the negative pole of a galvanometer and a fourth, or exploring, electrode can be attached
to the positive pole. The galvanometer can still only read the potential difference between two

1,3

l.................~ ....i!"...........l + f~
~2

>

Electrode axis

Time
B

1,3

2~

l.. ---......~ ....~ ................l +

Electrode axis

-V2

1,4

3~2

l.................t................l +
Electrode axis

Figure 2-12. The deflection of a galvanometer needle using standard


connections. (A) A wave of depolarization approaching the positive electrode causes an upward (positive) deflection. (8) A wave of depolarization
approaching a negative electrode
causes a negative deflection. (C) A
wave of depolarization that is proceeding perpendicular to the electrode axis produces no net deflection.

points, but, if the common electrode is at zero volts, then the other electrode will provide the
actual or absolute voltage at the body surface. This arrangement of connections is termed a
unipolar electrode and is used to record the precordial or chest leads from standardized
sites. There are six precordial leads, also termed V leads, in the standard EKG. These precordial
leads measure electrical activity in the horizontal plane of the body.
a. V 1 is in the fourth intercostal space (ICS) just to the right of the sternum.
b. V 2 is in the fourth ICS just to the left of the sternum.
c. V 4 is at the midclavicular line (MCl) in the fifth ICS.
d. V3 is halfway between V 2 and V 4
e. V 5 is in the anterior axillary line at the same level as V 4'
f. V 6 is in the midaxillary line at the same level as V 4 and V 5'
3. Augmented unipolar leads. Any of the three limb electrodes can be used to record cardiac
potentials in comparision to the common terminal. For example, the voltage recorded at RA
can be determined by the equation: RA - (RA + LA + LU. The resulting voltage is small,
because the potential difference is reduced by the RA potential in the common terminal.

RA~----~------~

A. Einthoven's triangle

aVL

B. Triaxial system

aVF
C. Hexaxial system

Figure 2-13. (A) Einthoven's triangle showing connections, leads (/, II, and 1/1), and lead polarity. Two other
axis systems, the triaxial system (8) and the hexaxial system (C), also are shown. The electrical zero occurs at the
center of Einthoven's triangle and at the center of the
triaxial and hexaxial systems. RA = right arm; LA = left
arm; LL = left leg; aVR, aVL, and aVF = augmented
unipolar limb leads.

Disconnecting the RA lead from the common terminal increases the potential difference by
50% and results in the augmented unipolar limb lead, aVR.
a. aVR is the potential difference betwen RA and (LA + LL).
b. aVL is the potential difference between LA and (RA + LL).
c. aVF is the potential difference between LL and (RA + LL).
d. Augmented limb lead axes bisect the angles of Einthoven's triangle. The axis of aVR is from
-150 to +30; the axis of aVL is from _30 to + 150; and the axis of aVF is from +90 to
-90.
4. A triaxial reference system is obtained by moving the sides of Einthoven's triangle so they
intersect at the center of the triangle. Lead I then divides the system into an upper (negative)
and a lower (positive) hemisphere (see Figure 2-13B).
5. Hexaxial reference system. Superimposing the axes of the augmented unipolar limb leads on
the triaxial system provides a hexaxial reference system with axes at every 30 (see Figure
2-13C). In this system lead I and aVF divide the system into quadrants with the zero isopotential
point for all leads at the central intersection.

6. A standard 12-lead EKG consists of the three bipolar limb leads

(I, II, and III), the three


augmented limb leads (aVR, aVL, and aVF), and the six chest leads (V,-V6).

C. Electrocardiography. EKGs are the tracings of the surface cardiac potentials recorded against
time. These tracings usually are made at a standard recording speed (25 mm/sec) and amplification
(1 mV = 1 cm deflection). With standard EKG paper, each small horizontal division represents
0.04 second and each large division is 0.2 second, while each small vertical division represents
0.1 mY. Each lead indicates the same electrical events in the heart but from a different angle in
anyone subject unless there is a change in the sequence of excitation. This can be compared to
looking at a house from the front, the side, and the back. It is all the same house, but it looks
different from different angles (Figure 2-14).
1. EKG waves and intervals (Figure 2-15)
a. The P wave is due to atrial depolarization, and it is normally positive (upright) in the standard
limb leads and inverted in aVR.
b. The P-R interval is measured from the onset of the P wave to the onset of the QRS complex,
and it normally varies between 0.12 and 0.21 second depending on heart rate. The P-R
interval is a measure of the AV conduction time, and it includes the delay through the AV
node.

~r?t
o :

______________

RA~-

~~~~

________--.
LEAD I

II

III
Figure 2-14. Reconstruction of the QRS complex from the depolarization loop for the three bipolar limb leads.
Arrows indicate the direction of the recording. RA = right arm; LL = left leg; LA = left arm.

S-Tsegment

P-R segment
I

1
1

I
I
I

I
I

I
I
I
I

I
I

I
I

:
I

-0.2 sec-

I
I
I
I

r---5mm-

I
I
I
I
I
I
I
I
I
I

r
I

I
I
I
I

I
l

1 mV

I
I
I

/~

:
:/
1-~n
1

I
I
I
I

i--P--:
I

I
I

I
I
I

IIQ Sl1
laRS:
I
I

I
I

\..
I
I
I

I
I
I

I
I

I
I
I
I

I
I
I

I
I

I-P-R---l
1
interval

1I

Ll

-:
I

I
I

1cm

I
I

O-T interval

..:
I

Figure 2-15. A typical electrocardiogram showing the various waves and segments as well as the standard voltages
and times.
c. The QRS complex is caused by ventricular depolarization, and its duration is normally less
than 0.08 second. A prolongation of the QRS duration indicates an intraventricular conduction block caused by blockage of one of the bundle branches or the presence of an ectopic
ventricular pacemaker.
d. The T wave is caused by ventricular repolarization, and it is normally in the same direction
as the QRS complex since ventricular repolarization follows a path that is opposite to
depolarization.
e. The R-R interval is the time between successive QRS complexes. The heart rate is equal to
60 divided by the R-R interval in seconds.
2. The mean electrical axis (MEA) is the average vector produced by any given wave in the
EKG. The MEA can be derived for the P, QRS, and T waves by using any two standard limb
leads or any two augmented limb leads. An augmented limb lead and a bipolar limb lead cannot
be used together because of the difference in amplification of the two leads.
a. Method of measurement. Clinically, the MEA of the QRS complex is determined by
algebraically summing the heights of the Q, R, and 5 waves in each of two leads. Figure
2- 16 shows this method.
LEAD I
o 5

LL

Figure 2-16. Vectorial analysis of mean electrical axis


determination. The Q and 5 waves are summed and
added algebraically to the R wave to give the magnitude
of the vector in two leads. The magnitudes are plotted
along the respective leads toward the appropriate polarity. Perpendiculars are drawn through the arrowheads,
and the intersection marks the head of the mean electrical
vector. The vector is drawn from the electrical zero (the
center of the triangle) to the intersection of the perpendiculars. The mean electrical axis, in degrees, and the magnitude are given by the direction and length of the mean
electrical vector (axis). RA = right arm; LL = left leg; LA
= left arm.

(1) Plotting of lead vectors. The vector from each lead is plotted on an appropriate scale,
using either the Einthoven triangle, the triaxial, or the hexaxial reference system. The
tail of the arrow is placed on the zero potential point, and the vector is drawn along
the respective lead with the head of the arrow pointing toward the correct pole.
Perpendiculars are then drawn to the heads of the two vectors.
(2) Drawing the mean vector. The head of an arrow is drawn at the intersection of the
two perpendiculars, and the tail is at the center of the triangle or the zero potential
point. The length of this arrow represents the magnitude of the MEA, and its direction,
in degrees, represents the electrical axis in the frontal plane.
b. Axis deviation. The MEA of the QRS determines if axis deviation is present.
(1) A normal axis is present if the MEA lies between -30 and + 120.
(2) Right axis deviation (RAD) is present when the MEA lies between + 120 and + 180.
RAD is caused by right ventricular hypertrophy secondary to chronic lung disease or
pulmonary valve stenosis or to delayed activation of the right ventricle as in right bundle
branch block.
(3) Left axis deviation (LAD) is present when the MEA lies between -30 and -90. This
condition is associated with obesity, left ventricular hypertrophy, or left bundle branch
block.
(4) An indeterminate axis is present if the MEA lies between -90 and -180. Indeterminate
axis may be due to either extreme right or extreme left axis deviation.
3. Electrical orientation in the horizontal plane is determined using the precordial or chest leads.
a. The transition zone, or null point, is defined as the electrode position in which the QRS
is biphasic (i.e., when the algebraic sum of positive and negative waves is zero). The null
point normally occurs between leads V 2 and V 3. A leftward shift in the null point is associated
with left ventricular hypertrophy; a right shift is associated with right ventricular hypertrophy.
b. The electrical axis in the horizontal plane is perpendicular to the null point.
D. Vector loops. The instantaneous cardiac vector represents the electrical vector generated by the
cardiac dipole during the depolarization process. This vector begins at the zero isopotential point
and inscribes a loop as the tissues are depolarized. Three loops can be recorded during one cardiac
cycle. The P loop is caused by atrial depolarization, the QRS loop is caused by ventricular
depolarization, and the T loop is due to ventricular repolarization (Figure 2-1 7A). Atrial repolarization cannot be recorded with standard techniques because of the prolonged time-course and the
small voltages involved.
1. The P loop is small and is directed leftward and inferiorly, resulting in a positive P wave in the
three bipolar limb leads.

LL
A. Vector loops

__________________

__________________

2. The normal QRS loop is inscribed counterclockwise and is directed toward the left hip,
posteriorly. The QRS loop generates the QRS complex on scalar EKGs and represents a specific
sequence of activation. Ventricular depolarization is a continuous process, but it can be divided
into stages for discussion (see Figure 2-17B).

LL
B. Stages of QRS

Figure 2-17. (A) Schematic representation of the vectorcardiographic loops P, QRS, and T, which can be recorded
with appropriate equipment and placement of electrodes. (B) A QRS loop that has been arbitrarily divided into five
stages.

a. Stage 1. The initial phase of ventricular depolarization involves the left endocardial surface
of the interventricular septum. Depolarization spreads superiorly and to the right, resulting
in a small vector directed toward the right shoulder. This initial vector produces small
negative waves in leads I and II and small positive waves in the right precordial leads.
b. Stage 2 represents depolarization of the remainder of the interventricular septum and the
subendocardial areas of both ventricles. The resultant vector is directed inferiorly and
produces large positive waves in leads II, III, and aVF.
c. Stage 3 depolarization involves the remainder of the right ventricle and a large portion of
the left ventricle. Since the left ventricle normally has a larger muscle mass than the right,
the stage 3 vector is directed toward the left. This vector produces R waves in lead I and
causes a progressive increase in the R waves in the precordial leads V 2 to V 6'
d. Stage 4 represents the last portion of the ventricle to depolarize, which is the posterior base
of the left ventricle. This vector is directed toward the left shoulder and produces small
S waves in leads III and aVF.
e. Stage 5 is the return of the cardiac potential to the zero potential point and the end of the
depolarization process.
3. T loop. The direction of the repolarization process is roughly opposite to that of depolarization.
This occurs because the inner layers of the myocardium are slightly hypoxic due to vascular
compression during systole, and hypoxia prolongs the relative refractory period and the onset
of repolarization. The opposite direction of repolarization compared to depolarization results
in T waves that normally are in the same direction as the QRS complex.

IV. CARDIAC RATE, RHYTHM, AND CONDUCTION DISTURBANCES


A. Cardiac rate
1. Normocardia is a normal resting heart rate of 60-100 beats/ min.
2. Tachycardia is a heart rate in excess of 100 beats/min.
3. Bradycardia is defined as a heart rate that is less than 60 beats/min, which commonly is seen
in well-trained athletes.
B. Cardiac rhythms
1. Sinus rhythm is present when the SA node is the pacemaker; sinus rhythm is assumed if each
P wave is followed by a normal QRS complex, the P-R and Q-T intervals are normal, and the
R-R interval is regular.
a. Sinus arrhythmia is present if all of the above criteria apply except that the R-R interval
(cardiac rate) varies with respiration. Heart rate normally increases during inspiration and
slows during expiration due to variations in vagal tone that affect the SA node. Sinus
arrhythmia is common in children and in endurance athletes with slow heart rates.
b. Sinus tachycardia is a normal response to exercise and also occurs in the presence of fever,
hyperthyroidism, and as a reflex response to low arterial pressures.
2. Atrial rhythms
a. Atrial tachycardia occurs when an ectopic atrial site (outside the SA node) becomes the
dominant pacemaker.
(1) Characteristics. Atrial rhythms are characterized by very regular rates ranging from
140 to 220 beats/min. Usually, atrial tachycardias have a very rapid onset and last
only for a few seconds or minutes, which is why they are termed paroxysmal atrial
tachycardia (PAT). Occasionally, these tachycardias may last for hours or days. A
prolonged bout of atrial tachycardia is termed a supraventricular tachycardia since it
is usually not possible to determine the actual pacemaker site.
(2) Causes. PATs may be precipitated by overindulgence in caffeine, nicotine, or alcohol,
or they may occur during anxiety attacks. PAT may be due to discharge of a single
ectopic site, or it may be due to a reentry phenomenon.
(a) Reentry occurs when a cardiac impulse reexcites an area of the heart that previously
had been depolarized. This phenomenon requires parallel paths of depolarization
that have different conduction velocities and refractory periods.
(b) Fast fibers regain excitability by the time the impulse arrives from the slow conducting fibers over connecting paths, and the fast fibers are reexcited.

(3) Treatment. Attacks of PAT can usually be interrupted by increasing vagal tone (carotid
sinus massage or a Valsalva maneuver); in persistent cases of PAT, administration of
digoxin or verapamil may be required, or, infrequently, cardioversion may be necessary
to break the attack.
b. Premature atrial contraction (PAC)
(1) Cause. PACs (also known as atrial extrasystoles) occur if an atrial ectopic site fires and
becomes the pacemaker for one beat.
(2) Significance. These beats are entirely normal, and the patient may note an occasional
irregularity in the cardiac rhythm.
(3) Diagnosis. The condition can be readily diagnosed if it occurs during an EKG tracing,
since a premature P wave is present followed by a normal QRS complex and a T wave.
The premature P wave may be of different configuration than normal, and the P-R
interval may also be altered due to the different path of depolarization that occurs. The
condition can usually be diagnosed at the bedside since the premature impulse discharges the SA node, which then must repolarize and fire after the normal interval. This
results in a shift in cardiac rhythm (Figure 2-18A). This condition is benign and usually
does not require treatment.
c. Atrial flutter occurs when atrial rates are 220-350 beats/min. During atrial flutter the AV
node is unable to transmit all of the atrial impulses so that a physiologic AV block develops,
and the ventricular rate is half, one-third, or one-fourth of the atrial rate.
(1) Cause. Atrial flutter may be due to a single ectopic focus or to a reentry phenomenon.
(2) Characteristics. The ventricular rate may be normal and very regular, although rapid
changes in ventricular rate may occur as the AV block changes from, for example, 4:1
to 3:1. The P waves produce a saw-toothed appearance on the EKG, which is virtually
diagnostic of atrial flutter.
(3) Significance. Atrial flutter differs from atrial tachycardia only in the atrial rate, and it
has a similar prognosis and treatment.
d. Atrial fibrillation is a totally irregular, rapid atrial rate in which there is contraction of only
small portions of the atrial musculature at anyone time, since large portions of the atria are
still refractory to depolarization. The ventricular rate is completely (irregularly) irregular since
only a fraction of the atrial impulses that reach the AV node are transmitted to the ventricles.
A. Premature atrial contraction
PAC
Pwave

ORS

Normal
R-R interval

t I.
'I 1

I.

I..

I.,

I.

I,

B_ Premature ventricular contraction


Pwave
ORS

Normal
R-R interval

I
,

I,

'1

Blocked
PVC

Figure 2-18_ The effect of premature contractions on the sequence and timing of atrial and ventricular events. (A) A
premature atrial contraction (PAC) causes a permanent shift in the atrial and ventricular events. (8) A premature
ventricular contraction (PVC) causes no permanent shift in rhythm, since the PVC and the subsequent beat typically
equal two normal cycle lengths.

(1) Characteristics. The baseline of the EKG shows small, irregular oscillations (F waves)
due to the depolarization of small units of the atrial musculature. There are no recognizable P waves, and the R-R interval is irregularly irregular. The QRS and T waves are
normal since the impulses that are transmitted through the AV node are normally
conducted through the ventricles.
(2) Significance. Atrial fibrillation represents an even higher rate of atrial activity than atrial
flutter. Atrial fibrillation frequently is associated with enlarged atria secondary to AV
valve disease. The cardiac output is reduced due to the valve disease as well as to the
loss of an effective atrial contraction. Prolonged periods of atrial fibrillation are associated with the presence of thrombi in the atrial appendages. Atrial thrombi may be the
source of pulmonary emboli (right atrium) or systemic emboli (left atrium).
(3) Treatment. Atrial fibrillation should be converted to normal sinus rhythm by cardioversion (defibrillation) once the presence of atrial thrombi is ruled out. The prognosis of
atrial fibrillation depends on the underlying cause.
3. AV junctional (nodal) rhythms. Pacemaker cells have been found only in the A-N and N-H
zones of the AV node; thus, the term junctional rhythm, or junctional arrhythmia, is used for
ectopic pacing from the AV node.
a. Junctional premature beats are characterized by an inverted P wave and normal QRS
complex. They have been subdivided into high or low junctional beats depending on
whether the P wave precedes or follows the onset of the QRS complex. Junctional beats
have the same significance as PAC. If the A V nodal tissues recover excitability, a retrograde
impulse may return and cause a second ventricular depolarization, which is known as a
reciprocal beat.
b. Transient or permanent junctional rhythms may arise in normal subjects due to suppression of the SA node. Permanent junctional rhythms may arise secondary to a large number
of organic heart diseases.
c. Junctional tachycardias are similar to atrial tachycardias and may be indistinguishable by
EKG; therefore, they should be included in the supraventricular tachycardias.
4. Ventricular rhythms
a. A premature ventricular contraction (PVC) can arise from any portion of the ventricular
myocardium, and, occasionally, it can occur in normal people. Frequent PVCs occur with
any form of heart disease, especially coronary heart disease, since ischemia increases the
irritability of the myocardium.
(1) Characteristics of a PVC are a prolonged (greater than 0.1 second), bizarre QRS
complex that does not have a preceding P wave. The T wave is usually oppositely
directed from the QRS complex. The origin of the PVC can be determined by vectorial
analysis using the initial portion of the QRS complex.
(2) Compensatory pause. Retrograde transmission of depolarization to the atria usually
does not occur with PVCs; thus, the atrial rate remains unaltered.
(a) The atrial depolarization that follows a PVC usually arrives while the AV node is
still refractory and, thus, is not conducted to the ventricles. This creates a pause in
the ventricular rhythm.
(b) This pause is usually fully compensatory so that the R-R interval of the beat preceding
the PVC plus the PVC interval is equal to two normal cycle lengths (see Figure
2-18B). The beat following the PVC is stronger than normal, due to the added stroke
volume, and is detectable by the patient as a palpitation of the heart.
(3) Interpolated beats. If the sinus rhythm is slow, a PVC may occur without altering the
normal R-R interval. The premature beat in this case is termed an interpolated beat.
b. Ventricular tachycardias result from a rapid, repetitive discharge from a ventricular site,
usually due to a reentry phenomenon. Alterations in vagal tone (carotid sinus massage,
Valsalva maneuver) do not affect ventricular tachycardias since the ventricles do not receive
any vagal innervation.
(1) Characteristics. The EKG reveals wide, bizarre QRS complexes occurring at a rapid
rate. The P waves are usually indistinguishable, although the SA node activity continues
independently of the ventricles.
(2) Significance. Ventricular tachycardias are almost always associated with serious heart
disease or digitalis or quinidine toxicity. However, ventricular tachycardias can also be
seen when intracardiac catheters stimulate the endocardial surface of the ventricles. A
slight movement or withdrawal of the catheter may terminate this condition. The cardiac

output is reduced during ventricular tachycardias due to a reduced filling time and a
decreased effectiveness of ventricular contraction due to asynchrony.
(3) Treatment. Cardioversion is frequently necessary to terminate an attack of ventricular
tachycardia due to organic heart disease.
c. Ventricular fibrillation is due to rapid, irregular, ineffective contractions of small segments
of the ventricular myocardium. The peripheral pulse is absent, and cardiac output is zero.
The diagnosis of ventricular fibrillation must be made by EKG since this condition cannot
be distinguished from cardiac standstill by physical signs.
(1) Characteristics. The EKG shows undulating waves of varying frequency and amplitude.
Ventricular fibrillation is often precipitated by one or more PVCs, one of which falls on
the vulnerable interval of the T wave.
(2) Treatment. Cardiopulmonary resuscitation must be started immediately until cardioversion can be carried out.

C. Conduction disturbances
1. SA nodal block. This condition, also termed the sick sinus syndrome, consists of the disappearance of the P wave for many seconds while an ectopic pacemaker, usually a junctional or
ventricular site, drives the ventricles. SA block results in bradycardia and is seen especially in
the elderly or after a coronary occlusion. Various drugs and implanted pacemakers have been
used to increase the heart rate to normal levels.

2. AV nodal block
a. First-degree AV nodal block is a prolongation of the P-R interval beyond 0.21 second due
to a slowed conduction through the AV node. This condition has no effect on the pumping
ability of the ventricles but indicates that some conduction disturbance is present. Firstdegree AV block can be caused by increased vagal tone, quinidine, acute infectious diseases,
and many diseases affecting the heart.
b. Second-degree AV nodal block is characterized by the failure of the AV node to transmit
all of the atrial impulses.
(1) Wenckebach block (Mobitz type I) is characterized by a progressive lengthening of
the P-R interval in successive beats and finally a failure of one impulse to be transmitted.
(2) Periodic block (Mobitz type II) is characterized by an occasional failure of conduction,
which results in an atrial-to-ventricular rate of, for example, 6:5 or 8:7. The P-R interval
is constant in this condition.
(3) Constant block. This form of second-degree block represents a higher degree of block
than the periodic type, and the atrial-to-ventricular rate is a constant small number ratio
(e.g., 2:1, 3:1).
c. Third-degree (complete) AV block causes the atria and ventricles to beat at independent
rates due to a complete interruption of AV conduction.
d. Significance of AV block depends upon the underlying cause. First-degree AV block due
to increased vagal tone is of no consequence. Second- and third-degree AV blocks are
usually due to organic heart disease. The onset of third-degree AV block may be associated
with prolonged ventricular standstill until a ventricular focus begins firing. The cardiac arrest,
if prolonged, can result in cerebral ischemia and syncope; this condition is Stokes-Adams
syndrome.
3. Wolff-Parkinson-White syndrome (ventricular pre-excitation or accelerated conduction)
is due to an aberrant conduction pathway between the atria and the ventricles. The aberrant
pathway eliminates or reduces the normal delay between atrial and ventricular activation.
a. Characteristic pattern. The EKG shows a shortened P-R interval, usually less than 0.1
second and, classically, a widened QRS complex with a slurred initial upstroke (delta wave).
The P-) interval may be normal or shortened, depending on whether the accessory bundle
inserts into the ventricular myocardium or the bundle of His, respectively.
b. Significance. The major significance of the accessory bundle is that it predisposes to
paroxysmal tachycardias due to a reentry phenomenon. The atrial impulse is normally
conducted to the ventricles and returns to the atria via the accessory bundle, thus triggering
the reentry mechanism.
c. Treatment. Surgical ablation of the accessory bundle is possible if it can be adequately
localized.

4. Bundle branch block. Blockage of the right or left common bundles or the left anterior or
posterior fascicles results in an abnormal sequence of ventricular excitation.
a. Right bundle branch block delays the activation of the right ventricle, and the QR5 duration
is prolonged beyond 0.12 second. The initial QR5 vectors are shifted to the left due to
unopposed activation of the left ventricle, and the final vectors are directed toward the right
as excitation spreads to the right ventricle through the myocardium. This sequence of
excitation produces the typical r5R' pattern in the right precordial leads, a wide 5 wave in
lead I, and a right axis deviation of the mean electrical axis. Repolarization of the right
ventricle follows the same path as depolarization, which causes an inversion of the T wave
in the right precordial leads.
(1) Incomplete right bundle branch block produces similar QR5 patterns as right bundle
branch block, except that the QR5 duration is 0.08-0.10 second.
(2) Significance. Right bundle branch block can occur in a normal individual as a transient
or a permanent manifestation. It occurs secondary to chronic pulmonary disease and
may appear acutely as a consequence of pulmonary embolism. Right bundle branch
block also occurs in conjunction with some forms of congenital heart disease such as
interatrial septal defect.
b. Left bundle branch block occurs in almost any form of heart disease, and its occurrence
is rare in the absence of organic heart disease. It is common in association with coronary
artery disease or conditions leading to left ventricular hypertrophy (hypertension, aortic
stenosis). Left bundle branch block is best diagnosed using the left precordial leads, which
show a QR5 duration of greater than 0.12 second and also show initial and final QR5 vectors
directed toward the left. The initial vector is due to septal activation from right to left, and
the final vector is due to delayed activation of the left ventricle.
c. Other forms of intraventricular block. Intraventricular conduction block may involve all
three fascicles of the bundle branches, a peri-infarction block, or a block in the peripheral
Purkinje network.
D. Other cardiac dysfunctions
1. Coronary insufficiency and myocardial infarction are due to a narrowing or occlusion of a
coronary artery, resulting in the reduction or absence of blood flow to an area of the myocardium. The most common cause of coronary artery narrowing is an arteriosclerotic plaque, and
the most common cause of coronary occlusion is an arteriosclerotic coronary thrombosis.
Reduction of coronary blood flow results in myocardial ischemia or infarction.
a. Myocardial ischemia
(1) Symptoms. The cardinal sign of coronary artery insufficiency is anginal pain related
either to exertion or emotional stress. The pain may be precordial or may radiate to the
shoulder, arm, or jaw. The pain is typically described as a tight band or weight upon
the chest, which results in a strangling sensation; thus, the term angina pectoris is truly
apt. The pain of coronary insufficiency typically lasts 1-10 minutes and is relieved by
rest.
(2) Diagnosis is usually made from patient history, since the resting EKG may be within
normal limits. Twenty-four-hour EKG (Holter) monitoring may provide evidence of
ischemic changes during anginal attacks. Stress (exercise) tests with EKG monitoring are
used to precipitate ischemic episodes for diagnostic purposes.
(3) EKG pattern. Myocardial ischemia results in 5-T segment and T-wave changes. An
ischemic area of the myocardium has a reduced membrane potential compared to
normal regions of the heart, which then results in a current flow from the normal regions
to the ischemic area. This current flow is termed a "current of injury" and results in an
5-T segment elevation from leads overlying the ischemic area, whereas 5-T segment
depression occurs in leads on the opposite side of the heart. Ischemia also alters the
path of repolarization and results in T-wave inversion.
(4) Treatment. The major aim is to reduce the work of the heart either with vasodilators
(nitroglycerin) or a /3-adrenergic blocker (propranolol), which reduces heart rate.
b. Myocardial infarction occurs when blood flow ceases or is reduced below a critical level.
The left ventricle is almost always the site of a myocardial infarction.
(1) Symptoms of myocardial infarction are similar to those of coronary insufficiency except
that the pain is more intense, generally lasts for more than 15-20 minutes, may not be
related to exertion or exercise, and is not relieved by nitrates.
(2) Diagnosis. The history is critical in making the diagnosis of a myocardial infarction,
while the EKG provides supportive evidence. Measurement of serum levels of cardiac

enzymes creatine phosphokinase (CPK), serum glutamic-oxaloacetic transaminase


(5GOT), and lactic acid dehydrogenase (LDH) may aid in the diagnosis, but no enzyme
is specific for the myocardium.
(3) EKG pattern. The EKG undergoes a series of changes following a myocardial infarction,
and these changes should be recorded with daily EKG tracings for diagnostic purposes.
A transmural infarct results in wide, deep Q waves from leads overlying the infarcted
area since these leads essentially record an intracavitary potential. These same leads
demonstrate an 5-T segment elevation, and, early after the infarct, there is an inversion
of the T wave.
(4) Treatment. The major aims are the relief of pain and the support of the circulation if
cardiogenic shock develops. In addition, continuous EKG monitoring should be carried
out in an intensive care unit to detect the presence of arrhythmias, especially PVCs,
which may trigger bouts of ventricular tachycardia or ventricular fibrillation. Cardioversion may be necessary to interrupt arrhythmias, or drug therapy (membrane stabilizers)
may be required to prevent the recurrence of arrhythmias.
2. Ventricular hypertrophy occurs if the work of one or both ventricles is increased sufficiently.
Ventricular hypertrophy increases the diameter but not the number of individual myocardial
cells. Thus, the diffusion distance increases for O 2 and other metabolites. The increased diffusion
distance may lead to borderline ischemic conditions.
a. Diagnostic criteria for ventricular hypertrophy
(1) R wave. There is a direct correlation between the thickness of the ventricular wall and
the height of the R wave in the overlying leads. The increased height of the R wave is
a reflection of the increased magnitude of the depolarization vector.
(2) QRS duration is increased slightly because of the increased muscle mass present. The
duration of the QR5 complex is usually less than 0.12 second, but, occasionally, it may
exceed this value.
(3) S-T segment and T-wave alterations. In the presence of ventricular hypertrophy there
is 5-T segment depression and inversion of the T wave in epicardial leads. The altered
repolarization process may be related to endocardial fibrosis and mild ischemia, which
are commonly present with ventricular hypertrophy. If changes in the 5-T segment and
T wave are the only manifestations, the condition is referred to as a ventricular strain
pattern.
b. Diagnostic criteria for left ventricular hypertrophy
(1) Axis. The mean electrical axis is shifted toward the left and superiorly, and the transition
zone of the precordial leads is shifted to the left.
(2) QRS configuration. Left ventricular hypertrophy is present if the R wave in V 5 or V 6
plus the 5 wave in V 1 is greater than 35 mm, the R wave in Vs or V6 is greater than
25 mm, the R in aVl is greater than 13 mm, or the R in aVF is greater than 25 mm. 5-T
segment and T-wave changes mayor may not be present.
c. Diagnostic criteria for right ventricular hypertrophy
(1) Axis. The mean electrical axis is usually vertical or greater than + 11 D in the frontal
plane.
(2) QRS configuration. Right precordial leads generally show tall R waves rather than the
normal 5 waves. The QR5 is usually prolonged but less than 0.12 second, the 5-T
segment is depressed, and the T wave is inverted in the right precordial leads.
(3) Diagnostic criteria for right ventricular strain (acute cor pulmonale). The appearance of 5-T segment and T-wave changes without abnormal R waves in the right
precordial leads is consistent with acute cor pulmonale. This condition occurs with
acute exacerbations of lung disease, and it is an important sign in the detection of
pulmonary emboli.

v.

THE HEART AS A PUMP


A. Pump components. The heart is two pumps in series (right and left sides), which are connected
by the pulmonary and systemic circulations. Each side of the heart is equipped with two valves
that normally maintain a one-way flow of blood.
1. The AV valves separate the atria from the ventricles and open during diastole to allow the
blood to fill the ventricles. The AV valves close during systole to prevent the back flow
(regurgitation) of blood from the ventricles into the atria. The right AV valve is called the
tricuspid valve; the left AV valve is the mitral or bicuspid valve.

2. The semilunar valves (aortic and pulmonic) open to allow the ventricles to eject blood into
the arteries during systole and close to prevent back flow of blood into the ventricles during
diastole.
3. The right ventricle is well designed to pump relatively large volumes of blood at relatively low
pressures through the pulmonary circulation.
4. The left ventricle is thick walled and can pump effectively against the higher pressures in the
systemic circulation. The left ventricle works much harder than the right ventricle because of
the high pressures and consequently is more commonly affected by disease processes than the
right ventricle.
B. Stroke volume is the volume of blood that is ejected with each beat of the heart. Both ventricles
must pump the same volume of blood during any significant time interval. Normally, the stroke
volume of each ventricle averages 70 ml of blood, but the stroke volume varies with changes in
ventricular contractility, arterial pressure (afterload), and end-diastolic volume (Figure 2-19).

1. Effects of contractility
a. Increased contractility (positive inotropism) results in a stronger contraction so that the
ventricles are able to eject more blood from the same diastolic volume, and stroke volume
increases. An increased contractility occurs with sympathetic stimulation to the heart (e.g.,
during exercise).
b. Decreased contractility (negative inotropism) occurs in heart failure and results in a
decrease of stroke volume. In this condition, the ventricle increases in size because of the
rise in residual ventricular volume. The increased ventricular volume tends to raise stroke
volume toward normal through the Frank-Starling mechanism (see Figure 2-9). Cardiac
output tends to be maintained by increasing the heart rate. In severe ventricular failure both
cardiac output and stroke volume are decreased and the ventricular diastolic pressure is
high. The increased ventricular diastolic pressure causes a high venous pressure, which is
reflected back to the capillaries and produces leakage of fluid into the tissues (edema).
(1) In left heart failure, fluid accumulates in the lungs (pulmonary edema), causing difficulty in breathing (dyspnea) and the inability to breathe when lying down (orthopnea).
(2) During right heart failure, edema occurs in the feet and ankles (dependent edema)
since the high venous pressures occur in the systemic veins.
2. Arterial pressure during systole affects the ejection of blood from the ventricle and is termed
the ventricular afterload. The higher the arterial pressure, the greater the afterload that the
ventricle must overcome. An increase in arterial pressure causes a reduction in the stroke
volume (see Figure 2-19B) and a decrease in the velocity of ejection from the ventricle (see
Figure 2-10).
3. Ventricular end-diastolic volume affects the stroke volume because of the Frank-Starling
relationship. The greater the ventricular end-diastolic volume, the stronger the contraction and
the larger the stroke volume. The ventricular end-diastolic volume increases if the pressure in
the atria or the central veins increases, because the higher pressure can stretch the ventricular
walls. The ventricular end-diastolic volume also increases if the distensibility of the ventricles
increases or if the heart rate slows, which allows more time for the blood to enter the ventricles.
4. Cardiac work is proportional to the pressure generated by the ventricular contraction and to
the stroke volume. Since both ventricles pump the same average stroke volume and since the
right ventricular systolic pressure is about one-seventh that of the left ventricle, then the work
of the right ventricle is about one-seventh that of the left ventricle.
a. Hypertension. The work of the ventricles is markedly increased if the blood pressure is
high (hypertension). Hypertension can occur in either the right or left side of the circulation.
Pulmonary, or right-sided, hypertension almost invariably is caused by some chronic
lung disease, whereas systemic hypertension most commonly is caused by an unknown
mechanism and is termed essential hypertension. Hypertension can also be caused by
endocrine, renal, or congenital conditions.
b. Hypertrophy. The increase in the work of the heart causes the ventricular muscle to enlarge
just like any other muscle if it is exercised sufficiently. The enlargement (hypertrophy) may
be harmful if the blood supply that provides O 2 does not increase in proportion to the size
of the muscle.

A. Normal pressure-volume curve

150

50

100

150

Left ventricular volume (ml)

B. EHacts of sfterload

150

50

100

150

Left ventricular volume (ml)

C. Changes In stroke volume

50

100
Left ventricular volume (ml)

150

Figure 2-19. Ventricular pressure-volume (PV) relationships and the effects of changes in afterload and end-diastolic
volume. (A) Normal PV loop. 1 = beginning of isovolumic contraction; 2 = onset of ejection; 3 = beginning of
isovolumic relaxation; 4 = onset of ventricular filling. The dashed line represents the maximal PV relationship, which
is determined by the level of contractility. (8) Increased afterload (thick fine) results in a reduction of stroke volume
since the myocardial shortening is abbreviated and the end-systolic volume is increased compared to normal.
(0 Curve 1 represents control conditions. An increase in end-diastolic volume (curve 2) results in an increased stroke
volume but the maximal PV relationship is unchanged. The slight increase in end-systolic volume is caused by the
increased aortic pressure resulting from the increased stroke volume. An increased contractility (curve 3) results in
an increased stroke volume by reducing the end-systolic volume since the maximal PV line is shifted to the left and
has a steeper slope.

c. Myocardial O 2 consumption. The amount of O 2 utilized by the heart depends on the


amount of work that is performed. The O 2 consumption can be calculated from the difference in O 2 content in the coronary arterial and venous blood and the coronary blood flow.
5. Myocardial O 2 supply. The heart is able to function without any blood flow for only very
short periods of time; thus, normal cardiac function requires a continuous delivery of O 2 to the
heart. The O 2 delivery to any tissue is equal to the arterial O 2 content (the volume of O 2 carried
per 100 ml of blood) multiplied by the blood flow. A reduction in O 2 delivery to the heart is
almost always due to a partial or complete obstruction of a coronary artery by fatty deposits
in the lining (atherosclerosis).

VI. CARDIODYNAMICS refers to the mechanical events that are associated with the contraction of
the heart.
A. Cardiac cycle events (Figure 2-20). The cardiac cycle includes both electrical (EKG) and mechanical (muscle contraction) events within the cardiovascular system. The pressure (potential energy)
is generated by myocardial contraction and is converted into flow (kinetic energy) in order to supply
all of the body cells with nutrients and to serve other circulatory functions. A full understanding of
the events of the cardiac cycle is essential to understand the physical signs of cardiovascular
disease that are elicited during the physical examination of a patient.

1. EKG. The electrical events precede and initiate the corresponding mechanical events; thus, the
P wave precedes atrial contraction, and the QRS complex precedes ventricular contraction.
2. Atrial contraction. The two atria contract almost simultaneously due to the specialized interatrial tracts that carry the excitation between the atria. The increase in atrial pressure that results
from atrial contraction is termed the a wave and is reflected back into the large veins, where
it can be recorded from the jugular vein with appropriate transducers. The level of filling of the
Reduced ejection
Rapid ejection
Isovolumic
contraction
Rapid active filling
(atrial contraction)

Protodiastole
Isovolumic relaxation
Rapid
passive filling

Reduced filling

120
100
~"-1---Aortic

80
C5
J:

60

pressure

140
_----i---Ventricular volume

M-

.s 40

C])

::J

en
en

70 ~

a.. 20
0

/~----

__ __ __ _
Atrial pressure
---"""Ventricular pressure

o
-EKG

Figure 2-20. The events of the cardiac cycle. 5" 52 (A 2 and P2), 5" and 54 represent heart sounds. The a wave (a)
is caused by atrial contraction. The c wave (c) is caused by increased pressure due to the AV valves ballooning in
the atria during isovolumic contraction. The v wave (v) indicates the opening of the AV valves.

jugular veins provides an estimate of the right ventricular filling pressure. The pressure of the
blood in the ventricles at the termination of the atrial contraction is termed the ventricular
end-diastolic pressure; this represents the maximal ventricular volume, which sets the preload
for the next cardiac contraction. The ventricular end-diastolic pressure for the left ventricle is
normally less than 12 mm Hg.

3. Ventricular contraction (systole)


a. A V valve closure
(1) Shortly after the QR5 complex begins, the pressure in the ventricular cavities rises due
to the onset of ventricular contraction. The rising ventricular pressure now exceeds the
atrial pressure and causes the AV valves to close.
(2) Closure of the AV valves is the major component in generating the first heart sound
(S,). The first heart sound is a low-frequency sound that occurs just after the onset of
ventricular contraction and that signals the onset of ventricular systole.
b. Isovolumetric (isovolumic) contraction. Once the A V valves close, the ventricular chambers are sealed from both the atria and the arteries. The ventricular volume remains constant
(isovolumic) until the ventricular pressure exceeds the pressure in the respective artery (aorta
or pulmonary artery); at this point ventricular ejection begins.
c. Rapid ejection. The onset of ventricular ejection is indicated by the rise in aortic and
pulmonary artery pressures and the decrease in ventricular volume. The initial part of
ejection is rapid; about two-thirds of the stroke volume is ejected in the first third of systole.
The end of the rapid ejection phase occurs at about the peak of ventricular and arterial
systolic pressure. The right ventricular ejection begins before that of the left and continues
after left ventricular ejection is complete. Since both ventricles eject (on the average) the
same volume of blood, the velocity of right ventricular ejection is less than that of the left
ventricle.
d. Reduced ejection. During the latter two-thirds of systole the rate of ejection declines, and
the ventricles begin to relax. Both ventricular and arterial pressures begin to decrease as the
rate of blood flow through the peripheral vessels exceeds the rate of blood flow from the
ventricles. The volume of blood remaining in the ventricles at the end of ejection is termed
the end-systolic volume.
e. Second heart sound (S2). The 52 is a relatively high-frequency sound that occurs as the
semilunar valves snap closed. The valve closure stops the movement of the blood toward
the ventricles and causes vibrations of the tissues that are heard as the 52. The closure of
the semilunar valves also causes the dicrotic notch in the downslope of the arterial pressure
pulse, the incisura. The incisura or 52 indicates the end of systole and the onset of the phase
of diastole.
4. Ventricular diastole refers to the relaxation phase of the cardiac cycle.
a. Isovolumetric (isovolumic) relaxation. Ventricular pressure falls rapidly as the ventricles
continue to relax; isovolumic relaxation only lasts about 0.04 second. The ventricular volume
remains constant from the closure of the semilunar valves (52) until the AV valves open,
which occurs when ventricular pressure falls below atrial pressure.
b. Rapid passive filling. During ventricular systole the venous return continues so that the
atrial pressure is high when the AV valves first open. The high atrial pressure causes a rapid,
initial flow of blood into the ventricles. Once the AV valves open, the atria and ventricles
are a common chamber and the pressure in both cavities falls as ventricular relaxation
continues.
c. Reduced filling and diastasis. Toward the end of diastole, the pressure in the atria and
ventricles rises slowly as blood continues to return to the heart. With very slow heart rates,
ventricular filling virtually ceases since the ventricles reach their volume limit. This phase is
termed diastasis. When there is an increase in heart rate the length of the cardiac cycle is
obviously shorter. The shortening of the cycle occurs primarily by an abbreviation of the
phase of diastasis or reduced filling. Thus, increases in heart rate to 140-150 beats/ min do
not cause a significant decrease in ventricular filling. Rapid heart rates produce an increased
cardiac output until rates of 180-200 beats/min are achieved, because the increase in rate
more than compensates for any slight decrease in ventricular filling. At heart rates in excess
of 200 beats/min the cardiac output declines due to a decrease in the ventricular filling
(ventricular end-diastolic volume is markedly reduced).
d. The third and fourth heart sounds (S3 and S4) occur during diastole and are not normally
heard in adults. 53 occurs during the rapid passive filling, whereas 54 occurs during atrial

contraction. S3 can be heard in children, and S4 is sometimes heard in people with congestive
heart failure.
5. Aortic and pulmonary arterial pressures. The recordings of the pulsatile pressure changes in
these vessels are termed pressure pulses.
a. Systole. During the period of rapid ventricular ejection, the pressure in the aorta is slightly
less than that in the ventricle. The peak of arterial pressure is the arterial systolic pressure
and occurs at the end of the rapid ejection phase. The normal aortic systolic pressure is
120 mm Hg; pulmonary arterial systolic pressure averages 15-18 mm Hg. The incisura,
during the downslope of the arterial pressure, indicates the end of ventricular systole.
b. Diastole. During diastole the arterial pressure declines as blood continues to flow from the
arteries out through the capillaries and into the veins. The lowest pressure in the artery
during a cardiac cycle is termed the arterial diastolic pressure, and it occurs just prior to
the onset of the next ventricular ejection. The average diastolic pressure in the aorta is about
80 mm Hg; pulmonary arterial diastolic pressure averages 8-10 mm Hg.
c. Pulse pressure is the difference between the systolic and the diastolic pressures, which
normally is 120 minus 80, or 40 mm Hg, for the aorta.
d. Mean arterial pressure (MAP) can be obtained by integrating the area under the arterial
pressure pulse and dividing by the cycle length. MAP can be approximated by adding
one-third of the pulse pressure to the arterial diastolic pressure.
e. Recorded blood pressure is normally written as arterial systolic pressure/diastolic pressure,
or 120/80 mm Hg.
6. Ventricular volume is difficult to measure clinically, but echocardiography can be used to
measure ventricular dimensions as well as ventricular wall and valve motion during the cardiac
cycle.
a. Ventricular end-diastolic volume is the volume of blood in the ventricle just prior to the
onset of ventricular contraction. The normal left ventricular end-diastolic volume is 120140 ml.
b. Ventricular end-systolic volume is the volume of blood remaining in the ventricle at the
end of ejection. The normal left ventricular end-systolic volume is 40-70 ml.
c. Stroke volume is the volume of blood that is ejected with each beat and equals ventricular
end-diastolic volume minus ventricular end-systolic volume, or 75-80 ml.
B. Valve lesions and murmurs (Figure 2-21). The cardiac valves may be abnormal due to either
congenital or acquired heart disease. The circulatory effect of valve lesions depends on the
reduction in cardiac output, the additional work imposed upon the heart by the extra pressure or
volume load that is generated, and the back up of blood that is produced by a particular lesion.
Abnormal cardiac valves produce signs of dysfunction that are recognizable during physical
examination. The physical signs are due to the abnormal blood flow and pressure gradients.
1. Stenosis of a valve is a narrowing of the valve orifice that retards the forward flow of blood.
Aortic or pulmonic valve stenosis causes the left or right ventricles, respectively, to generate
higher pressures in order to eject blood through the narrowed orifice. In these conditions, the
ventricular systolic pressure is much higher than the systolic pressure in the artery. This pressure
difference can be detected during cardiac catheterization. Auscultation of the chest reveals a
systolic, ejection-type murmur. These stenotic lesions can lead to ventricular hypertrophy and
eventually produce ventricular failure.
2. Insufficiency of a valve allows the backward flow of blood; for example, with aortic insufficiency there is a flow of blood from the aorta to the left ventricle during diastole, which reduces
the effective cardiac output. However, with aortic insufficiency, the left ventricle ejects a large
volume of blood so that systolic pressure increases markedly. During diastole, much of the
stroke volume returns to the ventricle, resulting in a very large pulse pressure, which is characteristic of aortic insufficiency. An insufficiency of an AV valve allows blood to flow from ventricle
to atrium during systole. Dilation of one or more heart chambers occurs to compensate for the
regurgitant blood and can eventually result in heart failure.
3. Murmurs. An important physical sign of a valve lesion is a murmur. Murmurs are caused by
turbulent blood flow resulting from narrowing of a valve or changes in the direction of the
blood flow. Normal flow is laminar or streamlined and is silent, whereas turbulent flow sets up
vibrations in the tissues that can be heard as murmurs.

Valvular events
Heart sounds
Aortic stenosis
Aortic insufficiency
Mitral stenosis

~r-~--------~~~~'--------I
I

Mitral insufficiency

Patent ductus arteriosus


(continuous murmur)
Figure 2-21. Schematic representation of the heart sounds (5, through 54)' valvular events, and murmurs generated
by left-sided valve lesions. Right-sided valve lesions begin and end in association with right heart valve events (not
shown). AC = atrial contraction; MC = mitral valve closure; TC = tricuspid valve closure; PO = pulmonic valve
opening; AO = aortic valve opening; A2 = aortic valve closure; P2 = pulmonic valve closure; MO = mitral valve
opening; TO = tricuspid valve opening.

a. Timing. Murmurs may be either systolic, diastolic, or continuous throughout the cardiac
cycle. Murmurs may also be classified as being presystolic, early diastolic, or middiastolic.
The timing of the murmurs is associated with specific valve lesions.
b. Location and radiation. Murmurs are best heard on the chest wall closest to their origin
and in the downstream direction of blood flow. For example, the murmur of aortic stenosis
is best heard at the right sternal border in the second interspace, and the murmur radiates
to, and is also clearly heard over, the vessels of the neck.

VII. HEMODYNAMICS
A. Basic units
1. Pressure gradient (dP). Fluid flow depends on a difference in total energy between two
locations within the system. The fluid always flows from an area of high pressure to one of
lower pressure; in other words, water (or blood) always flows downhill.
a. The total energy of a system equals the sum of the potential energy (pressure) and the
kinetic energy, which is due to the velocity of the blood. The energy can be converted
between potential and kinetic energy. Figure 2-22 is a system of pitot tubes, which can be
used to measure changes in flow velocity. A pitot tube facing upstream measures both
potential and kinetic energy, whereas a pitot tube facing downstream measures the potential
energy minus the kinetic energy. The difference in the fluid level in the two tubes provides
a measure of the velocity of flow.
b. Pressure is a force per unit area (dynes/cm2) and is usually expressed in terms of the height
of a column of fluid that the pressure will support. The common units of pressure are
mm Hg and cm H 2 0 (1 mm Hg = 1.36 cm H 20).

2. Flow (Q) is a volume movement per unit time, represented as the average velocity of movement

(cm/sec) times the cross-sectional area of the tube (cm 2). Flow equals the pressure gradient
between two points in the system divided by the resistance to flow: Q = dP/R. This is similar
to Ohm's Law in electricity, where Q would be amps, dP is voltage, and R is resistance. Thus,
an increase in the pressure gradient causes an increase in flow, whereas an increase in resistance
causes a decrease in flow.

----

-.-----1

Kinetic energy

____t__ _

pO""f e'e""

........................................

",
,,
,,
,

........

,,

,,

........

,,

--------

Figure 2-22. System of pitot tubes indicating the variation in total pressure, potential energy, and kinetic energy with
change in flow velocity. The arrows represent the relative velocities in the segments of the system. The narrow
section, a high-resistance segment, causes a large pressure drop. An increased flow velocity decreases lateral pressure
(increases kinetic energy) and is termed a Bernoulli effect.

B. Poiseuille's law. From Poiseuille's law, for laminar flow


dP

Q=R
R

8 x /-L x L
----=---

r4

where r = the radius of the tube, /-L = the viscosity of the fluid, and L= the length of the tube.
1. Resistance to blood flow. The major factors that determine the resistance to flow are the
radius of the vessels and the viscosity of the blood.
a. Effects of tube radius. Since the tube radius (r) is raised to the fourth power, any change
in tube radius produces a large change in resistance, which markedly alters the flow. For
instance, for a constant pressure gradient (dP), if the radius is reduced to half, the flow will
be one-sixteenth of its previous value. Blood flow to different organs or tissues is regulated
primarily by changing the radius of the arterioles. Arteriolar radius is controlled by altering
the sympathetic impulses to the blood vessels, by various drugs or hormones, or by the
release of various substances from the tissues (e.g., histamine, adenosine, prostaglandins).
b. Viscosity is the internal friction to flow in a fluid. Blood is a complex fluid because of the
presence of cells and proteins, and its viscosity varies as a function of the flow rate. Blood
with a normal hematocrit has a viscosity that is about three times that of water flowing
through medium-sized tubes or vessels.
c. Hematocrit. The amount of red cells present in blood is the major factor that alters the
blood viscosity in the body. Blood with a hematocrit of 60 has approximately twice the
viscosity of blood with a hematocrit of 40 (the relationship between hematocrit and viscosity
is exponential, not linear). An increase in hematocrit requires a greater pressure to produce
a given flow rate.
2. Total peripheral resistance is usually expressed as the ratio of the difference in pressure (dP)
to the flow (Q) such that R = dP /Q, where R is resistance. Expressed in this manner, resistance
has the units of mm Hg/L/min or peripheral resistance units (PRU). The normal systemic
circulation exhibits a total peripheral resistance of approximately 20 PRU in contrast to the
pulmonary circulation, where the resistance is about 2 PRU.
a. Systemic circulation represents a system of parallel resistances, since blood is supplied to

the body organs by individual arteries. The total resistance in a system of parallel vessels is
determined by adding the reciprocal of the resistance of each vessel

Rtatal

R,

R2

Rn

- - = - + - ..... -

Thus, the total resistance is less than the resistance of any of the individual resistances.
b. Each organ system represents a system of series resistances. A system of series resistances
is additive. The resistance in any organ (Ra) is composed of the resistance in the artery (R.),
arteriole (R ar ), capillary (Rc), and veins (R), so that

Ra

Ra + Rar + Rc + R,

C. Arterial pressure is determined by the interaction of four parameters: the elasticity of the arterial
system (E), the mean arterial blood volume (V), the pulse pressure (dP), and the systolic uptake of
blood by the arteries (dV). These parameters are interrelated by an equation derived from Hook's
law, which states
dP x V
E=-dV

1. Parameters
a. The elastic constant (E) is similar to Young's modulus of elasticity; E increases as an
individual ages, and the vascular system becomes less compliant (see Figure 2-1). An increase
in E, while stroke volume and arterial resistance remain normal, causes an increase in the
pulse pressure. E is less in the pulmonary artery than it is in the aorta.
b. The pulse pressure (dP) is largely determined by the elastic constant and the systolic uptake
of blood by the arteries. Increases in stroke volume, with other parameters constant, increase
dP.
c. Systolic arterial uptake (dV) is less than the stroke volume, because some blood flows out
of the arterial system during systole. Increases in stroke volume increase dV.
d. The mean arterial blood volume (V) normally is 10%-15% of total blood volume. Increases in V cause an increase in the arterial pressure by distending the vascular system,
which then recoils on the contained blood with a greater force.
2. Physiologic determinants of arterial pressure. The body adjusts arterial pressure by altering
heart rate, stroke volume, and peripheral resistance. It is important to understand the relationships between these parameters in order to analyze compensatory mechanisms during such
stresses as hemorrhage, syncope, and arrhythmias.
a. Factors that increase arterial blood pressure
(1) Increased heart rate raises arterial pressure, since it reduces the time for blood to leave
the arterial vessels (reduces diastole) and usually increases the cardiac output; both
factors raise the arterial blood volume and arterial blood pressure.
(a) At the onset of an increase in cardiac output, the arterial pressure rises until the
amount of blood leaving the arterial system per minute equals the amount entering.
(b) There now is a new equilibrium (i.e., inflow equals outflow), but there has been a
transfer of blood from the venous to the arterial system, which has raised the arterial
pressure so that the increased flow is forced through the systemic resistance.
(2) Increased peripheral resistance is caused by a contraction of the smooth muscle in
the walls of the arterioles, which narrows their lumens. This raises the arterial volume
by initially reducing the amount of blood that leaves the arterial system per minute. The
arterial pressure rises until the new pressure is sufficient to overcome the added resistance to flow, and outflow again equals inflow.
(3) Increased stroke volume primarily raises the arterial systolic pressure so that the pulse
pressure increases. Again, the arterial pressure increases until outflow equals inflow.
b. The elastic constant refers to the stiffness of the arterial system, which progressively
increases from in utero until death. An increased stiffness or elastance of the arterial system
causes an increase in the pressure for any increase in arterial volume. The increase in the
elastic constant of the arteries is one of the factors that raises the arterial pulse pressure
during the aging process. This increase in the arterial pressure raises the work of the ventricle
during ejection and can reduce the ventricular stroke volume since it represents an increase
in the ventricular afterload.

VIII. CARDIAC OUTPUT AND VENOUS RETURN. Over any significant period of time, the venous
return must equal the cardiac output. In the normal human, the cardiac output and the venous
return are about 5 l/min. A complicated interaction of neural, humoral, and physical factors on the
cardiovascular system maintains the flow rates at this level.
A. Vascular function curves refer to the relationship between venous return and venous pressure
in the circulatory system. With each contraction of the heart, blood is removed from the venous
side of the circulation and is ejected under higher pressure into the arteries. Increasing the cardiac
output (and venous return) increases the transfer of blood from the venous to the arterial vessels;
this causes the arterial pressure to rise and the venous pressure to fall. Thus, an increased venous
return, associated with an increased cardiac output, results in a decreased venous pressure. This
relationship is shown in Figure 2-23A.

10

10

E
::J

(/J

::J

'E
~6

::J

(/J

::J

Ql

Ql

> 2

> 2

-4

8
12
4
o
Venous pressure (mm Hg)

16

-4

c
10

C 8

C 8

:;

(/J

::J

8
12
4
Venous pressure (mm Hg)

16

10

.a
~

(/J

::J

Ql

Ql

> 2

> 2

O~----~----r---~-----r----~

-4

12
8
4
Venous pressure (mm Hg)

16

O~----r----r----r---~----'

-4

8
12
4
Venous pressure (mm Hg)

16

Figure 2-23. Vascular and cardiac function curves. In graphs A through C, venous pressure is the dependent variable
but is plotted on the x-axis in contrast to the usual method of plotting the dependent variable along the y-axis.
(A) Normal curve. Venous pressure decreases as venous return increases, since an increased flow results from blood
being transferred from the venous to the arterial system. When venous return (vascular flow) is zero, then venous
pressure is maximal and is referred to as mean circulatory pressure (MCP). (8) An increased blood volume or
sympathetic stimulation, which decreases venous compliance, shifts the vascular function curve to the right.
(0 Changes in arteriolar resistance alter the slope of the vascular function curve. An increased resistance results in a
reduction of pressure transmitted through the microcirculation so that venous pressure is lower for any given flow
rate. (0) Three cardiac function curves demonstrate the effect of changes in contractility on the relationship between
venous pressure and cardiac output.

1. Mean circulatory pressure (MCP). If the heart is stopped, blood continues to flow from the
arteries to the veins for the first few seconds until the pressures throughout the circulatory
system become equal. Since the veins now contain more blood than they did when the heart
was beating, the venous pressure is highest when the cardiac output (or venous return) is zero.
This pressure is termed the mean circulatory pressure (see Figure 2-23AJ. The Mep varies with
blood volume and the activity of the sympathetic nervous system.
a. Increased blood volume increases the Mep and shifts the entire vascular function curve
to a higher pressure, since the vascular system is more distended, just as the pressure in a
tire increases if more air is added. These relationships are shown in Figure 2-23B.
b. Sympathetic stimulation causes a contraction of the smooth muscle in the veins and
arteries. The vessels now squeeze down on the blood with more force, raising the Mep and
shifting the vascular function curve to the right.

2. Increased resistance to flow occurs when there is a constriction of the vessels, which creates
more friction between the blood and the vessels. Because of the increased friction, there is a
greater pressure drop between the arteries and the veins, resulting in a lower venous pressure
at any venous return and shifting the curve left as shown in Figure 2-23C.

B. Cardiac function curves refer to the relationship between venous pressure and cardiac output.
This is the Frank-Starling relationship or Starling's law of the heart (see II E). To summarize
this relationship: An increase in venous pressure causes a greater filling of the ventricle during
diastole resulting in a greater stretch of the cardiac muscle fibers (increased preload). The increased
preload produces a stronger contraction and a greater ejection of blood during systole. Thus, an
increased venous pressure produces an increased cardiac output. This relationship is shown in
Figure 2-230, which also depicts the effects on the heart of increased contractility (sympathetic
stimulation) and decreased contractility (heart failure).

C. Cardiovascular function curves. Because both venous return and cardiac output depend on
venous pressure, one can combine the normal curves from Figure 2-23A and 0 into one graph
(Figure 2-24). The intersection of the venous return and the cardiac output curves is the only point
where this system can function, since this is where the two flow rates are in equilibrium. Any
change in cardiac contractility, blood volume, or vascular resistance will cause the equilibrium
point to shift, either higher or lower, but again cardiac output and venous return must be equal.

1. Effects of exercise. During exercise there is an increase in cardiac output due to a number of
changes in the cardiovascular system. The effects of these changes are shown in Figure 2-25.

a. Activation of the sympathetic system causes the cardiac output curve to shift to a higher
level (see Figure 2-25, from point A to point B). Sympathetic stimulation also causes constriction of the venous smooth muscle, which increases the mean circulatory pressure, shifting
the vascular function curve to the right (see Figure 2-25, point O.
b. Vasodilation occurs in the skeletal muscles in order to increase the blood flow to provide
more O 2 and metabolites. The vasodilation reduces the total peripheral resistance, which
causes the vascular function curve to become steeper (see Figure 2-25).

10

E
:J

~
rn

:J

~c 6
cq~

-::J

S:J

III

:eIII

0
-4

4
8
12
16
Venous pressure (mm Hg)

Figure 2-24. The vascular and cardiac function curves


are superimposed. The only operating point on this graph
is the intersection of the two curves, since this is the point
where venous return equals cardiac output. In this graph
the cardiac output (venous return) equals 5 Llmin and
venous pressure is 4 mm Hg.

Exercise
Normal
10
c

:J

~
<Jl
:l

~c6
cn~

-::J
5.~
4
:;
0
<.l

co
~
co

()

-4

12

16

Venous pressure (mm Hg)

Figure 2-25. The effects of exercise on the vascular and


cardiac function curves (see VIII C 1).

2. Congestive heart failure results in another series of changes that involves both the cardiac
and vascular function curves (Figure 2-26).
a. Decrease in contractility of the heart is the primary defect in congestive heart failure. The
cardiac function curve is shifted downward, and cardiac output initially is reduced from
point A to point B in Figure 2-26.
b. Increased blood volume. Because of the reduced cardiac output, the kidneys retain fluids
and electrolytes, so that, over a period of several weeks or months, the blood volume
increases. This shifts the vascular function curve to the right, raising venous pressure, and
cardiac output returns toward normal (see Figure 2-26, point C).
c. High venous pressure invariably accompanies congestive heart failure. This high pressure
causes an increased filtration of fluid from the vascular system into the tissues and results in
edema formation. The edema may occur either in the lungs (pulmonary edema) or in the
feet (swollen ankles) depending on whether it is the left or right ventricle that is failing.

-------- Normal
Ventricular failure

10

4
<.l

co

'Eco

()

0
-4

12

Venous pressure (mm Hg)

16

20

Figure 2-26. The effect of congestive heart failure on the vascular and
cardiac function curves (see VIII C 2).

D. Measurement of cardiac output. The cardiac output (stroke volume x heart rate) averages
about 5 Llmin in a normal man and is about 20% less in a woman. Over any interval of time, the
cardiac output must equal the venous return. The cardiac output is exactly controlled to provide
an adequate mean blood pressure for delivery of nutrients and O 2 The cardiac output can be
measured using either of the following techniques.
1. The Fick principle is an example of the law of conservation of mass, a widely applied principle
in physiology. When applied to the cardiac output, the Fick principle states that the amount of
O 2 delivered to the tissues must equal the O 2 uptake by the lungs plus the O 2 delivered to the
lungs in the pulmonary artery. This is mathematically expressed as

Q
~here

V0 2

\;0 2

= ---"---

Ca0 2

Cv0 2

= cardiac output, Ca0 2 arterial O 2 content, Cv0 2


O 2 consumption (mil min).

mixed venous O 2 content, and

2. Indicator dilution
a. Theory
(1) If a suitable indicator is injected (A = amount injected) into an unknown volume of
distribution (V), the V can be estimated from the resultant indicator concentration (c)
with the equation: V = Ale.
(2) Similarly, the flow of a fluid (Q) can be measured if the mean concentration of the
indicator is determined for the time (t) required for that indicator to pass a given site.
This flow is measured with the equation: Q = A/ct.
b. Technique. In the body, the indicator is injected instantaneously but is dispersed in time
due to different transit times between the injection and sampling sites. Usually, recirculation
of some indicator occurs before passage of the indicator is complete (Figure 2-27). After the
peak concentration is reached, the indicator concentration follows an exponential timecourse, so that the curve can be extrapolated to zero concentration, and the duration of the
indicator passage (t) can be estimated (see Figure 2-27).
c. Indicators used for cardiac output measurement include Evans blue, Cardia-Green, hypertonic or hypotonic saline, ascorbate, and cold saline. Cold saline, used in thermodilution, is
the most commonly used indicator in human cardiac output studies.
d. Thermodilution uses a multilumened balloon catheter with a thermistor near the distal end.
The catheter is inserted into a peripheral vein, and the tip is floated into a branch of the
pulmonary artery; cold saline is injected through a proximal opening that is within the right
ventricle. The saline mixes with the blood in the right ventricle; the temperature change is
measured by the distal thermistor, which is downstream in the pulmonary artery. Thermodi-

...
I

I
I
I
I

I"" AT'"

-t---

Time after injection

Figure 2-27. Indicator dilution curve, which can be used


to measure cardiac output. AT = appearance time; c =
mean dye concentration for one passage of the dye; t =
passage time. The dotted line is an extrapolation of the
indicator curve to zero concentration. The area inside
the rectangle equals the area enclosed by the indicator
dilution curve. If c = 1.0 mg/L, t = 0.5 min, and a =
2.5 mg, then Q = a/ct = 2.5/(1) (0.5) = 5 L/min.

lution is a preferred technique because little or no recirculation occurs, cold saline is relatively
innocuous, and little temperature change occurs in the surrounding tissues.

IX. CONTROL OF THE CARDIOVASCULAR SYSTEM is designed primarily to maintain an adequate arterial pressure in order to provide sufficient blood flow to all of the body organs. To accomplish
this, the system is designed to control both arterial pressure and blood volume.

A. Control of the blood volume represents a complex interaction of neural, endocrine, and renal
mechanisms.
1. Neural component of blood volume control. The arterial baroreceptors, as well as stretch
receptors in the atria (low-pressure baroreceptors), serve as sensors to detect increases in blood
pressure or volume. These receptors are within the thoracic or cervical vascular system and
therefore detect changes in central vascular volume and not changes in the volume of the
extracellular space (e.g., standing causes fluid to shift from the thorax to the legs, which is
interpreted by these receptors as a decrease in the vascular volume). Impulses from both highand low-pressure receptors are transmitted to the hypothalamus where the neurohypophysial
cells are inhibited from releasing antidiuretic hormone (ADH).

2. Endocrine component
a. ADH is released from the posterior pituitary by an increased osmolality of the blood
perfusing the brain. The increased osmolality is detected by osmoreceptors located in the
anterior hypothalamus. ADH acts on the kidney to increase the reabsorption of water from
the distal convoluted tubule and especially the collecting duct; thus, water retention and
dilution of the solutes occur.
b. The renin-angiotensin-aldosterone system is the major control mechanism for electrolytes
within the body. A decrease in the mean blood pressure or pulse pressure within the kidney
causes the release of the enzyme renin from the juxtaglomerular cells of the nephron. Renin,
within the plasma, reacts with a substrate called angiotensinogen to form the decapeptide,
angiotensin I (AI). AI is converted to angiotensin II (All), an octapeptide, by angiotensinconverting enzyme (ACE), located primarily on the endothelial surface of the pulmonary
vessels and also on the renal vasculature. All is one of the most potent vasoconstrictive
agents known. All acts directly on arterioles to increase resistance and return blood pressure
toward normal. In addition, All causes the release of aldosterone from the zona glomerulosa
of the adrenals. Aldosterone causes a reabsorption of electrolytes (primarily Na+) and
osmotically obligated water in the distal tubule of the kidney. The increased fluid reexpands
circulating blood volume, which tends to increase blood pressure.
c. Atrial natriuretic factor (ANF) has been found in the form of storage granules in certain
cells of the left and right atria. This material, as well as natriuretic factors from other sources,
is released into the circulation when the atria are distended by increases in blood volume.
The ANF is transported to the kidneys where it dilates afferent arterioles, which increases
the glomerular filtration rate (GFR). An increase in GFR enhances the excretion of Na+
and water, which then lowers blood volume and the pressure in the vascular system. As
the blood volume decreases, the atrial distension is reduced, which lowers the rate of release
of ANF.
B. Control of arterial pressure
1. Baroreceptors are highly branched nerve endings located in the carotid sinuses (at the bifurcation of the carotid arteries) and the aortic arch. Baroreceptors are stimulated by distension of
the vessel walls; resultant nerve impulses are carried to the central nervous system (eNS) over
the sinus and aortic nerves (branches of the glossopharyngeal and vagus nerves, respectively).
The carotid sinus represents the most distensible area of the arterial system, which indicates
that these receptors respond to stretch of the arterial walls rather than to pressure per se.
2. Baroreceptor response. The baroreceptors generate a receptor potential in response to distension of the vascular walls. The receptor potential results in the generation of action potentials
in the afferent nerves, and the action potential frequency is linearly related to the receptor
potential. The receptor potential has two distinct components, the dynamic and static responses.
a. The dynamic receptor response is the initial receptor potential, which is proportional to
the rate of change of the arterial pressure.

b. The static receptor response is proportional to the new steady pressure and continues
without adaptation as long as the pressure is maintained. These responses provide information to the cardiovascular control centers in the CNS regarding the heart rate, the rate of
change in blood pressure (related to the ventricular contractility), and the pulse pressure
during each cardiac cycle.
c. Baroreceptor (moderator) reflex. Afferent neural activity over the sinus and the aortic
nerves causes a reflex slowing of the heart mediated by the vagus nerve and a withdrawal
of sympathetic tone to the arterioles, which results in vasodilation. Both of these effects lead
to a reduction in aortic pressure. Massage of the carotid sinus is used clinically to induce a
vagally mediated slowing of the heart to interrupt PAT. An increased sensitivity of the carotid
sinus is seen in some older individuals who experience syncope secondary to the vagally
mediated sinus arrest and a prolonged period of ventricular asystole. This condition is the
Stokes-Adams syndrome.
3. CNS centers
a. Medulla. Located within the reticular formation of the brain stem are diffuse collections of
cells involved in the integration of cardiovascular function.
(1) A pressor area is located irrthe lateral portion of the reticular formation, whereas
stimulation of the medial portion of the reticular formation causes a depressor response.
(2) The depressor response is due to the withdrawal of vasoconstrictor influence rather
than an active vasodilation.
(3) A cardioaccelerator center and a cardioinhibitory center are also located in the
reticular formation. These latter two centers send impulses to the heart over the cardiac
sympathetic and vagal nerves, respectively.
b. Hypothalamus. The medullary cardiovascular centers receive input from higher levels of
the brain. The effect of temperature changes on the hypothalamic centers is relayed to the
medulla and results in vasoconstriction (heat conservation) or vasodilation (heat dissipation)
in the vessels of the skin. Emotional stresses also influence heart rate and blood pressure,
and these effects are relayed from the higher centers to stimulate or inhibit the medullary
centers. In some animals, but apparently not in humans, the skeletal muscles receive a
sympathetic cholinergic innervation as well as the a-adrenergic innervation that normally
controls blood flow. Stimulation of these cholinergic fibers causes a vasodilation that occurs
in anticipation of exercise. The vasodilation opens the thoroughfare channels in skeletal
muscle, but, once exercise begins, the nutrient channels are dilated by the release of the
local vasodilator substances.
c. Cortex. Stimulation of various areas of the motor cortex leads to complex responses,
including appropriate cardiovascular adjustments. These complex motor patterns involve
autonomic responses. It is becoming more and more common to use biofeedback to attempt
to control heart rate and blood pressure, which emphasizes the control that the higher
centers can exert on the autonomic or vegetative nervous system. The sympathetic cholinergic outflow to the skeletal muscle arises from cerebral areas in lower animals and follows a
separate descending tract compared to the adrenergic sympathetic outflow.
4. Autonomic nervous system
a. Sympathetic preganglionic fibers arise from cell bodies in the intermediolateral gray
columns of the spinal cord of the thoracodorsal regions. Axons of these cells exit the CNS
through the ventral root and proceed to the paravertebral chain of ganglia where they
synapse with the postganglionic cells of the sympathetic system. All of the preganglionic
sympathetic fibers are cholinergic, while sympathetic postganglionic fibers may be either
adrenergic or cholinergic. Sympathetic postganglionic cholinergic fibers innervate some
exocrine glands and sweat glands and are the vasodilator fibers in skeletal muscles of certain
species.
(1) Vasomotor tone. Most of the sympathetic nerves to blood vessels exhibit some neural
activity that induces about 50% of the maximal vasoconstrictor activity in most of the
vascular beds. Thus, vasoconstriction or vasodilation is produced by increasing or
decreasing the sympathetic neural activity.
(2) Denervation hypersensitivity occurs following interruption of the sympathetic nerve
supply to vascular smooth muscle and certain glands. Normally, receptors are located
in the subjacent areas of the nerve terminals. However, following denervation, receptors
develop over the entire membrane; thus, the end organs are extremely sensitive to any
circulating catecholamines.

(3) Receptors to sympathetic mediators are of two major types.


(a) a-Receptors are stimulated most strongly by epinephrine and norepinephrine and
cause vasoconstriction.
(b) {3-Receptors are of two types, {31 and {32' both of which are most strongly stimulated
by isoproterenol and least stimulated by norepinephrine. The {31 receptors are
located in the myocardium (not the coronary arteries), and activation of these
receptors results in positive inotropic and chronotropic responses. The {32 receptors
are located in the other body tissues, and stimulation of these receptors leads to
inhibition (e.g., bronchodilation or vasodilation). Pharmacologic agents are available
that can stimulate or block the {31 or {32 receptors individually.
b. The parasympathetic nervous system arises from cranial and sacral outflows of the CNS.
The major effect of the parasympathetic system on cardiovascular function is due to its
action, through the vagus nerve, on the SA and AV nodes and its direct reduction of atrial
contractility. Marked efferent activity over the vagus nerve can result in extensive slowing
of the heart, sinus arrest, and/or AV nodal block.

x.

SPECIAL CIRCULATORY BEDS


A. Coronary circulation. The heart is supplied by the right and left coronary arteries; the left coronary
artery divides after about 1 cm into the anterior descending and circumflex branches. Normally,
the coronary arteries appear to function as end arteries; however, the presence of an arterial
plaque or occlusion allows anastomoses to become functional. The heart can withstand a very
limited O 2 debt so that adequate flow is essential to maintain an adequate supply of nutrients. The
heart metabolizes all substrates in approximate proportion to their vascular concentration, with a
preference for fatty acids.

1. Coronary flow rate represents approximately 5% of the resting cardiac output, or about 6080 ml blood/100 g tissue/min.
2. Myocardial O 2 consumption averages 6-8 ml O 2/1 00 g tissue/min; normally hemoglobin
releases approximately half of its arterial O 2 content to the myocardium.
a. In contrast, for the remainder of the body at rest, hemoglobin releases only about 25% of
the O 2 content in its transit through the systemic capillaries, which is equivalent to a P0 2 of
40 mm Hg. The coronary venous P0 2 is about iS-3~ mm Hg under resting conditions and
becomes even lower during exercise or stress.
b. During exercise, the cardiac O 2 consumption may increase five- to six-fold in well-trained
athletes partly due to increases in coronary flow and also by a widening of the arteriovenous
O 2 difference.
3. Coronary organization. The major coronary vessels travel in the epicardium of the heart and
subdivide, sending penetrating branches through the myocardium. It is these superficial vessels
that are corrected by coronary bypass operations when they develop atherosclerotic narrowing.
The penetrating branches further subdivide into arcades that distribute blood to the myocardium. It is important to understand that, during ventricular systole, myocardial wall tension
increases and produces a compression of the vessels, thus increasing the resistance to flow.
Consequently, the coronary flow pattern is complex due to this extravascular compression
(Figure 2-28), and maximal flow in the left coronary vessels usually occurs during isovolumic
relaxation, while arterial pressure is still relatively high and the myocardium is relaxed.
a. Left ventricular venous drainage occurs primarily through the coronary sinus with some
small contributions directly through the thebesian veins and coronary-luminal connections.
b. Right ventricular venous drainage occurs through the multiple anterior coronary veins,
and a larger proportion of flow occurs through the coronary-luminal connections than for
the left ventricle.
4. Control of coronary flow
a. Adenosine. The myocardium extracts large amounts of O 2 from the coronary blood even
at rest; thus, during exercise, it is critical that coronary flow increases to maintain an adequate
P0 2 in the myocardium. Recent evidence indicates that the major factor causing coronary
vasodilation during hypoxic states is adenosine, which is derived from adenosine monophosphate by the reaction catalyzed by the enzyme 5' -nucleotidase. Release of adenosine into
the myocardium results in an extremely strong vasodilator response.

Systole

Diastole

120
Aorta

80

-Ventricle

.................
O-t----f

:.,.,.........
:
............. ..

......... .

O~--~--~-------~

Time

Figure 2-28. Diagram of the right coronary artery flow


(solid line) and the left coronary artery flow (dashed line)
in association with pressure pulses from the systemic circulation. Peak left coronary flow occurs at the end of
isovolumic relaxation, when extravascular compression
is minimal and perfusion pressure (aorta) is high.

b. Sympathetic stimulation results in coronary vasodilation primarily because of an increase


in the metabolic rate of the myocardium. An increase in the myocardial metabolism without
a compensatory increase in coronary flow will cause the breakdown of ATP and the release
of adenosine. The coronary vessels contain both a and {3 receptors, but the a vasoconstrictor
activity is rather weak. There are no sympathetic cholinergic vasodilator fibers in the myocardium.
B. Cerebral circulation. The brain is surrounded by the rigid skull, but vascular diameter can be
altered in response to changing blood flow needs since vasodilation on the arterial side can be
compensated for by narrowing of the venous bed. Interruption of blood flow for only 5-1 0 seconds
causes loss of consciousness, while circulatory arrest for only 3-4 minutes results in irreversible
brain damage.
1. Organization. The major vessels supplying the brain in humans are the carotid arteries. This
is important since the presence of the carotid sinus provides a reflex input to maintain blood
pressure at an adequate level to maintain flow to the brain.
2. Cerebral autoregulation tends to be very effective in controlling blood flow over a range of
arterial pressures of 80-180 mm Hg. Hyper- or hypocapnia produces a doubling or a halving,
respectively, of cerebral blood flow compared to normal. The cerebral vasodilation produced
by both hypoxia and hypercapnia appears to be due to the H + that arises from lactic or
carbonic acids. The reduction in blood flow accompanying hyperventilation is sufficient to
produce cerebral hypoxia, which results in dizziness and, occasionally, fainting (syncope).
3. Neural control of cerebral blood flow seems to be limited to the larger vessels and those
located within the pia. The cerebral vessels are supplied by both sympathetic and parasympathetic neurons, but the role of the neurons remains unknown.
4. The Cushing reflex is a systemic vasoconstriction in response to an increase in the cerebrospinal
fluid (CSF) pressure. This reflex produces a marked systemic hypertension in response to
increases in the CSF pressure in order to maintain adequate blood flow to the brain. The reflex
is initiated by the medullary centers, which become activated when they become hypoxic due
to a limitation of blood flow.

C. Skeletal muscle comprises 40%-50% of the total body weight in the normal adult male. Because
of this large mass, even resting skeletal muscle utilizes approximately 20% of the total resting O 2
consumption.
1. Resting skeletal muscle receives a relatively large blood flow for its metabolic rate so that
venous O 2 content is about 17-18 mlldl. Blood flow in resting skeletal muscle varies from 1.5
to 6.0 ml/l 00 glmin, and the variability is largely due to the type of muscle. Muscles composed
mainly of red fibers receive a larger blood flow than muscles composed mainly of white fibers.

2. Exercising skeletal muscle receives about 80 ml blood 11 00 g tissue/min and extracts about
80% of the O 2 from the arterial blood. Even with marked vasodilation, the blood flow is
inadequate to maintain aerobic levels in the cells during moderate to heavy exercise. The
anaerobic threshold, which is defined as the level of exercise that produces a significant rise
in the level of lactic acid in the bloodstream, occurs at an O 2 consumption that is approximately
60% of the maximal.
D. Fetal circulation exhibits a number of differences compared to the postnatal condition. The
placenta is a major organ of the fetus and receives more than 50% of the combined output of
both fetal ventricles. At birth there are marked changes that occur in both the systemic and
pulmonary circulations as the placental blood supply is lost.
1. Pulmonary circulatory changes are primarily due to a marked decrease in pulmonary vascular
resistance as the lungs are inflated at birth. After inflation of the lungs, the pulmonary vascular
resistance is only about 10% of its value during fetal life.
a. Foramen ovale. The drop in pulmonary vascular resistance causes the pressure in the right
atrium to become less than that in the left atrium, which is opposite to the situation present
during fetal life. The pressure reversal causes the flap-like valve to close over the foramen
ovale, which then normally fuses to the septum over the next few days.
b. The ductus arteriosus, connecting the pulmonary artery and the descending aorta, conducts blood from the pulmonary artery to the aorta during fetal life. The flow through this
vessel reverses after birth as the pulmonary resistance is reduced, and the pulmonary artery
pressure becomes less than in the aorta. The ductus arteriosus begins to constrict shortly
after birth perhaps due to the higher O 2 content of the blood. There is evidence that the
patency of the ductus arteriosus is maintained by a dilator prostaglandin. Closure of this
vessel can be facilitated by the administration of aspirin, which inhibits prostaglandin formation. The ductus arteriosus may remain patent for several days, during which time a continuous murmur may be heard over the upper chest due to turbulent blood flow through the
narrowed structure.
c. Right ventricular wall thickness is about equal to that of the left ventricle during fetal life.
The thickness of the right ventricle diminishes over several months, and the electrical axis
of the heart swings toward the left over a period of several years until it lies within the
normal adult quadrant.
2. Systemic circulatory changes are caused by the loss of the placental circulation so that more
blood flow is available for all of the other body organs. The umbilical arteries constrict in
response to cooling, trauma, circulating catecholamines, and high Po 2 The loss of the placental
circulatory bed after birth causes a large increase in the systemic vascular resistance. The
increase in resistance raises the arterial pressure and consequently increases the afterload of
the left ventricle. The higher afterload leads to increases in left-ventricular and left-atrial pressures, both of which contribute to the closure of the foramen ovale.

XI. RESPONSES TO STRESS


A. Shock is defined as a condition characterized by inadequate blood flow to critical organs such as
the heart, brain, liver, kidneys, and gastrointestinal organs.
1. Causes
a. Primary shock or syncope (fainting) is usually caused by a reflex that produces arteriolar
dilation and/or cardiac slowing. Fainting is most commonly caused by pain, stress, fright,
heat, or the sight of blood. The vasovagal reflex results in vasodilation, peripheral pooling
of blood, and a decrease in the venous return. Without an adequate venous return, the
cardiac output declines, and blood pressure decreases. If the person is sitting, or especially
if standing, cerebral blood flow becomes inadequate because of the low blood pressure,
and fainting occurs (consciousness is lost). It is imperative to keep the person lying down
with feet raised to aid venous return to the heart, until the blood pressure has returned to
normal.
b. Hypovolemic shock is caused by a low blood volume and is caused by a number of
disorders.
(1) External or internal hemorrhage represents blood loss caused by injury, fractures, or
ruptured vessels.

c.

d.
e.

f.

(2) Fluid loss from the gastrointestinal tract (diarrhea or vomiting), kidneys (diabetes mellites, diabetes insipidus, or excessive use of diuretics), or skin (burns, sweating, or
exudation) can dehydrate the body and reduce the circulating blood volume.
Cardiac shock may be due to inadequate cardiac function following a heart attack, or it
may be caused by too rapid or too slow a ventricular rate. The cardiac output is markedly
decreased in either condition.
Septic shock is due to infections in the bloodstream, especially gram-negative septicemia,
or to bacterial toxins released from an infected site.
Obstruction to blood flow. This type of shock is associated with such conditions as massive
pulmonary emboli, tension pneumothorax, positive end-expiratory pressure respirations,
and cardiac tamponade. All of these conditions block the circulation mechanically, so that
cardiac output is reduced.
Other causes of shock include endocrine failure such as Addison's disease or myxedema,
severe hypoxia, severe allergic reaction, drug overdose, and trauma to the nervous system.

2. Symptoms of shock initially are agitation and restlessness, which later progress to lethargy,
confusion, and coma.
3. Physical signs of shock are dependent on the stage of shock and the severity of the dysfunction.
a. The initial stage can be characterized by a moderate reduction in cardiac output secondary
to fluid loss or a negative inotropic effect on the heart. In general, compensatory reflexes
may maintain the blood pressure at normal or low-normal levels. These reflex adjustments
result in an increased heart rate, an increase in sympathetic discharge as evidenced by
decreased blood flow to the skin (pallor), and a cold sweat.
b. The second stage of compensation occurs with a loss in blood volume of 15%-25%.
Intense arteriolar vasoconstriction results, which is usually not adequate to maintain a normal
blood pressure since the cardiac output is moderately reduced.
c. In the third stage, small additional losses of blood or cardiac function produce a rapid
deterioration of the circulation with inadequate blood flow to critical body organs. If this
state persists, a widespread tissue injury results due to the inadequate blood flow, which
causes hypoxic damage to the tissues. This condition rapidly progresses to irreversible
shock, which is fatal.
4. Treatment of shock requires attaining an adequate blood flow to all bodily tissues either by
raising the circulating blood volume (if this is depleted), reestablishing normal rhythm to the
heart, increasing the inotropic state of the ventricles with digitalis or ~-sympathetic agonists,
or administering vasodilators. The most useful measure for evaluating the extracellular fluid
volume and function of the left ventricle is the pulmonary wedge pressure. Low pressures
indicate a depletion of the extracellular fluid, while excessive pressures are associated with
large fluid volumes and/or inadequate function of the left ventricle.
B. Hypertension is high blood pressure. Hypertension causes an increased work load on the heart
and can cause damage to the blood vessels and other organs, especially the kidneys and the eyes.
Hypertension is generally diagnosed when systolic pressure exceeds 140 and diastolic pressure
exceeds 90 mm Hg. Recent evidence indicates that death rate and illness are increased in patients
with diastolic pressures in excess of 85 mm Hg, which may prove to be a new borderline for the
diagnosis of hypertension.

1. Etiology
a. Essential hypertension accounts for more than 90% of all cases of hypertension. The cause
of essential hypertension is unknown. Excessive salt intake, resetting of the baroreceptors,
increased salt and fluid in the arteriolar walls, and long-standing exposure to stress are some
of the theories that have been proposed to explain the increased systemic arterial pressures.
b. Systolic hypertension with a wide pulse pressure can be caused by various malformations
or conditions that decrease the compliance of the aorta (aging) or increase the cardiac
output or stroke volume of the left ventricle. These latter conditions include thyrotoxicosis,
patent ductus, A-V fistula, fever, and aortic insufficiency.
c. Systolic and diastolic hypertension can be produced by a variety of causes.
(1) Renal disease, if severe, results in systemic hypertension due to the release of renin
and the formation of angiotensin II. Destruction of the kidney by a severe renal disease
or narrowing of the renal arteries can result in hypertension.
(2) Endocrine diseases such as Cushing's syndrome, primary hyperaldosteronism, pheochromocytoma, and acromegaly may result in hypertension.

(3) Miscellaneous causes of hypertension include psychogenic causes, coarctation of


the aorta (important to diagnose because it can be cured surgically), polyarteritis nodosa,
and excessive transfusion.
2. Symptoms of hypertension are generally absent until the disease is far progressed, and the
hypertension has resulted in cardiac, renal, neural, or ocular damage. The disease is usually
diagnosed as the result of a blood pressure taken during a routine physical examination.
a. Cardiac symptoms are caused by the increased work required of the left ventricle. The
ventricle initially hypertrophies and eventually, if the hypertension is severe, fails, resulting
in pulmonary congestion and edema. Hypertension also increases the rate of formation of
arteriosclerosis (hardening of the arteries), which reduces blood flow to critical vessels in
the heart. The reduced blood flow can cause angina or a heart attack.
b. Neural symptoms of hypertension include occipital headaches especially in the morning,
dizziness, ringing in the ears, dimmed vision, and syncope. Permanent brain damage can
result from complete blockage of a blood vessel by arteriosclerosis or by hemorrhage
secondary to the stress imposed on the vessels by the hypertension.
c. Renal effects of hypertension are due to atherosclerotic lesions that affect the afferent
and efferent arterioles and the glomerular tufts. These lesions cause a decrease in GFR as
well as renal tubular dysfunction. These pathologic alterations eventually result in renal
failure.
d. Malignant hypertension occurs when arterial pressure becomes markedly increased and
vascular, CNS, renal, and cardiac deterioration occurs over weeks or months instead of
years.
3. Treatment of hypertension is critically important to prevent or minimize damage to cerebral,
ocular, renal, and cardiac tissues. Treatment of essential hypertension relies primarily on diuretics, p-adrenergic blocking drugs, or calcium channel blockers to lower the arterial blood
pressure. Treatment of secondary hypertension also depends on treating the underlying disease.

C. Exercise
1. Importance. The cardiovascular and respiratory response to exercise has become of major
interest due to the current concern with physical conditioning and prevention of cardiovascular
disease. In addition, exercise is used as a stress test to evaluate the respiratory and cardiovascular
systems.
a. Specificity. Endurance (aerobic) training produces different effects than weight training, and
only aerobic exercise produces cardiovascular conditioning. Training effects are specific for
the particular muscle groups involved; for instance, rowing does not benefit running since
different muscles are involved.
b. Characteristics. Muscles conditioned by endurance training exhibit increases in capillary
density, myoglobin concentration, glycogen, and mitochondrial enzymes of the citric acid
cycle. These enzymes are involved in the breakdown of long-chain fatty acids, which serve
as an energy source during exercise thus sparing muscle glycogen.

2. Cardiac responses
a. Heart rate increases linearly with work rate up to a maximum, which is determined by a
person's age. The maximal heart rate (HRmaxl can be estimated from the following equation:
HRmax = 210- [0.65 x age (years)]. Cardiovascular conditioning requires a heart rate of
60%-70% HRmax for 20-30 minutes, three to four times a week for at least 3 months to
obtain optimum benefits.
b. Stroke volume. Endurance training causes an increase in the stroke volume of the heart by
increasing the volume of the ventricles. Thus, conditioned athletes can maintain any level
of cardiac output at a lower heart rate than nonconditioned individuals.
c. Maximal cardiac output in conditioned athletes is greater than in nonconditioned individuals because of the larger stroke volumes that are attained. Conditioning does not have any
effect on the maximal heart rate for an individual. The maximal cardiac output may be on
the order of 30-35 L/min in Olympic-class runners, while the maximal cardiac output in
deconditioned adults is about 15-20 L/min.
d. The anaerobic threshold is defined as the minimal level of exercise that produces a
significant increase in the lactate concentration of the blood and occurs at approximately

60% of the maximal exercise level. The anaerobic threshold represents the point when
muscle metabolism shifts from an aerobic to an anaerobic state. Training improves the
absolute anaerobic threshold as well as the anaerobic threshold relative to the maximal O 2
consumption.
3. Respiratory responses
a. Minute ventilation increases linearly with work rate until the anaerobic threshold is
reached, when minute ventilation increases more steeply with work rate due to the additional
respiratory drive imposed by the liberated lactic acid (Figure 2-29). Endurance training
increases the maximal minute ventilation achieved during exercise, but the maximal voluntary ventilation does not improve with training.
b. CO 2 output increases linearly with the work rate until anaerobic threshold, when the
CO 2 output increases more steeply due to the increased respiration. Above the anaerobic
threshold, the arterial Peo 2 declines as the body stores are depleted, since excretion exceeds
production.
c. O 2 consumption increases linearly with work rate and is exactly dependent on the work
performed. The O 2 consumption does not change with training at any work load (Le.,
training does not improve the body's efficiency unless muscle coordination is improved by
practice). Conditioning produces a 5%-20% increase in the maximal O 2 consumption,
which comes from an increase in the cardiac output and a widening of the arteriovenous
O 2 difference at the maximal exercise level.
d. The alveolar-to-arterial O 2 gradient normally remains at 5-10 mm Hg during moderate
levels of exercise, but, beyond the anaerobic threshold the alveolar-to-arterial O 2 gradient
widens slightly due to an increase in the alveolar Po 2
e. The respiratory exchange ratio (R = Veo 2 l'Vo 2 ) rises to greater than 1 at work levels
above the anaerobic threshold but normally never exceeds 1.25 even at maximal levels of
exercise. The increase in R is due to the conversion of bicarbonate to carbonic acid in the
buffering process of lactic acid and the liberation of the CO 2 in the lungs.

4. Pathologic responses (Figure 2-30)


a. Chronic obstructive lung disease limits exercise mainly due to the onset of severe dyspnea.
In these patients the heart rate does not reach the minimal level for training effects, and the
anaerobic threshold occurs at very low levels of exercise due to the impairment of O 2
uptake. Thus, exercise training in these patients has only slight benefits, which seem to be
due mainly to desensitization of symptoms. It appears to be more worthwhile to train the
respiratory muscles in these patients.

100

OJ

80 E

.S-

a:

.>rS 3

a
()

ctl

60 ';
c:

rS

to

()

"2

.>

40

2-

20

20

40

60

Work rate (% max)

80

100

Figure 2-29. The effects of exercise


on respiratory parameters of gas exchange. The dashed line indicates the
onset of the anaerobic threshold.
VE = minute ventilation; veo 2 = CO 2
excreted in expired gas (L/min);
V02 = O 2 utilization (L/min); Paeo2 =
arterial CO 2 tension; R = the respiratory exchange ratio.

100

V0 2 (Umin)

Figure 2-30. The effect of training, heart disease, and


lung disease on the heart rate response to endurance
(aerobic) exercise. Vertical arrows mark the onset of the
anaerobic threshold, which occurs at about 60% of the
maximal exercise level for every individual.
2 = O2
utilization.

v0

b. Cardiac patients have an early onset of the anaerobic threshold due to the low cardiac
output, and they reach maximal heart rates at much lower levels of exercise than normal
individuals.

STUDY QUESTIONS
Directions: Each of the numbered items or incomplete statements in this section is followed by answers
or by completions of the statement. Select the one lettered answer or completion that is best in each
case.
1. In a recumbent person, the greatest difference
in blood pressure would exist between the
(A) ascending aorta and brachial artery
(B) saphenous vein and right atrium

(C) femoral artery and femoral vein


(D) pulmonary artery and left atrium
(E) arteriolar and venous end of a capillary

2. Distribution of blood flow is regulated mainly

6. All of the following mechanisms will increase


the transcapillary flux of liquids, solids, and gases
EXCEPT
(A) an increase in capillary surface area
(B) an increase in the concentration gradient
across the microvascular wall
(C) an increase in the diffusion distance
(D) an increase in the capillary transit time
(E) a decrease in arteriolar resistance

by
(A)
(B)
(C)
(D)
(E)

capillaries
arterioles
venules
arteriovenous anastomoses
postcapillary venules

3. Which of the following mechanisms is most important for maintaining an increased blood flow to
skeletal muscle during exercise?
(A)
(B)
(C)
(D)

An increase in aortic pressure


An increase in a-adrenergic impulses
An increase in ~-adrenergic impulses
A vasoconstriction in the splanchnic and renal
areas
(E) A vasodilation secondary to the effect of local
metabolites
4. Which of the following factors is most important to blood flow regulation at the local level?
(A) Vessel-tissue transmural pressure difference
(B) Metabolic activity of the organ or tissue
(C) Local neurotransmitters
(D) Circulating neurotransmitters
(E) Cardiac output
5. The most important mechanism for blood-tissue
transport of small hydrophilic solutes is
(A)
(B)
(C)
(D)
( E)

convection
facilitated diffusion
active transport
passive diffusion
cotransport

7. The rate of lymph flow in humans is approximately


(A)
(B)
(C)
(D)
(E)

10-20 L/day
100-200 ml/day
10-20 ml/hr
1.5-2 L/day
1-2 ml/hr

8. Venous return is enhanced during exercise by


all of the following factors EXCEPT
(A)
(B)
(C)
(D)
( E)

increased depth of respiration


pumping action of skeletal muscles
venoconstriction
reduced arteriolar resistance
an erect position

9. All of the following conditions would cause a


decrease in cardiac output EXCEPT
(A) ventricular tachycardia
(B) electrical stimulation of the distal end of a cut
vagus nerve
(C) electrical stimulation of the superior cervical
ganglion
(D) external compression of the area over the carotid sinus
(E) the Valsalva maneuver

10. An EKG that reveals no P wave in any leads


indicates a block of impulses coming from the
(A)
(B)
(C)
(D)

SA node
bundle of His
Purkinje fibers
left bundle branch

(E) ventricular muscle

11. Ventricular end-diastolic pressure can be used


on the abscissa (x-axis) when plotting a FrankStarling curve instead of the usual variable, which
is
(A) cardiac output
(B) ventricular stroke volume
(C) cardiac work
(D) ventricular contractility
(E) ventricular end-diastolic volume
12. Increasing the preload of cardiac muscle will
(A)
(B)
(C)
(D)

reduce the ventricular end-diastolic pressure


reduce the peak tension of the muscle
decrease the initial velocity of shortening
decrease the time it takes the muscle to reach
peak tension
(E) increase the ventricular wall tension
13. Which of the following events is represented
on an EKG?
(A)
(B)
(C)
(D)
(E)

SA node depolarization
AV node depolarization
Bundle of His depolarization
Depolarization of Bachmann's bundle
Atrial muscle depolarization

14. An increase in systemic blood pressure leads


to which of the following effects?
(A) An increase in the velocity at which blood is
ejected from the left ventricle
(B) An increase in cardiac output
(C) An increase in the residual volume of blood in
the left ventricle
(D) A decrease in the time it takes for the left ventricular wall to develop peak tension
(E) A decrease in the maximal wall tension developed in the left ventricular muscle
15. The time from the upstroke of the carotid artery volume pulse to the dicrotic notch (incisura) is
a measure of the period of
(A)
(B)
(C)
(D)
(E)

atrial diastole
ventricular ejection
reduced ventricular filling
rapid ventricular filling
ventricular isovolumic relaxation

16. The pulmonic valve normally closes after the


aortic valve because the
(A) diameter of the pulmonary artery is less than
that of the aorta
(B) right ventricular contraction begins after left
ventricular contraction
(C) velocity of ejection in the right ventricle is less
than that in the left ventricle
(D) diastolic pressure in the pulmonary artery is
less than that in the aorta
(E) leaflets of the pulmonic valve are stiffer and
harder to close compared to those of the aortic
valve
17. During diastole, blood flow into the ventricles
sometimes produces
(A)
(B)
(C)
(D)
(E)

a first heart sound


a second heart sound
a third heart sound
an ejection click
an ejection-type murmur

18. A patient with coronary artery disease undergoes coronary arteriography, which reveals a 50%
decrease in the lumen of the left anterior descending coronary artery. For any arteriovenous pressure
gradient, the flow through this artery (compared to
normal) will decrease by a factor of
(A) 2

(B) 4
8

(C)

(D) 12
(E) 16
19. A decrease in heart rate (while stroke volume
and peripheral resistance remain constant) will
cause an increase in
(A)
(B)
(C)
(D)
(E)

arterial diastolic pressure


arterial systolic pressure
cardiac output
arterial pulse pressure
mean arterial pressure

20. The hexaxial reference system consists of reference lines generated from
(A) leads V 1-V 6
(B) the coronal body plane
(C) the horizontal body plane
(D) standard and augmented limb leads

(E) all bipolar limb leads

21. The increased circulating fluid volume in


chronic congestive heart failure results from all of
the following factors EXCEPT
(A) increased sympathetic discharge to the kidney
(B) decreased rate of firing of atrial volume receptors
(C) decreased renal perfusion
(D) aldosterone activity
(E) stimulation of the arterial baroreceptors

23. Stimulation ofthe high-pressure baroreceptors


is associated with
(A) an increase in cardiac contractility
(B) an increase in heart rate
(C) an increase in the discharge rate of vagal efferent cardiac neurons
(D) a decrease in systemic blood pressure
(E) stimulation of the vasopressor center

22. The greatest resting arteriovenous difference


in O 2 content is found in the
(A)
(B)
(C)
(D)
(E)

liver
skeletal muscle
heart
kidney
lung

Directions: Each group of items in this section consists of lettered options followed by a set of numbered
items. For each item, select the one lettered option that is most closely associated with it. Each lettered
option may be selected once, more than once, or not at all.
Questions 24-26

Questions 27-31

For each condition listed below, select the mechanism that is associated with it.

For each of the conditions listed below, select the


most appropriate response.

(A) Increase in contractility

(A)
(B)
(C)
(D)

(B) Increase in fiber length

(C) Both
(D) Neither
24. An increase in the maximal velocity of shortening (V max)
25. A shift to a new Frank-Starling curve
26. An increase in intracellular ionic calcium

27.
28.
29.
30.
31.

Increased ventricular end-diastolic pressure


Increased aortic diastolic pressure
Both
Neither
Myocardial tissue damage
Increased arteriolar resistance
Decreased ventricular contractility
Increased heart rate
Increased central venous pressure

Questions 32-35

Questions 36-40

Match the cardiac event with the interval of the


cardiac cycle in which it occurs.

Match each cardiovascular effect with the factor


that is responsible for it.

(A)
(B)
(C)
(D)
(E)

Atrial contraction
Isovolumic contraction
Rapid ventricular ejection
Reduced ventricular ejection
Isovolumic relaxation

(A)
(B)
(C)
(D)
(E)

32.
33.
34.
35.

Second heart sound


Maximal ventricular volume
Closure of the AV valves
Aortic valve opening

36.
37.
38.
39.
40.

Functional hyperemia
Histamine
Hypertension
Peo 2
Capillary pressure
Regulation of capillary filtration rate
Elevation of arterial diastolic pressure
Metabolic regulation of blood flow
Regulation of cerebral blood flow
Increase in microvascular permeability

ANSWERS AND EXPLANATIONS


1. The answer is C. [I C; Figure 2-2] The largest pressure drop in the vascular system (40-50
mm Hg) occurs as blood passes through the systemic arterioles; this segment of the vasculature would
be represented by the region between the femoral artery and femoral vein. The large arteries provide
little resistance to flow, so that there is practically no pressure loss from the aorta to any of the distributing
arteries. The pressure loss across the venous system amounts to approximately 10 mm Hg, since the
pressure in the large veins may be 8-10 mm Hg whereas the pressure in the right atrium may be near
zero. The atrium is distended since the pressure outside the atrium (intrathoracic pressure) averages
about -5 mm Hg, which means that the pressure across the wall of the atrium (transmural pressure) is
5 mm Hg. The pulmonary circulation is a low-resistance circuit, and the gradient across this vascular
segment also is only about 8-10 mm Hg. The pressure at the arteriolar end of a capillary averages about
40 mm Hg, whereas the pressure at the venous end is 25-30 mm Hg. Thus, even the smallest vessels
in the body, which should have the highest resistance, exhibit only a 10-15 mm Hg pressure gradient.
This seeming paradox occurs because of the tremendous number of capillaries that exist, which are
arranged in a parallel circuit. Remember that the total resistance to flow in a parallel circuit is the sum
of the reciprocals of the resistance in each vessel, so that the greater the number of vessels, the lower
the total resistance.

2. The answer is B. [I C 2 a] The arterioles often are referred to as the stopcocks of the circulation,
since they act as valves to restrict or enhance blood flow to different tissues or organs. Small changes in
the diameter of these vessels can markedly alter the resistance to flow since resistance varies as the
fourth power of the radius in the arterioles, as in any other vessels. However, the arterioles have a thick
muscular wall in comparison to the lumen, so that a relatively small amount of smooth muscle contraction
can cause a marked change in lumen size.
3. The answer is E. [/ C 2 a] Vasodilation in active skeletal muscles reduces the resistance to flow and
results in an increased blood flow to the involved tissues. The increase in blood flow is dependent on
maintaining the mean arterial blood pressure, which normally rises only slightly with exercise. Thus, the
slight rise in arterial pressure coupled with a significant rise in blood flow indicates that the total vascular
resistance must decline during exercise. The decrease in vascular resistance occurs because skeletal
muscle represents about 50% of body weight in the normal adult, and a decrease in this tissue can more
than compensate for an increased resistance in such tissues as the gastrointestinal tract, kidneys, and
skin during periods of mild to moderate exercise.
4. The answer is 8. [/ C 2 a (1 J (aJ] Normally, the body adjusts blood flow by the release of vasodilator
substances from the active tissue or organ. This eliminates the need for the body to supply receptors and
afferent and efferent nerves in order to detect changes in metabolism in virtually every cell in the body.
Thus, most tissues autoregulate their own blood flow dependent on their own metabolic activity. An
increase in activity increases the release of vasodilators such as adenosine and potassium. The increased
metabolism results in a reduced Po 2 , which also produces vasodilation in the systemic circulation.
5. The answer is D. [/04 a] Small, hydrophilic molecules (e.g., O 2 , CO 2 , glucose, water) are thought
to pass readily through the pores in the endothelial cells of capillaries and are not impeded significantly
by capillary endothelium. The capillary pores are such that virtually all of the albumin (molecular
weight = 64,000 daltons) is retained.
6. The answer is C. [I 0 4 a (1 J] Diffusion rate is inversely related to the distance over which it must
occur, so that, while diffusion is a very effective means of moving substances over short distances, it is
a very inefficient and slow process over long distances. A decreased arteriolar diameter enhances the
flux of materials across capillaries, since it raises capillary pressure toward the arterial pressure, increasing
the filtration forces and leading to increased bulk flow or convection of fluid.
7. The answer is D. [/ E 3] The rate of lymph flow is about 1-2 L/day. Although this rate seems large,
it is quite small compared with the flow through the entire vascular system. The entire blood flow per
day is 5-6 L/min x 60 min/hr x 24 hr/day = 8000 L. Thus, approximately 99.98% of the blood flow
returns to the heart via the veins, whereas 0.02% of the blood flow is returned to the heart via the
lymphatics.

8. The answer is E. [I F 2 d] Standing erect exposes a column of blood to the effects of gravity, which
raises the pressure in the dependent veins. Since the venous system is very compliant, the increase in
pressure causes an increase in venous volume, which reduces venous return compared to the prone or
horizontal position. Venoconstriction, which is caused by sympathetic outflow during exercise, will
partially compensate for the peripheral pooling caused by the effects of gravity. An increased depth of
respiration causes a more negative pressure in the throrax, which increases the pressure gradient along
the venous system and improves venous return. Contraction of the skeletal muscles compresses the veins
and displaces the blood toward the heart; this effect is enhanced if the venous valves are competent
and prevent a backflow of blood between contractions. A reduced arteriolar resistance in the skeletal
muscles raises capillary and venous pressures and increases the flow rate through the systemic circulation.
9. The answer is C. [I F 2 c; II C 1 c; IV B 4 b] Stimulation of the superior cervical ganglion increases
the sympathetic outflow to the heart. This leads to increased heart rate and contractility, which raises
cardiac output. Ventricular tachycardia reduces cardiac output, since it interferes with diastolic filling of
the ventricles and markedly reduces stroke volume. Stimulation of the distal vagus nerve causes bradycardia or even cardiac arrest for a period of time, with an obvious reduction in cardiac output. Compression
over the carotid sinus results in a stimulation of the baroreceptors and a reflex increase in vagal outflow
to the heart, causing cardiac slowing and a decreased cardiac output. Cardiac output is reduced during
a Valsalva maneuver since the intrathoracic pressure is increased, which impairs venous return. Since
the heart can pump only the blood it receives, there is a decline in cardiac output during the Valsalva
maneuver. Immediately following the Valsalva maneuver, however, there is a transient increase in cardiac
output as the dammed up blood returns to the heart.

10. The answer is A. [II C 1, 2; 11/ C 1 a] The sinoatrial (SA) node is the normal pacemaker of the heart,
and its depolarization cannot be seen on the EKG because of the small mass of tissue whose depolarization
cannot be seen on surface recordings. SA node depolarization normally spreads to the atrial muscle,
causing depolarization, which is seen on the EKG as the P wave. The bundle of His, Purkinje fibers, left
bundle branch, and ventricular muscle all lie below the atria and normally do not cause depolarization
of the atria.
11. The answer is E. [II E] The x-axis usually represents the independent variable, so that, for the
Frank-Starling mechanism, this would be the factor that defines the length of the sarcomere. In the intact
heart, the sarcomere varies in length as a function of ventricular end-diastolic volume. Assuming that
there is no change in compliance of the ventricular muscle, the sarcomere will also vary as a function
of end-diastolic pressure. The factor plotted on the y-axis of such a graph represents the output of the
heart, which can include such functions as cardiac output, stroke volume, and cardiac work.

12. The answer is E. [II E 1] An increase in both end-diastolic pressure and volume is synonymous with
an increased preload. The increased preload causes a more forceful ventricular contraction, which results
in an increase in the peak pressure generated by the ventricle as well as an increase in the stroke volume.
This intrinsic property of the myocardium is termed the Frank-Starling mechanism. The increased preload
results in an increased velocity of shortening, but, since the peak pressure increases, the time to peak
pressure usually remains constant in these conditions.
13. The answer is E. [11/ C 1 a] Atrial muscle depolarization is seen as the P wave on a standard EKG.
An EKG does not reveal depolarization of the SA node, atrioventricular (AV) node, bundle of His, or
Bachmann's bundle, because the mass of the tissue involved is too small to cause a deflection on the
surface electrograms that represent the EKG. Depolarization of the bundle of His can be recorded
from electrode catheters located in the esophagus. Esophageal catheterization is frequently done to
characterize supraventricular arrhythmias, which cannot be diagnosed using the standard EKG.

14. The answer is C. [V B 2] An increase in the systemic pressure, or afterload, that the ventricle must
overcome reduces the velocity of shortening of the contractile elements in the muscle. An increase in
afterload also causes a reduction in the extent of muscle shortening, which results in a decreased stroke
volume and, consequently, an increased residual volume (i.e., the volume of blood left in the ventricles
at the end of ejection). The time that it takes for the left ventricular wall to develop peak tension is not
reduced, because the velocity of contraction would be reduced, and the peak tension would be increased.
Wall tension is a function of the product of ventricular pressure and volume. Since pressure and volume
are increased, wall tension also is increased in the presence of an increased afterload.

15. The answer is B. [VI A 3 c] The upstroke of the carotid artery pulse indicates the onset of ventricular
ejection, while the dicrotic notch (caused by semilunar valve closure) indicates the end of ejection. Atrial
diastole represents the entire cardiac cycle except for the period of atrial systole that is concomitant with
the a wave in the atrial pressure pulse. The periods of ventricular filling represent diastolic events that
can be determined from an atrial pressure pulse or the jugular volume pulse. The period of isovolumic
relaxation occurs from the incisura, or 52' to the peak of the atrial v wave, which represents the time
when the AV valves open.

16. The answer is C. [VI A 3 c, e] Right ventricular ejection begins before that of the left ventricle and
continues after left ventricular contraction ends. The normal sequence of semilunar valve closure, which
represents the major components of the 52' is closure of the aortic valve (A 2) followed by closure of the
pulmonic valve (P 2). During inspiration, P2 is delayed and A2 occurs slightly earlier, which widens the
interval between these events. This sequence can be appreciated during auscultation of the heart and
is referred to as normal splitting of the 52' If there is a delay in left ventricular activation, as in left bundle
branch block, A2 may follow P2 and, during inspiration, there is a narrowing of the interval between
these events. This is termed paradoxical splitting of the 52'
17. The answer is C. [VI A 4 d] The first and second heart sounds (5, and 52) indicate the beginning
and end of systole, respectively. An ejection click and an ejection-type murmur are systolic sounds that
are caused by blood leaving the ventricle. The third heart sound (53) occurs during mid-diastole when
the ventricular wall becomes tense toward the end of the rapid filling phase. 53 is not normally heard in
adults but can be heard in children with thin chest walls.

18. The answer is E. [VII B 1 a] The effect of vessel radius on flow is so powerful because the flow rate
is directly proportional to the fourth power of the radius (Poiseuille's law). Thus, decreasing the radius
to half reduces the flow to one-sixteenth of the normal value.

19. The answer is O. [VII C 2] A constant stroke volume coupled with a decreased heart rate produces
a decrease in cardiac output. The reduced flow in combination with a constant peripheral resistance
means that mean arterial pressure must decline. The fact that pulse pressure increases is obtained from
an analysis based on the vascular elastic modulus (E = V x dP IdV: where E is the elastic modulus; V
is arterial volume, which is determined by arterial pressure; dP is pulse pressure; and dV is arterial
uptake). Arterial pressure and volume decrease, since cardiac output is reduced (heart rate decreased
and stroke volume constant). The quantity dV remains constant, since it is primarily related to stroke
volume, which is constant. Thus, dP must increase in order to cancel the decreased V in order to maintain
E as a constant (E varies only as a function of age).

20. The answer is O. [/II A 5; Figure 2-13] The hexaxial reference system indicates six lines, which
would include the three bipolar limb leads and the three augmented limb leads. The three bipolar limb
leads are separated by 60 degrees and, thus, lie at 0, 60, 120, 180, and 240 degrees. The augmented
limb leads bisect the angles provided by the bipolar leads, which means that the hexaxial reference
system provides reference lines at every 30 degrees. The hexaxial reference system is in the frontal
plane, whereas the precordial (V) leads lie in the horizontal plane.
21. The answer is E. [V/II C 2; IX A] The arterial baroreceptors are stimulated by an increase in arterial
pressure, not a decrease, as is likely in cardiac failure. The increased fluid volume in congestive heart
failure occurs due to an increase in the release of renin from the juxtaglomerular cells of the kidney,
which is due to a generalized increase in sympathetic discharge and to the reduced perfusion of the
kidney. Renin, in turn, stimulates the release of aldosterone by the adrenal glands. There is evidence that
accommodation of the atrial volume receptors occurs when these receptors are exposed to prolonged
periods of vascular distension, causing decreased firing rate.
22. The answer is C. [X A 2] Under resting conditions, the heart removes approximately half of the O 2
from the blood, which results in a P0 2 of approximately 27 mm Hg. In contrast, blood flow through
other tissues is much greater in proportion to the metabolic rate and results in an arteriovenous (A-V)
O 2 difference of about 5 mlldl blood. The mixed venous O 2 content, which represents the weighted
average of blood flow to all parts of the body, is 15 ml 02/dl blood, which represents a hemoglobin
saturation of 75% and a P0 2 of 40 mm Hg.

23. The answer is C. [IX B 2 c] The high-pressure baroreceptors are stimulated by an increased arterial
blood pressure. The afferent impulses are carried over the glossopharyngeal and vagus nerves, where
they stimulate the cardioinhibitory and depressor center. The one portion of the efferent limb of the
reflex causes an increased number of vagal impulses to the heart, which results in cardiac slowing leading
to a decrease in cardiac output. This depressor response causes a decrease of sympathetic tone to the
vascular system, resulting in decreased peripheral resistance and a pooling of blood in the venous system.
Both of these peripheral effects also lead to a lowering of the arterial blood pressure.
24-26. The answers are: 24-A, 25-A, 26-A. [II 02 a, F 1] An increased maximal velocity of shortening
(V rna.) is one definition of increased contractility, or positive inotropism. Another way of looking at
increased contractility is an increase in stroke volume at a constant end-diastolic pressure or volume.
The presence of a new Frank-Starling curve implies a change in contractility. If the new curve is above
and to the left of the control curve, this indicates an increased contractility. Changes along anyone curve
indicate a change in ventricular fiber length (changing ventricular volume or pressure), which is the
mechanism for invoking the Frank-Starling mechanism of altering ventricular output, whether this is
cardiac work or stroke volume.
Positive inotropism is associated with an increase in the intracellular concentration of calcium. The
increased calcium allows a greater number of troponin molecules to become saturated, which then
provides more cross-bridges to be formed during the contraction process. The increased number of
cross-bridges increases the force of contraction and the velocity of contractile element shortening at any
load compared to the control condition.

27-31. The answers are: 27-A, 28-C, 29-A, 30-8, 31-C. [II E, F 2; VB 1 b; VII C 2 a (1), (2)] Myocardial
tissue damage results in a decline in ventricular force development, leading to a reduction in stroke
volume. Transiently, the volume of blood pumped by the ventricle is less than the venous return, so that
ventricular diastolic pressure and volume increase. The increase in ventricular volume represents an
increase in the preload so that the remaining muscle generates a greater tension than before, as long as
the ventricle remains on the ascending limb of the Frank-Starling curve. This mechanism is limited since
the ventricle may exceed the peak of the Frank-Starling curve, at which point there is a decline in output
as the preload increases further.
Increased arteriolar resistance results in a rise in arterial (aortic) pressure. If heart rate and stroke
volume remain constant, then the pulse pressure must decline in accordance with the principles determining arterial pressure. The increased aortic diastolic pressure represents an increase in the ventricular
afterload so that stroke volume will be reduced slightly, further decreasing the pulse pressure, as detailed
in Figure 2-19.
Decreased ventricular contractility results in a reduction in cardiac output as the ventricle shifts to a
lower Frank-Starling curve. As cardiac output decreases, there is a rise in the venous pressure so that
ventricular diastolic pressure and volume increase due to an added inflow into the ventricle. As long as
the ventricle remains on the ascending limb of the Starling curve, this will result in some restoration of
the cardiac output. However, the ventricular radius is larger so that the myocardium must generate a
higher wall tension in accordance with the Laplace equation. The one benefit of ventricular dilation is
that the myocardium can produce a given stroke volume with a smaller amount of myocardial shortening
because of the increased radius.
An increased heart rate results in an increased cardiac output and a decline in ventricular end-diastolic
pressure, since both ventricular filling time and central venous pressure are reduced. An increase in
cardiac output as the primary event decreases central venous pressure. This becomes more obvious if
one thinks about the heart pumping blood from the venous to the arterial side of the circulation. Thus,
with an increased cardiac output, an additional volume of blood is transferred from the venous to the
arterial system, which raises the pressure in the arteries and lowers the pressure in the veins.
An increase in central venous pressure raises the gradient for ventricular filling, which increases the
ventricular end-diastolic pressure and volume. This represents a Frank-Starling mechanism that increases
stroke volume and consequently cardiac output. As a result of the increased cardiac output, there will
be an increase in aortic diastolic and systolic pressures in accordance with the relationship between
pressure, flow, and resistance: [tldP = r Q x R.
32-35. The answers are: 32-E, 33-A, 34-8, 35-C. [VI A 2, 3 b, c, e] 52 is caused by closure of the
semilunar valves and the consequent vibrations in the tissues caused by stopping the backward flow of
blood in the aorta. Thus, the 52 represents the beginning of isovolumic relaxation.

Ventricular filling begins as soon as the AV valves open in diastole and depends on the rate of venous
return. Contraction of the atria, late in diastole, causes an additional increment in ventricular volume so
that ventricular volume reaches a maximum. The increased filling caused by atrial contraction is particularly important at fast heart rates, since the time for ventricular filling is abbreviated due to the shortened
diastolic interval.
Closure of the AV valves causes the left ventricular chamber to become isolated, since the aortic
valves also are closed due to the diastolic aortic pressure. Since both inflow and outflow valves are
closed, there is no change in ventricular volume for a time; hence the interval is called isovolumic (equal
or constant volume).
Opening of the aortic valve occurs when the pressure in the left ventricle exceeds the pressure in the
aorta, and blood begins to flow out of the ventricle. Since the rate of ventricular contraction is initially
high, the blood rapidly leaves the ventricle during this interval, causing aortic pressure to increase rapidly
in association with the pressure in the ventricle.

36-40. The answers are: 36-E, 37-C, 38-A, 39-D, 40-8. [I C 2 a, 04 b (1) (a); VI A 5 d; X B 2; XI B
1] Of the factors listed, capillary filtration rate is most closely associated with capillary pressure. Capillary
pressure and interstitial fluid oncotic pressure are direct determinants of capillary filtration rate, according
to the Starling hypothesis. Capillary pressure is counteracted by the oncotic pressure of the blood and
tissue pressure.
An elevated arterial diastolic pressure is a more important determinant of the mean arterial pressure
than the systolic pressure. The arterial systolic pressure is largely determined by the stroke volume,
whereas the arterial diastolic pressure reflects the resistance in the systemic circulation. The high arterial
diastolic pressure leads to vascular changes in certain organs, especially the kidneys and eyes, which
are characteristically affected by arterial hypertension.
During exercise, there is an increased blood flow to skeletal muscles, which can be defined as a
functional hyperemia, or increased blood flow. This increased flow is due to a local decrease in vascular
resistance brought about by an increase in tissue temperature, osmolality, PK and Peo 2 , and a decrease
in the local Po 2 None of these factors alone seems to be able to duplicate the effects of exercise.
Peo 2 is the most important factor that regulates the resistance in the cerebral vessels. Hyperventilation,
which results in a reduction of Peo 2 , produces a marked vasoconstriction of cerebral vessels that, at
times, can lead to hypoxia sufficient to cause dizziness and even fainting.
Histamine, released from mast cells in areas of inflammation, results in an increased permeability of
microvessels, primarily of the postcapillary venules.

3
Respiratory Physiology
Joseph Boyle, III

I. INTRODUCTION. The primary function of the respiratory system is to exchange oxygen (0 2 )


and carbon dioxide (C0 2 ) between the body and the environment. Secondary roles of the respiratory
system include aiding in acid-base balance, defending the body against inhaled particles such as
bacteria and pollens, acting as a circulatory filter to prevent clots from entering the systemic
circulation, and regulating several hormonal and humoral concentrations.
A. External respiration is concerned with the exchange of gas between the environment and the
lungs, the transfer of gas across the respiratory membrane, and the transport of the gas by the
blood to and from the body cells. This chapter is concerned primarily with the process of external
respiration.

B. Internal respiration is concerned with the oxygen consumption by the body cells through
metabolic transformations and is now considered to be the province of biochemistry.

II. RESPIRATORY MECHANICS refers to the study of factors involved in altering lung volume.
These factors include the muscular forces needed to expand the respiratory system, the forces that
impede expansion (resistance and elastance), and the determinants of lung volume.

A. Inspiration refers to an increase in lung volume requiring that a force be applied to the
respiratory system. The active force usually is generated by the inspiratory muscles, but this force
also can be supplied by a mechanical respirator if the muscles are weakened by a disease such as
polio or muscular dystrophy. The lungs are passive structures that follow any movements of the
chest wall. The small amount of fluid normally found in the pleural space causes the visceral and
parietal pleura to adhere to one another so that any movements of the chest wall cause similar
changes in lung volume. Normally, no gas exists in the pleural space, but gas can enter the pleural
space from a rupture of the lung or a penetrating wound of the chest wall. The presence of gas in
the pleural space is termed a pneumothorax.

1. Inspiratory muscles include the respiratory diaphragm, the external intercostals, and the
accessory muscles of inspiration. The accessory muscles include the sternomastoid and other
strap muscles of the neck.
a. The diaphragm, the major muscle of inspiration, is a dome-shaped sheet of muscle separating the thoracic and abdominal cavities. The diaphragm acts similarly to a piston or a syringe,
since contraction of the muscle causes the dome of the diaphragm to descend. Descent of the
diaphragm enlarges the volume of the thoracic cavity from below (Figure 3-1 ). Contraction of
the diaphragm also lifts the lower ribs since the diaphragm originates from these structures.
Elevation of the ribs causes thoracic expansion laterally and anteriorly because the ribs are
angled downward (see Figure 3-1). This is referred to as a bucket-handle effect.
b. The external intercostal muscle fibers originate from an upper rib and insert on a lower
rib more anteriorly. Contraction of these muscles causes an elevation of the ribs and enhances the bucket-handle effect.
c. The accessory muscles of respiration originate from the neck and skull and lift the clavicles
and the sternum. These muscles help to elevate the ribs and enlarge the thorax.

Accessory muscles

Internal

intercostals----,H:~....

I'r--t-\-External intercostals

Tendon of d iaphragm --1'--\*rj~~;?f~"'",Cs....-::::1hT- Spi ne


..JItf-~r--1'--Diaphragm

Figure 3-1. (A) The thorax and respiratory muscle relationships are shown during relaxation. (8) Maximal inspiration.
Note the flattening of the diaphragm and the more horizontal position of the ribs denoting the increased volume of
the thorax.

2. Inspiratory force is generated by contraction of the inspiratory muscles. This inspiratory force
expands the volume of the respiratory system. Boyle's law states that the product of pressure
times volume is a constant (P x V = C). Thus, if the volume of a container is increased, the
pressure will decrease so that the product of pressure and volume remains constant. Contraction
of the inspiratory muscles expands the gas in the respiratory system, causing the pressure in
the lungs to decrease below atmospheric pressure. If the airways are open, the gas can flow
from an area of higher pressure (atmosphere) to lower pressure (alveoli) to renew the O 2
concentration in the alveoli.
a. Maximal inspiratory force normally is -80 to -100 cm H 2 0 (80-100 cm H 2 0 pressure
below atmospheric pressure). In a procedure called a Muller maneuver, one inspires as
forcefully as possible, while holding the nose and closing the mouth. Note that the chest
does not expand much but that negative pressure causes a suction effect felt in the ears.
b. Normal inspiration requires pressures of only -3 to -5 cm H 2 0 to cause an adequate gas
flow into the respiratory system. Thus, a tremendous reserve of force is available, which
can be used to increase respiration during exercise or stress or if the forces that impede
inspiration are increased by disease or injury.
c. Decreased inspiratory force can be caused by many neurologic and muscular diseases
such as muscular dystrophy and poliomyelitis, both of which impair the ability of the muscle
to contract. Patients with these diseases may need respirators to maintain an adequate
volume of inspiration.

B. Expiration refers to a decrease in lung volume, which is normally a passive process. During
inspiration, the respiratory system is inflated above its equilibrium volume (see II C 4 a), just like
blowing up a balloon. During relaxation (releasing the neck of the balloon), the elastic forces
generated by the inflation compress the gas in the respiratory system, which raises its pressure (in
accordance with Boyle's law), and the gas flows out until the system has reached its equilibrium
volume. Expiratory muscle force may be needed when respiration must be increased during
exercise or when disease has increased the resistance and elastance forces.

1. Expiratory muscles include the muscles of the abdominal wall, especially the rectus abdominus, and the internal intercostal muscles.
a. Contraction of the rectus abdominus muscle decreases lung volume by pulling down on
the lower ribs; contraction of the abdominal wall muscles compresses the abdominal contents and forces the diaphragm upward.
b. Contraction of the internal intercostal muscles lowers the ribs, since these muscles
originate from the lower ribs posteriorly and insert on upper ribs more anteriorly.

2. Expiratory muscle force compresses the gas in the respiratory system, and the gas pressure
increases according to Boyle's law. Maximal contraction of the expiratory muscles can generate
forces of 100-150 cm H 2 0 above atmospheric pressure. Maximal expiratory force can be
generated from lung volumes near full inspiration when the nose and mouth are blocked.
Forced expiration against closed airways is termed a Val salva maneuver and is commonly
performed when lifting heavy objects or when moving one's bowels.
a. Maximal expiratory pressure can be measured with a mercury column or a blood pressure
gauge (aneroid gauge). One takes in a maximum inspiration, holds the nose with one hand,
and blows as forcefully as possible into the tubing to the gauge. The maximal pressure that
can be sustained for 5 seconds is read. Since these gauges are calibrated in mm Hg, the
gauge reading must be multiplied by 1.36 to convert it to cm H 2 0 pressure.
b. Normal expiration is passive, since the elastic forces stored during inspiration are used to
compress the alveolar gases. Thus, for expiration to occur, the inspiratory muscles gradually
relax to provide a slow and controlled expiration.
c. Decreased expiratory force also occurs in many neuromuscular diseases; however, since
expiration normally is passive, a weakness of the expiratory muscles is not so critical as a
weakness of the inspiratory muscles.

C. Events of the respiratory cycle. The two phases of respiration, inspiration and expiration, can
be measured by a spirometer, which is a simple device that can measure and record changes in
lung volume. Most spirometers use a water seal to collect the gas (Figure 3-2), which means that
the gas is at room temperature and is saturated with water vapor. The gas in the spirometer
occupies a different volume than it did in the body because of the change in the temperature and
the amount of water vapor present. The volume that the gas occupied in the body can be calculated
by using the general gas law, which is a combination of Boyle's and Charles' laws as shown in
Table 3-1.

Pulley

Pen

Paper supply

Water

Patient --::_=,t::.=.=~=:::~:::=------l----!="'<::--+--------~

.--;---+- - - ~

Motor
Figure 3-2. A water-sealed recording spirometer, which is used to measure changes in lung volume.

Table 3-1. The General Gas Law


The general gas law is stated as

P, x V,

T,
To convert gas volumes from spirometer to body conditions, the general gas
law is used in the following form

where V, = gas volume in the body; V 2 = gas volume in the spirometer; P, =


barometric pressure; P2 = standard pressure (760 mm Hg); T, = body temperature (310 K); T 2 = room temperature (0 K); PHP = water vapor pressure at
room temperature; and 47 = water vapor pressure at body temperature.
To identify the conditions for different lung volumes, the following definitions
are used:
atmospheric temperature, pressure, saturated (conditions in a
ATPS
spirometer)
body temperature, pressure, saturated (gas volumes in the body)
BTPS
standard temperature (00 C), pressure (760 mm Hg), dry (used
STPD
to express O 2 and CO 2 volumes for metabolic equivalence)

1. Alveolar pressure during inspiration is negative compared to atmospheric pressure, as shown


in Figure 3-3. This negative pressure in the alveoli creates the pressure difference between the
atmosphere and the alveoli, which causes the gas to flow into the lungs. At the end of inspiration,
the alveolar pressure becomes equal to atmospheric pressure, and the gas flow into the lungs
ceases.
2. Alveolar pressure during expiration is greater than atmospheric pressure, because the elastic
forces of the respiratory system and the contraction of the expiratory muscles compress the
gas and raise the pressure. Thus, the higher pressure in the alveoli causes gas to flow out of the
lungs, and lung volume returns to its initial level so that the next respiratory cycle can begin.
3. Rate of gas flow into or out of the lungs depends primarily on the pressure gradient between
the alveoli and the atmosphere. The highest gas flow rates occur when the pressure difference
between the alveoli and the atmosphere is greatest. Note, in Figure 3-3, that when the alveolar
pressure becomes zero (i.e., equals atmospheric pressure), the gas flow also is zero. Since the

0.5
Tidal volume
(L)

Flow
(Usee)
Alveolar
pressure
(em H 2 O)

low

0
-5

Interpleural
pressure
(em H 2 O)

-10

0
Time (sec)

Figure 3-3. Events of a respiratory cycle. Resistance forces on the alveolar pressure curve indicate only airway
resistance, while resistance forces on the pleural pressure curve include both airway and tissue resistance forces.
Note that, at the peak of inspiration, flow is zero, resistance forces are zero, and the change in the interpleural pressure
curve is due only to elastance forces.

pressure difference changes its direction between inspiration and expiration, the gas flow also
reverses.
4. Pressure in the interpleural space normally is below atmospheric pressure (negative).
a. Negative interpleural pressure. When the respiratory muscles are completely relaxed, the
lungs contain 2-2.5 L of gas. The lungs are distended to this degree because the chest wall
exerts a force tending to inflate the lungs. In turn, the elastic forces of the lungs try to
compress the gas in the lungs and cause them to deflate. These opposing forces are equal
but in opposite directions at this lung volume, which generates a negative pressure in the
interpleural space. During complete relaxation of the respiratory muscles, the interpleural
pressure is about -5 cm H 2 0 (Figure 3-4B). Because of the balanced forces under these
conditions of relaxation, the volume of gas in the lungs is termed the equilibrium volume.
(1) The equilibrium volume of the respiratory system represents the lung volume at the
end of a normal, relaxed expiration. The equilibrium volume is more commonly called
the functional residual capacity (see II H 2 c).
(2) Pleural fluid. The lungs and the chest wall are coupled by the small amount of fluid in
the interpleural space. This fluid acts as a lubricant so that the visceral pleura, which
covers the lungs, slides easily over the parietal pleura, which lines the inside of the
thoracic cavity. This system works similarly to two moistened pieces of glass. The glass
pieces slide easily back and forth, but it is difficult to separate them because of the
adhesion provided by the fluid.
b. During inspiration the interpleural pressure becomes progressively more negative (see
Figure 3-3), because the inspiratory muscles must stretch the lungs to take up the additional
volume of gas inspired with each breath. Therefore, at the end of inspiration the lungs recoil
away from the rib cage with more force. Also, the negative alveolar pressure is transmitted
through the lungs into the interpleural space. The change in pleural pressure, from the
beginning to the end of inspiration, measures the elastance forces for that breath. These
forces can be used to measure the dynamic compliance of the lungs.
c. During expiration the interpleural pressure returns to its resting level. During normal breathing, the interpleural pressure remains negative (i.e., below atmospheric pressure). However,
with forced expirations, the interpleural pressure can become positive, which compresses
the airways and slows the rate of expiration .

A. Residual volume

B. Functional residual capacity

C. Total lung capacity

Figure 3-4. Lung and chest wall forces at various lung volumes. The arrow outside the body indicates chest wall
forces, and the arrow inside the body indicates the retractive force of the lungs. (A) At small lung volumes, the lung
retractive forces are minimal, and the chest wall expansion forces predominate. The combination of the two forces
results in a large, negative interpleural pressure. (8) At the equilibrium volume of the respiratory system, the lung
retraction and chest wall forces are equal but in opposite directions. (C) At large lung volumes, both chest wall and
lung forces are directed inward, resulting in a positive interpleural pressure. PPL = interpleural pressure.

D. Pressure-volume (P-V) relationships of the respiratory system depend on the passive (completely
relaxed respiratory muscles) elastic properties of the chest wall and the lungs. These elastic
properties are defined by the relationship between the gas volume in the lungs and the transmural
pressure for the lungs and the chest wall under static conditions. Static conditions imply that
volume is constant, and no gas is flowing; therefore, no resistance forces are present. Figure 3-SA
indicates the anatomic locations that determine the transmural pressures for the lungs, chest wall
(thorax), and the respiratory system.
1. The transmural pressure (PTM), or the pressure across the wall, is the difference in pressure
between the inside (PI) and the outside (Po) of any structure: PTM = PI - Po. Pl refers
specifically to the transmural pressure across the lung. The equilibrium volume of a structure
is defined as the volume it contains when the PTM is zero, that is, when PI = Po. A positive
PTM (PI > Po) is a distending force that enlarges a structure above its equilibrium volume,
while a negative PTM is a force that tends to decrease the volume of a structure.
2. Transpulmonary pressure (PL) equals the pressure inside the lungs (PA) minus the pressure
just outside the lungs, which is the pressure in the interpleural space (PPL): Pl = PA - PPL. The
pressure in the interpleural space is estimated in humans by measuring the esophageal pressure
with a balloon-tipped catheter.
a. The lung is a passive structure whose volume is determined by its transmural pressure (PL).
Lung volume increases curvilinearly as PL increases until a limiting volume is approached
at a Pl of about 30 cm H 2 0 when the compliance of the lung approaches zero. This limiting
volume is thought to be determined by the complex arrangement of collagen fibers in the
interstitium of the lung.
b. The equilibrium volume of the lung normally is less than 10% of maximal lung volume.
At negative Pl, the airways close off so that a small volume of gas is trapped in the lungsthis is termed the minimal volume.
c. Compliance (distensibility) is the change in volume of a structure for each unit change in
pressure. The compliance is the slope (or steepness) of the P-V curve in Figure 3-5B, C,
and D.
(1) Normal compliance of the human lungs is about 0.2 L/cm H 2 0. This means that, for
each cm H 2 0 increase in transmural pressure, the volume increases 0.2 L of gas. The
amount of gas inspired with each breath is about 500 ml or 0.5 L. In order to "stretch"
the lungs to take up the normal tidal volume, an increase of about 2.5 cm H 2 0 in the
transmural pressure across the lungs is required. The change in the transmural pressure
normally is generated by the inspiratory muscles.
(2) Decreased compliance of the lungs is caused by certain lung diseases (e.g., tuberculosis, abscess, respiratory distress syndrome) that produce scarring or fibrosis of the lungs
or destroy functional lung tissue. This condition is termed restrictive lung disease.
Patients with restrictive lung disease must generate forces much greater than normal to
expand the lungs.
(3) Increased compliance occurs in emphysema, which destroys some of the alveolar
septa. This leads to a marked increase in airspace size and a reduction in the elastance
forces of the lungs.
3. Transthoracic pressure (Pew) represents the transmural pressure across the chest wall; thus,
Pew = PPl - PBS.
a. The chest wall in the adult functions like a bellows that contains compression springs so
that the equilibrium volume (see II D 2 b) is near maximal volume (see Figure 3-50. If the
lung volume is less than the chest wall equilibrium volume, then Pew is negative (PPl <
PBS), which indicates that the chest wall has a tendency to expand. Thus, the relaxation P-V
curve for the chest wall (Pew) is located largely on the negative side of the pressure axis in
Figure 3-SC. The expansion force exerted by the relaxed chest wall helps to create the
negative interpleural pressure and to keep the lungs expanded.
b. At low volumes, the slope of the Pew line (compliance) approaches zero, which is caused
by the thorax becoming essentially rigid.
4. Transrespiratory pressure (PRS) is the transmural pressure across the respiratory system. The
inside pressure is PA and the outside pressure is PBS, so PRS = PA - PBS. Normally, the pressure

100

6D

E
::>

E
'x<0
E
-;F.

PL = 20

Ql

E
::>

'0

>
PL = PA - PPL

PA

PAS = PA - PBS

PPL

Pew = PPL - PBS

PBS

PL = 2

0
0
-30
Pressure (em H 2O)

20

100

Ql::>

E.
::>x

-<0

illl

gE

-;F.

Pew

0
-40

-15

20

Pressure (em H 2O)


Figure 3-5. (A) Schematic relationships of the structures and pressures used to define the mechanical
properties of the respiratory system. PA = alveolar
pressure; PPL = interpleural pressure; PBS = pressure
at the body surface; PRS = transrespiratory pressure;
PL = transpulmonary pressure; and Pew = transthoracic pressure. (B) Pressure-volume curve of the lungs,
which are represented as a simple balloon whose
volume depends only on the transmural pressure.
(C) Pressure-volume curve of the chest wall, which is
represented as a bellows with internal compression
spri ngs that set its relaxation vol u me at about 80% of
maximum. (0) Pressure-volume curve of the respiratory system, which is represented as the combination
of the balloon and bellows. The system is shown at
three different volumes (1, 2, and 3), with representative alveolar (PA) and interpleural (PpL) pressures.

o
100
PA
PPL

10

O+-~--~--~---+--~--.-~

- 40

20

Pressure (em H 20)

at the body surface (PBS) is atmospheric, which is considered to be zero when dealing with
differential pressures. It can be shown that PRS = PL + Pew. *
a. The respiratory system P-V curve (see Figure 3-5D) is "5" shaped, steepest in the middle,
and almost flat at both high and low lung volumes. Normally, the respiratory system functions
near the middle of the curve, where the system is most distensible. This P-V curve is due
to the summation of the elastic properties of the chest wall and the lungs, so that PRS = PL
+ Pew.
*It can be proven that PRS = PL + Pcw by substituting the expressions for PL and Pcw; thus: PRS = PA - PPL
PBS). PPL cancels out, leaving: PRS = PA - PBS, which is the definition of the respiratory system pressure.

(PPL-

b. The equilibrium volume of the respiratory system occurs at about 40% of maximal lung
volume. This volume also is referred to as the functional residual capacity and represents
the normal end-expiratory lung volume.

E. Elastance is the force generated by distension of any structure, whereas the transmural pressure
represents the force that counteracts the elastance to keep a structure inflated. An example will
clarify this relationship: If one stretches an elastic band between one's hands, the transmural
pressure represents the force generated by the arms to lengthen the elastic band; in turn, the elastic
band recoils against the hands, and this force is termed the elastance. The smaller the compliance
(the thicker the elastic band), the greater the force (elastance) generated by any increase in volume.
Thus, compliance (C) and elastance (E) are inversely related (C = 1 IE). At a constant volume,
the elastance force and the transmural pressure are equal but in opposite directions. Elastic forces
in the lungs arise from two sources: tissue and surface forces.
1. Tissue forces. The lung contains large amounts of collagen and elastin, which are connective
tissue fibers whose roles in generating elastic (retractile) forces in the lungs are not fully understood. It is thought that elastic fibers are stretched at low and medium lung volumes, and that
collagen prevents overdistension of the lungs at high lung volumes. Smooth muscle fibers in
the lungs also may contribute to the tissue forces.

2. Surface forces. Whenever an interface exists between two different phases (e.g., between a
liquid and a gas), a surface force or surface tension is generated. The surface force tends to
minimize the surface area of the interface; this is what causes water droplets to bead up on a
waxed surface of a car. The surface force is caused by the unbalanced attraction of the surface
molecules by the molecules below the surface (Figure 3-6Al. The surface forces cause the lungs
to retract or empty, which is counteracted by the transmural pressure. Filling the lungs with a
liquid, such as saline, eliminates the surface forces and markedly reduces the recoil force
compared to the air-filled lung (Figure 3-7). In the fluid-filled lung, the retractive forces are the
result of only the tissue component. The conclusion from this experiment is that the surface
forces, which are present when the lungs are air filled, represent a more powerful cause of the
lungs' recoil pressure than the tissue forces.
a. Surface tension. If the surface is spherical, as in an alveolus, then an opposing force is
required to counteract the surface tension to keep the structure inflated. This opposing force
is the surface component of the transmural pressure as shown in Figure 3-6C.
b. Laplace's law gives the relationship between surface tension (ST), transmural pressure (P),
and radius (r) of the spherical structure: P = 2 x STir (see Figure 3-6B). An increase in the
surface tension or a decrease in the radius would require a higher pressure to keep the
alveoli inflated. Since the lungs normally have a low transmural pressure, any decrease in
lung volume or increase in alveolar surface tension leads to collapse of the alveoli. Collapse
of the lung air spaces is termed atelectasis.
c. Pulmonary surfactant lines the alveolar surface.
(1) Composition. Pulmonary surfactant is a complex mixture made up of phospholipids,
cholesterol, and a specific protein; all of these substances are formed by type II
alveolar cells.

(2) Functions
(a) Reduction of surface tension. Adsorption of surface active molecules at the
alveolar-air interface reduces the surface tension in the lungs. The reduction of the
surface tension, which is the prime function of pulmonary surfactant, increases the
compliance of the lungs and thereby decreases the work of respiration.
(b) Increase in alveolar radius. Surfactant fills irregularities in the alveolar surface,
which increases the mean alveolar radius. This increase reduces the transmural
pressure required as shown by the formula for the law of Laplace. This effect also
increases the compliance of the lungs and reduces the work of breathing.
(c) Reduction of pulmonary capillary filtration. Normal surfactant, by lowering the
alveolar surface tension, reduces the retraction forces in the lungs. This makes the
interstitial pressure of the lungs less negative, which reduces the filtration forces
across the pulmonary capillary (Frank-Starling mechanism; see Ch 2 V B 1 bl. Thus,
normal surfactant helps to keep the lungs from becoming edematous, which would
interfere with gas exchange by increasing the thickness of the alveolar-capillary
membrane or by causing alveolar flooding.
(d) Stabilization of the alveoli. Alveolar stability would be enhanced if the alveolar
surface tension changed in proportion to the size of the alveoli. The surface tension

B
Surface

{te",IOO~
p

"c

2 ST

Surface
tension

Bubble

Figure 3-6. (A) Schematic of the intermolecular forces for molecules in the gas phase, at the surface and in the bulk
phase of a liquid. Note that surface molecules have an unbalanced force tending to pull them into the liquid.
(8) Surface forces in a hollow sphere tend to reduce the volume of the sphere. The relationship between surface
tension and the surface component of transmural pressure is defined by Laplace's law. (C) To maintain a constant
volume in a spherical structure, the transmural pressure must counterbalance the surface tension forces. P = pressure;
ST = surface tension; and r = radius.

at an air-fluid interface changes when surface active agents are placed on a surface
balance, and the surface area is cyclically changed. Lung extracts containing surfactant change their surface tension from 0 to 40-50 dynes/em during such cycling
(Figure 3-8). Whether such changes in surface tension occur in the alveoli is still
controversial.
(3) Abnormalities
(a) Respiratory distress syndrome (RDS) of the newborn is due to abnormal surfactant that is formed because of immaturity of the lung. These infants have inadequate
gas exchange because of alveolar instability and consequent loss of surface available
for exchange.
(b) Atelectasis (aveolar collapse) occurs in several other conditions associated with
abnormalities in surfactant.
(c) Shock, systemic infections, and trauma can cause abnormal surfactant function
in adults (adult respiratory distress syndrome).

100
......._--Air

()

~ 50

Figure 3-7. Pressure-volume loops for isolated lungs


when filled with saline or air, showing the direction of
volume change (long arrows) as a percent of the total lung
O~--------------.-------------~
capacity (TLC). Note the marked increase in pressures
o
30 required during the air inflation. PL = transpulmonary
pressure.

100

E 80

'x
<1l

E 60

~
e.....

<1l

<1l

40

CD

(,)

<1l
't:

20

::l

en

20

40

Surface tension (dynes/em)

60

Figure 3-8. Surface tension as a function of surface area


obtained from bronchoalveolar lavage fluid. Note the
large hysteresis and that minimal surface tension approaches zero. The arrows indicate the direction of the
recording.

d. Tissue interdependence is another mechanism that stabilizes alveoli. Since the alveolar
walls form a continuous network with one another and are under some tension, the co "apse
of one area is counteracted by the tension that is generated in adjacent alveolar walls (Figure
3-9). Thus, alveoli are not independent units that can change volume independently of the
neighboring lung tissue.
F. Resistance forces are present whenever lung volume is changing. Resistance is caused by the
friction of gas molecules between themselves and the walls of the airways (airway resistance) as
we" as by the friction of the tissues as they expand or contract (tissue resistance).

Figure 3-9. A schematic representation of the concept of tissue interdependence. Alveolar collapse occurs if
surface tension increases or alveolar
ventilation is reduced sufficiently. Tissue interdependence prevents alveolar collapse because of the tension
generated in the surrounding alveolar
septa (indicated by the large open
arrows) and, therefore, stabilizes alveolar volume.

1. Airway resistance represents about 80%-90% of the total resistance forces during breathing.
The airway resistance depends on the rate of gas flow and the diameter and length of the
airways. Airway resistance can be calculated by dividing the alveolar pressure during respiration
by the rate of gas flow from the lungs. The usual units of airway resistance are em H 2 0/L/sec.
a. Flow rate of gas is determined by the velocity of the gas molecules as they pass through
the airways. The airway system is organized so that the total area of the airways increases
about 1000-fold as the gas proceeds from the trachea through about 23 generations of
branchings to the alveoli. This is similar to a river that widens gradually into a lake. The
velocity of flow is much higher in the river (trachea and bronchi) than it is in the lake
(alveolar ducts), because the river is much narrower. The highest resistance to flow occurs
in the intermediate-sized bronchi because of the velocity. The airways less than 2 mm in
diameter represent only about 10% of the total airway resistance.
b. Airway diameter is an extremely powerful factor that determines airway resistance. The
smaller the airway the higher the resistance for any flow rate. * Actually, a decrease in airway
diameter to half its original size causes the airway resistance to increase 16-32 times. People
with chronic obstructive pulmonary disease (COPD), such as asthma, bronchitis, and
emphysema, have a narrowing of their small airways and a very high airway resistance. The
high airway resistance causes COPD patients to generate very high pressures with their
respiratory muscles in order to breathe.
(1) Sympathetic nervous activity to the nerves in the airways causes only minor changes,
since these adrenergic nerves do not actually innervate the bronchial smooth muscle.
However, circulating adrenergic substances (e.g., epinephrine) or adrenergic-like
drugs cause a dilation of the airways, which reduces airway resistance. The dilation
occurs since the bronchial smooth muscle contains largely .B 2 -receptors, which are
activated by epinephrine or a .Bradrenergic drug, which can be administered by inhalation.
(2) Vagus nerve (cholinergic) activity to the lungs causes the smooth muscle in the walls
of the bronchi and bronchioles to contract and increases the formation of mucus. This
narrows the airways and increases the resistance to airflow. Cholinergic activity is a
major component in the pathogenesis of asthma in some individuals.
(3) Nonadrenergic, noncholinergic activity. The lungs are derived embryologically from
the foregut. Recently, it has been recognized that many of the neuropeptides that control
intestinal activity also control airway dimensions.
(a) Noncholinergic excitatory compounds, such as substance P and neurokinins A
and B, cause bronchoconstriction, mucus secretion, and increased vascular permeability. These compounds seem to be especially involved in the pathogenesis of
asthma in certain individuals.
(b) The nonadrenergic inhibitory system consists of postganglionic neurons that release
vasoactive intestinal peptide (VIP) and other mediators that cause smooth muscle
relaxation and inhibit mucus production.
c. Airway length changes relatively little during respiration and disease processes and, thus,
is not a significant factor that changes airway resistance.
2. Tissue resistance plays only a minor role in determining the forces required to breathe.
3. Effort-independent flow
a. Effort-independent flow occurs during forced expiration at volumes less than 80% of the
maximal lung volume. This relationship is shown in Figure 3-10, where expiratory flow is
plotted as a function of the interpleural pressure at different lung volumes. Under these
conditions, interpleural pressure can be used as a measure of the effort exerted during
expiration. In Figure 3-10, note that the expiratory flow rate remains constant at each lung
volume, while interpleural pressure varies over a large range (i.e., flow is independent of
effort). The factor controlling flow under these conditions is the transpulmonary pressure,
which is a function of lung volume.
b. Dynamic compression of the airways (Figure 3-11) occurs during forced expirations and
limits the flow rate during the terminal part of expiration. The amount of dynamic compression depends on several variables that determine the limitation of the expiratory gas flow.

'In the lungs, the smaller airways provide a larger total cross-sectional area, so that flow rates and airway resistance
are reduced as gases proceed toward the alveoli.

60% VC

UQl
C/)

2;;:

:;::
>.

B
l!'

40%VC
3

'5.

x
w

10% VC

O;-~~~Lr------~--------~

-30

30

60

Pleural pressure (em H2 0)

Maximal effort

10

UQl

C/)

2;;:

6
4

T!!

'5.

x
w

0
0

100
Lung volume (% VC)

Figure 3-10. (A) Isovolume flow diagram. At volumes less than 80% of total lung capacity, expiratory flow rate is
limited, and the flow rate is independent of effort as measured by the interpleural pressure. VC = vital capacity.
(8) Flow-volume loop. Much information can be obtained about pulmonary mechanics by recording flow rate as a
function of lung volume. The flow rate labeled maximal effort represents maximal flow rate and cannot be exceeded.
The maximal flow rate at each volume depends on the state of the airways and the lung parenchyma and provides
important diagnostic information. % VC = percent vital capacity.

(1) Lung volume is the most powerful modulator of expiratory gas flow. When lung volume
is increased, airway diameter expands because of the enhanced radial traction and
expiratory muscle force increases due to changes in the muscle length.
(2) Elastic recoil alters the diameter of the intrapulmonary airways also through radial
traction on the airway walls. Elastic recoil is decreased by processes such as aging and
emphysema and is increased in patients with restrictive lung disease.
(3) Increased airway resistance, caused by COPD, increases the pressure drop along the
small peripheral airways. This loss of pressure produces a greater compressive effect on
the airways, leading to a further narrowing and even greater reduction in the rate of gas
flow at any given lung volume. Patients with obstructive lung disease can reduce the
pressures needed to breathe by breathing slowly but with a larger tidal volume to
maintain a normal alveolar ventilation. This breathing pattern reduces the velocity of
gas flow, the pressures required to overcome resistance, and the work of breathing.

PL = + 3 - (- 3) = 6 cm H20
Forced expiration

+10

n------

Figure 3-11. Schematic representation of the flow limiting segment in an airway during forced expiration.
PPL = interpleural pressure; PA = alveolar pressure; and
PL = transpulmonary pressure. During eupneic expiration, the transmural pressure of the airways is a distending
force (PA > PPl), which dilates airways. During a forced
expiration, PPl > the pressure in the bronchi (indicated
by the arrows), which narrows airways, increases airway
resistance, and limits the expiratory flow rate.

G. The work of respiration equals the change in pressure needed to inflate the lungs times the
volume change: work = I:J. p x I:J. V. If either the resistance to breathing increases or the compliance
decreases, then the respiratory muscles must generate more force and, thus, use more O 2 to
overcome the added load.
1. Respiratory failure. The respiratory muscles, just as any skeletal muscles, can become fatigued
if exposed to a high work load for a long time. If the respiratory muscles cannot generate
sufficient force, then pulmonary ventilation becomes inadequate, and respiratory failure occurs.
In respiratory failure, there is an inadequate supply of O 2 to the body, and CO 2 excretion is
impaired.
2. The minimal work for respiration varies with the resistance and elastance of the respiratory
system; these factors are largely determined by the size of the airways and the lungs. All animal
species seem to use the combination of tidal volume and respiratory rate that creates the
minimal work of respiration. The mechanism by which the brain selects this combination is
unknown.
a. Increased resistance work occurs when the resistance forces are increased. Resistance
work can be minimized by breathing more slowly and deeply, since this reduces the flow
rates of gas and thereby decreases the pressure gradients needed to generate gas flow.
b. Increased elastance work occurs when the compliance is reduced (increased elastance).
The elastance work of breathing is proportional to the tidal volume as well as to the elastance
of the respiratory system. The elastance work of breathing can be minimized by decreasing
the tidal volume but breathing more rapidly to maintain an adequate ventilation of the lungs.

H. Lung volumes and capacities. The amount of gas in the lungs can be divided into various
compartments with specific functions. There are four lung volumes that are nonoverlapping
fractions of the gas content and four capacities that are combinations of two or more of the lung
volumes. Figure 3-12 shows how these volumes and capacities are organized.
1. Lung volumes
a. Tidal volume (VT) is the volume of gas inspired or expired with each breath. The normal
tidal volume is 500 ml (0.5 L).
b. Inspiratory reserve volume (IRV) is the additional volume of gas that can be inspired
above the tidal volume. The inspiratory reserve volume is in reserve and can be used if the
tidal volume needs to be increased (e.g., during exercise).
c. Expiratory reserve volume (ERV) is the volume of gas that can be expired after a normal
expiration. The expiratory reserve volume also is partially used when tidal volume needs to
be increased.
d. Residual volume (RV) is the volume of gas that remains in the lungs after a maximal
expiration. This gas prevents the alveoli from completely collapsing; complete collapse of

-r----Spirogram .........

IRV

---rr
vc

-20

TLC

+30

Time

Figure 3-12. (A) A pressure-volume curve of the respiratory system. Residual volume (R\I) and total lung capacity

are limited by the decreased compliance of the system, as indicated by the reduced slope of the curve.
Functional residual capacity (FRC) is determined by the volume at which the transrespiratory pressure (PRS) is zero.
(B) A spirogram showing several normal tidal volumes (Vn; a maximal inspiratory effort followed by a maximal expiratory effort, termed a vital capacity (VC) maneuver; and the return to normal VT. This tracing can be used to define
all of the subdivisions of lung volume. ERV = expiratory reserve volume; IRV = inspiratory reserve volume; and
Ie = inspiratory capacity.
(TLC)

an area of the lung (atelectasis) requires extremely large forces to reopen the alveoli. Since
the residual volume cannot be expired into a spirometer, it must be measured by either a
dilution method or a plethysmographic technique, as illustrated in the following examples.
(1) Dilution method
(a) Helium-an indicator gas that can be readily detected-is placed in a spirometer.
[In this example, the fraction of helium is assumed to be 0.2 (F, = 0.2) of the total
volume in the spirometer (Vsp ), which is 3 L.J
(b) If a person begins to breathe from the spirometer at the end of a normal expiration,
then the volume of gas in the lungs at the beginning of the test, or the functional
residual capacity (FRC), will be measured. The spirometer volume is maintained
at 3 L by the addition of O 2 ,
(c) At the end of the test, when the helium is equilibrated between the lungs and the
spirometer, the fraction of helium (F 2 ) is found to be 10%. The lung volume can be
easily calculated using the following relationship

FRC

F, x Vsp = F2(V sP + FRC)


F, x VSp/F2 - Vsp = 0.2 x 3/0.1 - 3

3 L

(d) If the expiratory reserve volume is measured from the spirometer tracing and found
to be 2 L, then the residual volume is 1 L.
(2) Plethysmographic method
(a) A person is placed in an airtight chamber (plethysmograph), and the pressures within
the chamber and within the airways are measured while the person pants against a
closed airway.
(b) With proper calibration of the system, it is possible to determine the volume of gas
in the thorax by applying Boyle's law (P,V, = P2 V2 ), where the two subscripts
indicate the pressure (P) and the volume (V) under the different conditions when
the number of gas molecules remains constant.
2. Lung capacities
a. Total lung capacity (TLC) is the volume of gas in the lungs after a maximal inspiration. It
represents the sum of the four lung volumes.
b. Vital capacity (VC) is the maximal volume of gas that can be expired after a maximal
inspiration. Measurement of the vital capacity under different conditions is the most commonly performed pulmonary function test.

(1) Slow vital capacity is used to evaluate the size of the lungs. Many lung diseases can
decrease the vital capacity by either reducing the total lung capacity or increasing the
residual volume. A patient's vital capacity can be compared to predicted normal values,
which are dependent on height, age, and sex.
(2) Forced vital capacity is used to evaluate the resistance properties of the airways and
the strength of the expiratory muscles.
(a) This test is performed by having a patient expire as rapidly as possible into a
recording spirometer. Patients with high airway resistance (obstructive lung disease)
are unable to achieve high expiratory flow rates and, therefore, require a longer
time to expire their vital capacity (Figure 3-13).
(b) A measurement that is commonly made is one that determines the percent of the
vital capacity that can be expired in 1 second. This is called the %FEV, (forced
expiratory volume in 1 sec/forced vital capacity x 100), which is normally greater
than 75%-80%.
(c) Figure 3-13 shows typical recordings of forced vital capacities and several measurements that can be obtained from normal, healthy patients and patients with obstructive and restrictive lung disease.
c. FRC is the volume of gas that remains in the lungs after a normal expiration. The FRC
represents a reservoir of gas that allows oxygenation of the blood between breaths. Since
the FRC contains the residual volume, it also cannot be measured by spirometry alone.
d. Inspiratory capacity (IC) represents the sum of the tidal volume and the inspiratory reserve
volume and is the maximal volume that can be inspired after a normal expiration.

III. GAS EXCHANGE BETWEEN THE ATMOSPHERE AND THE BODY


A. Composition of air. Air is a mixture of gases composed primarily of N2 and O 2, with a variable
amount of water vapor, a negligible quantity of CO 2 and a small amount of inert gases. These
inert gases usually are lumped together with N z. Dry atmospheric air can be considered to be
composed of 79% N2 and 21% 02.

1. Partial pressure of gases. Gas molecules are in constant, rapid motion causing them to strike
each other and the walls or surfaces of any container or structure. Because of this constant
bombardment, these molecules exert a pressure, which is termed the atmospheric or barometric pressure. The normal barometric pressure at sea level is 760 mm Hg (760 Torr). If there
is a mixture of gases, then each gas exerts its own partial pressure, which is equal to its
concentration (fraction of gas present) times the total pressure (Dalton's law).

2. Partial pressure of N2 (PN 2) in air is calculated by multiplying the fraction of N z in air by the
barometric pressure. PN z = 760 x 0.79 = 600 mm Hg. The term tension is synonymous with
partial pressure; thus, the N z tension

600 mm Hg.

TLC:
7

d:
C])

E
:::I

"0 4
>
Cl
c:
:::I

...J

2
Time (sec)

Figure 3-13. Forced vital capacity maneuvers from a


normal individual and from patients with chronic obstructive pulmonary disease (COPO) and restrictive lung disease (RLOl. The y axis represents the absolute lung volume. Note the differences in the rate of expiratory flow
for the three subjects. TLC = total lung capacity; RV =
residual volume.

3. Partial pressure of O 2 (P0 2 ) is determined the same way, so that the O 2 tension of air =
760 x 0.21 = 160 mm Hg. Note that the sum of the O 2 and the N2 tensions equals the total
barometric pressure.
B. Minute ventilation and alveolar ventilation
1. Minute ventilation (V E) is the volume of air inspired or expired per minute, which equals the
tidal volume (VT) multiplied by the respiratory rate or frequency (f): VE= VT x f. The tidal
volume at rest averages 500 ml (0.5 L), and the normal respiratory rate is 12-15 breaths/min.
Therefore, the normal minute ventilation is 6-7.5 L/min.
2. Dead space ventilation (VD). The upper airways, from the nose and mouth to the terminal
bronchioles, do not participate in gas exchange because of the thickness of their walls. This
area of the respiratory system, which contains a gas volume of approximately 150 ml, is termed
the conducting zone, or dead space (V D), because it does not participate in gas exchange.
The dead space ventilation is the. volume of gas per minute that enters the dead space of the
respiratory system. For example, Vo = Vo x f = 0.15 x 15 = 2.25 L/min.
a. Anatomic dead space is the volume of gas that occupies the nose, mouth, pharynx, larynx,
trachea, bronchi, and bronchioles. The anatomic dead space has several functions, including
conditioning of air and removal of foreign materials.
(1) Conditioning of air. During nasal breathing, the inspired gas is warmed to body temperature and is saturated with water vapor (has 100% relative humidity) by the time it
reaches the trachea. The addition of water vapor to the inspired air dilutes the O 2 and
N2 concentrations slightly. Mouth breathing or the presence of a tracheostomy allows
cool, dry air to reach the lower airways. This can result in the drying of airway secretions
and the formation of mucous plugs.
(2) Removal of foreign material occurs through the processes of filtration in the nose,
impaction in the lower airways, or dissolving in the moist surface of the airways. Small
particles (e.g., soot or pollen) impact on the surface of the airways and stick to the
mucous lining. The foreign material is carried in the mucus toward the mouth, where it
is either expectorated or swallowed. The mucus is propelled upward by the cilia of the
respiratory epithelium that line the airways.
(3) Reaction to foreign material. Foreign material in the inspired gas can stimulate irritant
receptors in the airways, which causes coughing, secretion of mucus, and hypertrophy
of the mucous glands. Prolonged breathing of air that contains foreign materials such
as cigarette smoke and smog can cause chronic bronchitis.
b. Total dead space. In normal persons, the anatomic dead space represents the entire dead
space, but, in patients with lung disease, some alveoli do not receive any blood flow and
therefore do not participate in gas exchange. These alveoli form the alveolar dead space.
(1) Bohr's method. The total dead space equals the anatomic dead space plus the alveolar
dead space. The total dead space can be determined by measuring CO 2 levels in the
expired gases.

Vo = VE x (1 - PEC0 2/PAC0 2)
where PEC0 2 and PAC0 2 represent Pco 2 in expired and alveolar gases, respectively. This
method requires measuring the Pco 2 in mixed expired gases and in end-tidal samples
of expired gases which, theoretically, represent pure alveolar gas.
(2) Fowler's method of determining the dead space requires measuring expired N2 tensions
after inspiring 100% O 2 (see VI C 1 b).
3. Alveolar ventilation (VA) is the volume of gas that participates in the exchange of O 2 and
CO 2, The normal alveolar ventilation is 4.5-5 L/min (alveolar ventilation equals the minute
ventilation minus the dead space ventilation). An adequate alveolar ventilation is critical since
it determines the alveolar P0 2 and Pco 2
a. Alveolar Po 2 O 2 is continually removed from the alveoli by its diffusion into the pulmonary
capillary blood. Inspiration brings fresh air into the alveoli, which maintains the alveolar P0 2
at about 100 mm Hg. The alveolar P0 2 (Table 3-2) is affected by changes in the barometric
pressure (mountain climbing or diving), the fraction of O 2 inspired (supplemental O 2), and
by the amount of CO 2 in the alveoli (hypoventilation or hyperventilation). The alveolar P0 2
at sea level can be raised above 600 mm Hg by administering 100% O 2 through a mask.
The prolonged use of 100% O 2 results in lung damage so that high concentrations of O 2
must be used with caution, similar to any other powerful drug, when treating a patient.

Table 3-2. Alveolar Air Equation


The alveolar air equation can be used to calculate the average alveolar oxygen
tension (PA0 2 ) and is stated as
PA0 2

(PB - 47)

FI0 2

PAC0 2
-

--

where PB = barometric pressure; 47 = vapor pressure of water at body temperature; FI0 2 = fraction of O 2 in inspired air; PAC0 2 = alveolar CO 2 tension; and
R = volume of CO 2 excreted per volume of O 2 absorbed per minute (R normally
is 0.8).
Normal PA0 2 at sea level is
PAO,

40

(760 - 47) x 0.21

0.8

100 mm Hg

b. Alveolar Peo 2
(1) Alveolar Pco 2 is determined by the ratio of the rate of CO 2 production (Vco 2 ) to alveolar
ventilation (VA)
where K is a constant.
(2) Alveolar and arterial CO 2 tensions are essentially equal due to the high diffusibility of
CO 2 and the shape of the CO2 content curve. Pco 2 normally is very closely regulated
to a value of 40 mm Hg, since changes in Pco 2 alter pH, which affects the rates of
many enzymatic reactions.
(a) An increase in alveolar ventilation causes the alveolar Pco 2 to decrease, since the
CO 2 is washed out of the lungs by the increased volume.
(b) A decrease in alveolar ventilation (hypoventilation) results in a rise in alveolar Pco 2
and is termed hypercapnia if it exceeds 45 mm Hg. The alveolar Pco 2 is determined
by the ratio of the rate of CO 2 production to the rate of alveolar ventilation.
Hypercapnia indicates that ventilation is inadequate for the metabolic rate and
usually is accompanied by hypoxia.

C. Diffusion. O 2 and CO 2 move across the respiratory membrane between the alveoli and the
pulmonary capillaries by diffusion. Diffusion is the movement of molecules from areas of high
concentration to areas of low concentration by random motion. The rate of diffusion is determined
by several factors as defined by Fick's law of diffusion (Table 3-3) [see Ch 1 I 1 C a (3)].
1. Diffusion capacity of the lungs (DL) is a pulmonary function test that measures the diffusion
properties of the respiratory system. The DL is measured with a dilute concentration of carbon
monoxide (CO). The DL is proportional to the surface area of the lungs divided by the length
of the diffusion path. Lung diseases can decrease the surface area for gas exchange or lengthen
the diffusion path and reduce the diffusion properties of the lungs. These changes can be
detected by the diffusion capacity test.
2. The respiratory membrane consists of several components.
a. Each layer of the respiratory membrane is extremely thin; the normal respiratory membrane
averages about 0.5 j.l (0.00002 inch) in thickness.
Table 3-3. Fick's Law of Diffusion
Fick's law of diffusion is stated as

K x (P 1 - P2 ) x A
V=------

where V = volume of gas diffusing per minute; K = a constant that includes


temperature and the diffusion coefficient of the gas; P1 = partial pressure of the
gas on one side of the membrane; P2 = partial pressure of the gas on the
other side of the membrane; A = surface area of the membrane; and
d = diffusion distance between the two sides of the compartment.

b. The surface area of the respiratory membrane is very large, averaging about 100 m 2 in the
normal adult. The large surface area and the small distance separating the alveolar gas from
the pulmonary capillary blood allows rapid diffusion of gases between the two compartments.

3. Diffusion of 02 occurs from the alveolar gas to the pulmonary capillary blood, because the
alveolar P0 2 usually is 100 mm Hg, while the blood entering the pulmonary capillary normally
has a P0 2 of 40 mm Hg.
a. The O 2 molecules dissolve in the plasma, which raises the P0 2 and allows O 2 to diffuse into
the red blood cell so that it can combine with hemoglobin. Normally, enough O 2 diffuses
across the respiratory membrane so that the blood P0 2 becomes equal to the alveolar P0 2
in about 0.25 second.
b. Pulmonary diseases that reduce the surface area or increase the thickness of the respiratory
membrane can reduce the rate of O 2 transfer (Figure 3-14). These diffusion abnormalities
can result in hypoxemia (i.e., a reduced O 2 level in the blood) if the P0 2 in the pulmonary
capillary blood never becomes equal to the alveolar Po 2 Under these conditions, O 2
exchange is termed diffusion limited, whereas O 2 exchange normally is limited by the rate
of blood flow (perfusion limited).

4. Diffusion of CO 2 occurs from the pulmonary capillary blood (Peo 2

= 46 mm Hg) to the alveoli


(Peo 2 = 40 mm Hg). CO 2 diffuses much more rapidly than O 2 , because it is about 25 times
more soluble in body fluids than is O 2 , This factor is included in the diffusion coefficient of the
Fick equation. The increased solubility means there are more molecules dissolved in the liquid,
which increases the rate of diffusion.

IV. GAS TRANSPORT. O 2 must dissolve in the plasma of the pulmonary capillaries before it can diffuse
to the hemoglobin inside the red blood cell. The volume of gas that dissolves in any liquid is determined
by Henry's law.

A. Henry's law states that the volume of gas "x" that dissolves in a liquid equals the product of its
solubility coefficient (Sx) and its partial pressure (Px): C = (Sx) (Px).
1. Gas molecules dissolve in liquids the same way that sugar dissolves in coffee. In order to know
how much sugar is dissolved in coffee, one needs to know how many scoops were put in as
well as the size of the scoops. Similarly, the gas content of fluids depends on two factors. The
solubility coefficient is similar to the size of the scoop, and the partial pressure is comparable
to the number of scoops.

100

End
Np
Normal DL0 2

Q.

: Abnormal DL0 2

50

CO
0
0

0.75

1.5

Time (sec)
Figure 3-14. Changes in pulmonary capillary gas tensions, as a percent of the alveolar gas tension (% PA). The x axis
represents the time the blood is in the pulmonary capillaries, which normally is 0.75 second. Dw, = diffusion
capacity of the lungs for 02; N 2 0 = nitrous oxide; and CO = carbon monoxide. Normally, O 2 is a perfusion limited
gas, as is N 20, since there is equilibration between pulmonary capillary blood and alveolar gas tensions. CO represents
a diffusion limited gas that is commonly used to measure the diffusion capacity of the lung.

Table 3-4. Solubility Coefficients of Some Important Gases in Respiratory


Physiology

Solubility Coefficient
(ml gas/ml saline/mm Hg gas tension)
Temperature ("C)

o
20
37

CO
0.049
0.032
0.024

0.024
0.016
0.012

1.71
0.90
0.58

0.035
0.023
0.019

2. It is important to understand the difference between the partial pressure of a gas and the gas
content of a liquid. The gas content represents the volume of the gas (in units of volume per
unit volume of liquid) that is present, while the partial pressure of the gas represents the pressure
it would exert in the gas phase. The solubility coefficient represents a proportionality constant
that equates gas content and partial pressure.

3. Table 3-4 shows the solubility coefficients of some gases that are important in respiratory
physiology. Note that the solubility coefficients for the various gases are different and that
increasing the temperature reduces the solubility of gases.

B. Hemoglobin markedly increases the amount of O 2 that is carried in the blood (the size of the
scoop is increased almost 50-fold). Each gram of hemoglobin can combine with 1.34 ml of O 2;
normally, there are 12-15 g hemoglobin/dl blood. The arterial blood carries approximately 20 ml
02/dl; about 98% of this O 2 is carried by hemoglobin, and 2% is dissolved in the plasma.

C. O 2 transport. O 2 dissolves in the plasma of the pulmonary capillaries after diffusing across the
respiratory membrane of the lungs. O 2 then diffuses into the red blood cell, where it combines
reversibly with the iron atom of hemoglobin and converts deoxyhemoglobin (reduced hemoglobin) into oxyhemoglobin. Normally, the P0 2 in the pulmonary capillary blood becomes equal to
the alveolar P0 2 before the blood leaves the pulmonary capillary.

1. Hemoglobin saturation is the percent of hemoglobin that is combined with O 2, The amount
of O 2 that combines with hemoglobin depends on the P0 2 as indicated by the hemoglobin
saturation curve (Figure 3-15).
a. Pso is the P0 2 that produces a 50% saturation of hemoglobin with O 2, The normal Pso for
arterial blood is 27 mm Hg.

b. Hemoglobin affinity for O 2 is inversely related to the Pso . An increase in hemoglobin


affinity means that O 2combines more readily with hemoglobin at the lungs, while a decrease
in the hemoglobin affinity causes hemoglobin to release O 2 more readily at the tissue level.
c. The loading zone of the 02-dissociation curve is the plateau above a P0 2 of about 60 mm
Hg. The loading zone provides a margin of safety, since the P0 2 in the lungs can be reduced

100

. - - : Loading ~

O~~~--------r-------~r-----------------------~
o
27
50
100
POz(mm Hg)
Figure 3-15. Hemoglobin saturation as a function of O 2 tension (Po2 ) in the blood. Normally, the hemoglobin
becomes 95%-98% saturated as it flows through the pulmonary capillaries (loading zone). When the blood enters
the systemic capillaries, O 2 diffuses from the red cells into the tissues (unloading zone), and the hemoglobin saturation
is reduced to about 75%, which is equivalent to a P0 2 of 40 mm Hg. At a hemoglobin saturation of 50%, the P0 2
is normally 27 mm Hg (Pso = 27 mm Hg).

substantially, and yet the O 2 content (hemoglobin saturation) remains relatively high. This
allows one to climb mountains to moderate altitudes and suffer only a small decrease in the
arterial O 2 content even though the arterial P0 2 has decreased substantially.
d. The unloading zone is the steep portion of the dissociation curve below a P0 2 of about 60
mm Hg. The P0 2 in the tissues is very low since O 2 is consumed by metabolism. A gradient
in P0 2 is established between the capillary blood and the tissues, which causes O 2 to diffuse
from blood to tissue. As O 2 diffuses into the tissues, the capillary P0 2 is reduced. The
capillary P0 2 is determined by the ratio of the blood flow to the rate of O 2 consumption; a
low blood flow or a high O 2 consumption leads to a low tissue and capillary Po 2 The steep
portion of the curve allows hemoglobin to release O 2 readily if the tissue O 2 consumption
increases; this minimizes the decrease in P0 2 that would otherwise take place.
e. Physiologic alterations in Pso are produced by changes in temperature and by changes in
the concentration of CO 2 , H+, other ligands such as diphosphoglycerate (DPG), ATP, ADP,
and other organic phosphates (Figure 3-16).
f. Bohr effect. Hemoglobin is an allosteric enzyme that reacts with O 2 , and the affinity
between hemoglobin and O 2 is altered by these different ligands. The Bohr effect refers to
the increased Pso of the Ordissociation curve caused by an increase in Peo 2

2. O 2 capacity is determined by the hemoglobin concentration. The O 2 capacity equals hemoglobin concentration in g/dl of blood times 1.34 ml 02/g. The normal O 2 capacity is 20.1 ml
02/dl blood (15 g hemoglobin/dl times 1.34 ml/g). It is important to realize that the O 2 capacity
varies only with the amount of hemoglobin in the blood. The O 2 capacity defines the maximal
amount of O 2 that can be carried in the blood by hemoglobin.

3. O 2 content is the actual amount of O 2 being carried in the blood. The O 2 content depends on
the hemoglobin concentration, the Po 2 , and the Pso of the hemoglobin. Figure 3-17 shows the
effect of varying these parameters on the O 2 content of blood.
a. The O 2 content usually is expressed as ml 02/dl blood (also termed volumes %, or vol%).
The normal arterial O 2 content is about 19.5 vol%, while the normal venous O 2 content is
about 15 vol%. Thus, 4.5 vol% of O 2 are released from every deciliter of blood in the
systemic capillaries to supply tissue metabolism.
b. The normal cardiac output is 5-5.5 L/min, so that 50-55 dl are pumped by the heart, and,
if each deciliter supplies 4.5 ml of O 2 , then the body consumes 225-250 ml 02/min.
c. Hemoglobin saturation is calculated by dividing O 2 content by O 2 capacity, then multiplying
by 100.
4. Carboxyhemoglobin (HbCO) is formed when CO combines with hemoglobin. This reaction
occurs at the same site as O 2 , which makes this hemoglobin molecule incapable of O 2 transport.
CO has about 200 times the affinity for hemoglobin as does O 2 ; therefore, HbeO is a rather
stable molecule.
a. The CO combines with the most reactive hemoglobin molecules, which are those molecules
that have the highest Pso' Therefore, the hemoglobin available for O 2 transport has a low
Pso (high O 2 affinity), which shifts the hemoglobin dissociation curve to the left and lowers
the tissue P0 2 even further than expected.
100

----I~~::~ruM
~PG

O~--~------T----------r-------------------------~

27

50

100

P~ (mm Hg)

Figure 3-16. The effect of increased CO 2 tension (Peo2 ), H+ concentration, temperature, and diphosphoglycerate
(OPC) concentration on the Pso of the hemoglobin saturation curve. The increase in the Pso indicates a reduced
affinity of hemoglobin for O 2 , which will raise the tissue O 2 tension (Po,). Decreases in these parameters cause a shift
of the curve to the left, which represents an increased affinity of the hemoglobin for O 2 ,

30

= 20 g/dl

HGB

= 15 g/dl,

HGB

= 10 g/dl

20

ON
<11

HGB

10

HbCO

5 g/dl

O~-=~-----------------.----------------------.---

50

100

POz (mm Hg)

Figure 3-17. O 2 content of arterial blood (Cao) as a function of O 2 tension (Po,) and hemoglobin (HCB) concentration. Note that carbon monoxide (HbCO) not only causes a reduction in O 2 content but also causes a decrease in
the Pso .

b. Because of the extreme affinity of CO for hemoglobin, a Pea of approximately 0.5 mm Hg


causes 50% of the hemoglobin to react with CO. The rapid loss of 50% of the functional
hemoglobin can be fatal.
c. Dissociation of CO from hemoglobin is enhanced by breathing 100% O 2 . Administration
of 100% O 2 provides an additional benefit, since significant amounts of O 2 (about 1 .5 vol%)
are dissolved in the plasma at this high Po 2 This dissolved O 2 helps to maintain an adequate
O 2 delivery to the body tissues.
d. HbCO gives a typical cherry-red color to the tissues.
5. Methemoglobin is formed when hemoglobin iron is oxidized from the ferrous (Fe 2 +) to the
ferric (Fe3+) state. Methemoglobin is incapable of carrying O 2 and has a bluish color that can
impart a cyanotic (blue) color to tissues. Methemoglobin is formed at a slow rate within the
red blood cell and is converted back to functional hemoglobin by the presence of reducing
compounds produced in the red cell by metabolic reactions. Many drugs and foods (e.g.,
nitrites, phenacetin, fava beans) can produce methemoglobin, especially in people who have
a genetic deficiency of glucose-6-phosphate dehydrogenase (G6PD).
6. Abnormal hemoglobins occur as genetic variants, which may have abnormal affinities for O 2
or abnormal physical characteristics, as in sickle cell anemia. Sickle cell hemoglobin (hemoglobin S) becomes very insoluble in the deoxygenated state. These molecules cause the red cells
to assume bizarre (sickle) shapes, causing them to lodge in capillaries and block the blood flow.
The blocked blood flow results in ischemic damage to many different organ systems in affected
individuals.
D. CO 2 transport. CO 2 is the usual end-product of oxidative metabolism and is formed in the tissues.
CO 2 diffuses out of the cells and into the capillary blood, which raises the Peo 2 of the capillary
and venous blood 5-6 mm Hg higher than the arterial Peo 2 . CO 2 is transported in the blood in
several different forms.

1. Physically dissolved CO 2 accounts for about 5% of the total CO 2 in the blood.


2. Carbamino-C0 2 refers to CO 2 that combines with terminal NH3 groups on protein side chains
to form NH 2 -COOH. Approximately 5% of the CO 2 is carried as carbamino compounds.
3. Bicarbonate. Approximately 90% of the CO 2 enters the red cell, where, in the presence of
carbonic anhydrase, it rapidly reacts with water to form carbonic acid (Figure 3-18). The
carbonic acid dissociates into bicarbonate (HC0 3 - ) and a hydrogen ion (H+). The HC0 3 diffuses into the plasma, whereas the H+ is buffered by the deoxygenated hemoglobin, which
is a weaker (less dissociated) acid than oxyhemoglobin. Although HC0 3 - is formed within the
red cell, most of the CO 2 is carried in the plasma as He0 3 - .
a. Chloride shift. As the HC03 - is formed in red cells at the tissues, it diffuses out of the cells
(see Figure 3-18). Since HC03 - is a negatively charged molecule, and the red cell membrane

Tissues ----.
Lungs~

CO 2

CI-

HC0 3 -

H+

+ HC0 3 -

CA

CO 2

- - -...~

+ H.o

H 2C0 3

- - - - - I....
......_ __

+
HGB - - - . H + + HGB

.....-

Red blood cell

Figure 3-18. The chloride shift. CO 2


tension increases in the blood as it
passes through the tissue capillaries,
which causes CO 2 to diffuse into the
red blood cell, where it is rapidly hydrated by carbonic anhydrase (CA).
The hydrogen ion (H+), derived from
the dissociation of carbonic acid
(H 2 C03 ), is buffered by the hemoglobin (HGB). Bicarbonate (HC0 3 - ) diffuses out of the red cell and is replaced by chloride (C/-)' Most of the
CO 2 picked up from the tissues is
transported in the plasma in the form
of bicarbonate.

is relatively impermeable to cations, as HC0 3 - diffuses out of the cell it causes the inside
of the cell to become less negatively charged. In order to neutralize this effect, the negatively
charged chloride ion (Ci-) diffuses from the plasma into the red cell to replace the HC03 - .
b. Reversal of these reactions occurs in the pulmonary capillaries (see Figure 3-18).
4. The Haldane effect refers to the fact that deoxyhemoglobin combines with more CO 2 compared to oxyhemoglobin at any Peo 2 The Haldane effect minimizes the increase in Peo 2 that
occurs in the venous blood, which, in turn, causes the venous blood to be less acid than it
would be otherwise.

V. PULMONARY CIRCULATION. The entire blood flow from the right ventricle normally is distributed to the pulmonary blood vessels. Since the output of the right heart must equal that of the left
heart, the pulmonary blood flow equals the cardiac output (approximately 5 L/min).
A. Pulmonary artery pressure is much lower than the pressure in the aorta, since the resistance to
blood flow in the pulmonary circulation is about one-tenth that of the systemic circulation. The
normal systolic pressure in the pulmonary artery is about 20 mm Hg, whereas the diastolic pressure
is about 10 mm Hg.
B. Flow distribution. Pulmonary blood flow varies in different parts of the lungs because of the low
pressures that are present, the distensibility of the vasculature, and the hydrostatic effects of gravity.
1. Hydrostatic pressure
a. Definition. Hydrostatic pressure is caused by the weight of the column of fluid and is
equal to

h x d x g
where h = height of the column of fluid (cm), d = density of the fluid (g/cm 3 ),
and g = the gravitational constant. The hydrostatic pressure adds to or subtracts from the
potential energy (pressure) at levels below or above the zero reference plane, respectively.
The zero reference plane is taken at the level of the atria that is approximately at the
middle of the lungs.
b. Example. Assuming the lungs to be 40 cm from base to apex, the pulmonary artery pressure
to be 20 cm H 2 0, and the density of blood to be 1 g/cm 3 , then the blood pressure would
be sufficient to lift the column of blood to a height of 20 cm. In this example, if the pulmonary
artery pressure decreased 4 cm H 2 0, then the top 4 cm of the lung would not be perfused,
since the pressure would not be adequate. In the upright position, the apex of the lungs
receives relatively little blood flow while the base of the lungs is well perfused.
c. Zones of the vertical lung. Various zones for the vertical lung (Figure 3-19) have been
defined as follows.
(1) Zone 1 is at the lung apex and is present if the regional pulmonary artery pressure (Pa)
is less than alveolar pressure (PA). Under these conditions, the pulmonary capillaries are
collapsed, and there is no blood flow (see Figure 3-19). Zone 1 is increased by factors
that reduce Pa or increase PA [e.g., positive end-expiratory pressure (PEEP)]; it is de-

Lung

Zone 1 PA > Pa

Cl

Q)
~

Cl

c:

::l

...J

.~~':1~.~ ~~.:: .~~\. ~~....


Pv
Zone 3 Pa > Pv :;:; PA
Zone4
..... .

Blood flow

Figure 3-19. The effect of hydrostatic pressure and level on the distribution of pulmonary blood flow in a vertical
lung. Pulmonary blood flow depends on the relationship between pulmonary artery pressure (Pa), alveolar pressure
(PA), and pulmonary venous pressure (Pv). (Adapted from West JB, et al: Distribution of blood flow in the isolated
lung: relation to vascular and alveolar pressure. J App/ Physio/19:713, 1964.)

creased by factors that increase Pa or reduce the vertical height of the lung (e.g., lying
down).
(2) Zone 2 is the region of the lung where Pa > PA > Pv (pulmonary venous pressure). In
this region, the pressure gradient producing flow is Pa - PA, not the arteriovenous
gradient, which is the usual mechanism. Since Pa increases 1 cm H 2 0 for each centimeter down the lung, there is an increase in the driving forces, and flow increases linearly
in zone 2 (see Figure 3-19).
(3) Zone 3 occurs in the lower part of the lung, where Pa > Pv > PA. Blood flow
increases down zone 3 as a result of distension and recruitment of vessels due to the
increasing hydrostatic pressure, which reduces the resistance to flow.
(4) Zone 4 is present only when there is an abnormally high pulmonary venous pressure
(e.g., due to left ventricular failure or mitral stenosis). Zone 4, when present, replaces
part of zone 3 at the base of the lung and represents a zone of reduced blood flow (see
Figure 3-19). Zone 4 results from edema occurring in the interstitium around blood
vessels, which is an early event in pulmonary edema. This edema is termed vascular
cuffing and results in an increased vascular resistance and a reduction in local blood
flow.
2. Hypoxic pulmonary vasoconstriction (HPV) is caused by alveolar hypoxia (Le., low alveolar
Po 2 ). HPV is a localized response in that it occurs only in areas of hypoxia; it is potentiated by
hypercapnia and acidosis.
a. The mechanism of this vasoconstriction is unknown, but it is important to remember that
hypoxia, hypercapnia, and acidosis produce vasodilation in the systemic circulation rather
than vasoconstriction as in the pulmonary circulation.
b. Function. HPV is an important mechanism of balancing ventilation and perfusion, since it
results in an increased vascular resistance in hypoxic areas of the lung, which shifts blood
flow to areas with a higher Po 2 If the hypoxia is generalized, as in hypoventilation or
exposure to high altitude, then HPV increases the total pulmonary vascular resistance, which
results in pulmonary hypertension.
c. Pulmonary hypertension increases the work of the right ventricle, resulting in right ventricular
hypertrophy, right axis deviation on the EKG, and, eventually, in right ventricular failure.
3. Changes in lung volume can alter markedly the pulmonary vascular resistance. Pulmonary
vascular resistance is minimal at approximately functional residual capacity and increases at
both higher and lower volumes. The effect of lung volume on pulmonary vascular resistance
is complex, because two different categories of blood vessels are involved.
a. Extra-alveolar vessels are the larger distributing arteries and veins in the lungs. These vessels
are dilated at high lung volumes because of the radial traction exerted on their walls by
connective tissue septa. The increased diameter reduces the flow resistance in these vessels.

b. Alveolar vessels, mainly capillaries, are located in the alveolar septa and are compressed
by the alveoli at high lung volumes. During inspiration, the capillary vascular resistance
increases while the larger vessels dilate. Thus, much of the increased right ventricular output
during inspiration is stored in the pulmonary arteries until expiration, when it reaches the
left ventricle. This alternate fluctuation in the stroke volume of the right and left ventricles
prolongs the respective ventricular ejection and represents the major factor that alters the
splitting of the second heart sound (see Ch 2 VI A 3 e).
4. Local reduction in blood flow results from the presence of diseases that can obstruct or
destroy pulmonary vessels and alter the normal flow pattern. In the presence of pulmonary
disease, the distribution of blood flow can be very uneven in adjacent areas of the lung rather
than having the predictable pattern as shown in Figure 3-19.

VI. THE DISTRIBUTION OF VENTILATION is uneven in the lungs and is affected by the pleural
pressure gradient, the time constant of the lung, and airway closure.
A. Pleural pressure gradient is caused by the effects of gravity on the lung within the intact thorax.
While the cause of this variation in pleural pressure is incompletely understood, the lung behaves
as if it were a low-density fluid (because of its gas content) when exposed to gravity. The density
of the normal lung is about 0.25-0.3 g/cm 3 , which produces a hydrostatic pressure difference of
7.5-10 cm H 2 0 between the apex and the base of a vertical lung. The interpleural pressure in a
standing adult increases from about -10 cm H 2 0 at the apex to about -2 cm H 2 0 at the base.
Astronauts, exposed to zero-gravity conditions, have neither a pleural pressure gradient nor regional variations in lung volume.
1. Regional lung volume is affected by the local interpleural pressure, since this is one of the
determinants of the transpulmonary pressure (Pl = PA - PPll. Since PA is zero throughout
the lung under static conditions, the Pl varies from 2 cm H 2 0 at the base to 10 cm H 2 0 at the
apex of the vertical lung. These two values of Pl place the apex and the base of the lung on
widely separated parts of the P-V curve, which causes the apex and the base to function at
very different compliances as shown in Figure 3-20.
2. Tidal volume is unevenly distributed in the erect lung because of the variation in regional
compliance due to the effects of gravity. In a vertical lung, most of the tidal volume goes to
the base of the lung when inspiration starts at functional residual capacity. There is a linear
reduction in regional tidal volume from base to apex under these conditions (see Figure 3-201.
The distribution of the tidal volume is much more even in a supine individual because of the
reduction in the interpleural pressure gradient.
B. Time constant of the lung (RC)-the product of resistance (R) and compliance (C)-sets the
time course for acinar filling and emptying. The functional unit of the lung is the acinus, which
consists of a respiratory bronchiole and all of the structures distal to it. The acinus can be modeled
by a balloon and a tube; the balloon provides compliance, and the tube provides airway resistance.
This type of system fills and empties exponentially; for each time constant the volume will change

100
PL

Apex---

= 0-

(-10)

= 10

Alveolus

e-t---\-PA = 0
e-t--+IPA = 0
~--+PA=

PL = 0 - (-2.5) = 2.5

0-2.5 10

-20

-30

Pleural pressure (em H20)

Figure 3-20. The transpulmonary pressure (PL) varies


from the apex to the base of the upright lung due to the
interpleural pressure gradient. The result is that basal
areas of the lung are located on a more compliant portion
of the pressure-volume curve than apical regions. During
inspiration the transmural pressure of the lung increases
and the lungs expand (volume at end-inspiration is indicated by arrow heads>. Basal regions take up more of the
tidal volume than apical regions, resulting in an uneven
distribution of alveolar ventilation. ~V = change in volume; and PA = alveolar pressure.

8. "
~

(5

>

Figure 3-21. Models of acini showing the effect of nor-

--------~-----------

0)

I
I
I
I

Inspiration

Expiration

Time

mal compliance (A), decreased compliance (8), and high


airway resistance (C) on the time course of filling. With a
high airway resistance, the acini fill slowly, and the volume change varies with the duration of inspiration. A
decreased compliance results in rapid filling but a reduction in the volume change.

63% toward the equilibrium value (Figure 3-21). The normal lung behaves as if all acini have the
same time constant. Variations of the time constant due to disease can cause an even more unequal
distribution of the tidal volume than the effect of the plural pressure gradient.

1. An increased RC is caused primarily by an increased airway resistance and results in a slower


rate of acinar filling and emptying (see Figure 3-21 C).

('V E) for
any driving pressure (dP): \IE = dP/R. The presence of lung segments having increased RC
results in the phenomenon of frequency-dependent compliance, since the extent of lung
filling varies as a function of inspiratory time. When respiratory rate increases, lung filling
decreases due to insufficient filling time, and the dynamic lung compliance decreases.
b. The presence of frequency-dependent compliance indicates the presence of obstructive
airway disease rather than parenchymal or restrictive lung disease as the name implies.
a. The slower rates are caused by the high resistance (R), which reduces the flow rate

2. A decreased RC results primarily from a decreased lung compliance due to lung fibrosis or
destruction of functional lung units. The decreased compliance allows these units to fill and
empty rapidly but to a smaller extent than normal (see Figure 3-21 B). The decreased compliance
alters the distribution of the tidal volume and markedly increases the work of expanding the
lungs.

C. Airway closure occurs when the regional transpulmonary pressure is reduced to a critical level.
Transpulmonary pressure varies primarily as a function of lung volume. Closing volume is the
volume of the vital capacity that is present when airway closure begins. Closing capacity is the
closing volume plus the residual volume.

1. The single breath N2 test can be used to detect airway closure. In this test, an individual
expires to residual volume, then inspires to total lung capacity using 100% O 2 and, finally,
expires slowly and steadily to residual volume. The PN 2 in the expired gas is measured during
the second expiration and is depicted in Figure 3-22, which shows four distinct phases.

200
3

i.'
Il..
"0

100

a.
x
w

TLC

RV
Vital capacity (L)

Figure 3-22. Record of a single-breath nitrogen (N,) test.


The tracing shows the expired N, tension (PN2 ) following
an inspiratory vital capacity of 100% 0,. CV = closing
volume; TLC = total lung capacity; and RV = residual
volume.

a. Phase 1 of the expired PN 2 represents the first gas from the lungs, which is pure dead space
gas (100% O 2 ),

b. Phase 2 is a mixture of gas from the dead space and alveoli and represents a transition
phase. The volume expired at the midpoint of this phase represents the anatomic dead
space. Measuring the dead space in this fashion is referred to as Fowler's method.
c. Phase 3 represents alveolar gas and is referred to as the alveolar plateau. The slope of
phase 3 varies as a function of the unevenness of tidal volume distribution.

d. The onset of phase 4 (terminal rise) indicates the closing volume (onset of airway closure).
The higher PN 2 in phase 4 is due to the loss of the dilution effect of gases coming from the
base of the lung. Recall that, at residual volume, the base of the lungs is almost completely
deflated, while the apex is moderately inflated with gas containing 80% N2 After inspiring
100% O 2 to total lung capacity, the base is maximally inflated by almost pure O 2 while the
apex is maximally inflated but contains significant amounts of N 2 During the subsequent
expiration, when basal airways begin to close, only gas from the apex continues to be
expired, which has a higher PN 2
e. Closing capacity is the closing volume plus residual volume.

2. Early airway closure is considered to occur when airways close prematurely compared to the
predicted value for a person's age and sex. Airway closure begins at the base of the lung where
transpulmonary pressure and airway diameter are smallest. Early airway closure occurs in the
presence of airway disease, excess mucus, contraction of bronchial smooth muscle, or
loss of radial traction on the airways due to destruction of alveolar septa. In other words,
anything that narrows the airways leads to early airway closure.

VII. VENTILATION:PERFUSION RATIO (VA/Q) is merely the ratio of the alveolar ventilation to
the pulmonary blood flow. The VAtO at the acinar level determines the alveolar P0 2 and Peo 2 and
is, therefore, critical in determining the gas exchange in each alveolus of the lung. The most efficient
gas exchange occurs with VA/O ratios of about 1.0. Abnormal VAtO (either too high or too Iowa
ratio) is by far the most common cause of hypoxia, which can result from many different pulmonary
diseases.

A. Normal VAtQ
1. Recall that the alveolar ventilation is normally 4.5-5.0 Ltmin and that the cardiac output is
about 5 Ltmin. If the alveolar ventilation and pulmonary blood flow were both evenly distributed throughout the lungs, then the VA/O would be about 0.9.
2. Even in normal lungs, both ventilation and blood flow vary in different regions (low at the apex
and high at the base of a vertical lung). Due to the higher density of the blood compared to
the lung tissue, the change in blood flow is much greater than the change in ventilation per
unit distance (Figure 3-23). Thus, the lung base is overperfused compared to the ventilation,
while the apex is underperfused compared to the ventilation; the base of the lung has a low
VA/O while the apex has a higher VA/O.
3. The VAtO normally ranges from about 0.6 at the base to about 3 at the apex of the lung. This
relatively narrow range impairs gas exchange only slightly. In the presence of pulmonary
disease, the VAtO can vary from 0 to infinity.

0.15

ilA or Q

VL

0..10.

0..05

0.

20.
Distance (cm)

40.

Figure 3-23. Diagram showing the distribution of alveolar ventilation (VA), pulmonary blood flow (Q), and the
ventilation:perfusion ratio (VA/QJ in a normal lung. Because the blood flow gradient is steeper than the ventilation gradient, the base of the lung is overperfused, and
the apex of the lung is overventilated. Thus, the VA/Q is
low at the base and high at the apex of the normal,
vertical lung. % VL = percent of the regional lung volume.
(Adapted from West JB: Ventilation/Blood Flow and Cas
Exchange. Oxford, Blackwell Scientific, 1970, p 33.)

B. Low VA/Q is caused either by a low rate of ventilation or by excessive blood flow to an area
of the lung (Figure 3-24A). Either condition causes the alveolar P0 2 to decrease and Pe0 2 to
increase, since an inadequate amount of O 2 is brought into the lungs by ventilation, compared
with the amount of O 2 that is carried away by the pulmonary capillary blood.

1. Due to the low alveolar P0 2, the blood leaving this area of the lungs also contains a low P0 2
and consequently has a decreased O 2 content. When this blood mixes with blood from
well-ventilated areas, the arterial O 2 content is reduced, which results in hypoxemia. The blood
flow from low VA/Q areas is termed a physiologic shunt.
2. If there is no alveolar ventilation (VA/Q = 0), then there is no gas exchange at all, and
the blood flow is referred to as an anatomic shunt. An anatomic shunt may be intra- or
extrapulmonary. Extrapulmonary anatomic shunts may be the result of congenital heart malformations resulting in a right-to-Ieft shunt of blood. The presence of an anatomic shunt means
that true venous blood is mixing with arterial blood, and even the administration of 100% O 2
will not correct this decrease in the arterial P0 2.

l. The fraction of shunted blood (Qs/QtJ can be measured by means of the shunt equation
Qs/Qt
where Ci0 2
content.

Ci0 2 - Ca0 2/Ci0 2 - Cv0 2

ideal O 2 content, Cv0 2

mixed venous O 2 content, and Ca0 2

arterial O 2

C. High VA/Q is caused by excessive ventilation or inadequate blood flow (see Figure 3-240.
1. The P0 2 in these areas of the lung is high, but O 2 exchange is inefficient for the level of ventilation
because very little extra O 2 is added to the blood due to the plateau of the 02-dissociation curve
(see Figure 3-15). In addition, O 2 exchange is impaired since the high VA/Q is commonly due
to a low blood flow so that there is little blood flow to carry the O 2 away. The extra ventilation
needed to raise the P0 2 above normal can be considered ventilation of alveolar dead space.
2. If the blood flow to an area of the lung is zero (VA/Q = infinity), then there is no gas exchange
in these alveoli, and all of the ventilation represents alveolar dead space ventilation. Any gas
going to alveolar dead space is wasted and contributes to dead space ventilation.
D. The 02-C02 diagram (Figure 3-25) is one method of visualizing the effects of ventilation-perfusion
alterations on the blood gas tensions. This graph contains a V A/Q line that is defined by
three points representing the venous blood composition (v), ideal alveolar gas composition (i), and
the inspired gas composition (I).

1. R Lines represent the P0 2 and Pe0 2 that would be present under conditions where the rates
of O 2 uptake and CO 2 release vary. R is the ratio of the volume of CO 2 released per minute
(Ve0 2) to the volume of O 2 absorbed per minute (V0 2). R lines can be determined for both the
gas and blood phase, and these lines intersect the VA/Q line for each respective value of R.
a. Gas R lines radiate from the inspired gas point and represent the change in gaseous O 2 and
CO 2 tensions for a particular R value.
b. Blood R lines radiate from the venous blood point and represent blood gas tensions for
various R values.
2. Alveolar dead space can be represented in the gas R line as a movement of the alveolar gas
point away from the ideal point. The alveolar gas point represents the mean value for all
alveoli, so that addition of alveolar dead space, with a gas composition equal to the inspired
gas, moves the mean alveolar value toward the inspired gas point. Unless the overall alveolar
A. Shunt

B. Normal

C. Alveolar dead space

AAA
VA

=0

VA = a

VA/a

=0

VA/a = 1

=0

VA/a =

00

Figure 3-24. Models depicting the


effect of variations in alveolar ventilation (VA) and blood flow (0) on the
ventilation:perfusion ratio. Areas of
the lung with reduced alveolar venti-

lation result in a shunt effect or venous admixture (A), while other areas
with reduced blood flow contribute
to alveolar dead space (C).

Gas R line

40

Blood R line

OJ

'\
'\

.......-..................

'\ I

30

EON

20

U
Il..

10
0
0

40

80

120

P0 2 (mm Hg)
Figure 3-25. Ventilation:perfusion line, which is determined by the composition of mixed venous blood (ii) and
inspired gas (I). The arterial blood gas composition (point a) is determined by the amount of shunt flow, while the
mixed-expired gas (point f) is determined by the amount of dead space. Point i represents the average gas composition
of the pulmonary capillary blood. R lines represent the ratio of carbon dioxide production to oxygen consumption,
and the blood and gas R lines meet at the VA/Q line.
ventilation is increased, an increase in alveolar dead space results in CO 2 retention as well as
hypoxia. In the typical patient with increased alveolar dead space, the initial phase of hypercapnia stimulates the respiratory centers, so that minute ventilation increases, and the Peo 2 returns
to normal, while the hypoxia remains due to the effects of a physiologic shunt.
3. A physiologic shunt causes the arterial blood gas composition (see Figure 3-25, point a) to
move along the blood R line toward the venous point. The more severe the VA/Q abnormality,
the further the arterial point will move to the left. Note that there is a marked reduction in P0 2
with only a minimal rise in the Peo 2 as one proceeds along the blood R line. This is typical of
the blood gas values obtained from patients with VA/Q abnormalities.
E. Distribution of VA/Q ratios in the lungs normally can be represented by a log-normal distribution
<;urv~ (Figure 3-26A) having a range that covers about 1 decade of VA/Q values. Patients with
VA/Q abnormalities may have a single mode of distribution with a much wider range of values
than normal (see Figure 3-26B), or there may be multiple modes of distribution (see Figure 3-260.
Either of these abnormalities causes a worsening of the blood gas values.

VIII. RESPIRATORY CONTROL


A. Neural mechanisms. Unlike the heart, which automatically generates its own rhythm, the respiratory muscles are typical skeletal muscles requiring electrical stimulation via somatic nerves to
initiate contraction. The major muscle of inspiration, the diaphragm, is innervated by motor fibers
in the phrenic nerve. Impulses reach this nerve from either voluntary or involuntary paths in
the central nervous system (CNS). This dual pathway allows voluntary breath holding or control
of breathing during talking, singing, or swimming. The involuntary pathway functions during most
of the day and night, allowing humans to breathe automatically without conscious effort. (Hiccups
represent involuntory spasms of the diaphragm frequently caused by irritation from surrounding
structures such as the stomach or gallbladder.)

1. Medullary centers. The automatic basic rhythm of respiration is generated within the medulla,
but its exact source and mechanism of generation is unknown. Respiration continues in animals
and humans as long as the medulla and the spinal cord are intact. Patients who are "brain
dead" and require a respirator to breathe have destroyed the medullary centers as well as the
higher brain functions. There are two groups of neurons on each side of the medulla that
contain cells that discharge rhythmically during either inspiration or expiration. These groups
of cells are called the ventral respiratory group (VRG) and the dorsal respiratory group
(DRG). These centers are influenced by nervous activity in other areas of the brain such as the

'E

~
~
0

u:::
0

Shunt

0.001

0.01

0.1

10

0.1

10

\//>,/0
B
3

'E

~
~

u:::
0

Shunt

0.001

0.01

VA/a

c
3

'E

~
~
0

u:::
0

Shunt

0.001

0.01

VA/a

0.1

Figure 3-26. Distribution of VA/Q


ratios. The y axis represents the
amount of blood flow going to each
VA/Q value. (A) Normal, showing a
mean VA/Q of 1 and a standard deviation of the VA/Q population of 0.2,
which results in a variation ofthe ratio
from about 0.6 to 3. (8) An increased
standard deviation of the population
results in a wider and shorter pattern
of flow distributions, while (0 shows
a bimodal distribution of the flow.
10 Both Band C produce a decrease in
the arterial Po 2

pons, the reticular activating system (RAS), and the cerebral cortex as well as by afferent
activity in the vagus, glossopharyngeal, and somatic nerves.
a. The ORG is a bilateral group of cells that lies within the nuclei of the tractus solitarius. The
DRG cells are primarily inspiratory cells (Le., they discharge during inspiration).
(1) Activity of the ORG. The DRG may be the primary rhythm generator for respiration,
since the activity in these cells gradually increases during inspiration. The electrical
activity of this center has been likened to a ramp, since the activity rises to a crescendo
during inspiration and then rapidly disappears.
(2) Afferent input to the ORG comes mainly from the vagus and glossopharyngeal
nerves, which carry information from the peripheral chemoreceptors and mechanical
receptors in the lungs. The DRG activity is influenced through these nerves by low Po 2 ,
high Peo 2 , and low pH as well as by changes in lung volume and the neural activity
within the RAS. During sleep, the RAS activity is diminished, which results in a lessened
inspiratory drive, a decrease in alveolar ventilation, and a slight rise in arterial Peo 2
(3) Efferent connections from the ORG go to the contralateral phrenic and intercostal
motoneurons and to the VRG.
b. The VRG contains neurons that are active during both inspiration and expiration. The
expiratory activity does not result in activation of the expiratory muscles during normal

respiration (eupnea), since expiration is normally passive. The VRG is comprised of the
upper motor neurons of the vagus and the nerves to the accessory muscles of respiration.
2. Pontine centers are areas of the brain stem that modify the activity of the medullary respiratory
centers.
a. The pneumotaxic center lies in the upper part of the pons and functions to inhibit the
apneustic center. Stimulation of the pneumotaxic center shortens inspiration, leading to a
shallower and more rapid respiratory pattern.
b. The apneustic center lies in the caudal area of the pons, but it has not been identified
with any specific collection of cells. The efferent outflow from the apneustic center stimulates
inspiration. Stimulation of this area of the brain increases the duration of inspiration,
which results in a deeper and more prolonged inspiratory effort. Consequently, the rate of
respiration becomes slowed because of the greater depth of inspiration. Loss of inhibitory
activity on the apneustic center, caused by vagotomy and destruction of the pneumotaxic
center, results in prolonged periods of inspiration termed apneusis. The apneustic center is
normally inhibited by impulses carried in the vagus nerves and also by the activity of the
pneumotaxic center.
3. Central chemoreceptors are cells that lie just beneath the ventral surface of the medulla and
that respond to the H+ concentration in the cerebrospinal fluid (CSF) and the surrounding
interstitial fluid.
a. Charged ions do not readily cross the endothelium of the blood vessels in the brain, which
constitutes the blood-brain barrier (Figure 3-27).
b. However, CO 2 does cross this barrier, since it is a small uncharged molecule. An increase
in CO 2 stimulates the central chemoreceptors after hydration to carbonic acid and dissociation into H + and HC03 - . A decrease in CO 2 inhibits respiration, since there is a reduction
in the H+ concentration within the CSF and the brain tissue.
c. About 85% of the resting ventilatory drive is due to the stimulatory effect of CO 2 on the
central chemoreceptors.
B. Peripheral chemoreceptors are located in the carotid and aortic bodies. These receptors respond
to lowered Po 2 , increased Peo 2 , and increased H+ concentration in the arterial blood. The
peripheral chemoreceptors receive a tremendous blood flow for their size and thus can be considered to monitor the P0 2 of arterial blood. Decreases in the O 2 content of blood caused by
anemia, methemoglobinemia, and carbon monoxide poisoning do not stimulate the peripheral
chemoreceptors, since the Po 2 , which is determined by the amount of dissolved O 2 , remains
normal.
1. O 2 The peripheral chemoreceptors are the only site in the body that detect changes in the
Po 2 These receptors rapidly increase their firing rate as the arterial P0 2 falls below 100 mm
Hg. Impulses from these receptors are carried to the brain via the vagal and glossopharyngeal
nerves and result in an increased rate and depth of respiration.
2. Decreased pH (increased H+ concentration), caused by the addition of fixed acids (e.g.,
lactate, acetoacetate, butyrate) into the blood, also stimulates the peripheral chemoreceptors
and ventilation increases. The increased alveolar ventilation lowers the Peo 2 in the arterial
blood and reduces the amount of acid in the blood, which tends to return the arterial pH
Brain

H2 O

CO 2

H 2 C0 3

HC0 3 -

~
%
%
%
%
%
%
%
%
%
%
/-

H+---+-~
Central
chemoreceptor

Blood-brain
barrier

Blood
H+
CO 2

Figure 3-27. The effects of the


blood-brain barrier on the transport
of charged ions such as H+ and the
diffusibility of CO 2 CO 2 can readily
cross the blood-brain barrier and
serve as a source of H+ to stimulate
the central chemoreceptors.

toward normal. Acutely, the fixed acid cannot cross the blood-brain barrier, so that the resultant
decrease in Pco 2 results in an alkalosis in the CSF. This central alkalosis inhibits respiratory
drive for several days until CSF pH is returned to normal by ion transport by the meningeal
tissues. The respiratory response to a low arterial pH is termed respiratory compensation.
3. CO 2 also stimulates the peripheral chemoreceptors, but its major effect is on the central
chemoreceptors as explained above.

C. Other respiratory reflexes


1. The Hering-Breuer reflex is initiated by inflation of the lungs and functions to terminate
inspiration. Inflation of the lungs stimulates stretch receptors that are located in the small
airways. These receptors send impulses to the respiratory centers via the vagus nerves that
inhibit the pontine and medullary respiratory centers.
2. Irritant receptors are located in the large airways and are stimulated by smoke, noxious
gases, and particles in the inspired air. These receptors initiate reflexes that cause coughing,
bronchoconstriction, mucus secretion, and breath holding (apnea).
3.

J receptors are stimulated by distension of the pulmonary vessels as in heart failure and
pulmonary embolization. These receptors initiate reflexes causing rapid breathing (tachypnea).

4. Chest wall receptors can detect the force generated by the respiratory muscles during breathing. If the force required to distend the lungs becomes excessive (high airway resistance or low
compliance) the information from these receptors gives rise to the sensation of dyspnea
(difficulty in breathing).
D. Respiratory function tests. There are many tests that can be used to evaluate the function of the
respiratory system. Some of the most commonly used tests are summarized below.
1. Maximal voluntary ventilation represents the maximum volume of gas that a patient can
breathe. This test is very fatiguing and depends on a patient's complete cooperation (thus the
term "voluntary" in the name). The patient breathes as rapidly and forcefully as possible while
the volume changes are recorded with a spirometer. The test is carried out for 12 seconds, and
the results are expressed in Llmin. A normal value for a young adult male is about 200 L/min.
The test evaluates all of the mechanical factors in breathing and results in the highest rate of
ventilation for any condition.
2. Respiratory responses to CO 2 Gases containing different amounts of O 2 or CO 2 can be
administered to evaluate the respiratory drive of the respiratory system. Increasing the Pco 2 in
inspired gas causes a rise in the minute ventilation. A decrease in P0 2 is synergistic with
hypercapnia, so that the respiratory response is much greater than with either stimulus alone
(Figure 3-28). Patients vary in their response to this test, depending on, for example, the
50

o l
o

30

Figure 3-28.
obtained at
(PA0 2 ) of 50,
Hg. PAC02 =

Cal response curves


alveolar O 2 tensions
100, and > 500 mm
alveolar CO 2 tension.

Chronic hypoxia

Normal

Asleep

~ 20

2-

c 15
0

CD

>

Opioid

~overdose

C 10

.l!l
::J

Increased
resistance

0
0

25

30

40
Pc02 (mm Hg)

50

Figure 3-29. The effect of various


pathologic conditions on CO2 response curves.

sensitivity of their chemoreceptors, activity of the reticular activating system, airway resistance,
and lung volume (Figure 3-29).
3. Forced vital capacity is a vital capacity maneuver performed with maximal effort. Many
different measurements can be performed on the spirometer recording that is obtained. A
decrease in the FEV, can be caused by either a high airway resistance or a lack of expiratory
muscle strength. These two conditions can be differentiated by measuring the maximal expiratory force. The patient with a high airway resistance can generate a normal expiratory effort
of about 100 cm H 2 0, with the low flow rate due to the high airway resistance. In contrast,
the patient with expiratory muscle weakness will be unable to generate a normal maximal
expiratory pressure.
4. Other tests. Measurement of a patient's diffusion capacity, lung volumes (e.g., FRC or VC),
and the concentration of O 2 and CO 2 in the arterial blood can provide physicians with valuable
information about respiratory function.
E. Dysfunction of respiratory control can occur due to changes in the environment or to diseases
affecting the respiratory system, cardiovascular system, or brain. Various respiratory patterns are
seen clinically (Figure 3-30).

1. Cheyne-Stokes respiration is an abnormal respiratory pattern that occurs with depression of


the brain due to disease, drug overdose, congestive heart failure, and hypoxia from other
causes. Cheyne-Stokes respiration is characterized by periods of waxing and waning tidal
volumes separated by periods of apnea.
2. Biot's breathing is another type of periodic breathing that consists of one or more large tidal
volumes separated by periods of apnea. The condition occurs in many diseases producing brain
damage.
Respiratory patterns
Cheyne-Stokes
respiration
Biot's
breathing
Kussmaul's
respiration
Apneustic
breathing
Normal
Time

Figure 3-30. Illustration of various breathing patterns.


Vr = tidal volume.

3. Ondine's curse gets its name from Greek mythology. Ondine was a water nymph who fell in
love with a human. Ondine's father was king of the nymphs and placed a curse on the man
that took away all of his automatic functions. Presumably, the man spent all of his time willing
his heart to beat and his muscles to breathe and had little time for his lover. Automatic
respiratory control is lost when there is a destruction of the involuntary neural pathways.
Patients with this condition can breathe only by conscious effort and, therefore, cannot sleep
without becoming apneic. This condition is treated with a mechanical respirator that maintains
breathing while the patient sleeps.
4. Sleep-apnea syndromes have been recognized as disorders of respiratory control that affect
large numbers of the population, especially elderly men. Many sleep centers have been established to study patients by recording physiologic parameters such as respiratory movements,
air flow, EEG, and hemoglobin saturation during sleep.
a. Obstructed apnea
(1) Some individuals experience a marked loss of muscle tone in the pharyngeal muscles
during REM (rapid eye movement) sleep. The loss of muscle tone results in partial or
complete obstruction of the pharynx during inspiration. Partial obstruction is associated
with snoring, as the inspired air causes these tissues to vibrate. During complete airway
obstruction, there are contractions of the respiratory muscles, but no air can move
because of the blocked airway. The person usually awakens because of the hypoxia
that develops; muscle tone is reestablished, and sleep returns. These episodes may
occur hundreds of times each night and are associated with a marked drop in the
hemoglobin saturation of the arterial blood. The person wakes up in the morning still
tired and unrested because of the poor sleep pattern. Frequently, these people fall asleep
at work, in lectures, or even while eating because of the lack of restful nighttime sleep.
(2) When this condition is associated with extreme obesity, it is referred to as the Pickwickian syndrome (read Dickens' Pickwick Papers to find out which character was affected).
(3) Severe cases may require a tracheostomy in order to bypass the airway obstruction.
b. Nonobstructed (central) apnea refers to a complete cessation of respiratory muscle activity
due to a loss of rhythmic activity from the respiratory centers. These periods of apnea can
last 30-60 seconds, with a marked drop in arterial hemoglobin saturation. Affected persons
and some of their family members have been shown to have a decreased chemoreceptor
sensitivity to O 2 and CO 2 Sleep-apnea syndrome has been proposed as one of many
possible causes of sudden infant death syndrome (SIDS, or crib death).

IX. HYPOXIA is defined as an inadequate O 2 supply to the body tissues. The O 2 supply can be limited
anywhere along the O 2 pathway from the atmosphere to the body tissues. Hypoxia will occur
downstream (toward the tissues) of the limitation, whereas normal P0 2 may be present upstream
(toward the environment).
A. Symptoms of hypoxia depend on how fast and how severely the P0 2 declines.
1. Fulminant hypoxia occurs within seconds upon exposure to P0 2 less than 20 mm Hg. This
could happen if an aircraft lost cabin pressure at altitudes above 30,000 feet and there were
no supplemental O 2 available. It also could happen if the O 2 were consumed by combustion
in a closed space or was displaced by some other gas. Unconsciousness results in as little as
15-20 seconds, and brain death may follow in 4-5 minutes.
.
2. Acute hypoxia is produced by exposure to P0 2 equivalent to altitudes of 18,000-25,000 feet.
Symptoms of acute hypoxia are very similar to the effects of ethyl alcohol; there is incoordination, slowed reflexes, slurred speech, overconfidence, and eventually unconsciousness. Coma
and death can occur in minutes to hours if the compensatory mechanisms of the body are not
adequate.
3. Chronic hypoxia occurs during exposure to arterial P0 2 of 40-60 mm Hg for long periods of
time. Most clinical causes of hypoxia fall into this category. Chronic hypoxia produces symptoms similar to those of severe fatigue, and there is a sensation of difficulty in breathing
(dyspnea) and shortness of breath. These patients may be bedridden or limited to sitting in a
chair, because they are unable to increase their O 2 supply to the tissues due to either respiratory
or cardiac disease. Respiratory arrhythmias (see VIII E) can occur in these people, especially
during sleep, which can contribute to the hypoxic state.

B. Signs of hypoxia

1. Cyanosis is the bluish color of tissue caused by the presence of more than 5 g of deoxyhemoglobin/dl in the capillary blood. Patients with anemia may never develop cyanosis because of
their inadequate hemoglobin concentration. Methemoglobin, because of its slate gray color,
also can cause cyanosis. Cyanosis is most readily seen in the nail beds, lips, mucous membranes, and ear lobes, but it may not be recognized because of skin pigmentation or poor
lighting. Cyanosis is not a reliable sign of hypoxia.

2. Tachycardia (rapid heart rate) occurs as a reflex response to the low P0 2 in the arterial blood.
The hypoxia is detected by the aortic and carotid chemoreceptors, which then activate the
sympathetic outflow to the heart. The increased heart rate is an attempt to increase cardiac
output to enhance the O 2 delivery to the tissues.
3. Tachypnea (rapid breathing) and hyperpnea (deep breathing) also are reflex responses to
hypoxia, which are activated by the arterial chemoreceptors. The resultant increase in minute
ventilation is an attempt to raise the alveolar P0 2 toward its normal value of 100 mm Hg.

C. Causes of hypoxia. All types of hypoxia reduce the amount of O 2 that is available to the body
tissues. The O 2 delivery is a measure of the amount of O 2 that is available to the body and is
calculated as the product of cardiac output (l/min) and O 2 content of the blood/L. The normal
O 2 delivery is about 1000 ml 02/min (5 l/min x 200 ml 02/U, Not all types of hypoxia cause
a decrease in the O 2 delivery. Table 3-5 provides the characteristics of the different types of
hypoxia and various tests that can be used to differentiate the types of hypoxia.

1. Arterial hypoxia (Figure 3-31 A) results from inadequate oxygenation of the arterial blood,
which is caused by breathing gas with a low P0 2 or by one of four pathophysiologic mechanisms. Notice in Figure 3-31 that arterial hypoxia is the only type of hypoxia in which trere
is a decreased arterial Po 2 Arterial hypoxia is due to impaired gas exchange in the lungs.
Pulmonary gas exchange is evaluated by using the alveolar-to-arterial P0 2 difference. A
wide alveolar-to-arterial P0 2 indicates impaired pulmonary gas exchange and is caused by a
decreased arterial Po 2.
a. Hypoventilation results from an inadequate rate and/or depth of respiration. There is a
reduction in both alveolar and arterial P0 2 and an increase in alveolar and arterial Peo 2
(hypercapnia) because of the inadequate alveolar ventilation. Hypercapnia is synonymous
with hypoventilation because of the relationship between arterial Peo 2 and alveolar ventilation.
b. Diffusion limitation occurs if there is a severe loss of surface area of the alveolar membrane
or a large increase in the diffusion distance in the lungs due to pulmonary disease. These
changes reduce the amount of O 2 that diffuses into the pulmonary capillary blood. Consequently, the P0 2 in the pulmonary capillaries remains below the alveolar P0 21 which results
in the abnormally low arterial Po 2.
c. Ventilation:perfusion ratio imbalance (physiologic shunt) produces low P0 2 in areas of
the lung with low VA/Q ratios. When the blood leaves these areas of the lungs and mixes
with blood from other regions of the lung, the overall arterial O 2 content and P0 2 are

Table 3-5. Differentiating Types of Hypoxia


Type of Hypoxia
Arterial hypoxia
Hypoventilation
Diffusion
Ventilation:perfusion
Anatomic shunt
Anemic hypoxia
Hypokinetic (ischemic) hypoxia
Histotoxic hypoxia
Pa0 2 = arterial O 2 tension; Paco 2
decreased; + = increased.
*Critical characteristic.

- =

Pao 2

Paco 2

n
n
n

++*
n
n/n/n
n
n

Pvo 2

Exercise Pao 2

100% O 2

+/-

CO 2++
n
n
<500 mm Hg*
n
n
n

*
+/+/n
+/+/-

*
*
+*

arterial CO 2 tension; PV0 2

mixed venous O 2 tension; n

normal;

B. Hypokinetic hypoxia

A. Arterial hypoxia

a
20

20

"'0

"'0

:c

:c

0
0

0
0

"'N

=0
0

=0
"'N
0

E
0

0
0

40

100
PO. (mm Hg)

D. Histotoxic hypoxia
a

C. Anemic hypoxia
20
"'0
0
0

:c
~

.-

..

40
100
PO. (mm Hg)

------

2
"'0
0
0

:c

=0
"'N
0

0
0

40

100
PO. (mm Hg)

100
40
PO. (mm Hg)

Figure 3-31. Oxyhemoglobin dissociation curves illustrating the four types of hypoxia; a = arterial point and v =
mixed venous point. Curve C shows a normal curve (15 gldl of hemoglobin) as the dashed line, whereas the solid
line represents the effect of 7.5 gldl of hemoglobin. Note that the arterial P0 2 is below normal only in arterial hypoxia,
whereas the mixed venous point is below normal in all cases except in histotoxic hypoxia, where it is increased.

reduced. Administration of 100% O 2 to these people can correct the hypoxia; this fact can
be used as a diagnostic test to separate VA/Q imbalance from anatomic shunts.
d. Anatomic shunts are caused by blood flow that is never exposed to ventilated alveoli so
that true venous blood enters the systemic (arterial) circulation. Anatomic shunts can occur
through abnormal connections between the right and left sides of the heart caused by
congenital heart disease, or it may represent blood flow through atelectatic (completely
collapsed) areas of the lungs. The shunted blood represents venous blood that is dumped
into the arterial circulation and results in a dilution of the normally oxygenated blood. Since
the shunted blood is never exposed to ventilated alveoli, the administration of 100% O 2
does not raise the arterial P0 2 to its maximal levels.
2. Hypokinetic (ischemic) hypoxia is due to an inadequate blood flow. The arterial P0 2 and
content may be normal, but, because of inadequate blood flow, the tissues withdraw more O 2
from each unit of blood, so that the venous O 2 content is decreased (see Figure 3-31 B). The
reduced blood flow may involve the whole body, as in congestive heart failure, or it may
involve only a localized area of the body due to abnormalities in a regional blood vessel. The
most common cause of arterial obstruction is arteriosclerosis, where deposits of cholesterol
and other lipids in the endothelium cause narrowing of a vessel's lumen, which reduces the
blood flow. If this occurs in the coronary arteries, it can cause a myocardial infarction (heart
attack), since a portion of the heart muscle dies from lack of O 2 , This condition is fatal if a large
portion of the heart muscle is affected, and cardiac output declines or arrhythmias occur.

3. Anemic hypoxia is caused by an insufficient amount of functional hemoglobin. The decrease


in functional hemoglobin may be caused by deficiency of essential nutrients such as iron (iron
deficiency anemia), or it may be due to the presence of abnormal amounts of methemoglobin
or carboxyhemoglobin. Patients with anemic hypoxia have a reduced O 2 capacity and consequently a decreased O 2 content, whereas the arterial P0 2 remains normal (see Figure 3-31 C).
4. Histotoxic hypoxia is caused by an inactivation of certain metabolic enzymes by chemicals
such as cyanide. if these enzymes are not functioning, the tissues are unable to use O 2 , even
though there may be adequate O 2 in the tissues. in this condition, the O 2 delivery remains
normal, and the venous P0 2 and O 2 content are high because the O 2 is not being consumed
by the tissues (see Figure 3-31 D).
D. Physiologic responses to chronic hypoxia. The body has many ways to compensate for decreases in the O 2 supply to the body.

1. Accommodation represents the immediate adjustments by the body to hypoxia. These mechanisms involve reflex adjustments of the cardiovascular and respiratory systems, whereas longterm adjustments can involve structural changes in the tissues.
a. Hyperventilation occurs secondary to a stimulation of the peripheral chemoreceptors due
to the low P0 2 in the arterial blood. The increased ventilation reduces alveolar Peo 2 , which
raises alveolar P0 2 proportionately. The reduced Peo 2 results in an alkalosis, which, in turn,
lowers the respiratory drive. The alkalosis is slowly corrected by renal mechanisms over 12 weeks, so that respiration may continue to increase as the alkalosis is corrected.
b. Tachycardia is a reflex response to carotid body stimulation by hypoxia. In humans, the
cardiac output returns to normal after spending several weeks at high altitude.
c. Increased diphosphoglycerate (DPG) concentration occurs during either hypoxia or alkalosis. The increased DPG concentration increases the Pso of the hemoglobin, which helps
to maintain tissue P0 2 at slightly higher levels than it would be otherwise.
2. Acclimatization refers to changes in the body tissues in response to long-term exposure to
hypoxia.
a. Polycythemia is an abnormally high number of red cells/microliter of blood. This condition
is also diagnosed if the hemoglobin level exceeds 18 g/ dl in males or 16 g/ dl in females or
if the hematocrit exceeds 50%. The polycythemia usually is secondary to tissue hypoxia,
which causes the release of renal erythropoietic factor, which acts on a plasma globulin to
form erythropoietin. Erythropoietin stimulates the production of erythrocytes by the bone
marrow, which eventually increases the number of circulating erythrocytes and the hematocrit. The increased number of red cells allows each unit of blood to carry additional O 2 ,
which compensates for the decreased Po 2
b. Pulmonary hypertension is secondary to the generalized pulmonary hypoxia that results
from hypoxic pulmonary vasoconstriction. The increased pulmonary artery pressure
causes a more even distribution of the pulmonary blood flow, which can improve gas
exchange by reducing the range of VA/Q values. The increased pulmonary artery pressure
can induce right ventricular hypertrophy, right-axis deviation on the EKG, right bundle
branch block, and right ventricular failure.
c. Effects at the cellular and tissue level. Chronic hypoxia has several effects including:
(1) Increased enzyme activity
(2) Increased mitochondrial density
(3) Increased ability of cells to function at low P02
(4) Increased capillary density, which occurs in skeletal and cardiac muscle
(a) The increased number of capillaries reduces the diffusion distance.
(b) This results in a decrease in the P0 2 diffusion distance from the blood into the cells.
d. Life-long exposure to hypoxia causes additional alterations that seem to improve the
body's function.
(1) Decreased respiratory drive to hypoxia has been discovered in individuals exposed
to hypoxia for prolonged periods. The reduced drive results in a higher Peo 2 and lower
P0 2 but diminishes the work of respiration, which reserves more O 2 for use by other
skeletal muscles.
(2) Total lung capacity and diffusing capacity are increased in high-altitude natives
compared to their sea-level controls. The increase in total lung capacity is apparent by
the enlarged chest that high-altitude natives develop.

3. Acute mountain sickness (AMS) occurs in many individuals who stay at altitudes in excess
of 9000-10,000 feet above sea level.
a. Symptoms of AMS include fatigue, nausea, loss of appetite, headache, dyspnea, palpitations,
and sleep disturbance. In some individuals, these effects progress and result in cerebral or
pulmonary edema.
b. Exercise during the first several days at high altitude can result in severe pulmonary edema
due to the high blood flow coupled with the hypoxic pulmonary vasoconstriction. Pulmonary
edema is very unevenly distributed throughout the lungs and is thought to occur in areas of
the lung that are not fully vasoconstricted. In these areas, the pulmonary artery pressure is
transmitted to the pulmonary capillaries, and the higher pressure increases the transudation
of fluid into the interstitium and the alveoli.
c. Treatment relies on increasing the P0 2 either by supplemental O 2 or, preferably, byevacuating the individual to lower altitude.
4. Chronic mountain sickness (Monge's disease) occurs in some long-term residents of high
altitudes who develop extreme polycythemia, cyanosis, malaise, fatigue, and exercise intolerance. These individuals must be removed to lower altitude to prevent fatal pulmonary edema
from rapidly developing.

STUDY QUESTIONS
Directions: Each of the numbered items or incomplete statements in this section is followed by answers
or by completions of the statement. Select the one lettered answer or completion that is best in each
case.
1. A 62-year-old male patient is known to have
chronic lung disease and hypercapnia. He needs a
major operation to remove an intestinal tumor. To
insure that he has adequate alveolar ventilation
while being anesthetized, which of the following
should be available?
(A)
(B)
(C)
(D)
(E)

Tank of 100% O 2
Tank of 95% O 2 , 5% CO 2
Mechanical respirator
Cardiac defibrillator
EKG machine

2. All of the following can reduce vital capacity


EXCEPT
(A)
(B)
(C)
(D)
(E)

a decreased total lung capacity


an increased residual volume
a weakness of the inspiratory muscles
a weakness of the expiratory muscles
a decreased alveolar surface tension

3. Maximal inspiratory gas flow occurs when


(A)
(B)
(C)
(D)

lung volume approaches total lung capacity


lung volume approaches residual volume
alveolar pressure is most negative
interpleural pressure is approximately -5 cm
H 20
(E) the abdominal muscles are maximally contracted

4. Which of the following is true regarding the


transmural pressure for the lungs?
(A) It always is negative
(B) It is equal to interpleural pressure minus atmospheric pressure
(C) It is equal to interpleural pressure minus alveolar pressure
(D) It is equal to alveolar pressure minus interpleural pressure
(E) It is independent of lung volume when the muscles are relaxed

5. During inspiration, as the diaphragm contracts,


the pressure in the interpleural space becomes
(A)
(B)
(C)
(D)
(E)

equal
more
more
equal
equal

to zero
positive
negative
to the pressure in the alveoli
to the pressure in the atmosphere

6. The slope of a pressure-volume curve represents


(A) resistance
(B) compliance
(C) conductance
(D) reluctance
(E) inductance
7. Which of the following statements regarding the
compliance of the respiratory system is true?
(A) It is greater than the compliance of the chest
wall
(B) It is greater than the compliance of the lungs
(C) It is equal to the compliance of the chest wall
(D) It is equal to the compliance of the lungs
(E) It is less than the compliance of the chest wall
8. The respiratory system is at the equilibrium position in all of the following conditions EXCEPT
(A) at the end of a normal expiration
(B) when the trans respiratory pressure is zero
(C) when lung recoil is balanced by chest wall
expansion
(D) when lung volume is at residual volume
(E) when the respiratory muscles are relaxed and
the airway is open
9. A lack of normal surfactant, as occurs in infants
with respiratory distress syndrome, results in
(A) an increased lung compliance
(B) stabilization of alveolar volume
(C) an increased retractive force of the lungs
(D) a reduced alveolar to arterial P0 2 difference
(E) a decrease in the filtration forces in the pulmonary capillaries

10. The primary defect responsible for respiratory


distress syndrome in infants is

16. During effort-independent flow, the flow rate


of gas depends on the

(A) a high airway resistance

(A) alveolar pressure


(B) interpleural pressure

(B) hypoventilation
(el immature inspiratory muscles
(D) abnormal pulmonary surfactant
(E) left ventricular failure
11. Effort-independent flow occurs during
(A) normal tidal volume breathing
(B) a relaxed vital capacity maneuver
(C) forced inspiration
(D) the initial part of a maximal expiratory effort
(E) the terminal part of a maximal expiratory effort
12. The major area of airway resistance during
breathing is located in the
(A) oropharynx
(B) trachea and large bronchi
(e) intermediate-sized bronchi
(D) bronchioles less than 2 mm in diameter
(E) alveoli
13. Airway resistance of the small peripheral airways (less than 2 mm in diameter) is
(A) high because of the small diameter of each
tube
(B) high due to the high velocity of flow
(e) high due to the high viscosity of gas
(D) low because of the turbulent flow that is normally present
(E) low because of the low velocity of flow
14. The major factor that determines airway resistance is the
(A) length of the airways
(B) radius of the airways
(C) gas density
(D) gas viscosity
( E) alveolar pressure
15. Airway resistance can be reduced by
(A) increasing vagal impulses to the lungs
(B) administering a J3-adrenergic blocking drug
(C) decreasing the radial traction exerted by lung
tissue
(D) performing a maximal forced expiration
(E) increasing lung volume

(C) transmural pressure of the chest


(D) transmural pressure of the lungs
(E) transmural pressure for the respiratory system
17. A patient with restrictive lung disease typically
has
(A) an increased forced expiratory volume in 1
second (FEV,) and a normal lung compliance
(B) a decreased FEV, and an increased lung compliance
(e) a decreased FEV, and a decreased lung compliance
(D) an increased FEV, and an increased lung compliance
(E) an increased FEV, and a decreased lung compliance
18. During the effort-independent portion of a
forced vital capacity, the expiratory flow rate
(A) varies as a function of the pleural pressure
(B) is limited by compression of the airways
(e) depends on the alveolar pressure
(D) is maximal for that individual
( E) is constant
19. An increased airway resistance is caused by
all of the following factors EXCEPT
(A) increase in lung volume
(B) parasympathetic stimulation
(C) aging
(D) chronic bronchitis
(E) asthma
20. The timed vital capacity, FEV" is used to evaluate the
(A)
(B)
(C)
(D)
(E)

flow resistance properties of the airways


compliance properties of the lungs
pulmonary blood flow resistance
elastance properties of the lungs
ventilation:perfusion ratio

21. The volume of gas in the lungs at the end of a


normal expiration is referred to as the
(A) residual volume
(B) expiratory reserve volume
(C) functional residual capacity
(0) inspiratory reserve volume

(E) total lung capacity


22. The volume of N2 dissolved in body fluids is
greatest while breathing which of the following gas
mixtures?
(A) Air at sea level
(B) Air at an altitude of 15,000 feet
(C) 20% O 2 ,20% N 2 , 60% He, while scuba diving
at 2 atm of pressure
(0) 20% O 2 ,30% N 2 , 50% He, while scuba diving
at 2 atm of pressure
(E) 20% O 2 , 10% N 2 , 70% He, while scuba diving
at 5 atm of pressure

26. Alveolar ventilation is equal to


(A) dead space ventilation
(B) tidal volume times respiratory rate
(C) minute ventilation
(0) minute ventilation minus dead space ventilation
(E) CO 2 production/min

27. A reduction in local alveolar ventilation is associated with


(A) an increase in regional pulmonary blood flow
(B) a decrease in regional alveolar Peo 2
(C) a decrease in regional alveolar P0 2
(0) an increase in regional tissue pH
(E) an increase in capillary hemoglobin saturation
28. Which of the following statements best characterizes the relationship between alveolar ventilation and alveolar Peo 2 ?

(A) dead space ventilation


(B) functional residual capacity
(C) inspiratory capacity
(0) alveolar ventilation
( E) alveolar Pco 2

(A) An increase in alveolar ventilation causes a decrease in alveolar Peo 2


(B) An increase in alveolar ventilation causes an
increase in alveolar Peo 2
(C) A decrease in alveolar ventilation causes a decrease in alveolar Peo 2
(0) Alveolar ventilation has no effect on alveolar
Peo 2

24. Which of the following statements is true regarding the fraction of O 2 in inspired (tracheal) gas?

29. Arterial Peo 2 is increased in a normal individual while

(A) It equals 0.25 at sea level


(B) It decreases as a function of altitude
(C) It varies as a function of the weather
(0) It is less than the fraction of O 2 in the atmosphere
(E) It equals the fraction of O 2 in the alveoli

(A) exercising
(B) breathing a gas mixture with a high P0 2
(C) ascending a mountain
(0) hypoventilating
(E) scuba diving

23. Increasing tidal volume, while keeping everything else constant, will result in an increase in

30. The major sign of hypoventilation is


25. Which of the following statements regarding
the Pco 2 in mixed expired gas is true?
(A) It is
(B) It is
(C) It is
(0) It is
(E) It is

greater than the alveolar Pco 2


less than the alveolar Pco 2
equal to the alveolar Pco 2
equal to atmospheric Pco 2
greater than the Peo 2 in venous blood

(A) cyanosis
(B) increased airway resistance
(C) hypercapnia
(0) dyspnea
(E) hypoxia

31. Which of the following statements regarding


the normal alveolar Pco 2 is true?
(A) It is equal in all alveoli
(B) It is highest at the base of vertical lungs
(C) It is directly proportional to the inspired P0 2
(0) It is directly proportional to the alveolar ventilation
(E) It is equal to 46 mm Hg

32. Which of the following gases normally is diffusion limited?


(A) O 2

(B) N 2 0
(C) He
(0) CO 2
(E) CO
33. The diffusion coefficient of O 2 , as compared
to that of CO 2 , is
(A) greater because O 2 combines with hemoglobin
(B) less because O 2 is less soluble
(C) greater because of a higher pressure gradient
(0) less because of the lower molecular weight of

O2
(E) essentially the same
34. The pH of venous blood, compared to the pH
of arterial blood, is
(A) higher because of the additional CO 2
(B) lower because of the additional CO2
(C) higher because of the removal of O 2
(0) lower because of the removal of O 2
( E) exactly the same
35. The CI-concentration in red cells of venous
blood, compared to the CI- concentration in red
cells of arterial blood, is
(A) lower due to the loss of CI- during capillary
transit
(B) lower due to the increase in red cell volume
(C) unchanged
(0) higher due to an exchange with HC03 (E) higher due to an increase in Na + concentration

36. The distribution of pulmonary blood flow is


(A) equal throughout the lungs
(B) increased in the dependent portions of the lung
(C) not affected by changes in pulmonary vascular
resistance
(0) not affected by changes in alveolar pressure
(E) not affected by changes in interpleural pressure

37. The variation in the ventilation:perfusion ratio


in different areas of the normal lung is due primarily
to the effects of
(A) neural control on the distribution of blood flow
(B) neural control on the distribution of tidal
volume
(C) gravity
(0) the hemoglobin dissociation curve
(E) CO 2 on smooth muscle in the lungs
38. Areas of the lung near the base in a normal,
standing human have which of the following ventilation:perfusion ratios (VA/O) and which volume?
(A) A VA/O less than 1 and a volume less than that
of the apex of the lung
(B) A VA/O less than 1 and a volume greater than
that of the apex of the lung
(C) A VA/O greater than 1 and a volume less than
that of the apex of the lung
(0) A VA/O greater than 1 and a volume greater
than that of the apex of the lung
(E) A VA/O of 1 and a volume less than that of the
apex of the lung
39. The alveolar P0 2 in the apex of a vertical lung
is high because the apex of the lung
(A) has a low metabolic rate
(B) receives a high blood flow
(C) has a high ventilation:perfusion ratio
(0) receives the major portion of the tidal volume
(E) does not participate in gas exchange

40. If an area of the lung is not ventilated, but


blood flow continues, then the blood leaving that
area will have a composition equal to
(A) the inspired gas
(B) the normal systemic arterial blood
(C) the composition of blood in the pulmonary
artery
(0) the composition of blood in the coronary arteries
(E) an indeterminate value because of an unknown
ventilation:perfusion ratio

41. The most common cause of hypoxia is

46. Vital capacity is the sum of

(A)
(B)
(C)
(D)
(E)

(A) residual volume, tidal volume, and expiratory


reserve volume
(B) residual volume, tidal volume, and inspiratory
reserve volume
(C) residual volume, expiratory reserve volume,
and inspiratory reserve volume
(D) expiratory reserve volume, inspiratory reserve
volume, and tidal volume
(E) functional residual capacity and inspiratory capacity

hypoventilation
anemia
ventilation:perfusion abnormalities
anatomic shunt
diffusion block between alveoli and pulmonary
capillaries

42. The following figure shows two ventilatory


patterns: one normal and the other abnormal.
Which experimental maneuver listed below will
create the abnormal pattern?

47. Tissue hypoxia can be produced by all of the


following factors EXCEPT
Abnormal

Normal
Time

(A) Midpons transection with vagi intact


(B) Transection of the brain stem between the pons
and medulla
(C) Midpons transection with vagi cut
(D) Transection rostral to the pons with vagi cut
(E) Transection rostral to the pons with vagi intact
43. Peo 2 affects respiration primarily by stimulating the
(A)
( B)
(C)
(D)
( E)

carotid and aortic bodies

J receptors
medullary chemoreceptors
baroreceptors
hypoglossal nerve

44. Lactic acidemia can increase ventilation


through its effect on receptors located in the
(A) small airways
(B) trachea and large bronchi
(C) medulla oblongata
(D) carotid bodies
( E) baroreceptors
45. Obstructed sleep apnea is characterized by
airway occlusion at the level of the
(A)
(B)
(C)
(D)
(E)

external nares
naso- or oropharynx
larynx
trachea
alveoli

(A) decreased ventilatory drive


(B) decreased hemoglobin concentration in the
blood
(C) decreased diffusion capacity of the lungs
(D) decreased CO 2 concentration in inspired air
(E) cyanide poisoning
48. The best measure of the gas exchange properties of the lungs is
(A)
(B)
(C)
(D)
(E)

the alveolar-to-arterial difference in P0 2


arterial Peo 2
arterial P0 2
arterial blood pressure
arterial hemoglobin saturation

49. A reduction of arterial P0 2 is typical of


(A)
(B)
(C)
(D)
(E)

anemia
CO poisoning
moderate exercise
cyanide poisoning
hypoventilation

50. Venous P0 2 is higher than normal in which of


the following conditions?
(A) Cyanide poisoning
(B) Exercise
(C) Decreased cardiac output
(D) Anemia
(E) CO poisoning

Directions: Each group of items in this section consists of lettered options followed by a set of numbered
items. For each item, select the one lettered option that is most closely associated with it. Each lettered
option may be selected once, more than once, or not at all.
Questions 51-55
The following graph shows a normal respiratory cycle followed by a maximal inspiration, a maximal
forced expiration, and another normal respiratory cycle. Match each of the lung volumes listed below
to the appropriate lettered arrow on the graph.

II

d:
Q)

:::l

(5

>

Ol

:::l

....J

O~~-T-----.-----r-----r--L--.----~

Time (sec)

51.
52.
53.
54.
55.

Vital capacity
Forced expiratory volume in 1 second
Functional residual capacity
Total lung capacity
Residual volume

Questions 56-59
Match each description of gas volume to the appropriate division of lung volume.
(A) Total lung capacity
(B) Residual volume
(C) Functional residual capacity
(D) Vital capacity
(E) Closing capacity
56.
57.
58.
59.

Lung volume that can be measured with just a spirometer


Lung volume at the end of a normal expiration
Maximal volume of gas that can be expired after a maximal inspiration
Lung volume at the end of a maximal expiration

ANSWERS AND EXPLANATIONS


1. The answer is C. [11/ B 3 b] Administering supplemental O 2 may correct the hypoxia, but, to maintain
an adequate ventilation and correct hypercapnia, the lungs must be adequately ventilated. The only
treatment option that provides ventilation is the respirator. The patient certainly does not need extra
CO 2 , because he is hypercapnic.
2. The answer is E. [/I A 2 a, B 2 a, HId, 2 b] A decreased alveolar surface tension will lead to an
increased lung compliance, resulting in an increased total lung capacity and an increased vital capacity.
Vital capacity equals total lung capacity minus residual volume, so that either a decrease in total lung
capacity or an increase in residual volume can reduce the vital capacity. Expanding the lungs to the
normal total lung capacity requires a strong inspiratory muscle force. Thus, weakness of the inspiratory
muscles will produce a decrease in total lung capacity. Similarly, expiratory muscle force is required to
decrease the lung volume to the normal level of the residual volume. A decrease in expiratory muscle
force can result in an increase in the residual volume.
3. The answer is C. [/I C 3] The driving force for gas flow is the pressure in the alveoli; negative
pressures cause inspiratory flow, and positive pressures cause expiratory flow. It is the ratio of alveolar
pressure to airway resistance that determines the actual flow of gas. As lung volume approaches total lung
capacity or residual volume, much of the muscle force is expended in overcoming the low compliance of
the respiratory system. When the interpleural pressure is approximately -5 cm H 2 0, it is possible for
alveolar pressure to be either positive or negative depending on muscle activity. The abdominal muscles
are expiratory muscles and, when contracted, generate positive alveolar pressures and expiratory flow.
4. The answer is D. [/I C 4, 0 1] Transmural pressure represents the pressure across the wall of a
hollow organ and is defined as the inside pressure minus the outside pressure. For the lungs, the internal
pressure is the pressure in the alveoli or airways, while the pressure immediately outside the lungs is the
pressure in the pleural space Onterpleural pressure). In humans, the interpleural pressure is estimated by
measuring the esophageal pressure, since the esophagus represents a flaccid tube that essentially traverses
the pleural space. The interpleural pressure in animal experiments usually is measured by inserting a
needle into an intercostal space and connecting it to a pressure gauge.
5. The answer is C. [/I C 4 b] The increased negativity in the pleural space is caused by the chest wall
being expanded by the inspiratory muscles. This negative pressure increases the transmural pressure of
the lungs so that the lungs expand and fill with air, which enters through the airways.
6. The answer is B. [/I 02 c] The slope of a pressure-volume curve represents the change in volume
for a unit change in pressure, which is the definition of compliance. If the graph were plotted as a
volume-pressure relationship, then the slope would be elastance, which is the inverse of compliance.
7. The answer is E. [/I 02 c; Figure 3-5] The compliance of the respiratory system (CRS) is determined
by the compliance of the lungs (CL) and the chest wall (Ccw) and can be calculated as 1 /CRS = 1 /Cl +
1 /Ccw. Because of the need to add reciprocals, the compliance of the respiratory system always is less
than the compliance of either of its parts. These relationships are apparent in Figure 3-5.
8. The answer is D. [/I 0 3 b, 4 b, H 2 c] At residual volume, the chest wall has a strong tendency to
expand because it is far from its equilibrium position, which is about 80% of total lung capacity. At the
same time, the recoil force of the lung is reduced since residual volume is close to the equilibrium
position of the lung. Due to these unequal forces, either the expiratory muscles must be contracting in
order to hold the respiratory system at that level, or the glottis must be closed to prevent gas from
entering the airways. If the glottis is closed and the respiratory muscles are relaxed, then the strong
expansion force of the chest wall will cause the gas in the airways to expand, the alveolar gas pressure
will become less than atmospheric, and the transrespiratory pressure will be negative.
9. The answer is C. [/I E 2 c (3) (a)] The lack of normal surfactant results in a high alveolar surface
tension, which increases the retractile force of the lungs, resulting in a high transmural pressure. The
high transmural pressure means that the lungs are less distensible, and the alveoli tend to collapse due
to the increased surface forces. Due to the edema and atelectasis, there is an abnormal range of
ventilation:perfusion ratios, which impairs gas exchange. The alveolar-to-arterial P0 2 difference is a good
measure of the gas exchange capabilities of the lungs. This difference increases in the presence of
ventilation:perfusion or diffusion abnormalities.

10. The answer is D. [/I E 2 c (3) (a)] Alveolar lavage fluid from premature infants with respiratory
distress syndrome shows abnormally high surface tension, which is thought to result in the alveolar
collapse and edema characteristically found in the lungs of these infants. Several medical centers presently
are running clinical tests to determine the effects of administering artificial or concentrated animal
surfactants to these infants via the trachea. Preliminary results indicate that these substances are very
effective in correcting the hypoxia and disability caused by lung immaturity.
11. The answer is E. [/I F 3] As the name implies, effort-independent flow is not related to the expiratory
force or effort that is expended. This phenomenon occurs when radial traction on the outside walls of
the airways is overpowered by a negative transmural pressure. Thus, effort-independent flow does not
occur at high lung volumes because of the large radial traction that occurs. Airway compression begins
in the large airways at about 80% of the total lung capacity and proceeds peripherally as lung volume
decreases. During normal tidal breathing, relaxed vital capacity maneuver, or forced inspiration, airway
transmural pressure is always positive, so that compression of the airways and the formation of a
flow-limiting segment is not feasible.
12. The answer is C. [/I F 1 a] The airway tree is constructed such that each generation of airways is
only slightly smaller in diameter than the parents. Thus, there is almost an exponential increase in
cross-sectional area proceeding toward the periphery of the lung. Because of this relationship, the linear
velocity of gas molecules decreases markedly as these molecules approach the terminal bronchioles.
Therefore, very little pressure is required to achieve this velocity (i.e., there is a low resistance). Direct
measurements indicate that bronchioles less than 2 mm in diameter represent less than 10% of the total
airway resistance.
13. The answer is E. [/I F 1 a, b] The small peripheral airways normally provide between 10%-20%
of the total airway resistance. Although each airway is small in diameter, the large number of these
airways provide a large total cross-sectional diameter. This results in a very low linear velocity of the gas
molecules, which requires only a small pressure gradient. The viscosity of the gas in the small airways
is no different than in the large airways.
14. The answer is B. [/I F 1 b] While the length and radius of the airways, the gas density and viscosity,
and the alveolar pressure all are involved in determining airway resistance, the most important factor is
the radius of the airways. This is true because the flow rate varies as a function of the fourth or fifth
power of the radius depending on whether flow is laminar or turbulent. Thus, small changes in the radius
can produce large changes in the flow rate or the pressure needed to generate any flow.
15. The answer is E. [/I F 1 b, 3] Increases in lung volume produce an increase in the radial traction
forces that dilate the airways. This is one of the most powerful factors that can alter airway resistance.
Increasing vagal impulses to the lungs or administering a J3-adrenergic blocking drug increases airway
resistance by narrowing the airways. The vagus nerve represents the motor nerve for bronchial smooth
muscle that constricts the airways, while J3-adrenergic blockers remove a dilating mechanism from the
airways. A decrease in radial traction, as caused by the aging process, also narrows the airways by
removing a mechanical dilating force on the outside of the airway walls. A forced expiration increases
airway resistance, since the positive pleural pressure causes a compression of the large intrathoracic
airways, which represent a flow-limiting segment under these conditions.
16. The answer is D. [/I F 3]lf the airways were rigid tubes, then gas flow would depend on the alveolar
pressure, since this would determine the transairway pressure gradient. Since the airways are collapsible
(and distensible), the presence of a positive pressure in the pleural space (caused by expiratory muscle
contraction) represents a compressive force, since the transmural airway pressure is negative for the
large airways. The driving pressure for airflow represents the difference between alveolar and pleural
pressures, or transmural pressure of the lungs. The flow is limited by the narrowed segment, and the
pressure downstream from this point (toward the mouth) has no influence on the flow rate.
17. The answer is C. [/I 02 c, F 3, H 2 b (2); Figure 3-13] A patient with restrictive lung disease has
a decrease in the forced expiratory volume in 1 second (FEV 1 ) because of the reduced vital capacity.
These patients typically can expire a larger fraction of their own vital capacity in 1 second because of
the greater radial traction that results from the decreased compliance. The increased radial traction
reduces airway resistance in the lungs. These patients have no difficulty breathing at high frequencies
because of the low airway resistance and usually choose a high rate of respiration coupled with a reduced
tidal volume to minimize the work of breathing.

18. The answer is B. [/I F 3 b] The effort-independent portion of a forced expiration begins when lung
volume reaches about 80% of total lung capacity. During the effort-independent period, the flow rate
declines as a function of lung volume. Actually, the driving force for expiration during this period is the
transpulmonary pressure, which is a function of lung volume. The term indicates that flow is independent
of effort and that changes in expiratory force do not alter the flow rates. Therefore, the expiratory flow
rate neither varies as a function of the pleural pressure, nor depends on the alveolar pressure, because
these values are altered by expiratory effort. The expiratory flow rate is not maximal for the individual
because maximal flow can be achieved only when lung volume is just below total lung capacity.

19. The answer is A. [/I F 3 b (T)] Increases in lung volume reduce airway resistance because of the
increased radial traction that is generated on the airway walls. Parasympathetic stimulation, chronic
bronchitis, and asthma are associated with increased airway resistance because of bronchial smooth
muscle contraction. Aging results in an increased airway resistance due to the destruction of alveolar
septa, which reduces the radial traction and allows the airways to constrict.
20. The answer is A. [/I H 2 b (2)] The FEV 1 represents the volume of gas expired in the first second
of a maximally forced expiration. This volume is frequently expressed as a ratio to the forced vital
capacity and is then termed the %FEV 1 The %FEV 1 compensates for different sized vital capacities and
normally exceeds 80%. The %FEV 1 is reduced either by a high airway resistance or by a reduced
expiratory force due to muscle weakness or a less than maximal effort by the patient.

21. The answer is C. [/I H 2 c] Since expiration is passive, the lung volume decreases during expiration
until the equilibrium volume (functional residual capacity) is reached. The equilibrium volume represents
the volume of a distensible structure when the transmural pressure (pressure inside minus pressure
outside) is zero.
22. The answer is A. [11/ A 2; IV A] Henry's law states that the volume of gas dissolved in a liquid
equals the partial pressure of the gas times the solubility coefficient. Since the partial pressure of N2 (PN 2)
at an altitude of 15,000 feet would be less than the PN 2 at sea level, the amount of N2 dissolved in the
tissues also would be less. The PN 2 when breathing air at sea level is 0.79 atm. Gas equilibration requires
approximately 12 hours after any change in pressure and occurs at different rates in different tissues. For
N 2, equilibration takes the longest in the fatty tissues because of the high N2 solubility and the low blood
flow to this tissue. Scuba diving at 2 atm while breathing a gas with 20% N2 provides 0.4 atm of PN 2.
The other two scuba conditions yield N2 tensions less than 0.79 atm.
23. The answer is D. [/1/ 8 1, 3] If respiratory rate, dead space, and the ventilation:perfusion ratio
remain constant, then an increase in tidal volume will increase minute ventilation and alveolar ventilation.
Since dead space ventilation = dead space volume times respiratory rate, dead space ventilation is
unchanged under these conditions. Functional residual capacity is not altered, while the inspiratory
capacity is reduced by increases in tidal volume. Alveolar CO 2 tension (Peo 2) is reduced by an increased
alveolar ventilation.
.
24. The answer is D. [/II A 3, 82 a (T)] The fraction of O 2 in the air is constant from sea level to several
hundred thousand feet altitude and equals 0.21. However, the P0 2 decreases with increased altitude,
since the total pressure declines as one ascends. The partial pressure of any gas is given by the product
of the mole fraction of the gas and the total or barometric pressure. Nasal breathing adds water vapor
to inspired air, so that the O 2 content of the air is decreased somewhat by the time it reaches the trachea.
Alveolar gas is diluted by the addition of CO 2 from the blood, so that alveolar O 2 is less than that in the
trachea.
25. The answer is B. [11/ 8 2 b] Mixed expired gas has a lower Peo 2 than alveolar gas because it is a
mixture of alveolar and dead space gas. The composition of this gas varies depending on the ratio of
dead space ventilation to alveolar ventilation. This fact can be used to determine the dead space tidal
volume ratio (VD/Vr) which is given by 1 - [expired Peo 2/alveolar Peo 2].
26. The answer is D. [11/ 8 3] Alveolar ventilation is minute ventilation minus dead space ventilation.
Minute ventilation is the volume of gas expired per minute, which is equal to the product of tidal volume
times respiratory rate. Not all of the minute ventilation reaches the gas exchange region of the lungs,
since some must occupy the conducting system of the lungs.

27. The answer is C. [III B 3; VB 2] A decrease in local or regional alveolar ventilation results in a
decreased influx of gas to that region of the lung, so that, transiently, more O 2 is absorbed, and more
CO 2 is released by the capillary blood. Consequently, alveolar P0 2 declines, while alveolar Peo 2 increases
in this area of the lung. The hypoxia, hypercapnia, and resultant local acidosis cause pulmonary vasoconstriction, which results in a decrease in pulmonary blood flow. Due to the decrease in local po 2 , there
is a reduction in the hemoglobin saturation of the blood that leaves this area of the lung. These changes
represent the effects of a low ventilation:perfusion ratio on gas exchange in a small area of the lung.
28. The answer is A. [11/ B 3 b] Alveolar Peo 2 and alveolar ventilation are inversely related, so that a
decrease in alveolar ventilation results in an increase in alveolar Peo 2 CO 2 can only be eliminated by
adequate ventilation of the alveoli. It does not matter what type of gas is used to flush out CO 2 , provided
the flow is adequate. Of course, an adequate level of O 2 in the gas is required to maintain life.
29. The answer is D. [11/ B 3 b] Hypoventilation is synonymous with hypercapnia. The arterial Peo 2 is
determined by the ratio of CO 2 production to alveolar ventilation. During exercise, the alveolar ventilation
is adjusted so that arterial Peo 2 remains normal and may even decline under moderate and heavy
exercise loads. During O 2 breathing there is little, if any, effect on alveolar ventilation in the normal
individual. Mountain climbing exposes one to hypoxia with an increase in respiratory drive and a
consequent decrease in Peo 2 Scuba diving, at least at moderate depths (i.e., less than 100 feet), should
not alter the Peo 2 , since alveolar ventilation should be maintained at a normal level.

30. The answer is C. [11/ B 3 b] Hypercapnia and hypoventilation are synonyms, since the alveolar
Peo 2 is primarily determined by the alveolar ventilation. Cyanosis may be present with hypoventilation,
but it is a sign (and not a very good one) of hypoxia, not hypoventilation. Both an increased airway
resistance and dyspnea may be symptoms of airway disease but are not characteristic of hypoventilation.
31. The answer is B. [11/ B 3 b; VII B] Alveolar Peo 2 varies inversely as a function of the ventilation:perfusion ratio (V A/o.) or alveolar ventilation. With a high VA/o., the ventilation exceeds blood flow so that
CO 2 delivery to the lungs via the pulmonary blood flow is low and is diluted by the ventilation. The
opposite is true at the base of the lungs. The P0 2 varies reciprocally with the Peo 2 At sea level, while
breathing air, the sum of the two gas tensions cannot exceed 150 mm Hg, which is the inspired Po 2 ,
because the remaining gas fraction is occupied by N 2 , which is not utilized by the body. Alveolar Peo 2
is maintained at 40 mm Hg to maintain the pH of body fluids in the normal range.
32. The answer is E. [11/ C " 4] Normally only CO is diffusion limited, but, in the presence of severe
lung disease, O 2 also may be diffusion limited if the diffusion capacity of the lungs is reduced sufficiently.

33. The answer is B. [11/ C 4] The diffusion coefficient concerns the movement of molecules in solution
and is determined by the molecular weight and the solubility of any substance. O 2 has a lower molecular
weight than CO 2 but is much less soluble-about 24 times less. The overall effect is that CO 2 is much
more diffusible than is O 2 , which is important because there is only about a 5 mm Hg gradient to cause
CO 2 to diffuse from the pulmonary capillary blood to the alveoli.
34. The answer is B. [IV 0] The pH of venous blood is lower than the pH of arterial blood because of
the addition of CO 2 at the tissue level. CO 2 is an acid molecule because it becomes hydrated to carbonic
acid (H 2 C0 3 ), which can dissociate into H+ and HC0 3 - .
35. The answer is C. [IV 0 3 a] The CI- concentration in red blood cells is higher in venous blood
than in arterial blood because of the chloride shift that occurs in the tissue capillaries. As blood picks up
CO 2 at the tissues, some of it diffuses into the red cells, where it is hydrated to H 2 C0 3 , which then
dissociates into H + and HC03 - . The H + is buffered by hemoglobin, and the HC0 3 - diffuses out of the
red cells. In order to maintain electrical neutrality in the cells, the HC0 3 - is replaced by CI-, which
diffuses in and raises the CI- concentration in the venous red cells.
36. The answer is B. [V B] The pulmonary vasculature is a low pressure-low resistance system that is
very distensible. Gravity causes a linear loss of the driving force at locations above the heart, so blood
flow decreases to apical areas. The dependent vessels are distended due to the higher hydrostatic forces
and flow resistance decreases, so that flow increases toward the base of the lung. Changes in alveolar
or interpleural pressure alter flow through the alveolar and extra-alveolar vessels in a complex fashion
that alters the resistance in these vessels.

37. The answer is C. [V B; VI A] Gravity affects both the distribution of the tidal volume and pulmonary
blood flow because of the hydrostatic effects on the driving pressure and the interpleural pressure. The
autonomic nervous system has only a minor role in adjusting both pulmonary vascular and airway
resistance under normal conditions. While an increased Peo 2 causes pulmonary vasoconstriction, and
a decreased expired Peo 2 results in bronchoconstriction, neither of these responses represents a major
factor under normal conditions. The hemoglobin dissociation curve is not involved in controlling blood
or air flow.
38. The answer is A. [VI A T; VII A] The ventilation:perfusion ratio eVA/Q) is less than 1 at the base of
the lungs because blood flow exceeds ventilation in this area. Blood flow increases more rapidly per
vertical distance down the lung, since gravity has a greater effect on blood due to its higher density
compared with lung tissue. The transmural pressure for the lung varies between base and apex because
of the pleural pressure gradient. The pleural pressure is near zero at the base of the lung, whereas it is
more negative at the apex. Thus, the transmural pressure is minimal at the base and greater at the apex,
which results in the base of the lung being less distended than the apex.
39. The answer is C. [VII A] The P0 2 in the lungs depends on the regional ventilation:perfusion ratio.
A high ratio results in a high alveolar P0 2 and a low alveolar Peo 2 . The high ventilation:perfusion ratio
at the apex of a vertical lung occurs because the perfusion rate is reduced much more than the rate of
ventilation. However, both ventilation and blood flow are reduced compared to what is present in the
base of the lung.

40. The answer is C. [VII B]lf an area of the lung is not ventilated, then there will be no gas exchange
in that area. If there is no gas exchange, then the gas tensions will be equal to that in the venous blood.
Since the blood in the pulmonary artery represents mixed venous blood from the systemic circulation,
then blood leaving an unventilated area of the lungs will have a gas composition equivalent to that in
the pulmonary artery.
41. The answer is C. [VII] Due to the complexity of factors involved in regulating ventilation and
perfusion in the lungs, many diseases can lead to an imbalance in these factors. It has been estimated
that 85% of patients with hypoxia suffer from ventilation:perfusion imbalance. Hypoventilation, anatomic
shunt, and diffusion block represent three other abnormalities that result in arterial (hypoxic) hypoxia.
Anemia is an additional mechanism of hypoxia that is less common than ventilation:perfusion imbalance.
42. The answer is C. [VIII A 2 b; Figure 3-30] This pattern of breathing is called apneustic breathing,
or inspiratory breath holding. This respiratory pattern occurs when the inhibitory influences from the
periphery via the vagus nerves and the pneumotaxic center are interrupted. The pneumotaxic center lies
in the rostral pons, whereas the apneustic center (which enhances inspiratory drive) lies in the caudal
pons. The vagus nerve carries impulses from stretch receptors in the airways that tend to inhibit inspiration
after a certain threshold of tidal volume is exceeded.
43. The answer is C. [VIII A 3] Approximately 85% of the effect of CO 2 is mediated through the
medullary chemoreceptors; only 15% of the effect comes from the carotid and aortic bodies. J receptors,
baroreceptors, and the hypoglossal nerve are not affected by changes in CO 2 ,

44. The answer is D. [VIII B 2] H+ affects ventilation by stimulating the peripheral chemoreceptors or
carotid bodies. H+ does not readily cross the blood-brain barrier and therefore cannot affect the central
chemoreceptors in the medulla.
45. The answer is B. [VIII E 4] Obstructed sleep apnea is caused by a partial or complete obstruction
of the upper airway. During sleep, the muscle tone of the pharyngeal dilator muscles is lost so that there
is airway narrowing. This results in a higher velocity of gas flow through this narrowed area, producing
a more negative pressure in the airway (Bernoulli effect). These very negative pressures result in fluttering
of the tissues, causing the typical snoring indicative of intermittent closure of the airway in the oro- or
nasopharynx.
46. The answer is D. [Figure 3- T2] Vital capacity-which is made up of expiratory reserve volume,
inspiratory reserve volume, and tidal volume-is the maximal amount of gas that can be expired after
a maximal inspiration. Residual volume is not included in vital capacity.

47. The answer is 0. [IX C] Air normally has only a negligible quantity of CO 2 , so further reduction in
the CO 2 will have no effect. Decreased ventilatory drive and decreased diffusion capacity of the lungs
result in arterial hypoxia; decreased hemoglobin concentration in the blood represents hypemic hypoxia;
and cyanide poisoning causes histotoxic hypoxia.
48. The answer is A. [IX C 1] Normally, the difference between the alveolar and arterial P0 2 is 5-10
mm Hg, which is due to a small anatomic shunt and to some ventilation:perfusion ('VA/o.) imbalance.
Any further increase in the difference between the alveolar and arterial P0 2 indicates that the gas
exchange function is impaired by a \JA/o. imbalance (physiologic shunt), anatomic shunt, or diffusion
limitation. Arterial Peo 2 depends primarily on the adequacy of alveolar ventilation. With hypoventilation,
the alveolar Peo 2 and, consequently, the arterial Peo 2 rise while alveolar and arterial P0 2 fall; however,
the difference between the alveolar and arterial P0 2 remains in the normal range. Arterial blood pressure
is altered reflexly through the chemoreceptors by changes in the blood gases, while hemoglobin saturation
is largely a function of the Po 2 The alveolar-to-arterial difference in P0 2 is the best measure of gas
exchange.

49. The answer is E. [IX C 1 a] A normal arterial P0 2 is typical of anemia, CO or cyanide poisoning,
and moderate exercise. It is either the tissue or the venous P0 2 that is reduced as a consequence of
inadequate O 2 delivery. Hypoventilation results not only in hypoxia but also, even more characteristically,
in hypercapnia.

50. The answer is A. [IX C 4] Cyanide poisoning causes venous P0 2 to be higher than normal, because
the tissue oxidative enzymes are inactivated by cyanide and unable to use O 2 Therefore, less O 2 diffuses
out of the blood in the systemic capillaries, which leaves a greater amount in the venous blood. During
exercise, the tissues remove a greater amount of O 2 from the capillary blood, which results in a decreased
venous Po 2 A decreased cardiac output, anemia, and CO poisoning all reduce O 2 delivery to the tissues
so that more O 2 than normal is extracted from the capillary blood, and venous P0 2 declines.
51-55. The answers are: 51-0, 52-C, 53-B, 54-A, 55-E. [/I H; Figures 3-12,3-13] Vital capacity is the
maximal volume of gas that can be expired after a maximal inspiration (indicated by the arrow labeled
0). Vital capacity equals the sum of tidal volume, inspiratory reserve volume, and expiratory reserve
volume.
The forced expiratory volume in 1 second (FEV,) is the volume of gas that can be expired in 1 second
during a maximal forced expiration. The forced expiration begins at the third second on the graph; the
volume expired 1 second later is indicated by the arrow labeled C.
The functional residual capacity is the reservoir of gas that remains in the lungs after a normal expiration
(indicated by the arrow labeled 8). This gas buffers the changes in O 2 and CO 2 tensions in the blood
that traverses the capillaries between inspirations.
The total lung capacity represents the amount of gas in the lungs after a maximal inspiration (indicated
by the arrow labeled A).
The residual volume is the amount of gas in the lungs after a maximal expiration (indicated by the
arrow labeled fl. The residual volume cannot be removed from the lungs, because the chest wall
becomes rigid at this volume and lung volume cannot be reduced further. In older individuals, the
residual volume is set by closure of the airways, which prevents further emptying of the lungs.

56-59. The answers are: 56-0, 57-C, 58-0, 59-B. [/I H 1 d, 2 a-c; VI C 1 e] A simple spirometer can
only measure lung volumes that can be expired; thus, any division of lung volume that contains the
residual volume (RV) requires the use of much more sophisticated equipment. The total lung capacity
(TLC), functional residual capacity (FRC), and closing capacity all include the RV. (The closing capacity
consists of the closing volume and the RV.)
The respiratory system normally reaches its equilibrium position at the end of a normal expiration. At
this volume, the expansion force of the chest wall is counteracted by the retractile force of the lung, and
the transrespiratory pressure is zero.
The maximal volume of gas that can be expired after a normal expiration is the vital capacity (VC). It
is important that patients inspire maximally in order to achieve TLC and then expire maximally to reach
RV. The VC equals TLC minus RV. Weakness of the inspiratory muscles will reduce TLC, while weakness
of the expiratory muscles will increase the RV.
The RV represents the volume of gas that cannot be expired from the lungs with an intact chest. The
RV is determined in young adults by the chest wall, which reaches a point where it cannot be compressed
further. In older people and in patients with obstructive lung disease, the RV is set by airway closure.
Once the airways close off, the gas distal to the closure cannot be expelled and represents the RV.

4
Renal Physiology
John Bullock

I. INTRODUCTION
A. Kidney function. The kidneys maintain the constant state of the body's internal environment by
regulating the volume and composition of the extracellular fluids. To accomplish this, the kidneys
balance precisely the intake, production, excretion, and consumption of many organic and inorganic compounds. This balancing requires that the kidneys maintain the volume and composition
of body fluids by the conservation and excretion of water and solutes to meet bodily requirements.

1. Water and electrolyte intake and metabolism


a. The gastrointestinal system provides the primary source for the normal oral intake of water
and electrolytes.
b. The oxidation of food provides a secondary but important source of water. The ordinary
mixed diet will lead to the production of approximately 300 ml of metabolic water per day,
mostly from the oxidation of fat.
c. Metabolism also produces urea, a nontoxic product of protein metabolism; uric acid, the
end product of purine metabolism; and creatinine, an endogenous anhydride of muscle
creatine.
2. Excretion of water and solutes
a. A nonperspiring young man in a basal metabolic state loses approximately 30 g of
water/hr by insensible perspiration. This, together with water loss from the lungs, constitutes
the insensible water loss, which can range between 800 and 1400 ml/day, depending on
body surface area and metabolic rate. Sensible sweat is a hypotonic solution with a salt
concentration ranging from 10 to 70 mEq/L of water.
b. Less than 100 ml of fluid together with 1 mEq each of sodium and chloride are excreted in
the feces per day.
c. Normally, the kidneys excrete about 1500 ml of hypertonic urine in 24 hours.
3. The formation and release of renin, which is a major component of the renin-angiotensinaldosterone mechanism, allows the kidneys to regulate blood pressure. In addition, the renal
control of fluid volume is essential for the regulation of blood pressure.
4. The formation and release of renal erythropoietic factor (erythropoietin) increases the
number of circulating erythrocytes. Erythropoietin is synthesized in the epithelial cells of the
renal cortical glomerular tuft and the juxtaglomerular cells. The liver is the main source of
erythropoietin in the prenatal period.
S. Vitamin D activation. Dietary vitamin D must undergo two hydroxylations in order to be
useful to the body. The first step is performed by the liver. The final hydroxylation, performed
by the kidneys, converts this vitamin to its most biologically active form (1,25-dihydroxycholecalciferol).
6. Gluconeogenesis. The kidney acquires the important ability to synthesize and secrete glucose
produced from noncarbohydrate sources (glutamine) only in unusual circumstances such as
prolonged starvation and chronic respiratory acidosis.

199

B. Kidney structure (Figure 4-1). The kidneys are paired organs that are located retroperitoneally in
the upper dorsal region of the abdominal cavity. Each human kidney is composed of approximately
1 million nephrons, is about the size of a fist, and weighs about 150 g.

1. Nephron. This basic functional unit of the kidney is composed of a glomerulus, with its
associated afferent and efferent arterioles, and a renal tubule.

a. Glomerulus. The glomerulus consists of a tuft of 20-40 capillary loops protruding into
Bowman's capsule, which is the beginning of the renal tubule. The capillary endothelium
is fenestrated with an incomplete basement membrane, and together these structures provide
a minimal resistance for filtration of plasma while providing a sieving function for retention
of plasma proteins and blood cells.
b. Renal tubule (Figure 4-2). The renal tubule begins as Bowman's capsule, which is an
expanded, invaginated bulb surrounding the glomerulus. The epithelium of Bowman's
capsule is an attenuated layer that is about 400 A. in thickness. The renal tubule consists of
the proximal convoluted tubule, the loop of Henle, the distal convoluted tubule, and
the collecting duct that carries the final urine to the renal pelvis and the ureter.

c. Types of nephrons
(1) Cortical nephrons comprise about 85% of the nephrons in the kidney and have
glomeruli located in the renal cortex. These nephrons have short loops of Henle, which
descend only as far as the outer layer of the renal medulla.

Capsule
Nephron

Ql

t::

Pyramid

()

_________ J
Outer stripe

"1

--------- c:

Inner stripe Q;

:;

__________ J

Hilus

Renal vein

Ii
c.
til

a.

j
Figure 4-1. Structure of the human kidney, cut open to show the various zones. (Reprinted from Marsh DJ: Renal
Physiology. New York, Raven, 1983, p 37.) Inset: Two principal nephron types and their collecting duct systems.
Superficial (cortical) nephrons have their glomeruli near the surface of the kidney and do not possess long loops of
Henle. Juxtamedullary nephrons have their glomeruli at the corticomedullary junction. Glomeruli and proximal tubules
are shown as solid black. The remaining nephron is shown as white. (Reprinted from Brenner B, Coe FL, Rector FC;
Transfer functions of renal tubules. In Renal Physiology in Health and Disease. Philadelphia, WB Saunders, 1987,
p 29.)

Proximal tubule

----\o\:---Bowman's space

Urinary space of
Bowman's capsule

Endothelial cell
Capillary lumen of glomerulus
Figure 4-2. Organization of the glomerulus. The afferent arteriole forms a capillary plexus, which then fuses to form
the efferent arteriole. The outer lining of the glomerulus, called Bowman's capsule, is continuous with the proximal
tubule. (Reprinted from Marsh OJ: Renal Physiology. New York, Raven, 1983, p 41.) Inset: Glomerular capillary wall
in cross-section. The luminal surface is covered by fenestrated endothelial cells. The basement membrane has a
middle lamina densa surrounded by lamina rara interna and externa. Overlying this are the foot processes of epithelial
cells, separated by small slit diaphragms. (Illustration by Nancy Lou Gahan Markris. Reprinted from Brenner BM,
Beeuwkes R III: The renal circulations. Hosp Pract 13:35-46, 1978.)
(2) Juxtamedullary nephrons are located at the junction of the cortex and the medulla of
the kidney. Juxtamedullary nephrons have long loops of Henle, which penetrate deep
into the medulla and sometimes reach the tip of the renal papilla. These nephrons are
important in the countercurrent system, by which the kidneys concentrate urine.
2. Renal blood vessels
a. Renal arteries. Each kidney receives a renal artery, which is a major branch from the aorta.
The kidneys receive approximately 25% of the total resting cardiac output or about 1.25 L
blood/min. The sympathetic tone to renal vessels is minimal at rest but increases during
exercise to shunt renal blood flow to exercising skeletal muscles.
(1) Afferent and efferent arterioles. Each renal artery subdivides into progressively smaller
branches, and the smallest branches give off a series of afferent arterioles. Each afferent
arteriole forms a tuft of capillaries, which protrudes into Bowman's capsule. These
capillaries come together and form a second arteriole, the efferent arteriole, which
divides shortly after to form the peritubular capillaries that surround the various portions
of the renal tubule.
(2) Peritubular capillaries differ in organization depending on their association with different nephrons.
(a) The efferent arterioles of cortical nephrons divide into peritubular capillaries that
connect with other nephrons, forming a rich meshwork of microvessels. This meshwork functions to remove water and solutes that have diffused from the renal
tubules.

(b) Efferent arterioles of juxtamedullary nephrons also form peritubular capillaries, a


special portion of which are the vasa recta. The vasa recta descend with the long
loops of Henle into the renal medulla and return to the area of the glomerulus. The
vasa recta form capillary beds at different levels along the loop of Henle.
b. Renal veins are formed from the confluence of the peritubular capillaries and exit the kidney
at the hilus. The pattern of the renal venous system is similar to that found in the end arterial
system except for the presence of multiple anastomoses between veins at all levels of the
venous circulation.

II. BODY FLUID COMPARTMENTS


A. Water content and distribution (Figure 4-3). The two major fluid compartments are the intracellular fluid and extracellular fluid volumes.

1. Total body water (TBW) constitutes 55%-60% of the body weight in young men and 45%50% of the body weight in young women. (The lower percentage in women largely is due to
the relatively greater amount of adipose tissue in women than in men.)
a. Distribution. About one-third of the total body water is in the extracellular fluid compartment, and the remaining two-thirds is in the intracellular fluid compartment. (The TBW
distribution in a young, 70-kg man is summarized in Table 4-1.) Distribution of TBW is as
follows:
(1) Muscle (50%)
(2) Skin (20%)
(3) Other organs (20%)
(4) Blood (10%)
b. Lean body mass. Although the percentage of TBW declines with advancing age and with
obesity, this percentage for any individual (regardless of sex) represents a constant 70% of
that individual's lean body mass (LBM or fat-free mass). * Based on this constant relationship,
the amount of body fat can be determined as
body fat (%)

100 -

percentage of TBW
=------...:=----0.7

and the LBM can be estimated as


LBM (kg)

TBW (L)
= ---

0.7

2. Extracellular fluid (ECF). The ECF compartment includes several subcompartments.


a. Plasma volume, which represents the fluid portion of the blood, is about 3.5 L in a young,
70-kg man and represents about 25% of the ECF.

Table 4-1. Distribution of Body Water in a Young, 70-kg Man


Percent
Compartment
Total body water
Extracellular fluid
Plasma
Interstitial fluid
Intracellular fluid

Volume
(L)

Body
Weight

Lean Body
Mass

Body
Water

42*
14
3.5
10.5
28

60 t
20
5
15
40

70
24
6
18
48

100
33
8
25
67

*Total body water is 35 L in a young, 70-kg woman.


t80dy water accounts for 50% of the total body weight of a young, 70-kg woman.

'Lean body mass is defined as 15% bone, 10% fat, and 75% tissue.

BONE AND
(3% = 2L)

:::J
l!J

co?
II
-!.
0

!:!!.

U>

--1
a..

D
D
D
D
D

CONNECTIVE TISSUE
(4.5% = 3 L)

I l

CU

INTERSTITIAL
FLUID
(15% = 10.5 L)

--I
:II

D
D
D
D
D

Z
U>

()

r
r

INTRACELLULAR
FLUID
(40% = 28 L)

S;
:II
"T1

a
~

tn

-!.
0

II
~

.c:

Figure 4-3. Distribution of the total body water in an average 70-kg man. The intracellular fluid compartment is not
subdivided, but the extracellular fluid compartment consists of four subdivisions: plasma, interstitial fluid, transcellular
fluid, and the water of bone and connective tissue. Relative volumes are expressed as percentages of the body weight,
assuming a fat content of about 10%. Fat has relatively little water associated with it. (Reprinted from Bauman JW Jr,
Chinard FP: Body fluids. In Renal Function: Physiological and Medical Aspects. St. Louis, CV Mosby, 1975, p 19.)
(1) Blood volume constitutes about 8% of the total body weight and can be obtained from

plasma volume and the hematocrit as


blood volume (U

100
plasma volume (U x - - - - - - (100 - hematocrit)

(2) Blood volume occupies approximately 80 ml/kg of body weight.


b. Interstitial fluid is the fluid between cells and includes lymph, which constitutes 2%-3%
of the body weight. The volume of interstitial fluid in a young, 70-kg man is about 10.5 L
and represents about 75% of the ECF. The interstitial fluid is also referred to as the internal
environment, or milieu interieur, which surrounds all cells except the blood cells.
c. Transcellular fluid volume is about 1 L in most humans and occupies about 15 ml/kg of
body weight (1.5% of body weight). This ECF subcompartment represents fluid in the lumen
of structures lined by epithelium and includes digestive secretions; sweat; cerebrospinal fluid
(CSF); pleural, peritoneal, synovial, intraocular, and pericardial fluids; bile; and luminal fluids
of the gut, thyroid, and cochlea.
(1) Gastrointestinal luminal fluid constitutes about half of the transcellular fluid and occupies
about 7.4 ml/kg of body weight.
(2) CSF occupies about 2.8 ml/kg of body weight.
(3) Biliary fluid volume is about 2.1 ml/kg of body weight.
3. Intracellular fluid (ICF). The volume of the ICF compartment varies but usually constitutes
30%-40% of the body weight. This is the larger of the two major fluid compartments, containing about two-thirds of the TBW.

B. Volume measurement in the major fluid compartments

1. Indicator dilution principle. The volume of water in each fluid compartment can be measured
by the indicator dilution principle. This principle is based on the relationship among the amount
of a substance injected intravenously (Q), the volume in which that substance is distributed
(V), and the final concentration attained (C).

a. The equation for this relationship, based on the definition of concentration (C), is
C

9 or V = 9
V

(1)

where V = the volume (in ml or L) in which the quantity, Q (in g, kg, or mEq), is distributed
to yield the concentration, C (in glml or l or in mEq/ml or L).
Example. If 25 mg of glucose are added to an unknown volume of distilled water and the
final concentration of glucose after mixing is 0.05 mglml, then the volume of solvent is

V=

25 mg
0.05 mglml

= 500 ml

b. Volume measurement by the dilution principle requires that the introduced substance be
distributed evenly in the body fluid compartment being measured.
(1) Equation (1) must be altered if the solute leaves the compartment through one of the
following mechanisms:
(a) Excretion in the urine or transfer to another compartment where it exists in a
different concentration
(b) Metabolism of the solute
(c) Vaporization of the solute from the skin and respiratory tract
(2) The amount of substance lost from the fluid compartment, then, is subtracted from the
quantity administered, as

Q administered - Q removed

V = -'--------'---C
Example. A 60-kg woman is infused with 1 millicurie (mCn of tritium oxide (lHzO).
After 2 hours, 0.4% of the administered dose is lost in the urine and by vaporization
from the skin and respiratory tract. The radioactivity of a plasma sample is measured
by liquid scintillation spectrometry and indicates a concentration of 0.03 mCi/l of
plasma water. Since the concentration of 3H zO throughout the body fluids should be
the same as in plasma after equilibration, the TBW can be calculated as
V

Q infused - Q excreted
C

= ---------

1 mCi - (1 mCi x 0.004)


0.03 mCi/l
0.996
0.03
=

33.2 l

c. 3H zO is an unstable isotope and the substance of choice for measuring TBW. It is a weak
beta emitter with a biologic half-life of 10 days but a physical half-life of 12 years. Other
substances used to measure TBW include:
(1) Antipyrine and N-acetyl-4-amino antipyrine (NAAP), which rarely are used
(2) Deuterium oxide (2H zO), which is a stable isotope
2. ECF volume
a. Substances used to measure the ECF volume are of two types:
(1) Saccharides such as inulin, sucrose, raffinose, and mannitol
(2) Ions such as thiosulfate, thiocyanate, and the radionuclides of sulfate (50/-), chloride
(CI-), bromide (Br-), and sodium (Na+)
b. Interstitial fluid volume cannot be measured directly, because no substance is distributed
exclusively within this compartment. To measure the interstitial fluid volume, the capillary
membranes must be permeable to a substance injected intravenously, which becomes
distributed throughout the ECF and not exclusively in the interstitium. Therefore, the interstitial fluid volume is determined as the difference between the ECF volume and plasma
volume.
c. Plasma volume is measured using either of two dilution methods.
(1) The first method employs substances that neither leave the vascular system nor penetrate
the erythrocytes. Such substances include:
(a) Evans blue dye (T-1824)

(b) Radioiodinated human serum albumin (RISA)*


(c) Radioiodinated gamma globulin and fibrinogen, which generally do not leak out of
the bloodstream
(2) The second method is based on the fact that the radioisotopes of phosphorus (32P), iron
51
(55, 59 Fe), and chromium ( Cr) penetrate and bind to erythrocytes, The tagged cells are
injected intravenously, and their volume of distribution is measured, Plasma volume
(PV), then, is calculated from the measured erythrocyte volume (BVl and hematocrit
(Hctl as
PV (in L) = BV (in L) x

(100 - Hctl

lOa

PV (in L)
; BV (in L) = - - 1 - Hct

3. ICF volume cannot be measured directly by dilution, because no substance is confined exclusively to this compartment after intravenous injection of a marker substance, The ICF volume
is obtained by subtracting the ECF volume from the TBW,t

C. Disturbances of volume and concentration (osmolality) of body fluids (Figure 4-4; Table 4-2l
1. Terms and general concepts
a. The general clinical terms for volume abnormalities are dehydration and overhydration.
Dehydration is an ambiguous term that does not distinguish between simple water loss and
loss of Na +. Both conditions are associated with a decrease in ECF volume, The ECF volume
is determined by the amount of Na+ in the body (Na+ content) and not by the Na+
concentration in the plasma!
(1) A simple water deficit reduces the ECF and ICF proportionately.
(2) A sodium chloride (NaCi) deficit will always cause a decrease in ECF volume.
b. The adjectives isosmotic, hyperosmotic, and hyposmotic are used to describe the changes
in volume (i.e., dehydration and overhydrationl and refer to the osmolar concentration of
ECF in its new steady state. Since the concentration of the body fluids is regulated by
antidiuretic hormone, and the volume of the body fluid compartments is regulated by
aldosterone, these two hormones attempt to reestablish the normal volumes and concentrations by an increase or decrease in their secretion.
c. The distribution of body water in the ECF and ICF compartments is determined by osmosis,
which is the diffusion of water across a membrane. Thus, water moves from a region of
lower osmolality to a region of higher osmolality. Note that osmosis can also be defined as
the movement of water from a region of lower solute concentration to a region of higher
solute concentration.
2. Dehydration (volume contraction) states
a. Isosmotic dehydration (see Figure 4-4Al
(1) Description
(a) Initially, fluid is lost from the plasma and then is repleted from the interstitial space.
No major change occurs in the osmolality of the ECF; therefore, no fluid shifts into
or out of the ICF compartment.
(b) Finally, the volume of the ECF is reduced with no change in osmolality.
(2) Causes of isosmotic dehydration include hemorrhage, plasma exudation through
burned skin, and gastrointestinal fluid loss (e.g., vomiting, diarrheal.
b. Hyperosmotic dehydration (see Figure 4-4Al
(1) Description
(a) Initially, fluid is lost from the plasma, which becomes hyperosmotic, causing a fluid
shift from the interstitial fluid to the plasma.
(b) The rise in interstitial fluid osmolality causes fluid to shift from the ICF back to the
ECF compartment.
(c) Finally, the ECF and ICF volumes both are decreased, and the osmolality of both
major fluid compartments is increased.

*RISA slowly leaks out of the circulation into the interstitial fluid; therefore, the plasma volume is slightly overestimated
when RISA is used.
tThere is a good correlation between the ICF volume and total exchangeable K+. ICF volume also is related to muscle
mass, which decreases with age.

Normal state

q;

>-J: 300j

200

ICF

~ ~ 100

r:(:::~:-)t1

ECF

Cl

.!!!-"
0-

00

.s

10

20

30

40

Volume (L)

A. Dehydration states
1. Isosmotic
>-

q;300~
;200

o~

~ ~ 100

00

.s

.~I~
Volume(L)

q; 300

~J:

~~200
0E E

00E 100
~

2. Hyperosmotic

3. Hyposmotic

Volume(L)

B. Overhydration states
1. Isosmotic

2. Hyperosmotic

Volume(L)

3. Hyposmotic

j i~lI~ ~
Volume (L)

Volume (L)

Figure 4-4. A Darrow-Yannet diagram representing the volume (abscissa) and osmolality (ordinate) of the intracellular
and extracellular fluid compartments (lCF and ECf) in a 70-kg man. The combined plasma and interstitial fluid spaces
are represented by the ECF compartment. This diagram is useful to simplify the clinical analysis of fluid balance. The
area of each fluid compartment rectangle represents the total milliosmoles of solutes in that compartment. In all
diagrams, the normal state is indicated by solid lines and the shifts from normality are indicated by dashed lines.
(Reprinted from Valtin H: Renal Function: Mechanism Preserving Fluid and Solute Balance in Health, 2nd edition.
Boston, Little, Brown, 1983, p 272.)

Table 4-2. Steady State Changes in Volume and Osmolal Concentration of Body Fluids
Osmolality (mOsm/kg H 2 0)

Volume (L)
Type of Change
Contraction (dehydration)
Isosmotic
Hyperosmotic
Hyposmotic
Expansion (overhydration)
Isosmotic
Hyperosmotic
Hyposmotic

ICF

ECF

ICF

ECF

o
t
~

t
t

Note-The changes in volume and in osmolality refer to the ECF compartment in the new steady state.

(2) Causes of hyperosmotic dehydration include water deficits due to decreased intake,
diabetes insipidus (neurogenic or nephrogenic), diabetes mellitus, alcoholism, administration of lithium salts, fever, and excessive evaporation from the skin and breath.
c. Hyposmotic dehydration (see Figure 4-4A)
(1) Description
(a) Initially, loss of NaCi causes a loss of water. This is followed by water retention but
a continued loss of NaCI.
(b) A net loss of NaCi in excess of water loss results in a decreased osmolality of the
ECF and a subsequent shift of fluid from the ECF to the ICF compartment.
(c) Finally, the ECF volume is decreased, the ICF volume is increased, and the osmolality
of both major fluid compartments is decreased.
(2) Causes of hyposmotic dehydration include loss of NaCi due to heavy loss of hypotonic
sweat and renal loss of NaCi due to adrenal insufficiency [e.g., primary hypoadrenocorticalism (Addison's disease)].
d. General signs of dehydration include:
(1) Decrease in plasma volume, decrease in blood volume and, in turn, a decrease in
cardiac output with poor filling of veins below a venous tourniquet
(2) Decrease in skin turgor ("tenting")
(3) Sunken and soft eyeballs
(4) Dry mucous membranes
(5) Sunken fontanelles (in babies)
(6) Gray, cool skin
(7) Increased pulse rate

3. Overhydration (volume expansion) states


a. Isosmotic overhydration (see Figure 4-4B)
(1) Description. Isosmotic overhydration is characterized by an overall expansion of the
ECF volume with no change in the osmolality of the ICF and ECF compartments.
(2) Causes of isosmotic overhydration are edema and oral or parenteral administration of
a large volume of isotonic NaCI.
b. Hyperosmotic overhydration (see Figure 4-4B)
(1) Description
(a) Oral intake of large amounts of salt or intravenous infusion of a hypertonic saline
solution leads to an increase in the plasma osmolality.
(b) The rise in plasma osmolality causes a shift of water from the interstitium into the
plasma, thereby initially increasing plasma volume.
(c) Concomitantly, the increase in plasma salt concentration causes NaCi to diffuse into
the interstitium. The net result is an increase in the osmolality of the ECF without a
change in volume.
(d) The increase in the osmolality of the ECF causes water to flow out of the ICF, which
eventually decreases the volume of the ICF and increases the volume of the ECF.
The osmolality of both major fluid compartments is increased.
(2) Cause. Oral or parenteral intake of large amounts of hypertonic fluid causes hyperosmotic overhydration.
c. Hyposmotic overhydration (see Figure 4-4B)
(1) Description
(a) Initially, water enters the plasma, causing a decline in the plasma osmolality, a shift
of water into the interstitial space, and a decrease in the interstitial fluid osmolality.
(b) The decrease in interstitial fluid osmolality causes water to shift from the ECF to the
ICF compartment.
(c) Finally, the ECF and ICF volumes increase and the osmolality of both major fluid
compartments decreases.
(2) Causes of hyposmotic overhydration include ingestion of a large volume of water and
renal retention of water due to the syndrome of inappropriate antidiuretic hormone
(ADH) secretion (SIADH).

D. Ionic composition of body fluids (Figure 4-5; Table 4-3)


1. General considerations
a. Ions constitute about 95% of the solutes in the body fluids.
b. The sum of the concentrations (in mEq/ L) of the cations equals the sum of the concentrations
(in mEq/L) of the anions in each compartment, making the fluid in each compartment
electrically neutral.
c. Physicians rely on the changes in electrolyte concentrations in the ECF compartment,
particularly in the plasma, in the diagnosis and treatment of patients with fluid or electrolyte
imbalances, or both.
2. Cations
a. The monovalent cations Na+ and K+ are the predominant cations of the ECF and ICF
compartments, respectively. Essentially all of the body K+ is in the exchangeable pool,
whereas only 65%-70% of the body Na+ is exchangeable. Only the exchangeable solutes
are osomotically active.
b. The divalent cations Mg2+ and Ca2+ exist in body fluids in relatively low concentrations.
Almost all of the body Ca2+ (in bone) and most of the body Mg2+ (in bone and cells) is
nonexchangeable. After K+, Mg2+ is the main cation ofthe ICF. After Na+, Ca2+ is the main
cation of the ECF.
3. Anions. The chief anions of the body fluids are CI-, HC0 3 - , phosphates, organic ions, and
polyvalent proteins.
a. Organic phosphates, proteins, and organic ions are the predominant anions in the ICF.
b. CI- and HC03 - are the predominant anions in the ECF.

350

= Cations

~ = Anions

Creatinine .

= Nonelectrolytes

300

c:
.Q

'

250

CI.l

'-'

200

Urea

150

CI~

100

x
H2P04~

50
SO;~

\~~2Z!i:!II',Org

Mg'-=~~~Prot"-

SO.'~
'.

Mg2.
Na.

Prot~

I--Extracellular fIUid--1 Llntracellular

'.

Org ac"-

flUid~ I , - - - - - - u r i n e - - - - - - - '

Figure 4-5. Millimolar and osmolal concentrations of body fluids. Total electrolyte concentrations are expressed in
mmol/L to emphasize the qualitative heterogeneity of the body fluids. In the ECF, the cation and anion concentrations
are almost equal and exert similar osmotic effects. In the ICF, the millimolar cation concentration is greater than the
anion concentration, and cations exert a proportionately greater osmotic effect. However, the osmotic activity of the
ECF and the ICF are equal. Although the ICF has a higher ion concentration than the ECF, there is no difference in
osmolality because many of the cellular ions are bound to protein and are osmotically inactive. X- = undetermined
anions. The shaded bars represent concentrations measured in mOsm/kg water; the unshaded bars represent concentrations measured in mmol/L.

Table 4-3A. Electrolyte Concentration of Body Fluids


Plasma
Ion

(mg/L)

Cations
Na+
K+
Ca2+
Mg2+
NH4+
Total cations
Anions

CIHC03 Phosphate"
Protein
SO/-t
Organic acids
Total anions
Total electrolytes

(mEq/L)

(mmol/L)

Interstitial
Fluid
(mEq/L)

ICF
(mEq/L)

Urine
(mEq/L)

50-130
20-170
5-12
2-18
30-50
107-380

3266
156
100
24

142
4
5
2

142
4
2.5
1

145
4
3
2

10
145

3546

153

149.5

154

195

3692
1525
106
65,000
16
175
70,514
74,060

104
25
2
15
1
6
153
306

104
25
1.1
1.1
0.5
3
134.7
284.2

117
27
2
1
1
6
154
308

5
10
100
60
20

40

195
390

50-130
20-40
30-45
20-50 t
120-265
227-645

Note-At the pH of body fluids, the proteins have multiple charges per molecule (average valence of -15). Hence,
the ICF has more total charges than does the ECF. The total concentration of cations in each compartment must equal
the total concentration of anions in each compartment when expressed in mEq/L. (After Frisell WR: Intra- and
extracellular electrolytes. In Acid-Base Chemistry in Medicine. New York, Macmillan, 1968, p 5.)
*HPO/- (ECF, ICF); H 2 P0 4 - (urine).
lAs free sulfur.
lAssuming average valence of -2.

Table 4-3B. Nonelectrolyte Concentration of Body Fluids


Plasma
Solute
Urea
Creatinine
Osmolarity

Urine

(mg/L)

(mmol/L)

(mg/L)

(mmol/L)

150
11

2.5
0.08

12,000--24,000
678-2260

200--400
6-20
500-800

III. OVERVIEW OF RENAL TUBULAR FUNCTION. The constancy of the body's internal environment is maintained, in large part, by the continuous functioning of its roughly 2 million nephrons. As
blood passes through the kidneys, the nephrons clear the plasma of unwanted substances (e.g.,
urea) while simultaneously retaining other, essential substances (i.e., water). Unwanted substances
are removed by glomerular filtration and renal tubular secretion and are passed into the urine.
Substances that the body needs are retained by renal tubular reabsorption (e.g., Na+, HC03 -) and
are returned to the blood by reabsorptive processes.

A. Glomerular filtration is the initial step in urine formation. The plasma that traverses the glomerular
capillaries is filtered by the highly permeable glomerular membrane, and the resultant fluid, the
glomerular filtrate, is passed into Bowman's capsule. Glomerular filtration rate (GFR) refers to
the volume of glomerular filtrate formed each minute by all of the nephrons in both kidneys.
B. Renal tubular secretion and reabsorption (Table 4-4) refer to the direction of transport and not
to the differences in the underlying mechanisms of transport (see III DJ.
1. Secretion refers to the transport of solutes from the peritubular capillaries into the tubular
lumen.
2. Reabsorption denotes the active transport of solutes and the passive movement of water from
the tubular lumen into the peritubular capillaries.

Table 4-4A. Tubular Function: Proximal Tubules


Reabsorption

Na+
K+
Ca2+
Mg2+

HPO/SO/N0 3 -

Secretion

Active

Passive

Nonreabsorption

Active

Urate
Glucose
Amino acids
Protein
Acetoacetate
13- H ydroxybutyrate
Vitamins

CI-

Inulin
Creatinine
Sucrose
Mannitol

H+t
Urate
PAH
Penicillin
Sulfonamides
Creatinine

HC0 3 -t

HPO/Water
Urea

Passive

"Most secretion involves active transport into the proximal tubule.


tCoupled with Na+ secretion.
tHC0 3 - reabsorption occurs by H+ secretion, and, in this sense, HC03 - is active. However, CO 2 , formed by the
association of H+ and HC03 - in the tubular lumen, diffuses into the tubular cell passively and forms HC03 - within
the cell, which is passively reabsorbed. HC03 - is largely reabsorbed into the tubular cell as CO 2

Table 4-4B. Tubular Function: Distal Tubules


Reabsorption
Active

Passive

Na+
Ca 2 +
Mg2+
Urate

CI-

Secretion
Nonreabsorption

Active

Passive

Urea'

HC0 3 Water

"Urea is passively reabsorbed from the thick segment of the ascending limb of the loop of Henle and from the lower
part of the collecting tubule.
tCoupled with Na+ reabsorption.
tK+ secretion in the distal tubule occurs primarily by passive electrical coupling of Na+-K+ on the luminal side and
secondarily by active Na+-K+ exchange on the peritubular side of the cell.

C. Capillary membrane transport in the kidney


1. Convection is the bulk transport of a fluid with its dissolved small solutes across a membrane.
a. Convection is caused by a hydrostatic pressure difference between the glomerular capillaries and Bowman's capsule.
b. Since there are no concentration gradients for inorganic ions, glucose, amino acids, and
insulin, the concentrations in plasma and the ultrafiltrate in Bowman's space are the same.
c. In the absence of concentration gradients there can be no diffusion.
d. The filtered load' of the solutes is proportional only to the GFR and to the forces that affect
filtration, namely hydrostatic and oncotic pressures.
2. Simple diffusion is the movement of solutes from areas of higher concentration to areas of
lower concentration. The movement of the individual solute particles is random, and the
speed of the movement increases with increased temperature. While diffusion ceases when
equilibrium is reached, the random motion of the particles continues.
a. Diffusion is the principal mode of transport in the interstitial space and within cells.
b. For an ion, the driving force for diffusion consists of two gradients: a concentration gradient
and an electric (voltage) gradient.
D. Renal epithelial transport (Figure 4-6)
1. Basolateral (abluminal) membrane transport systems
a. Primary active transport mechanism of the Na + -K+ -ATPase pump system. Na + transport through the basolateral membrane is mediated principally by the Na+-K+-ATPase

'Filtered load is the amount of solute transported across the glomerular membranes per unit time. Mathematically,
it is equal to the GFR times the plasma concentration of that solute.

Lumen

Cell

Apical
membrane

Interstitial
fluid

Tight junction

Basement
membrane
Intercellular channel

,/'"
Basolateral
membrane

Water~===~)

+-

Facilitated/ "
diffusion
O-Glucose
Cotransport
Figure 4-6. Transport systems of the proximal tubule. Sodium (Na+) enters the apical (adluminal) membrane and is
actively transported across the basolateral (abluminal) membrane by Na+ -K+ -ATPase. Although Na+ influx in the
tissue is favored by the electrochemical potential difference, it is mainly carrier-mediated. Water also follows this
transcellular route taken by Na +. Other solutes (o-glucose) enter the cell against a concentration gradient by coupling
to a Na+ carrier in the apical membrane. This transport of glucose is called secondary active transport because the
uphill transport of glucose is driven by the energy provided by the transport of Na+ down its electrochemical gradient.
(Reprinted from Marsh DJ: Renal Physiology. New York, Raven, 1983, p 68.)
pump, which transports Na+ against an electrochemical potential from the cell interior and
maintains a low intracellular Na + concentration. *
b. Facilitated diffusion. This system translocates glucose from the intracellular fluid across
the membrane to the interstitial fluid. Facilitated diffusion involves the passive transport of
a substance by a protein carrier from a region of higher concentration to a region of lower
concentration.
2. Apical (adluminal) membrane transport systems. The apical surface of the renal epithelium
of the proximal tubule differs from the other cell membranes in that simple diffusion plays
almost no role in the transport of Na+, K+, CI~, H+, glucose, or amino acids. The movement
of Na+ across the luminal membrane (influx) is favored by the electrochemical gradient, but
the bulk of Na+ transport is mediated by specific membrane transport proteins and not by
simple diffusion. The apical surface does possess two transport mechanisms for Na+.
a. A diffusion mechanism is available for Na+ to pass through the tight junction in association
with CI~. Most of the CI~ that is reabsorbed does not enter the cell at all.

*Active transport of Na + (efflux) across all cells of the body consumes


metabolism in most cells.

30%~50%

of the energy derived from

b. Carrier-mediated mechanism. The transport of Na + down its gradient provides energy for
active (uphill) transport of other solutes (e.g., glucose, amino acids). Once inside the cell,
glucose diffuses to the basolateral membrane, where it enters the interstitial fluid by
facilitated diffusion. Na+-dependent transport mechanisms are of two types and are also
called secondary active transport systems.
(1) Cotransport (symport) denotes the transport of two substances by a protein carrier in
the same direction.
(2) Countertransport (anti port) defines the transport of two substances by a protein carrier
in opposite directions.
3. Transepithelial transport. The differences between apical and basolateral membrane properties account for the transepithelial transport of all solutes, which can occur via the transcellular
or the paracellular pathway.
a. The transcellular pathway is used for active transepithelial transport. Approximately twothirds of the Na+ transport is active and transcellular. There are three principal mechanisms
of Na+ transport in the proximal tubule. All involve protein carriers that also combine with
Na+ and thus are secondary active transport processes. These are:
(1) Electrogenic Na+ cotransport (symport) with solutes such as sugars and amino acids
(2) Electroneutral Na+ countertransport (anti port) with H+
(3) Electroneutral Na+ cotransport (symport) with phosphate and organic anions such as
acetate, citrate, and lactate
b. The paracellular (intercellular) pathway is used for passive transepithelial transport; this
pathway consists of lateral intercellular spaces. About one-third of NaCi transport is passive
and paracellular.
4. Osmosis is the process of net diffusion of water from a region of higher water activity (or lower
solute concentration) to a region of lower water activity (or higher solute concentration).
a. Osmosis is the only important mechanism of water movement across cell membranes.
b. Virtually all cells are permeable to water.
E. Energy requirements for membrane transport
1. The energy required for the secondary active transport of glucose, amino acids, H +, and organic
ions is derived from the Na + concentration gradient.
2. The energy for the work of primary active transport is provided by the A TP. The Na + -K + -ATPase
pump maintains the Na+ gradient across the membrane by maintaining the Na+ concentration
difference across the membrane.

IV. RENAL CLEARANCE (Figure 4-7)


A. Definitions
1. Clearance is a measure of the volume of plasma completely freed of a given substance per
minute by the kidneys. It is the efficiency with which the plasma is cleared of a given substance.

2 6'
I:
Q)

~oh;PP""oacld
\

200

C,)

c:
~

~atinine,

Q)

100

Inulin

L -_ _ _L -_ _ _ _ _ _ _ _ _ _ _ _ _ _

Plasma concentration (mg/dl)

Figure 4-7. Clearances of several substances plotted


against their plasma concentrations. Units of plasma concentrations vary over a wide range so that values on
the abscissa are relative only. Note that the ordinate is
interrupted, and para-aminohippuric acid (PAH) clearance at low plasma concentration is greater than 600 mil
min. (Reprinted from Bauman JW Jr, Chi nard FP: Measurement of renal integrity. In Renal Function: Physiological and Medical Aspects. St. Louis, CV Mosby, 1975,
p 43.)

2. Renal clearance of a given substance is the ratio of the renal excretion rate of the substance
to its concentration in the blood plasma. This ratio is expressed as

Ux x

(2)

=--

where Cx = clearance of the substance (ml/min); U x and Px = concentration (mg/mi) of the


substance in urine and plasma, respectively; V = urine minute volume or volume of urine
output per minute (ml/min); and U x V = amount of substance excreted per unit time (mg/
min). [Equation (2) is an expression of the indicator dilution principle, which is discussed in II

B1.]
a. Equation (2) can be used to measure the clearance of any substance that is present in the
plasma and excreted by the kidneys.
b. Plasma concentrations rather than whole blood concentrations are used in calculations of
renal clearance, because only the plasma is cleared by filtration. An analysis of this relationship demonstrates that the unit for clearance is ml/min, as
Ux x V
(mg/mi) (ml/min)
C = --- =
= ml/min
x
Px
(mg/mi)
c. Because substances can be cleared by filtration, tubular secretion, or a combination of these,
the magnitude of the tubular clearance depends on the plasma concentration and the tubular
transport capacity (Tm) for reabsorption or secretion. (The phenomenon of Tm is discussed
in more detail in VI A 1.)
3. Amount filtered and excreted. Clearance is a comparison of the amount of substance removed
from plasma per unit time (C x x Px ) with the amount of substance excreted in the urine per
unit time (U x x V), which is demonstrated by a simple rearrangement of equation (2); that is
C x x Px

Ux x

(3)

8. Inulin clearance. Inulin, a fructo-polysaccharide that does not occur naturally in the body, can
be used in a test for determining renal function.

1. As a measure of GFR
a. Inulin clearance (C in ) is a measure of GFR because the volume of plasma completely cleared
of inulin per unit time equals the volume of plasma filtered per unit time (i.e., Cin = GFR).
The following characteristics of inulin account for this quality.
(1) Inulin is only freely filtered. Because no inulin is secreted or reabsorbed, all excreted
inulin must come from the plasma.
(2) Inulin is biologically inert and nontoxic.
(3) Inulin is not bound to plasma proteins, and it is not metabolized to another substance.
b. Although inulin is used most commonly, other substances can measure GFR. The most
frequently used agents include mannitol, sorbitol, sucrose (intravenous), iothalamate, radioactive cobalt-labeled vitamin Bw 5 1Cr-labeled edetic acid (EDTA), and radioiodine-labeled
Hypaque. (Endogenous creatinine clearance is used clinically as an estimate of GFR, as
discussed in IV D 2 a, because a small amount of creatinine is secreted in humans.)
2. As an indicator of plasma clearance mechanisms. A comparison of the clearance of a given
substance (C x ) with the clearance of inulin (C in ) provides information about the renal mechanisms used to remove the substance from plasma.
a. When C. equals C in , excretion is by filtration alone. In terms of equation (3), this means
that the mass of the substance excreted in the urine per unit time equals the mass of the
substance filtered during the same time, or

Ux x

V = Cx

x P/

'The mass filtered per unit time (also called the amount filtered or filtered load) is equal to GFR x Px (or
and is expressed in mg/min (see VI A 3).

Cin

X P.)

b. When C. is less than Cin , excretion is by filtration and reabsorption. In this case, the
mass of the substance excreted in the urine is less than the mass of the substance filtered
during that time, or
Ux x

V<

Cx x Px

c. When C. is greater than Cin , excretion is by filtration and secretion. In this case, the mass
of the substance excreted in the urine is greater than the mass of the substance filtered
during the same time, or

Ux x

V>

C x x Px

C. Application of the clearance concept to tubular function (Figure 4-8)

1. Clearance ratios
a. Definition. Clearance ratio is the ratio of the clearance of any substance (Cx) to the clearance
of inulin (C in ). *
(1) Clearance ratio has the formula

Cx

Ux x

Cin

Px

----

U in X

Pin

(2) Clearance ratio is a double ratio (i.e., a ratio of two ratios), and it can be determined
without measuring urine flow
ance ratio is reduced to

(V)

[see equation (2)]. Because

V cancels out,

Cx
U x U in
-=--:--

C in

Px

Pin

the clear-

(4)

b. Tubular clearance ratio


(1) The local tubular clearance of a solute compared to that of inulin at a specific site along
the nephron is termed the tubular clearance ratio. This relationship is a reexpression of
equation (4)

TFx

TF

-x - - - -in
Cin
Px
Pin
where TF = the tubular fluid concentration at the local site of micropuncture. Micropuncture techniques have made it possible to analyze fluids collected with micropipettes
(outer diameter = 6--10 JLm) from segments of the nephron. This technique permits the
quantitative analysis of the tubular transport process along the length of a given single
nephron segment.

o
I

50

100
I

Percent tubular length

10.0

CK'"

1.0

Figure 4-8. Idealized ratios of K+ clearance (CK +) to the


clearance of inulin (C;n), as a function of tubular length.
Since inulin is neither secreted nor reabsorbed, an increase in the CK + IC;n ratio represents tubular reabsorption of K+. In evaluating solute transport, the TF IP concentration ratio of a solute divided by the TF I P
concentration ratio of inulin has the same significance as
the clearance ratio. In fact, the TF I P clearance ratio is
the clearance ratio for the site of sampling. When the
TF I P concentration ratio of a solute is divided by the
TF I P concentration ratio of inulin, it is called the tubular
clearance ratio. (Reprinted from Rose BD: Evaluation of
micropuncture data. In Clinical Physiology of Acid-Base
and Electrolyte Disorders. New York, McGraw-Hili,

1977, p 67.)

0.1
'Creatinine clearance also can be used in the clearance ratio instead of inulin clearance (see IV D).

(2) The tubular clearance ratio is used for two purposes.


(a) It is used to measure the fraction of a solute remaining in the lumen at a specific
tubular segment along the nephron.
(b) It is used to determine the nature of tubular solute transport (Le., reabsorption or
secretion). Changes in CJC in ratios represent net solute reabsorption from, or solute
secretion into, the lumen because inulin is neither reabsorbed nor secreted. Thus,
to examine solute transport, the effect of water transport is eliminated by comparing
the TF I P solute concentration to the TF I P concentration of a substance that remains
in the tubule (Le., inulin).

c. Interpretation of clearance ratios


(1) Case 1: When C./C;n = 1. A ratio of 1 indicates that the amount of the substance
excreted per unit time equals the amount of the substance filtered in the same time.
This is simply a restatement of equation (3).
Ux x

V=

Cin

Px

(a) A clearance ratio of 1 indicates that, on a net basis, the substance is neither
reabsorbed nor secreted and, therefore, is only filtered.
(b) Substances with clearance ratios close to 1 include mannitol, sorbitol, thiosulfate,
ferricyanide, iothalamate, vitamin B]2< and sucrose (intravenous).
(2) Case 2: When C./C in < 1. A clearance ratio of less than 1 demonstrates that the
amount of the substance excreted per unit time is less than the amount filtered per unit
time, or

Ux x V < Cin X Px
(a) A clearance ratio of less than 1 indicates that, on a net basis, the substance
undergoes reabsorption.
(b) Substances with clearance ratios of less than 1 include glucose, xylose, and fructose.
(3) Case 3: When C./C;n > 1. A clearance ratio that is greater than 1 indicates the net
excretion of the substance over time is greater than the quantity filtered over time, or

V > Cin

Px
(a) A clearance ratio of more than 1 represents net secretion of the substance into the
Ux x

lumen; therefore, the substance is cleared by filtration and secretion.


(b) Substances with clearance ratios greater than 1 include para-aminohippuric acid
(PAH), phenol red, iodopyracet, certain penicillins, and creatinine (in humans).
2. Tubular fluid concentration ratio (Figure 4-9)
a. Definition. A comparison of the tubular fluid (TF) solute concentration to the plasma (P)
solute concentration is called the TF IP concentration ratio. The TF I P concentration ratio
is not compared to that of inulin and is, therefore, a single ratio.

b. Uses of the TF/P concentration ratio


(1) This ratio measures the tubular solute concentration along the nephron, which is
a function of solute transport (Le., reabsorption or secretion) as well as water reabsorption. The following example illustrates this function.
(a) The TF/P concentration ratio for K+ (TF/P [K+]) increases in the collecting duct,
which indicates either water reabsorption or K+ secretion.
(b) However, the C K + IC in (Le., clearance ratio) remains constant, indicating that K+ is
neither secreted nor reabsorbed (see Figure 4-8).
(c) Therefore, the elevated TF/P [K+] represents water reabsorption in the collecting
duct.
(2) TF I P solute concentration ratios also can represent total solute concentration rather
than the concentration of a single solute. When measuring the total solute concentration,
the TF I P or urine-to-plasma (U I P) ratio is expressed in terms of osmolality (osmolarity)
and can be interpreted as follows.
(a) Case 1: When TFosm/Posm = 1 or Uosm/Posm = 1, the tubular fluid or urine is
isosmotic with respect to plasma.
(b) Case 2: When TFosm/Posm or Uosm/Posm < 1, the tubular fluid or urine is hyposmotic with respect to plasma.
(c) Case 3: When TFosm/Polm or UOlm/Po.m > 1, the tubular fluid or urine is hyperosmotic with respect to plasma.
(d) Example. States of hydropenia (dehydration) lead to ADH secretion. This, in turn,
leads to increased water reabsorption and an elevation of Uos m' which has an upper

5.0

R S><l
~\\\\\\1

4.0

Secretion
Reabsorption

3.0

2.0

TF
p
1.4

1.0 1t~~~iW~~~~n
0.8

0.6
0.4

0.2

o
o

75
50
25
% Proximal tubule length

100

Figure 4-9. Transport of various solutes along the length


of the proximal tubule. TFIP = tubular fluid-to-plasma
concentration ratio for a particular solute. A TF / P for
inulin approaches 3 at the end of the proximal tubule,
indicating that approximately two-thirds of the filtered
fluid is normally reabsorbed in the proximal tubule. Inulin
is neither reabsorbed nor secreted. PAH = paraaminohippuric acid; OSM = osmolarity or osmolality,
which denotes the total concentration of all solutes in the
tubular fluid. (Reprinted from Brenner B, Coe FL, Rector
FC: Transport functions of renal tubules. In Renal Physiology in Health and Disease. Philadelphia, WB Saunders,
1987, p 33.)

limit in humans of about 1400 mOsm/kg. With a Posm of about 300 mOsm/kg, this
is equivalent to a maximal Uosm/Posm of approximately 4.7.

D. Creatinine clearance and creatinine concentration

1. Normal levels and the effect of aging


a. Creatinine clearance has a normal range of 80-11 0 ml/min per 1.73 m 2 body surface area
(estimated average body surface area of 25-year-old humans) and declines with age in
healthy individuals. Because creatinine clearance is an index of GFR (see IV D 2 a), normally
GFR also declines with age.
b. Plasma creatinine concentration remains remarkably constant through life, averaging
0.8-1.0 mg/dl in the absence of renal disease.
c. Because creatinine clearance normally declines with age and plasma creatinine concentration does not, the decline in clearance is attributed to a parallel decrease in the excretion
rate of creatinine. (The decline in creatinine clearance is not due to a decreased tubular
secretion of creatinine, because the clearance ratio of creatinine to inulin is a quite constant
1 .2, about 120% of inulin clearance.) Thus, urinary creatinine excretion exceeds the amount
filtered, since approximately 20% of the urinary creatinine is derived from tubular secretion
by the secretory pathway in the proximal tubule.
d. The decline in creatinine excretion with age is due to a primary decline in renal function
and a secondary decline in muscle mass.
(1) The decline in GFR is due to declines in renal plasma flow, cardiac output, and renal
tissue mass. (Decreased GFR is the most clinically significant renal functional deficit
occurring with age.)
(2) The decline in creatinine excretion with a decline in muscle mass over time accounts
for the relatively constant plasma creatinine concentration among healthy individuals.

2. Estimating GFR
a. Creatinine clearance
(1) Although inulin clearance can be used to measure GFR, in clinical practice it is more

common to determine the 24-hour endogenous creatinine clearance as an estimate of


GFR.
(2) Creatinine clearance determinations do not require administration of exogenous creatinine, as creatinine is a product of muscle metabolism.
b. Creatinine concentration. The tendency to rely on serum creatinine concentration rather
than creatinine clearance as an estimate of renal function is a serious error, because plasma
creatinine concentration overestimates renal function (i.e., GFR).
(1) Using serum creatinine concentration as a basis for drug dosages can lead to drug
overdose, especially in elderly individuals. Overdose is a particularly serious risk with
drugs that are cleared primarily by renal mechanisms (e.g., digoxin and aminoglycoside
antibiotics>. The dosages for these drugs frequently are based on serum creatinine
concentration, although it is incorrect to do so.
(2) To avoid this error, patient age should be considered together with measurements of
creatinine clearance and blood levels of the drug. Drug dosages should be adjusted to
the 30%-40% decrease in GFR that normally occurs in individuals 30-80 years old.

v.

GLOMERULAR FILTRATION AND GFR


A. Factors affecting glomerular filtration
1. Role of hydrostatic and oncotic pressures. Fluid movement is proportional to the membrane
permeability and to the balance between hydrostatic and osmotic (oncotic) forces across the
glomerular membrane. When hydrostatic exceeds oncotic pressure, filtration occurs. Conversely, when oncotic exceeds hydrostatic pressure, reabsorption occurs.
a. Effective filtration pressure (EFP) refers to the net driving forces for water and solute
transport across the glomerular membrane. EFP is a function of two variables: the hydrostatic pressure gradient driving fluid out of the glomerular capillary and into Bowman's
capsule and the colloid osmotic pressure gradient bringing fluid into the glomerular
capillary. Using these variables, EFP is expressed as
EFP

(P eap - PBow )

COPeap*

where (P eap - PBow ) = the outward forces promoting filtration, and COPeap = the inward
forces opposing filtration.
b. The filtration coefficient (Kf ) of the glomerular membrane is a function of the capillary
surface area and the membrane permeability. Kf is expressed in ml/min/mm Hg.
c. Using EFP and Kf , then, the GFR is expressed as
GFR

Kf x EFP

(1) Kf normally equals about 12.5 ml/min/mm Hg.

(2) EFP normally equals about 10 mm Hg, which can be determined from the normal values
for Peap (45 mm Hg), PBow (10 mm Hg), and COP eap (25 mm Hg) using the equation for
EFP.
(3) The normal GFR for both kidneys is about 125 ml/min per 1.73 m 2 of body surface
area.

2. Role of capillaries. Although the principle of opposing forces accounts for the magnitude and
direction of flow, the main mechanism that alters intracapillary hydrostatic pressure probably
is not the resistance along the length of the capillary but the contractile state of the precapillary

sphincters.
B. Glomerular filtration and systemic filtration: a comparison. The amount of filtration performed
by the renal glomerular capillaries is better appreciated when this filtration is compared to that of
the extrarenal (systemic) capillaries.

*The colloid osmotic pressure in Bowman's capsule is not expressed in this equation, although it is understood to be
zero, because the glomerular filtrate normally contains little protein.

1. Capillary exchange area and Kf


a. The total glomerular capillary exchange area is estimated to be 0.5-1.5 m 2 / 100 g of
renal tissue.
b. In the adult human, if the entire systemic capillary bed were patent at a given time, it would
afford a total capillary exchange area of 1000 m 2 . In a human at rest, approximately 25%35% of the systemic capillaries are open at any time, and the effective pulmonary capillary
surface area is about 60 m 2 .
c. The Kf of the glomerulus is 50-100 times greater than that of a muscle capillary.

2. GFR and systemic filtration rate


a. About 20 L of fluid are filtered daily from the systemic capillaries. Of this, 16-18 L/ day are
reabsorbed in the venular ends of the capillaries, and the remaining 2-4 L/day represent
lymph flow. (Diffusional exchange across the entire systemic capillary bed, however, is
about 80,000 L/day!)
b. Approximately 180 L of fluid are filtered daily from the glomerular capillaries. Therefore,
the transtubular flow of fluid out of the glomerular capillaries (GFR = 180 Llday) far exceeds
the filtration from systemic capillaries (20 L/day).

C. Determinants of GFR. The major determinant of GFR is the hydrostatic pressure within the
glomerulus. In addition, the renal blood flow (RBF) through the glomeruli has a great effect on
GFR; when the rate of RBF increases, so does GFR. The following factors determine the RBF.

1. Autoregulation of arterial pressure (Figure 4-10)


a. Over a wide range of renal arterial pressures (i.e., 90-190 mm Hg), the GFR and RBF remain
quite constant. This intrinsic phenomenon observed in the renal capillaries also occurs in
the capillaries of muscles and is termed autoregulation. Intrarenal autoregulation is virtually
absent, however, at mean arterial blood pressures below 70 mm Hg.
(1) Autoregulation has been observed to persist after renal denervation, in the isolated
perfused kidney, in the transplanted kidney, after adrenal demedullation, and even in
the absence of erythrocytes.
(2) Autoregulation of RBF is necessary for the autoregulation of GFR.

GFR
....................

50

100

(ij

200

..... y............ ..

Q)

a:

150

.. j....

50
100
150
200
Mean arterial blood pressure
(mm Hg)

.. '

Figure 4-10. Autoregulation of renal blood flow (RBFJ


and glomerular filtration rate (CFR) and the effect of arterial blood pressure on RBF, renal plasma flow (RPFJ, GFR,
and volume flow of urine (0. (Reprinted from Harth 0:
The function of the kidneys. In Human Physiology. Edited
by Schmidt RF and Thews G. New York, Springer-Verlag,
1983, p 615,)

b. This phenomenon probably is mediated by changes in preglomerular (afferent) arteriolar


resistance. The major factors that determine blood flow (F) are a pressure gradient (ilP) and
resistance (R). This relationship is expressed as
ilP

F =R
It becomes clear that to maintain a constant blood flow with a concomitant increase in renal
perfusion pressure (ilP), there must be a commensurate increase in renal vascular resistance.

2. Sympathetic stimulation (Table 4-5)


a. In addition to the autoregulatory response to hypotension (i.e., vasodilation), the reflex
increase in sympathetic tone results in both afferent and efferent arteriolar vasoconstriction
and an increase in renal vascular resistance together with a decrease in GFR.
b. Conversely, the autoregulatory response to increases in renal arterial blood pressure (i.e.,
vasoconstriction) causes the reflex decrease in sympathetic tone, which results in renal
arteriolar vasodilation and a decrease in renal vascular resistance together with an increase
in GFR.

3. Arteriolar resistance
a. The afferent arteriole is the larger diameter arteriole and the major site of autoregulatory
resistance. When resistance is altered in the afferent arterioles, GFR and RBF change in the
same direction. Therefore, changes in afferent arteriolar resistance do not affect the
filtration fraction [i.e., the ratio of GFR to renal plasma flow (RPF)). *
(1) Constriction of the afferent arteriole decreases both RPF and GFR.
(a) An increase in vascular resistance proximal to the glomeruli leads to a decrease in
RPF.
(b) A decrease in hydrostatic pressure within the glomerular capillaries leads to a
decrease in GFR.
(2) Dilation of the afferent arteriole increases both RPF and GFR.
(a) A decrease in vascular resistance proximal to the glomeruli leads to an increase in
RBF.
(b) An increase in glomerular capillary perfusion pressure leads to an increase in GFR.
b. When resistance is altered in the efferent arterioles, GFR and RPF change in opposite
directions. Therefore, changes in efferent arteriolar resistance do affect the filtration

fraction.
(1) Constriction of the efferent arteriole decreases RPF and increases GFR.
(a) An increase in vascular resistance distal to the glomeruli leads to a decrease in RPF.

Table 4-5. Effect of Changes in Renal Vascular Resistance with a Constant


Renal Perfusion on GFR, RPF, and FF

Renal Arteriolar
Vascular Resistance
Afferent

Efferent

GFR

RPF

(ml/min)

(ml/min)

t
t
!

t
!
t

FF
no change
no change
t
U

Note-GRF = glomerular filtration rate, RPF = renal plasma flow, and FF = filtration
fraction. Note that GFR and RPF exhibit parallel shifts with changes in afferent arteriolar
resistance but exhibit divergent shifts with changes in efferent arteriolar resistance. Increases in vascular resistance always lead to a decline in RPF, and decreases in arteriolar
resistance always lead to an increase in RPF. In the first two columns, upward arrows
denote the effect of vasoconstriction, and downward arrows denote the effect of vasodilation.

Filtration fraction is a measure of the fraction of the entering plasma volume (RPF) that is removed through the
glomeruli as filtrate. Filtration fraction refers to the bulk volume of fluid (not solutes) and normally measures about
20% of RPF.

(b) An increase in the hydrostatic pressure within the glomerular capillaries leads to an
increase in GFR.
(c) Following an increase in efferent arteriolar resistance, the filtration fraction increases.
(2) Dilation of the efferent arteriole increases RPF and decreases GFR.
(a) A decrease in vascular resistance distal to the glomeruli leads to an increase in RPF.
(b) A decrease in the hydrostatic pressure within the glomerular capillaries leads to a
decrease in GFR.
(c) Following a decrease in efferent arteriolar resistance, the filtration fraction decreases.

VI. RENAL TUBULAR TRANSPORT: REABSORPTION AND SECRETION


A. Active transport processes
1. Renal tubular transport maximum (Tm) refers to the maximal amount of a given solute that
can be transported (reabsorbed or secreted) per minute by the renal tubules.

a. The highest attainable rate of reabsorption is called the maximum tubular reabsorptive
capacity and is designated Tm or Tr. Substances that are reabsorbed by an active carriermediated process and that have a Tm include phosphate ion (HPO/~), SO/~, glucose
(and other monosaccharides), many amino acids, uric acid, and albumin, acetoacetate,
,8-hydroxybutyrate and a-ketoglutarate.
b. The highest attainable rate of secretion is called the maximum tubular secretory capacity
and is designated Tm or Ts. Substances that are secreted by the kidneys and that have a
Tm include penicillin, certain diuretics, salicylate, PAH, and thiamine (vitamin B1 ).
c. Uric acid is the only organic substance to be both reabsorbed and secreted by the kidney.
K+ is the only inorganic cation that is both reabsorbed and secreted by the kidney.
d. Some solutes have no definite upper limit for unidirectional transport and, hence, have
no Tm.
(1) The reabsorption of Na+ along the nephron has no Tm.
(2) The secretion of K+ by the distal tubules has no Tm.

2. Threshold concentration, which refers to the plasma concentration at which a solute begins
to appear in the urine, is characteristic of that substance.

3. Filtered load and excretion rate


a. Definitions. Reabsorption and secretion are not directly measured variables but are derived
from the measurements of the amount of solute filtered (filtered load) and the amount of
solute excreted (excretion rate).
(1) The filtered load is the amount of a substance entering the tubule by filtration per
unit time and is mathematically equal to the product of GFR (or C in ) and the plasma
concentration of the substance (P.), as
filtered load = GFR x Px; (ml/min) (mg/ml) = mg/min
= C in X Px ; (ml/min) (mg/ml) = mg/min

(2) The excretion rate is the amount of a substance that appears in the urine per unit
time and is mathematically equal to the product of urine flow rate (V) and the urine
concentration of the substance (U x ), as
excretion rate = U x x

V;

(mg/ml) (ml/min) = mg/min

b. Secretion. If the excretion rate exceeds the filtered load [or if the clearance of the substance
(C x) is greater than Cinl, net tubular secretion of that substance has occurred. Thus, secretion
is expressed as

Ux x

V>

C in

Px or Cx > Cin

c. Reabsorption. If the filtered load exceeds the excretion rate (or if Cx is less than Cin ), net
reabsorption of that substance has occurred. Thus, reabsorption is expressed as
Cin

Px > Ux x

V or Cx <

Cin

d. Calculation of Tm (Tr or Ts). Tm is the difference between the filtered load (Cin
the excretion rate (U x x

'if).

Tr and Ts, then, are expressed as

Tr
Ts

=
=

C in X Px - U x x V (in mg/min)
U x x V - Cin X Px (in mg/min)

P.) and

B. Graphic representation of renal transport processes: renal titration curves (Figure 4-11)
1. Construction of renal titration curves
a. A titration curve is constructed by plotting the following pairs of variables:*
(1) The filtered load (C in X Px) against the plasma concentration (Px)
(2) The excretion rate (U x x \/) against Px
(3) The difference between the filtered load and the excretion rate (i.e., Tr or Ts) against Px
b. The plotted renal titration curve, therefore, can be used to determine the plasma concentration (Px) at which the renal tubular membrane carriers are fully saturated (Tm).

A. Glucose titration curve

B. PAH titration curve


150

600
"2

"2

'E

500

C,

Ol

.s 100

.s 400
15

g.
c:

15

Q.

300

Ul

c:

~
::c 50

~Q) 200

Ul

Il..

g 100

10

20

30

Plasma PAH concentration (mg/dl)

Plasma glucose concentration (mg/dl)


C. Clearance curves

700
600
"2500

'E

400

Q)

g
a;

III

()

300
200
Gin = GFR = 125 ml/min

100~--~---------------------

o
o

10

20

30

40

50

60

70

80

Plasma concentration
Figure 4-11. Relationships of renal titration curves and clearance curves to plasma concentration. The splay (curved
portions) observed in the glucose reabsorption and excretion curves (graph A) and the splay in the para-aminohippuric
acid (PAH) secretion and excretion curves (graph B) are due to the kinetics of tubular transport and the heterogeneity
of the nephron population in terms of number, length, and functional variations in transport. Graph e demonstrates
that the clearance of glucose (ee) increases and approaches the clearance of inulin (ein ) asymptotically as the plasma
glucose concentration increases beyond the tubular transport maximum for glucose (Tm e). Conversely, graph e shows
that the clearance of PAH (epAH ) declines and approaches the C'n asymptotically as the plasma PAH concentration
increases above the tubular transport maximum for PAH (Tm pAH ). In graph C, the plasma PAH concentration is in
mg/dl and the plasma glucose concentration is plotted as one-tenth of the actual concentration (i.e., mg/dl x 10- 1 ).
= per 1.73 m 2 body surface area. (Adapted from Selkurt EE: Renal function. In Physiology, 5th edition. Edited by
Selkurt EE. Boston, Little, Brown, 1984, pp 424 and 429.)

'The dependent variables (i.e., filtered load, excretion rate, and Tr or Ts) are plotted on the ordinate as a function of
the independent variable (i.e., Px)'

2. Glucose transport: reabsorption


a. Characteristics of glucose transport
(1) Glucose is an essential nutrient that is actively reabsorbed into the proximal tubule by
a Tm-limited process. The Tm for glucose (Tmd is about 340 mg/min (or about 2 mmol/
min) per 1.73 m 2 body surface area.
(2) Because the Tmc is nearly constant and depends on the number of functional nephrons,
it is used clinically to estimate the number of functional nephrons, or the tubular
reabsorptive capacity (Tr).
b. Glucose titration curve. Figure 4-11 A illustrates that glucose transport and excretion processes are functions of the plasma glucose concentration (Pc).
(1) Increasing the Pc results in a progressive linear increase in the filtered load (C in X Pc).
(2) At a low Pc, the reabsorption of glucose is complete; hence, the clearance of glucose
(C c ) is zero (see Figure 4-11 C).
(3) When Pc in humans reaches the renal threshold concentration of 180-200 mg/dl
(10-11 mmol/U, glucose appears in the urine (glycosuria). (Note that the threshold
concentration is not synonymous with the Pc that completely saturates the transport
mechanism, which is about 300 mg/dl.)
(4) As the Tmc is approached, the urinary excretion rate increases linearly with increasing
Pc (first-order reaction kinetics).
(5) When the Tmc is reached, the quantity of glucose reabsorbed per minute remains
constant and is independent of Pc (zero-order reaction kinetics).
(6) When the Tmc is exceeded, the C c becomes increasingly a function of glomerular
filtration; therefore, the Cc approaches Cin , and the amount reabsorbed becomes a
smaller fraction of the total amount excreted.

3. PAH transport: secretion


a. Characteristics of PAH transport
(1) PAH is a weak organic acid that is actively secreted into the proximal tubule by a

Tm-limited process. PAH is a foreign substance that is not stored or metabolized and is
excreted virtually unchanged in the urine. By filtration and secretion, PAH is almost
entirely cleared from the plasma in a single pass through the kidney, if the Tm for PAH
(Tm pAH ) has not been reached. (The Tm pAH in humans is about 80 mg/min per 1.73 m 2
body surface area.)
(2) Because about 10% of the plasma PAH is bound to plasma proteins, PAH is not entirely
freely filterable, and the concentration of PAH in the plasma (PpAH) is greater than that
in the glomerular filtrate. However, PAH does not need to be dissolved in plasma to be
transported (secreted) by the carrier into the tubular fluid, and protein binding does not
diminish the effectiveness of tubular excretion.
(3) Since the Tm pAH is nearly constant, it is used clinically to estimate tubular secretory
capacity (Ts).
b. PAH titration curve. Figure 4-11 B illustrates that the filtration and secretion of PAH are
functions of the PPAH' The amount of PAH excreted per minute (U PAH x V) exceeds the
amount filtered (C in X PPAH) at any PPAH'
(1) When the PPAH is low, virtually all of the PAH that is not filtered is secreted, and PAH
is almost completely cleared from the plasma by the combined processes of glomerular
filtration and tubular secretion.
(2) When PPAH is above 20 mg/dl, the transepithelial secretory mechanism becomes saturated, and the Tm pAH is reached.
(3) When the Tm pAH is reached, the quantity of PAH secreted per minute remains constant
and is independent of PPAH'
(4) When the Tm pAH is exceeded, the C PAH becomes progressively more a function of
glomerular filtration; hence, the CPAH approaches Cin (see Figure 4-11 C), and the constant
amount of PAH secreted becomes a smaller fraction of the total amount excreted.

c. PAH clearance and RPF


(1) Fick principle. According to the Fick principle, blood flow through an organ can be
determined with the equation

BF=-Ax - Vx
where BF

the blood flow (in ml/min), R

(5)

the rate of removal (or addition) of a

substance (in mg/min) from (or to) the blood as it flows through an organ, Ax = the
concentration of that substance (in mg/mi) in the blood entering the organ, and Vx =
the concentration of that substance (in mg/mi) in the blood leaving the organ.
(2) Since C PAH can be used to measure RPF, equation (5) can be modified to
RPF

U PAH x

(6)

ApAH - VPAH
where RPF = renal plasma flow (in ml/min), U PAH V = the rate of PAH excretion (in
mg/min), and A pAH and VPAH = concentrations of PAH (in mg/mi) in the renal artery
and the renal vein, respectively.
(a) At low concentrations of PAH in arterial plasma, the renal clearance is nearly
complete; therefore, as a first approximation, the PAH concentration in renal venous
plasma may be taken to be zero.
(i) About 10%-15% of the total RPF perfuses nonexcretory (nontubular) portions
of the kidney, such as the renal capsule, perirenal fat, the renal medulla, and the
renal pelvis; therefore, this plasma cannot be completely cleared of PAH by
filtration and secretion.
(ii) Because about 10% of the PAH remains in the renal venous plasma, the RPF
calculated from C PAH underestimates the actual flow by about 10%. Accordingly,
the C PAH actually measures the effective renal plasma flow (ERPF), and
U PAH x V
ERPF =
= C PAH
(7)
A pAH
(b) Since PAH is not metabolized or excreted by any organ other than the kidney, a
sample from any peripheral vein can be used to measure arterial plasma PAH
concentration, and the equation is written as
ERPF

U PAH

C PAH
PPAH
(3) Effective renal blood flow (ERBF) is calculated from the relationship between plasma
volume (PV), hematocrit (Hct), and blood volume (BV), as*
=

--- =

PV x 100
BV=---(100 - Hct)
and, therefore,
ERBF

ERPF x 100

=---(100 - Hct)

(4) True renal plasma flow (TRPF) can be determined if the extraction ratio (E) is known,
where
E

A pAH - VPAH
A pAH

and

ERPF
TRPFx 100
TRPF = - - . TRBF = - - - E'
(100 - Hct)

C. Renal transport of common solutes and water (see Table 4-4)


1. General considerations (see Figure 4-9)
a. The proximal tubules reabsorb 70%-85% of the filtered Na +, CI-, HC0 3 - , and water and
virtually all of the filtered K+, HPO/-, and amino acids. In addition, glucose is reabsorbed
almost completely by the proximal tubules and begins to appear in the urine when the renal
threshold is exceeded. t
b. Reabsorption of water is passive, and reabsorption of solutes can be passive or active; solute
reabsorption generates an osmotic gradient, which causes the passive reabsorption of water
(osmosis).

*In humans, the C PAH is 600-700 ml/min and, when corrected to ERBF (assuming a hematocrit of 45%), is approximately 1100-1200 ml / min, which is about 20% of the resting cardiac output.
Ii.e., approximately 200 mg glucose/dl arterial plasma, or 11 mmol/l.

2. Na+ reabsorption
a. Proximal tubules. Transtubular reabsorption of Na+ across the proximal tubules occurs in
two steps (see Figure 4-6).
(1) Step 1: Through the luminal membrane. Although Na+ is transported through the
luminal (adluminal) membrane passively down an electrochemical gradient, the transport is mediated by specific membrane proteins and is not by simple diffusion. This first
step involves two processes.
(a) First, Na+ undergoes passive cotransport with the active cotransport of glucose
or amino acids. Thus, the energy for the uphill transport of glucose from lumen to
cell is derived from the simultaneous downhill movement (influx) of Na+ along a
concentration gradient, which is maintained by the primary active Na+ pump.
(b) Next, Na+ reabsorption occurs, coupled with H+ secretion across the luminal
membrane (cation exchange) to maintain electroneutrality. (Na+ transport in the
proximal segment is not associated with K+ secretion.) Because tubular H+ is
buffered by HC0 3 - , Na + influx must be electrically balanced by an anion. * This is
accomplished by passive inward diffusion of CI- down its concentration gradient
or by regeneration of HC03 - from cellular CO 2 and water in the presence of
carbonic anhydrase.
(2) Step 2: Through the basolateral membrane
(a) Na+ is actively transported through the basolateral membrane via the Na+-K+
pump. Na+ reabsorption is equivalent to CI- reabsorption + (H+ + K+) secretion.
Approximately 67% of the proximal reabsorption of Na+ is based on the active
transport process.
(b) The remaining percentage of Na+ reabsorption across the basolateral membrane of
the proximal tubule occurs passively by solvent drag.
(i) The efflux of Na+ into the intercellular spaces creates an electrical gradient for
the efflux of CI- and HC03 - , which, in turn, generates an osmotic gradient for
water transport into the interspaces. About 20 0/0 of the Na + movement is loosely
coupled to the active secretion of H +, resulting in the reabsorption of HC03 - .
(ii) The osmotic movement of water generates a small increment in hydrostatic
pressure, and some bulk flow of water containing Na+, CI-, and HC0 3 - proceeds into the peritubular capillaries.
(c) As a result of these active and passive transport processes, Na+ reabsorption together with anions along the proximal tubule allows for the isosmotic reabsorption
of fluid.
b. Distal and collecting tubules
(1) Na+ transport via cation exchange. At the luminal membrane of the distal and collecting tubules, Na+ is reabsorbed by two electrically linked cation-exchange processes. t
(a) Na+ -H+ exchange involves Na+ reabsorption and H+ secretion.
(b) Na+ -K+ exchange involves Na+ reabsorption and K+ secretion. The quantity of
Na+ reabsorbed distally is much greater than the amount of K+ secreted.
(c) These cation-exchange processes are competitive (i.e., K+ competes with H+ for
Na+) and, therefore, are not one-to-one. Both cation-exchange processes, however,
are enhanced by aldosterone (see IX B).
(2) Na+ transport via CI- reabsorption. Na+ also is transported by the distal and collecting tubules by reabsorption largely in association with CI- reabsorption.
(a) When the availability of either K+ or H+ is reduced, the exchange of the more
available ion for Na+ is increased.
(i) Example 1. In hypokalemic alkalosis induced by chronic vomiting or hyperaldosteronemia, the exchange of H+ for Na+ is greater than the exchange of K+
for Na+, the urine becomes more acidic, and the plasma becomes more alkaline.
(ii) Example 2. In hypoaldosteronemia, the reduced secretion of K+ and H+ (due to
the decreased rate of distal Na+ reabsorption) causes hyperkalemia and acidosis.
(b) The Na+-H+ exchange mechanism is not mandatory for H+ secretion.
c. Nephron. Na+ reabsorption along the nephron occurs in the following proportions:
(1) Proximal tubule (75%)

'The transcellular transport of Na + is passive.


'Only a small amount of the filtered Na+ that is reabsorbed in the distal and collecting tubules is by cation exchange
with H+ and K+.

(2) Ascending limb of the loop of Henle (22%)*


(3) Distal tubule (4%-5%)
(4) Collecting tubule (2%-3%)

3. K+ transport
a. General considerations
(1) K+ is the only plasma electrolyte that is both reabsorbed and secreted into the renal

tubules. Virtually all of the K+ is actively reabsorbed by the proximal tubule, whereas
K+ secretion is a passive process and is largely a function of the distal tubule.
(2) The net secretion of K+ by the nephron can cause the excretion of more K+ than was
filtered, as in hyperaldosteronemia and in metabolic alkalosis. (Elevated K+ excretion
also can occur, in part, as a result of decreased K+ reabsorption, as observed in the
osmotic diuresis seen with uncontrolled diabetes mellitus.)
(3) The distal tubule has the capacity for both net secretion and net reabsorption of K+,
depending on the plasma [K+] and the acid-base state of the body.
(a) Metabolic acidosis usually is associated with cellular K+ efflux, a decreased K+
secretion, and hyperkalemia.
(b) Metabolic alkalosis usually is associated with cellular K+ influx, an increased K+
secretion, and hypokalemia.
b. K+ reabsorption. K+ is completely filterable at the glomerulus, and tubular reabsorption
occurs by active transport in the proximal tubule, ascending limb of the loop of Henle, distal
tubule, and collecting duct.
c. K+ secretion involves an active and a passive process and is influenced by aldosterone.
(1) The critical step is the active transport of K+ from the interstitial fluid across the basolateral membrane into the distal tubular cell. This active transport step is associated with
active Na+ extrusion via the Na+-K+-ATPase system.
(2) The elevated intracellular [K+] achieved by the basolateral pump favors the net K+
diffusion at the luminal membrane down a concentration gradient into the tubular
lumen. The transcellular transport of K+ is passive.
(3) Aldosterone increases the activity of the basolateral Na + -K+ -ATPase pump, which
increases K+ entry into the distal cells and the [K+] gradient from cell to lumen.
Aldosterone also increases the permeability of the luminal membrane to K+, further
increasing K+ diffusion (secretion).
4. H+ secretion and HeOl - reabsorption (Table 4-6; see Figures 5-3,5-4). H+ secretion is the
process by which the filtered HC03 - is reabsorbed and the tubular fluid becomes acidified.
The process of H+ secretion and HC0 3 - reabsorption occurs throughout the nephron, except
in the descending limb of the loop of Henle.
a. H+ secretion into the tubular lumen occurs by active transport and is coupled to Na+
reabsorption; therefore, for each H+ secreted, one Na+ and one HC0 3 - are reabsorbed.
The renal epithelium secretes about 4300 mEq (mmol) of H+ daily.
(1) Approximately 85% of the total H+ secretion occurs in the proximal tubules.
(2) Approximately 10% of the total H + secretion occurs in the distal tubules, and about
5% occurs in the collecting ducts.

Table 4-6. Renal Regulation of H + and HC0 3 Ion


HC03 HPO/-*
NH4 +t
H+j

Filtered
mEq/day

Reabsorbed
mEq/day

Secreted
mEq/day

Excreted
mEq/day

4320
144
None
<0.1

4318
124

None
None

4315

4375

2
20
40
58

*Main constituent of titratable acid is H 2 PO. -.


tSecreted mainly as NH3 and NH. +.
ISum of titratable acid (HPO/-) and NH. + minus He03-.

*Na+ is passively reabsorbed in the thin and thick segments of the ascending limb of the loop of Henle.

b. He0 3 - reabsorption
(1) Most of the H + secreted into the tubular fluid reacts with He03- to form carbonic acid
(H 2 C0 3), which is dehydrated (via carbonic anhydrase) to form CO 2 and water. The
CO 2 diffuses back into the proximal tubular cells, where it is rehydrated to H 2 C0 3,
which dissociates into HC0 3- and H +.
(2) The buffering of secreted H+ by filtered HC0 3- is not a mechanism for H+ excretion.
The CO 2 formed in the lumen from secreted H+ returns to the tubular cell to form
another H +, and no net H + secretion occurs. *
c. He0 3 - secretion by cortical collecting tubules has recently been shown.
d. Nonbicarbonate buffering of H+. The H+ secreted in excess of those required for HC03reabsorption are buffered in the tubular fluid either by nonbicarbonate buffer anions (chiefly,
HPOl-) or by NH3 to form NH4+'
(1) About 20 mEq (mmol) of H+ per day are buffered by HPOl- (and organic ions) and
excreted as titratable acid. The nonbicarbonate buffering of H+ occurs mainly in the
distal and collecting tubules.
(2) About 40 m Eq (mmol) of H + per day are buffered by N H 3 and excreted as N H4 + .
(3) The total amount of H+ excreted daily by an individual with a normal diet equals the
sum of titratable acid and NH4 + excreted, or about 60 mEq (mmol) of H+ per day.
Thus, only a minute concentration of free H+ normally exists in the final urine despite
the 4300 mEq (mmol) of H+ secreted daily!
5. CI- transport
a. General considerations
(1) CI- reabsorption occurs at a rate of 20,000 mEq/day and is inversely related to HC0 3reabsorption; that is, when HC03- reabsorption increases, CI- reabsorption decreases
and vice versa. Therefore, the plasma [CI-] varies inversely with the HC0 3- reabsorption rate.
(2) CI- is the chief anion to accompany Na+ through the renal tubular epithelium. Since
Na+ reabsorption is under control of aldosterone, the plasma [CI-] is secondarily
influenced by aldosterone.
(3) CI- is excreted with NH4 + to eliminate H+ in exchange for Na+ reabsorption.
b. Passive transport. When Na+ is actively reabsorbed across the luminal membrane of the
proximal tubule, CI- is passively reabsorbed along an electrochemical gradient.
(1) Passive proximal diffusion (reabsorption) of CI- is aided by the increased [CI-] within
the luminal fluid of the late proximal tubule.
(2) This increased [CI-] is due to the reabsorption of NaHC0 3 and, therefore, water into
the early proximal tubule.
c. Active transport. CI- is actively reabsorbed in the thick segment of the ascending limb of
the loop of Henle.
(1) CI- reabsorption induces the passive reabsorption of Na+ in the thick segment.
(2) The reabsorption of NaCI in the thick and thin segments together with the water
impermeability of the ascending limb lead to the dilution of the tubular fluid.
6. Water reabsorption. Water movement across membranes is determined by hydrostatic and
osmotic pressure gradients. Water is reabsorbed passively by diffusing along an osmotic gradient, which primarily is established by the reabsorption of Na + and CI-.
a. Proximal tubules
(1) In humans, about 75%-80% of the reabsorption of filtered water occurs in the proximal
tubule.
(2) The proximal reabsorption of water is invariant and involves no change in osmolality.
b. Loop of Henle
(1) Unlike the proximal tubule, the loop of Henle reabsorbs considerably more solute than
water. Only about 5% of the reabsorption of filtered water occurs here.
(2) The tubular fluid entering the descending limb of the loop of Henle always is isosmotic,
regardless of the hydration state.
c. Distal and collecting tubules

'The reabsorption of HC0 3 - across the luminal membrane does not occur in the conventional way (i.e., via an active
HC0 3 - pump); instead, HC0 3 - is transported across the luminal membrane mainly as CO 2 ,

(1) The distal and collecting tubular reabsorption of water occurs only in the presence of
ADH."
(2) The tubular fluid entering the distal tubule always is hyposmotic to plasma, regardless
of the hydration state. In the terminal distal tubule and the collecting ducts, the osmolality
in the tubular fluid changes according to the water permeability of the tubule.

VII. RENAL CONCENTRATION AND DILUTION OF URINE


A. Functional considerations

1. Purpose. The kidney can alter the composition of the urine in response to the body's daily
needs, thereby maintaining the osmolality of body fluids. When it is necessary to conserve
body water, the kidney excretes urine with a high solute concentration. When it is necessary
to rid the body of excess water, the kidney excretes urine with a dilute solute concentration.
2. Role of ADH. The principal regulator of urine composition is the hormone ADH. In the absence
of ADH, the kidney excretes a large volume of dilute urine; when ADH is present in high
concentration, the kidney excretes a small volume of concentrated urine.
3. Components of the concentrating and diluting system. The formation of urine that is
dilute (hyposmotic to plasma) or concentrated (hyperosmotic to plasma) is achieved by the
countercurrent system of the nephron (Figure 4-12.)t This system consists of:
a. Descending limb of the loop of Henle (DLH)
b. Thin and thick segments of the ascending limb of the loop of Henle (ALH)
c. Medullary interstitium
d. Distal convoluted tubule
e. Collecting duct
f. Vasa recta, which are the vascular elements of the juxtamedullary nephrons
4. Mechanisms of dilution and concentration. The kidney forms a dilute urine in the absence
of ADH. In the presence of ADH, the kidney forms a concentrated urine via the functioning of
the countercurrent multipli~rs (i.e., the loop of Henle and collecting duct) and the countercurrent exchangers (i.e., the vasa recta). Regardless of ADH, however, the fluid osmolality in
the loop of Henle, vasa recta, and medullary interstitium always increases progressively from
the corticomedullary junction to the papillary tip (see Figure 4-12D).
a. The fundamental processes involved in the excretion of a dilute or concentrated urine
include:
(1) Variable permeability of the nephron to the passive back-diffusion (reabsorption) of
water along an osmotic gradient and of urea along its concentration gradient
(2) Passive reabsorption of NaCi by the thin segment of the ALH
(3) Active reabsorption of CI- and passive reabsorption of Na+ by the thick segment of
the ALH
b. The formation of a hyperosmotic urine involves the following steps.
(1) The medullary interstitium becomes hyperosmotic by the reabsorption of NaCi and
urea.
(2) The urine entering the medullary collecting ducts equilibrates osmotically with the
hyperosmotic interstitium, resulting in the excretion of a small volume of concentrated
urine in the presence of ADH.
c. The final osmolality of the urine is determined by the permeability of the collecting ducts to
water.
d. The vascular loop (vasa recta) prevents the dissipation of the osmotic (hypertonic) "layering"
in the interstitium (see VII C).
B. Countercurrent multipliers (see Figure 4-12)

1. General considerations
a. The countercurrent multiplier system is analogous to a three-limb model consisting of the
two limbs of the loop of Henle, which are connected by a hairpin turn, and the collecting

"In the presence of a maximal ADH effect, 99% of the water is reabsorbed; in the absence of ADH, 88% of the
water is reabsorbed.
tOnly the juxtamedullary nephrons, with their long loops of Henle, contribute to the medullary hyperosmolality.

PCT

Cortex

OCT
ALH

Outer
medulla
Inner
medulla

(thick)

OLH

ALH

CO

(thin)

To ureter

Urea

285

400

400

200

400

600

600

400

600

H 20 800

800

600

800

1000
NaCI

1200

1000
1200 mOsm/kg

Figure 4-12. The three-limb model of the countercurrent multiplier system of the juxtamedullary nephron. Model A
represents the directions of tubular flow (countercurrent); A = concentrating segment; B = diluting segment; C =
collecting duct; and I = interstitium. Model B represents the major components of the nephron; PCT = proximal
convoluted tubule; OLH = descending limb of the loop of Henle; ALH = ascending limb of the loop of Henle (thick
and thin segments); OCT = distal convoluted tubule; and CD = collecting duct. Model C represents solute and
solvent transfer; the heavy line indicates water impermeability. Model 0 represents the vertical (longitudinal) and
horizontal (transverse) osmotic gradients in the presence of antidiuretic hormone. (After Jamison R, Maffly RH: The
urinary concentrating mechanism. N Engl J Med 295:1059-1067, 1976.)

duct. The fluid flow through the three limbs is countercurrent (i.e., in alternating opposite
directions; see Figure 4-12A).
b. The countercurrent multiplier process has two important consequences (Figure 4-13; see
Figure 4-120).
(1) At equilibrium in the presence of AOH, a maximal vertical gradient of 900 mOsm/kg
is established between the corticomedullary junction (300 mOsm/kg) and the renal
papillae (1200 mOsm/kg). This explains the origin of the term "countercurrent multiplier."
(2) At any given level in the renal medulla, the osmolality is almost the same in all fluids
except that in the ALH. The fluid in the ALH is less concentrated than that in either the
OLH or the interstitium and becomes hyposmotic to plasma (see Figure 4-120).

2. Loop of Henle
a. The DLH is the concentrating segment of the nephron (see Figures 4-12C, 4-120, 4-13).
The following characteristics of the OLH account for this.
(1) The OLH is highly permeable to water. Solute-free water leaves the OLH, causing the
fluid in the OLH to become concentrated to a degree that is consistently higher than
that of the fluid in the ALH.
(2) The OLH has a low permeability to NaCI and urea. The predominant solute is NaCl,
with relatively small amounts of urea existing in the OLH.

PCT

OLH

ALH

OCT

CD

Final
urine

TFosm

Uosm

Posm

Posm

Interstitial fluid (outside nephron)


Tubular fluid in the presence of AOH (antidiuresis)
Tubular fluid in the absence of AOH (water diuresis)

Figure 4-13. TF/P osmolality ratios as a function of length along the nephron. A ratio of 1 indicates isosmolality, a
ratio of more than 1 indicates hyperosmolality, and a ratio of less than 1 indicates hyposmolality. Solute-free water
reabsorption occurs in the early distal tubule, in the descending limb of the loop of Henle, and in the collecting duct.
PCT = proximal convoluted tubule; DLH = descending limb of the loop of Henle; ALH = ascending limb of the
loop of Henle; OCT = distal convoluted tubule; CD = collecting duct; and ADH = antidiuretic hormone. (Adapted
from Koch A: The kidney. In Physiology and Biophysics: Circulation, Respiration and Fluid Balance, 20th edition.
Edited by Ruch TC and Patton HD. Philadelphia, WB Saunders, 1974, p 445.)

(3) The fluid in the OLH nearly attains the osmolality of the adjacent medullary interstitium.
[The interstitial osmolality is maintained by solvent-free solute (NaCI) that is transported
out of the ALH.)
b. The ALH. The thick and thin segments of the ALH constitute the diluting segments of the
nephron. Both segments are impermeable to water and both are permeable to NaCl; in
addition, the thin segment of the ALH is permeable to urea.
(1) As fluid passes up the thin segment of the ALH, NaCi diffuses down its concentration
gradient into the interstitial fluid. This contributes to the increased interstitial osmolality
and renders the tubular fluid hyposmotic to the peritubular interstitium.
(2) The thick segment of the ALH actively transports CI- out of the lumen, with Na+
following the electrical gradient established by CI- transport. The active CI- transport
from the ALH to the interstitium is the most important single effect in the countercurrent system.
(3) The active and passive transport of NaCI from the ALH to the interstitium forms a
horizontal osmotic gradient of up to 200 mOsm/kg between the tubular fluid of the
ALH and the combined fluid of the interstitium and the OLH (Figure 4-14; see Figures
4-12C, 4-120). This leaves behind a smaller volume of hypotonic fluid rich in urea,
which flows into the distal convoluted tubule and the cortical and medullary collecting
ducts.
(4) The fluid in the ALH becomes diluted by the net loss of NaCi in excess of the net gain
of urea by diffusion. This efflux of NaCI causes the tubular fluid within the ALH to
become hyposmotic and that in the interstitial fluid to become hyperosmotic.

3. Simultaneous events in the interstitium


a. Fluid in the interstitium contains hyperosmotic concentrations of NaCI and urea.
(1) As the collecting ducts join in the inner medulla, they become increasingly permeable
to urea (especially in the presence of AOH), allowing urea to flow passively along its
concentration gradient into the interstitium (see figures 4-12C, 4-13).
(2) The increase in medullary osmolality causes water to move out of the adjacent tubules,
the terminal collecting ducts, and the OLH.

Distal tubule

I
~

100300
600
A 1000
1400
Loop of Henle

To ureter

Figure 4-14. Countercurrent system for production of


urine. Numerals denote osmolality; 0 = descending limb
of the loop of Henle; A = ascending limb of the loop of
Henle; and the capillary denotes the vasa recta countercurrent exchange system. (Reprinted from Wilatts SM:
Normal water balance and body fluid compartments. In
Lecture Notes on Fluid and Electrolyte Balance. Boston,
Blackwell Scientific, 1982, p 19.)

b. Water is reabsorbed in the last segment of the distal convoluted tubule and in the collecting
duct in the cortex and outer medulla.
c. In the inner medulla both water and urea are reabsorbed from the collecting duct (see Figure
4-120.
(1) Some urea reenters the ALH but at a slower rate than the efflux of NaCI.
(2) This medullary recycling of urea, in addition to solute trapping by countercurrent
exchange, causes urea to accumulate in large amounts in the medullary interstitium,
where it osmotically abstracts water from the DLH and thereby concentrates NaCi in
the DLH fluid.
4. Distal convoluted tubule (see Figures 4-12, 4-14)
a. General considerations. Before reaching the collecting duct, which is the next major
component of countercurrent multiplication, the tubular fluid must pass through the distal
convoluted tubule, where the role of ADH again is apparent.
b. Role of ADH. Whether in the presence or the absence of ADH, fluid entering the distal
convoluted tubule always is hypotonic to plasma in cortical as well as juxtamedullary
nephrons (see Figure 4-13).
(1) During diuresis (Le., in the absence of ADH), this fluid remains hypotonic to plasma as
it traverses the distal convoluted tubule and collecting duct; in fact, the solute concentration of the final urine may be lower than that of the fluid entering the distal convoluted
tubule (see Figure 4-14). Thus, in the absence of ADH, Na+ reabsorption is not retarded.
(2) During antidiuresis (Le., in the presence of ADH), the hypotonic fluid entering the distal
convoluted tubule becomes isosmotic and the fluid entering the collecting duct is
isosmotic (see Figure 4-13). This is because ADH increases the water permeability of
the cells of the late distal convoluted tubule and collecting duct.
(3) The osmolality of the glomerular filtrate, the cortical interstitial fluid, and the fluid leaving
the distal convoluted tubule (in the presence of ADH) is isosmotic (300 mOsm/kg).
5. Collecting duct. The cortical and upper medullary collecting ducts are relatively impermeable
to water, urea, and NaCI.
a. The relative impermeability to water occurs in the absence of ADH.
b. The relative impermeability to NaCi permits the high interstitial concentration of NaCi to
act as an effective osmotic gradient between the tubular fluid and the interstitium. (Some
Na+ and CI- are reabsorbed from the distal convoluted tubule and collecting duct in the
presence or absence of ADH.)
c. When the kidney forms a concentrated urine, the collecting duct receives an isosmotic fluid
from the distal convoluted tubule, the collecting duct is made permeable to water, and the
urine equilibrates with the hyperosmotic medullary interstitium, resulting in the excretion of
a low volume, hypertonic urine.
d. If the collecting duct is impermeable to water (ADH absent), the dilute tubular fluid entering
the collecting duct from the distal convoluted tubule remains hypotonic and is excreted as
a higher volume, hypotonic urine.

C. Countercurrent exchangers (Figure 4-15; see Figure 4-14). The vasa recta function as countercurrent diffusion exchangers, making the concentrating mechanisms far more efficient.

_325

AVA

Figure 4-15. Vasa recta as countercurrent exchangers.


AVR = ascending limb of the vasa recta; DVR = de-

1200
---. H 2 0
- - . Solutes

scending limb of the vasa recta; and I = interstitium.


(Adapted from Pitts RF: Physiology of the Kidney and
Body Fluids, 3rd ed. Chicago, Year Book, 1974, p 131.)

1. Functional considerations of the vasa recta (see Figure 4- 1 5)


a. The vasa recta are derived from the efferent arterioles of the juxtamedullary glomeruli and
function to maintain the hyperosmolality of the medullary interstitium. The vasa recta are
in juxtaposition with the loops of Henle.
b. These vessels are permeable to solute and water and reach osmotic equilibrium with the
medullary interstitium.
c. The vasa recta return the NaCi and water reabsorbed in the loops of Henle and collecting
ducts to the systemic circulation.
d. The concentrations of Na+ and urea in the medullary interstitium are kept high by the slow
blood flow in the vasa recta.
2. Countercurrent exchange effects of the vasa recta (see Figure 4-15)
a. Na+, CI-, and urea are passively reabsorbed from the ALH, diffuse across the interstitial
fluid into the descending limb of the vasa recta, and are returned to the interstitium by the
ascending limb of the vasa recta.
(1) These solutes recirculate in the vasa recta capillary loops and increase the medullary
osmolality.
(2) Na+, CI-, and urea also recirculate in the loop of Henle.
b. Water diffuses from the descending limb of the vasa recta across the interstitial fluid and
into the ascending limb of the vasa recta. * As a result, the cortex receives blood that is only
slightly hypertonic to plasma (see Figure 4-15).
D. Role of urea. The main function of urea in the countercurrent system is to exert an osmotic effect
on the DLH, promoting the abstraction of water and raising the intraluminal concentration of NaCI.
1. In the medulla, ADH enhances both water and urea permeabilities of the collecting duct.
a. Urea diffuses into the thin segment of the ALH via secretion but is more concentrated in
the fluid entering the distal convoluted tubule (concentrated in a smaller volume) than in
the filtrate entering the proximal convoluted tubule.
b. In the presence of ADH, additional water is reabsorbed from the collecting duct, which
further increases the urea concentration in the tubular fluid (urine).
c. The thin segment of the ALH is less permeable to urea than to NaCl, and the thick segment
of the ALH is impermeable to urea.
2. Urea recirculates in the loops of Henle and the vasa recta. The vasa recta are permeable to
urea so that urea diffusing out of the papillary collecting duct is trapped in the medullary
i nterstiti um.
a. Of the 1200 mOsm/kg of solute present in the renal papillary loop during antidiuresis, half
is composed of NaCI and half of urea.

'Water is continually removed by the ascending limb of the vasa recta.

b. During water diuresis, about 10% of the total medullary solute concentration is urea.
c. The maximal urine osmolality (Uos m) cannot exceed that in the interstitium, and the ability
to conserve water by excreting a concentrated urine is reduced when the papillary concentration is reduced.
E. Measurement of renal water excretion and conservation (Figure 4- 16)

1. Renal water excretion. The quantitative measure of the kidney's ability to excrete water is
termed free-water clearance.
a. General considerations
(1) Free-water clearance is not a true clearance, because no osmotically free water exists

in plasma.
(2) Free-water clearance denotes the volume of pure (Le., solute-free) water that must be
removed from, or added to, the flow of urine (in ml/min) to make it isosmotic with
plasma.

b. Measurement
(1) Free-water clearance (C H20) is calculated using the equation

V=
C H20 =

Cos m

V-

C H20

Cos m

where
Uos m

Cos m =
Pos m

Substituing for

Cos m

the equation becomes


.

CH

Uos m

and by factoring out

V,

= V - ---

Pos m

the final equation is


C H20

V.

1 - Uos
- -)

(8)

Pos m

where V
urine volume per unit time (in ml/min); Uos m = urine osmolality; Pos m =
plasma osmolality; and Cos m = osmolal clearance, which is the volume of plasma (in
ml) completely cleared of osmotically active solutes that appear in the urine each
minute.
(2) From equation (8), during water diuresis (Le., in a hydrated state and in the absence
of ADH), the Uos m is less than the P osm (hyposmotic urine), and the Uos m / P osm ratio is
less than 1. Therefore, C H20 is positive, indicating that water is being eliminated by the

Solvent (water)

~Solute

"E
Q)
0

c
0
0

Water reabsorption

Q)

>

Qi

II:

Urine

Plasma

Water excretion

Urine

Figure 4-16. Renal free-water reabsorption (A) and free-water clearance (C). In column A, the stippled area represents
the amount of free water that was removed from the urine by tubular reabsorption to raise the Uosm to the observed
hypertonic value. In column C, the cross-hatched area denotes the amount of free water that was added to the urine
by tubular excretion to lower the Uos m to the observed hypotonic value. Plasma osmolality (B) serves as the reference
point for describing the osmolality of urine.

excretion of a large .volume of dilute urine. The maximum C H20 in humans is 15-20
Llday (10-15 ml/mml.
(3) From equation (8), during antidiuresis (i.e., in a dehydrated state and in the presence
of ADH), the Uos m is greater than the Posm (hyperosmotic urine), and the Uos ml Posm ratio
is greater than 1. Therefore, CH~O is negative, indicating that water is being conserved
by the excretion of a small volume of concentrated urine. (To avoid the use of the
term "negative free-water clearance," the symbol, F H20, is used to denote free-water

reabsorption.)

2. Renal water conservation. A quantitative measure of the ability of the kidney to reabsorb
water is termed free-water reabsorption.
a. General considerations. Free-water reabsorption denotes the volume of free water reabsorbed per unit time, or, the amount of free water that must be removed from the urine by
tubular reabsorption to make it hyperosmotic with plasma.

b. Measurement
(1) Free-water reabsorption (TCH2o )* is calculated using the equation
TC H20

Cosm -

where

Cosm =

Uos m V
---

Posm

Substituting for Cos m, the equation becomes

TcH

0
2

and by factoring out

V,

- Uos m V
Posm

the final equation is

(Uos m _ 1) V
(9)
Posm
(2) From equation (9), during antidiuresis (i.e., in a dehydrated state and in the presence
of ADH), the Uos m exceeds Posm, and the Uosm/Posm ratio is greater than 1. Therefore,
TC H 0 is positive, indicating that water is being conserved by the elimination of a small
vol5me of concentrated urine.
TC

H20

3. Antidiuresis and diuresis (Table 4-7) can be examined in terms of the volume of urine excreted
per unit time (V). Recall that V is the algebraic sum of osmolar clearance (Cosm) and either
free-water clearance (C H 0) or free-water reabsorption (TC H20l.
a. Antidiuresis (i.e., a d~crease in V) can result from:
(1) Water deprivation, which leads to an increase in plasma osmolality
(2) Reduced circulating blood volume
b. Diuresis (i.e., an increase in V) can result from:
(1) Reduced osmotic reabsorption of water, leading to increased solute-free water clearance
and water diuresis
(2) Reduced solute reabsorption (primarily Na + with associated anions), leading to increased osmolar clearance and osmotic diuresis

Table 4-7. Renal Water Excretion and Reabsorption


Renal State
Antidiuresis
Water diuresis
Osmotic diuresis

COlm

Uosm

(ml/min)

(ml/min)

(mOsm/kg)

~
~

Note-V = urine excretion; Cosm = osmolar clearance; CH 0= free-water clearance; TC H 0


2
2
and U osm = urine osmolality.

free-water reabsorption;

'The superscript "c" in TC H20 signifies that the net reabsorption of water occurs in the collecting tubule.

VIII. ADH
A. Synthesis and chemical characteristics
1. ADH, also known as vasopressin, * is a hypothalamic hormone synthesized mainly by the cell
bodies of the neurosecretory neurons of the supraopticohypophysial tract that terminate
in the pars nervosa of the posterior lobe of the pituitary gland.
a. Neurosecretory neurons are peptidergic neurons that conduct action potentials like all
neurons and, unlike ordinary neurons, synthesize and secrete peptide hormones. The hypothalamic nuclei that represent the main source of ADH are the supraoptic nuclei, which are
called magnocellular neurosecretory neurons.
b. ADH is stored (but not synthesized) in the pars nervosa. In the absence of a pars nervosa,
the newly synthesized hormone still can be released into the circulation from the ventral
diencephalon (hypothalamus).
2. ADH is classified as a polypeptide. Specifically, it is an octapeptide or a nonapeptide with
a molecular weight of approximately 1000.
a. If the two cysteine residues are considered as a single cystine residue, ADH is an octapeptide.
b. ADH is a nonapeptide if the two cysteine residues are numbered individually.
3. The neurophysins are the physiologic carrier proteins for the intraneuronal transport of ADH
and are released into the circulation with the neurosecretory products (ADH and oxytocin)
without being bound to the hormone.
4. The biologic half-life of ADH is 16-20 minutes.
8. Control of ADH secretion: stimuli and inhibitors
1. Osmotic stimuli
a. Hyperosmolality. Under usual conditions, a 1%-2% increase in plasma osmolality is the
prime determinant of ADH secretion, and the most common physiologic factor altering the
osmolality of the blood is water depletion or water excess. (In clinical medicine, volume
deficits are much more prevalent than volume excesses.)
(1) The stimulation of the osmoreceptors located in the anterior hypothalamus causes the
reflex secretion of ADH.
(2) However, not all solutes (e.g., urea) stimulate the osmoreceptors, despite increasing
plasma osmolality. Only those solutes to which cells are relatively impermeable make
up the effective osmotic pressure, in response to which ADH is secreted.
(a) Na+ and mannitol, which cross the blood-brain barrier relatively slowly, are potent
stimulators of ADH release. Since the plasma [Na+] accounts for 95% of the
effective osmotic pressure, the osmoreceptors normally function as plasma Na+
receptors.
(b) Hyperglycemia is a less potent stimulus for ADH production and secretion than
hypernatremia for the same level of osmolality.
b. Volume disturbances. The control of ADH secretion by osmolality can be overridden by
volume disturbances. For example, marked hyponatremia will be tolerated in order to
maintain circulating blood volume.
2. Osmotic inhibitors. Expansion of the intracellular volume of the osmoreceptors secondary to
hyposmolality of the ECF (water ingestion) inhibits ADH secretion.
3. Nonosmotic stimuli
a. Hypovolemia (i.e., decreased effective circulating blood volume) is a more potent stimulus
to ADH release than is hyperosmolality. A 10%-25% decrease in blood volume will evoke
ADH release. A 10% decrease in blood volume is sufficient to cause the release of enough
ADH to participate in the immediate regulation of blood pressure. Contraction of blood
volume without an alteration in the tonicity of body fluids may cause ADH release.

"The term vasopressin denotes an excitatory action on the blood vessels, causing vasoconstriction of the arterioles
and an increase in systemic blood pressure. This effect is observed only when relatively large quantities of vasopressin
are released from the posterior lobe (e.g., during hemorrhage) or when pharmacologic amounts are injected. Therefore,
the vasopressor effect usually is not considered to be a physiologic effect. The biologically active form of ADH in
humans is arginine vasopressin.

(1) A lower osmotic threshold is required to cause ADH secretion in a volume-depleted

state.
(2) A decrease in the stretch (tension) of the volume receptors (low-pressure baroreceptors)
located in the left atrium, vena cavae, great pulmonary veins, carotid sinus, and aortic
arch causes an increase in ADH secretion. Increased firing rates from these baroreceptors elicited by increased blood pressure exert an inhibitory effect on ADH secretion.
(3) Any reduction in intrathoracic blood volume (e.g., due to blood loss, quiet standing,
upright body position, and positive-pressure breathing) leads to ADH secretion. Upon
standing, the left atrial pressure falls markedly, leading to an increase in ADH release
and antidiuresis.
b. Other nonosmotic stimuli for ADH release include:
(1) Pain
(2) Certain drugs (e.g., nicotine, morphine, barbiturates, acetylcholine, chlorpropamide,
.a-adrenergic agonists)
(3) Various pulmonary or CNS disorders (e.g., pneumonia, tuberculosis, stroke, meningitis,
subdural hematoma)
(4) Angiotensin II
4. Nonosmotic inhibitors
a. Increased arterial pressure secondary to vascular or ECF volume expansion inhibits ADH
release.
(1) Thus, ADH release is inhibited by increased tension in the left atrial wall, great veins,
or great pulmonary veins secondary to increased intrathoracic blood volume due to
hypervolemia, a reclining position, negative-pressure breathing, and water immersion
up to the neck.
(2) In the recumbent position, the increase in central blood volume leads to an increase in
left atrial pressure and inhibition of ADH release. During sleep, the production of a
concentrated urine is by and large due to a reduction of blood pressure, which offsets
the effect of reduced ADH secretion due to a change in body position.
b. Other nonosmotic inhibitors of ADH release include:
(1) Certain drugs (e.g., anticholinergic agents, ethanol, phenytoin, lithium, caffeine)
(2) CO 2 inhalation

C. Role in regulation of renal water excretion


1. Renal effects. The major site of action of ADH is the luminal membrane of the cortical and
medullary collecting tubules, where ADH increases the permeability to water. ADH, through
its effect on increased water reabsorption, leads to the production of a urine that has a decreased
volume and an increased osmolality. ADH also decreases renal medullary blood flow.
2. Extrarenal effects. In addition, ADH stimulates the release of adrenocorticotropic hormone
(ACTH) from the anterior lobe of the pituitary gland. ACTH plays a relatively small role in
controlling aldosterone secretion (see IX D 1).

IX. ALDOSTERONE has the primary function of promoting Na+ retention. In this way, aldosterone
plays a major role in regulating water and electrolyte balance. Together with ADH, aldosterone
influences urine composition and volume.
A. Synthesis, secretion, and inactivation

1. Synthesis
a. Aldosterone is a C-21 (21 carbon atoms) corticosteroid that is synthesized in the outermost
area of the adrenal cortex, the zona glomerulosa.
b. Aldosterone represents less than 0.5% of the corticosteroids, and as all of the corticosteroids,
it is stored in very small quantities. However, aldosterone is the major mineralocorticoid
in humans.
c. The circulatory half-life of aldosterone is about 30 minutes in humans during normal activity.
d. Cholesterol (esterified) is the precursor for steroidogenesis and is stored in the cytoplasmic
lipid droplets in the adrenocortical cells. (Free plasma cholesterol appears to be the preferred
source of cholesterol for corticosteroid synthesis.)

2. Secretion. Most of the secreted aldosterone is bound to albumin, * with a lesser amount bound
to corticosteroid-binding globulin (CBG; transcortin). CBG preferentially binds cortisol,
which is a glucocorticoid.
3. Inactivation. Aldosterone is metabolized mainly in the liver, where greater than 90% of this
corticosteroid is inactivated during a single passage.
a. Most aldosterone inactivation is by saturation (reduction) of the double bond in the A-ring.
b. The major metabolite is tetrahydroaldosterone, most of which is conjugated with glucuronic acid at the carbon-3 position of the A-ring. The resultant glucuronides are more
polar and, therefore, more water-soluble, making them readily excreted by the kidney.
B. Physiologic effects

1. Conservation of Na +
a. Aldosterone stimulates Na+ reabsorption in the connecting segment of the distal tubule and
in the cortical and medullary collecting tubules.
b. Of the amount of filtered Na+ (25,000 mEq/day), only 1%-2% (250-500 mEq or 600012,000 mg/day) are actively reabsorbed via an aldosterone-dependent mechanism in the
distal nephron. t
c. Aldosterone also promotes Na+ reabsorption in the epithelial cells of the sweat glands,
salivary glands, and the gastrointestinal mucosa.
d. By restricting the renal excretion of Na +, which is the main determinant of plasma osmolality,
aldosterone regulates the ECF volume. Therefore, aldosterone regulates the total body
Na+ content, while ADH regulates the plasma Na+ concentration.
2. Secretion and excretion of K+
a. Aldosterone promotes K+ secretion as a secondary effect of its action on Na+ reabsorption.
(1) In the distal tubule, the linked Na+ reabsorption-K+ secretion is referred to as the distal
Na+ -K+ exchange process.
(2) Although K+ appears to be secreted in exchange for Na+, the distal secretion of K+ is
only indirectly related to Na+ reabsorption. Distal Na+ reabsorption is linked to the
secretion of both K + and H +. t
(3) Aldosterone increases the [K+] in sweat glands and saliva.
b. Stimulation of K+ excretion greatly depends on dietary Na +, as indicated by a lack of K+
excretion after aldosterone administration in animals with Na+ -deficient diets.
c. More than 75% of the K+ excreted in the urine is attributed to distal K+ secretion.
(1) Excess aldosterone secretion causes a decline in the urinary Na+ /K+ concentration
ratio (from a normal value of about 2) because it decreases Na+ excretion and increases
K+ excretion.
(2) Elevated aldosterone release increases the plasma Na + / K+ concentration ratio (from a
normal value of 30) due to the increased excretion of K+. Aldosterone can cause an
isotonic expansion of the ECF volume with no change in the plasma [Na+] or a hypertonic expansion of the ECF volume with a rise in the plasma [Na+]. In any case, the
total amount of body Na+ is increased.
3. Water excretion and ECF volume regulation
a. The effect of aldosterone on water excretion is not important physiologically, as aldosterone
treatment fails to correct the impaired water excretion by patients with hypofunctional
adrenal glands (due to Addison's disease or adrenal insufficiency) and by patients who have
undergone adrenalectomy.
b. Aldosterone has no direct effect on GFR, RPF, or renin production; however, by stimulating
Na+ reabsorption aldosterone causes water retention, and the resultant expansion of the
ECF volume then leads to an increase in GFR and RPF and a decrease in renin production.
c. A high circulating aldosterone level is a common finding in edema and is due primarily to
the increased aldosterone secretion induced by the depletion of the effective circulating
blood volume.
(1) It is unlikely that the plasma [Na+] is a major regulator of aldosterone secretion because
the plasma [Na+] during Na+ depletion is normal in humans.

Protein-bound hormones are biologically inactive.


IFiltered load refers to the amount of Na+ in the glomerular filtrate and not in the distal nephron.
INa+ transport in the proximal tubule is not associated with K+ or H+ exchange processes.

(a) Hyponatremia often is a consequence of ADH secretion and is accompanied by


an increase in the ECF volume, which tends to suppress rather than stimulate
aldosterone secretion. When ADH and water are given to Na + -depleted individuals,
aldosterone secretion falls despite a fall in plasma [Na+].
(b) By and large, hyponatremia in the clinical setting is due to excess body Na+ caused
by a decreased effective blood volume. The low blood volume leads to an increase
in ADH secretion and aldosterone secretion which, in turn, lead to edema. Over
90% of the cases of hyponatremia should be treated by restriction of NaCI
and water.
(2) Therefore, it is the volume of the ECF rather than the plasma [Na+] that influences
aldosterone secretion in most circumstances.

C. Aldosterone and acid-base balance. Aldosterone affects acid-base balance through its control
of K+ secretion. As stated above, aldosterone promotes increased distal tubular secretion and,
therefore, excretion of K +. Aldosterone also promotes the excretion of H + and N H4 + .
1. Hyperaldosteronemia (Le., increased aldosterone secretion) is one cause of K+ depletion, a
condition termed hypokalemia.
a. Hypokalemia is characterized by an increase in intracellular [H+], which favors distal tubular
secretion of H+ over K+ and results in a state of metabolic alkalosis.
b. Conversely, if metabolic alkalosis is the primary event, there is a decrease in intracellular
[H+]. This results in an increase in distal tubular intracellular [K+], which favors increased
urinary loss of K+ over H+ and results in a state of hypokalemic metabolic alkalosis (as
occurs in hyperaldosteronemia).
2. Hypoaldosteronemia (Le., decreased aldosterone secretion) is one cause of excess K+, a
condition termed hyperkalemia.
a. Hyperkalemia is characterized by an increase in intracellular [K+], which favors distal tubular
secretion of K+ over H+ and results in a state of metabolic acidosis.
b. On the other hand, when metabolic acidosis is the primary event there is an increase in
intracellular [H+]. This results in an increase in distal tubular intracellular [H+], which favors
increased secretion of H+ over K+ and results in a state of hyperkalemic metabolic
acidosis (as occurs in hypoaldosteronemia).
3. Hypokalemia causes the following conditions.
a. Impaired renal concentrating ability leads to the formation of hyposmotic urine and
polyuria that is resistant to ADH.
b. Reduced carbohydrate tolerance leads to a decline in insulin secretion. * As a rule, fasting
hyperglycemia is not present with reduced insulin secretion.
D. Control of aldosterone secretion. At least three well-defined mechanisms control aldosterone
secretion: ACTH, plasma [K+], and the renin-angiotensin system.
1. Extrarenal control mechanisms. The following mechanisms cause the release of aldosterone
by direct action on the adrenal cortex.
a. Hypothalamic-hypophysial-adrenocortical axis. Under normal conditions, ACTH is not
a major factor in the control of aldosterone synthesis or secretion. However, the pituitary
gland plays an important role in the maintenance of the growth and biosynthetic capacity
of the zona glomerulosa.
(1) ACTH, also known as corticotropin, is a 39-amino-acid polypeptide that supports
steroidogenesis in the zona glomerulosa. ACTH enhances aldosterone production by
stimulating the early biosynthetic pathway (Le., the 20,22-desmolase enzyme complex
that catalyzes the conversion of cholesterol to pregnenolone). ACTH also plays a minor
role in mediating the diurnal rhythmic secretion of all of the corticosteroids.
(2) Corticotropin releasing hormone (CRH) is a hypothalamic hypophysiotropic polypeptide made up of 41 amino acid residues. CRH is secreted into the hypophysial portal
system and causes release of ACTH from the pituitary gland. Since CRH secretion is
regulated by higher brain centers (e.g., the limbic system), these centers playa role in
Na+ balance.

'The reduction in carbohydrate tolerance due to hypokalemia occurs in only about half of patients with elevated
plasma aldosterone.

(a) Hypophysectomized patients and those with pituitary insufficiency exhibit normal
aldosterone secretion on a moderate salt intake; however, these individuals demonstrate a suboptimal aldosterone response to Na+ restriction.
(b) Normal individuals injected chronically with ACTH show an acute rise in aldosterone secretion followed by a return to control level or below in 3-4 days, despite
ACTH administration over a period of 7-8 days.
(c) When aldosterone is administered for several days to normal individuals, the kidney
"escapes" from the Na+-retaining effect but not from the K+-excreting effect. The
escape phenomenon prevents the appearance of edema in individuals treated with
aldosterone for prolonged periods and in patients with primary aldosteronism.
b. Hyperkalemia. A 10% increase in plasma [K+] can stimulate the synthesis and release of
aldosterone by a direct action on the zona glomerulosa. In the anephric human, K+ appears
to be the major regulator of aldosterone even though aldosterone levels are low.
(1) This release probably occurs by the depolarization of the glomerulosa cell membrane
by the elevated plasma [K+].
(2) Stimulation of aldosterone secretion by K+ loading will be limited by the simultaneous
reduction in renin release.
(3) K+ stimulates an early step in the biosynthetic pathway for aldosterone synthesis.
(4) K+ loading increases the width of the zona glomerulosa layer in experimental animals;
prolonged aldosterone administration results in atrophy of this layer due to depressed
renin secretion.
c. Hyponatremia. A 10% decrease in plasma [Na+] also appears to stimulate the synthesis
and release of aldosterone directly at the level of the zona glomerulosa. However, this effect
usually is overridden by changes in the effective circulating volume. Thus, aldosterone
secretion is increased in the hyponatremic patient who is volume-depleted but is reduced
in the hyponatremic patient who is volume-repleted.
2. Intrarenal control mechanism. Aldosterone secretion also is regulated by the renin-angiotensin
system, the major component of which is the juxtaglomerular apparatus (JGA). The reninangiotensin-aldosterone system is regulated by the sympathetic nervous system.
a. Anatomy of the ,GA (Figure 4-17). The JGA is a combination of specialized tubular and
vascular cells located at the vascular pole where the afferent and efferent arterioles enter
and leave the glomerulus. The JGA is composed of three cell types.
(1) ,uxtaglomerular (,G) cells are specialized myoepithelial (modified vascular smooth
muscle) cells located in the media of the afferent arteriole, which synthesize, store, and
release a proteolytic enzyme called renin. Renin is stored in the granules of the jG cells.
(a) The jG cells are baroreceptors (tension receptors) and respond to changes in the
transmural pressure gradient between the afferent arteriole and the interstitium.
They are innervated by sympathetic nerve fibers.
(b) These vascular "volume" receptors monitor renal perfusion pressure and are stimulated by hypovolemia, or decreased renal perfusion pressure.

Proximal
tubule-------,~-

Renal nerves

Juxtaglomerular cells.".---+---t:;,.~

Figure 4-17. The anatomic components of the juxtaglomerular apparatus. (Reprinted from Yates FE, et al:
The adrenal cortex. In Medical Physiology, 14th edition. Edited by Mountcastle VB. St. Louis, CV Mosby, 1980,
p 1590.)

(2) Macula densa cells are specialized renal tubular epithelial cells located at the transition
between the thick segment of the ALH and the distal convoluted tubule (see Figure
4-17).
(a) These cells are in direct contact with the mesangial cells, in close contact with the
jG cells, and contiguous with both the afferent and efferent arterioles as the tubule
passes between the arterioles supplying its glomerulus of origin.
(b) The macula densa cells are characterized by prominent nuclei in those cells on the
side of the tubule that is in contact with the mesangial and vascular elements of the
jGA.
(c) The macula densa cells function as chemoreceptors and are stimulated by a decreased Na+ (NaCI) load. This inverse relationship between Na+ load and renin
release provides a reasonable explanation for the clinical problems involving a
decreased filtered load of Na+ and CI- in association with increased renin release.
(d) Macula densa cells are not innervated.
(3) Mesangial cells also are referred to as the polkissen (asymmetrical cap) and are the
interstitial cells of the jGA. Mesangial cells are in contact with both the jG cells and the
macula densa cells. A decreased intraluminal Na+ load, CI- load, or both in the region
of the macula densa stimulates the jG cells.
b. Role of the sympathetic nervous system. The sympathetic nervous system plays an
important role in the control of renin release via the renal nerves.
(1) The jG cells of the afferent arterioles are innervated directly by the sympathetic postganglionic fibers (unmyelinated). In the absence of renal nerves, the renal response to Na+
depletion is attenuated.
(2) Circulating catecholamines (i.e., epinephrine and norepinephrine) and stimulation of the
renal nerves produce vasoconstriction of the afferent arterioles, which causes renin
release by a decrease in perfusion pressure.
(a) This renin response caused by catecholamines and renal nerve stimulation is mediated via the J3-adrenergic receptor and can be elicited by the synthetic sympathomimetic amine, isoproterenol, which is a J3-agonist.
(b) Renal denervation and J3-adrenergic receptor blockade by propranolol inhibit the
release of renin.
(3) Renal innervation is not a requisite for renin release because the denervated kidney can
adapt to a variable salt intake.
(4) In humans, exercise for assuming an upright posture increases renal sympathetic activity,
which produces renal arteriolar vasoconstriction and an increase in renin release. Thus,
the sympathetic nervous system, by modulating the secretion of renin, has an indirect

effect on aldosterone secretion.


c. Role of renin: stimuli for release
(1) Renin has a circulatory half-life of 40-120 minutes in humans. The common denominator for renin release by the intrarenal mechanism is a decrease in the effective circulating
blood volume, which is induced by:
(a) Acute hypovolemia associated with hemorrhage, diuretic administration, or salt
depletion
(b) Acute hypotension associated with ganglionic blockade or a change in posture
(postural hypotension)
(c) Chronic disorders associated with edema (e.g., cirrhosis with ascites, congestive
heart failure, nephrotic syndrome)
(2) Renin release is increased by K+ depletion, epinephrine, norepinephrine, isoproterenol,
and standing.

d. Role of renin: inhibition of renin secretion


(1) Renin release is inhibited by angiotensin II, angiotensin III, ADH, hypernatremia, hyperkalemia, and atrial natriuretic factor.
(2) K+ loading leads to the inhibition of renin release and to the direct stimulation of the
glomerulosa cells to secrete aldosterone. In contrast to its effect on normal indiViduals,
aldosterone administration to patients with heart failure, cirrhosis with ascites, or nephrosis causes Na+ retention without K+ excretion, because of greater proximal reabsorption
of Na + with less Na + available for the distal exchange with K+.

e. Angiotensin synthesis
(1) Renin is secreted into the bloodstream, where it combines with the renin substrate,
angiotensinogen, which is an arglobulin synthesized in the liver.

(a) Renin is not saturated with its substrate in normal plasma; the same amount of
renin generates more angiotensin I if the substrate concentration is increased above
normal.
(b) Oral contraceptives increase plasma angiotensinogen concentration and decrease
plasma renin concentration.
(2) The only physiologic effect of renin is to convert angiotensinogen to the biologically
inactive decapeptide, angiotensin I.
(3) Angiotensin I is converted primarily in the lung by pulmonary endothelial cells to the
physiologically active octapeptide, angiotensin II (Figure 4-18).
(a) The enzyme that forms angiotensin II is a peptidase (dipeptidyl carboxypeptidase)
called angiotensin-converting enzyme. It is found chiefly in pulmonary tissue and
to a lesser degree in renal tissue and blood plasma.
(b) The converting enzyme is identical to kininase II, which converts the nonapeptide
vasodilator, bradykinin (kallidin-9), to inactive peptides, thereby diminishing the
circulating levels of a vasodepressor substance and enhancing the vasoconstrictive
action of angiotensin II.
(4) Angiotensin II, with a circulatory half-life of 1-3 minutes, has several important physiologic actions.
(a) It functions as the tropic hormone for the zona glomerulosa and stimulates the
secretion (and synthesis) of aldosterone. Angiotensin II is the aldosteronestimulating hormone.
(b) It is a potent local vasoconstrictor (vasopressor) of the renal arterioles at low
plasma concentrations; at higher concentrations, angiotensin II exerts a general
vasopressor effect on the smooth muscle cells of arterioles throughout the cardiovascular system, leading to an elevation of systemic mean arterial blood pressure.
(c) Angiotensin II stimulates the secretion of ADH and ACTH.
(d) Angiotensin II stimulates thirst, which leads to increased fluid consumption. It also
stimulates the release of epinephrine and norepinephrine from the adrenal medulla.
(5) Angiotensin II is inactivated by two angiotensinases (peptidases), and it can be converted
to angiotensin III, which is a heptapeptide and a very potent stimulator of aldosterone
secretion. Angiotensin III is not an effective vasoconstrictor in contrast to angiotensin II.
(6) Renin, converting enzyme, angiotensinogen, and angiotensin II have been found in brain
tissue.
f. Plasma renin activity and plasma renin concentration. Plasma renin activity is defined
as the rate of angiotensin I formation when plasma renin acts on endogenous substrate.
Plasma renin concentration is measured when exogenous substrate is added to plasma
to saturate the enzyme and increase the velocity of angiotensin I formation to a maximal
rate.
(1) Oral contraceptive administration is known to increase plasma renin activity and aldosterone secretion via a marked increase in renin substrate concentration, whether the
woman is normotensive or hypertensive prior to the therapy.

RENINANGIOTENSIN
SYSTEM

KININ
SYSTEM

Angiotensinogen

t . . .;...----

Prorenin
Renin ...,,1--------'1 ....,,;...----Kallikrein -

Angiotensin I

Kininogen

Kinins

!. .

enzyme (kininase II)

tI ~_

Angiotensinases

1 - - - - - - - - - Angiotensin-converting - - - - - _
..

Angiotensin II

Angiotensin III

-----------'-+--....

Inactive
fragments

Inactive
fragments

Figure 4-18. The renin-angiotensin system and its relationship to the kinin system.

(2) As a result of the increased angiotensin II formation, plasma renin concentration is


suppressed. Thus, the increase in plasma renin activity in this situation occurs without
an increase in renin concentration.
(3) Oral contraceptives are a cause of hypertension in women through this mechanism.
E. Aldosteronism refers to a condition of excessive aldosterone secretion.
1. Primary hyperaldosteronism (Conn's syndrome)
a. Etiology. Primary hyperaldosteronism results from a tumor of the zona glomerulosa.
b. Characteristics of primary hyperaldosteronism include:
(1) Elevated plasma (and urinary) aldosterone
(2) Hypertension due to Na+ and water retention
(3) Hypokalemic alkalosis with a K+ excretion rate of greater than 40 mEq/day*
(4) Decreased levels of angiotensin and renin
(5) Decreased hematocrit due to the expansion of the plasma volume
(6) Polyuria and dilute urine due to secondary nephrogenic diabetes insipidus
(7) Absence of peripheral edema
(8) Decreased plasma colloidal osmotic (oncotic) pressure due to ECF expansion
2. Secondary hyperaldosteronism
a. Etiology. Secondary hyperaldosteronism is caused by the following extra-adrenal factors:
(1) Diuretic therapy, which is the most common cause
(2) Extravascular loss of Na+ and water, which is associated with edema (due to such
underlying factors as nephrosis, cirrhosis, and congestive heart failure), an increase in
the total ECF volume, and a loss of effective blood volume
(3) Hyperreninism caused by a tumor of the JG cells
(4) Renovascular disease (e.g., renal artery stenosis)
b. Characteristics. Secondary hyperaldosteronism is characterized by edema and Na+ retention. Urinary K+ excretion is not increased because there is a reduced flow of fluid into and
through the distal segments of the nephron. This low fluid flow reduces K+ secretion and
offsets the stimulating effect of aldosterone. Also, with decreased Na+ and water delivery
to the distal tubule, the quantity of K+ (and H+) secreted in the urine is limited. Additional
characteristics of secondary hyperaldosteronism include:
(1) Increased plasma (and urinary) aldosterone
(2) Hypertension with edema (due to Na+ retention and water accumulation in the interstitial fluid compartment) and a decrease in plasma volume
(3) Hypokalemic alkalosis
(4) Increased angiotensin and plasma renin activityt
(5) Peripheral edema
3. Chronic licorice ingestion in excessive amounts can mimic primary aldosteronism, because
licorice contains the salt-retaining substance, glycyrrhizinic acid. Patients with this condition
present with:
a. Hypertension and hypokalemic alkalosis
b. Suppressed plasma renin levels
c. Reduced aldosterone secretion due to chronic volume expansion
F. Aldosterone antagonism: spironolactone
1. Renal effects. Spironolactone is a steroidal aldosterone antagonist, which competitively blocks
the Na+ -retaining and K+ -excreting effects that aldosterone exerts on the distal renal tubule.
As a result, spironolactone leads to an increase in urinary Na+ excretion and a decrease in
K+ excretion. This antagonist is efficacious only in the presence of aldosterone or another
mineralocorticoid; spironolactone is without effect in adrenalectomized individuals.
2. Blood pressure effect. Because the drug enhances Na+ diuresis, spironolactone is effective
in potentiating the action of many antihypertensive drugs whose dosage should be reduced in
its presence.

'Hypokalemia is not always concomitant with hypermineralocorticoidism (e.g., in llj3-hydroxylase deficiency).


tThe elevated renin level is the characteristic that differentiates secondary aldosteronism from the primary form.

3. Clinical application. Spironolactone is useful in the differential diagnosis of primary and


secondary hyperaldosteronism.
a. If both plasma [K+] and blood pressure are returned to normal with spironolactone, primary
hyperaldosteronism is suspected.
b. If spironolactone causes the plasma [K+] to return to normal without the antihypertensive
effect, secondary hyperaldosteronism is suspected.

x.

ATRIAL NATRIURETIC FACTOR (ANF) refers to a group of polypeptides (also abbreviated


ANP) produced by the atrial muscle cells. ANF exerts a hormonal influence on the kidney, which
results in changes in intrarenal hemodynamics through its effects on fluid volume, electrolyte (Na+)
balance, and blood pressure homeostasis.
A. Synthesis
1. In mammals, ANF is synthesized, stored, and released from atrial cardiocytes. The secretory
activity of these cells is evidenced by the presence of membrane-bound storage granules with
electron-dense cores.
2. The peptides that comprise the ANF group have molecular weights ranging from about 2500
to 13,000 daltons and lengths ranging from 21 to 73 amino acid residues. All are derived
from a common 126-amino-acid precursor called pro-ANP (atriopeptigen), which is the
predominant form of ANF in the atrium.
3. In humans, ANF is synthesized in a prepro (Le., pre-atriopeptigen) form containing 151 amino
acid residues.
4. The predominant circulating form of ANF is the 28-amino-acid peptide.
B. Secretion. Stimuli for ANF release include atrial distension (hypervolemia), epinephrine, arginine
vasopressin (ADH), and acetylcholine.

C. Physiologic effects of ANF


1. Renal and adrenal responses
a. An increased GFR is associated with constriction of the efferent arteriole, increasing the
glomerular hydrostatic pressure. It is important to note that ANF induces relaxation (dilation)
of precontracted renal arteries.
b. Dilation of the afferent arteriole increases the hydraulic pressure in the glomerular capillary.
c. Natriuresis (i.e., increased urinary Na+ excretion) occurs primarily as a result of the increase
in GFR, the increase in renal medullary or papillary blood flow, or both.
d. Inhibition of aldosterone secretion. ANF blocks aldosterone secretion that was prestimulated by Na+ depletion, angiotensin II, ACTH, K+, and cyclic adenosine 3',5'-monophosphate (cAMP).
e. Inhibition of renin secretion occurs via the increase in NaCI delivery to the macula densa
or the increase in hydrostatic pressure at the JGA due to afferent arteriolar vasodilation.
2. Cardiovascular effects
a. Decreases in mean systemic arterial blood pressure occur due to vasorelaxation or
suppression of renin secretion.
b. Reduction in cardiac output occurs primarily due to bradycardia.

STUDY QUESTIONS
Directions: Each of the numbered items or incomplete statements in this section is followed by answers
or by completions of the statement. Select the one lettered answer or completion that is best in each
case.
1. The renal threshold for glucose excretion normally corresponds to a plasma glucose concentration of
(A)
(B)
(C)
(0)
(E)

80 mg/dl
100 mg/dl
120 mg/dl
160 mg/dl
180 mg/dl

2. The following renal function data were collected for substance x: urine flow rate = 90 ml/
hr, concentration of substance x in urine = 480
mg/ml, and concentration of substance x in
plasma = 6 mg/ml. What is the renal clearance of
substance x?
(A)
(B)
(C)
(0)
(E)

12 ml/min
120 ml/min
120 mg/min
240 ml/min
480 ml/min

3. The following data were derived from a renal


function test in a normal person: GFR = 180 LJ
day, plasma uric acid concentration = 5 mg/dl,
and uric acid excretion rate = 700 mg/day. What
conclusion can be reached regarding renal processing of uric acid?
(A) Uric acid is secreted and reabsorbed but at
different rates
(B) Uric acid is only secreted
(C) Uric acid is only filtered
(0) Uric acid is not metabolized by the kidney
(E) Uric acid is filtered and reabsorbed but not
secreted

4. The primary renal site for the secretion of organic ions (e.g., PAH, urate, creatinine) is the
(A) proximal tubule
(6) thin segment of the loop of Henle
(C) thick segment of the loop of Henle
(0) distal tubule
(E) collecting duct

5. T ranscellular fluid is found in the lumen of structures lined by epithelial cells. An example of transcellular fluid is
(A) plasma
(B) CSF

(C) interstitial fluid


(0) edematous fluid

(E) intravascular fluid


6. The renal transport maximum (Tm) for a substance is defined as the maximal
(A) GFR

(B)
(C)
(0)
(E)

urinary excretion rate


tubular reabsorption or secretion rate
renal clearance rate
amount of a substance filtered by the glomeruli
per minute

7. Of the renal systems available for the excretion


of H +, the one with the greatest activity is
(A)
(B)
(C)
(0)
(E)

H + secretion
NaH 2 P0 4 excretion
SO/- excretion
titratable acid excretion
NH4+ excretion

8. Which of the following substances undergoes


both reabsorption and secretion in the kidney?
(A)
(B)
(C)
(0)
(E)

PAH
Lactate
Urea
Uric acid
Creatinine

9. When the secretion of PAH reaches the Tm,


a further increase in plasma PAH concentration
causes its clearance to
(A) increase in proportion to its plasma concentration
(B) approach glucose clearance asymptotically
(C) remain constant
(0) approach inulin clearance asymptotically
(E) increase in proportion to GFR

10. All of the following statements concerning renin are true EXCEPT

14. All of the following statements regarding K+


transport are true EXCEPT

(A) renin release from the JG cells is inversely related to the degree of stretch in the wall of the
afferent arteriole
(B) renin is a secretory product of the JG cells
(C) renin substrate is a hepatic globulin
(0) the renin response to Na+ depletion is attenuated in the absence of renal nerves
(E) renin secretion is a prerequisite for aldosterone
secretion

(A) K+ is the only inorganic electrolyte that is both


reabsorbed and secreted by the kidney
(B) the filtered load of K+ is more than that of Na+

11. All of the following conditions will evoke an


increase in plasma AOH level EXCEPT
(A) dehydration
(B) trauma
(C) hemorrhage
(0) hyponatremia
(E) pain
12. Ions constitute approximately 95% of the solutes in body water. The body fluid compartment
with the highest protein concentration is the
(A) ICF
(B) interstitial fluid
(C) plasma
(0) CSF
(E) transcellular fluid
13. Which of the following statements regarding
Na+ transport is correct?
(A) Active transport of Na + across all cells consumes most of the energy derived from cellular
metabolism
(B) The Na+ concentration is highest in the intracellular fluid
(C) Na+ reabsorption across proximal tubular cells
is mainly active and transcellular
(0) The Na+ concentration gradient provides energy for the cotransport of H +
(E) The transport of Na+ across the luminal membrane of the nephron is an active transport
process

(C) insulin stimulates cellular uptake of K+


(0) urinary K+ excretion is primarily dependent on
K+ secretion from the distal tubular cell into
the lumen
15. Small solutes are transported across the glomerular capillaries by the process of
(A) simple diffusion
(B) convection
(C) facilitated diffusion
(0) primary active transport
( E) secondary active transport
16. The renal threshold for a solute denotes the
(A) maximum filtration rate
(B) maximum reabsorption rate
(C) plasma concentration at which a solute begins
to appear in the urine
(0) maximum secretion rate
(E) maximum tubular secretory capacity
17. Which of the following conditions causes a
decrease in the ECF volume, an increase in the ICF
volume, and a decrease in the osmolar concentration of both compartments?
(A) Hyperosmotic dehydration
(B) Hyposmotic dehydration
(C) Isosmotic dehydration
(0) Hyperosmotic overhydration
(E) Hyposmotic overhydration
18. The volume of the ECF is regulated primarily
by the reabsorption and excretion of
(A) sodium lactate
(B) sodium bicarbonate
(C) sodium chloride
(0) sodium phosphate
(E) sodium citrate

19. If the GFR remains unchanged, a decrease in


the urine-to-plasma inulin concentration ratio (U in /
Pin) indicates that
(A) free-water clearance has decreased
(B) urine flow has increased
(C) inulin clearance has decreased
(D) plasma inulin concentration has decreased
(E) creatinine clearance has increased
20. The major contributor to the osmotically active solutes in the ECF is
(A)
(B)
(C)
(D)
(E)

sodium
albumin
urea
creatinine
potassium

24. Given a GFR of 125 ml/min, a plasma glucose


concentration of 400 mg/l 00 ml, a urine glucose
concentration of 75 mg/ml, and a urine flow of 2
ml/min, what is the Tm for glucose?
(A)
(B)
(C)
(D)
(E)

300
350
400
500
550

mg/min
mg/min
mg/min
mg/min
mg/min

25. The following graph shows the tubular loss of


glucose (excretion rate) plotted against the rate at
which glucose is filtered at the glomerulus (filtered
load). Which lettered point on the curve corresponds to the Tm for glucose?

21. Which of the following substances has the


lowest renal clearance?
(A)
(B)
(C)
(D)
(E)

Glucose
Urea
Inulin
Creatinine
PAH

22. Stimulation of renin secretion will cause an


increase in the
(A) K+ concentration in the blood
(B) volume of the ECF
(C) hematocrit
(D) plasma colloid osmotic pressure
(E) H + concentration in the blood
23. A drug that completely blocks the secretion of
PAH is given to a laboratory animal. The clearance
of which of the following substances will be the
same as the PAH clearance under this condition?
(A)
(B)
(C)
(D)
(E)

Water
Sodium
Urea
Inulin
Bicarbonate

Filtered load
26. Which of the following factors can best explain an increase in the GFR?
(A) Increased arterial plasma oncotic pressure
(B) Increased hydrostatic pressure in Bowman's
capsule
(C) Increased glomerular capillary hydrostatic
pressure
(D) Decreased net filtration pressure
(E) Vasoconstriction of the afferent arteriole
27. Which of the following forces opposes ultrafiltration at the glomerulus?
(A)
(B)
(C)
(D)
(E)

Plasma colloid osmotic pressure


Glomerular capillary hydrostatic pressure
Colloid osmotic pressure in Bowman's capsule
Concentration of NaCI in Bowman's space
Crystalloid osmotic pressure of the final urine

28. Measurements taken after an intravenous injection of inulin indicate that the substance appears
to be distributed throughout 30%-35% of the total
body water. This finding suggests that inulin most
likely is
(A) excluded from the cells
(8) distributed uniformly throughout the total body
water volume
(C) restricted to the plasma volume
(0) neither excreted nor metabolized by the body
(E) not freely diffusible through capillary membranes

Directions: Each question below contains four suggested answers of which one or more is correct.
Choose the answer.

A if 1, 2, and 3 are correct


B if 1 and 3 are correct
C if 2 and 4 are correct
D if 4 is correct
E if 1, 2, 3, and 4 are correct

29. There is an inverse relationship between the


rate of ADH secretion and the rate of discharge
in afferent neurons from stretch receptors in the
vascular system. These pressure receptors are
found in which of the following regions of the vascular system?
(1) Great pulmonary veins
(2) Left atrium
(3) Aortic arch
(4) Carotid sinuses

30. A 38-year-old male patient presents with serum Na+ concentration of 155 mEq/L together
with a urine osmolality of 50 mOsm/kg H 2 0.
These findings could be explained by
(1) a lack of ADH
(2) volume expansion with isotonic saline
(3) an increase in free-water clearance
(4) excessive ingestion of water

31. The proximal tubular reabsorption of Na +


would be markedly reduced by
(1) removal of the pituitary gland
(2) a decrease in plasma glucose concentration
(3) an increase in H+ secretion
(4) a deficiency of luminal membrane transport
proteins
32. Vasoconstriction of the renal artery can lead
to an elevation in mean arterial pressure by
(1) increased renin secretion
(2) increased angiotensin II formation
(3) increased aldosterone secretion
(4) decreased plasma bradykinin levels

Directions: Each group of items in this section consists of lettered options followed by a set of numbered
items. For each item, select the one lettered option that is most closely associated with it. Each lettered
option may be selected once, more than once, or not at all.
Questions 33-37

Questions 38-42

Match each of the following descriptions with the


most appropriate lettered region of the nephron
pictured below.

Match each of the following filtered substances


with the appropriate renal transport process.

(A)
(B)
(C)
(D)

Reabsorption
Secretion
Both
Neither

38. K+
39. Mannitol
40. HC0 3 -

Q)

t:::

()

41. Creatinine
42. Urea

Questions 43-45
Match each description with the most appropriate
morphologic segment of the nephron.

33.
34.
35.
36.
this
37.

Tubular fluid always is hyposmotic at this site


The TF / P ratio for glucose is 1.0 at this site
The urea concentration is highest at this site
The Uosm can reach 1200-1400 mOsm/L at
site
The macula densa is closest to this site

(A) Glomerulus
(B) Proximal convoluted tubule
(C) Thin descending limb of the loop of Henle
(0) Thick ascending limb of the loop of Henle
(E) Collecting tubule
43. Major site of ADH-induced water permeability
44. Site where largest fraction of water is reabsorbed
45. Concentrating segment of the nephron

Questions 46-51
Match each of the following substances with the appropriate lettered TF / P curve on the graph below.
A

5.0

4.0

3.0

2.0

TF
P
1.4

1.0

,,

0.8

' ........

0.6
0.4

.... .....

......

......

0.2

------- __ D

0
0

25

50

75

100

% Proximal tubule length

46. Na+
47. Glucose
48. Glycine

49. eI50. Heo)51. PAH

ANSWERS AND EXPLANATIONS


1. The answer is E. [VI A 2; Figure 4-11A] The renal threshold for glucose is the plasma glucose
concentration at which glucose begins to appear in the urine and is equal to 180 mg/di. It is important
to note that this concentration is not the plasma glucose concentration that completely saturates the
renal glucose transport mechanism (Tm = 340 mg/min); glycosuria occurs long before the transport
mechanism is fully saturated.
2. The answer is B. [IV A 1,2] The renal clearance of a substance (C) is defined as the plasma volume
from which a given substance is removed per minute. Thus, it is an empiric measure of the volume of
plasma that contains the same amount of the substance as is excreted in urine in 1 minute. The clearance
equation is

Ux x

=--

Px

where U x and Px = the urinary and plasma concentrations of substance x in mg/ ml, respectively;
V = the urine flow rate in ml/min; and Cx = the clearance of substance x in ml/min. Using the data
given
C

480 mg/ml x 1.5 ml/min


=

120 ml/min

6 mg/ml

Notice that the units of concentration cancel out, leaving the units for clearance as ml/min.
3. The answer is A. [VI A 1 a, 3 a] Clearance of a substance provides no information about the
mechanism by which the kidney removes a substance from the plasma. In relation to this question, the
.
data can be analyzed as follows:
input - output
(GFR x Puric aCid)

amount transported

(U uric acid

V)

amount transported

Substituting,
(180 L/day x 50 mg/L) - 700 mg/day
9000 mg/day - 700 mg/day

amount transported

8300 mg/day

Notice that approximately 8% of the filtered urate is excreted in the urine. Since the amount of uric acid
filtered is higher than the amount excreted, this substance on a net basis is filtered and reabsorbed.
However, in mammals (including humans), the tubular transport of uric acid is bidirectional in the
proximal tubule, with reabsorption and secretion occurring within the same tubular cell. Uric acid is the
end product of purine metabolism, with approximately 70% of the uric acid eliminated by the kidney.
In humans, the bulk of the excreted urinary nitrogen is in the form of urea and ammonia, with excretion
of uric acid accounting for only 5% of the total. Normally, more than 90% of the filtered uric acid is
reabsorbed in the proximal tubule. Hence, tubular secretion must account largely for the bulk of uric
acid excreted in the urine.
4. The answer is A. [VI 8 3 a, b] The proximal tubule is the most important site of organic ion secretion
because it contains an abundance of protein carriers. Secretion refers to the active or passive transport
of a substance from the peritubular blood, interstitium, or tubular cell into the tubular lumen. The
term "secretion" excludes the entry of substances into the tubular lumen by glomerular filtration.
Para-aminohippuric acid (PAH) has a higher clearance than almost any other known substance. It is a
weak organic acid, actively secreted primarily into the proximal tubule by a Tm-limited process. Creatinine, an endogenous product of muscle metabolism, enters the tubular lumen by the organic secretory
pathway in the proximal tubule. Like PAH, creatinine is freely filtered and is not reabsorbed, synthesized,
or metabolized by the kidney. Uric acid, derived from the metabolism of purine nucleotides, exhibits
bidirectional tubular transport mainly in the proximal tubule, with reabsorption exceeding secretion.
5. The answer is B. [II A 2 c] Transcellular fluid is the extracellular fluid in the lumen of structures lined
by epithelium. The largest component of the transcellular fluid is the intraluminal gastrointestinal water;
other transcellular fluids include the cerebrospinal fluid, bile, synovial fluid, and ocular fluid. Transcellular
fluid accounts for about 1 L of body fluid, which is equivalent to 2.5% and 1.5% of the total body
water and body weight, respectively. The plasma, interstitial, edematous, and intravascular fluids all are
extracellular fluids but do not contribute to the transcellular fluid compartment.

6. The answer is C. [VI A 7 a, b, 3 d] Tubular maximum or transport maximum, both abbreviated as


Tm, is defined as the upper limit for the unidirectional rate of active transport, either reabsorptive or
secretory, depending on the direction of solute transport. The units for Tm are mg/min. Thus, there is a
maximal rate of reabsorption called maximum tubular reabsorptive capacity and a maximal rate of
secretion called maximum rate of tubular secretory capacity. Tubular maxima vary with the substance
involved. Renal reabsorption of glucose and secretion of PAH are examples of actively transported
solutes exhibiting tubular transport maxima. An example of a substance that is not Tm-limited is the
reabsorption of Na+ along the nephron. Tubular maximum is the difference between the filtered load
and the rate of excretion of a solute. Glomerular filtration rate (GFR) is defined as the volume of plasma
filtered per minute (ml/min), while clearance is the virtual volume of plasma from which a substance is
removed per minute (ml/min).
7. The answer is E. [VI C 4] The concentration of free H+ at urinary pH above 4.4 is negligible;
therefore, acid must be excreted in a buffered form. The two main urinary buffers are phosphate (HP0 4-)
and ammonia (NH 3 ). The amounts of secreted H+ excreted bound to ammonia and phosphate are
measured as ammonium (NH 4+) and titratable acidity, respectively. The sum of ammonium and titratable
acidity minus the amount of excreted bicarbonate (He0 3 - ) equals net acid excretion, which normally
is approximately 1 mEq/kg body weight/24 hr. Ammonia production occurs primarily in the proximal
tubule, and it represents the major adaptive mechanism available to the kidney for the increased excretion
of H+ during states of acidosis. In contrast, the phosphate buffer enters the tubular lumen by glomerular
filtration. The normal kidney excretes almost twice as much acid combined with ammonia (40 mEq H+ /
day) than it excretes titratable acid (20 mEq H+ /day). Approximately 4300 mEq of H+ must be secreted
per day to accomplish the reabsorption of 4300 mEq of bicarbonate. In this process, most of this secreted
H+ is reabsorbed in the form of water. Sulfate (SOi-) in combination with ammonium forms a neutral
salt [(NH4)2S04], which makes the tubular urine less acidic.

8. The answer is D. [VI A 7 c] Uric acid is the only organic substance that is both reabsorbed and
secreted-two carrier-mediated processes that occur mainly in the proximal tubule. The reabsorption of
uric acid exceeds its secretion. Lactate normally is completely reabsorbed by active transport. Urea, the
nitrogenous end product of protein catabolism, is partially reabsorbed (35%-60%) from the tubular fluid
by simple diffusion in many regions of the nephron and, therefore, does not exhibit a tubular maximum.
Para-aminohippuric acid (PAH) and creatinine are secreted mainly by the proximal tubules. The secretion
of PAH involves an active, carrier-mediated transport system and, therefore, exhibits a tubular (transport)
maximum. The clearance of PAH is higher than that of almost any other known substance. When the
plasma PAH concentration is low, virtually all the PAH that escapes filtration is secreted.

9. The answer is D. [VI B 3; Figures 4-7, 4-7 7C] At low plasma concentrations of PAH, PAH is almost
completely cleared from the plasma by a combination of glomerular filtration and tubular secretion.
When plasma concentrations of PAH are elevated beyond 30 mg/dl, the secretory mechanism becomes
saturated, and the tubular transport maximum is reached. As the tubular secretory mechanism becomes
saturated and is exceeded by progressive increments in plasma PAH concentration, the clearance of
PAH declines and becomes more a function of glomerular filtration. The PAH clearance asymptotically
approaches the inulin clearance. Thus, the amount of PAH secreted becomes a smaller fraction of the
total amount of PAH excreted. The clearance of PAH always is greater than the clearance of inulin, since
some PAH is always secreted.

10. The answer is E. [IX 0 2 c] Renin secretion is not always required for aldosterone secretion. Renin
is a proteolytic enzyme produced by the juxtaglomerular (JG) cells, which are modified smooth muscle
cells in the renal afferent arterioles. The major stimulus for the release of renin is a decrease in the
perfusion pressure of blood traversing these afferent arterioles. Thus, the JG cells function as low-pressure
baroreceptors (volume receptors) where the secretion of renin varies inversely with the degree of stretch
in the wall of the afferent arteriole. The major extrinsic factor influencing renin secretion is the renal
postganglionic sympathetic vasomotor nerves, which are not necessary for renin secretion but exert a
stimulatory effect (via ~-adrenergic receptors) on the magnitude of renin release in response to a given
stimulus. Increased K+ concentration elicits aldosterone secretion by depolarizing the zona glomerulosa
membranes; hyperkalemia inhibits renin secretion. Renin substrate, called angiotensinogen, is an
u 2-globulin of hepatic origin containing a tetradecapeptide moiety that serves as the pro hormone for the
angiotensins.

11. The answer is D. [VIII B 7 a, 3 a, b] Normally, a 1%-2% increase in plasma osmolality causes an
increase in plasma antidiuretic hormone (ADH) level. The central hypothalamic osmoreceptors are the

primary regulators of ADH synthesis and release. The osmoreceptors communicate with the ADHproducing cells, mainly in the supraoptic nucleus. The renal regulation of blood volume and osmolality
involves factors that modify the synthesis and release of ADH and aldosterone. Dehydration (hydropenia),
trauma, hemorrhage (hypovolemia), pain, and anxiety are stimuli of ADH release. With massive blood
loss, osmolality is sacrificed in order to maintain the volume of the circulation. Hyponatremia represents
a decline in the osmolar concentration of plasma and, therefore, an inhibitory signal for ADH secretion.
Hypertonic solutions of saline, sucrose, and mannitol stimulate ADH release, while increases in plasma
osmolality due to glucose or urea have little or no effect.
12. The answer is A. [/I D 3; Figure 4-5; Table 4-3A] The body fluid compartment with the highest
protein concentration is the ICF, which has a protein concentration of about 60 mEq/L (4 mmol/ U. In
decreasing order, the concentration of protein in the body fluid compartments is as follows:
Plasma
Interstitial fluid
Cerebrospinal fluid (CSF)

15 mEq/L
1 mEq/L
0.05 mEq/L

1 mmol/L
0.06 mmol/L
0.003 mmol/L

CSF is part of the transcellular fluid compartment. The milliequivalent concentrations of proteins in the
body fluids are based on an average valence of -15 and an average molecular weight of 65,000 daltons.
13. The answer is C. [11/ D 3; VI C " 2 a; Table 4-4] The Na+-K+-ATPase enzyme in all cell membranes
is the carrier molecule responsible for the active Na+ transport (efflux) and the active K+ transport
(influx). This enzyme maintains the low intracellular Na+ concentration and high extracellular Na+
concentration. Active Na+ transport consumes 30%-50% of the energy derived from metabolism in
most cells. Although the influx of Na+ from the tubular lumen to the proximal tubular cell is in the
direction favored by the electrochemical potential, this transport is mediated by specific membrane
carrier proteins and not by simple diffusion. These membrane proteins couple the active movement of
other solutes to the passive movement of Na+. Examples include Na+ -glucose and Na+ -amino acid
cotransporters and a Na+-H+ countertransporter. In each case, the potential energy released by the
downhill transport of Na+ is used to power the uphill transport of the other substance. These transport
systems are referred to as Na+-coupled, secondary active transport processes. Most of the proximal
reabsorption of Na+ occurs by active transport and is transcellular. The transcellular pathway consists
of luminal (apical) and basolateral membranes. It is essential to understand that reabsorption includes
not only Na+ influx from the tubular lumen but also active transport of Na+ out of the cell into the
bloodstream.
14. The answer is B. [VI C 3 a-c] Since the plasma K+ concentration is approximately one-thirtieth
that of Na+, the filtered load of K+ is much less than that of Na+. K+ and uric acid transport are
bidirectional in the proximal tubule, with reabsorption and secretion occurring within the same tubular
cell. Thus, K+ is the only inorganic cation that is both reabsorbed and secreted by the kidney. Most of
the filtered K+ is actively reabsorbed in the proximal tubule, whereas the excreted K+ is derived mainly
from K+ secretion in the distal tubule. Secretion involves the active pumping of K+ into the distal tubular
cells across the peritubular membrane followed by the passive diffusion across the luminal membrane
into the tubular lumen. Insulin stimulates K+ uptake by cells.
15. The answer is B. [11/ C 1] Small solutes are transported across glomerular capillaries by the passive
process of convection (bulk flow). Convection is caused by a hydrostatic pressure difference causing the
fluid with its dissolved material to flow from a region of high pressure to one of lower pressure. As there
are no concentration gradients for substances across the glomerular membrane, there can be no simple
diffusion. Primary active transport denotes a carrier mechanism for the movement of a solute against a
concentration difference or a combined concentration and electrical potential difference, which is
directly coupled to metabolic energy. Secondary active transport denotes a process that mediates the
uphill movement of solutes, which is not directly coupled to metabolic energy expenditure; the energy
required is derived from the concentration gradient of another solute (Na+).
16. The answer is C. [VI A 2; Figure 4-11A] The renal threshold for a substance denotes the plasma
concentration at which the solute begins to appear in the urine. It is not the plasma concentration that
completely saturates the transport mechanism either for reabsorption or secretion. When the plasma
concentration of a solute exceeds the renal threshold, the amount of that solute transported through the
nephron exceeds the tubular transport maximum, and the solute appears in the urine in increaSing

amounts. The amount of a substance entering the tubule by filtration per unit time defines the filtered
load.
17. The answer is B. [/I C 1-3; Figures 4-3, 4-4; Table 4-2] Hydration states are named in terms of the
ECF compartment. Overhydration, or fluid and salt retention, results from excessive influx of water and
NaCI. Dehydration, or fluid and salt depletion, usually involves both the ECF and the ICF. The state of
overhydration or dehydration is named only in terms of the volume of the ECF compartment that
increases or decreases. The volume of the ECF compartment is determined by the Na+ content and not
by the Na+ concentration. The situation described in the question represents a state of dehydration
because of the contraction of the ECF volume. Since the solute concentration of the ECF also is decreased,
there is a hyposmotic dehydration (e.g., as occurs with excessive salt loss in adrenal insufficiency). A
net loss of salt in excess of water loss leads to hyposmolality of the ECF and a shift in water from the
ECF to ICF. Thus, the volume of the ECF is decreased, the volume of the ICF is increased, and the
osmolality of both is decreased.
18. The answer is C. [/I C 1 a; Figure 4-5] The major factor regulating ECF volume is the reabsorption
and excretion of sodium chloride. More than 9D% of the osmotically active solutes in the ECF volume
are salts of sodium. Most of these salts (e.g., sodium lactate, sodium citrate) exist in concentrations too
small to affect the volume of the ECF. The concentrations of other sodium salts (e.g., sodium bicarbonate,
sodium phosphate) are maintained within very narrow limits in order to maintain homeostatic functions
other than ECF volume regulation.
19. The answer is B. [IV B " C 2 b (1), (2); Figures 4-8,4-9,4-13] The urine-to-plasma concentration
ratio (U / P) reflects the concentration gradient between the urine and the plasma. Since inulin is neither
reabsorbed from nor secreted into the nephron, the inulin concentration increases as water is reabsorbed.
In fact, the inulin concentration in the tubular fluid or urine will be solely a function of the amount of
filtered water that is reabsorbed. Normally, 7D% of the filtered water is reabsorbed in the proximal
tubules, and more than 99% is reabsorbed by the entire tubular system. A decrease in the urine-to-plasma
inulin concentration ratio (Uin/P in ) with a constant GFR indicates that less water was reabsorbed, and
the urinary inulin concentration was decreased. Thus, the urine flow was elevated.
20. The answer is A. [/I C 1 a, 02 a] Osmotic forces are the primary determinant of water distribution
in the body. Consequently, the distribution of the total body water within the ICF and ECF compartments
is determined by the number of osmotically active particles in each compartment. Sodium salts are the
principal extracellular osmotically active solutes and act to hold water in the extracellular space. Urea
is an ineffective osmole, while potassium salts account for almost all of the intracellular osmoles. The
plasma concentration of protein is low (1.1 mmol/L or 15 mEq/L) as is that of creatinine (D.l mmol/L).
21. The answer is A. [IV C 1 c (1)-(3); Figure 4-7] Renal clearance can be measured for any solute
present in the plasma and excreted by the kidney; clearance best represents the rate of elimination of
a substance by the kidney. If the kidney removes a substance (e.g., inulin) from the plasma by filtration
only, the tubular clearance will be zero and the excretion rate will be proportional to the plasma
concentration. If the clearance involves filtration and either tubular reabsorption or secretion, then
clearance depends on the plasma concentration and the tubular transport capacity for reabsorption or
secretion. Thus, clearance is higher for substances that are filtered and secreted than for substances that
are filtered and reabsorbed. The lowest clearance is observed with substances that are completely
reabsorbed, such as glucose. Approximately 5D% of the filtered urea is reabsorbed. Creatinine and PAH
are filtered and secreted, which increases their clearance compared to inulin.
22. The answer is B. [IX B 3, 0 2 c, d] The only known physiologic effect of renin is to cause the
formation of angiotensin I from its plasma substrate, angiotensinogen. Angiotensin I, in turn, appears to
serve only as a specific substrate for angiotensin-converting enzyme (ACE), a peptidase that converts
angiotensin I to angiotensin II, which does have significant biologic activity. Angiotensin II exerts a potent
vasoconstrictive action on the vascular smooth muscle of peripheral arterioles, which causes an increase
in the mean arterial blood pressure. It also stimulates the release of aldosterone from the zona glomerulosa
of the adrenal cortex, causing an increase in the ECF volume through the increased active reabsorption
of Na+ by the distal tubules and collecting ducts. The increased ECF volume accounts for the decreased
concentration of K+, the decreased hematocrit, and the decreased plasma oncotic pressure. Aldosterone
also acts on the distal tubule and collecting duct to increase the net reabsorption of Na+ in exchange
for the secretion of K+ and H +. Thus, aldosterone tends to produce hypokalemia and metabolic alkalosis.

23. The answer is D. [IV 8 7 a; VI 8 3 a, b; Figures 4-7, 4-9] Secretion refers to the transport of a
substance from the peritubular blood, to the interstitium, to the tubular cell, and then into the tubular
lumen regardless of whether the movement is active or passive. Secretion does not refer to the movement
of substances across the lumen by glomerular filtration. Secretion is an active transport process that
occurs mainly in the proximal tubule. PAH is actively secreted by the proximal tubular cells and is
excreted without being metabolized. If a substance blocks the secretion of PAH, which is only filtered
and secreted, then PAH is only filtered and neither reabsorbed nor secreted. Such a substance would
resemble inulin, which is only filtered.
24. The answer is B. [VI A 3; Figure 4-7 7A] Using the data given, the Tm for glucose is found to be
350 mg/ min. Tm refers to the maximal rate of transport of a substance. All substances that are reabsorbed
or secreted have a Tm, which is expressed as an amount per minute. To determine whether the excretion
of a substance involves either reabsorption or secretion (in addition to glomerular filtration), it is necessary
to compare the excretion rate of the substance with the rate at which the substance is filtered at the
glomerulus (filtered load). The algebraic difference between filtered load and the amount excreted
determines which of the two processes is used by the kidney to excrete a substance. In the case of
glucose, the amount excreted is less than the amount filtered, which is consistent with reabsorption, as
Tm

amount filtered - amount excreted


(125 ml/min x 4 mg/ml) - (75 mg/ml x 2 ml/min)
= 500 mg/min - 150 mg/min
= 350 mg/min
=
=

25. The answer is B. [VI 8 7-3; Figure 4-7 7A] When the filtered load of a substance exceeds the Tm
for that substance, the excess is not reabsorbed, and, thus, proportionately more of the substance is
excreted. In this example, the Tm for glucose (point 8 on the curve) is determined by extrapolation of
the linear portion of the curve (from point C to point 0) to the abscissa (filtered load). The splay of the
curve (from point A to point C) indicates that all of the tubules are not uniform in length, number of
glucose transporters, or renal threshold.
26. The answer is C. [V A 7 a, C 7-3] The major determinant of GFR is the hydrostatic pressure within
the glomerulus. An increase in glomerular capillary hydrostatic pressure is a major outward directed
force for fluid to leave the glomerular capillary and enter Bowman's capsule. In glomerular capillaries,
the net movement of fluid is primarily or solely out of the capillaries, whereas in systemic capillaries the
change in the balance of Starling forces is such that net movement out of the capillaries is nearly balanced
by net return of fluid into the vessels. Increases in plasma oncotic pressure, hydrostatic pressure in
Bowman's space, and afferent arteriolar resistance lead to a decrease in GFR, as does a decrease in net
filtration pressure.

27. The answer is A. [V A 7 a] Ultrafiltration separates water and nonprotein constituents (the "crystalloids") of plasma-which enter Bowman's space-from the blood cells and protein macromolecules
(the "colloids")-which remain in the blood. The glomerular ultrafiltration mechanism is governed by
the same passive Starling's forces that determine the translocation of fluid across other capillaries in the
body, namely, the imbalance between transcapillary hydrostatic and colloid osmotic (oncotic) pressures.
The hydrostatic pressure of the afferent arteriolar blood exceeds the sum of the antifiltration forcestubular hydrostatic pressure plus glomerular capillary oncotic pressure-thereby causing net filtration.
Since albumin is the main component of the plasma oncotic pressure, which serves to hold fluid within
the vascular space, the glomerular impermeability to albumin contributes to the maintenance of plasma
volume by preventing renal loss of albumin.
28. The answer is A. [/I A 2; Figures 4-3, 4-4; Table 4-7] The ECF volume constitutes approximately
one-third of the total body water, or about 12-19 L. This compartment includes two subcompartments
separated by the capillary membrane: intravascular fluid or blood plasma, and interstitial fluid. The ECF
volume is measured with a test substance that does not penetrate the cells. Therefore, it has become
useful to measure the volume distribution of a specific substance and refer to it as, for example, the
inulin space, if the test substance is inulin. All substances used to measure ECF volume must cross
capillaries and distribute at the same concentration in plasma and interstitial fluid.
29. The answer is E (all). [VIII 8 3 a (2)] The major stimuli for ADH (vasopressin) secretion are an
increase in the plasma osmolality and a decrease in the effective circulating blood volume. There is an

inverse relationship between ADH secretion and the nerve impulse frequency in the autonomic (visceral)
afferent neurons from the baroreceptors (also called volume receptors, pressoreceptors, or stretch
receptors) in both low- and high-pressure regions of the vascular system. Thus, a decrease in vagal
impulses (left atrium and aortic arch) and glossopharyngeal nerve impulses (carotid sinus) results in an
increase in ADH secretion. The afferent neurons from the left atrium, carotid sinus, and aortic arch form
a primary synapse in the nucleus tractus solitarius. The left atrial receptors are more sensitive to small
changes in blood volume than are the carotid sinus and aortic arch baroreceptors.

30. The answer is 8 (1, 3). [VII E; Figure 4-16] The signs of hypernatremia and the excretion of a
hyposmotic urine are consistent with a lack of ADH. The polyuria accounts for the water diuresis
observed in this patient. The lack of ADH increases free-water clearance and results in the tendency to
increase the concentration of the serum electrolytes, including Na+.
31. The answer is C (2, 4). [VI C 2 a (1)-(2), 4 a, b; Figure 4-6] Although the influx of Na+ across the
luminal membrane is in the direction favored by the electrochemical gradient, Na+ transport is mediated
by specific membrane carrier proteins and is not by simple diffusion. In the proximal tubule, Na+
combines with a glucose carrier and glucose is transported in the same direction against a concentration
gradient. The Na+ and glucose transport system is an example of cotransport (symport). The energy for
the active transport of glucose comes from the Na+ concentration gradient. Thus, a reduction in the
intraluminal glucose concentration or a reduction in the number of carrier proteins in the proximal tubular
cells will reduce Na+ reabsorption. Na+ reabsorption also is dependent on active H+ secretion. Removal
of the pituitary gland does not have a major effect on Na + reabsorption, because adrenocorticotropic
hormone (ACTH) is not necessary for aldosterone secretion.

32. The answer is E (all). [IX 0 2] The release of renin from the JG cells is inversely related to the
degree of stretch in the wall of the afferent arteriole containing these cells. Thus, any factor that decreases
renal perfusion pressure (e.g., hemorrhage, renal arterial vasoconstriction) will tend to reduce the renal
perfusion pressure. The resultant increase in renin secretion will lead to the formation of angiotensin I
which, in turn, is converted to angiotensin II by ACE, an enzyme that is identical to kininase II. Kininase
II inactivates bradykinin, which is a vasodilator. The angiotensin II stimulates aldosterone secretion from
the zona glomerulosa of the adrenal cortex.
33-37. The answers are: 33-C, 34-A, 35-E, 36-E, 37-C. [11/ C 1 b; VII BIb, 2, 3 a, c, 4 b, 02 a; IX

o2

a (2); Figures 4-120, 4-13, 4-17] The solute concentration in the ascending limb of the loop of
Henle (site C) is less than that in any segment of the descending limb. The tubular fluid leaves the
ascending limb at a lower concentration than it had when it entered the descending limb. Thus, the fluid
presented to the distal tubule always is hyposmotic, regardless of the body's state of hydration.
Ultrafiltration separates water and nonprotein constituents (the "crystalloids") of plasma from the blood
cells and protein macromolecules (the "colloids"). Except for proteins and lipids, the concentrations of
crystalloids (e.g., Na+, glucose) in the plasma and in Bowman's space (site A) are nearly the same.
The wall of the ascending limb of the loop of Henle is relatively impermeable to water. Therefore,
NaCi in this segment is reabsorbed to the virtual exclusion of water, a process that renders the medullary
and papillary interstitium hyperosmotic to plasma. The medullary interstitial osmolality is higher in
antidiuresis than diuresis, due largely to urea. Thus, the highest osmolality exists in the papillary interstitium. With continued reabsorption of water, urea becomes even more concentrated at the terminals of
the collecting ducts (site fl.
During dehydration with maximal ADH secretion, the Uosm/Posm approaches 4 to 1 (at site fl because
of the increased free-water reabsorption.
The macula densa is located at the junction of the thick segment of the ascending limb of the loop of
Henle and the distal convoluted tubule (site C).

38-42. The answers are: 38-C, 39-0, 40-C, 41-8, 42-A. [IV BIb, C 1 C (1) (b), 0 1 c, 2 a; VI C 3
a- c, 4; VII 0] K+ is the only plasma inorganic electrolyte that is both reabsorbed from and secreted
into the renal tubule. K+ is largely actively reabsorbed by the proximal nephron, whereas the excreted
K+ is derived mainly from K+ secretion in the distal nephron. Secretion involves active pumping of K+
into the distal tubular cells across the peritubular membrane followed by passive diffusion across the
luminal membrane into the tubular lumen.
Mannitol is a 6-carbon sugar alcohol that is filtered but not reabsorbed or secreted. Therefore, the
intravenous administration of mannitol raises the osmolality of tubular fluid and decreases the reabsorption of water, so that an osmotic diuresis ensues.

HCO l - is completely filterable at the glomerulus. In a normal person, virtually all of HCO l - is actively
reabsorbed, mainly by the proximal tubule. The remainder of the HCO l - reabsorption occurs in the
loop of Henle and distal tubule. HCO l - also can be secreted by the cortical collecting tubules.
For practical purposes, endogenous creatinine clearance is equal to inulin clearance. However, exogenous creatinine is not only freely filtered but also is secreted by the proximal tubule. Thus, creatinine
clearance exceeds inulin clearance by 20%-30%.
Urea is filtered and passively reabsorbed by the nephron. The reabsorption of urea is not limited to
the proximal tubule. The net reabsorption of urea ranges between 40% and 60%.

43-45. The answers are: 43-E, 44-8, 45C. [VI C 6 a; VII A 4; V/II C 1; Figures 4-12, 4-13] ADH, a
hormone primarily synthesized by the supraoptic nucleus of the diencephalon, is the major determinant
of water permeability in the entire cortical and medullary collecting duct system. ADH also increases
the passive permeability of the inner medullary collecting duct to urea.
Two-thirds of the glomerular filtrate is reabsorbed isosmotically by the proximal convoluted tubule. In
this process, the total osmolality remains essentially unchanged.
The thin descending limb of the loop of Henle is relatively impermeable to urea but is highly permeable
to water. Thus, water is reabsorbed from the descending limb in response to an osmotic gradient
established by urea. The thin descending limb also is relatively impermeable to NaCI. Thus, as water (but
not NaCIl is reabsorbed from this limb, the NaCi concentration of the tubular fluid increases until it
exceeds the NaCi concentration in the interstitium. This latter process provides for the passive reabsorption of NaCi from the ascending limb of the loop of Henle.

46-51. The answers are: 46-C, 47-E, 48-E, 49-8, 50-0, 51-A. [/II D 3 a; VI B 2, 3 a, b, C 2 a, 4 a, b,
5 a, b; Figures 4-6, 4-7, 4-9, 4-11; Table 4-6] Two-thirds of the filtered Na + is reabsorbed by the proximal
tubule. Since two-thirds of the water also is reabsorbed proximally, the concentration of Na+ remains
unchanged.
The rate of urinary glucose excretion always is less than the rate of glucose filtration at the glomerulus.
Thus, there is a net reabsorption of glucose that occurs solely in the proximal tubule. The tubular
fluid-to-plasma concentration ratio (TF / P) for glucose falls to a value of 0.1, indicating that 90% of the
filtered glucose is reabsorbed in the early portion (first quarter) of the proximal tubule. The filtered glucose
is reabsorbed by an active, carrier-mediated process with Tm-limited characteristics.
Like glucose, glycine is transported from the tubular fluid into the proximal tubular cell by specific
carrier molecules that also combine with Na +. Thus, glycine transport is a Na + -coupled, secondary
active transport process. The TF / P for amino acids also falls to a value of 0.1, indicating that about 90%
of the filtered glycine is reabsorbed in the initial 25% of the proximal tubule. The active reabsorption of
amino acids also involves a Tm-limited process.
Early in the proximal tubule, the reabsorbed anion is chiefly HCOl - , leaving behind a fluid enriched
in CI-. The rise in tubular fluid CI- concentration creates a gradient favoring the diffusion of CI- from
tubular fluid to the interstitial space. Most of the CI- that is absorbed does not enter the cell at all but
moves primarily via the paracellular pathway (i.e., through the tight junctions into the intercellular
channell. About 60% of the filtered CI- is reabsorbed proximally.
Early in the proximal tubule, the reabsorbed anion is chiefly HCOl - and constitutes about 90% of
the filtered HCO l - . The remaining 10% is reabsorbed in the distal tubule and the collecting ducts.
Reabsorption of HCO l - is accomplished by the active transport of H+ from the cell to the lumen.
HCO l - added to the peritubular capillary is derived from the intracellular dissociation of H 2 CO l , whereas
the filtered HCO l - is reabsorbed from the tubular fluid as CO 2 and H 2 0. There is no absolute Tm for
HCO l - because the reabsorption of HCOl - depends on Na+ reabsorption.
PAH is an organic anion secreted by the proximal tubule, where there is an abundance of protein
carriers. At low plasma concentrations, this exogenous substance is completely cleared by the kidney
via filtration and secretion.

5
Acid-Base Physiology
John Bullock

I. ACID PRODUCTION AND ELIMINATION. Although the body produces large amounts of
acid in two forms-carbonic (volatile) and noncarbonic (nonvolatile, or fixed) acids-the pH of the
body fluids is maintained in an alkaline state (7.4). Most of the hydrogen ion (H+) is formed as an
end product of metabolism. The pathways for acid removal include the kidneys, lungs, and gastrointestinal tract.
A. Carbonic acid (H 2 C0 3 ) formation. Because carbon dioxide (C0 2 ) can be formed from H 2C0 3
and, in turn, CO 2 can be eliminated by the lungs, H 2 C0 3 is called a volatile acid.
1. Unfortunately, the proton donor/acceptor terminology of BrQnsted prevents the classification
of CO 2 as an acid, but CO 2 functions as the single most important weak acid in the body fluids.
2. Most of the CO 2 is derived from oxidative metabolism.
B. Noncarbonic acid formation. Much smaller amounts of fixed acids are produced, and, because
they cannot be converted to CO 2 , they are called nonvolatile acids. Noncarbonic acids are
derived from three sources: diet, intermediary metabolism, and stool bicarbonate (HC0 3 -) loss.
1. Diet. A high protein diet, which is common in Western countries, accounts for the formation
of more acids than bases.
a. Foodstuffs such as glucose and triglyceride are not acids in the body fluids but are converted
to CO 2 during the course of their metabolism, and much of this CO 2 is hydrated to form
H 2 C0 3 , which then dissociates into H+ and HC0 3 -
b. A vegetarian diet will produce an excess of alkali, which must be excreted by the kidneys
as HC03 - .
2. Intermediary metabolism. The metabolism of foodstuffs does not always tend to acidify the
body fluids; some foodstuffs have an alkalizing action. For example, the ingestion of large
amounts of salts of organic acids found in fruit (e.g., lactate, isocitrate, citrate) alkalinize the
body fluids because these organic ions are metabolized to CO 2 and H 2 0, a process that involves
the consumption of H +. Between 40 and 60 mmol of inorganic and organic acids that are not
derived from CO 2 are produced daily. About half of the metabolically produced acids are
neutralized by bases in the diet, but the remainder must be neutralized by the buffer systems
of the body.
a. The metabolism of foodstuffs is a significant source of noncarbonic acids.
(1) Lactate is produced from the anaerobic metabolism of glucose or glycogen. In cases of
exercise or hypoxia, the excessive production of lactic acid can result in a transient
increase in noncarbonic acid production.
(2) Acetoacetic and ,B-hydroxybutyric acids are produced from the metabolism of triglycerides. These are the ketone bodies that are noncarbonic acids produced by normal
subjects during fasting. Upon eating, acetoacetic and ,B-hydroxybutyric acids are further
catabolized to CO 2 and H 2 0. Excessive production of ketone bodies contributes to the
noncarbonic acid "pool."
(3) Phosphoric acid, which is produced from the metabolism of phospholipids, provides
a major source of H + .
(4) Sulfuric acid is produced from the catabolism of proteins containing sulfhydryl groups
in their cysteine, cystine, and methionine residues.
(5) Uric acid is produced from the metabolism of nucleoproteins.

?E>7

b. Acetic acid (vinegar) functions transiently as a noncarbonic acid because humans can
convert it rapidly to CO 2 and H 20.

3. Stool HC0 3 - loss


a. The diet also contains organic cationic and anionic salts that may be metabolized to yield
noncarbonic acids and bases (HC0 3-). Organic anions that are metabolized in the body to
HC0 3- include acetate, citrate, and-in the presence of insulin-the anions of the ketoacids.
b. Digestive processes result in the loss of 20-40 mmol of alkali in the stool, and this loss is
equivalent to the addition of nonvolatile acid in the body.

C. Acid elimination. There are three processes available for maintaining H+ concentration within
normal limits:
1. Combination of H + with a chemical buffer such as HC0 3 -, protein, phosphate, or hemoglobin

2. Elimination of CO 2 by pulmonary ventilation


3. Excretion of H + by the kidneys

II. THE HYDROGEN ION AND pH


A. Fundamental chemistry (Table 5-1). H+ is a proton (Le., a hydrogen atom without its orbital
electron); H+ in aqueous solution exists as a hydrated proton called the hydronium ion, or H30+.
pH refers to the negative Briggsian logarithm of the H+ concentration. The gain and loss of protons
constitutes acid-base chemistry. The currently accepted model of acid-base relationships is that
proposed by Bronsted.

1. An acid is a substance that acts as a proton donor.


2. A base is a substance that accepts protons (Le., H+) in solution. Thus, bicarbonate ion (HC0 3-),
phosphate ion (HPO/-), ammonia (NH 3), and acetate ion (CH 3COO- ) all are bases.

3. Some substances are nearly equally divided between the acidic and basic forms at the normal
H+ concentration of the body. For example, the imidazole side groups of hemoglobin undergo
the following reaction:
HHb~

H+

+ Hb-

The acid, deoxyhemoglobin (HHb), dissociates to form H+ and the conjugate base, Hb-. HHb
and Hb- occur in about equal concentrations in blood cells.
B. Concept of pH and H+ concentration (Table 5-2)

1. H+ concentration is expressed in two different ways, either directly as [H+] or indirectly


as pH. (The symbol, [H+], refers to H+ concentration in mol/L or Eq/L.) The relationship
between [H+] and pH can be expressed as:
1
a. pH = log,o - [H+]
b. pH

c. [H+]

-Iog,o [H+]
=

10-pH

Table 5-1. Important Buffer Acids at Physiologic [H+]


Proton Donor
(Conjugate Acid)*
(C0 2 )H 2 C0 3
H 2 P0 4 H Protein
HHb02
HHb

Proton
(H+)
----->.
...-----...---------->.
...------

----->.

----->.
...--------'"
...------

H+
H+
H+
H+
H+

Proton Acceptor
(Conjugate Base)*

+
+

+
+

HC03HPO/ProteinateHb0 2 Hb-

A conjugate acid can be an anion or a cation; a conjugate base usually is an anion; He03 - , a conjugate base, also
can be an acid.

Table 5-2. Relationship between pH and [H+]


[H+]
(nEq/L)

pH

7.70
7.40 (plasma)
7.30 (CSF)
7.10 (lCF)
7.00
6.90
CsF

20
40
50
80
100
126

cerebrospinal fluid; ICF

intracellular fluid.

2. pH is a dimensionless number and should be treated as such; it should not be referred to in


concentration units or as "pH concentration." In fact, the quantity whose logarithm determines
the pH is a volume per equivalent, which is the inverse of concentration. Thus, pH could be
correctly conceptualized as a logarithmic expression of the volume required to contain 1
equivalent of H+. In human plasma at pH 7.4 [i.e., [H+] of 40 x 10- 9 mol (Eq)/L or 40 nmol
(nEq)/L], that volume is 25 million L!

3. Because pH is the logarithmic expression of [H+], it permits a graphic representation of a wide


range of [H+] values. (It is important to note that pH and [H+] are inversely related.) Another
advantage of the pH concept is that when the pK' of a buffer system is known, it is immediately
possible to determine the effective pH range of the buffer. *
4. A disadvantage of the pH system is that it both inverts and uses the logarithmic scale to
express [H+]. For example, it is not immediately apparent that a decrease in pH from 7.4 to
7.1 represents a doubling of the [H+] from 40 nmol/L to 80 nmol/L.

C. H+ concentration of body fluids (Figure 5-1; Table 5-3)

1. Blood and plasma. With regard to [H+], the body fluid compartment most studied is arterial
blood plasma. The term Blood pH always refers to plasma pH (7.4), which is higher than the
intracellular pH of the erythrocyte (7.2).
a. In normal individuals, the [H+] is approximately 40 nmol (nEq)/L, which is equivalent to
pH 7.4. The range of [H+] that is compatible with life is 20-126 nEq/L, which is equivalent
to a pH range of 7.7-6.9.

t/::]Slood

7.6

~ CSF

60

7.5

50

7.4

30

Cl

~ 7.3

E 40
E

30

7.2
7.1
0

O--J---L:...=........,."",",",'---

pH

Peo2

Figure 5-1. Comparison of the relationship between pH, Pco 2 , and [HC0 3 -] in arterial blood and cerebrospinal
fluid (CSF) in normal adult humans. Each vertical bar represents 1 sD. (Reprinted from Seldin DW, Giebisch G
(eds): Acid-base balance in specialized tissues: central nervous system. In The Regulation of Acid-Base Balance. New
York, Raven, 1989, p 109.)

OK = the ionization or dissociation constant; pK = the negative logarithm of K (- log K) and is equal to the pH at
which half of the acid molecules are dissociated and half are undissociated; pK' = the apparent pK (see II D 2 a).

Table 5-3. H+ Concentration ([H+]) and pH of Biologic Fluids


Fluid

pH

[H+]
(nEq/ or nmoI/L)*

[H+]
(Eq/L or mol/L)

10- 7

Pure water

7.0

100

Blood
Normal mean
Normal range
Acidosis (severe)
Alkalosis (severe)

7.40
7.36-7.44
6.9
7.7

40
44-36
126
20

3.98
4.36
1.26
2.00

CSF (normal range)

7.36-7.44

44-36

4.36 x 10- 8 -3.63 x 10- 8

Pure gastric juice (normal)

1.0

100,000,000

Urine
Normal average
Maximum acidity
Maximum alkalinity

6.0
4.5
8.0

1000
31,600
10

1 X 10- 6
3.16 x 10- 5
1 X 10- 8

ICF (muscle)

6.8

158

1.58 x 10- 7

x
x
x
x

10- 8
10- 8-3.63
10- 7
10- 8

10- 8

10- 1

CSF = cerebrospinal fluid; I.CF = intracellular fluid. (Adapted from Brobeck JR (ed): Regulation of hydrogen ion
concentration in body fluids. In Best and Taylor's Physiological Basis of Medical Practice, 10th edition. Baltimore,
Williams & Wilkins, 1979, p 5-13.)
*n = nano- = 10- 9 Thus, 100 nEq/L = 100 X 10- 9 Eq/L, where nEq = nmol of a monovalent ion.

b. In normal individuals at rest, the pH of mixed venous blood is 7.38 compared to 7.41 for
arterial blood because of the uptake of CO 2 by blood as it perfuses the tissues.
c. The [H+] of plasma is very small compared to that of other ions in plasma (e.g., the plasma
Na+ concentration is about 142 million nmol/L, and the K+ concentration is about 4 million
nmol/U.
2. Cerebrospinal fluid (CSF) is essentially a bicarbonate buffer with a negligible concentration
of protein (between 2 x 10- 2 g/dl and 4 x 10- 2 g/dl) or other non bicarbonate buffers.
a. The arterial pH (7.4) is higher than that of CSF (7.32), because the CO 2 tension (Pea 2 ) is
about 48 mm Hg in the CSF and about 40 mm Hg in the arterial blood (see Figure 5-1).
b. Generally, it is possible to characterize acid-base disturbances precisely in terms of the blood
data; however, the extracellular changes do not always reflect the intracellular changes. Also,
acid-base alterations in arterial blood may produce similar or opposite changes in the
CSF depending on whether the blood [H+] changes are due to respiratory or metabolic
abnormalities.
(1) An increase in arterial Peo l (respiratory acidosis) leads to a parallel rise in the CSF
Pea 2 , with a resulting increase in the [H+] of both arterial blood and CSF.
(2) A decrease in arterial Peo l (respiratory alkalosis) leads to a parallel decline in CSF
Pea 2 , with a resulting decrease in the [H+] of both arterial blood and CSF.
(3) An increase in arterial [H+] (metabolic acidosis) stimulates ventilation, lowering the
Pea 2 of the arterial blood and CSF, resulting in CSF alkalosis and blood acidosis.
(4) A decrease in arterial [H+] (metabolic alkalosis) decreases ventilation, raising the
Pea 2 of the arterial blood and CSF, resulting in CSF acidosis and blood alkalosis.

D. Henderson-Hasselbalch equation
1. Significance. Acid-base balance is maintained primarily through the control of two organ
systems. The lungs control the Pea 2 through the regulation of alveolar ventilation, and the
kidneys control the HC03 - concentration ([HC0 3 -]). The classic description of the acid-base
state is based on the Henderson-Hasselbalch equation, which is an expression of three
variables (pH, Pea 2 , and [HC0 3 -]) and two constants (pK' and S).

2. Definition of parameters. The Henderson-Hasselbalch equation could functionally be written


as
pH

kidneys
pK' + log - - lungs

(1)*

However, the equation is expressed more usefully as


pH

[HC0 3 -]
pK' + log - - 5 X Pco 2

(2)

or, using specific values for pK' and 5 (defined in II D 2 a and b), as
pH

6.1 + log

[HC0 3 -]

(3)

0.03 x Pco 2

From equation (3) it is clear that the value of arterial pH depends on the ratio of [HC0 3 -] to
5 x Pco 2 , not on the individual value of each variable. In clinical medicine, pH, Pco 2 , and
[HC0 3 -] can be measured directly. With equation (3), however, anyone of the variables can
be calculated if the other two are known.
a. pK is defined as the negative logarithm of the [H+] at which half the acid molecules are
undissociated and half are dissociated. When equimolar concentrations of weak acid and
conjugate base exist, the pH value equals the pK (i.e., the log of 1 is 0).
(1) The actual dissociation constant (K) for carbonic acid (H 2C0 3 ) in dilute aqueous
solution at 38 C is 1.6 X 10- 4 mol/L (pK = 3.8). Thus, H 2 C0 3 is almost completely
dissociated in the body where [H+] = 4 X 10- 8 mol/L, and it exists in quantities that
are too small to be analyzed (i.e., 2.4 x 10- 4 mEq/U. The formation and dissociation
of H 2 C0 3 is expressed as
carbonic
anhydrase
CO 2 ~ CO 2 + H 2 0
' H 2 C0 3 ~ H+ + HC03 alveolar
plasma
500:1
4000:1
gas

(4)

At equilibrium there are approximately 500 mmol of CO 2 for every 1 mmol of H 2 C0 3


and approximately 4000 mmol of H 2 C0 3 for every 1 mmol of H+. Because of the
presence of carbonic anhydrase, equilibrium between CO 2 and H 2 C0 3 is rapid and
constant. t
(2) Because the denominator of equation (3) is increased by a factor of 500, the apparent
dissociation constant (K') for the CO 2 /HC0 3 - buffer system in plasma at 38 C is
correspondingly smaller (8 x 10- 7 mol/L or 800 nmol/U, and the apparent pK (pK')
for this same buffer pair is correspondingly larger (6.1). As a rule, the optimal buffer
region of a buffer pair system is within a range of 1 pH units of its pK value.
(3) It is important to note that, because CO 2 increases [H+], as shown in equation (4), it is
considered an acid even though it is an acid anhydride. Thus, dissolved CO 2 is present
as a potential H+ donor, and its concentration is proportionate to the true donor,
H 2C0 3 For these reasons, it is more meaningful to characterize acid-base disturbances
in terms of the CO 2 /HC0 3 - buffer system instead of the H 2 CO)HC0 3 - system. For
all practical purposes, the H 2C0 3 /HC0 3 - buffer pair can be considered to be composed
of HC0 3 - (conjugate base) and dissolved CO 2 (conjugate "acid").
b. S is defined as the solubility constant for CO 2 in plasma at 38 C and is equal to 0.03
mmol/L/mm Hg. 5 represents the proportionality constant between CO 2 and Pco 2 For
blood plasma at 38 C, the amount of dissolved CO 2 is expressed as
dissolved CO 2

0.03

Pco 2

where CO 2 = the millimoles of dissolved CO 2 per liter of plasma. At an arterial Pco 2 of 40


mm Hg, the CO 2 concentration ([C0 2 ]) is expressed more usefully as

0.03 mmol/L/mm Hg x 40 mm Hg

1.2 mmol/L

Multiplying the proportionality constant by milliliters of CO 2 per millimole (22.3 ml/mmoll


"The kidneys primarily regulate [HeO, -], the numerator; the lungs mainly regulate Pco 2 , the denominator.
tCarbonic anhydrase is found in erythrocytes, gastric parietal cells, renal tubular cells, pancreatic and pulmonary
tissue, bone, and the eye; it is not found in muscle, peripheral nerves, or skin.

yields a solubility constant of 0.67 ml/L/mm Hg for CO 2 in plasma. At this solubility constant
and at a Peo 2 of 40 mm Hg, the concentration of dissolved CO 2 is expressed as

0.67 ml/l/mm Hg x 40 mm Hg

26.8 ml/l *

c. [HCO l - ] denotes the bicarbonate ion concentration, which is expressed in millimolarity


rather than molarity. The normal value of [HC0 3 -] in plasma is 24 mmol (mEq)/L.

3. Calculations with the Henderson-Hasselbalch equation


a. Traditional calculation of pH. pH in normal arterial plasma = 7.4; normal Peo 2 = 40 mm
Hg, and normal [HC0 3 - ] = 24 mmol/l (or 24 mEq/U. Applying the Henderson-Hasselbalch equation, pH is calculated as
[HC0 3 -]
pH = 6.1 + log
0.03 x Peo 2
24 mmol/l
= 6.1 + log
0.03 x 40 mm Hg
24 mmol/l
= 6.1 + log
1.2 mmol/l
= 6.1 + log 20
= 7.4
b. Simple calculation of [H+] and pH
(1) Conversion of pH to [H+]: a close approximation
(a) Equation. The [H+] in plasma is expressed in nmol/L. The equation for [H+] now
becomes
log [H+]- 9 = - pH or [H+] = antilog (9 - pH)
(b) Examples (see Table 5-2)
(i) At pH 7.4
[H+] = antilog (9 - 7.4)
= antilog 1.6
= 39.6 nmol/l
(ii) At pH 6.9
[H+] = antilog (9 - 6.9)
= antilog 2.1
= 126 nmol/l
(2) Conversion of [H+] to pH
(a) Equation. The [H+] in plasma is expressed in nmol/L. The equation for pH is
pH = 9 - log [H+]
(b) Examples (see Table 5-2)
(i) At [H+] = 40 nmol/l
pH = 9 -log 40
= 9 - 1.6
= 7.4
(ii) At [H+]

126 nmol/l
pH

=
=
=

9 - log 126
9 - 2.1
6.9

E. Henderson equation
1. Use of the Henderson equation. This equation provides a simple means for converting pH
to [H+], since Peo 2 and [HC0 3 -] will have been provided. Additionally, this equation provides
a simple arithmetic estimate of [HC0 3 -I, because the value of [H+] often will have been
approximated from the pH value reported by the laboratory.

This amounts to about 5% of the total amount of CO 2 carried in the arterial blood.

2. Calculations with the Henderson equation. The Henderson equation represents the nonlogarithmic (arithmetic) method of determining [H+] or [HC0 3 -] as
[H+]

K'

Pea
2

[HC0 3 -]

or [HCO -]

Pea
K' _ _2
[H+]

(5)

a. A value for K', is derived by converting the apparent pK for carbonic acid (6.1) into a
dissociation constant (expressed in nmol/U and multiplying that value by the solubility
constant for CO 2 in plasma at 38 C (i.e., 0.03). K' has a value of 800 nmol/L [see II D 2 a
(2)], which, when multiplied by 0.03, equals 24.
b. Using normal values for Pea 2 , [H +], and [HC0 3 -], the Henderson equation is applied as

[H +]

PC0 (mm Hg)

2
---=------=:....-

24

40
24 24

[HC0 3 - ] (mmol/U
=

40 nmol/L

Similarly,
Pea 2 (mm Hg)
[HC0 3 -]

24 [H+] (nmol/U

40
= 24 - = 24 mmol/L
40
Additionally,
Pea 2

[H+] X [HC0 3 -]
= "----:...----''----::........:

24
40 x 24
- - - = 40mm Hg
24

III.

BODY BUFFER SYSTEMS.

A buffer is a solution consisting of a weak acid and its conjugate base.


Buffering is the primary means by which large changes in [H+] are minimized.

A. Types of body buffer systems


1. Blood buffers (Table 5-4)
a. It is important to note that the blood buffers are not solely plasma buffers, but that hemoglobin, HC0 3 - , and phosphate are found in erythrocytes and act as important blood buffers.
(1) In blood, the chief H+ acceptor is HC03 - , which exists in a concentration of 20-30
mEq/L.
Table 5-4. Whole Blood Buffers
Buffer Type
Bicarbonate
Plasma
Erythrocyte
Total bicarbonate
Nonbicarbonate
Hemoglobin and oxyhemoglobin
Plasma proteins
Organic phosphate
I norganic phosphate
Total nonbicarbonate

Buffering Capacity of
Whole Blood (%)

35
18

53
35
7
3
2

47

Adapted from Brobeck JR (ed): Regulation of hydrogen ion concentration in body fluids. In Best and Taylor's
Physiological Basis of Medical Practice, 10th edition. Baltimore, Williams & Wilkins, 1979, p 5-14.

(2) H+ acceptor is available in the hemoglobin system, where reduced hemoglobin (deoxyhemoglobin) is a stronger base than oxyhemoglobin (Le., deoxyhemoglobin has a
stronger capacity to combine with H+).
(3) The hemoglobin buffer system is quantitatively as important as the bicarbonate buffer
system.
b. The major buffer anions of whole blood-HC0 3 - , protein, and hemoglobin-have a total
concentration of approximately 48 mEq{L [see VII B 1 a (2)].
c. With the bicarbonate and hemoglobin systems taken together, 5 L of blood of a normal
adult have sufficient buffer capacity to combine with almost 150 mmol of protons (150 ml
of 1 N HC!) before the pH of body fluids becomes dangerously acidic.
2. Tissue buffers. The major buffer capacity of the body is not in the blood but in the H +
acceptors found in other tissues, principally in the muscle and in bone. These tissues can
neutralize about five times as much acid as the blood buffers.
a. Muscle
(1) Since skeletal muscle represents about half of the cellular mass, most intracellular
buffering presumably occurs in muscle.
(2) For any individual, the body He0 3 - concentration averages 13 mEq{kg body weight.
Muscle cells contain HC03 - at a concentration of about 12 mEq{L, and most other
cells contain it at higher concentrations. The intracellular fluid (lCF) and extracellular
fluid (ECF) compartments each contain about 50% of the total body HC0 3 - .
b. Bone plays an important role in buffering H+.
(1) The total body store of bone carbonate is about 50 times the amount of HC0 3 - in the
ICF and ECF compartments together. Bone carbonate appears to be the predominant
source of base for neutralizing excess noncarbonic acid in the ECF. Indeed, it has long
been recognized that chronic noncarbonic acidosis causes bone dissolution (resorption)
through the loss of calcium carbonate (CaC0 3 ). The early carbonate release is in the
form of sodium carbonate (Na 2 C0 3 ).
(2) Bone contains about 80% of the total CO 2 (including CO/- , HC0 3 - , and CO 2 ) in the
body. About two-thirds ofthis CO 2 is in the form of CO/- complexed with Ca2+, Na+,
and other cations located in the lattice of the bone crystals. The other third consists of
HC03 - and is located in the hydration shell of the hydroxyapatite crystal, an inorganic
compound found in the matrix of bone and teeth.
B. Distribution of body buffer systems in major fluid compartments

1. Blood, in regard to its buffering activity, usually is considered as a whole rather than in terms
of its components. Blood buffers are described here in terms of separate compartments (i.e.,
plasma and erythrocytes) for didactic purposes only.
a. Important concepts
(1) Whole blood is an excellent buffering system for noncarbonic acids because of its
nonbicarbonate buffers as well as its bicarbonate buffer system.
(2) More than 90% of the blood's capacity to buffer carbonic acid is attributed to the
hemoglobin buffer system. Thus, the nonbicarbonate buffer in the erythrocyte is quantitatively more important than the bicarbonate buffer in that compartment. The bicarbonate buffer system remains quantitatively important, however, in the erythrocyte (see
Table 5-4).
(a) The bicarbonate buffer system does not function as a buffer for carbonic acid.
(b) The non bicarbonate buffer systems can buffer both noncarbonic and carbonic acids.
b. Buffer capacity of blood components
(1) Plasma, which contains three buffer systems, has a considerable capacity for buffering
noncarbonic acids but a much smaller capacity for buffering carbonic acid.
(a) Bicarbonate buffer system (HC0 3 -{H 2 C03 or HC03 -{C02 ), This buffer exists in
a concentration of 24 mmol{L of plasma. When noncarbonic acid is added to
normal plasma, more than 75% of the buffering capacity of plasma is due to the
HC0 3 -{C0 2 system. Most of the remaining buffering of noncarbonic acid involves
the plasma protein buffers and, to a small degree, the phosphate buffer system.
Again, the HC0 3 -{C0 2 system plays no role in the buffering of carbonic acid.
(b) Nonbicarbonate buffer systems
(i) Plasma protein (Protein-{H Protein). Plasma is a salt solution containing 7%
protein, which exists as polyanions at the pH of plasma. Plasma protein H+

acceptors exist in a concentration of 1.5 mmol/L of plasma and account for less
than one-sixth of the total buffering capacity of whole blood.
(ii) Inorganic orthophosphate (H PO/- / H 2 P0 4 - ). Because this buffer system exists in a concentration of only 0.66 mmol/L of plasma, it contributes little to the
total buffering activity of plasma. * At a plasma pH of 7.4, the concentration ratio
of HPO/- /H 2 P0 4 - is 4:1. Therefore, 80% of the inorganic phosphate exists as
disodium phosphate, and 20% is in the form of monosodium phosphate. The
HPO/- /H 2 P04 - system is a major elimination route for H+ via the urine, which
has a relatively high phosphate content.
(2) Erythrocytes. Although the blood contains other cells, the erythrocyte is the only
important cellular component of the blood buffer system. The erythrocyte contains four
buffer systems.
(a) Bicarbonate buffer system. The HC03 - /C0 2 system exists in a concentration of
15 mmol/L of erythrocytes, compared to a concentration of 21 mmol/L and 25
mmol/L of whole blood and plasma, respectively.

(b) Nonbicarbonate buffer systems


(i) Hemoglobin buffers (Hb- /HHb and Hb02 - /HHb0 2 ) [see III C]. One L of
erythrocytes contains 334 g (5.1 mmol) of hemoglobin. One L of whole blood
contains 150 g (2.3 mmol) of hemoglobin.
(ii) Organic phosphate. Although the erythrocyte contains a significant amount of
organic phosphate buffer, this amount is quantitatively small compared to the
bicarbonate and hemoglobin buffer concentrations in the erythrocyte.
(iii) Inorganic orthophosphate. The HPO/- /H 2 P0 4 - system exists in a concentration of 2 mmol/L of erythrocytes.

2. Interstitial fluid (including lymph)


a. Bicarbonate buffer system. The HC0 3 - /C0 2 system is quantitatively the most important
buffer of noncarbonic acid. The [HC0 3 -] of the interstitial fluid is 27 mmol/L, which is
similar to, or about 5% higher than, the [HC0 3 -] of plasma. It is important to consider that,
in humans, the interstitial fluid volume is about three times that of plasma; therefore, the
total capacity of the interstitial fluid to buffer noncarbonic acid is considerably greater than
that of the total blood volume to buffer these acids. On a per unit of volume basis,
however, the interstitial fluid has nearly the capacity of plasma to buffer noncarbonic

acids.
b. Nonbicarbonate buffer system. The HPO/- /H 2 P04 - system exists in a concentration of
0.7 mmol/L of interstitial fluid; therefore, this compartment has little capacity to buffer
carbonic acid. The interstitial fluid is essentially free of protein.
3. ICF (excluding erythrocytes)

a. Bicarbonate buffer system. The ICF contains only about 12 mmol of HC03 - /L in skeletal
and cardiac muscle.

b. Nonbicarbonate buffer systems. Protein and organic phosphate compounds exist in quantitatively significant amounts in the ICF, giving this compartment the capacity to effectively
buffer both noncarbonic and carbonic acids as well as alkali. The ICF concentrations of
these major buffer anions are:
(1) HPO/- /H 2 P0 4 - (skeletal muscle) = 6 mmol/L
(2) Protein- /H Protein (skeletal muscle) = 6 mmol/L
(3) Organic anions (skeletal muscle) = 84 mmol/L

C. Hemoglobin: an "extracellular" buffer


1. Important concepts. Although hemoglobin is found intracellularly, it is more conventionally
regarded as extracellular and, therefore, part of the extracellular buffer system because:
a. Hemoglobin is confined to the erythrocyte, which is a cellular component of the ECF.
b. Hemoglobin is readily available for the buffering of extracellular acids.
c. Hemoglobin is the primary nonbicarbonate buffer of the blood.

*The pK' of the acid form (i.e., H 2 P0 4 -) is 6.B. With this pK', the HPO/- IH 2 PO. - system would be a more effective
buffer than the HC03 - IC0 2 system (pK' = 6.1) if it were present in an appreciable concentration. Many of the
organic phosphate compounds found in the body have pK' values within half of a pH unit from 7.0.

2. Hemoglobin as a buffer. Like all proteins, hemoglobin is a buffer. At pH 7.2 (i.e., the pH of
normal arterial erythrocytes), the buffering action of hemoglobin is due mainly to the imidazole
groups of the histidine residues.
a. The titration curves of deoxyhemoglobin and oxyhemoglobin in Figure 5-2 illustrate the
basis for the ability of hemoglobin to neutralize H+ formed subsequent to the diffusion of
CO 2 into the erythrocyte. Most of the H + is buffered by hemoglobin and most of the HC03 diffuses into the plasma. *
b. Oxyhemoglobin dissociates more completely than does deoxyhemoglobin, and, as a result,
deoxyhemoglobin produces less H+ at a given pH than does oxyhemoglobin, which is a
stronger acid. Thus, hemoglobin becomes a more effective buffer when CO 2 and, hence,
H + are added from the tissues. This is important because the diffusion of CO 2 from the tissues
to the capillary blood is accompanied by the simultaneous reduction of oxyhemoglobin.
c. As the uptake of CO 2 depends on H+ acceptors, this increase in H+ acceptors in the form
of deoxyhemoglobin facilitates the uptake and buffering of the H+ generated by the hydration of CO 2 and the dissociation of H 2 C0 3 As a result of these reactions, CO 2 is converted
into HC0 3 - within the erythrocyte.
(1) For each mmol of oxyhemoglobin that is reduced, about 0.7 mmol of H+ can be taken
up and, consequently, 0.7 mmol of CO 2 can enter the blood without a change in pH
(see Figure 5-2, point A to point C).
(2) A reaction that causes no change in [H+] (or pH) is called isohydric buffering.
d. Respiratory quotient (R.Q.). When the body is at rest, the rate of O 2 consumption (V0 2 )
under standard conditions is 250-350 ml/min, and the rate of CO 2 production (Ve0 2 ) is
200-250 ml/min. (The dot over the symbol V denotes volume per unit time.) R.Q.
represents the ratio of Ve0 2 to V0 2 Normally, on a mixed diet, the V0 2 exceeds the Ve0 2 ,
and the R.Q. is less than 1.
(1) The metabolic R.Q. equals the molar ratio of CO 2 production rate to the corresponding
O 2 consumption rate by metabolizing tissues.
(2) If the R.Q. is 0.7, then, for 1 mmol of O 2 consumed, 0.7 mmol of CO 2 is produced,
which, when converted to H 2 C0 3 , yields 0.7 mmol of H+ upon dissociation.
(3) The complete deoxygenation of 1 mmol of oxyhemoglobin to liberate 1 mmol of O 2
results in the neutralization of 0.7 mmol of H+ without a change in pH. Thus, all of the
H+ produced when the R.Q. is 0.7 can be buffered by deoxyhemoglobin with no
change in pH (see Figure 5-2).

50

[H + I (nmoI/L)
40
30

7.3

74

20

+1.0
mmol H+
added to 1
mmol Hb0 2 - or
Hb-

+0.5

0
mmol of H+
removed from 1
mmol Hb0 2 or
Hb

-0.5

7.5
pH

7.6

77

Figure 5-2. Titration curves of oxyhemoglobin (Hb0 2 -) and deoxyhemoglobin (Hb-), illustrating the importance of hemoglobin as a buffer.
The complete deoxygenation of 1
mmol of Hb0 2 - to liberate 1 mmol
of O 2 results in the neutralization of
0.7 mmol of H+ without a change in
pH. Arrow AC indicates the amount
of H+ that can be added during the
reduction of hemoglobin without
causing a pH change. Arrow AB represents the pH change that would occur if the oxyhemoglobin at pH 7.4
was completely reduced. The reduction of Hb0 2 - to Hb- would cause
a large increase in pH if CO 2 and,
hence, H+ were not added simultaneously to the system. Reduction of
hemoglobin denotes the O,-free state
without a change in the valence of
iron. (Adapted from White A, et al:
Hemoglobin and the chemistry of respiration. In Principles of Biochemistry, 5th edition. New York, McGrawHill, 1973, P 843.)

'The buffer capacity of non bicarbonate buffers is dependent primarily on the hemoglobin concentration.

e. At pH 7.2, about 84% of the deoxyhemoglobin is in the form of HHb, whereas only about
23% of the oxyhemoglobin is in the form of HHb0 2. Of the total oxyhemoglobin that
causes O 2 to form deoxyhemoglobin:
(1) 23% was already combined with H+,
(2) 16% will not combine with H+, and
(3) 61% will take up H+ before a pH decrease occurs.
f. CO 2 entering the erythrocytes rapidly undergoes two reactions.
(1) CO 2 is hydrated to form H 2C0 3 , a reaction that is catalyzed by carbonic anhydrase in
the erythrocyte. The H 2C0 3 dissociates to form HC0 3 - and H +, which is buffered
primarily by the hemoglobin buffers. Much of the HC0 3 - formed within the erythrocyte
diffuses into the plasma in exchange for CI-.
(2) CO 2 combines with the amino groups of deoxyhemoglobin to form carbaminohemoglobin. The carbaminohemoglobin dissociates to a carboxylate anion and H+, which is
buffered primarily by the hemoglobin buffers. In summary:
Hb NH2 + CO 2 ~ Hb NHCOO- + H+
g. The unloading of O 2 from oxyhemoglobin to the tissues causes the formation of deoxyhemoglobin that is better able to tie up the H+ produced by the simultaneous uptake of CO 2, The
loss of O 2 from hemoglobin facilitates the uptake of CO 2 in the form of carbaminohemoglobin by the erythrocytes. Oxyhemoglobin contains about 0.1 mmol and deoxyhemoglobin
about 0.3 mmol of carbaminohemoglobin per mmol.

IV. GENERATION AND ELIMINATION OF H+. The greatest source of H+ is the CO 2 produced
as one of the end products of the oxidation of glucose and fatty acids during aerobic metabolism.
A. Carbonic acid (H 2 C0 3 ) is an acid that forms a volatile end product upon dehydration and can,
therefore, be excreted by the lungs.

1. Hydration-dehydration: dissociation-association reaction


a. CO 2 can form H+ according to the following series of reactions:
hydration
dissociation
CO 2 + H 20
<
H 2C0 3
dehydration
association

H+ + HC0 3 -

HC0 3 - is formed through the hydration of CO 2 and the subsequent dissociation of H 2C0 3
A significant concentration of HC0 3 - cannot be formed by this reaction because the
dissociation of H 2C0 3 forms equal numbers of H+ ions and HC0 3 - ions. Therefore, the
significant concentration of HC0 3 - can only be attained after the H+ formed from H 2C0 3
is buffered by blood buffer anions (Le., reduced hemoglobin, protein, and phosphate) as:
H+ + HC0 3 - + Na+ + buffer

H buffer + Na+ + HC0 3 -

b. Thus, for every H 2C0 3 molecule that has its H+ taken up by a buffer, one HC0 3 - appears
in the blood. These HC03 - ions are distributed between the erythrocytes and the plasma.

2. CO 2 production (Table 5-5). CO 2 is the chief product of metabolism and represents the greatest
portion of the acid that is continuously eliminated from the body by the lungs.
a. Most CO 2 in the body is produced from the decarboxylation reactions of the tricarboxylic
(citric) acid cycle.
b. HC03 - is an excellent buffer with respect to noncarbonic acid added to the blood by diet,
metabolism, and disease, but the HC03 - buffer system plays no role in the buffering of
carbonic acid.
Table 5-5. CO 2 Production under Basal Conditions*
Molarity

Volume
Time Elapsed
1 minute
1 hour
1 day
*Data for a respiratory quotient of 1.0.

(L)

0.2
12.0
300

(mmol)

540
13,500

(mol)

10- 3

0.54
13.5

c. Comparing CO 2 and HCO l - , there is more CO 2 than HCOl - produced metabolically in


the body. However, the ECF contains a preponderance of the HCOl - form.
d. Under basal conditions, with a R.Q. of 0.82, the average adult produces about 300 l (13
mol of CO 2 per day.
e. The lungs and kidneys are the principal routes for the elimination of protons and for maintaining normal [HCO l -]IS x Peo 2 ratios.
(1) In the course of a day, the equivalent of 20-40 l of 1 N acid (20,000-40,000 mEq
H+)t are eliminated via the lungs, or, more correctly, the amount of CO 2 produced
during a day in a normal individual is potentially capable of forming 20-40 Eq of H+.
(2) During a 24-hour period, the equivalent of 50-150 ml of 1 N acid (50-150 mEq H+)
is excreted via the kidneys.
f. CO 2 must also be removed from the blood by the lungs at a rate of 200 ml/min during
resting conditions. t With a resting cardiac output of 5 l/min, 40 ml of CO 2 must be added
to each liter of blood per minute (Le., 5 l/min x 40 mill = 200 ml/min).

3. Buffering of H 2 C0 3 is primarily by the intracellular buffers (Le., proteins, organic and inorganic
phosphates, and hemoglobin). Most of the buffering of H 2 CO l occurs in the erythrocyte, where
the hemoglobin buffer system is quantitatively the most important buffer.
B. Noncarbonic acids are acids that do not form a volatile end product and, therefore, must be
excreted by the kidneys (Table 5-6).

1. Source
a. The primary source of noncarbonic acid in humans is the metabolism of exogenous
protein. The body produces 30 mEq of H+ daily from the catabolism of proteins such as
phosphoprotein and methionine, which are metabolically equivalent to phosphoric and
sulfuric acids, respectively.
b. Other noncarbonic acids are the products of intermediary metabolism.
(1) Lactic acid, a product of anaerobic intermediary metabolism, accumulates in the ECF
in large quantities during heavy exercise. This excess acid must be buffered until excreted
or metabolized to CO 2 and water.
(a) Although most organs generate lactic acid, the largest amount of lactic acid is
produced by skeletal muscle, erythrocytes, and the skin. (Liver, kidney, and muscle
can convert lactic acid to HCO l - .)
(b) Tissue hypoxia leads to hyperlacticemia, which reduces the plasma [HC0 3 -] and
increases the anion gap (see VII B 2 for discussion of anion gap). The increment in
the anion gap in this situation is a measure of serum lactate. Clinical conditions
associated with tissue hypoxia include cardiac arrest, shock, severe cardiac failure,
and severe hypoxemia.
(c) Alkalosis stimulates glycolysis and generates lactic acid mainly by activation of
phosphofructokinase. Respiratory alkalosis is a more effective stimulus for glycolysis,
because CO 2 crosses cellular membranes more readily.
(2) Acetoacetic acid and ~-hydroxybutyric acid also are products of intermediary metabolism. These acids are formed almost exclusively by the liver during fasting.
Table 5-6. Sources of Noncarbonic (Nonvolatile) Acids in Humans
Source

Amount
(mEq/day)

Noncarbonic Acid

Diet

30

Phosphoproteins, sulfur-containing
amino acids, chloride salts

Intermediary
metabolism

30

Ketoacids, lactic acid

Stool loss

30

loss of HCOl

*1 mol of CO 2 = 22.26 L at STPD (standard conditions of temperature and pressure, dry).


t*Depending on the level of physical activity; during resting metabolism, a normal individual is potentially capable
of forming 13,000 mEq of H+.
lEqual to CO 2 production at rest.

c. Excess fluid intake (hydration) reduces the [HC0 3 -), which causes a corresponding increase in [H+) if Peo 2 is constant. This type of noncarbonic acidosis is corrected more
rapidly by the renal excretion of excess water than by renal secretion of the apparent
excess of H +. Dehydration has the opposite effect.

2. Excretion
a. lactic acid production usually is transient; that is, after exercise is completed or the hypoxemic condition is removed, the lactic acid is metabolized to CO 2 and water. Lactic acid,
therefore, can be eliminated by metabolic destruction of the acid rather than by renal
excretion under normal conditions.
b. Phosphoric and sulfuric acids cannot be further metabolized to CO 2, and the elimination
of these acids and the H+ that they yield can only be accomplished by the kidneys.
c. Even noncarbonic acids that can be catabolized to CO 2 and water are excreted by the
kidneys when present in large excess, as are ,a-hydroxybutyric and acetoacetic acids in
diabetes mellitus. This condition is called ketonuria. Clinically more important is the acidosis
of uncontrolled diabetes mellitus, where the accumulation of acetoacetic and ,a-hydroxybutyric acids in the ECF may produce coma and death.
3. Buffering of noncarbonic acids. In contrast to the bicarbonate buffer system, which can
buffer only noncarbonic acids, the nonbicarbonate buffer systems can buffer both noncarbonic
and carbonic acids.
a. The plasma bicarbonate system is quantitatively the most important buffer in the ECF for
noncarbonic acids.
b. When noncarbonic acids are being buffered in the erythrocytes, more than 60% of the
buffering occurs by the hemoglobin and more than 30% by the bicarbonate system in the
erythrocytes. Approximately 10% of the buffer capacity in erythrocytes is attributed to the
organic phosphate esters.
c. The bodily response to mineral acids (e.g., HO) is different in that lactic acid is distributed
throughout the body water and is metabolized, whereas mineral acids are confined to the
ECF and are buffered and excreted without being metabolized.

v.

RESPIRATORY REGULATION OF ACID-BASE BALANCE. It is not immediately apparent


that the actual quantities of CO 2 and O 2 transported in the blood are far greater than the amounts of
these gases in physical solution, because these gases are transported mainly in the form of chemical
derivatives. Further, it is even less apparent that there is far more total CO2 than O 2 in every liter of
blood. At the level of the lung, the HC0 3 - combines with the potential protons (i.e., the protons
associated with HHb0 2) to produce Hb02- and H 2C0 3 resulting in the elimination of the protons as
CO 2 and water. * Thus, free H+ never appear to any appreciable extent but are in the form of HHb
or H 2C0 3 . The entire respiratory cycle can be regarded as an exchange of a HC03 - for a
Hb0 2 - , with the HC0 3 - transported from the tissues to the lungs and the Hb02- transported to the
tissues from the lungs.
A. Blood forms of CO2
1. CO 2 is transported in three forms. However, H 2C0 3 ultimately must be converted to CO 2
in order to be eliminated via the lungs.
a. HC03 - accounts for 90% of the total CO 2 in plasma.
b. Carbamino compounds (i.e., carbamates of hemoglobin and protein) represent the combination of CO 2 with free NH2 groups of blood proteins according to the equation
R . NH2 + CO 2 R NHCOOAbout 5% of the CO2 normally carried in arterial blood
compounds (carbamates).
c. A small amount of CO 2 gas (5%) is physically dissolved in
H 2C0 3 Also, approximately 500 mol of CO 2 exist for every
(1 ).

+ H+
is in the form of carbamino
plasma and also hydrated as

1 mol of H 2C0 3 [see II D 2 a

*HHb0 2 and Hb0 2 - are forms of oxygenated hemoglobin and constitute a buffer pair where the protonated form,
HHb0 2 , is the conjugate acid and the unprotonated form, Hb0 2 - , is the conjugate base.

2. It is important to recognize that the term "bicarbonate" is used interchangeably with the term
"total CO 2 combining power" or "total CO 2,'' This must never be confused with the term
"partial pressure of CO 2,'' which is the Peo 2 of the arterial blood gas measurement.
3. In blood with an arterial P0 2 of 100 mm Hg and a mixed venous P0 2 of 40 mm Hg, there are
200 ml 02/L and 150 ml 02/L of arterial and mixed venous blood, respectively. However, the
arterial Peo 2 and mixed venous Peo 2 are associated with 480 ml CO 2/L and 520 ml CO 2/L of
arterial and mixed venous blood, respectively.

B. H 2 C0 3 and other CO 2 -forming acids

1. Source
a. The oxidation of glucose and triglyceride leads to the formation of CO 2, much of which is
hydrated to H 2C0 3 H 2 C0 3 in turn dissociates into H+ and HC0 3 - .
b. CO 2 and water are the most abundant end products of metabolism.
(1) Lactic acid, a noncarbonic acid, normally is metabolized to CO 2 and water.
(2) The ketone bodies, ,8-hydroxybutyric acid and acetoacetic acid, also are noncarbonic
acids, which are produced primarily by the liver during fasting. Upon reingestion,
these acids that have accumulated are further catabolized to CO 2 and water by the
extrahepatic tissues.

2. Elimination
a. Respiration accounts for the greatest portion of acid eliminated continuously from the body.
b. The lungs eliminate H 2C0 3 in the dehydrated (anhydrous) form of CO 2, Indeed, the unique
property of the CO 2 /HC0 3 - buffer system lies in the ability of the lungs to eliminate
undissociated H 2C0 3 as non ionizable CO 2 ,
3. Buffering of H 2 C0 3 Because CO 2 readily penetrates cellular membranes, H 2C0 3 is buffered
by the entire body.
a. Most of the buffering of H 2C0 3 occurs via the nonbicarbonate buffer systems within erythrocytes. The H+ derived from the dissociation of H 2 C0 3 is buffered primarily by the hemoglobin buffer system.
b. The erythrocyte bicarbonate buffer and the plasma bicarbonate buffer play no role in the
buffering of H 2 C0 3

VI. RENAL REGULATION OF ACID-BASE BALANCE


A. Overview
1. Normal acid-base conditions met by the kidneys
a. The kidneys are responsible for ridding the body of metabolically produced noncarbonic
acids.
(1) Among these are sulfuric and phosphoric acids and smaller amounts of hydrochloric,
lactic, uric, ,8-hydroxybutyric, and acetoacetic acids. The most abundant of the weak
acid waste products of metabolism is acid phosphate (H 2 P0 4 -).
(2) About half of these metabolically produced acids are neutralized by base in the diet.
The other half must be neutralized by buffer anion systems of the body. Of those
noncarbonic acids buffered in the ECF, 97%-98% are buffered by reacting with HC0 3 - .
b. Type of diet is a major determinant of the daily acid-base conditions that must be regulated
by the kidneys.
(1) High-protein diets contain large amounts of sulfur in the form of sulfhydryl groups. The
sulfur is oxidized to sulfate ion (50/-), a process that tends to lead to metabolic
acidosis.
(2) Vegetarian diets are associated with large intakes of lactate and acetate, and the metabolites of these anions tend to lead to metabolic alkalosis.

2. Kidney function
a. The primary role of the kidneys in acid-base regulation is to conserve major cations and
anions in the body fluids. To maintain the total quantities and concentrations of the major
electrolytes within normal limits, the kidneys perform two major functions.
(1) The kidneys stabilize the standard He03 - pool by obligatory reabsorption (mainly by
the proximal tubule) and by controlled reabsorption of filtered HC03 - (by the distal
and collecting tubules).

(2) The kidneys excrete a daily load of 50-1 00 mEq of metabolically produced noncarbonic
acid.' This represents a H+ excretion of 1 mEq/kg of body weight/day.
(a) In most cases, 25% of this noncarbonic acid (10-30 mEq/day) is excreted in the
form of titratable acid (see VI D).
(b) About 75% (30-50 mEq/day) is excreted in the form of acid combined with
ammonium (i.e., NH4 +; see VI E).
(c) The normal urinary ratio of NH4 + to titratable acid is between 1 and 2.5.
b. The major sites of urine acidification are the distal and collecting tubules.
(1) Essentially all of the H+ within the tubular lumen is from the tubular secretion of H+
generated by metabolism. There is no significant contribution of H+ from the glomerular
filtrate, which accounts for less than 0.1 mmol of H+ per day.
(2) Since the lowest pH attainable in urine is 4.4 (i.e., a [H+] of 40 x 10- 6 Eq/U and the
plasma [H+] is 40 X 10- 9 Eq/L, the kidney can cause a 1000-fold [H+] gradient
between plasma and urine.
c. An important difference between the acidification of urine and free H+ excretion is that the
ability to reduce urinary pH (acidification) does not reveal a great deal about the amount of
free H+ excreted. This is because most of the H+ that is excreted occurs in association with
an anion (mostly as H 2 P0 4-) or in combination with ammonia (as NH4 +).
d. The majority of secreted H+ is used to bring about HC03 - reabsorption and, therefore, is
not excreted.
B. Basic ion exchange mechanisms (Figures 5-3; 5-4)

1. Passive transport of Na+. Entry of Na+ into the luminal (adluminai) membrane of the tubular
cell is operationally linked to the secretion of H+ into the tubular lumen. (It is a cation-exchange
process that maintains intracellular electroneutrality.)
a. Although the movement of Na + across the luminal membrane is favored by the electrochemical gradient, the transport is mediated by specific membrane transport proteins (i.e., it is
carrier-mediated) and is not by simple diffusion.
C.A. = carbonic anhydrase
- _ = passive process
- . = active process
Tubular lumen
Na+

Peritubular capillary

Proximal tubular cell

Na+

()

--- ........ -

--'- ... -

---.... HC0 3 -

+
H2 0 -- ----- -----.

Na+ HC0 3 -

Na+ HC03 -

Figure 5-3. Mechanism of obligatory HC0 3 - reabsorption in the proximal tubule. The Na+ is reabsorbed from the
tubular lumen by a passive process and from the cell into the blood by an active process. Similarly, K+ is reabsorbed
as KHC0 3 or KCI. In contrast to Na+, the transluminal reabsorption of K+ is active, while transcellular K+ transport
is passive.

'In normal individuals in Western countries, a net of 40-60 mEq of noncarbonic acid is excreted daily.

CA = carbonic anhydrase
-_ = passive process
----+
active process

Distal and collecting


tubular cells

Tubular lumen
Na+

,,

HC03-

,f

Na + .--- -------. Na +

--- --_

K+

~+,

1
H C0
2

HC03-

------.

tCA
CO 2

H2 0 ----

------.

H2 O

K+ CI-

+
Na+ CI-

---- --.. HC03-

H2C0 3

3
----

cO 2

K+ HC03-

Na+

\.
HC03".
K'
'.
~
-----.
Na+
.---------- - --. Na+
HCO:?

HC0 3-

--

Blood

Na+

HC03

Na+ CI-

Figure 5-4. Mechanism of controlled HC03 - reabsorption in the distal and collecting tubules. Unlike the proximal
tubule, the distal and collecting tubules lack luminal carbonic anhydrase. The exchange of K+ for Na+ leads to the
reabsorption of 1 mol of NaCI for every 1 mol of KHC0 3 excreted. Note that intracellular carbonic anhydrase is
found throughout the nephron, including the distal and collecting tubules.
b. Secretion of H+ is active in that it occurs against an electrochemical gradient.
c. The process of H+ secretion and HCO, - reabsorption occurs throughout the nephron, with
the exception of the descending limb of the loop of Henle.
d. The Na+ -H+ antiporter is the major mechanism for H+ secretion in the proximal tubule
and thick ascending limb of the loop of Henle. This anti porter system is termed "secondary
active" transport because the energy is derived from the Na + concentration gradient and
not from the hydrolysis of adenosine triphosphate (ATPl.
e. The Na+-H+ antiporter exchanges one Na+ ion for one H+ ion and, therefore, is electroneutral.
2. Active transport of Na +. Subsequent to the passive Na + transport across the luminal membrane, Na+ is actively reabsorbed across the basolateral (abluminal) membrane into the peritubular capillary in association with HCO, - that was formed within the cell. The secretion of 1
mol of H+ leads to the reabsorption of 1 mol of Na+ and 1 mol of HCO, - in the peritubular
blood.
a. H+ for secretion originates from the hydration-dissociation reaction within the tubular cell
[see II D 2, equation (4)] and from the splitting of water, which liberates hydroxyl ion (OH-)
within the cell; the OH- then reacts with CO 2 to form HCO, -.
b. H+ secretion exceeds H+ excretion; the amount of H+ excreted in the form of free H+ is
negligible.
c. The net effect of H+ secretion is to facilitate the transfer of HCO, - from tubular lumen to
tubular cell.
d. It is probable that H+ secretion along the entire tubule is mediated by an active transport
process.

C. Reabsorption of HC03 - (see Figures 5-3; 5-4). The filtered HCO, - is not directly reabsorbed by
the renal tubules; instead, it must first be converted to H 2 CO l , with the H+ secreted into the
tubular lumen in exchange for the Na+ that is transported from the lumen. The HC03 - is reabsorbed indirectly by conversion to CO 2 within the tubular lumen, and most of the H + is reabsorbed
following its conversion to water within the lumen.

1. Proximal reabsorption. Approximately 90% of the filtered HCO l - is reabsorbed into the
proximal tubules as a result of the Na+ -H+ exchange and represents more than 4000
mEq/day.
a. The presence of carbonic anhydrase on the microvilli of the luminal border (brush border)
of the proximal tubules catalyzes the rapid dehydration of H 2 CO l to form CO 2 and water.
b. CO 2 diffuses back into the proximal tubular cell, where it is rehydrated by intracellular
carbonic anhydrase into H 2 CO l The HCOl - formed by the dissociation of H 2 CO l is
passively reabsorbed into the peritubular blood along with equimolar amounts of Na +,
which is actively transported into the peritubular blood. The H+ formed by the dissociation
of H 2 CO l serves as a source for another H+ to be secreted.
c. The entry ofNa+ into the tubular cell is balanced electrochemically by two mechanisms.
(1) CI-, which is the only quantitatively important reabsorbable anion in the filtrate, is
transported from lumen to cell by passive diffusion.
(2) In the presence of carbonic anhydrase and with OH - as its substrate, HCO l - is regenerated from the cellular CO 2 and water.
(3) The net result of either the CI- diffusion into the cell or the regeneration of HCO l is a net reabsorption of sodium chloride (NaCi) or sodium bicarbonate (NaHCO),
respectively, into the peritubular capillaries. It is important to note that the Na + delivered
to the peritubular capillaries comes from the filtrate in the lumen, but the HCOl transported into the capillaries is synthesized within the proximal tubular cell.
d. Any secreted H+ that combines with He0 3 - in the lumen to bring about He03 reabsorption DOES NOT contribute to the urinary excretion of acid. Thus, the majority
of the secreted H+ is used to accomplish HCO l - reabsorption.
e. Referring to this process as HCOl - reabsorption, then, seems inaccurate, since the HCO l that appears in the peritubular capillaries is not the same HCOl - that was filtered. Moreover,
most of the H+ that was secreted into the lumen is not excreted in the urine but is incorporated into water and reabsorbed.
f. There is no absolute Tm for HCOl - since the reabsorptive capacity for HCO l - varies
directly with the fractional reabsorption of Na +.
2. Reabsorption from the distal and collecting tubules
a. The remaining 10%-15% of the filtered HCOl - is reabsorbed by the distal and collecting
tubules via a mechanism that involves the exchange of Na + for K+ or H +. As in the proximal
tubules, every H+ secreted into the distal tubular lumen leaves a HCO l - within the tubular
cell.
b. Except for the Na+ -H+ anti porter or Na+ -K+ exchange pump at the luminal border of the
distal cellular membranes and the absence of carbonic anhydrase on the distal luminal
border, the reabsorption pathways for HCOl - are analogous to those described for the
proximal tubule. *
3. Addition of new HeOl - . In addition to the renal conservation of HCO l - , the kidneys add
newly synthesized HCOl - to the plasma so that the quantity of HCO l - in the renal vein
exceeds the amount that entered the kidneys.
a. The addition of new HCOl - to the plasma does not involve the HCOl - reabsorbed into
the tubule but the HCO l - generated within the tubular cell via the hydration of CO 2 and
dissociation of H 2 CO l This process is similar to the scheme for the reabsorption of filtered
HCOl - ; however, the HeOl - that is generated within the tubular cell does not represent filtered HeO l - .
b. The renal contribution of new HCOl - is accompanied by the excretion of an equivalent
amount of acid in the urine in the form of titratable acid, NH4 +, or both.
c. The amount of new HCO l - formed per day (approximately 70-100 mEq) is much less than
the quantity of filtered HCOl - reabsorbed per day (more than 4300 mEq).
D. Excretion of titratable acid (Figure 5-5; Table 5-7)
1. Filtration of HPO/-. Titratable acid denotes that portion of H + bound to filtered buffers and
equals the amount of alkali, in the form of sodium hydroxide (NaOH), required to titrate urine
back to the normal pH of blood (i.e., the number of milliequivalents of H+ added to the tubular
fluid that combined with phosphate or organic buffers).

Intracellular carbonic anhydrase is found throughout the renal tubule, including the distal convoluted tubule and
collecting duct. In the proximal convoluted tubule, carbonic anhydrase also is located in the luminal cell membranes.

.-

C.A. = carbonic anhydrase


= .passive process
= active process

-+

Proximal
tubular cells

Lumen

Blood

Na+

Na+

HPO/HPO/- Na+

'-

H+

Na+ H2P0 4

T i\----+Na +

./

H+ + HC03 -

y
k.A.
+
H2C0 3

CO 2

/
'-.

Na+

-----

--------- HC0

H2O

+
Na

HC03

Metabolism

Figure 5-5. Mechanism of controlled excretion of titratable acid in the proximal tubules.

a. Titratable acid is largely attributed to the conversion of HPO/- to H 2 P0 4 - . Additional


buffer species that may contribute to titratable acid are creatinine, uric acid, J3-hydroxybutyrate, and 50/-.
b. Titratable acid is a poor measure of the total amount of H+ secreted by the tubules, since
most of the acid produced by this secretion is H 2 C0 3 , which disappears from the urine as
CO 2 ; however, the H+ that is trapped by anions of noncarbonic acids remains in the final
urine in the form of titratable acid.
c. Titratable acid is a measure of the content of weak acids. It is not a measure of the H+ that
combines with ammonia (NH 3 ) to yield NH4 +, because the pK' of the NH)NH4 + buffer
system is high (9.2).
d. The proximal tubule is the major nephron site where titratable acid is formed. Additional
titratable acid is generated along the collecting duct by a H + -ATPase pump.

2. Excretion of H 2 P0 4 - . The exchange of H + for Na + converts dibasic sodium phosphate


(Na 2 HP0 4) in the glomerular filtrate into dihydrogen phosphate (NaH 2 P0 4), which is excreted
in the urine as titratable acid. The H+ secreted into the tubules, therefore, can react with filtered
HPO/- rather than the filtered HC03 - .
a. Besides HC0 3 - , HPO/- represents a major filtered conjugate base. * Furthermore, the
HPO/- /H 2 P0 4- buffer pair provides an excellent buffer system because it has a pK' of

6.8.
Table 5-7. Urinary Acid Excretion in Health and Disease (in mEq H+ /day)
Urinary Acid

Normal Excretion

Excretion in
Diabetic Ketoacidosis

Titratable acid

10-30
30-50
40-80

75-250
300-500
375-750

Ammonium
Total

'Since about 75% of the filtered HPO/- is reabsorbed, only about 25% of the filtered HPO/- is available for
buffering.

b. The blood HPO/- /H 2 P04- molar ratio is 4:1. The H+ secretory system converts much of
the HPO/- to H 2 P0 4- in the tubular fluid, resulting in a urinary HPO/- /H 2 P04- molar
ratio of 1 :4. The acidification of the phosphate buffer system occurs significantly in the
proximal tubule, and much of the H+ generated by the formation of H 2 S0 4 and H 3P04
during protein and phospholipid metabolism is excreted in this way.
c. H+ secreted into the lumen that reacts with HC03- is not excreted, whereas H+ secreted
into the lumen that reacts with a nonbicarbonate buffer remains in the tubular fluid and is
excreted.
d. Urinary phosphate is the major anion contributing to urine acid excretion.
e. Virtually all of the filtered phosphate is reabsorbed by the Na +-phosphate symporter of the
proximal tubular brush border.

3. Role of aldosterone in H+ excretion. Aldosterone exerts its major renal effect on the collecting
duct.
a. Aldosterone plays an important role in regulating collecting duct HC03- reabsorption via
its stimulatory effect on H+ secretion.
b. Aldosterone stimulates H+ secretion directly on the tubular cell.
c. Aldosterone stimulates H+ secretion indirectly by increasing Na+ reabsorption.
E. Excretion of ammonium (NH 4+) [Figures 5-6; 5-7; see Table 5-7]. Unlike phosphate, NH3 enters
the tubular lumen not by filtration but by tubular synthesis and secretion, which normally are
confined to the distal and collecting tubules.
1. Secretion of NH 3 NH3 is synthesized mainly by the deamidization and deamination of glutamine in the presence of glutaminase. Most of the NH4 + excreted in the urine is produced in
the proximal tubular cells from amino acids, primarily glutamine.
a. The NH4 + that are formed are actively secreted into the lumen and are excreted in the final
urine at a rate equal to their secretion rate.
b. With regard to proximal NH4 + secretion, it must be emphasized that the traditional view
that states that NH3 combines with the H+ derived from the dissociation of H 2 C0 3 is
incorrect. Instead, the H+ produced in the metabolism of glutamine results in the formation
of NH4 + within the proximal tubular cell (see Figure 5-6).
c. The mechanism of distal nephron NH4 + excretion is different from that in the proximal
tubule (see Figure 5-7).
2. Formation of NH4 +. The NH3/NH4 + system has a very high pK' (about 9.2), which means
that, at the usual urine pH, virtually all the nonpolar NH3 that enters the distal tubular lumen
immediately combines with H+ to form NH4 +, which is nondiffusible because it is lipid insoluble. The renal excretion of NH4 + causes the net addition of HC0 3- to the plasma.
a. The important physiologic characteristic of the NH)NH4 + buffer pair is that, as H+ is
combined with intraluminal buffer (NH 3), H+ is excreted in the urine as NH 4+, a substance
that does not cause the pH of urine to fall.
b. Under most conditions, the excretion of NH4 + is quantitatively more important to the
acid-base balance of the body than is the excretion of titratable acid. The normal kidney
excretes almost twice as much acid combined with NH3 (30-50 mEq/day) as titratable acid
(10-30 mEq/day) [see Table 5-7].

....

aKG 2- -.2HC0 3 Glutamine _ - '


..... 2NH+

..

....

Urine

Extracellular
fluid

Proximal
tubule
Figure 5-6. Formation and active secretion of NH. + by the proximal tu
buies. aKG 2 - = aketoglutarate .
(Reprinted from Seldin DW, Giebisch
G (eds): New concepts in renal ammonium excretion. In The Regulation
of AcidBase Balance. New York, Ra
ven, 1989, p 170.)

= carbonic anhydrase
-.. = passive process
,..--, = active process

C.A.

Distal and collecting


tubular cells

Lumen

Blood

Na+
I
I

CI-

~-

,,

" ......... 7

CI

N-----. Na+

\. .J

H+

H+

H2C03

r
)
HC03- \..
----------- ......

CI

.... ---------

Na

HC03
+

HC03-

Metabolism

NH3

....

AC.A.
CO 2 + H2O
H+

Na+

-------. NH3

Glutaminase

Glutamine

NH4+ CIFigure 5-7. Mechanism of controlled excretion of NH4 + in the distal and collecting tubules.

(1) In chronic severe metabolic acidosis, NH3 serves as the major urinary buffer, and NH4 +
excretion can increase from a normal value of 30 mEq/day to 500 mEq/day. However,
in chronic acidosis, the excretion of H2P04 - may increase by only 20-40 mEq/day.
(2) In diabetic ketoacidosis, the excretion of titratable acid may reach 75-250 mEq/day,
while the excretion of acid combined with NH3 may reach amounts of 300-500 mEq/
day. However, the ratio of NH4 + to titratable acid remains within the normal range of
1 to 2.5.

3. Excretion of H+. The sum of titratable acid and urinary NH4 + excretion represents the net
gain of HC03- for the body fluids.
a. The net amount of fixed acid excreted (in mEq/day) equals the sum of titratable acid and
NH4+ minus the urinary [HC0 3-].
b. Normally, the urine is free of HC0 3- because all the HC03- remaining in the distal and
collecting tubules has combined with secreted H+ and been reabsorbed.
c. Aldosterone enhances NH3 production via an effect on cellular metabolism.

VII. PRIMARY AND SECONDARY ACID-BASE ABNORMALITIES


A. Definitions and basic concepts (Tables 5-8; 5-9)
1. Acidosis and alkalosis
a. Acidosis or acidemia is an abnormal clinical condition or process caused by the bodily
accumulation of acid (or the loss of base) sufficient to decrease pH below 7.36 or to increase
the [H+] above 43.6 nEq/L of blood in the absence of compensatory (secondary) changes.
b. Alkalosis or alkalemia is an abnormal condition or process caused by the accumulation of
base (or the loss of acid) sufficient to raise pH above 7.44 or to decrease the [H+] below
36.3 nEq/L of blood in the absence of compensatory changes.

Table 5-8. Characteristics of the Uncompensated Acid-Base Abnormalities


Effect on:
Acid-Base
Abnormality

Primary
Disturbance

Acidosis
Respiratory
Metabolic

t [HC0 3 -]

Alkalosis
Respiratory
Metabolic

[HCO l -]IS x Peo 2

[H+]

Compensatory
Response

pH

< 20
< 20

t Peo 2

> 20
> 20

t Peo 2

t [HC03 -]

Note-These conditions are simple disturbances (i.e., they represent the effects of one primary etiologic factor) and,
thus, are termed uncompensated or pure acid-base abnormalities. The compensatory response always occurs in the
same direction as the primary disturbance.

2. Respiratory and metabolic


a. The adjective respiratory denotes that the primary abnormality involves impairment in
alveolar ventilation, which results in an abnormally high or low [total CO 2 ] of the ECF. *
(1) Respiratory acidosis refers to a condition of abnormally high arterial Peo v which is
termed hypercapnia (hypercarbia).
(2) Respiratory alkalosis refers to a condition of abnormally low arterial Peo v which is
called hypocapnia (hypocarbia).
b. The adjective metabolic denotes that the primary abnormality involves an abnormal gain
or loss of noncarbonic acid by the ECF, which affects [HC0 3 -].
(1) Metabolic acidosis refers to a disturbance that leads to the accumulation of noncarbonic acid in the ECF or to the loss of HC03 - from the ECF.
(2) Metabolic alkalosis refers to an imbalance characterized by a loss of noncarbonic acid
or a gain of HC0 3 - by the ECF.
3. Primary and secondary factors
a. The factor in the [HC0 3 -115 x Peo 2 ratio (i.e., [HC0 3 - ] or Peo 2 ) that undergoes the
greater degree of displacement (Le., the larger proportional change) indicates the primary
abnormality (see Tables 5-8; 5-9).
(1) If the [HC0 3 -]/5 x Peo 2 ratio becomes less than 20:1, by either decreasing the
numerator or increasing the denominator, pH falls (t[H+]), and acidosis occurs.
Table 5-9. Primary and Secondary* Changes in the CO 2 /HC0 3 - System
Alkalosis

Acidosis
[HC03 -]
Respiratory

Peo 2

+
I

...

..
..

< 20:1

[total C0 2F . t
[HC0 3 -]
Metabolic

Peo 2

[total CO 2 ]

I
I

[HC0 3 -]
5

Peo 2

[total CO 2 ]

< 20:1

..
I
I

+
t

[HC0 3 -]

...

5 x Peo 2

[total CO 2 ]

> 20:1

> 20:1

...

Note-Dashed arrows denote direction of compensatory responses. Thin solid arrows show changes in [total C0 2 l
during respiratory imbalances. Wide arrows depict direction of change of primary acid-base imbalance and direction
of change in [total CO 2 ]. (Adapted from Christensen HN: Diagnostic Biochemistry. New York, Oxford University
Press, 1959, p 106.)
*A compensatory (secondary) response in the alternate variable occurs in the same direction and counteracts the
effect of the primary disturbance by returning the [HCO) -liS x Pc0 2 ratio toward normal (20:1).
t[Total C0 2 l does not always increase in alkalosis or decrease in acidosis. Dissolved CO 2 contributes little to the [total
C0 2 l. [Total C0 2 l provides little information about pulmonary function.

*[Total

C0 2 l refers to the dissolved CO 2 (S x Pc0 2 ) plus the [HCO) -l.

(2) If the [HC0 3 -]lS x Peo 2 ratio becomes more than 20:1, by either increasing the
numerator or decreasing the denominator, pH rises U[H+]), and alkalosis occurs.
b. A secondary response in the alternate variable, which occurs in the same direction as the
primary abnormality, counteracts the effect of the primary abnormality on the [HC0 3 -]I
S X Peo 2 ratio. The secondary change represents compensation and acts to minimize the
pH alteration produced by the primary disorder.
4. Simple and mixed acid-base disturbances
a. Simple acid-base imbalances are caused by one primary factor.
b. Mixed acid-base imbalances are caused by more than the primary factor.
(1) Mixed-type disturbances are not uncommon. Sometimes a primary abnormality of one
type is superimposed on a primary abnormality of another type.
(a) A patient with respiratory acidosis from pulmonary emphysema may develop metabolic acidosis from uncontrolled diabetes or metabolic alkalosis from large doses of
corticosteroids used in the treatment of an attack of status asthmaticus.
(b) A patient with a metabolic acid-base disturbance, in turn, may develop a respiratory
acid-base abnormality.
(2) The changes in the variables may be difficult to interpret in mixed acid-base disturbances,
because manifestations of one primary abnormality may be either cancelled out or
augmented by those of the other primary abnormality. Therefore, a normal pH may
not necessarily mean a normal compensation in mixed acid-base imbalances.

5. Plasma [HC0 3 -], Peo 2 , and [total CO 2 ], For an adequate analysis of any acid-base disorder,
it is necessary to measure pH, Peo 2 , and the total CO 2 content of blood.
a. An acid-base disorder cannot be diagnosed with certainty from the plasma [HC0 3 - ]
alone. Although a reduction in the plasma [HC0 3 -] may be due to metabolic acidosis, it
also can indicate a renal compensation for respiratory alkalosis. Similarly, an elevated
[HC0 3 -] can result from a metabolic alkalosis or the secondary response to respiratory
acidosis. Since the aim of therapy is to bring about acid-base balance and not to normalize
[HC0 3 -], it is necessary to measure the pH (i.e., [H+]) when acid-base imbalance is
suspected.
b. low Peo 2 may be due to respiratory alkalosis or a respiratory compensation to metabolic
acidosis. Similarly, a high Peo 2 may be caused by respiratory acidosis or a respiratory
compensatory response to metabolic alkalosis.
c. [Total CO 2 ] does not always increase in alkalosis or decrease in acidosis (Figure 5-8; see
Table 5-9). About 90% of the [total CO 2 ] of plasma is contributed by [HC0 3 -], and 5% is
contributed by dissolved CO 2 and H 2 C0 3 ; therefore, dissolved CO 2 contributes little to the
[total CO 2 ], The [total CO 2 ] gives little information about the functional status of the lungs.
d. The physician accepts [total CO 2 ] and [HC0 3 -] as essentially interchangeable terms. The
[total CO 2 ] normally exceeds the [HC0 3 - ] by 1.2 mmol. Most laboratories measure the
[total CO 2] and not the [HC0 3 -].

35

c5' 20

()

15

7.0

7.1

7.2

7.3

7.4

pH

7.5

76

7.7

Figure 5-8. A pH-[HC0 3 -l diagram illustrating that acidosis (points A and C) and alkalosis (points B and D)
cannot be differentiated solely on the basis of [HC0 3 -l
or [total C0 2l. The rHCa 3 - J pertains to plasma. A =
respiratory acidosis; B = respiratory alkalosis; C = metabolic acidosis; D = metabolic alkalosis; and N = normal
point. The slope of line ANB is a measure of the nonbicarbonate buffer capacity of whole blood. (After Davenport
7.8 7.9 HW: The ABC of Acid-Base Chemistry, 5th edition. Chicago, University of Chicago Press, 1969, p 65.)

[BB)

BE

(mEq/L)

(mEq/L)

58

"""7'/.n"777?//i"7'/.n"777?//i'77r
///'/////////////////////

+10

::::~::Metabolic alkalosis:::::::::
///////////////~~////////

53
48

43

+5

-----liormalvvhole-----arterial blood
----------------------

//////////////////////////

::::::::Metabol!c acido~il>;::::::::::

38

/////////~~/////////////~~

-10

Figure 5-9. Diagram illustrating the relationship between


base excess [BE] and the acid-base status of normal whole
arterial blood. BE is the base concentration (in mEq/L),
as measured by the titration with strong acid to pH 7.4,
at Pco 2 of 40 mm Hg, and at 37" C. For negative values
of BE, the titration must be carried out with strong base.
BE measures the change in the concentration of the buffer
base ([BB]) from its normal value, which is 48 mEq/L in
normal whole arterial blood. [BB] is the sum of the buffer
anions of blood or plasma. When observed [BB] is less
than normal [BB], BE is a negative value; when observed
[BB] is greater than normal [BB], BE is a positive value;
and when observed [BB] equals normal [BB], BE equals
zero. (After Winters RW, et al: Acid-Base Physiology in
Medicine. Westlake, Ohio, London Co, 1967, p 45.)

B. Clinical expressions for evaluation of acid-base status

1. Base excess (BE) [Figure 5-9; Table 5-10]


a. Definition. BE refers to the change in the concentration of buffer base [BB] from its normal
value, or, the observed [BBJ minus the normal [BBJ in mEq/L of whole blood. The range
of normality for BE is -2 to +2 mEq/L in arterial whole blood.*
(1) The [BB] in normal whole blood is 48 mEq/L, which is arbitrarily set to 0, and deviations
(i.e., ::!: BE) are measured from this reference point.
(2) The normal [BB] of 48 mEq/L equals the sum of all the conjugate bases in 1 L of arterial
whole blood. These bases and their concentrations are:
(a) [HC0 3 -] = 24 mEq/L
(b) [Protein-] = 15 mEq/L
(c) [Hb- /Hb0 2 - ] = 9 mEq/L
b. Clinical application. BE refers principally to the [HC0 3 -] but also to other bases in the
blood (mainly plasma protein and hemoglobin). However, the [HC0 3 - ] or [total CO 2 ] is
used in acid-base problems as the indicator of BE.
(1) Because [HC0 3 -] and BE are influenced only by metabolic processes, there are only
two acid-base conditions associated with abnormalities in [HC0 3 -] and BE.
(a) Metabolic acidosis. When a metabolic process leads to the accumulation of noncarbonic acid in the body or the loss of HC0 3 - , the [HC0 3 -] falls, and the BE value
becomes negative. Metabolic acidosis is associated with a BE below -5 mEq/L.
(b) Metabolic alkalosis. When a metabolic process causes a loss of acid or an accumulation of HC03 - , the [HC0 3 -] rises above normal, and the BE value becomes
positive. Metabolic alkalosis is associated with a BE above +5 mEq/L.
(2) The BE value serves only as a rough guide for alkalosis and acidosis therapy.
2. Anion gap
a. Definition
(1) The ionic profile of normal serum is depicted in Figure 5-10. The law of electroneutrality states that the number of positive charges in any solution must equal the number
of negative charges. If every ion present in serum were measured, the concentration of
Table 5-10. Base Excess and Metabolic Acid-Base Abnormalities

Base
Excess

Metabolic
Abnormality

Characteristics

Metabolic alkalosis

Noncarbonic acid is lost;


HC0 3 - is gained

Metabolic acidosis

Noncarbonic acid is gained;


HC0 3 - is lost

Adapted from Broughton JO: Understanding Blood Cases. Madison, Wisconsin, Ohio Medical Products, form no.
456, 1979, p B.

*Because BE is a negative or positive value, the term "base deficit" is avoided.

160

I
I
I

Ca 2 + Mg 2 +
K+ (5)

140

Protein anions

I
I
I
I

K+ (5)

PO/" ,SO/ ,
Organic anions
120 - I - -

HC03 - (25)

//////////////
/////////////
///"~"/"'//

/ / / Anion gap/ / r--:::j;///(15f'//;;


////// ... /////
/////////////~

H-

HC0 3 - (25)

r---

100 - I - -

I-+-

r--

--

H--

r--

....J

;Z-80
E

Na + (140)

CI- (105)

Na + (140)

CI- (105)

60 - I - -

-+-

r--

40 -I---

H-

r--

20 - I - -

f.---\-

r--

Cations

Anions

True circulating electrolytes


True cation-anion equality

Cations

Anions

Measured electrolytes
Apparent catlon-anion disparity

Figure 5-10. Normal plasma concentrations of anions and cations in the mEq/L show that circulating cations always
counterbalance anions, thereby maintaining electroneutrality (left). Routine electrolyte assays only measure a portion
of circulating anions, and an apparent cation-anion disparity exists (right). This difference, in mEq/L, is termed the
anion gap. Parentheses indicate concentrations in mEq/L. (Reprinted from Narins RG, et al: Lactic acidosis and
elevated anion gap (I). Hosp Pract 15:125-136, 1980. Original drawing by Albert Miller.)

cations would equal the concentration of anions if these concentrations were expressed
in mEq/L. Routine serum electrolyte determinations measure essentially all cations but
only a fraction of the anions. This apparent disparity between the total cation
concentration and the total anion concentration is termed the anion gap. It is a
virtual measurement and does not represent any specific ionic constituent.
(2) The anion gap, which has a normal value of 12 4 mEq/L, reflects the concentrations
of those anions actually present but routinely undetermined (i.e., other than [CI-] and
[HeO l -]), such as:
(a) Polyanionic plasma proteins (primarily albumin)
(b) Inorganic phosphates
(c) Sulfate
(d) Ions of organic acids (e.g., lactic, ,8-hydroxybutyric, and acetoacetic acids)
(3) When the anion gap is increased, unmeasured anions fill this gap. (Most of these anions
usually are the products of metabolic processes that generate H +.)

b. Determination of anion gap


(1) The anion gap (AG) equals 16 mEq/L of anions other than HCO) - and CI- when
determined by the following equation
AG = ([Na+] + [K+]) - ([HCO)-] +[CI-])
= (142 mEq/L + 4 mEq/L) - (25 mEq/L + 105 mEq/L)
= 146 mEq/L - 130 mEq/L
= 16mEq/L
(2) Often the anion gap is calculated with Na+ as the major cation, because K+ has a
relatively minor quantitative contribution. The equation, then, becomes:
AG = [Na+] - ([HCO) -] + [CI-])
= 142 mEq/L - (25 mEq/L + 105 mEq/L)
= 142 mEq/L - 130 mEq/L
= 12mEq/L
(3) The plasma proteins contribute a significant amount of negative charges and, thus,

anionic equivalence (16 mEq/L). The plasma proteins account, almost stoichiometrically, for the difference between the [Na+] and the sum of [HCO) -] and [CI-].
c. Clinical application
(1) Acidification with acids other than HCI
(a) Organic acids increase the anion gap. In this case, the lost (neutralized) HCO) - is
not replaced by the routinely unmeasured anions of noncarbonic acids such as
lactic acid and the ketoacids. Thus, the anion gap is increased by all metabolic
acidoses except the hyperchloremic acidoses. With the accumulation of acid,
there is rapid extracellular buffering by HCO) -. If the acid is HCI, then
HCI + NaHCO) ~ NaCi + H 2 CO)
The net effect is the milliequivalent-for-milliequivalent replacement of extracellular
HCO) - by CI-. Since the sum of [CI-] and [HCO) -] remains constant, the anion
gap is unchanged in hyperchloremic acidosis.
(b) It is apparent that the decrease in plasma [HCO) -] equals the increase in the anion
gap (see Figure 5-10). The presence of an increased anion gap usually indicates an
excess of H+ derived from noncarbonic acid. Whatever the size of the anion gap,
it is the retention of H+-not the particular anion-that is responsible for the
acidosis.
(i) An increased anion gap due to excess H+ derived from noncarbonic acid occurs
in metabolic acidosis; this also may result from a compensatory increase in
lactic acid production in response to respiratory alkalosis via activation of the
phosphofructokinase step in glycolysis.
(ii) In respiratory acidosis the anion gap is not increased, because excess H+ is
derived from the H 2 CO) pool, not the noncarbonic acid pool.
(2) Conditions that increase the anion gap include diabetic and alcoholic ketoacidosis,
intoxicant and lactic acidoses, and renal failure.
(3) Conditions that cause metabolic acidosis without an increase in the anion gap
are associated with a high serum [CI-] and include diarrhea; pancreatic drainage;
ureterosigmoidostomy; ileal loop conduit; treatment with acetazolamide, ammonium
chloride, or arginine-HCI; renal tubular acidosis; and, rarely, intravenous hyperalimentation.

3. Osmolar gap
a. Definition. Osmolar gap refers to the disparity between the measured serum (plasma)
osmolality and the calculated serum osmolality.
b. Clinical application. Osmolar gap measurement provides a reasonably good screening
procedure for toxins. Other causes of metabolic acidosis do not affect the osmolar gap,
since the metabolic acid simply replaces the HCO) - with another anion and HCO) - is lost
as CO 2 ,
(1) Serum osmolality is estimated by dividing blood urea nitrogen (BUN) and glucose
concentrations (in mg/ L) by their molecular weights, * thereby converting them to
milliosmoles (mOsm)/L. Doubling the serum [Na+] (in mEq/L) provides an estimate of

*The molecular weight of urea (60) is not used because it is BUN that is measured, and urea contains 2 nitrogen
atoms (2 x 14 = 28).

40

Peo 2 = 40 mm Hg

35
~
30
E

E
::::25

8'

20

15
10~--.--.--.---~-.---.--.--,

7.0
pH

Figure 5-11. The pH-[HCO, -] diagram showing the


buffer curves of separated plasma (5) and true oxygenated
plasma (n. Point N denotes the intercept for a normal
plasma [HCO, -] of 24 mmol/L and a normal pH of 7.4.
The slope of the normal in vitro buffer line of true plasma
is a function of the hemoglobin content of blood. Lines 5
and T are called non bicarbonate buffer curves, and the
steepness of the slopes of these lines is a quantitative
assessment of the amount of non bicarbonate buffer present in the system. These same two lines represent CO,titration curves. The steeper the buffer line, the smaller
the pH change resulting from a given increase or decrease
in Pco,. Vertical arrows indicate increases and decreases
in Pco" fixed acid, or fixed base. (Adapted from Davenport HW: The ABC of Acid-Base Chemistry, 5th edition.
Chicago, University of Chicago Press, 1969, p 48.)

the serum ion concentration. With a BUN of 14 mg/dl, plasma glucose of 90 mg/dl,
and a plasma [Na+] of 142 mEq/L, serum (plasma) osmolality (Posm) is estimated as

+ [BUN (mg/U/28] + [glucose (mg/U/180]


2 (142) + 140/28 + 900/180
286 + 5 + 5 = 296 mOsm/kg

Pos m = 2 [Na+]
=
=

(2) Circulating intoxicants increase measured serum osmolality without altering serum
[Na+]. Therefore, the measured serum osmolality exceeds the calculated serum osmolality, and the difference closely reflects the osmolar concentration of the circulating
toxins.

C. Graphic evaluation of acid-base status: the pH-[HC0 3 - ] diagram (Figures 5-11; 5-12)
1. Value of diagram. When acid-base data obtained from blood are plotted on the pH-[HC0 3 -]
diagram, the physician has a tool that can be used to:
a. Diagnose the primary cause of an acid-base abnormality
b. Determine the degree of compensatory response of the kidneys and of the lungs by monitoring the [HC0 3 -] and Peo 2 , respectively
c. Estimate the concentration of noncarbonic and carbonic acids in the ECF
d. Aid in the choice of therapy to correct an acid-base imbalance
e. Measure the nonbicarbonate buffer power of whole blood by the slope of the buffer line
f. Indicate not only the total buffer activity but also the distribution of the buffering activity
between the bicarbonate and non bicarbonate buffer systems
2. Construction of diagram: effects of H+ and CO 2 on pH. The basis for plotting pH and
[HC0 3 - ] on cartesian coordinates is best understood from a consideration of the CO 2 /HC0 3 buffer system

(6)

40

35

7.8

Figure 5-12. The pH-[HCO, -] diagram with arterial


Pco, isobars for 20, 40, 60, and 80 mm Hg. Each arterial
Pco, isobar is the titration curve of a HCO, - -H,CO,
solution with arterial PCo, held constant. Point N denotes the intercept for a normal plasma [HCO, -] of 24
mmol/L and a normal pH of 7.4. The dashed arrow
represents the buffer line. (After Davenport HW: The ABC
of Acid-Base Chemistry, 5th edition. Chicago, University
of Chicago Press, 1969, p 45.)

a. Effect of fixed acid or base on pH


(1) Addition of H+ at a constant Peo 2
(a) When H+ ions, in the form of fixed (noncarbonicl acid, are added to the system
depicted in equation (6), most of them combine with HC03 - to form H 2 C0 3 , which
dehydrates to form CO 2 and water. Thus, the series of reactions in equation (6) is
driven to the left.
(b) The decrease in [HC0 3 - ] closely approximates the amount of H+ added only if the
solution has no other nonbicarbonate buffer substances. Otherwise, the amount of
acid added would be greater than the decrease in [HC0 3 -].
(2) Addition of base at a constant Peo 2
(a) When a base is added to the system depicted in equation (6), some of the base
combines with H+, causing more H 2 C0 3 to dissociate into H+ and HC03 - . Here
the series of reactions in equation (6) proceeds to the right.
(b) The increase in [HC0 3 -] determines the amount of base added.
(3) Principles (see Figure 5-11)
(a) When fixed acid or base is added under the conditions described, there is an inverse
relationship between [H+] and [HC0 3 -], and the primary acid-base disturbance is
metabolic. Thus, when fixed acid or base is added, the [H+] and [HC0 3 -] change
in opposite directions.
(b) Or, if the direction of change in [HC0 3 -] is the same as that for pH, the primary
acid-base abnormality is metabolic.
b. Effect of CO 2 on pH
(1) Addition of CO 2 (see Figure 5-12)
(a) An increase in arterial Peo 2 titrates the blood (and ECF) in the acid direction. This
is clear from equation (6), which shows that an increase in Peo 2 causes more CO 2
to hydrate to form H 2 C0 3 ; the H 2 C0 3 in turn dissociates into H+ and HC03 - ,
moving the reactions to the right.
(b) In the system shown in equation (6), every molecule of CO 2 that hydrates and
dissociates forms one H + and one HC0 3 - ; therefore, the changes in [H +] and
[HC0 3 -] are exactly equal.
(c) The final [H+] depends not only on the change in Peo 2 but also on the buffers in
the system. For every H 2 C0 3 molecule that has its H+ taken up by a buffer ion, one
HC0 3 - appears in the solution. For example, the reaction with hemoglobin is
H 2 C0 3

+ Hb-

HHb

+ HC0 3 -

(d) If CO 2 is added to whole blood (which contains many buffer systems), the H+,
rather than remaining in solution, are mostly bound to protein buffers (Pr-) as
CO 2 + H 2 0

H 2 C0 3

HC0 3 - + H+

+
Pr-

1~
H Pr
Therefore, the CO 2 /HCO l - system alone is not effective in buffering the
changes in pH that are induced by the addition of CO 2 .
(2) Removal of CO 2
(a) When CO 2 is removed from the system shown in equation (6), fewer CO 2 molecules
are available for combination with water to form HC0 3 - .
(b) Thus, not only is CO 2 decreased but [H+] and [HC0 3 - ] also are decreased and the
reactions shift to the left.
(3) Principles. The concentration of acid added to a buffer solution by a change in Peo 2
is equal to the change in [HC0 3 -].*
(a) If CO 2 is added or removed, the [H+] and [HC0 3 - ] change in the same direction,
and the primary acid-base disturbance is respiratory.
(b) Or, if the direction of change in [HC0 3 -] is opposite to that for pH, the primary
acid-base disturbance is respiratory.

*In humans, the serum [HC0 3 -] is 24 x 10- 3 Eq/L (24 x 10- 3 mollL), whereas the serum [H+] is 40 X 10- 9
Eq/L (40 X 10- 9 mol/l). Therefore, the [HC0 3 -] is 6 X 10 5 greater than [H+], and any shift (left or right) in
equilibrium has a much greater proportionate effect on [H+] than on [HC0 3 -].

(c) As Peo 2 is increased or decreased, the values for pH and [HCO J -] form coordinates
that define a nearly straight line termed the buffer line or the CO 2 titration curve
(see Figure 5-12).
(4) Summary
(a) When H+ is added or removed, the [H+] and [HCO J -] change in opposite directions and the acid-base imbalance is metabolic in origin.
(b) When CO 2 is added or removed, the [H+] and [HCO J -] change in the same
direction and the acid-base imbalance is respiratory in origin.

3. Properties of Peo2 isobars


a. At constant [HC0 3 - ] , Pco 2 is proportional to [H+]. This property can be illustrated using
the following form of the Henderson equation to solve for Peo 2

Pea

[H+]

Peo 2

= -'-----~

24

[HCO J -]

[H+][HCO J - ]

24

b. At constant pH (i.e., along any vertical line in Figures 5-11 and 5-12), Peo 2 is proportional
to [HCO J -].

D. Interpretation of acid-base abnormalities using the pH-[HC0 3 - ] diagram (Figure 5-13). Acidbase imbalances are determined graphically with reference to the intercept of the Peo 2 isobar of
40 mm Hg and the normal buffer line. The intercept of these two curves marks the point of
normality (N), which is associated with a pH of 7.4 (the abscissa) and a [HCO J -] of 24 mmol/L
(the ordinate). Point N is the triple intercept that defines the pH, [HC0 3 -], and Peo 2 of true
arterial plasma of a normal individual.

1. Points to the left of point N (points A, C, E, and F) indicate acidosis. Points to the right of
point N (points B, 0, G, and H) indicate alkalosis.
a. Line NA represents the direction of an individual's response to an increase in Peo 2 Points
to the left of the normal Peo 2 isobar (points A, E, and F) indicate respiratory acidosis.
(1) Point A denotes a condition of uncompensated respiratory acidosis, which is characterized by a low pH, high [HCO J -], and high Pco 2
(2) Point E denotes respiratory acidosis (tPeo 2 ) with metabolic acidosis U[HCOJ -]).
(3) Point F denotes respiratory acidosis (tPeo 2 ) with metabolic alkalosis (t[HCO J -]).
b. Line NB represents the direction of an individual's response to a decrease in Peo 2 Points
to the right of the normal Peo 2 isobar (points B, G, and H) indicate respiratory alkalosis.
(1) Point B denotes a condition of uncompensated respiratory alkalosis, which is characterized by high pH, low [HCO J -], and low Pco 2
(2) Point G denotes respiratory alkalosis UPeo 2 ) with metabolic acidosis U[HCOJ -]).
(3) Point H denotes respiratory alkalosis UPeo 2 ) with metabolic alkalosis (t[HCO J -]).

40

35
~
30
E

.s 25

I~

o() 20
E.

15
10~-''--.---.--+---.--.---r--'

70

pH

Figure 5-13. Pathways of acid-base


imbalance. The {HC0 3 - J pertains to
plasma. Line ANB represents the normal blood buffer line, and line CND
represents the normal PCo, isobar (40
mm Hg). N = normal point; A = respiratory acidosis (uncompensated);
B = respiratory alkalosis (uncompensated); C = metabolic acidosis (uncompensated); D = metabolic alkalosis (uncompensated); E = respiratory
acidosis + metabolic acidosis; F =
respiratory acidosis + metabolic alkalosis; C = respiratory alkalosis +
metabolic acidosis; and H = respiratory alkalosis + metabolic alkalosis.
(After Davenport HW: The ABC of
Acid-Base Chemistry, 5th edition.
Chicago, University of Chicago Press,

1969, p 65.)

2. Points below the normal buffer line (points C, E, and C) indicate conditions with a component
of metabolic acidosis (~ [HC0 3 -]). Points above the normal buffer lines (points 0, F, and H)
indicate conditions with a component of metabolic alkalosis (t[HC0 3 -]).
a. Line NC represents the direction of the development of metabolic acidosis in an individual;
the [HC0 3 -] of this individual moves down the normal Peo 2 isobar.
(1) Point C denotes a condition of uncompensated metabolic acidosis, which is characterized by low pH, low [HC0 3 -], and normal Peo 2
(2) Point E denotes metabolic acidosis (![HC0 3 -]) with respiratory acidosis (fPeo 2 ).
(3) Point C denotes metabolic acidosis (![HC0 3 -]) with respiratory alkalosis (!Peo 2 l.
b. Line NO represents the direction of the development of metabolic alkalosis in an individual;
the [HC0 3 -] of this individual moves up the normal Peo 2 isobar.
(1) Point 0 denotes a condition of uncompensated metabolic alkalosis, which is characterized by high pH, high [HC0 3 -], and normal Pco 2
(2) Point F denotes metabolic alkalosis (t[HC0 3 -]) with respiratory acidosis (fPeo 2 ).
(3) Point H denotes metabolic alkalosis (t[HC0 3 - ] ) with respiratory alkalosis (!Peo 2 ).
3. Mixed acid-base disturbances, in which the primary state of acidosis or alkalosis has an
additional abnormality with respect to Peo 2 and [HC0 3 -], are denoted by points E, F, C,
and H.
VIII.

COMPENSATORY MECHANISMS FOR PRIMARY ACID-BASE ABNORMALITIES (see


Tables 5-8; 5-9)
A. Terminology
1. Compensation is the secondary physiologic process occurring in response to a primary acidbase disturbance by which the deviation of blood pH is ameliorated. Thus, abnormal pH is
returned toward normal by altering the component that is not primarily affected.
a. The terms secondary and compensatory may be used to describe a change in the composition of the blood or to describe a process.
b. The terms secondary and compensatory may be used to describe a process, but they should
not be used to describe acidosis or alkalosis.
2. Correction denotes the therapeutic course of action that is directed toward counteracting or
altering the factor that is primarily affected. Correction involves the amelioration of the
primary underlying abnormality in [H+], [HC0 3 -], or Peo 2 In correction, as in compensation,
the pH is returned toward normal.
B. Basic mechanisms of compensatory responses
1. Buffers of the ECF represent the body's first defense mechanism for neutralizing noncarbonic
acid. H+ is transferred across tissue cell membranes in a direction that normalizes blood pH.
2. A deviation in plasma pH acts on the respiratory center to change alveolar ventilation, which,
in turn, alters the alveolar Peo 2 and arterial Peo 2 so as to counteract the change in plasma pH.
This is called respiratory compensation of a primary noncarbonic acid excess or deficit;
respiratory compensation usually is not complete (i.e., does not restore pH to normal).
3. Carbonate ion is released from bone, which is the predominant source of alkali for neutralizing excess noncarbonic acid added to the ECF.
4. Renal excretion of noncarbonic acid or base increases in order to eliminate an excess of
noncarbonic acid or base or to compensate for the pH changes due to an abnormal arterial
Peo 2 This is called renal compensation of a primary hyper- or hypocapnia.
5. The responses to respiratory acidosis and alkalosis differ from responses to metabolic acid-base
disorders in that there is virtually no extracellular buffering of H 2 C0 3 in respiratory acidosis
because HC03 - is not an effective buffer for H 2C0 3

C. Principles of respiratory and renal compensatory responses


1. General considerations
a. Physiologic compensation for major acid-base abnormalities rarely is complete. Therefore,
the singular concern of clinical diagnosis is to differentiate the primary cause of the imbalance
from the secondary (compensatory) response.

b. Most data indicate that the pH ([H+]) is the most important factor in determining the bodily
response to acid-base imbalances. (In both acid and base disturbances, the abnormal pH is
returned toward normal.) However, in spite of its critical biologic significance, a change in
pH does not provide all the information needed for quantitative assessment and, thus,

for planning therapy.


(1) The key determinants of the cause and compensation of acid-base abnormalities are

Pco 2 and [HC0 3 -], not pH.


(2) The most important factor in determining the efficacy of the bodily responses to devia-

tions from the normal acid-base status is pH or [H+].

2. Respiratory and renal processes (see Tables 5-8; 5-9)

a. Both respiratory and renal compensatory responses tend to restore the abnormal [HC0 3 - ] /

5 x Peo 2 ratio toward its normal value of 20:1.


(1) Primary acid-base disturbances of metabolic origin lead to secondary adjustment of
the Peo 2 by changes in the rate of alveolar ventilation.

(2) Primary acid-base disturbances of respiratory origin lead to secondary changes in


blood [HC0 3 -] by appropriate adjustment of the rate of H+ secretion/excretion from
the renal tubular cell into the tubular lumen.
b. These two processes interact to control the [H+] of the ECF, and this integration is clearly
understood using the mathematical relationship of the Henderson equation
[H+] = 24

Peo
2

[HC0 3 -]

From this equation it is apparent that:


(1) An increase in the [H+], regardless of cause, can be reduced toward normal by a
decrease in PC0 2 , an increase in plasma [HC0 3 -], or by both changes.
(2) A decrease in the [H+], regardless of cause, can be increased toward normal by an
increase in Peo 2, a decrease in plasma [HC0 3 -], or by both changes.

D. Etiology of acid-base disorders and their compensatory responses


1. Metabolic acidosis is a disorder characterized by a low arterial pH (t[H +]) or a reduced
plasma [HC0 3 -].
a. Causes of metabolic acidosis include:
(1) Gain of noncarbonic (fixed) acid by the ECF, such as excess quantities of the ketoacids
(J3-hydroxybutyric acid and acetoacetic acid) or lactic acid or the ingestion of alcohol,
salicylates, or N H 4 CI
(2) Loss of HC0 3 - and other conjugate bases, such as occurs with severe diarrhea and
fistulas (which cause loss of bowel fluids containing HC03 -)
b. Compensation for metabolic acidosis includes:
(1) Extracellular buffering primarily by HC03 (2) Intracellular buffering primarily by proteins and phosphates
(3) Respiratory compensation by an increase in alveolar ventilation resulting in a decline in
the Peo 2
(4) Renal compensation by an increase in H + excretion and an increase in HC03 - reabsorption
2. Metabolic alkalosis is a disorder characterized by an elevated arterial pH U[H+]) or an
increased plasma [HC0 3 -].
a. Causes of metabolic alkalosis include:
(1) Decreased production of noncarbonic acid or a loss of noncarbonic acid via the kidneys
or the gastrointestinal system (vomiting)
(2) Excess HC0 3 - or other conjugate base by ingestion or infusion
(3) Excessive renal reabsorption of HC03 (4) Overtreatment with HC0 3 - or lactate
b. Compensation for metabolic alkalosis includes:
(1) Respiratory compensation by hypoventilation resulting in an increase in plasma Peo 2
(2) Renal compensation by an increase in HC03 - excretion
3. Respiratory acidosis is a disorder characterized by a reduced arterial pH (t[H+]), an elevated
Pco 2 (hypercapnia), and a variable increase in the plasma [HC0 3 -j.

a. Causes
(1) The common denominator in respiratory acidosis is a reduction in alveolar ventilation.

(2) The most common causes are chronic obstructive pulmonary disease and the overuse
of respiratory depressant drugs.
b. Compensation for respiratory alkalosis includes:
(1) Increased renal reabsorption of HC03 (2) Increased renal H+ secretion and excretion
4. Respiratory alkalosis is a disorder characterized by an elevated arterial pH U[H+j), a low
Peo 2 (hypocapnia), and a variable decrease in the plasma [HC0 3 -j.
a. Causes of respiratory alkalosis include:
(1) Hyperventilation (hyperpnea)
(2) Anxiety and hysteria
(3) Salicylate overdosage
b. Compensation for respiratory alkalosis includes:
(1) Decreased urinary H+ excretion
(2) Increased urinary HC03 - excretion
E. Pathways for compensation of primary acid-base disturbances: the pH-[HCO] -] relationship (Figure 5-14)
1. Respiratory acidosis (point A). The system at fault in this condition is the respiratory system,
and compensation occurs through metabolic processes.
a. The kidneys excrete more acid and reabsorb more HC0 3 - , returning the [HC0 3 - liS X
Peo 2 ratio toward 20:1 and, therefore, returning pH toward normal (point A,).
b. If the Peo 2 is elevated but the pH is normal, the kidneys had time to retain HC03 - to
compensate for the elevated Peo 2 and the process is not acute; that is, the condition has
existed at least a few days to give the kidneys time to compensate (point A 2 ).
c. Usually, the body does not fully compensate for respiratory acidosis.
2. Respiratory alkalosis (point B). In this condition, the acid-base abnormality again is respiratory,
and compensation again occurs through metabolic means.
a. The kidneys compensate by excreting HC03 - , thus returning the [HC0 3 - liS X Peo 2 ratio
toward 20:1; this compensation takes 2-3 days (point B,).
b. Of the four acid-base abnormalities, only in respiratory alkalosis is the body able to compensate fully; the [HC0 3 -liS x Peo 2 ratio and pH, therefore, return entirely to normal (point

B2 )
3. Metabolic acidosis (point q. In this condition, the major abnormality is low [HC0 3 -j or
negative BE, and the compensation is a respiratory process.
a. By hyperventilation, the Peo 2 is lowered so that the [HC0 3 - liS X Peo 2 ratio is returned
toward 20:1 (point e,).
b. Since the compensatory system is the lungs, compensation can occur rapidly. If the metabolic acidosis is severe, however, the lungs may not be able to expel sufficient CO 2 to
compensate fully.

40
:J

35

~30
E
E

:::::25
I

020
u
~

15
10-r--.--.--~-,,--.--.--.--,

70

Figure 5-14. Effects of renal and respiratory compensation on the pH


and plasma [HC0 3 -l. N = normal
point; A = respiratory acidosis; A, =
respiratory acidosis with partial renal
compensation; A2 = respiratory acidosis with complete renal compensation; 8 = respiratory alkalosis; 8, =
respiratory alkalosis with partial renal
compensation; 8 2 = respiratory alkalosis with complete renal compensation; C = metabolic acidosis; C, =
metabolic acidosis with partial respiratory compensation; D = metabolic
alkalosis; and D, = metabolic alkalosis with partial renal compensation.
(After Davenport HW: The ABC of
Acid-8ase Chemistry, 5th edition.
Chicago, University of Chicago Press,

1969, p 65.)

4. Metabolic alkalosis (point 0). In this condition, the major abnormality is a high [HCO l -1, and
the compensation is a respiratory mechanism.
a. By hypoventilaton, the Pea 2 is elevated so that the [HCO l -]/S X Pea 2 ratio is increased
toward normal (point 0/).
b. The body usually cannot compensate fully for metabolic alkalosis.
F. Summary: characteristics of uncompensated acid-base disturbances and their compensatory
responses (see Table 5-8)
1. Metabolic acidosis
a. An uncompensated metabolic acidosis is characterized by a low pH, a low CO 2 content,
and a normal Pea 2 .
b. When a metabolic acidosis is compensated by a respiratory alkalosis, the pH tends to return
to normal but the CO 2 content and Pea 2 decrease.
2. Metabolic alkalosis
a. An uncompensated metabolic alkalosis is characterized by high pH, a high CO 2 content,
and a normal Pea 2
b. When a metabolic alkalosis is compensated by a respiratory acidosis, the pH tends to return
to normal but the CO 2 content and Pea 2 increase.
3. Respiratory acidosis
a. An uncompensated respiratory acidosis is characterized by a low pH, a high CO 2 content,
and a high Pea 2
b. When a respiratory acidosis is compensated by a metabolic alkalosis, the pH tends to return
to normal but the CO 2 content increases; the Pea 2 remains unchanged.
4. Respiratory alkalosis
a. An uncompensated respiratory alkalosis is characterized by a high pH, a low CO 2 content,
and a low Pea 2
b. When a respiratory alkalosis is compensated by a metabolic acidosis, the pH tends to
become normal but the CO 2 content decreases; the Pea 2 remains unchanged.

STUDY QUESTIONS
Directions: Each of the numbered items or incomplete statements in this section is followed by answers
or by completions of the statement. Select the one lettered answer or completion that is best in each
case.

1. Most of the body's total daily acid production


is derived from
(A) protein catabolism
(B) triglyceride catabolism
(C) phospholipid catabolism
(0) oxidative metabolism

6. In metabolic acidosis, the fall in arterial pH is


associated with which of the following arterial
blood conditions?

normal

(A)
(B)
(C)

2. All of the following are intracellular buffers


EXCEPT

(0)

(A) hemoglobin
(B) protein
(C) organic phosphate
(0) HC03 -

7. Respiratory alkalosis is characterized by which


of the following arterial blood conditions?

normal

(E)

pH

(E) carbonate

(A)
3. CO 2 , which is not an acid, can lead to an increase in the [H+] of body fluids through the formation of
(A) HC03 (B) H 2 C0 3
(C) lactic acid
(0) acetic acid
(E) phosphoric acid
4. Extracellular HC0 3 - is an effective buffer for
all the following acids EXCEPT
(A) phosphoric acid
(B) lactic acid
(C) sulfuric acid
(0) carbonic acid
(E) l3-hydroxybutyric acid
5. Which of the following statements about the
pH of CSF is FALSE?
(A) It is the same as plasma pH during normal
acid-base conditions
(B) It decreases during metabolic alkalosis
(C) It increases during metabolic acidosis
(0) It increases during hyperventilation
(E) It changes in the opposite direction as plasma
pH during metabolic acidosis

(B)
(C)
(0)
(E)

normal

8. Which of the following sets of values is indicative of compensated metabolic alkalosis?

(A)

(B)
(C)
(0)

(E)

[HC0 3 - ]
(mEq/L)

(mm Hg)

pH

20
40
17
34
17

25
46
30
10
19

7.50
7.56
7.3
7.7
7.9

Peo 2

9. Compared to normal, the urinary excretion of


acid in metabolic acidosis is best characterized by
which of the following patterns?
Urinary excretion of
Titratable acid
(A)
(B)
(C)
(0)
(E)

10. The following arterial blood data are obtained


from a hospitalized 55-year-old man:
[Na+] = 140 mEq/L
[HC0 3 -] = 15 mEq/L
[CI-] = 113 mEq/L
pH = 7.2
From these data, which of the following statements
about this patient is true?
(A) The patient has an elevated anion gap
(B) The patient has respiratory acidosis
(C) The [H+] of this patient is higher than 100

nmol/L
(0) The arterial Pco 2 of this patient is approximately normal
(E) The pH of this patient's CSF is decreased
markedly
11. An hysterical, 35-year-old woman is admitted
to the hospital and the following blood data are
collected: [HC0 3 -] = 22.2 mmol/L, Pco 2 = 30
mm Hg, and P0 2 = 98 mm Hg. From these data,
the blood [H+] of this patient would be expected
to be
(A) 17.4
(B) 22.5
(C) 28.1
(0) 32.4
(E) 36.7

nmol/L
nmol/L
nmol/L
nmol/L
nmol/L

12. A semi-comatose, 63-year-old male cigarette


smoker enters the hospital complaining of labored
breathing, chronic coughing, and drowsiness. He
has had a "smoker's cough" for 15 years. Physical
examination reveals wheezing and slowing of
forced expiration as well as cyanosis and bilateral
leg edema. Initial laboratory data include the following:
hemoglobin concentration
serum pH = 7.32
Pco 2 = 68 mm Hg
P0 2 = 33 mm Hg

18.5 mg/dl

From the above findings on history, physical examination, and laboratory testing, the most likely diagnosis is
(A) metabolic acidosis
(B) metabolic alkalosis
(C) respiratory acidosis
(0) respiratory alkalosis

13. The following blood data are collected from a


58-year-old man: [H+] = 49 nEq/L, Pco 2 = 30
mm Hg, and P0 2 = 95 mm Hg. From these data,
this patient's blood pH would be expected to be
(A) 4.23
(B) 7.31
(C) 8.24
(0) 3.56
(E) 6.17

Questions 14-15
The blood pressure falls during a surgical procedure
on an anesthetized animal. The arterial blood becomes cyanotic despite the maintenance of normal
ventilation. The arterial pH is 7.25, and the arterial
Pco 2 is 40 mm Hg.

14. The acid-base status of the animal is most


likely to be
(A) metabolic acidosis with respiratory compensation
(B) respiratory acidosis with metabolic compensation
(C) metabolic acidosis
(0) respiratory acidosis
(E) respiratory alkalosis with renal compensation
15. To increase pH toward normal, in which direction would the ventilation rate be changed and
what would be the corresponding change in arterial
Pco/

Ventilation Rate

Arterial Pco 2

(A)

(B)
(C)
(0)

(E)

no change

Questions 16-19

Questions 20-23

The following data were obtained from an arterial


blood sample drawn from a hospitalized patient:
pH = 7.55, Pco 2 = 25 mm Hg, and [HC0 3 -] =
22.5 mEq/L.

A 25-year-old male patient suffers from severe diarrhea for about 1.5 hours and reports to the emergency room, where several tests are performed.

16. This patient's arterial blood findings are consistent with a diagnosis of

20. Based on his history, this patient's arterial pH


would be expected to be approximately

(B) respiratory alkalosis


(C) metabolic acidosis
(D) respiratory acidosis

(A) 7.32
(B) 7.40
(C) 7.42
(D) 7.45
(E) 7.50

17. These findings indicate that the ratio of


[HC0 3 -] to dissolved CO 2 is

21. The pH of this patient's CSF would be expected to be approximately

(A) 5:1

(A) 7.12

(B) 10:1
(C) 20:1
(D) 30:1

(B) 7.22
(C) 7.30
(D) 7.32
(E) 7.40

(A) metabolic alkalosis

18. The data indicate that the CO 2 content is approximately


(A) 22 mmol/L

(B) 23
(C) 24
(D) 25
(E) 26

mmol/L
mmol/L
mmol/L
mmol/L

22. During the compensatory response to his


acid-base disorder, this patient's arterial Peo 2
would most likely be
(A) 32 mm Hg
(B) 40 mm Hg
(C) 46 mm Hg
(D) 50 mm Hg

60 mm Hg

19. The major compensatory response for this patient's acid-base disorder is

(E)

(A) hyperventilation
(B) hypoventilation
(C) increased renal HC03 - excretion
(D) increased H+ excretion

23. Following the compensatory response, a urinalysis of this patient would most likely indicate
(A) high pH
(B) decreased HC0 3 - excretion
(C) decreased titratable acid excretion
(D) increased Na+ excretion

Directions: Each question below contains four suggested answers of which one or more is correct.
Choose the answer.
A if 1, 2, and 3 are correct
B if 1 and 3 are correct
C if 2 and 4 are correct
D if 4 is correct
E if 1, 2, 3, and 4 are correct

24. Statements that correctly describe noncarbonic acids include which of the following?

27. Acid-base disturbances associated with an elevated arterial CO 2 content include

(1) They are called fixed acids

(1) diabetic ketoacidosis

(2) They are classified as nonvolatile acids

(2) respiratory alkalosis

(3) They are excreted by the kidneys

(3) metabolic acidosis

(4) They are buffered by both HC03 - and non-

(4) metabolic alkalosis

HC03 - buffers

25. Which of the following volatile acids can be


transiently classified as a nonvolatile acid when
present in high amounts?
(1) Lactic acid
(2) Acetic acid
(3) Citric acid
(4) Acetoacetic acid

26. The HC0 3 - buffer system is quantitatively the


most significant buffer in the
(1) interstitial fluid
(2) tissue cells
(3) CSF
(4) erythrocyte

28. Chronic acid-base disorders associated with


decreased arterial Peo 2 and [HC0 3 -] include
(1) respiratory acidosis
(2) metabolic acidosis
(3) metabolic alkalosis
(4) respiratory alkalosis

Directions: Each group of items in this section consists of lettered options followed by a set of numbered
items. For each item, select the one lettered option that is most closely associated with it. Each lettered
option may be selected once, more than once, or not at all.

Questions 29-32

Questions 33-37

For each acid-base disorder listed below, select the


appropriate pair of [HCO l -] and Pea 2 values.

For each patient described below, select the set


of arterial blood values that coincides with that
patient's acid-base condition.

[HC0 3 -]
(mmoI/L)

Pea 2
(mm Hg)

Patient
(A)

(B)
(C)
(D)

30
10
20
40

65
25
20
45

(A)
(B)
(C)
(D)

(E)
29.
30.
31.
32.

Metabolic acidosis
Metabolic alkalosis
Respiratory acidosis
Respiratory alkalosis

pH

Pea 2
(mm Hg)

[HC0 3 - ]
(mmoI/L)

7.00
7.10
7.34
7.50
7.56

70
27
70
46
26

16

8
39
36
23

33. Patient with metabolic alkalosis and partial


respiratory compensation
34. Patient with metabolic acidosis partially compensated by hyperventilation
35. Patient with the highest CO 2 content
36. Patient with the highest [H+]
37. Patient with the highest [HCO l -]/S x Pco 2
ratio

Questions 38-41

Questions 42-47

Points A-D on the pH-[HC0 3 -] diagram below


indicate states of acid-base imbalance; point N indicates a normal acid-base state. Match each of the
following conditions with the appropriate lettered
point on the diagram.

For each patient described below, select the set


of arterial blood values that coincides with that
patient's acid-base condition.

;J
E

E E
~

:::24

(A)

7.2
7.3
7.3
7.5
7.6

(C)

(D)

-I

c..

pH

(B)

(5
<ll

Patient

(E)

[HC0 3 - ]
[Peo 2 ]
P0 2
(mEq/l) (mm/Hg) (mm/Hg)

20
33
16
30
20

50
60
30
38
20

62
45
105
95
120

42. Patient with chronic respiratory acidosis and


partial renal compensation
7.4

Plasma pH

38.
39.
40.
41.

Hypocapnia
Hypercapnia
Ketoacidosis
NaHC03 ingestion

43. Patient with metabolic acidosis and partial respiratory compensation


44. Patient with uncompensated metabolic alkalosis
45. Patient with respiratory alkalosis and partial
renal compensation
46. Patient with highest dissolved CO 2
47. Patient with highest CO 2 content

ANSWERS AND EXPLANATIONS


1. The answer is D. [I A 2; IV A 2 a] Most of the body's daily CO 2 production occurs from chemical
reactions of the tricarboxylic acid cycle. From this metabolic activity, humans produce about 13,000
mmol of CO 2 daily, or, in acid-base terms, about 13,000 mEq of H+ per day. The mammalian body
produces large amounts of acids from two major sources. The volatile acid H 2 CO) is produced from
CO 2 , the end-product of oxidative metabolism. A variety of nonvolatile acids (e.g., H 2 S0 4 , H)P04 ) are
produced from dietary substances.

2. The answer is E. [III A 2 b, B 3, C 1] Carbonate is a major buffer system in bone and, therefore, is
not an intracellular buffer; bone contains approximately 35,000 mEq of carbonate that contributes to
the buffering of acid and base loads. The intracellular fluid (lCF) does not have a high concentration of
the bicarbonate (HCO) -) buffer system but does contain significant amounts of the non-HCO) - buffer
systems. Intracellular protein, with its histidine residues, and organic phosphate are the quantitatively
significant non-HCO) - buffer systems. Thus, the ICF functions to buffer both noncarbonic and carbonic
acids well. The red blood cell compartment, by virtue of its hemoglobin content, is regarded in the
physiologic context of body buffers as a part of the extracellular buffer system, although it is clearly
intracellular. Hemoglobin is quantitatively more important than the erythrocyte HCO) - buffer.

3. The answer is B. [I A] Carbon dioxide (C0 2 ) is a hydrogen ion (H+) generator. It is evident that the
total reservoir of carbonic acid (H 2 CO)) [i.e., both dissolved CO 2 and H 2 CO) termed the total "carbonic
acid pool"] should be considered the acid component of the HCO) - buffer system, and HCO) - the
conjugate base. The HCO) - IC0 2 buffer system is the most important ECF buffer pair. CO 2 , which is
not an acid, increases the acidity of a solution through the formation and dissociation of H 2 CO). Students
often are confused by the following series of reactions
CO 2 + H 2 0 -. H 2 CO) -. H+ + HCO)because both H+ and HCO) - are formed simultaneously, and there should be no change in pH.
However, the HCO) - concentration is several orders of magnitude (6 x 1 0 5 ) greater than the H+
concentration and is correspondingly less affected by any change in the concentration of CO 2 or of
H2 CO).

4. The answer is D. [IV B 3; VB 3] Extracellular HCO) - is not an effective buffer for H 2CO). The
HCO) - buffer system plays no role in the buffering of H 2 CO), but it is an effective buffer for noncarbonic
acids. HCO) - cannot buffer H 2 CO), because the combination of H+ with HCO) - results in regeneration
of H 2 CO) as
H 2 CO) + HCO) - ++ HCO) - + H 2 CO)
Most buffering of H 2 CO) occurs within the erythrocytes by hemoglobin. In contrast, the non-HCO)buffer systems (hemoglobin, protein, phosphate) can buffer both noncarbonic acids and H 2 CO). The
HCO) - IC0 2 buffer system accounts for 97%-98% of the buffering in the extracellular fluid (ECF), which
includes the interstitial fluid, lymph, and cerebrospinal fluid (CSF). A stipulation here is that HCO) - is a
major buffer for noncarbonic acids.
5. The answer is A. [II C 2] The [HCO)-] of CSF and arterial blood are similar (about 24 mmol/U;
however, the arterial pH (7.4) is higher than the pH of CSF (about 7.32), because the Peo 2 of CSF is
about 48 mm Hg compared to 40 mm Hg in arterial blood. Acute metabolic alkalosis depresses ventilation, increasing the Peo 2 of the blood and CSF, resulting in CSF acidosis and blood alkalosis. Thus, in
metabolic acid-base disturbances the [H+] of CSF and blood change in opposite directions, and in
respiratory acid-base disturbances the [H+] of CSF and blood change in the same direction.
6. The answer is C. [VIII 0 1, E 3; Table 5-8] Metabolic acidosis is an acid-base disturbance characterized by a decreased arterial pH (or increased [H+]), a decreased plasma [HCO) -], and a compensatory
hyperventilation resulting in a decreased arterial Peo 2
7. The answer is A. [VIII 0 4, E 2; Table 5-8] Respiratory alkalosis is an acid-base disturbance characterized by an increased arterial pH (or decreased [H+]), a decreased Peo 2 (hypocapnia), and a variable
reduction in arterial [HCO) -] due to renal compensation.
8. The answer is B. [VII 0 2 b; VIII 0 2 b; Figure 5-13] The hallmarks of metabolic alkalosis are elevated
[HCO) -], which is the primary event, and elevated Peo 2 , which is the compensatory event. Also, the

[H+] decreases and the [HC0 3 -] increases. The only other patient set of values showing an increased
[HC0 3 -]-set D-does not exhibit an increase in Peo 2
9. The answer is C. [VI D 2, E 2 b, 3; VIII D 1 b; Table 5-7] The kidney responds to an increased acid
load by augmenting renal ammonia (NH 3 ) production and, consequently, ammonium (NH 4+) excretion.
There is a smaller increase in the excretion of titratable acid (primarily in the form of H 2 P04-). Thus,
the decreased pH and decreased HC03 - excretion and increased H + excretion (in the form of N H4 +
and H 2 P0 4-) are consistent with metabolic acidosis.

10. The answer is D. [1/ D 3, E 2; Figure 5-13] From the data given, this patient's arterial Peo 2 is
determined to be 38 mmol/L, which is close to normal (40 mmol/U. The Henderson equation is used
to calculate Peo 2 The equation typically is expressed as
Peo
[H+] = 24
2
[HC0 3 -]
Note that when [HC0 3 -] is low, as in this patient, a change in Peo 2 will have a greater effect on [H+]
than when [HC0 3 -] is normal or high.
To determine Peo 2 , the Henderson equation is rearranged as
[H+] [HC0 3 -]
Peo 2 = -=----=--=----~
24
First, the [H+] is calculated from the pH value as
[H+] = antilog (9 - pH)
= antilog (9 - 7.22)
= antilog (1.78)
= 60 nmol/L
Then, Peo 2 can be determined as
[H+] [HC0 3 - ]
Peo 2 = - - - - 24
(60) (15)

38 mm Hg
24
Thus, the Peo 2 of this patient is within normal limits and, therefore, respiratory compensation has not
occurred. The acid-base disturbance of this patient is attributable to a hyperchloremic metabolic acidosis.
Notice that, in this type of acidosis, the anion gap (AG) is normal, as
AG = [Na+] - [HC0 3 - + CI-]
= 140 mEq/L - (15 mEq/L + 113 mEq/U
= 140 mEq/L - 128 mEq/L = 12 mEq/L
The pH of the CSF in metabolic acidosis usually is elevated because of the compensatory hyperventilatory
response to the metabolic acidosis. In this patient, there is no evidence of significant hyperventilation
and, therefore, the pH of the CSF would not increase significantly. The axiom to remember is that
metabolic acid-base alterations evoke opposite changes in the pH of the CSF and blood, while respiratory
acid-base disturbances bring about parallel changes in the pH of CSF and blood.
--- =

11. The answer is D. [1/ E 2 b] From the data given, this patient's arterial [H+] is determined to be 32.4
nmol/L. The Henderson equation is used in this case, which is
[H+]

24

Peo
2

[HC0 3 -]

It is necessary to keep in mind that the units for these factors are: [H+] (nmol/U, [HC0 3 -] (mmol/U,
and Peo 2 (mm Hg).
Substituting,
[H+]

30
2422.2

32.4 nmol/L

This patient has a partially compensated respiratory alkalosis. Note that her arterial [H+] and [HC0 3 -]
exhibit a parallel decrease.

12. The answer is C. [/I 0 3, E 2; VIII 03] This patient has an acidosis caused by ca 2 retention (Le.,
respiratory acidosis). The condition is partially compensated by renal retention of HCa3 - . The [HCa 3 -]
can be determined using either the Henderson-Hasselbalch equation or the Henderson equation.
Using the Henderson-Hasselbalch equation, the [HCa 3 -] is determined as
pH

[HCa 3 -]
pK + l o g - - -

Peo 2
[HCa

-]

3
---=----=--=--

7.32

6.1 + log

7.32

6.1 + log [HCa 3 -]

0.03 x 68 mm Hg
-

log 2.04

log [HCa 3 -] = 7.32 - 6.1 + 0.31

antilog 1.53

1.53

33.9 mmol/L

Using the Henderson equation, the [Hea 3 -] is determined as


[Hea 3 -]

Peo 2 (mm Hg)


24 [H+] (nmol/L)

68 mm Hg
24----=47.9 nmol/L

34.1 mmol/L

The patient's hypoxemia could be due to a pulmonary diffusion barrier. The cyanosis is caused by a
greater than normal concentration of reduced hemoglobin (greater than 5 g/dl of arterial blood in the
deoxy state). The leg edema could be due to the increased retention of Hea 3 - and Na+ resulting in an
increase in plasma volume. The hypoxemia causes polycythemia and constriction of vascular smooth
muscle of the pulmonary circulation, leading to pulmonary hypertension, elevated right atrial pressure,
and increased capillary pressure. The increase in capillary pressure causeS fluid movement into the
interstitial space and, ultimately, edema.
13. The answer is B. [/I 0 3 a, E 2 b] From the data given, this patient's blood pH is determined to be
7.31. To determine the arterial pH, it is first necessary to calculate the arterial [Hea 3 -] using the
Henderson equation, as
[Hea 3 -]

Peo 2
24 [H+]

30
2449

14.7 mmol/L

Then, applying the Henderson-Hasselbalch equation, the arterial pH is determined as


pH

[Hea 3 -]

pK +

14.7 mmol/L
6.1 + log - - - - 0.9 mmol/L

6.1 + log 16.3

6.1 + 1.21

7.31

log~-~
S X Peo 2

This patient has a partially compensated metabolic acidosis. It is important to note that the [HCa 3 -]I
S x Peo 2 ratio is equal to 16.3, indicating a significant respiratory response (hyperventilation) to the
metabolic problem. Thus, this metabolic acidosis is alleviated by a response that involves the respiratory
system.

14-15. The answers are: 14-C, 15-8. [VII 02 a; VIII 0 1 b] The acid-base abnormality is metabolic
acidosis without respiratory compensation. Since the respiratory response to metabolic acidosis is hyperpnea, hypocapnia would be expected, which is not observed in this case. Moreover, the imbalance
cannot be attributed to respiratory acidosis, which would be characterized by an elevated arterial Pea 2
It is apparent that a mechanical respirator maintained the animal's Pea 2 at a constant level.
The pH of arterial blood is a function of two variables: [HCO l -] and Pea 2 . A compensatory response
occurs in the alternate variable and in the same direction as the primary abnormality. In this case, the
decline in [HCO l - ] (i.e., metabolic acidosis) would be associated with a parallel decline in Pea 2 , the
alternate variable. To bring this about, there would be a hyperventilatory response and a resultant decline
in Pea 2

16-19. The answers are: 16-8, 17-D, 18-8, 19-C. [VII A 5 C, 0 1 a; VIII 04, E 2; Figure 5-8; Table
5-9] The blood findings indicate that this patient has a respiratory alkalosis, an acid-base disturbance
characterized by increased arterial pH (or decreased [H+]), decreased arterial Pea 2 (hypocapnia), and
decreased plasma [HCO l -]. It should be noted that both [H+] and [HCO l - ] are decreased in this
patient, which is consistent with the axiom that [H+] and [HCO l - ] change in the same direction in
respiratory acid-base imbalances. The decline in [HCO l - ] indicates that renal compensation has begun.
In alkalotic states, the [HCO l -]IS x Pea 2 ratio exceeds the normal 20:1, due to either an increase in
[HCO l -] (metabolic alkalosis) or a decrease in Pea 2 (respiratory alkalosis). The normal ratio of 20:1 is
derived as
24 mmol/L

24 mmol/L

20

Pea 2 0.03 x 40 mm Hg
1.2 mmol/L
In this alkalotic patient, the [HCO l -]IS x Pea 2 ratio is 30:1. This ratio can be determined by substituting
the patient's blood data into the above equation, as
S

[HCO l - ]

22.5 mmol/L

22.5 mmol/L

S x Pea 2

0.03 x 25 mm Hg

0.75 mmol/L

30

The total CO 2 content for this patient is approximately 23 mmol/L. Total CO 2 equals the sum of all
forms of CO 2 in the blood (i.e., HCOl - , H 2 CO l , dissolved CO 2 , and CO 2 bound to proteins). Dissolved
CO 2 content ([C0 2 ]) is calculated as
[C0 2 ] = 0.03 x Pco 2
= 0.03 x 40 mm Hg
= 1.2 mmol/L
Since [H 2 CO l ] is negligible, the total CO 2 content normally exceeds the [HCO l -] by 1.2 mmol/L and,
therefore, equals 25.2 mmol/L when normal plasma [HCO l - ] and Pea 2 values exist, as
total CO 2 content

=
=
=

[HCO l -] + (S x Pea 2 )
24 mmol/L + 1.2 mmol/L
25.2 mmol/L

Total CO 2 content is decreased in respiratory alkalosis. From this patient's blood data, the CO 2 content
is calculated as
total CO 2 content

=
=

[HCO l -] + S x Pea 2
22.5 mmol/L + 0.75 mmol/L
23.25 mmollL

Respiratory alkalosis decreases the renal reabsorption of HCOl - , causing a transient HCOl - diuresis
and a decline in net acid secretion. Since the major change in respiratory alkalosis is a decrease in arterial
Pea 2 , the compensation will be in the alternate variable (kidney) and in the same direction as the primary
event. Thus, there is a decline in plasma [HCO l -], which is indicative of a partial renal response (i.e.,
increased HCOl - excretion).

20-23. The answers are: 20-A, 21-E, 22-A, 23-8. [lie 2; VIII 01, E 3, F 1; Figures 5-1,5-12,5-13]
The fluid in the small and large intestine is relatively high in HCOl - . Therefore, severe diarrhea is a
cause of metabolic acidosis, which is the likely diagnosis in this patient. Only one of the given arterial
pH values-7.32-is consistent with acidosis.
Increased arterial [H+] stimulates ventilation, lowering the Pea 2 of the blood and CSF, resulting in
blood acidosis and CSF alkalosis. Thus, in metabolic acid-base alterations the pH of the blood and CSF
change in opposite directions, whereas in respiratory acid-base abnormalities the pH of the blood and
CSF change in the same direction. Since the normal pH of CSF is 7.32, the pH would increase in
metabolic acidosis. The only given pH value indicating a CSF alkalosis is 7.40.

Metabolic acidosis is compensated by hyperventilation, which reduces the Pco 2 and thereby attenuates
the reduction in pH. This hyperventilatory response to metabolic acidosis is due to direct stimulation of
the medullary respiratory center and of the peripheral chemoreceptors in the carotid and aortic bodies.
Only one of the given Pco 2 values-32 mm Hg-is associated with hypocarbia.
In the absence of therapy with NaHC03 , the renal compensation for a metabolic acidosis requires an
increased HC03 - reabsorption together with an increased H+ secretion and excretion. This adaptive
response is accomplished by an increased NH4 + excretion with a limited ability to enhance titratable
acidity via H 2 P0 4- excretion. Thus, the urine will exhibit a low pH.

24. The answer is E (all). [I B; 11/ B 1-3; IV B] Noncarbonic acids include all acids other than H 2 C0 3 ,
such as lactic, acetoacetic, ,a-hydroxybutyric, hydrochloric, and sulfuric acids. Some noncarbonic acids
(lactic, acetoacetic, and ,a-hydroxybutyric acids) can be converted to CO 2 and eliminated by ventilation
(the fate of "volatile" acids); however, when present in large amounts, these acids are eliminated by the
kidney (the fate of "nonvolatile" acids). Sulfuric, hydrochloric, and phosphoric acids cannot be catabolized to CO 2 and H 2 0 and, thus, must be eliminated by renal excretion. Noncarbonic acids can be
buffered by both HC03 - and non-HC0 3 - buffer systems. Fixed acid is another name for nonvolatile
acid.

25. The answer is E (all). [I B 2; IV B 1-2] Acids that can be converted to carbonic acid are classified
as volatile acids. In high amounts, many of these volatile acids are excreted by the kidneys without being
converted to CO 2 and H 2 0 and, thus, are classified as nonvolatile acids. Examples of such acids are
citric, isocitric, acetic, and lactic acids and the ketoacids. Acetic acid can increase following the ingestion
of even small amounts of vinegar. All of these acids can be either eliminated by pulmonary ventilation
or excreted by the kidneys. Thus, it is important to remember that nonvolatile (fixed) acids are dissociated
at body pH, and the H+ can be excreted via the urine.
26. The answer is B (1, 3). [II C 2; 11/ B 2 a] In plasma, interstitial fluid, lymph, and CSF the HC03 buffer system is the most important means of buffering noncarbonic acid. The ICF does not have a high
[HC0 3 -], but the intracellular protein and the large amount of organic phosphate compounds are
quantitatively significant non-HC03 - buffer systems. Thus, the ICF can buffer both carbonic and noncarbonic acids. Additionally, noncarbonic acids are buffered in the erythrocytes by hemoglobin. It should
be noted that the [HC0 3 - ] in plasma and CSF are similar.

27. The answer is D (4). [VII A 5 b; Figure 5-7; Table 5-9] The total CO 2 content is mainly a function
of HC0 3 - and dissolved CO 2 , Therefore, CO 2 content is elevated in metabolic alkalosis and respiratory
acidosis. Conversely, CO 2 content declines with decreases in [HC0 3 - ] and [C0 2 ], conditions that are
associated with metabolic acidosis and respiratory alkalosis, respectively. Note that [C0 2 ] denotes the
concentration of dissolved CO 2 and is calculated as
[C0 2 ]

=
=

S X Pco 2
0.03 x 40 mm Hg
1.2 mmol/L (normal)

28. The answer is C (2, 4). [VII A 5 a-b; Table 5-9] The changes in arterial [HC0 3 -] and Pco 2 during
various acid-base disturbances are illustrated, here, in terms of the [HC0 3 -liS x Pco 2 ratio:
[HC0 3 -]

..

[HC0 3 -]

..

S X Pco 2 !
metabolic acidosis

S X Pco 2 t
metabolic alkalosis

respiratory acidosis

respiratory alkalosis

The large arrows depict the major changes, and the small arrows depict the compensatory responses.
Changes in the numerator indicate metabolic acid-base imbalances, whereas changes in the denominator
show respiratory acid-base disturbances. Clearly, both Pco 2 and [HC0 3 -] decrease in respiratory alkalosis and metabolic acidosis, and both increase in respiratory acidosis and metabolic alkalosis. Note that
the sum of [HCO) -] and S x Pco 2 equals the total CO 2 content.

29-32. The answers are: 29-8, 30-0, 31-A, 32-C. [VII A 1-3, 0 1-2; VIII F; Figure 5-13; Table 5-8]
In analyzing these four acid-base disorders, it is necessary to consider both the primary abnormality (Le.,
the variable that undergoes the greater degree of change) and the compensatory response (i.e., the
alternate variable, which undergoes a lesser degree of change in the same direction).
In metabolic acidosis, there is a primary decrease in [HC0 3 -] and a compensatory decrease in Peo 2
Only one pair of values (8) shows this pattern. In metabolic alkalosis, there is a primary increase in
[HC0 3 -] and a compensatory increase in Peo 2 Only one pair of values (0) shows this pattern. In
respiratory acidosis, there is a primary increase in Peo 2 and a compensatory increase in [HC0 3 -]. Only
one pair of values (A) shows this pattern. In respiratory alkalosis, there is a primary decrease in Peo 2 and
a compensatory decrease in [HC0 3 -]. Only one pair of values (0 shows this pattern.

33-37. The answers are: 33-0, 34-8, 35-C, 36-A, 37-E. [II 0 3 b, E 2 b; VII A 3 a (2), 5 C, 02; VIII
1, 2, f] Patient 0 has a metabolic alkalosis with partial respiratory compensation. Metabolic alkalosis

can result from ingestion of alkaline substances (e.g., antacids) or from loss of acid (as occurs in vomiting).
The total CO 2 content also is increased in metabolic alkalosis. The compensation for this condition is
hypoventilation or excretion of an alkaline urine. Note that patient 0 shows partial compensation, as
evidenced by the increased Peo 2 The [HC0 3 -liS x Peo 2 ratio and pH are increased.
Patient 8 has a metabolic acidosis with partial respiratory compensation. Metabolic acidosis can result
from increased fixed (noncarbonic) acid production (as occurs in diabetic ketoacidosis), increased acid
retention (as occurs in renal failure with subsequent accumulation of H 2 P0 4-), or loss of base (as occurs
in diarrhea). The total CO 2 content also is decreased in metabolic acidosis. The compensation for this
condition is hyperventilation or excretion of an acid urine. Note that patient B shows partial compensation, as evidenced by the decrease in Peo 2 The [HC0 3 -liS x Peo 2 ratio and pH are decreased.
Patient C has the highest plasma CO 2 content (Le., [HC0 3 - ] + 5 x Peo 2 ). This patient has respiratory
acidosis, a condition caused by CO 2 retention resulting from hypoventilation, which may be due to
pulmonary disease. The compensation for this condition is an increase in both HC0 3 - reabsorption and
in H+ excretion in the form of H 2 P0 4- and NH4 +. Note that patient Cshows nearly complete compensation, as evidenced by the increased [HC0 3 -] and return of pH toward normal. The [HC0 3 -]/5 x Peo 2
ratio is only slightly decreased (i.e., 18: 1 versus the normal 20: 1).
Patient A has the lowest pH and, thus, the highest [H+]. This patient has metabolic acidosis without
compensation. [H+] can be calculated as
[H+]

=
=
=

antilog (9 - pH)
antilog (9 - 7)
antilog 2
100 nmol/L

or estimated as
[H+] = 24

Pea
2

[HC0 3 - ]

70
2416

24 (4.38)

105 nmol/L
Patient E has the highest [HC0 3 -liS x Peo 2 ratio (Le., 29.5:1). This patient has respiratory alkalosis,
=

a condition caused by loss of CO 2 due to anxiety or hysteria leading to hyperventilation. The compensation for respiratory alkalosis is excretion of an alkaline urine. This patient shows partial renal compensation, as evidenced by the slight decline [HC0 3 -]. Patient E has a slightly lower than normal CO 2 content.

38-41. The answers are: 38-0, 39-C, 40-A, 41-8. [VII 0 1-2; VIII 0; Figure 5-13; Table 5-8] In
analyzing these four uncompensated acid-base disturbances, it is important to consider the Peo 2 and
[HC0 3 - ] rather than the pH, as Peo 2 and [HC0 3 - ] are the key determinants of the cause of, and
compensation for, these disturbances.
Point 0 represents a patient with increased pH U[H+]) brought about by respiratory alkalosis. This
condition is characterized by a primary decrease in Peo 2 (hypocapnia) and a variable secondary decrease
in plasma [HC0 3 -]. Metabolic acidosis also is characterized by declines in these two variables, but the
pH is decreased (t[H+]) as well. Respiratory alkalosis is defined as alveolar ventilation in excess of the
existing need of the body to eliminate CO 2 This excess in alveolar ventilation, called hyperventilation,
results in a reduced arterial Peo 2 Hyperpnea is the general term used to describe any increase in
ventilatory effort. With respiratory alkalosis there is a decline in the [total CO 2 ].

Point C represents a patient with decreased pH (t[H+]) caused by respiratory acidosis. This clinical
disorder is characterized by a primary increase in Pea 2 (hypercapnia) and a variable secondary increase
in plasma [HC0 3 -]. The common denominator in respiratory acidosis is hypoventilation, which is
defined as alveolar ventilation insufficient to excrete CO 2 rapidly enough to meet the existing needs of
the body. With respiratory acidosis there is a relatively small increment in the [total CO 2 ], because the
major fraction of the CO 2 content is comprised of HC03 -
Point A represents a patient with diabetes mellitus, which is the most common cause of ketoacidosis.
This overproduction of ketoacids is caused by a deficiency of insulin, which leads to: (1) increased
lipolysis and an increased delivery of free fatty acids to the liver and (2) the preferential conversion of
free fatty acids to ketoacids rather than to triglycerides. Thus, metabolic acidosis is characterized by a
low arterial pH (f[H+]), a reduced [HC0 3 -], and a compensatory hyperventilation resulting in hypocapnia. The renal compensatory response for respiratory alkalosis also diminishes the plasma [HC0 3 -], but
the pH in that disorder is elevated U[H+]). Overproduction of ketoacids causes acidosis by two mechanisms: (1) a decrease in plasma [HC0 3 -] with an increase in the anion gap and (2) overloading of the
renal capacity to excrete H+ resulting in a loss of Na+ and a failure to recover NaHC0 3 In metabolic
acidosis, there is a decline in the [total CO 2 ], Furthermore, ketoacidosis, like lactic acidosis, differs from
other forms of metabolic acidosis in that the anion associated with H+ can be metabolized back to
HC03 - , as
,8-hydroxybutyrate- + O 2

....

CO 2 + H 2 0 + HC03 -

,8-hydroxybutyrate represents about 75% of the circulating ketoacids in diabetic ketoacidosis. It can
be seen from the above chemical equation that the metabolism of the ,8-hydroxybutyrate anion results
in the regeneration of the HC03 - that was neutralized in buffering the H +. Since the HC03 - is replaced
by an anion that is metabolized back to HC0 3 - , there is no actual loss of HC0 3 - from the body in
ketoacidosis (or lactic acidosis). Insulin administration decreases the accumulation of ,8-hydroxybutyric
acid and allows the metabolism of the acid ions back to HC03 - .
Point 8 represents a patient with metabolic alkalosis. Excessive ingestion of NaHC03 can result in
metabolic alkalosis and an increase of pH U[H+]). Metabolic alkalosis is characterized by an increase
in the plasma [HC0 3 - ] and a compensatory increase in the Pea 2 produced by a decline in alveolar
ventilation. Since elevation of plasma [HC0 3 -] can be due to the renal compensation for chronic
respiratory acidosis, the diagnosis of metabolic alkalosis cannot be made without measuring the pH.
Metabolic alkalosis is associated with a large increase in the [total CO 2 ],

42-47. The answers are: 42-8, 43-C, 44-0, 45-E, 46-8, 47-8. [II 02 b, c; VII A 5 c, 0 1-2; VIII 0F; Figures 5-12, 5-13] Patient 8 has respiratory acidosis with partial renal compensation. Respiratory
acidosis is caused by increased Pea 2 , which reduces the [HC0 3 -]IS x Pea 2 ratio and, thus, the pH.
Whenever Pea 2 rises, [HC0 3 -] also must increase somewhat because of the dissociation of H 2 C0 3
formed by the hydration of CO 2 , The kidney responds by conserving [HC0 3 -] via increased H+
secretion, with acid excreted as H 2 P0 4- and NH4+'
Patient C has metabolic acidosis with partial respiratory compensation. Metabolic acidosis means a
primary decrease in [HC0 3 -], which decreases the [HC0 3 -]/S x Pea 2 ratio and, thus, the pH. The
[HC0 3 -] may be lowered by the addition of H+ or by the loss of HC03 - . Respiratory compensation
occurs by increased ventilation via the action of H+ in the peripheral and central chemoreceptors.
Patient 0 has uncompensated metabolic alkalosis. In this condition, the increase in [HC0 3 -] causes
an increase in the [HC0 3 -]lS x Pea 2 ratio and, thus, the pH. Respiratory compensation occurs by a
reduction in alveolar ventilation, which tends to raise Pea 2 (not evident in patient 0).
Patient E has respiratory alkalosis with partial renal compensation. Respiratory alkalosis is caused by
a decrease in Pea 2 , which increases the [HC0 3 -]IS x Pea 2 ratio and, thus, the pH. Renal compensation
occurs by increased HC03 - excretion.
Patient 8 has both the highest amount of dissolved CO 2 and the highest total CO 2 content. Dissolved
CO 2 is determined by multiplying pea 2 (in mm Hg) by the Pea 2 solubility constant, S, as
dissolved CO 2 = S X Pea 2
= 0.03 x 60
= 1.8 mmol/L
Total CO 2 content is the sum of [HC0 3 - ] and dissolved CO 2 , It is calculated in patient 8 as
total CO 2 content

[HC0 3 -] + S x Pea 2
33 + 0.03 x 60

33 + 1.8

34.8 mmol/L

6
Gastrointestinal Physiology
Michael B. Wang

I. DIGESTIVE SYSTEM: OVERVIEW

A. Structure. The digestive system is composed of a long muscular tube-the gastrointestinal (GI)
tract, or alimentary canal-and a set of accessory organs (Figure 6-1).

1. The GI tract consists of the oral cavity, pharynx, esophagus, stomach, small intestine, large
intestine, rectum, and anal canal.

2. The accessory organs include the tongue, teeth, salivary glands, pancreas, liver, and gallbladder.

B. Function. The digestive system is responsible for breaking down food and supplying the body
with the water, nutrients, and electrolytes needed to sustain life. Before food can be used by the
body, it must be ingested, digested, and absorbed-processes that involve coordinated movement
of muscle and secretion of various substances.

1. Ingestion involves:
a. Placing food into the mouth
b. Chewing the food into smaller pieces (mastication)
c. Moistening the food with salivary secretions
d. Swallowing the food (deglutition)

2. Digestion. During digestion, food is broken down into small particles by the grinding action
of the GI tract and then degraded by digestive enzymes into usable nutrients.
a. Starches are degraded by amylases into monosaccharides.
b. Proteins are degraded by a variety of enzymes (e.g., pepsin, trypsin) into dipeptides and
amino acids.
c. Fats are degraded by lipases and esterases into monoglycerides and free fatty acids.

3. Absorption. During absorption, nutrients, water, and electrolytes are transported from the GI
tract (principally from the small intestine) to the circulation .

.,-II. THE ORAL CAVITY, PHARYNX, AND ESOPHAGUS are involved in the chewing and swallowing, or ingestion, of food. Saliva helps ready the food to be moved from the mouth to the
esophagus, and peristalsis is the muscle pattern primarily responsible for moving the food through
the esophagus.

A. Mastication. After food is placed into the mouth, it is cut and ground into smaller pieces by
chewing (mastication).

1. The chewing reflex. Although chewing is a voluntary act, it is coordinated by reflex centers
in the brain stem that facilitate the opening and closing of the jaw.
a. When the mouth opens, stretch receptors in the jaw muscles initiate a reflex contraction of
the masseter, medial pterygoid, and temporal is muscles, causing the mouth to close.
b. When the mouth closes, food comes into contact with buccal receptors eliCiting a reflex
contraction of the digastric and lateral pterygoid muscles, causing the mouth to open.
c. When the jaw drops, the stretch reflex causes the entire cycle to be repeated.

Mouth

~~~:::::::=jf'_ Salivary
glands

<::

"'\
t

_~_~Stomach

-=::::;"::::""-';'---1'"""-

Duodenum--~~~~

Pancreas

2J.~p+-r-Jejunum

Ascending
colon

Figure 6-1. The gastrointestinal (GI)


tract.

d. The tongue contributes to the grinding process by positioning the food between the upper
and lower teeth.

2. Function of mastication
a. Chewing breaks food into smaller pieces, which:
(1) Makes it easier for the food to be swallowed
(2) Breaks off the undigestible cellulose coatings of fruits and vegetables
(3) Increases the surface area of the food particles, making it easier for them to be digested
by the digestive enzymes
b. Chewing mixes the food with salivary gland secretions (see II B), which:
(1) Initiates the process of starch digestion by salivary amylase
(2) Initiates the process of lipid digestion by lingual lipase.
(3) Lubricates and softens the bolus of food, making it easier to swallow
c. Chewing brings food into contact with taste receptors and releases odors that stimulate
the olfactory receptors. The sensations generated by these receptors increase the pleasure
of eating and initiate gastric secretions.
B. Saliva
1. Salivary glands. Saliva is secreted primarily by three pairs of glands.
a. The parotid glands, located near the angle of the jaw, are the largest glands. They secrete
a watery fluid.
b. The submandibularand sublingual glands secrete a fluid that contains a higher concentration of proteins and so is more viscous.

c. Smaller glands are located throughout the oral cavity. Those in the tongue secrete lingual
lipase.
2. Composition of saliva. The salivary glands secrete a relatively high volume of fluid (0.5-1
L/day) containing electrolytes and proteins.
a. Electrolytes. Electrolyte concentration and osmolality both vary with secretory flow rates,
but generally, in comparison to plasma, saliva is hypotonic and contains higher concentrations of potassium (K+) and bicarbonate (HCO l -) and lower concentrations of sodium (Na+)
and chloride (0-).
b. Proteins. Two types of proteins are found in saliva.
(1) The enzymes, a-amylase (ptyalin) and lingual lipase, begin the process of starch and
fat digestion.
(2) Mucin is a glycoprotein that lubricates the food.
3. Control of salivary secretion
a. Salivary secretion is controlled entirely by autonomic nervous system reflexes.
(1) Parasympathetic nerve stimulation causes the salivary gland cells to secrete a large
volume of watery fluid that is high in electrolytes but low in proteins.
(2) Sympathetic nerve stimulation causes the salivary glands to secrete a small volume
of fluid containing a high concentration of mucus.
b. Salivary reflexes are elicited by the thought, aroma, or taste of food or by the presence of
food within the alimentary canal.
c. Salivary gland metabolism and growth are both stimulated by increased autonomic nervous system activity.
4. Functions of saliva. Salivary secretions perform a number of important functions.
a. Protection. Salivary secretions protect the mouth by:
(1) Cooling hot foods
(2) Diluting any hydrochloric acid (HC!) or bile regurgitated into the mouth
(3) Washing food away from the teeth and destroying harmful bacteria within the mouth
b. Digestion. Salivary secretions begin the process of starch and fat digestion.
(1) a-Amylase can digest most of the ingested starches into disaccharides before they
reach the small intestine. a-Amylase is ultimately inactivated by the low pH of the
stomach.
(2) Lingual lipase begins to break down ingested fats while they are in the mouth, stomach,
and upper portions of the small intestine.
c. Lubrication. Salivary secretions lubricate the food, making swallowing easier, and moisten
the mouth, facilitating speech.

C. Peristalsis
1. Definition. Peristalsis is a coordinated pattern of smooth muscle contraction and relaxation.
2. Function. Peristalsis helps move food through the pharynx and esophagus and within the
stomach. Peristalsis plays a minor role in propelling food through the intestine.
3. Mechanics. During peristalsis, contraction of a small section of proximal muscle is followed
immediately by relaxation of the muscle just distal to it. The resulting wavelike motion moves
food along the GI tract in a proximal (orad) to distal (caudad) direction (see also II D 1 c; III B
3, 6 c; IV B 2 b; V A 2 b).
D. Deglutition (swallowing)
1. Phases of swallowing
a. Oral (voluntary) phase. During the voluntary phase, the tongue forms a bolus of food and
forces it into the oropharynx by pushing up and back against the hard palate.
b. Pharyngeal phase
(1) The pharyngeal phase is coordinated by a swallowing center in the medulla and lower
pons. It begins when the food reaches the oropharynx and progresses as follows.
(a) The nasopharynx is closed by the soft palate, preventing regurgitation of food into
the nasal cavities.
(b) The palatopharyngeal folds are pulled medially, forming a passageway for the food
to move into the pharynx.
(c) The glottis and vocal cords are closed and the epiglottis swings down over the
larynx, guiding the food toward the esophagus and away from the airways.

(d) The bolus of food is pushed into the esophagus by the peristaltic contractions of the
pharynx and the opening of the upper esophageal sphincter.
(2) Respiration is inhibited for the duration of the pharyngeal phase of swallowing (1-2
seconds).
c. Esophageal phase. After reaching the esophagus, food is propelled into the stomach by
peristalsis. The strength of peristaltic contractions is proportional to the size of the bolus
entering the esophagus.
(1) Sphincters involved in esophageal peristalsis. The esophagus is isolated from the oral
cavity by the upper esophageal sphincter and from the stomach by the lower esophageal
sphincter.
(a) The upper esophageal sphincter is composed of striated muscle and is completely
under the control of the vagal fibers innervating the esophagus. Tone is maintained
by the continuous firing of vagal postganglionic neurons. The transmitter released
by these fibers is acetylcholine (ACh).
(b) The lower esophageal sphincter is composed of smooth muscle. Its tone is
maintained by a myogenic process. Intrinsic nervous system neurons (under vagal
control) cause the lower esophageal sphincter to relax during peristalsis. The synaptic
transmitter released by these neurons is neither ACh nor norepinephrine and is thus
called a nonadrenergic noncholinergic neural transmitter. Its identity is not known
but may be either adenosine triphosphate (ATP) or vasoactive intestinal peptide
(VIP).
(2) Types of esophageal peristalsis. There are two types of esophageal peristalsis, primary
and secondary.
(a) Primary esophageal peristalsis is initiated by swallowing. It begins when food
passes into the esophagus from the pharyngeal cavity.
(i) As soon as the food enters the esophagus, the upper esophageal sphincter
contracts to prevent regurgitation of food into the mouth.
(ii) The peristaltic wave travels rather slowly (3-4 cm/sec), taking about 9 seconds
to push the food from mouth to stomach (a distance of about 30 cm). The force
of gravity causes liquids to fall through the esophagus at a much faster rate.
(iii) After food enters the stomach, the lower esophageal sphincter contracts to
prevent regurgitation of food into the esophagus.
(iv) If swallowing is not accompanied by the passing of food into the esophagus the
ensuing peristaltic wave will be very weak or may not occur at all.
(b) Secondary peristalsis is initiated by the presence of food within the esophagus.
(i) After primary peristalsis is completed, any food remaining in the esophagus
stretches mechanical receptors, initiating another peristaltic wave.
(ii) Secondary peristaltic waves continue until all of the swallowed food is removed
from the esophagus.
(3) Coordination of esophageal peristalsis
(a) Primary esophageal peristalsis is coordinated by vagal fibers emerging from the
swallowing center within the medulla that are activated as part of the swallowing
reflex. Vagotomy, which eliminates the efferent fibers emerging from the swallowing center, would prevent the initiation of primary esophageal peristalsis.
(b) Secondary esophageal peristalsis is coordinated by the intrinsic nervous system of the esophagus. Afferent fibers innervate stretch receptors within the wall of
the esophagus and thereby activate the appropriate fibers in the intrinsic nervous
system. Since intrinsic rather than vagal nerves are involved, vagotomy would
reduce the intensity of secondary esophageal peristalsis but would not prevent it
from occurring.
2. Disorders of swallowing
a. Esophageal reflux may occur if the intragastric pressure rises high enough to force the
lower esophageal sphincter open, if the lower esophageal sphincter is unable to maintain
its normal tone, or if the lower esophageal sphincter is forced through the diaphragm and
into the thoracic cavity.
(1) During pregnancy, the growing fetus may push the top of the stomach into the thorax.
The low intrathoracic pressure (compared to the higher intra-abdominal pressure) caus~s
the lower esophageal sphincter to expand, allowing reflux to occur.
(2) Reflux of stomach acid causes esophageal pain (heartburn) and may lead to esophagitis.

b. Belching (eructation). Following a heavy meal or the ingestion of large amounts of gas
(e.g., from carbonated beverages), the gas bubble that is usually in the fundus of the stomach
is displaced to the cardia (see Figure 6-2). When the lower esophageal sphincter relaxes
during the swallowing process, gas enters the esophagus and is regurgitated.
c. Achalasia is a neuromuscular disorder of the lower two-thirds of the esophagus that leads
to absence of peristalsis and failure of the lower esophageal sphincter to relax. Food accumulates above this sphincter, taking hours to enter the stomach and dilating the esophagus.

III.

THE STOMACH. Complex patterns of motility move food through the stomach where it can be
broken down further by gastric secretions and then propelled into the small intestine. Only small
amounts of food are digested or absorbed in the stomach.

A. Anatomy (Figure 6-2)


1. Functional components
a. The three functional parts of the stomach are the fundus, corpus (body), and antrum.
Gastric contents are isolated from other parts of the GI tract by the lower esophageal
sphincter proximally and by the pylorus (pyloric sphincter) distally.
b. The antrum and pylorus are anatomically continuous and respond to nervous control as a
unit.

2. Musculature
a. As elsewhere in the gut, each muscle layer in the stomach forms a functional syncytium and
therefore acts as a unit. In the fundus, where the layers are relatively thin, strength of
contraction is weak; in the antrum, where the muscle layers are thick, strength of contraction
is greater.
b. The stomach and duodenum (the uppermost part of the small intestine) are divided by a
thickened muscle layer called the pyloric sphincter.

3. Innervation
a. Intrinsic. The interconnected myenteric (Auerbach's) plexus and submucosal (Meissner's) plexus within the stomach wall comprise the intrinsic innervation of the stomach, as
they do elsewhere in the gut. They are directly responsible for peristalsis and other
contractions. Because this system is continuous between the stomach and duodenum (see
Figure 6-4), peristalsis in the antrum influences the duodenal bulb.
(1) The myenteric plexus is located between the layers of the circular and longitudinal
muscles of the stomach.
(2) The submucosal plexus is located between the layers of the circular muscle and mucosa
on the luminal surface of the stomach.

Cardia-"""'r-

Greater
curvature
Antrum

Figure 6-2. The stomach.

b. Extrinsic. Autonomic innervation is dual-both sympathetic, via the celiac plexus, and
parasympathetic, via the vagus nerve. Sympathetic innervation inhibits motility, and parasympathetic innervation stimulates motility. Together, the two systems modify the coordinated motor activity that arises independently in the intrinsic system.
B. Motility
1. Function. Gastric motility serves three basic functions.
a. Storage. When food enters the stomach, the orad region-primarily the fundus-enlarges
to accommodate the food by a process referred to as receptive relaxation.
b. Mixing. The presence of food in the caudad stomach-primarily the corpus and antrumincreases the contractile activity of the stomach.
(1) The enhanced contractile activity (a combination of peristalsis and retropulsion) mixes
the food with stomach acid and enzymes, breaking it into smaller and smaller pieces.
(2) When the food is mixed into a pasty consistency, it is called chyme.
c. Emptying. When the chyme is broken down into small enough particles, it is propelled
through the pyloric sphincter into the intestine.

2. Receptive relaxation. When food is passed from the esophagus to the stomach, the contractile
activity of the fundus is inhibited, enabling it to easily accommodate 1-2 L of food.
a. Innervation. Vagal nerve fibers innervate the intrinsic (enteric) nervous system of the
orad stomach to cause receptive relaxation. During this process, postganglionic fibers within
the enteric nervous system release a noncholinergic nonadrenergic transmitter whose identity is unknown but may be ATP or VIP.
b. Initiation. Receptive relaxation may be initiated as part of the peristaltic process causing
swallowing and esophageal motility or in response to a bolus of food entering the stomach.
Stretch receptors in the orad stomach detect the presence of food and initiate a vago-vagal
reflex producing receptive relaxation.
c. Effects of vagotomy. Sectioning the vagus nerve will prevent or greatly diminish receptive
relaxation since the process is controlled by the enteric nervous system.
3. Peristalsis. Peristaltic contractions are initiated near the fundal-corpus border and proceed
caudally, producing a peristaltic wave that propels the food towards the pylorus.
a. Mechanics of peristalsis. Peristaltic contractions are produced by periodic changes in
membrane potential, called slow waves, or the basic electrical rhythm (BER) [Figure 6-3].
These waves are responsible for the rhythm and force of gastric contractions.
(1) Gastric slow waves originate in a pacemaker within the longitudinal muscle high on the
greater curvature of the orad corpus.
(2) Slow waves consist of an upstroke and plateau phase and occur at a rate of approximately 3-4/min.
(3) The velocity of the waves is 1 cm/sec when they sweep over the corpus and increases
to 3-4 cm/sec in the antrum.
A

I
5

I
0

h
I
10

I
15

I
20

I
25

I
30

I
35

I
40

h
I
45

I
50

I
55

Em (mV)
-40

lao

I
60

Time (sec)

Em (mV)

I
0

10

15

20

25

30

35

40

45

50

55

60

Time (sec)

Figure 6-3. Basic electrical rhythm (BER), or slow waves, as recorded from smooth muscle cells of (A) the stomach
and (B) the middle of the intestine. The slow waves of the stomach are 5-7 seconds in duration and occur at a rate
of 3-5 waves/min, while the slow waves of the intestine are more frequent (12 waves/min in the duodenum and 8
waves/min in the ileum) and have spikes superimposed on their plateaus.

(4) Although the electrophysiological basis for the slow waves is not entirely known, it is
assumed that the upstroke is due to the flow of Na+ and calcium (Ca H ) into the cell
and that the plateau is dependent primarily on the flow of Ca H into the cell.
b. Force of peristalsis. The force of peristaltic contractions is regulated by gastrin and ACh.
These hormones:
(1) Increase the size of the slow wave plateau potential, which increases the amount
of CaH entering the cell from the extracellular fluid
(2) Activate second messengers that release CaH from the sarcoplasmic reticulum
4. Retropulsion. Retropulsion is the back and forth movement of the chyme caused by the
forceful propulsion of food against the closed pyloric sphincter (Figure 6-4).
a. The wave of peristaltic contraction reaches the pyloric sphincter before the chyme does.
Thus, when the chyme reaches the sphincter, it is pushed back into the body of the stomach.
b. The forward and backward movement of the chyme (caused by peristalsis and retropulsion)
breaks the chyme into smaller and smaller pieces and mixes it with the gastric secretions
present within the stomach.
S. Gastric emptying occurs when the chyme is decomposed into small enough pieces to fit
through the pyloric sphincter.
a. Each time the chyme is pushed against the pyloric sphincter, a small amount (2-7 mls) may
escape into the duodenum.
b. The amount of chyme passing through the pylorus depends on the size of the particles. If
the particles are too large, none of the chyme will enter the duodenum.
c. Thus, the rate of gastric emptying of solids depends on the rate at which the chyme is broken
down into small particles.
d. Liquids empty much faster than solids. The rate of liquids emptying is proportional to pressure
within the orad stomach, which increases slowly during the digestive period.
6. Regulation of gastric emptying
a. Local reflexes
(1) Excitatory reflexes, initiated by expansion of the antrum and by the digestive products
of food, are responsible for increasing gastric motility. Although these reflexes do not
require the vagus nerve, vagotomy decreases the magnitude and coordination of stomach contractions.
(2) Inhibitory reflexes. A variety of stimuli act on the duodenum to initiate enterogastric
reflexes that slow the rate of gastric emptying.
(a) Purpose. Enterogastric reflexes prevent the flow of chyme from exceeding the
ability of the intestine to handle it.
(b) Causes. High osmolarity, low pH, fat and protein digestion products, and distension
of the duodenal wall all elicit an enterogastric reflex.
B

Esophagus

Food particles

Pylorus

Antrum

Antral ring

Figure 6-4. Peristaltic contractions begin in the midstomach (A) and proceed caudally, pushing the food toward the
pylorus (8). When the food reaches the pylorus (where small enough pieces of food flow into the duodenum), a mass
contraction of the terminal antrum pushes the food back toward the corpus through a narrow antral ring (C). The
backward movement of the food is called retropulsion.

b. Hormones released from the stomach and intestine also influence gastric motility.
(1) Excitatory effects. Gastrin, released into the circulation in response to antral distension
or food breakdown products, enhances gastric contractions.
(2) Inhibitory effects. A variety of intestinal hormones, collectively called enterogastrones, inhibit gastric contractions. Cholecystokinin (CCK) and secretin are two
known enterogastrones. The identity and mode of action of other enterogastrones
remain to be discovered.
(a) CCK is released from the duodenum in response to fat or protein digestion products.
CCK probably acts by blocking the excitatory effects of gastrin on gastric smooth
muscle (see also IV C 3 c).
(b) Secretin is released from the duodenum in response to the presence of acid. Secretin
most likely has a direct inhibitory effect on smooth muscle (see also IV C 3 c).
c. Migrating motor complex (MMC). During the interdigestive period, any food left in the
stomach is removed by the MMC.
(1) The MMC is a peristaltic wave that begins within the esophagus and travels through the
entire GI tract (see IV B 2 c).
(2) The peristaltic wave occurs every 60-90 minutes during the interdigestive period.
(3) The hormone motilin, which is released from endocrine cells within the epithelium of
the small intestine, increases the strength of the MMC.
7. Vomiting (or emesis) is the forceful expulsion of the food from the stomach and intestine.
a. Initiation. Vomiting may be initiated by direct activation of the vomiting center in the
medulla or by activation of the chemoreceptor trigger zone within the area postrema of
the brain stem.
(1) The vomiting center may be directly activated by afferent fibers or by irritation due to
injury or increases in intracranial pressure. When the vomiting center is directly activated, it causes projectile vomiting-a rapid, forceful emesis not accompanied by
nausea.
(2) The chemoreceptor trigger zone may be activated by afferent nerves originating within
the GI tract or by circulating emetic agents such as apomorphine or copper sulfate.
Vomiting caused by activation of the chemoreceptor trigger zone is accompanied by
nausea.
b. Mechanical sequence of vomiting
(1) Vomiting begins with a deep inspiration followed by the closing of the glottis.
(2) Next, a pressure wave originating in the intestine propels chyme into the orad stomach.
(3) Finally, an increase in abdominal pressure forces the chyme into the esophagus and out
of the mouth.
(4) Retching may proceed vomiting. Retching involves all of the involuntary motions of
vomiting but without the production of vomitus. The chyme is not ejected because the
abdominal and thoracic pressures are not sufficient to overcome the resistance of the
upper esophageal sphincter.

C. Gastric secretion
1. General considerations
a. Function. Gastric secretions aid in the breakdown of food into small particles and continue
the process of digestion begun by salivary enzymes. About 2 Llday of gastric secretions are
produced.
b. Phases of gastric secretion
(1) The cephalic phase of gastric secretion is initiated by the thought, sight, taste, or smell
of food. It is dependent on the integrity of the vagal fibers innervating the stomach.
(a) Secretion of HCi from parietal cells, gastrin from G cells, and pepsinogen from chief
cells is stimulated by vagal efferent fibers.
(b) Almost half of the gastric secretions released during a meal occur as a result of
cephalically induced vagal stimulation.
(2) The gastric phase of secretion is initiated by the entry of food into the stomach.
(a) Distension of the corpus, acting through local and vago-vagal reflexes, results in an
increase in HCi secretion.
(b) Distension of the antrum initiates vagally mediated and local reflexes that result in
gastrin release from antral G cells. Gastrin release is inhibited at low pH 3).
(c) Low pH activates local reflexes, which enhance pepsinogen secretion.

(d) Although the rate of gastric secretion during the gastric phase is less than during the
cephalic phase, it continues for a longer time. Thus, the two phases contribute about
the same amount of secretion.
(3) The intestinal phase of secretion begins as the chyme begins to empty from the
stomach into the duodenum. Overall, little gastric secretion occurs during the intestinal
phase.
c. Gastric secretory cells are located on the surface of the stomach and in glands that are
buried within the mucosa.
(1) Oxyntic glands are located in the fundus and corpus of the stomach. They contain
three types of secretory cells.
(a) The parietal (oxyntic) cells secrete HCI. These cells are also responsible for the
secretion of intrinsic factor, which is necessary for the absorption of vitamin B12
by the ileum of the small intestine (see III C 5; IV F 5 c).
(b) Peptic (chief) cells secrete pepsinogen, the precursor for the proteolytic enzyme

pepsin.
(c) Mucous cells secrete mucus.
(2) Pyloric glands are located in the antrum and pyloric regions of the stomach. They
contain G cells and some mucous cells. G cells are responsible for the release of the
hormone gastrin.
(a) There are two forms of gastrin, G-17 (little gastrin, a 17-amino-acid peptide) and

G-34 (big gastrin, a 34-amino-acid peptide). Although G-17 is more potent than
G-34, the larger gastrin is found in higher concentrations within the circulation.
(b) Gastrin is released from the basolateral surface of the G cells, enters the circulation,
and travels to the orad stomach where it stimulates parietal-cell HCI secretion.

2. HCI secretion
a. Functions of HCI
(1) HCI participates in the breakdown of protein.
(2) It provides an optimal pH for the action of pepsin.
(3) It hinders the growth of pathogenic bacteria.

b. Mechanism of HCI secretion (Figure 6-5)


(1) HCI is secreted into the parietal cell canaliculi by a three-step process.
(a) The active transport process is begun by the transport of K+ and CI- into the

canaliculi. CI- is transported either by a pump or through a channel. The flow of


CI- creates a negative potential inside the canaliculi, causing K+ to flow passively
into the canaliculi.
(b) Hydrogen ion (H+) is then exchanged for K+ by a H+-K+ ATPase.
(c) Water enters the canaliculi down the osmotic gradient created by the movement of
HCI.
(2) The H+ entering the canaliculi is supplied by the dissociation of carbonic acid (H 2 C0 3 )
into H + and HC03 - .
(a) H 2 C0 3 is formed within the parietal cell from the reaction:
CO 2

H 20 -

H 2 C0 3

H+

HC03 -

(b) The formation of H 2 C0 3 from carbon dioxide (C0 2 ) is catalyzed by the enzyme
carbonic anhydrase (CA). Acetazolamide, a CA inhibitor, blocks the formation of
HCI by the parietal cell.
(c) The HC0 3 - diffuses back into the plasma (creating the alkaline tide associated with
gastric secretion) in exchange for CI-, thus providing CI- for the initial step in the
secretory process [see III C 2 b (1) (a)].
(3) Most of the HCI that is secreted into the stomach is neutralized and reabsorbed within
the small intestine. However, if the gastric contents are lost before they enter the small
intestine (e.g., by vomiting), a severe alkalosis may ensue.
(4) The active transport processes involved in the generation of HCI require a large amount
of ATP. The ATP is generated by mitochondria found in very high concentration (40%
of cell volume) within the parietal cell.
(5) The pH of the parietal cell secretion can be as low as 0.8 (i.e., a H+ concentration of
approximately 150 mmol, or almost 4 million times as great as the H+ concentration
of plasma).

Secretion into stomach

Parietal
(oxyntic) cell

Figure 6-5. HCI being formed by the parietal cell. CI- and K+ are secreted into canaliculi by separate transporters,
which may be channels or carriers. H+ is exchanged for K+ by an ATPase active transport system, allowing HCI to
be secreted into the stomach. Numerous mitochondria provide energy for the active transport process. (Adapted
from Guyton AC: Textbook of Medical Physiology, 8th edition. Philadelphia, WB Saunders, 1991, P 64.)
c. Substances affecting HCI secretion
(1) Stimulation of HCI secretion. ACh, histamine, and gastrin act directly on the parietal
cell to stimulate HCI secretion (Figure 6-6). In addition, ACh and gastrin may directly
stimulate the mast cell to secrete histamine.
(a) ACh, a neurotransmitter, is released from nerve cells innervating the parietal cell.
(b) Histamine is released from mast cells located within the corpus.
(i) Histamine can stimulate HCI secretion directly or can potentiate the secretion
produced by ACh or gastrin.
(ii) Histamine is classified as a paracrine agent because it diffuses from its release
site to the parietal cells (rather than traveling within the circulation as does a
hormone).
(iii) The most commonly used anti-ulcer drugs (i.e., cimetidine and ranitidine) are
histamine antagonists that block the H2 receptor on the parietal cell.
(c) Gastrin is released from G cells in the distal stomach [see III C 1 c (2)]. Gastrin is
classified as a hormone because it travels to its target cell through the circulation.
A variety of substances affect gastrin secretion (see III C 3).
(2) Inhibition of HCI secretion. Somatostatin inhibits HCI secretion by parietal cells and
gastrin secretion by G cells. Somatostatin is released from interneurons within the enteric
nervous system.
d. Regulation of gastric acid (HCi) secretion
(1) Stimulation during the cephalic phase. The vagus nerve stimulates the release of ACh
and inhibits the release of somatostatin from interneurons within the enteric nervous
system, thus enhancing the secretion of HCI (see Figure 6-6).
(2) Stimulation during the gastric phase. The amount of ingested protein is the most
important determinant of acid secretion during the gastric phase.
(a) Protein is a good buffer and thus keeps the pH of the stomach at an optimal level
for acid secretion.

Vagal nucleus

Vagus nerve

~
~
0\

<$>

&;

Figure 6-6. Many substances affect HCI secretion by the parietal cell. ACh from the vagus nerve, histamine (H) from
mast cells, and gastrin (G) from G cells all stimulate the parietal cell directly to secrete HCI. The release of gastrin
into the circulation, in turn, is stimulated by gastrin-releasing peptide (GRP) and protein digestion products (P).
Somatostatin (5), released from somatostatin cells (55), inhibits the release of both gastrin and HCI. Thus, the stimulation
of vagal fibers, which causes the release of ACh and GRP but inhibits the release of somatostatin, has an amplified
positive effect on parietal-cell HCI secretion. Plus signs = stimulation; Minus signs = inhibition. (Adapted from
Johnson LR led]: Gastrointestinal Physiology, 3rd edition. St. Louis, CV Mosby, 1985, p 72.)

(b) Amino acids and peptides directly stimulate parietal cells to secrete acid.
(3) Inhibition during the gastric phase. The most potent inhibitor of Hel secretion during
the gastric phase is the presence of acid in the stomach. If the pH of the stomach falls
below 2, acid secretion stops. Acid secretion is inhibited by two mechanisms.
(a) A low pH directly inhibits Hel and gastrin secretion.
(b) Lowering the pH also releases somatostatin, which inhibits the secretion of gastrin
by the G cells and Hel by the parietal cells (see Figure 6-6).
(4) Stimulation during the intestinal phase. The presence of protein digestion products
within the duodenum causes an increase in Hel secretion.
(a) Although G cells have been identified within the duodenum, gastrin is not thought
to cause the increase in acid secretion.
(b) An as yet unidentified hormone, called entero-oxyntin, is postulated to be responsible for the increase in acid secretion.

(c) Amino acids circulating in the blood after being absorbed from the intestine may
also stimulate HCI secretion.
(S) Inhibition during the intestinal phase. The inhibition of HCI secretion is accomplished
by the same mechanisms responsible for inhibiting gastric motility.
(a) H +, fatty acids, and increased osmolarity stimulate the release of enterogastrones
from the duodenum.
(b) The most important of the enterogastrones may be gastrin inhibitory peptide
(GIP), which inhibits both gastrin release and parietal cell secretion of HCI. GIP is
thought to act by stimulating the release of somatostatin, which, in turn, inhibits the
parietal and G cells.

3. Gastrin secretion
a. Functions of gastrin
(1)
(2)
(3)
(4)

Gastrin stimulates HCI secretion.


It increases gastric and intestinal motility.
It increases pancreatic secretions.
It is necessary for the proper growth of GI mucosa.

b. Substances affecting gastrin secretion


(1) Stimulation of gastrin secretion. Bombesin [gastrin-releasing peptide (GRP)] is
most likely the neurotransmitter responsible for stimulating G cells to secrete gastrin.
The vagus nerve increases the release of GRP during the cephalic phase (see Figure

6-6).
(2) Inhibition of gastrin secretion. Somatostatin inhibits gastrin secretion. The vagus nerve
inhibits the release of somatostatin during the cephalic phase.

c. Regulation of gastrin secretion


(1) In general, gastrin secretion is regulated by the same mechanisms that regulate HCI
secretion (i.e., vagal stimulation, pH, enterogastrones).
(2) In addition, several foods and food breakdown products (secretagogues) directly stimulate the release of gastrin. These include protein digestion products, alcohol, and coffee
(both caffeinated and decaffeinated).

4. Pepsinogen secretion
a. Function of pepsinogen. Pepsin, the active form of pepsinogen, is a proteolytic enzyme
that begins the process of protein digestion [see IV F 2 b (1)].

b. Regulation of pepsinogen secretion. Pepsinogen is released from the chief cells of the
oxyntic glands during all three phases of digestion.
(1) Cephalic stage. During the cephalic stage of digestion, vagally stimulated cholinergic
neurons within the enteric nervous system directly stimulate chief cells to release pepsinogen.
(2) Gastric phase. During the gastric phase of digestion, low pH activates local reflexes
that enhance pepsinogen secretion. The low pH of the stomach is also responsible for
converting pepsinogen into pepsin. Again, ACh is the transmitter that stimulates the
chief cells.
(3) Intestinal phase. Secretin enhances pepsinogen release. Thus, the presence of H+
within the duodenum during the intestinal phase of digestion may contribute to pepsinogen secretion.

S. Intrinsic factor
a. Definition. Intrinsic factor is a glycoprotein secreted by the parietal cells of the gastric
mucosa, chiefly by those in the fundus.

b. Function. Intrinsic factor is required for the absorption of vitamin B12


(1) Intrinsic factor forms a complex with vitamin B12
(2) The intrinsic factor-B 12 complex is carried to the terminal ileum, where the vitamin is
absorbed (see IV F 5 c).

D. The gastric mucosal barrier protects the gastric lining cells from damage by intraluminal HCI, or
autodigestion. Its chief component is a thick viscous alkaline mucous layer that measures over 1
mm thick and is secreted by the mucous cells. The mucous cells cover the surface between the
various gastric glands and outnumber all other cell types found in the gastric mucosa.
1. The turnover rate of the gastric mucosa is extremely high; 5 X 10 5 mucosal cells are shed each
minute, and the entire mucosa is replaced in 1-3 days.

2. Mild injury results in increased mucus secretion and surface desquamation followed by regeneration.
3. More serious injury denudes the mucosal surface, forming an ulcer, and produces bleeding.
Ulceration results when damage to the mucosa (e.g., due to a highly concentrated HCI, 10%
ethanol, salicylic acid, or acetylsalicylic acid) allows acid to penetrate the mucosal barrier and
destroy mucosal cells. This liberates histamine, which increases acid secretion and produces
increased capillary permeability and vasodilation. The latter two effects lead to edema. It is the
exposure of mucosal capillaries to the digestive process that leads to bleeding.
4. The rate of repair of mucosal injury depends on the extent of injury and varies from as little as
48 hours for restricted desquamation to up to 3-5 months if damage has left only the deepest
portions of the gastric pits intact.
E. Gastric digestion and absorption
1. Digestion
a. Carbohydrate digestion in the stomach depends on the action of salivary amylase, which
remains active until halted by the low pH in the stomach.
b. Protein digestion. About 10% of ingested protein is broken down completely in the
stomach. Gastric pepsin facilitates later digestion of protein by breaking apart meat particles.
c. Fat digestion is minimal due to the restriction of gastric lipase activity to triglycerides
containing short-chain ( < 10 carbons) fatty acids. Acid and pepsin break emulsions so that
fats coalesce into droplets, which float and empty last.
2. Absorption
a. Nutrients. Very little absorption of nutrients takes place in the stomach. The only substances
absorbed to any appreciable extent are highly lipid-soluble substances (e.g., the un-ionized
triglycerides of acetic, propionic, and butyric acids). Aspirin at gastric pH is un-ionized and
fat soluble; after absorption, it ionizes intracellularly, damaging mucosal cells and ultimately
producing bleeding. Ethanol is rapidly absorbed in proportion to its concentration.
b. Water moves in both directions across the mucosa. It does not, however, follow osmotic
gradients. Water-soluble substances, including Na+, K+, glucose, and amino acids, are
absorbed in insignificant amounts.

IV. THE SMALL INTESTINE is the major site of digestion and absorption of carbohydrates, proteins,
and fats in the GI tract. The action and secretions of several accessory organs (see IV A 2) are
essential to the digestive and absorptive functions of the small intestine. Nutrients and fluids that are
not absorbed in the small intestine are passed on to the colon.
A. Anatomy
1. Small intestine
a. The small intestine has three parts: the duodenum, the jejunum, and the ileum (see Figure
6-1 ).
b. Although the small intestine is approximately 5 m long, it has an absorptive area of over
250 m2
(1) Its large surface area is created by numerous folds of the intestinal mucosa (valvulae
conniventes); by densely packed villi, which line the entire mucosal surface; and by
microvilli, which protrude from the surface of the intestinal cells.
(a) The epithelial cells from which the microvilli protrude are called enterocytes.
(b) The microvilli (about 1JLm long and 0.1 JLm in diameter) give the intestinal mucosa
its characteristic brush border appearance.
(2) The blood supply of the villus is ideally organized to collect the nutrients after they are
absorbed across the brush border membrane.
(a) Each villus is supplied by an arteriole, which gives rise to a capillary tuft at the tip
of the villus. The capillaries coalesce into venules, which drain into the portal vein.
The portal vein carries the absorbed nutrients to the liver.
(b) Branches of the lymphatics, called lacteals, also extend to the tip of the villus. These
vessels carry absorbed fats to the thoracic duct from which they enter the general
circulation.

2. The accessory organs involved in intestinal digestion and absorption are: the pancreas, the
liver, and the gallbladder.
a. The pancreas secretes various substances that aid in intestinal digestion, including HC0 3 - ,
which neutralizes the acidic content of chyme entering the small intestine.
b. The liver secretes bile, which is necessary for fat digestion and nutrient absorption.
(1) Bile travels from the liver through bile ducts to reach the duodenum of the small
intestine.
(2) The sphincter of Oddi, located at the distal end of the duodenum, forms the opening
that connects the small intestine to the common bile duct.
(a) This sphincter controls the flow of bile into the small intestine [see IV D 1 d, 5 b
(1 )j.
(b) When the sphincter is closed, bile cannot enter the small intestine and must be
stored in the gallbladder.
(3) The portal circulation carries bile that has been absorbed from the terminal ileum back
to the liver (see IV D 3 and Figure 6-7).
c. The gallbladder stores bile during the interdigestive period.
B. Motility
1. Contractile activity
a. Function. Contractile activity of the smooth muscles lining the small intestine serves two
major functions:
(1) Mixing the chyme with the digestive juices and bile to facilitate digestion and absorption
(2) Propelling the chyme from the duodenum to the colon
b. Transit time. It usually takes about 2-4 hours for the chyme to move from one end of the
small intestine to the other.
2. Types of movements
a. Segmentation is the most common type of intestinal contraction.
(1) During segmentation, about 2 cm of the intestinal wall contracts, forcing the chyme
back toward the stomach (oradly) and toward the colon (aboradly).
(2) When the muscle relaxes, the chyme returns to the area from which it was displaced.
(3) This back-and-forth movement enables the chyme to become thoroughly mixed with
the digestive juices and to make contact with the absorptive surface of the intestinal
mucosa.
(4) Segmentation contractions occur about 12 times/min in the duodenum and 8
times/min in the ileum. The contractions last for 5-6 seconds.
(5) Segmentation occurs throughout the digestive period.
b. Peristaltic contractions also occur in the small intestine.
(1) Although peristaltic contractions occasionally propel food along the entire length of the
intestine, they rarely involve more than a short segment of the intestine.
(2) Peristalsis is not considered to be an important component of intestinal transit.
c. The MMC (see III B 6 c) spreads over the intestine during the interdigestive period.
(1) The MMCs sweep out the chyme remaining in the small intestine during the interdigestive
period.
(2) MMCs occur every 60-90 minutes and last for about 10 minutes.
3. Propulsion of chyme. During the digestive period, the higher frequency of segmentation in
the proximal intestine (duodenum) than in the distal intestine (ileum) propels the chyme slowly
toward the colon.
a. Thus, when the chyme is pushed aboradly, it is less likely to be forced back by a segmentation
contraction.
b. In contrast, when the chyme is pushed oradly, it will be quickly pushed toward the colon
again by a segmentation contraction in the more proximal region of the small intestine.
4. Control of intestinal motility. The frequency and strength of segmentation contractions in the
intestine are controlled by the slow waves (see Figure 6-3).
a. Generation. Segmentation contractions can occur only if the slow waves produce spikes,
or action potentials. Spikes appear on the slow waves when the membrane potential is
sufficiently depolarized.
b. Frequency
(1) The frequency of segmentation contractions is directly related to the frequency of the
slow waves.

(2) Slow wave frequency is controlled by pacemaker cells within the wall of the intestine
and is not influenced by neural activity or circulating hormones.

c. Strength
(1) The strength of a segmentation contraction is proportional to the frequency of the spikes
generated by the slow wave. This frequency is controlled by the amplitude of the slow
wave. Thus, the greater the slow wave amplitude, the greater the frequency of spikes
generated and the greater the strength of the contraction.
(2) Slow wave amplitude is controlled by the hormones released during digestion.
(a) Gastrin, CCK, motilin, and insulin increase the slow wave amplitude.
(b) Secretin and glucagon reduce the slow wave amplitude.

C. Pancreatic secretions
1. Pancreatic cell types and their functions. The pancreas contains endocrine, exocrine, and
ductal cells.

a. The endocrine cells, arranged in small islets within the pancreas, secrete insulin, glucagon,
somatostatin, and pancreatic polypeptide directly into the circulation.
b. The exocrine cells are organized into acini that produce four types of digestive enzymes:
peptidases, lipases, amylases, and nucleases, which are responsible for digesting proteins,
fats, carbohydrates, and nucleic acids, respectively. In their absence, malabsorption syndromes develop.
c. Each day, the ductal cells secrete about 1200-1500 ml of pancreatic juice containing a
high concentration of HCOl - . The HC0 3 - neutralizes gastric acid and regulates the pH of
the upper intestine. Failure to neutralize the chyme as it enters the intestine will result in
duodenal ulcers.

2. Composition of pancreatic secretions


a. Electrolytes
(1) Na+ and K+ concentrations in pancreatic juice are the same as those in plasma water
(i.e., 142 mEq/L and 4.8 mEq/L, respectively).
(2) HC03 - concentration in pancreatic juice is much higher than it is in plasma water
(100 mEq/L as opposed to about 24 mEq/L).
(3) Pancreatic juice also contains small amounts of other ions such as Ca2+, magnesium
(Mg2+), zinc (Zn 2+), monohydrogen phosphate (HPO/-), and sulfate (SO/-)'
(4) Secretion of HC0 3 - by the ductal cells requires at least one active transport process.
(a) HC0 3 - and H+ are formed from the dissociation of H 2C0 3 via the same reaction
that occurs in the parietal cells of the stomach [see III C 2 b (2)].
(b) H + is actively transported out of the cell across its basal membrane by an Na +-H +
anti porter.
(c) HC0 3 - is transported across the apical membrane of the ductal cells. The transport
process responsible for HC0 3 - secretion is not yet known.
(d) Na+ follows the HC0 3 - into the pancreatic ducts. Most of the Na+ flows passively
between the ductal cells. However, some may be actively transported across the
apical membranes.
(e) Water flows into the ducts down the osmotic gradient established by the secretion
of sodium bicarbonate (NaHC0 3 )' Flow rates as high as 1 ml/min can be established.
(f) As the pancreatic juice flows along the ducts, CI- is exchanged for HC0 3 - . The
higher the flow rate, the smaller the exchange. Thus, HC03 - concentration is highest
when pancreatic secretion is greatest.
b. Enzymes. Three major types of pancreatic enzymes are secreted by the pancreas: amylases,
lipases, and proteases.
(1) Pancreatic a-amylase is secreted in its active form. It hydrolyzes glycogen, starch, and
most other complex carbohydrates, except cellulose, to form disaccharides.
(2) Pancreatic lipases (lipase, cholesterol lipase, and phospholipase) [see IV F 3 a (1)] are
secreted in their active forms. The enzymes that hydrolyze water-insoluble esters require
bile salts to work. Water-soluble esters can be hydrolyzed without the action of bile
salts.
(3) Pancreatic proteases (trypsin and the chymotrypsins) are secreted in their inactive
zymogen form (trypsinogen and the chymotrypsinogens, respectively) [see IV F 2 b (2)].
(a) Trypsinogen is converted to trypsin by enterokinase or by trypsin itself (autocatalysis).
(b) The chymotrypsinogens are converted to their active form by trypsin.

(4) Trypsin inhibitor is secreted by the same cells and at the same time as the pancreatic
proenzymes. Trypsin inhibitor protects the pancreas from autodigestion.
3. Control of pancreatic secretion. Like gastric secretion, pancreatic secretion is divided into
the following three phases.
a. Cephalic phase. The thought, sight, smell, or taste of food produces the cephalic phase
of pancreatic secretion. Both acinar and ductal cell secretions are enhanced by vagal
stimulation.
(1) Enzyme secretion by the acinar cells is stimulated by enteric neurons that release ACh.
(2) HCa l - secretion by ductal cells is stimulated by enteric neurons that release a noncholinergic, nonadrenergic transmitter that is thought to be VIP.
b. Gastric phase. Pancreatic secretion is enhanced during the gastric phase by distension and
food breakdown products.
(1) Distension of the antrum and corpus initiates a vago-vagal reflex resulting in a low
volume of pancreatic secretion containing both HCal - and enzymes. ACh is the
transmitter.
(2) Food breakdown products (primarily amino acids and peptides) can stimulate pancreatic secretions because of their ability to cause the G cells of the antrum to release
gastrin. Gastrin produces a low-volume, high-enzyme pancreatic secretion.
c. Intestinal phase. The major stimulants for pancreatic secretion are the hormones CCK and
secretin. They are released from endocrine cells in the duodenum and jejunum during the
intestinal phase of pancreatic secretion.
(1) CCK, in addition to its effect on the gallbladder [see IV D 5 b (1)], is a potent stimulant
of pancreatic enzyme secretion.
(a) Like gastrin, CCK is found in two physiologically active forms, an octapeptide called
CCK-8 and a 33-chain polypeptide, CCK-33.
(b) The actions of CCK are potentiated by secretin. By itself, secretin has no effect on
enzyme secretion.
(2) Secretin was the first hormone ever discovered. Its primary effect is to increase HCal secretion by the pancreas.
(a) The actions of secretin are potentiated by CCK. By itself, CCK has no effect on
HCa l - secretion.
(b) Because they are potentiators of each other's action, small concentrations of CCK
and secretin together can produce significant amounts of pancreatic HCal - and
enzyme secretions, while either one alone would have little or no effect.
(3) Control of CCK and secretin release. CCK and secretin are secreted from endocrine
cells in response to the entrance of chyme into the small intestine.
(a) Amino acids (primarily phenylalanine), fatty acids, and monoglycerides are the
major stimuli for CCK secretion.
(b) Low pH 4.5), caused by the presence of gastric acid (HCIl in the intestine, is a
potent stimulus for the release of secretin.
(4) A vago-vagal reflex, which greatly potentiates the effects of secretin and CCK, is activated during the intestinal phase of digestion.
(5) ACh potentiates the effects of both CCK and secretin. Thus, vagal stimulation is much
more potent in stimulating pancreatic secretions when CCK and secretin are present in
the plasma.

D. Biliary secretions
1. General features of bile
a. Function. Bile is required for the digestion and absorption of fats and for the excretion of
water-insoluble substances such as cholesterol and bilirubin.
b. Formation. Bile is formed by liver epithelial cells, called hepatocytes, and by epithelial
cells lining the bile ducts, called ductal cells. Between 250 and 1100 ml of bile are secreted
daily.
c. Storage. Although it is secreted continuously, bile is stored in the gallbladder during the
interdigestive period.
d. Release. Bile is released into the duodenum during the digestive period only after chyme
has triggered the release of CCK, which then produces contraction of the gallbladder and
relaxation of the sphincter of addi.

2. Composition of bile
a. Bile acids
(1) Primary bile acids (trihydroxycholic acid and dihydroxychenodeoxycholic acid)
are synthesized from cholesterol and converted into bile salts by the hepatocytes as
follows.
(a) Cholesterol is absorbed through microvilli lining the serosal (antiluminal) border of
the hepatic epithelial cells.
(b) The bile acids are conjugated with either taurine or glycine to form bile salts.
(c) The bile salts are actively secreted into a canaliculus on the lateral (luminal) surface
of the hepatocyte, from which they then drain into the bile duct.
(d) Because bile salts are not lipid soluble, they remain within the intestine until reaching
the ileum, where they are actively absorbed (see IV F 3 b).
(2) Secondary bile acids are formed by deconjugation and dehydroxylation of the primary
bile salts by intestinal bacteria, forming deoxycholic acid and lithocholic acid.
b. Bile pigments
(1) Bilirubin and biliverdin, the two principal bile pigments, are metabolites of hemoglobin
formed in the liver and conjugated as glucuronides for excretion. They are responsible
for the golden yellow color of bile.
(2) Intestinal bacteria metabolize bilirubin further to urobilin, which is responsible for the
brown color of stool.
(3) If bilirubin is not secreted by the liver, it builds up in the blood and tissues, producing
jaundice (see IV D 6 b).
c. Phospholipids (primarily lecithins) are, after bile salts, the most abundant organic compound in bile.
(1) Although the phospholipids are normally insoluble in water, they are solubilized by the
bile salt micelles.
(2) The micelles are able to solubilize other lipids more effectively when they are composed
of bile salts and phospholipids than when they are composed of bile salts alone.
d. Cholesterol, although present only in small amounts, is an important component of bile.
(1) Cholesterol is essentially insoluble in water and thus must be solubilized by bile salt
micelles before it can be secreted in the bile (see IV F 3 b).
(2) Biliary secretion of cholesterol is important because it is one of the few ways in which
cholesterol stores can be regulated.
e. Electrolytes. The electrolyte composition of bile is similar to that of pancreatic juice and
plasma (see IV C 2 a).
3. Enterohepatic circulation is the recirculation of bile salts from the liver to the small intestine
and back again. This circulation is necessary because of the limited pool of bile salts available
to help break down and absorb fat (Figure 6-7).
a. Path of circulation. Bile salts travel from the liver to the duodenum via the common bile
duct. When the bile salts reach the terminal ileum, they are reabsorbed into the portal
circulation. The liver then extracts them from the portal blood and secretes them once again
into the bile.
(1) Bile salts are reabsorbed only in the terminal ileum. No reabsorption of bile salts occurs
in the duodenum or jejunum.
(2) From 90%-95% of the bile salts that enter the small intestine are actively reabsorbed
from the lower ileum back into the portal circulation.
(3) The remaining bile salts are excreted into the feces.
b. Circulating pool. The total circulating pool of bile salts (consisting of primary and secondary
bile acids) is approximately 3.6 g. Because 4-8 g of bile salts are required to digest and
absorb a meal (more if the meal is high in fat), the total pool of salts must circulate twice
during the digestion of each meal. Consequently, the bile salts usually circulate 6-8 times
daily.
c. Bile salt synthesis and replacement. The rate of bile salt synthesis is determined by the
rate of return to the liver. The usual rate is 0.2-0.4 g/day, which replaces normal fecal
losses. The maximal rate is 3-6 g/day. If fecal losses exceed this rate, the total pool size
decreases.
d. Clinical implications. Because bile salts are required for proper digestion and absorption
of fats, any condition that disrupts the enterohepatic circulation (e.g., ileal resection or small
intestinal diseases such as sprue or Crohn's disease) leads to a decreased bile acid pool and
malabsorption of fat and fat-soluble vitamins. The clinical manifestations of such conditions

Hepatic duct

Figure 6-7. The enterohepatic circu-

- . = Direction of bile
salt circulation

= Bile salt active

transport system

lation. Bile acids are absorbed from


the terminal ileum by a Na+-dependent active transport system and then
rapidly sequestered by hepatocytes in
the liver and returned to the gallbladder or duodenum. Each day, about
20% of the bile acid pool escapes the
enterohepatic circulation (thus becoming lost to excretion) and must be
resynthesized by the liver.

are steatorrhea and nutritional deficiency. An increase in fecal losses of bile salts results in
watery diarrhea, since bile salts inhibit water and Na+ absorption in the colon.

4. Control of biliary secretion. The volume of biliary secretion and the amount of bile in that
secretion are regulated separately.

a. The bile-independent fraction of biliary secretion refers to the amount of fluid, composed of electrolytes and water, that is secreted each day by the liver. Although this
fluid, by definition, is secreted with the bile, its secretion is controlled separately from bile
secretion.
(1) Secretion of this fluid is controlled by the hormone secretin.
(2) The fluid resembles the secretion of the pancreatic ductal cells in the following ways
(see IV C 2 a, 3).
(a) The fluid is secreted by ductal cells.
(b) Its secretion is controlled by secretin.
(c) It has a high concentration of HC0 3 - .
b. The bile-dependent fraction of biliary secretion refers to the quantity of bile salts
secreted by the liver.
(1) The amount of bile salts secreted is directly related to the amount of bile reabsorbed by
the hepatocytes (i.e., the more bile reabsorbed from the portal circulation, the more
bile secreted by the liver).
(a) The total amount of bile is relatively constant. Because the liver has limited synthetic
capacity, there is a limit to the amount of bile that can be secreted.
(b) Substances that enhance bile secretion are called choleretics. Bile salts and bile
acids are the major choleretics.
(2) Unlike the bile-independent secretion, the synthesis and secretion of bile by the liver
is not under any direct hormonal or nervous control. However, CCK increases bile flow
indirectly by increasing the release of bile from the gallbladder [see IV D 5 b (1 )j.

5. Gallbladder
a. Functions. The gallbladder stores and concentrates the bile during the interdigestive period
and empties its contents into the duodenum during digestion.
(1) Storage. During the interdigestive period, the bile secreted by the liver is collected in
the gallbladder. The gallbladder typically stores 20-50 ml of bile.
(a) The bile is highly concentrated within the gallbladder by the reabsorption of water.
(b) Water is reabsorbed by the osmotic gradient produced by the active reabsorption
of Na+ and HC03 - .
(2) Contraction. During digestion, the gallbladder contracts, emptying its contents into the
duodenum.

b. Control
(1) CCK is the major stimulus for gallbladder contraction and sphincter of Oddi relaxation.
When chyme enters the small intestine, fat and protein digestion products directly
stimulate the secretion of CCK [see IV C 3 c (3) (a)].

(2) Vagal stimulation of the gallbladder also causes gallbladder contraction and sphincter
of Oddi relaxation. Vagal stimulation occurs directly during the cephalic stage of digestion and indirectly via a vago-vagal reflex during the gastric phase of digestion.
c. Effects of cholecystectomy. Bile, not the gallbladder, is essential to digestion. After removal
of the gallbladder, bile empties slowly but continuously into the intestine, allowing digestion
of fats sufficient to maintain good health and nutrition. Only high-fat meals need to be
avoided.
d. Gallstones form in an estimated 10%-30% of the population, although only a fraction,
perhaps 20% of these, ever produce symptoms. In Western societies, about 85% of gallstones are composed chiefly of cholesterol; the remainder are pigment stones, composed
chiefly of calcium bilirubinate.
(1) Cholesterol and lecithin, which are both insoluble in water, are kept in solution in bile
through the formation of micelles [see IV F 3 b (1 )]. When the proportions of lecithin,
cholesterol, and bile salts are altered, cholesterol crystalizes, leading to stone formation.
Cholesterol stones are radiolucent.
(2) Calcium bilirubinate stones can form when infection of the biliary tree leads to
bacterial deconjugation of conjugated bilirubin. Unconjugated bilirubin, which is insoluble in bile, then precipitates to begin the stone-forming process. Calcium bilirubinate
stones are radiopaque.

6. Bilirubin metabolism
a. Formation of bilirubin. Bilirubin is a yellowish pigment formed as an end product of
hemoglobin catabolism.
(1) Hemoglobin is released from red blood cells when their membranes rupture at the end
of their life span.
(2) Hemoglobin is taken up by the cells of the reticuloendothelial system where the porphyrin ring and iron of the heme moiety are separated.
(3) The porphyrin ring is then converted to bilirubin and gradually released into the plasma.
(4) The bilirubin combines with plasma albumin and circulates to the liver where it is
absorbed by the hepatocytes, conjugated with glucuronic acid, and secreted into the
gallbladder along with the bile salts.
b. Jaundice is a yellowing of the skin due to the accumulation of bilirubin within the tissues.
Jaundice may result from:
(1) Excess production of bilirubin caused by excessive red blood cell destruction (e.g., in
hemolytic anemia)
(2) Obstruction of the bile ducts or liver cells preventing the secretion of bilirubin

E. Intestinal secretion
1. Mucus most likely serves a protective role, preventing HCI and chyme from damaging the
intestinal wall. Mucus is secreted by:

a. Brunner's glands, which are located within the duodenum


b. Goblet cells located along the length of the intestinal epithelium and in the intestinal crypts,
called the crypts of Lieberklihn

2. Enzymes capable of breaking down small peptides and disaccharides are associated with the
microvilli of the epithelial cells lining the intestine. Although these enzymes are not secreted
into the intestine, they are able to digest small peptides and disaccharides during the absorptive
process.

3. Water and electrolytes are secreted by all the epithelial cells of the intestine.
a. The watery secretion provides a solvent into which the products of digestion are dissolved.
b. If excessive amounts of fluid are produced (as happens when the enterotoxin responsible
for cholera stimulates massive fluid secretion), potentially life-threatening watery diarrhea
can result.
F. Digestion and absorption
1. Carbohydrates. The three major carbohydrates in the human diet are the disaccharides,
sucrose (cane sugar) and lactose (milk sugar), as well as the polysaccharide starches (which
may be in either the straight chain form, amylose, or the branched chain form, amylopectin).
Cellulose, another plant polysaccharide, is present in the diet in large amounts, but no enzymes
in the human digestive tract can digest it, so it is excreted unused. Dietary intake of carbohydrates is 250-800 g/day, which represents 50%-60% of the diet.
a. Digestion. Carbohydrates must be digested into monosaccharides before being absorbed
from the GI tract.
(1) Although starch digestion, by salivary a-amylase, begins in the mouth, almost all
carbohydrate digestion occurs within the small intestine.
(2) Pancreatic a-amylase digests carbohydrates into a variety of oligosaccharides.
(3) The oligosaccharides are digested into monosaccharides by brush border enzymes such
as maltase, lactase, and sucrase.
(4) The end products of carbohydrates are fructose, glucose, and galactose.
b. Mechanisms of absorption
(1) Glucose and galactose are absorbed by a common Na+ -dependent active transport
system.
(a) The carrier has two binding sites for Na+ and one to which either one molecule of
glucose or galactose can bind.
(b) Because two Na+ are transported down their electrochemical gradient, a large
amount of energy is available for transport; thus, almost all of the glucose and
galactose present in the intestine can be absorbed.
(2) Fructose is absorbed by facilitated transport. Fructose absorption occurs readily because most of the fructose is rapidly converted into glucose and lactic acid within the
intestinal epithelial cells, thus maintaining a high concentration gradient for diffusion.
(3) After being absorbed into the enterocytes, the monosaccharides are transported across
the basolateral membrane by facilitated diffusion. They then diffuse from the intestinal
interstitium into the capillaries of the villus.
(4) Absorption of monosaccharides is not regulated. The intestine can absorb over 5 kg of
sucrose each day.
(5) Failure to absorb carbohydrates results in diarrhea and intestinal gas.
(a) The unabsorbed carbohydrates act as osmotic particles and draw excessive fluids
into the intestine, which results in diarrhea.
(b) The flora of the intestine and colon metabolize the unabsorbed carbohydrates
producing a variety of gases [hydrogen (H 2), methane (CH 4 ), and CO 2 ], as well as
a variety of intestinal irritants.
(c) Lactose intolerance is the most common cause of carbohydrate malabsorption. It
results from the inability of the goblet cells to produce lactase.
(i) Avoidance of milk or milk products prevents the symptoms from developing.
(ii) Lactose intolerance in adults, where it is most common, is not usually a problem.
However, in infants, the diarrhea-produced dehydration can be life threatening.
2. Proteins. The daily dietary protein requirement for adults is 0.5-0.7 g/kg of body weight. For
children 1-3 years old, it is 4 g/kg.
a. Sources. The protein that is found in the intestines comes from two sources.
(1) Endogenous proteins, totaling 30-40 g/day, are secretory proteins as well as the
protein components of desquamated cells.
(2) Exogenous proteins are dietary proteins, which total at least 75-100 g daily in the
average American diet.
b. Digestion. Proteins must be digested into small polypeptides and amino acids before being
absorbed.
(1) About 10%-15% of the protein entering the GI tract is digested by gastric pepsin
secreted by chief cells. Protein digestion within the stomach is important primarily

because the protein digestion products act as secretagogues, stimulating the secretion
of proteases by the pancreas.
(2) Pancreatic proteases playa major role in protein digestion. The proteases, such as
trypsin, are secreted in an inactive form and must be converted into an active form
within the intestine [see IV C 2 b (3)].
(a) Enterokinase, an enzyme secreted by the epithelial cells of the duodenum and
jejunum, converts the inactive trypsinogen into trypsin.
(b) Trypsin then autocatalizes the conversion of trypsinogen to trypsin as well as activating the other proteases.
(3) Peptidases, secreted by the intestinal epithelial cells, continue the digestive process
begun by the pancreatic proteases, eventually converting the ingested proteins to small
polypeptides and amino acids.
c. Mechanisms of absorption
(1) A variety of Na + -dependent active transport systems have been identified for the
transport of tripeptides, dipeptides, and amino acids. Polypeptides with more than three
peptides are poorly absorbed.
(a) Separate transporters are present for the absorption of basic, acidic, and neutral
amino acids. At least two different polypeptide transporters exist.
(b) Tripeptides and dipeptides are absorbed in greater quantities than amino acids.
(2) Once inside the enterocytes, intercellular peptidases digest some of the polypeptides to
amino acids.
(3) Amino acids and the remaining polypeptides are transported across the basolateral
membrane of the enterocytes by facilitated or simple diffusion. They then enter the
capillaries of the villus by simple diffusion.
(4) Almost all of the ingested protein is absorbed by the intestine. Any protein that appears
in the stool derives from the bacteria within the colon or from cellular debris.
(5) Malabsorption of amino acids due to lack of adequate transporters (e.g., the malabsorption of neutral amino acids that occurs in Hartnup disease) is relatively rare. Inadequate
absorption of proteins due to lack of trypsin is a common consequence of pancreatic
diseases.
3. Fats. Daily dietary fat intake varies widely from 25-160 g.
a. Digestion. Although the serous glands of the tongue secrete lingual lipase, very little, if any,
lipid digestion occurs in the mouth or stomach. Unlike carbohydrates and proteins, lipids
are absorbed from the GI tract by passive diffusion. However, before the lipids can be
absorbed, they must first be made soluble in water. Bile salts are required for the solubilization of lipids.
(1) Pancreatic lipases. The pancreas secretes three different lipases [see IV C 2 b (2)].
(a) Pancreatic lipase is a fairly specific lipase that cleaves fatty acids from the 1 and
l' positions of triglycerides, leaving a 2-monoglyceride.
(b) Cholesterol esterase cleaves the fatty acid from cholesterol esters, leaving free
cholesterol.
(c) Phospholipase A2 cleaves the fatty acids from phospholipids such as phosphatidylcholine.
(2) Emulsification of lipids. Lipids must be broken down into small droplets (less than 1
JLm in diameter) or emulsified into fat globules by bile acids and lecithin (a component
of bile) before being digested.
(3) Fat digestion by the pancreatic lipases occurs very rapidly after emulsification because
of the large surface-to-volume ratio of the small globules.
b. Mechanism of absorption
(1) Micelle formation. The emulsified products of lipid digestion (e.g., monoglycerides,
cholesterol) must form micelles with bile salts before they can be absorbed.
(a) Micelles are small (about 5 nm in diameter) spherical aggregates containing some
20-30 molecules of lipids and bile salts.
(b) The bile salts are on the outside of the micelle. The 2-monoglycerides and Iysophosphatides have their hydrophobic chains facing the interior of the micelle and their
polar ends facing the surrounding water phase. The cholesterol and fat-soluble
vitamins are located within the fat-soluble interior of the micelle.

(2) Absorption of lipids and bile salts from micelles


(a) The micelles move along the microvilli surface allowing their lipids to diffuse across
the microvilli membrane and into the enterocytes.

(b) Lipids, cholesterol, and the fat-soluble vitamins are removed rapidly from the micelles once the micelles make contact with the microvilli.
(c) The rate-limiting step in lipid absorption is the migration of the micelles from the
intestinal chyme to the microvilli surface.
(d) The bile salts, freed of their associated lipids, are absorbed in the terminal ileum by
a Na + -dependent active transport process.
(e) Normally, all of the ingested lipid is absorbed. Fat present in the stool is derived
from the intestinal flora.
(3) Formation of chylomicrons by enterocytes
(a) Once inside the enterocytes, the digested lipids enter the smooth endoplasmic
reticulum (ER), where they are reconstituted.
(i) 2-Monoglycerides are combined with fatty acids to produce triglycerides.
(ii) Lysophosphatides are combined with fatty acids to form phospholipids.
(iii) Cholesterol is re-esterified.
(b) The reformed lipids coalesce into chylomicrons (small lipid droplets about 1 nm in
diameter) within the smooth ER.
(c) The chylomicrons are transported out of the cell by exocytosis. ~-lipoprotein,
which is synthesized by the enterocytes, covers the surface of the chylomicrons. In
the absence of ~-lipoprotein, exocytosis will not occur, and the enterocytes become
engorged with lipids.
(4) Transport of lipids into circulation
(a) After exiting the cell, the chylomicrons merge into larger droplets that vary in size
from 50-500 nm, depending on the amount of lipids being absorbed
(b) The large lipid droplets then diffuse into the lacteals, from which they enter the
lymphatic circulation.
(c) Almost all digested lipids are totally reabsorbed by the time the chyme reaches the
midjejunum, with most of the absorption occurring in the duodenum.
(5) Lipid malabsorption is much more common than carbohydrate or protein malabsorption. It usually results from one of the following two conditions.
(a) The pancreas does not secrete sufficient quantities of lipase.
(b) The liver does not secrete sufficient quantities of bile.

4. Water and electrolytes


a. Water
(1) The small intestine, in addition to absorbing most of the dietary Na+ and water, must
also absorb the 7-8 L of water and 20-30 g of Na+ that are contained in salivary,
gastric, biliary, and pancreatic secretions. Failure to reabsorb water from the intestine
can lead to rapid dehydration and circulatory collapse.
(2) Water undergoes passive, iso-osmotic reabsorption in the small intestine.
(a) Active reabsorption of electrolytes and nutrients creates an osmotic gradient favoring
the reabsorption of water.
(b) Because osmotic equilibrium is rapidly achieved, the fluid in the intestine is always
isotonic to plasma.
(3) In the duodenum, the osmotic pressure created by the entering chyme causes water to
flow into the intestine.
(4) In the jejunum and ileum, the reabsorption of sodium chloride (NaCI) creates an osmotic
gradient favoring the reabsorption of water.
b. NaCI
(1) Na + reabsorption is a two-step process.
(a) First, Na+ and Ci- are transported from the lumen into the enterocyte.
(b) Then, they are transported across the basolateral membrane into the intestinal
interstitium.
(2) Na+ enters the enterocyte in three ways.
(a) About 30% is transported into the cell by a Na+-glucose, Na+-amino acid, or
Na+- (di- or tri-) peptide cotransport system.
(b) About 30% is transported into the cell by a neutral Na+-CI- cotransport system.
(c) The remainder enters the cell passively down an electrochemical gradient.
(3) Once inside the enterocyte, Na+ is transported across the basolateral membrane by a
Na + -K + ATPase active transport system.
(4) For the most part, Ci- flows passively through the enterocyte down the electrochemical
gradient established by the active transport of Na+.

5. Vitamins and minerals


a. Fat-soluble vitamins (A, D, E, and K) become part of the micelles formed by bile salts and
are absorbed along with other lipids in the proximal intestine.
b. Water-soluble vitamins (C, and the 8 vitamins biotin, folic acid, nicotinic acid, 8 6 or
pyridoxine, 8 2 or riboflavin, and 8 1 or thiamine) are absorbed by facilitated transport or
a Na+ -dependent active transport system in the proximal small intestine.
c. Vitamin 8 12 absorption is more complex than other vitamins.
(1) In the stomach, vitamin B12 is bound to an R protein, which is a specific binding protein.
(2) The gastric parietal cells secrete another vitamin B12-bindirig. protein called intrinsic
factor. However, the affinity of intrinsic factor for vitamin B12 is less than that of R
protein, so most of the B12 is bound to R protein in the stomach.
(3) In the intestine, pancreatic proteases cleave vitamin B12 from the R protein, allowing it
to bind to intrinsic factor.
(4) The intrinsic factor-B 12 complex binds to a receptor on ileal enterocytes.
(a) Absorption of the vitamin B12 from the intrinsic factor-B 12 complex can occur only
after the complex binds to the receptor.
(b) In the absence of intrinsic factor, minimal amounts of vitamin B12 can be absorbed
by diffusion. Thus, if large amounts of the vitamin are ingested, enough B12 can be
absorbed to prevent pernicious anemia.
d. Ca2+ absorption within the small intestine is regulated to maintain Ca2+ balance. Normally,
about 25%-80% of the daily intake of Ca2+ (1000 mg) is absorbed.
(1) Ca2+ absorption occurs via a membrane-bound carrier that is activated by vitamin D
(see also Ch 7 XIV E 1 C).
(a) Vitamin 0 3 is converted to 25-hydroxyvitamin D3 by the liver.
(b) The kidney converts the 25-hydroxyvitamin 0 3 to 1,25-dihydroxyvitamin D3 by a
process that is regulated by parathyroid hormone.
(c) 1,25-dihydroxyvitamin 0 3 then enters the enterocyte where it induces the formation
of a Ca2+ carrier that inserts on the luminal surface of the enterocyte.
(2) Ca2+ is transported out of the cell by a Ca H -ATPase active transport system and by a
Na+ -Ca H exchange system.
e. Iron absorption is necessary to maintain normal iron balance. However, very little (0.75 mg
for men and 1.5 mg for women) of the 15-25 mg of iron ingested each day is actually
absorbed.
(1) Iron is absorbed primarily within the duodenum and jejunum.
(2) Iron can be absorbed either as heme (derived from meat) or as a free ion.
(3) The ferrous ion (FeH ) is absorbed more efficiently than the ferric ion (Fe3+).
(4) Ascorbic acid (vitamin C) promotes iron absorption by reducing Fe3+ to Fe H and by
preventing iron from forming insoluble complexes within the chyme.
(5) Stomach acid tends to break insoluble iron complexes apart and thus facilitates iron
absorption.
(6) Four separate steps are involved in the transport of iron from the intestine to the plasma.
(a) First, the iron is transported across the apical membrane of the enterocyte by a
specific iron carrier system.
(b) Second, the iron binds to apoferritin, an iron-binding protein, to form ferritin.
(c) In order to leave the enterocyte, the iron must dissociate from ferritin and bind to
an intracellular carrier protein that shuttles it to the basolateral membrane, where it
is transported out of the cell.
(d) Upon entering the intestinal interstitium, the iron is transported to the plasma by
transferrin, a /3-globulin.
(7) The amount of iron absorbed depends on the amount of intracellular and extracellular
transport protein (transferrin) compared to the amount of ferritin.
(a) If a large amount of transferrin is available, iron can be transported rapidly from the
enterocyte to the plasma.
(b) If little transferrin is available, most of the iron remains trapped in the enterocyte
and is eventually excreted when the cells are desquamated.
(c) When iron stores are depleted, such as after a hemorrhage, transferrin synthesis
increases.

V. THE COLON, OR LARGE INTESTINE (Figure 6-8), absorbs some of the nutrients and most of
the fluids passed into it from the small intestine. Under normal circumstances, all but 50-100 ml of
the 1500 ml received from the small intestine is absorbed. Any nutrients or fluids that cannot be
absorbed are passed into the feces.

A. Motility
1. Function. The contractile activity of the large intestine serves two functions.
a. It enhances the efficiency of water and electrolyte absorption.
b. It promotes the excretion of the fecal material remaining in the colon.

2. Types of movements
a. Haustral shuttling

b.
c.
d.

e.

(1) Bands of muscle divide the large intestine into sac-like segments called haustrations.
Although the haustrations are present when the colon is empty, the entry of food into
the colon causes an increase in colonic contractile activity.
(2) The dynamic formation and disappearance of haustrations squeeze the chyme, moving it
back and forth along the colon in a manner similar to that described for the segmentation
contractions in the small intestine (see IV B 2 a).
Peristalsis, here, as elsewhere in the gut, is a progressive contractile wave preceded by a
wave of relaxation. Peristaltic-like segmentation contractions move the chyme very slowly
(5 cm/hr) along the colon. It can take up to 48 hours for chyme to traverse the colon.
Mass movements. OccaSionally (three to four times daily) the chyme is swept rapidly along
the colon by a peristaltic wave called a mass movement. The mass movement forces fecal
material into the rectum.
The frequency of contractions is greater in the rectum than in the sigmoid colon, causing
retrograde movement of fecal material. Because of this orad movement of fecal material,
the rectum is usually empty and material placed into it, such as a suppository, will be pushed
up into the colon.
The overall effect of the neural input to the colon is inhibitory. Thus, elimination of the
enteric nervous system, as occurs in Hirschsprung's disease, leads to a large increase in
colonic tone.

3. Defecation
a. Fecal material entering the rectum is evacuated by defecation, during which:
(1) The smooth muscles of the distal colon and rectum contract, propelling the fecal material
into the anal canal
(2) The internal and external anal sphincters both relax
(3) The abdominal and diaphragmatic muscles contract, increasing the intra-abdominal
pressure and forcing the feces through the anal canal
Transverse colon

--?-

.
Ascendln g
colon

$"

~---<~J)~~

~'~~~I~j
-

-='

,;

-t-"

,-~(....:I

,~

---<

1
~

,=,d

\..~

Taenia coli

Ileum

/: (j~
/
\ ; 'I~
t)))~'\\\ \

Descending
colon

Sigmoid
colon

~-:
~--'

~)~
~
\\.'-

Y(

\('

~ ~v

~~~jJ
l

RectumiJ~

I~

External
anal sphincter

Internal
anal sphincter
Figure 6-8. The large intestine.

b. Defecation involves both voluntary and reflex activity.


(1) When fecal material expands the rectum, a rectosphincteric reflex relaxes the anal
sphincters and generates the urge to defecate.
(2) Defecation can be prevented, however, by voluntarily contracting the external anal
sphincter (which is composed of skeletal muscle innervated by the pudendal nerves).
(3) If defecation does not occur, the internal anal sphincter closes, and the rectum relaxes
to accommodate the fecal material within it.
(4) Individuals lacking a-motoneuronal control over the external anal canal will defecate
whenever the rectum is filled with fecal material.
B. Absorption, secretion, and gas production
1. Water. The colon is unable to absorb more than 2-3 L/day. Thus, if most of the 8-10 L entering
the intestine (either as ingested water or as gastric, pancreatic, or biliary secretions) is not
absorbed in the small intestine, severe diarrhea can occur.

2. Na+ and CI-. The colon absorbs most of the Na+ and CI- that escapes absorption in the
small intestine.
3. K+, on the other hand, is secreted by the colon. Its concentration typically rises from its ileal
concentration of 9 mEq/L to 75 mEq/L by the time the fluid reaches the end of the large
intestine.
4. Aldosterone. While the small intestine has no way to regulate Na+ or K+ absorption, in the
colon, the hormone aldosterone controls these processes. Aldosterone enables the colon to
absorb all of the Na+ in the fecal fluid. However, in doing so, it causes significant amounts of
K+ to be lost from the body.
5. Intestinal gas
a. There are three sources of gas in the GI tract.
(1) Swallowed air, including air released from food and carbonated beverages, enters the
stomach, from which it is removed by eructation or passed into the intestines with
chyme.
(2) Gas is formed by bacterial action in the ileum and large intestine.
(3) Some gases diffuse into the GI tract from the bloodstream.
b. Gas in the colon differs in volume and source from gas in the small intestine.
(1) Small intestine. The small amount of gas present is usually the result of swallowed air.
This gas most likely will be passed on to the colon.
(2) Colon
(a) Colonic gas, or flatus, is produced in large volumes-up to 7-10 L/day.
(b) The gas is produced chiefly through the breakdown of undigested nutrients that
reach the colon.
(c) The main components of flatus are CO 2, CH 4 , H 2, and nitrogen gas (N 2). Since all
of these gases except N2 diffuse readily through the intestinal mucosa, the volume
of flatus expelled is reduced to about 600 ml/day.

STUDY QUESTIONS
Directions: Each of the numbered items or incomplete statements in this section is followed by answers
or by completions of the statement. Select the one lettered answer or completion that is best in each
case.
1. Which of the following secretions is most dependent on vagal stimulation?

6. Na+-dependent transport is responsible for the


absorption of all of the following EXCEPT

(A) Saliva

(A) vitamin E
(B) amino acids
(C) glucose
(0) bile salts

(B) HCI
(C) Pepsin
(0) Pancreatic juice
(E) Bile

2. The major stimulus for primary peristalsis in the


esophagus is
(A)
(B)
(C)
(0)
(E)

placing food in the esophagus


swallowing
regurgitation of food from the stomach
closing of the upper esophageal sphincter
opening of the lower esophageal sphincter

3. Which of the following will inhibit stomach


contractions?
(A)
(B)
(C)
(0)
(E)

ACh
Motilin
Gastrin
Secretin
Histamine

7. Which of the following can occur without brain


stem coordination?
(A)
(B)
(C)
(0)
(E)

Chewing
Swallowing
Primary esophageal peristalsis
Vomiting
Gastric emptying

8. The major stimulus for gastric acid (HCIl secretion during the cephalic stage is
(A)
(B)
(C)
(0)
(E)

histamine
gastrin
secretin
somatostatin
ACh

9. The major stimulus for the release of secretin is


4. Gastric acid secretion increases when food enters the stomach because
(A) protein digestion products directly stimulate
the parietal cells to release HCI
(B) food raises the pH of the stomach, allowing
more acid to be released
(C) both
(0) neither
5. Gastric parietal cells secrete
(A) gastrin
(B) motilin
(C) CCK
(0) intrinsic factor
(E) secretin

(A) protein digestion products


(B) histamine
(C) somatostatin
(0) HCI
(E) CCK

10. Fats are transported from intestinal cells to


blood plasma primarily in the form of
(A) micelles
(B) chylomicrons
(C) triglycerides
(0) fatty acids
(E) monoglycerides
11. ACh is required for the contraction of the
(A) lower esophageal sphincter
(B) upper esophageal sphincter
(C)

both

(0) neither

12. The major stimulus for receptive relaxation of


the stomach is

17. Micelle formation is necessary for absorption


of

(A) food in the stomach


(B) food in the intestine
(C) secretin
(D) CCK
(E) motilin

(A) bile salts


(B) iron
(C) cholesterol
(D) alcohol
(E) B vitamins

13. The motility pattern primarily responsible for


the propulsion of chyme along the small intestine
is

18. All of the following stimulate CCK secretion


EXCEPT

(A)
(B)
(C)
(D)
(E)

the migrating motor complex (MMC)


peristaltic waves
myogenic contractions
haustrations
segmentation

14. Intestinal motility is increased by all of the


following EXCEPT
(A)
(B)
(C)
(D)
(E)

CCK
secretin
gastrin
insulin
motilin

(A)
(B)
(C)
(D)

amino acids
fatty acids
HCI
bile acids

19. All of the following effects are caused by secretin EXCEPT


(A) stimulation of pancreatic HC0 3 - secretion
(B) enhancement of bile acid secretion
(C) potentiation of pancreatic enzyme secretion by
CCK
(D) inhibition of gastric muscle contraction

15. Gastric acid (HC!) secretion is inhibited by

20. A Na + -dependent active transport system is


necessary for the intestinal absorption of all of the
following EXCEPT

(A)
(B)
(C)
(D)
(E)

(A)
(B)
(C)
(D)
(E)

somatostatin
entero-oxynti n
high pH
amino acids
ACh

16. The major factor controlling the secretion of


bile salts from the liver is the amount of
(A)
(B)
(C)
(D)
(E)

secretin released during a meal


fat entering the small intestine
bile acids produced by the liver
bile reabsorbed from the intestine
CCK released during a meal

dipeptides
bile salts
fructose
vitamin C
glucose

21. Secondary bile acids are formed


(A) in the liver from cholesterol
(B) by the conjugation of bile acids with taurine or
glycine
(C) both
(D) neither

Directions: Each question below contains four suggested answers of which one or more is correct.
Choose the answer.
A if 1, 2, and 3 are correct
B if 1 and 3 are correct
C if 2 and 4 are correct
D if 4 is correct
E if 1, 2, 3, and 4 are correct

22. Removal of the duodenum will cause an increase in

23. Elimination of pancreatic secretions to the


small intestine may cause

(1) gastric acid (HCIl secretion

(1) fat malabsorption

(2) pancreatic HCOl

(2) increased small bowel motility

secretion

(3) gastric emptying

(3) ulceration of the duodenum

(4) gallbladder emptying

(4) increased gastric emptying

Directions: Each group of items in this section consists of lettered options followed by a set of numbered
items. For each item, select the one lettered option that is most closely associated with it. Each lettered
option may be selected once, more than once, or not at all.
Questions 24-28

Questions 29-33

For each of the following GI processes, choose the


stimulus that is most important for its regulation.

For each of the following functions of the GI tract,


choose the area where it occurs.

(A) Secretin
(B) Histamine
(c) CCK
(D) Bombesin
(E) Motilin

(A)
(B)
(C)
(D)
(E)

24.
25.
26.
27.
28.
tive

29.
30.
31.
32.
33.

Gastric acid (HeI) secretion


Gastrin secretion
Pancreatic enzyme secretion
Gallbladder emptying
Emptying of the intestine during the interdigesperiod

Fundus of the stomach


Antrum of the stomach
Duodenum of the intestine
Ileum of the intestine
Colon

Absorption of bile acids


Secretion of gastrin
Secretion of intrinsic factor
Secretion of K+
Absorption of iron

ANSWERS AND EXPLANATIONS


1. The answer is A. [/I B 3 a; 11/ C 2 c-d, 4 b (1)-(2); IV C 3, D 4 b] Salivary flow is entirely dependent
on the autonomic nervous system. Vagal stimulation produces a large volume of watery fluid, while
sympathetic stimulation causes the secretion of proteins (mucus and some enzymes). Secretion of HCi,
pepsin, pancreatic juice, and bile is influenced by vagal stimulation but can occur without it.

2. The answer is B. [/I D 7 c (2) (a)] Primary esophageal peristalsis is part of the swallowing response
and occurs whether or not food enters the esophagus. The intensity of the peristalsis, however, increases
if food is present. If the esophagus is not emptied by primary peristalsis, the presence of food in the
esophagus will initiate another peristaltic reflex, called secondary peristalsis.
3. The answer is D. [11/ B 6 b (2) (b)] Secretin has a direct inhibitory effect on the smooth muscle fibers
forming the stomach wall. Acetylcholine (ACh) and motilin, and possibly gastrin, increase the force of
stomach contractions. Histamine has no direct effect on stomach contractions.
4. The answer is C. [/II C 2 d (2)] Gastric acid secretion is increased directly by protein digestion
products and inhibited when the pH of the stomach is reduced. The buffering action of food promotes
gastric acid secretion by keeping the pH from falling too low.
5. The answer is D. [11/ C 7 c (1) (a)] Parietal cells, located in the oxyntic glands of the orad stomach
(fundus and corpus) secrete HCi and intrinsic factor. Gastrin is secreted by G cells located in the pyloric
glands of the distal stomach (antrum). Secretin and cholecystokinin (CCK) are secreted by endocrine
cells in the proximal intestine. Motilin is secreted from endocrine cells within the epithelium of the small
intestine.
6. The answer is A. [IV F 7 b (1),2 c (1),3 b (2) (d), 5 a] Amino acids and glucose are absorbed in the
proximal intestine by Na+-dependent active transport systems. Bile salts are reabsorbed by a Na+dependent active transport system located within the terminal ileum. Vitamin E is a fat-soluble vitamin
and is absorbed by passive diffusion along with other lipids in the proximal intestine.

7. The answer is E. [11/ B 5, 6 a (7)] Gastric emptying of solids occurs when the contractile activity of
the stomach reduces the size of the particles within the food sufficiently for them to pass through the
pyloric sphincter. Stomach contractions are elicited by reflexes initiated by antral distension and by
gastrin. Although the strength of the contractions is reduced by vagotomy, contractions can still occur.
Chewing, swallowing, primary (but not secondary) peristalsis, and vomiting all are coordinated by specific
regions of the brain stem.
8. The answer is E. [/II C 2 d] During the cephalic stage of gastric acid secretion, the sight, smell, or
thought of food activates cholinergic (acetylcholine-releasing) vagal fibers, which stimulate the release
of HCi from antral parietal cells.
9. The answer is D. [IV C 3 c (3) (b)] Secretin and cholecystokinin (CCK) are hormones released from
endocrine cells located in the proximal intestine. Although the release of both hormones is stimulated
by the presence of chyme in the small intestine, it is low pH resulting from HCi in the chyme that is the
major stimulus for the release of secretin. Protein digestion products (e.g., amino acids, particularly
phenylalanine) stimulate both secretin and CCK secretion, but are the major stimuli for CCK secretion.
CCK potentiates the effects of secretin but does not affect its release. Somatostatin and histamine have
no effect on CCK or secretin secretion.
10. The answer is B. [IV F 3 b (3)] Chylomicrons are small lipid droplets within the enterocytes that
are reconstituted from the lipid digestion products formed in the intestine. The chylomicrons are transported from the enterocytes by exocytosis by the intestinal fluid surrounding the intestine. The chylomicrons are then absorbed into the intestinal lacteals, from which they enter the circulation.

11. The answer is B. [1/ 0

T c (T)] The upper esophageal sphincter is composed of striated muscle and


is stimulated by cholinergic (acetylcholine-releasing) vagal fibers. The lower esophageal sphincter is
composed of smooth muscle, which normally is maintained in a contracted state by a myogenic process.

12. The answer is A. [11/ B 2 b] When food distends the orad stomach (fundus and corpus), it produces
a vago-vagal reflex by which noncholinergic, nonadrenergic fibers relax the stomach. About 2 L of food
can be accommodated in the stomach when receptive relaxation is at its maximum.
13. The answer is E. [IV B 2 a] Although the major functions of segmentation are the mixing of chyme
with digestive juices and exposing the products of digestion to the intestinal wall, segmentation is also
responsible for pushing the chyme along the intestine. Propulsion occurs because the frequency of
segmentation is higher in the more proximal intestine than it is in the distal intestine. Thus, it is more
likely for the chyme to move towards the colon than it is to move towards the stomach. Peristaltic waves
will move chyme along the intestine, but these are not frequent enough to propel the food into the colon.
The migrating motor complex (MMC) empties the intestine of the small amount of chyme remaining in
the intestine during the interdigestive period.
14. The answer is B. [IV B 4 c (2)] Secretin decreases intestinal and gastric contractile force. Cholecystokinin (CCK), gastrin, insulin, and motilin all increase intestinal contractions.
15. The answer is A. [11/ C 2 c (2)] Gastric acid (HCIl secretion is inhibited by low pH in the stomach
and by somatostatin released from interneurons within the enteric nervous system. Acetylcholine (ACh),
the hormone known as entero-oxyntin, and amino acids all stimulate gastric acid secretion.
16. The answer is D. [IV 04 b (1)] The amount of bile synthesized each day is not sufficient to absorb
all of the fat digested. However, because the bile salts are absorbed by the intestine and returned to the
liver, they can be used over and over again. The amount of bile secreted each day is thus proportional
to the amount absorbed from the intestine.
17. The answer is C. [IV F 3 b (1), (2) (b)] Micelles are necessary for absorption of dietary lipids such
as cholesterol. Bile salts, iron, and B vitamins are absorbed by membrane-bound active transport systems.
Alcohol is both water and fat soluble and so can diffuse directly across the membranes.
18. The answer is D. [IV C 3 c (3)] Amino acids, particularly phenylalanine, fatty acids, and monoglycerides all directly stimulate the release of cholecystokinin (CCK) from intestinal endocrine cells. HCI, which
lowers the pH of chyme entering the intestine, also causes CCK secretion. Bile acids do not influence
CCK secretion.
19. The answer is B. [11/ B 6 b (2); IV C 3 c (2); 04 a (1)] Although secretin regulates the volume of
biliary secretion, it neither enhances nor inhibits the secretion of bile salts. Secretin's major effect is
to stimulate HCa l - secretion by the pancreas and the liver. In addition, it potentiates the effect of
cholecystokinin (CCK) on pancreatic enzyme secretion. Secretin also diminishes gastric emptying, probably by directly reducing gastric smooth muscle contractility.

20. The answer is C. [IV F 1 b (1)-(2), 2 c (1), 3 b (2) (d), 5 b] Unlike glucose, which depends on a
Na+-dependent active transport system for absorption, fructose is absorbed by facilitated diffusion.
Active transport for fructose is not required because the intracellular fructose concentration is maintained
at a low value by intracellular enzymes that rapidly convert fructose to glucose and lactose. Bile salts,
vitamin C, and dipeptides all rely on Na+-dependent active transport systems to be absorbed.
21. The answer is D. [IV 02 a] Secondary bile acids are formed in the intestine by the deconjugation
and dehydroxylation of primary bile acids. Primary bile acids are synthesized from cholesterol in the
liver and then conjugated with taurine or glycine to form primary bile salts.

22. The answer is B (1,3). [11/ B 6 b (2), C 2 d (5) (b); IV C 3 C, 05 b (1)] The duodenum releases
cholecystokinin (CCK) and secretin when chyme enters the small intestine. Since both of these hormones
inhibit gastric emptying, removal of the duodenum will cause gastric emptying to increase. The duodenum
also releases gastrin inhibitory peptide (GIP), which inhibits the release of gastrin and HCI. Thus, removing
the duodenum also will enhance gastric acid (HCI) secretion. Because secretin stimulates pancreatic
HCal - secretion and CCK stimulates gallbladder contraction, removing the duodenum would hinder
both of these activities.
23. The answer is B (1,3). [IV C 1 b-c] Pancreatic secretions contain lipases, which are necessary for
the proper absorption of fats, and large amounts of HCa l - , which are necessary for the neutralization

of acids entering the duodenum. In the absence of HC03 - , gastric acid (HCI) will produce ulcerations
of the duodenum.

24-28. The answers are: 24-B, 25-D, 26-C, 27-C, 28-E. [III B 6 c (3), C 2 c (1), 3 b (1); IV C 3 c (1),
05 b (1)] Gastric acid (HCI) secretion is stimulated by acetylcholine (ACh), histamine, and gastrin. It is
inhibited by somatostatin. Gastrin secretion is stimulated by bombesin [also called gastrin-releasing
peptide (GRP)] and inhibited by somatostatin. Cholecystokinin (CCK) and secretin both stimulate the
pancreas. CCK, however, is responsible for the secretion of pancreatic enzymes. CCK is also responsible
for stimulating the gallbladder to contract. Motilin, a hormone released from the small intestine, increases
the strength of the migrating motor complex (MMC) and may be responsible for initiating it. The MMC,
which occurs every 60-90 minutes during the interdigestive period, starts in the stomach and sweeps
along the entire GI tract. It is thought to be responsible for emptying the small intestine of any chyme
remaining after the completion of a meal.

29-33. The answers are: 29-D, 30-B, 31-A, 32-E, 33-C. [III C 1 c (1) (a), (2); IV F 3 b (2) (d), 5 e (1);
VB 3] Bile acids are absorbed in the terminal ileum by a Na+-dependent cotransport process. Gastrin
is secreted by G cells contained in the pyloric glands of the distal stomach (antrum and pylorus). Intrinsic
factor, necessary for the absorption of vitamin B12f is secreted by parietal cells located in the proximal
stomach (fundus). The colon secretes K+ into the chyme. About 10% of the daily K+ load is excreted
by the colon; the remainder is excreted by the kidney. Iron is absorbed primarily within the duodenum
and jejunum.

7
Endocrine Physiology
John Bullock

I. GENERAL FEATURES OF HORMONES


A. Definition. In the classic definition, hormones are secretory products of the ductless glands, which
are released in catalytic amounts into the bloodstream and transported to specific target cells (or
organs), where they elicit physiologic, morphologic, and biochemical responses. In reality, the
requirement that hormones be secreted into the bloodstream is too restrictive, because they also
can act locally (Figure 7-1). For example:

1. Paracrine hormones can be conveyed over short distances by diffusion through the interstitial
space, to act on neighboring cells as regulatory substances
2. Autocrine hormones can regulate the activity of the same cells that produce them
B. Hormone-secreting tissues. Virtually all organs in the body exhibit endocrine function.

1. The most-studied endocrine organs and examples of the hormones they produce are listed in
Table 7-l.
2. Other organs with endocrine function and the hormones they produce are:
a. Heart: atrial natriuretic factor (ANF)
b. Kidney: 1,2S-dihydroxycholecalciferol (calcitriol)
c. Liver: 2S-hydroxycholecalciferol (calcidiol), somatomedin
d. Pineal gland: melatonin
e. Skin: calciferol (vitamin D 3 )
f. Gastrointestinal tract: gastrin, pancreozymin, secretin, vasoactive intestinal peptide (VIP)

C. Functions. Hormones regulate existing fundamental bodily processes but do not initiate cellular
reactions de novo. In contrast to vitamins, hormones serve no nutritive role in responsive tissues
and are not incorporated as a structural moiety into another molecule.
Table 7-1. Principal Endocrine Organs and Some Hormones They Produce
Organ

Examples of Hormones

Pituitary gland

Tropic hormones (e,g., adrenocorticotropic hormone, growth


hormone, prolactin)

Hypothalamus

Releasing hormones (e.g., thyrotropin releasing hormone),


antidiuretic hormone, oxytocin

Thyroid gland

Thyroxine, 3,S,3'-triiodothyronine

Adrenal glands

Mineralocorticoids (e.g., aldosterone), glucocorticoids (e.g.,


cortisol), catecholamines (e.g., epinephrine, norepinephrine)

Parathyroid glands

Parathyroid hormone

Gonads

Testosterone, estradiol

Pancreatic islets

Insulin, glucagon

1. Regulation of biochemical reactions. As regulators, hormones stimulate or inhibit the rate


and magnitude of biochemical reactions by their control of enzymes and, thereby, cause
morphologic, biochemical, and functional changes in target tissues. Although they are not used
as energy sources in biochemical reactions, hormones modulate energy-producing processes
and regulate the circulating levels of energy-yielding substrates (e.g., glucose, fatty acids).

2. Regulation of bodily processes. Hormones regulate growth, maturation, differentiation, regeneration, reproduction, pigmentation, behavior, metabolism, and chemical homeostasis.
Slower processes (e.g., growth, reproduction, metabolism) require longer periods of continual
hormone stimulation in contrast to rapid coordination of the body (e.g., reflex contraction of
a somatic muscle), which is regulated by the nervous system.

D. Physical characteristics
1. Chemical composition. The three major classes of hormones are: steroids, proteins and
polypeptides, and amino acid derivatives (i.e., catecholamines and thyroid hormones). No
polysaccharides or nucleic acids are known to function as hormones.

2. Plasma concentration. Hormones usually are secreted into the circulation in extremely low
concentrations.
a. Peptide hormone concentration is between 10- 12 mollL and 10- 10 mol/L.
b. Epinephrine and norepinephrine concentrations are 2 x 10- 10 mol/L and 13 x 10- 10
mol / L, respectively.
c. Steroid and thyroid hormone concentrations are 10- 9 mollL and 10- 6 mol/L,
res pectivel y.

3. latent period is the time interval between the application of a stimulus and a response. In
contrast to a latent period of 8 msec between a neural stimulus and the contraction of a muscle,
the latent period associated with hormones can be as long as seconds, minutes, hours, or days.
a. Following the administration of oxytocin, milk ejection occurs in a few seconds.

A. Endocrine signaling

~
..

Blood vessel

Extracellular
signal

Receptor

--:---;-.. .)

_ . 0. :::.::::.=~:::::~::.~;:=:..~-=--:-._~...
~

Hormone secretion
into blood by endocrine gland

Distant target cells

B. Paracrine signaling

[~" 9] 0.
"

...
@

. ~ ',', .., . ~~

........

. : .....

Q,

..
~

~.",

Secretory cell

.'

Adjacent target cell

c. Autocrine signaling

Target sites on same cell

Figure 7-1. The three methods of hormone information


transfer. Cell-to-cell signaling can occur over long distances via hormone secretion into the bloodstream (A) or
over short distances via hormone diffusion through the
interstitium (B). A hormone also may act directly on the
cell that produces it (0. (Adapted from Darnell J, Lodish
H, Baltimore D (eds): Molecular Cell Biology, 2nd edition.
New York, WH Freeman, 1990, p 710.)

b. The metabolic response to thyroxine can take as long as 3 days.


4. Postsecretory modification of hormones occurs by the proteolytic cleavage of peptide hormones or by enzymatic conversion of steroids and thyroid hormones at sites beyond the site
of secretion. This peripheral conversion to more active hormonal forms occurs in the liver,
kidney, fat, or bloodstream as well as in the target tissues themselves.
5. Circulating forms. Protein binding of hormones protects them against clearance by the kidneys,
slows the rate of degradation by the liver, and provides a circulating reserve of hormones. Only
unbound hormones pass through capillaries to produce their effects or to be degraded.
6. Hormone receptors are unique molecular groups in or on target cells that interact with
hormones to initiate a characteristic response. The specificity of a hormone depends on the
formation of a strong covalent bond with its hormone receptor (see II A).
7. Half-life. Most hormones are metabolized rapidly after secretion. In general, peptide hormones
are short-lived in the circulation, whereas steroids have a significantly longer half-life.
8. Degradation. The interaction of hormones with their target cells is followed by intracellular
degradation.
a. Degradation of protein hormones and amines occurs after binding to membrane receptors
and internalization of the hormone-receptor complex.
b. Degradation of steroids and thyroid hormones occurs after binding of the hormone-receptor
complex to the chromatin.
9. Inactivation and excretion. Only a small fraction of the circulating hormone is removed by
most target tissues. Hormone inactivation occurs in the liver and kidney.
a. Hormone degradation uses many enzymatic mechanisms such as hydrolysis, oxidation,
hydroxylation, methylation, decarboxylation, sulfation, and glucuronidation.
b. Only a small fraction 1%) of any hormone is excreted intact in the urine or feces.
E. Chemistry (Table 7-2)

1. Proteins and polypeptides generally are water soluble and circulate unbound in plasma.
a. Structure
(1) The peptide and protein hormones vary greatly in size. For example, thyrotropin
releasing hormone (TRH) is a tripeptide, whereas human chorionic gonadotropin
(HCG) consists of 243 amino acid residues.
(2) The molecular weights of the pituitary tropic hormones, which consist of about 200
amino acid residues, vary from 23,000 to 25,000 daltons. (The average molecular
weight of an amino acid residue is 120; thus, multiplying the number of residues by 120
is a good estimate of the molecular weight of a peptide or protein.)
b. Synthesis. Many of the protein-type hormones are synthesized on the rough endoplasmic
reticulum as prohormones or preprohormones. These precursor hormones undergo posttranslational cleavage by an endopeptidase within the Golgi complex prior to secretion of
the biologically active hormone.
(1) Growth hormone (GH) and prolactin are synthesized as pro hormones.
(2) Pro-opiomelanocortin (POMC), synthesized in the pituitary and hypothalamus, is a
prohormone complex that contains peptide hormone moieties including adrenocorticotropic hormone (ACTH; corticotropin), melanotropin, lipotropin, and endorphins.
(3) Insulin and parathyroid hormone (PTH) are synthesized as preprohormones, which
are hydrolyzed to prohormones and then further hydrolyzed to the hormone that is
secreted.
c. Storage and secretion. Protein and polypeptide hormones are secreted by endocrine organs
derived from ectoderm (pituitary gland; tuberoinfundibular, supraoptic, and paraventricular
nuclei) as well as organs derived from endoderm (pancreatic islets of Langerhans, thyroid
gland, parathyroid glands). The primordial cells that give rise to the parafollicular cells (C
cells) of the thyroid gland are derived from neural crest precursors.
(1) Protein and polypeptide hormones probably are stored exclusively in subcellular
membrane-bound secretory granules within the cytoplasm of endocrine cells.

The anterior and posterior lobes of the pituitary gland are derivatives of buccal ectoderm and neural ectoderm,
respectivel y.

Table 7-2. Characteristics of the Principal Classes of Hormones


Amines
Peptides

Characteristic
Solubility
In aqueous solvents
In nonaqueous solvents
Biosynthetic pathway

Steroids and Calcitriol*

Catecholamines

Thyroid Hormone

Excellent
Poor

Limited
Excellent

Good
Limited

Limited
Good

Single peptide, prohormone,


or preprohormone

Multiple enzymes

Multiple enzymes

Multiple enzymes

Postsecretory modifications

Very rare

Common

None

Common

Storage of preformed hormone

Often substantial

Minimal

Substantial

Substantial

Degradation products

Irreversibly inactive

Sometimes retain or
regain activity

Inactive

Inactive

Plasma binding proteins

Very rare

Yes

Limited

Yes

Half-life

Short (minutes)

Long (hours)

Very short (seconds)

Very long (hours to days)

Receptors

Cell surface

Nucleus

Cell surface

Nucleus

Site of action

Plasma membrane

Nucleus

Plasma membrane

Nucleus

Mechanism of action

Stimulates production of
second messenger

Stimulates production of
specific mRNAs

Stimulates production of
second messenger

Stimulates production of
specific mRNAs

Calcitriol = l,25-dihydroxyvitamin 0 3

..................... '- "TJ''''''''OT

(2) These hormones are released into the blood by exocytosis, which involves fusion of the
secretory granule and cell membrane followed by extrusion of the granular contents
into the bloodstream.
d. Half-life of various peptide/protein hormones is as follows:
(1) Antidiuretic hormone (ADH) and oxytocin: < 1 minute
(2) Insulin: 7 minutes
(3) Prolactin: 12 minutes
(4) ACTH: 15-25 minutes
(5) Luteinizing hormone (LH): 15-45 minutes
(6) Follicle-stimulating hormone (FSH): 180 minutes
2. Amino acid derivatives. Catecholamines, which are water soluble, and thyroid hormones,
which are lipid soluble, circulate in the plasma bound mainly to binding globulins.
a. Structure
(1) Catecholamines are derived from the amino acid, tyrosine. Thyroid hormones are
derived from two iodinated tyrosine residues. Hormones derived from tyrosine also are
called phenolic derivatives.
(2) Catecholamines and thyroid hormones both retain the aliphatic a-amino group. Introduction of a second hydroxyl group in the ortho- position on the benzene ring is
characteristic of the catecholamines, whereas iodination of the benzene ring distinguishes the thyroid hormones.
(3) Thyroid hormones are the only substances in the body that contain iodine.
b. Synthesis. Epinephrine and norepinephrine are synthesized in the chromaffin cells, which
are modified postganglionic neurons (see VII B-D). Thyroid hormones are synthesized in
thyroid follicular cells (see XIII A and D).
c. Storage and secretion. The amine hormones are secreted by endocrine tissues derived
from the neural crest (adrenal medulla) and from endoderm (thyroid gland).
(1) Catecholamines are stored in secretory granules. Secretion occurs when the membrane
of the chromaffin granules fuses with the plasma membrane, causing the granular
contents to be extruded into the circulation.
(2) Thyroid gland secretions are called iodothyronines-compounds resulting from the
coupling of two iodinated tyrosine molecules. Thyroid hormones are stored outside
follicular cells in the form of thyroglobulin, a glycoprotein precursor found in the lumen
of the cells. Several weeks' supply of thyroid hormone is stored in this form. Following
endocytosis and proteolysis of thyroglobulin, thyroid hormone is secreted into the
bloodstream by simple diffusion.
d. Circulation and half-life
(1) Epinephrine and norepinephrine exist in plasma either in the free form or in conjugation
with sulfate or glucuronide. Most circulating epinephrine is bound to blood proteins
(mainly albumin); norepinephrine does not bind to blood proteins to any appreciable
degree.
(2) Most thyroid hormones are bound to thyroxine-binding globulin.
(3) The half-life of various amine hormones is as follows:
(a) Epinephrine: 10 seconds
(b) Norepinephrine: 15 seconds
(c) Triiodothyronine: 1 day
(d) Thyroxine: 7 days

3. Steroid hormones are lipid soluble and circulate in the plasma bound to carrier proteins called
steroid-binding globulins.
a. Structure. Steroids are a group of biologically active substances, including androgens,
estrogens, progesterone, glucocorticoids, and mineralocorticoids. 25-Hydroxyvitamin D3
and 1,25-dihydroxyvitamin D3 are modified steroids called secosteroids.
(1) Steroids consist of three cyclohexyl rings and one cyclopentyl ring combined into a
single structure. They are derived from the cyclopentanoperhydrophenanthrene nucleus
consisting of a fully hydrogenated phenanthrene (rings A, B, and C), to which is attached
a hydrogenated cyclopentane ring (D-ring). This fully saturated, four-ring structure con
sisting of 17 carbon atoms is the hypothetical parent compound, gonane or sterane.
(2) The secosteroids, which are vitamin hormones, lack a B-ring and, therefore, consist of
two cyclohexyl rings (rings A and C) and one cyclopentyl ring (D-ring).

b. Synthesis and secretion. Steroids are synthesized and secreted by the endocrine organs
derived from mesoderm (adrenal cortex, testis, ovary). The placenta also synthesizes and
secretes steroids.
(1) Steroids are derived from cholesterol. The first, and rate-limiting, step in steroid synthesis
is conversion of cholesterol to pregnenolone. Depending on the product, hydroxylations
of the steroid nucleus may occur at carbons 11, 17, 18, or 21.
(2) There is little storage of steroids. Instead, steroid-producing cells store esterified cholesterol in the form of lipid droplets, which serve as prohormones. Steroids are released
in the circulation by simple diffusion.
c. Half-life of various steroid hormones is as follows:
(1) Aldosterone: 30 minutes
(2) Cortisol: 90-100 minutes
(3) 1,25-Dihydroxyvitamin 0 3 : 15 hours
(4) 25-Hydroxyvitamin 0 3 : 15 days

II. TYPES OF HORMONE ACTION


A. Hormone-receptor interaction. Hormones produce their effects by first combining with specific
cell components called receptors. Only cells with receptors for a specific hormone respond.
Hormones may be found on the target cell membrane (external receptors) or within the cytoplasm
or nucleus (internal receptors).

1. Mechanisms (Figure 7-2). Hormones and receptors interact in the following ways to affect
intracellular metabolism.
a. Polypeptide hormones and catecholamines bind irreversibly to fixed receptors on the outer
surface of target cells, which initiates the transduction of the hormone signal across the
membrane. Transduction occurs after activation of adenyl cyclase (see II B 1).

B. Cytoplasmic or nuclear receptors

A. Cell-surface receptors

....
Q.

Hormone-receptor
complex
,..,.-+.---:-:--- Surface

receptors

... .... -. .
.
----..
<~ .~.;.:.~:. ~~;

Low concentration
of second messengers

High concentration
of second messengers

Cytoplasmic
receptor
Nucleus
Hormone-receptor
complex

Extracellular
signal
Y

Receptor

Altered transcription
of specific genes

Figure 7-2. Mechanisms of hormone-receptor interaction. (A) Some hormones (e.g., polypeptide hormones, catecholamines) bind to specific receptors on the surface of target cells, triggering an increase or decrease in the
concentration of cyclic adenosine 3',S'-monophosphate (cAMP) or some other second messenger (e.g., calcium,
1,2-diacylglycerol>. (B) Other hormones (e.g., steroids, thyroid hormones) are transported by carrier proteins in the
blood and, once dissociated from the carrier, enter the target cell and bind to specific receptors in the cytoplasm or
nucleus. These hormone-receptor complexes then act on nuclear DNA to alter transcription of specific genes.
(Adapted from Darnell j, Lodish H, Baltimore D (eds); Molecular Cell Biology, 2nd edition. New York, WH Freeman,
1990, p 711.)

(1) Approximately 104-10 5 receptors exist on the surface of a polypeptide hormone target
cell.
(2) Polypeptide hormones that are known to enter cells include insulin, prolactin, PTH, and
gonadotropins.
(a) HCG remains bound to its receptor when it enters ovarian cells.
(b) Since receptors may enter cells, it is possible that receptors (not hormones) are the
biologically active component. For example, insulin antibody causes the insulin
receptor to change its configuration, allowing the receptors to exert insulin-like
effects on cells.
b. Steroids enter target cells and bind to specific mobile receptors in the cytoplasm. The
hormone-receptor complex then binds to specific DNA sequences to initiate transcription
and translation (see II B 2). Approximately 3000 to 104 intracellular receptors exist per
steroid target cell.
c. Thyroid hormones enter target cells and combine with nuclear receptors. The hormonereceptor complex then binds to specific DNA sequences to initiate transcription and translation (see II B 2).
2. Effects
a. Increased membrane permeability. Hormonal control of membrane permeability is well
documented for ions and for metabolites such as glucose and amino acids.
(1) The increased entry of glucose into cells, which is mediated by a special transport
system, is the major method by which insulin controls glucose utilization by muscle.
(2) Other peptide hormones that change membrane permeability include GH, ACTH, calcitonin, thyroid-stimulating hormone (TSH), and thyroxine.
(3) Steroids that alter membrane permeability include mineralocorticoids, glucocorticoids,
estrogens, and androgens.
b. Changes in receptor number
(1) Hormone-sensitive cells respond to high concentrations of certain hormones by reducing the number of cell surface receptors, thus decreasing sensitivity to the circulating
hormone. For example, elevated ambient insulin concentrations cause a loss or inactivation of insulin receptors in liver cells, fat cells, and white blood cells.
(2) Catecholamines exert their effects via plasma membrane receptors, and thyroid hormones have receptors in the target cell nucleus. Excess thyroid hormone leads to an
increased number of catecholamine receptors in the myocardium of experimental animals. This may explain the catecholamine-like effects noted in hyperthyroid patients
who produce normal amounts of catecholamines.
(a) Hyperthyroid patients have tachycardia and palpitations, effects observed with
excessive catecholamines (pheochromocytoma). These cardiac symptoms can be
ameliorated by the administration of a ,a-blocker such as propranolol.
(b) The increase in cardiac ,a-adrenergic receptors in hyperthyroidism makes the heart
more responsive to catecholamines.
(3) Angiotensin II decreases the number of its receptors on adrenocortical cells and increases the number of its receptors in vascular smooth muscle.
B. Mechanisms of information transfer: distant signals. For hormones that communicate via the
bloodstream, two mechanisms for hormone-receptor coupling operate.
1. Cyclic AMP-mediated hormone activity (see Figure 7-2A) involves cyclic adenosine 3',5'monophosphate (cAMP) as a second messenger that changes enzyme activities. cAMP is the
intracellular nucleotide messenger that mediates the effects of certain hormones on subcellular
processes, leading to a variety of physiologic responses.
a. Most polypeptide hormones, many biogenic amines, and some prostaglandins activate their
target cells by stimulating the synthesis of cAMP, which causes enzyme phosphorylation, a
reaction usually associated with enzyme activation. The first messenger is the hormone that
binds to the membrane receptor and leads to activation of membrane-bound adenyl
cyclase. This enzyme converts adenosine triphosphate (ATP) to cAMP in the presence of
magnesium ion (MgH).
b. cAMP exerts biologic activity via the phosphorylation of cAMP-dependent protein kinases.
(1) Kinases are a family of enzymes that phosphorylate their substrate. Protein kinases, a
subgroup of the kinases, can be soluble or membrane-bound. Protein kinases transfer a

Co

phosphate group from ATP to the hydroxyl group of the substrate. Only the protein
kinases that are regulated by cAMP are called cAMP-dependent protein kinases.
(2) The phosphorylation of a cAMP-dependent protein kinase can lead to the activation
(e.g., via phosphorylase kinase, phosphorylase, and triglyceride lipase) or the inactivation (e.g., via glycogen synthetase and pyruvate dehydrogenase) of the substrate.
(3) The effects of the cAMP-dependent protein kinases on their substrates are reversed by a
group of enzymes called phosphoprotein phosphatases, which remove the phosphate
group from the protein enzyme by hydrolysis.
(4) The enzyme that inactivates cAMP is called cyclic nucleotide phosphodiesterase.
Most of the phosphodiesterases are cytosolic enzymes.
An example of cAMP-mediated hormone activity is the hormone regulation of glycogen
metabolism. Glucagon and epinephrine stimulate glycogenolysis and also inhibit glycogen
synthesis. * Insulin has the opposite effect on both processes.
(1) The enzymes that promote glycogenolysis are active in their phospho- form and inactive
in their dephospho- form. The reverse is true for glycogen synthetase, which is the
major enzyme in glycogen synthesis.
(2) In muscle, a cAMP-dependent protein kinase is activated either by catecholamines (via
their ~-adrenergic receptors) or by glucagon (via its receptor in the liver).
(3) This activated protein kinase phosphorylates another protein kinase, phosphorylase
kinase, thereby activating it.
(4) The activated phosphorylase kinase phosphorylates the enzyme, phosphorylase, which
initiates glycogen breakdown. Phosphorylase kinase also phosphorylates glycogen synthetase, thereby inactivating it.

2. Transcription and translation effects (see Figure 7-2B). Steroids and thyroid hormones modulate transcription in specific areas of the nuclear chromatin by interacting with DNA molecules
in the chromatin to cause enzyme induction. Steroid-receptor complexes are translocated
through the cytosol and enter the nucleus. Thyroid hormones enter the nucleus in the free state
and then combine with nuclear receptors.
a. Chromatin consists of DNA, histone proteins, and nonhistone (acidic) proteins. The specificity of nuclear binding is a property of a particular acidic protein.
b. As a result of the interaction of steroid and thyroid hormones with the chromatin, transcription is stimulated and specific messenger RNA (mRNA) synthesis increases.
c. The specific mRNAs enter the cytoplasm, where they direct the synthesis (translation) of
specific proteins. These proteins may be enzymes, structural proteins, receptor proteins, or
secretory proteins.

C. Mechanisms of information transfer: local signals. Some cell-to-cell signaling occurs over very
small distances.
1. Paracrine communication (see Figure 7-1 B) involves local diffusion of a peptide or other
regulatory molecule to its target cell through the interstitium. Target cells are in the vicinity of
the paracrine cell that releases the messenger. Although paracrine substances may diffuse into
the blood, it is not necessary that they do.
a. Somatostatin is a paracrine substance whose secretion is stimulated by glucose, glucagon,
and gut hormones. Somatostatin is enzymatically degraded in the blood, a process that
protects distant cells from the paracrine substance, should it enter the blood.
b. Signal transmission also occurs through the release of peptides into the lumen of the gastrointestinal tract, where these peptides interact with endocrine cells to cause the release of a
second endocrine or paracrine messenger. Gastrin, somatostatin, and substance P are released into the gut lumen following nerve stimulation. Possibly, these substances are released
as precursor molecules that are activated by digestive enzymes. Gastrin also is secreted into
the bloodstream.
2. Neurocrine communication involves the release of chemical messengers from nerve terminals. Neurocrine substances may reach their target cells via one of three routes.
a. The neurotransmitter can be released directly into the intercellular space, cross the
synaptic junction, and inhibit or activate the postsynaptic cell.

"The glycogenolytic effect of epinephrine in human liver probably occurs via activation of a cAMP-independent
phosphorylase.

(1) Neurocrine substances are inactivated by degrading enzymes and by reuptake of the

substances by neurons.
(2) Examples of neurocrine substances secreted by this route are acetylcholine (ACh) and
norepinephrine.
b. A neural signal also can be transferred via a gap junction, which is a membrane specialization between nerve cells, between nerve terminals and endocrine cells, and between
endocrine cells. Gap junctions allow the movement of small molecules and electric signals
from one cell to another, creating a functional syncytium.
c. The third potential route for the transmission of a neural signal is identical to the classic
neurosecretory mechanism, which involves the release of a peptide or neurohormone
from a neurosecretory neuron into the blood followed by the interaction of this neurohormone with specific receptors on distant target cells (see III C-D). Examples of such neurocrine
substances are oxytocin and ADH. The effector sites of neurohormones are not always
endocrine cells.
3. Autocrine communication (see Figure 7-1 C). Autacoid is a term used to designate a compound that is synthesized at, or close to, its site of action. This is in contrast to the circulating
hormones, which act on tissues distant from their site of synthesis. Prostaglandins are autacoids.

III. BASIC CONCEPTS OF ENDOCRINE CONTROL


A. Homeostasis and steady state. A major function of the endocrine system is to maintain the
homeostasis of the internal environment. This condition of relative constancy in the concentration
of dissolved substances, in temperature, and in pH is a basic requirement for the normal function
of cells.

1. The concept of homeostasis as a constancy of physiologic variables must be modified, because


many regulated organismic processes are not constant but conform to a persistent endogenous
or exogenous rhythm. For example, humans demonstrate a circadian pattern in the levels of
plasma 17-hydroxycorticosteroids. Such 24-hour cycles are not solely a response to fluctuating
environmental stimuli but also are due to internal endogenous oscillators whose phases are
influenced by environmental stimuli.
a. In humans, certain corticosteroids (e.g., cortisol) have a rhythmic pattern of secretion, with
secretory rates highest early in the morning and lowest late at night. Accordingly, plasma
cortisol concentration is at a peak between 6 A.M. and 8 A.M. and at a nadir between
midnight and 2 A.M.
(1) This circadian rhythm persists but shifts to correspond with a change in sleeping habit
(e.g., during illness, night work, changes in longitude, and total bed rest or confinement).
(2) For this reason, treatment of patients with exogenous corticosteroids is on an alternateday dosage regimen, whereby the entire dose is given in the morning of every other
day. This dosage schedule simulates the normal adrenocortical secretory rhythm.
b. The rhythmic pattern of corticosteroid secretion occurs in isolated adrenal glands and even
in single adrenocortical cells.
2. The term steady state indicates that a function or a system is unvarying with time, but that the
system is not in true equilibrium. The system is said to be in a dynamic equilibrium, because
matter and energy flow into the system at a rate equal to that at which matter and energy flow
out of the system.
B. Hypothalamic-hypophysial axes and feedback control. The hypothalamus has neural control
over hormone secretion by the posterior lobe of the pituitary gland. The secretory activity of the
anterior lobe is controlled by hypothalamic hormones, which are secreted into the
hypothalamic-hypophysial portal system (the hypophysial portal system). Only those hypothalamic hormones that regulate the anterior pituitary are hypophysiotropic hormones.

1. Hypophysial portal system (Figure 7-3).The median eminence has a poorly developed bloodbrain barrier, and there is relatively little arterial blood perfusing the cells of the anterior lobe.
The blood supply of the anterior lobe is derived from branches of the internal carotid arteries
(mainly the superior hypophysial artery).
a. The posterior lobe derives its blood from a capillary plexus emanating from the inferior
hypophysial artery. This capillary plexus drains into the dural sinus. The neural tissue of the
upper infundibular stem (neural stalk) and of the median eminence is supplied largely by

Anterior lobe

Posterior lobe

Inferior hypophysial artery

Short portal
veins

Figure 7-3. Anatomic relationship of


the hypothalamus and pituitary gland,
showing the hypophysial portal system and neurons involved in control
of the pituitary gland. Neuron five (5)
represents the peptidergic neurons of
the supraopticohypophysial and paraventriculohypophysial tracts. Neurons four (4) and three (3) are the peptidergic neurons of the tuberohypophysial tract. Neuron one (1) and
neuron two (2) are monoaminergic
neurons. (Adapted from Gay VL: The
hypothalamus: physiology and clinical use of releasing factors. Fertil Steril
23:51,1972.)

branches of the superior hypophysial artery. The median eminence is the specialized area
of the hypothalamus located beneath the inferior portion of the third ventricle. It is a release
center for hypophysiotropic hormones.
b. The primary capillary plexus, which emanates from the superior hypophysial artery, forms
a set of long portal veins that carry blood downward into the anterior lobe.
(1) The portal veins, which give rise to the secondary capillary plexus, constitute about
90% of the blood supply to the cells of the anterior lobe. The secondary capillary plexus
drains into the dural sinus.
(2) The anterior lobe receives its remaining blood from the short portal veins, which originate in the capillary plexus of the inferior hypophysial artery at the base of the infundibular stem.
2. Feedback control is an important mechanism regulating hormone synthesis and secretion
(Figure 7-4).
a. Hypothalamic-pituitary-target gland model. The paradigm for feedback control is the
interaction of the pituitary gland with target endocrine tissues (thyroid gland, adrenal cortex,
gonads). Unbound circulating hormones produced by target endocrine organs inhibit the
hypothalamic-pituitary system, causing a decrease in the secretion of pituitary tropic hormones, which, in turn, control the secretion by the endocrine target glands. Virtually a"
hormone secretions are controlled by some type of feedback control.
b. Negative feedback control occurs on three levels.
(1) Long-loop feedback. Peripheral gland hormones and substrates arising from tissue
metabolism can exert long-loop feedback control on both the hypothalamus and the
anterior lobe of the pituitary gland. long-loop feedback usually is negative but occasionally can be positive and is particularly important in the control of thyroidal, adrenocortical, and gonadal secretions.
(2) Short-loop feedback. Negative feedback also can be exerted by the anterior pituitary
tropic hormones on the synthesis or release of the hypothalamic releasing or inhibiting
hormones, which co"ectively are called hypophysiotropic hormones.
(3) Ultrashort-loop feedback. Evidence suggests that the hypophysiotropic hormones may
inhibit their own synthesis and secretion via a control system referred to as ultrashortloop feedback.

C. Neurosecretory neurons (see Figure 7-3). Neural control of the pituitary gland is exerted through
neurohumoral secretions that arise from specialized neurosecretory neurons (peptidergic neurons)
and are carried by the bloodstream to a target site.

Pituitary tropic
hormone

Bioaminergic neuron
= Peptidergic neuron

Figure 7-4. The three levels of feedback mechanisms for


controlling hormone secretion: long-loop feedback (A),
short loop feedback (8), and ultrashort-loop feedback (0.
Plus signs indicate stimulation, and minus signs indicate
negative feedback.

1. Structure and function


a. Neurosecretory neurons are glandular, unmyelinated neurosecretory cells with two functions.
(1) They function as typical neurons, in that they conduct action potentials.
(2) They also function as endocrine glands, in that they synthesize and release neurohormones either directly into the general circulation (as in the case of the neurosecretory
neurons of the pars nervosa) or into a portal system (as in the case of the hypophysiotropic neurons, which release their neurohormones into the primary plexus of the
hypophysial portal system).
b. A neurosecretory cell system consists ofaxons that terminate directly on or near blood
vessels. This differentiates these cells from typical neurons, which release neurotransmitters
at localized synaptic regions. The functional complex of a neurosecretory neuron together
with a blood vessel (hemocoele) is called a neurohemal organ.
2. Classification. Neurosecretory neurons in humans are restricted to the hypothalamus, where
they occur as two distinct populations of cells that secrete neurohormones (Tables 7-3, 7-4).
a. The magnocellular neurosecretory system refers to the neurosecretory neurons of the
supraoptic and paraventricular nuclei, which together form the supraopticohypophysial
tract. Magnocellular neurons synthesize and secrete the neurohormones ADH and oxytocin.
b. The parvicellular (also called parvocellular) neurosecretory system refers to the neurosecretory neurons of the tuberoinfundibular tract. These neurons of the medial basal hypothalamus have axons that terminate directly on the capillaries of the portal vessels in the median
eminence, and they form a final common pathway for neuroendocrine function.
(1) The parvicellular neurosecretory neurons mainly are peptidergic. An important exception, however, is the dopaminergic neurosecretory neurons that form and secrete
prolactin-inhibiting factor (PI F).
(2) The secretory products of the parvicellular neurosecretory neurons are called hypophysiotropic hormones.

Table 7-3. Neuroendocrine Transducer Systems


Hormone

Neuroendocrine System
Magnocellular neurosecretory neurons
Posterior lobe
Supraoptic
Paraventricular

Antidiuretic hormone
Oxytocin

Parvicellular neurosecretory neurons


Median eminence

Hypophysiotropic hormones

Preganglionic fibers
Adrenal medulla

Epinephrine*

Postganglionic fibers
Pineal gland
Juxtaglomerular apparatus

Melatonin t
Renin t

After Martin JB, et al: Neuroendocrine transducers and neurosecretion. In Clinical Endocrinology. Philadelphia, FA
Davis, 1977, p 4.
Acetylcholine is the neurotransmitter preceding epinephrine release.
tNorepinephrine precedes the release of both melatonin and renin.

3. Neural control. Neural information is transmitted to the parvicellular and magnocellular neurosecretory cells by monoaminergic neurons. Most of the cell bodies of the monoaminergic
neurons are located in the mesencephalon and lower brain stem.
a. The monoaminergic neurons that innervate the parvicellular neurons produce and secrete
biogenic amines, which modulate the hypothalamic release of the hypophysiotropic hormones.
b. The function of the magnocellular neurosecretory neurons is controlled by cholinergic and
noradrenergic neurotransmitters.
(1) ACh stimulates the release of ADH and oxytocin.
(2) Norepinephrine inhibits the secretion of ADH and oxytocin.
c. Since the secretion of the parvicellular and magnocellular peptidergic neurons is regulated
by biogenic amines, the neurosecretory neurons can correctly be viewed as neuroeffector
cells.
D. Neuroendocrine transducers are endocrine glands that convert neural signals into hormonal
signals. Neural control of endocrine tissues occurs in three ways (Figure 7-5).
1. Direct innervation by autonomic secretomotor neurons
a. Pancreatic islets of Langerhans
(1) The islets of Langerhans have a postganglionic parasympathetic innervation. Increased
vagal activity to the beta cells stimulates insulin release only during periods of elevated
blood sugar.
Table 7-4. Characteristic Features of the Neurosecretory Control Systems
Feature

Parvicellular System

Magnocellular System

Neural input

Norepinephrine, dopamine,
serotonin

Acetylcholine

Nuclei

Arcuate nucleus

Supraoptic and paraventricular


nuclei

Tract

Tuberoinfundibular

Supraopticohypophysial

Terminus

Median eminence, upper


infundibular stem

Pars nervosa (infundibular


process)

Neurohormones

Neuropeptides (hypophysiotropic
hormones), polypeptides

Neuropeptides (arginine-ADH,
oxytocin), nonapeptides

Type of endocrine neuron

Peptidergic

Peptidergic

Stimuli

Monoamines

Acetylcholine

Vascular elements

Hypophysial portal system

Capillary bed (systemic)

Preganglionic
neuron

Preganglionic :
neuron

ACh

ACh

Postganglionic
neuron

Adrenomedullary
cell

Tropic hormones

Posterior
lobe

Figure 7-5. The three types of neuroendocrine transducers. (A) Secretomotor neurons control endocrine glands by
direct innervation via autonomic fibers; the adrenal medulla is the only autonomic neuroeffector that is innervated
by preganglionic neurons. AI = the parasympathetic nervous system; A2 = the sympathoadrenomedullary axis:
ACh = acetylcholine. (8) Magnocellular neurosecretory neurons control the posterior lobe of the pituitary gland, and
(0 parvicellular neurosecretory neurons control the anterior lobe. (Reprinted from Martin JB, et al: Neuroendocrine
transducers and neurosecretion. In Neuroendocrinology. Philadelphia, FA Davis, 1977, p 5.)
(2) The islets of Langerhans also have a postganglionic sympathetic innervation. When the
sympathetic nerves to the beta cells are stimulated or when norepinephrine or epinephrine is infused, the predominant effect is inhibition of insulin secretion.
b. Pineal gland. This endocrine structure of the diencephalon is classified as a periventricular
organ because it borders on the third ventricle.
(1) The pinealocytes are innervated by the postganglionic (adrenergic) sympathetic fibers,
which originate in the superior cervical ganglia of the sympathetic chain.
(2) When the neurotransmitter norepinephrine is released by the autonomic fibers, it stimulates the synthesis and release of melatonin and other indoleamine hormones.
(3) The pineal gland, like other periventricular organs (e.g., the median eminence), has a
poorly developed blood-brain barrier.
c. Juxtaglomerular cells. These granular cells of the juxtaglomerular apparatus receive a
postganglionic input, which, when stimulated, leads to the release of the proteolytic enzyme
renin. Renin can be classified, according to the neuroendocrine transduction concept, as a
hormone.
d. Adrenal medulla. This endocrine structure is composed of chromaffin cells and is innervated
by preganglionic (cholinergic) sympathetic fibers, which, when stimulated, cause the release
of the adrenomedullary hormones epinephrine and norepinephrine. The release of ACh at
the synapses causes the secretion of these catecholamines.
2. Magnocellular neurosecretory regulation of the posterior lobe (see Table 7-4)
a. Depolarization of the magnocellular neurosecretory cells by ACh released at synapses on
the cell bodies of these neurons causes the release of ADH and oxytocin. The axons of
these neurons terminate directly on the blood vessels of the posterior lobe.

b. The neural input to the cell bodies of the magnocellular neurons is cholinergic, and the
hormonal output consists of peptide hormones.
3. Parvicellular neurosecretory regulation of the anterior lobe (see Table 7-4)
a. The anterior lobe lacks a direct nerve supply, but the pituitary gland does possess an
innervation. The neurons in the anterior lobe are exclusively postganglionic sympathetic,
which are vasomotor fibers and not secretomotor fibers.
(1) The hypothalamic regulation of the anterior lobe is achieved through the tuberohypophysial neurons of the medial basal hypothalamus. These peptidergic neurons synthesize
and secrete specific hypophysiotropic hormones, which enter the hypophysial portal
system and stimulate or inhibit the secretion of anterior pituitary hormones.
(2) The arcuate nucleus (nucleus infundibularis) is the main site of origin of the fine unmyelinated axons of the tuberoinfundibular pathway; however, tuberohypophysial neurons
exist throughout the hypophysiotropic area, including the ventromedial nuclei and the
periventricular area. The arcuate nucleus serves as the final neural link in the neurovascular connection between the hypothalamus and the anterior lobe.
b. The cells of the intermediate lobe, unlike those of the anterior lobe, receive a direct bioaminergic secretomotor supply from the hypothalamus and are not perfused directly by the
hypophysial portal system.

IV. ENDOCRINE CELLS OF THE GASTROENTEROPANCREATIC (GEP) SYSTEM


A. Neuropeptides of the GEP system
1. The endocrine cells of the gut produce peptides that also are found in a variety of other tissues,
including the pancreas, pituitary gland, and central and peripheral nerves. The same peptide
might function as both a gut hormone and a neurotransmitter. The pancreatic hormones insulin,
glucagon, and somatostatin are considered to be part of the GEP system.
2. The cells of the GEP system share with certain neural tissues the capacity of amine precursor
uptake and decarboxylation and, therefore, are called APUD cells. APUD cells are located in
the gastrointestinal tract and pancreatic islets.
a. The endocrine cells of the gastrointestinal tract are found singly or in small groups and are
dispersed among the other epithelial cells of the mucosa.
b . The GEP cells usually are situated at or near the base of the intestinal glands.
8. APUD cells represent a group of neurons and endocrine cells, which take up amino acids and
modify them into amines and peptides. APUD cells synthesize and secrete all of the hormones of
the body except the steroids.
1. Most APUD cells are of ectodermal origin. Those in the gut and pancreas, however, originate
from endoderm. Some APUD cells, such as those of the adrenal medulla and neurons of
sympathetic ganglia, are derived from the neuroectodermal component called the neural crest.
2. Many APUD endocrine cells contain dopamine, serotonin, and histamine as well as polypeptide
hormones, and these substances tend to be released together.
a. In some cases, enough hormone is secreted to enter the plasma and be transported to other
tissues in the body and, therefore, an endocrine function is performed.
b. In other instances, little hormone is bound to distant receptors (due to too few receptors,
too little available hormone to bind them, or both) and, therefore, a paracrine action results.

V. ENDOGENOUS OPIOID PEPTIDES


A. Terminology
1. Opioid denotes opiate-like in terms of a functional similarity with a chemical dissimilarity.
a. Opioid peptides refer to endogenous or synthetic compounds that have a spectrum of
pharmacologic activity similar to that of morphine.
b. Since the opioid peptides bind with morphine receptors in the central nervous system (eNS),
they are called morphinomimetic peptides.

2. The two chemical groups of opioid peptides are called endorphins and enkephalins.
a. The term endorphin originally was used to designate all opiate-like compounds occurring in
the brain. This term now is restricted to the specific endogenous morphine-like substances,
a-, ~-, and y-endorphins.
b. The enkephalins include met-enkephalin, leu-en kephalin, dynorphin, and a-neo-endorphin.

B. Endorphins
1. Chemistry. Endorphins are structural derivatives of ~-lipotropic hormone (~-LPH). The
lipotropin-related peptides (~-LPH and ~-endorphin) and ACTH have a common glycoprotein
precursor molecule (molecular weight of about 31,000 daltons), which is proopiomelanocortin (POMC).
a. POMC is activated by proteolytic cleavage to yield 10 individual peptides that have been
classified into four groups:
(1) ACTH and corticotropin-like intermediate lobe peptide (CLIP)
(2) Lipotropic hormones (lipotropins), represented by ~- and y-LPH
(3) Melanocyte-stimulating hormones (MSHs), represented by a-, ~-, and y-MSH
(4) Opioid peptides or endorphins, represented by a-, ~- and y-endorphins
b. POMC consists of three major chemical moieties.
(1) The N-terminal fragment (16 K) is cleaved to form y-MSH.
(2) ACTH consists of the fragment containing amino acid residues 1-39.
(a) ACTH is cleaved to produce a-MSH, which consists of the fragment containing
amino acid residues 1-13. In humans, a-MSH normally is not synthesized in significant quantities.
(b) ACTH also is cleaved into CLIP, which consists of the fragment containing amino
acid residues 18-39.
(3) ~-LPH consists of the fragment containing amino acid residues 1-91. ~-LPH is the
precursor of the endogenous opioids, y-LPH and ~-endorphin.
(a) y-LPH consists of the ~-LPH fragment containing amino acid residues 1-58.
(i) In nonhuman species, y-LPH is converted by proteolytic cleavage to ~-MSH,
which is the fragment containing amino acid residues 41-58.
(ii) ~-MSH and a-MSH are not formed in humans in significant quantities because
the intermediate lobe is vestigial in adult humans. ~-MSH does not exist as such
in normal human plasma. *
(b) ~-Endorphin consists of the ~-LPH fragment containing amino acid residues 6191.
(i) ~-Endorphin is cleaved into y-endorphin, which is the fragment containing amino
acid residues 61-77.
(ii) y-Endorphin forms a-endorphin, which is the fragment containing amino acid
residues 61-76.
2. Distribution. The endogenous opioids (endorphins) do not cross the blood-brain barrier.
a. POMC is synthesized in the anterior and intermediate lobes of the pituitary gland, in the
hypothalamus and other areas of the brain, and in several peripheral tissues including the
placenta, gastrointestinal tract, and lung.
(1) In the human pituitary gland, POMC exists principally in the anterior lobe, where it is
synthesized by basophils. POMC is the precursor of ACTH and ~-LPH, which are
secreted together from the basophils.
(a) ACTH and ~-LPH occur within the same pituitary cell and possibly within the same
secretory granule.
(b) The intermediate lobe does not produce ACTH and ~-LPH as final secretory
products.
(2) In the brain, the concentrations of ACTH and ~-LPH are much lower than in the anterior
lobe, and all neural cells or fibers that contain ACTH also contain ~-LPH. The highest
extrapituitary concentrations of ACTH, a-MSH, ~-LPH, y-LPH, and ~-endorphin exist
in the hypothalamus followed by the limbic system.

*,8-MSH has been isolated from the human pituitary gland, but the MSH activity in human plasma probably is due
largely to the MSH-related peptides, ,8-LPH and ,},-LPH.

b. J3-Endorphin
(1) J3-Endorphin is the principal opioid peptide in the pituitary gland, and it exists in the
highest concentration in the intermediate lobe of experimental animals. In the human
pituitary, J3-endorphin generally is confined to the cells of the anterior lobe. In response
to acute stress, the pituitary gland secretes concomitantly ACTH and J3-endorphin.
(2) Other sites
(a) J3-Endorphin has been found in the human pancreas, placenta, semen, and in the
male reproductive tract.
(b) Immunoassayable J3-endorphin also exists in the plasma and cerebrospinal fluid
(CSF).
c. CNS endorphins. The endorphin system exists in the CNS and is characterized by long
fiber projection systems from the arcuate region of the hypothalamus to the periventricular
region.
(1) Only in the arcuate area do ACTH, J3-LPH, and the endorphins occur within cell bodies
of neurons. Therefore, endorphin cell bodies exist only in the ventral hypothalamus.
(2) Outside the arcuate area, the endorphins are found within the fibers of neurons, which
project mainly to the mesencephalic periaqueductal gray area. Other areas of projection
include the periventricular thalamus, medial amygdala, locus ceruleus, and the zona
incerta.
(3) Brain J3-endorphin content is unaltered by hypophysectomy, suggesting that the pituitary
gland is not the source of CNS endorphins.
3. Physiologic effects
a. Opiate-receptor binding. Endorphins bind to opiate receptors in the brain to cause analgesia, sedation, respiratory depression, and miosis (pupillary contraction).
(1) The intraventricular administration of J3-endorphin in humans relieves intractable pain.
(a) It is believed that this analgesic effect involves the pituitary release of endorphins
because the effect is abolished with hypophysectomy.
(b) The characteristic effect of opiates in humans is less a specific blunting of pain than
it is an induced state of indifference or emotional detachment from the experience
of suffering.
(2) The intracerebral administration of endorphins produces a profound sedation and immobilization (catatonia).
(3) Endorphins playa central role in controlling affective states and may be involved in
controlling the drive for food, water, and sex, all of which are associated with the limbic
nervous system.
b. Stimulation of the hypothalamus. The intracisternal or intracerebral injection of J3-endorphin elicits a hypophysiotropic effect via the stimulation of the hypothalamus.
(1) This hypothalamic effect on pituitary function causes the secretion of GH, prolactin,
ACTH, and ADH.
(2) In addition, there is a diminished pituitary secretion of TSH and the gonadotropic
hormones, LH and FSH.
(3) J3-Endorphin in the gastrointestinal tract stimulates the secretion of glucagon and insulin
but inhibits the secretion of somatostatin.
c. J3-LPH is a more potent stimulator of aldosterone synthesis than is angiotensin II, and
J3-LPH is thought to be the stimulator of the zona glomerulosa in primary aldosteronism.
d. J3-LPH, 'Y-LPH, and MSH have a lipolytic effect resulting in fat mobilization; however,
these compounds do not have significant regulatory effects on fat metabolism in humans.
(1) J3-MSH has been isolated from the human pituitary gland.
(2) MSH activity in human plasma is due largely to the MSH-related peptides, J3-LPH and
'Y- LPH .
e. J3-LPH, 'Y-LPH, and ACTH have a weak MSH activity, which explains the hyperpigmentation associated with increased ACTH secretion.

C. Enkephalins
1. Chemistry
a. Enkephalins, represented by met-enkephalin and leu-enkephalin, are pentapeptide opioids,
which are not breakdown products of POMe. The precursor substance for met- and leuenkephalin is proenkephalin A.

b. Proenkephalin B contains the sequences of a-neo-endorphin, dynorphins A and B, and


leu-enkephalin. Although a-neo-endorphin and dynorphin contain the leu-en kephalin sequence, neither is considered a precursor for leu-enkephalin.
2. Distribution
a. The pituitary gland is virtually devoid of enkephalins.
(1) Of the pituitary enkephalins, including dynorphin (amino acid residues 1-13), the greatest concentrations exist in the pars nervosa.
(2) Brain enkephalins are not depleted following hypophysectomy, suggesting a neural
origin for these peptides.
b. eNS. Enkephalins usually are localized in neuronal processes and terminals. The enkephalincontaining neurons have the widest distribution throughout the CNS and are characterized
by short axon projections. Dynorphin also is distributed widely in the CNS, with the highest
concentrations in the hypothalamus, medulla-pons, midbrain, and spinal cord.
(1) The highest concentrations of met-enkephalin are in the basal ganglia (globus pallidus)
and substantia nigra. *
(2) In the spinal cord, enkephalins exist in highest concentration in the dorsal gray matter,
which corresponds to the synapses of primary sensory nerve endings. Enkephalins also
are found in other areas of the spinal cord, notably the substantia gelatinosa, which is
known to be involved in the transmission of pain impulses.
(3) Many limbic structures have relatively high enkephalin levels, including the lateral
septal nucleus, the interstitial nucleus of the striae terminalis, the prefornical area, and
the central amygdala.
c. Other sites
(1) Enkephalin-like material has been demonstrated in the CSF and has been isolated
from the adrenal medulla, where it exists in high concentrations both in axon terminals
of the splanchnic nerve and in adrenomedullary chromaffin cells.
(a) Met-enkephalin in the circulation originates from the adrenal medulla.
(b) High plasma concentrations of met-en kephalin are found in patients with pheochromocytoma.
(2) Enkephalin immunoreactivity has been reported in the human gastrointestinal tract
(myenteric plexus and mucosa), gallbladder, and pancreas.
(3) Ectopic production of enkephalins has been reported in carcinoid tumors of the lung
and thymus.
3. Physiologic effects. Enkephalins are most appropriately classified as neurotransmitters or
neuromodulators. A neuromodulator is a substance that modifies (positively or negatively) the
action of a neurotransmitter at a presynaptic site (by modulating the release of a neurotransmitter) or at a postsynaptic site (by modulating the neurotransmitter action).
a. Spinal cord. The enkephalins in the dorsal gray matter of the spinal cord function to suppress
substance P-containing nerve endings and provide an analgesic (anti nociceptive) role. Electric stimulation of the periaqueductal gray area of the spinal cord of humans, which causes
analgesia, is associated with an increase in the concentration of enkephalins in the CSF.
(1) Opioids are believed to modulate pain impulses by acting to reduce sensitivity to pain
peripherally as well as centrally by activating opiate receptors in the periaqueductal
gray area.
(2) Acupuncture is associated with the release of enkephalins into the CSF.
b. Medulla. The vagal nuclear localization of enkephalins corresponds to the emetic actions
and antitussive properties of morphine.
c. Brain stem. Morphine and enkephalins inhibit the firing of the noradrenergic neurons of the
locus ceruleus, which is the origin of the ascending noradrenergic fibers.
(1) The localization of enkephalins in the locus ceruleus may account for the euphoriaproducing actions of morphine.
(2) The amygdala is considered the prime site for morphine-generated euphoria.
(3) The enkephalin tracts and opiate receptors in the limbic system may explain the euphoric
effects of opiates.

'The substantia nigra is classified by some neuroanatomists as a component of the basal ganglia.

(4) Respiratory depression, which accounts for the lethal effects of opiates, may involve
receptors in the nucleus solitarius of the brain stem, which regulates visceral reflexes
including respiration.

VI. PITUITARY GLAND


A. Embryology. The pituitary gland is in close anatomic relation to the hypothalamus. This relationship has both embryologic and functional significance.
1. The anterior lobe of the pituitary gland, the adenohypophysis, is derived from the primitive
gut by an upward extension (Rathke's pouch) of the epithelium of the primitive mouth cavity
(stomodeum). The adenohypophysis is a derivative of buccal ectoderm.
2. The neural or posterior lobe, the neurohypophysis, develops as a downward evagination of
the neural tube at the base of the hypothalamus (infundibulum) and, therefore, represents a
true extension of the brain. Neuroregulation of this structure is achieved by direct neural
connections. The neurohypophysis is a derivative of neural ectoderm.
B. Morphology
1. Gross anatomy (see Figure 7-3)
a. General structure
(1) The pituitary gland lies in a bony walled cavity, the sella turcica, in the sphenoid bone
at the base of the skull.
(2) The dura mater completely lines the sella turcica and nearly surrounds the gland.
(3) The pituitary (hypophysial) stalk and its blood vessels reach the main body of the
gland through the diaphragma sellae. The pituitary stalk consists of the infundibular stem
and the adenohypophysial tissue that is continuous with the infundibular stem.
b. Adenohypophysial structure. The anterior lobe has three components.
(1) The pars distalis represents the bulk of the anterior lobe in humans and receives most
of its blood supply from the superior (anterior) hypophysial artery, which gives rise to
the hypophysial portal system.
(2) The pars intermedia lies between the pars distalis and the neural lobe and is a vestigial
structure in humans. It is relatively avascular and is considered almost nonexistent in
humans.
(3) The pars tuberalis is an elongated collection of secretory cells, which superficially
envelops the infundibular stem and extends upward as far as the basal hypothalamus.
It is the most vascular portion of the anterior lobe.
c. Neurohypophysial structure
(1) Components
(a) The median eminence, located beneath the third ventricle, is a small, highly vascular protrusion of the dome-shaped base of the hypothalamus, which is designated
grossly as the tuber cinereum. The floor of the third ventricle is designated as the
infundibulum because it resembles a funnel.
(b) The infundibular stem (neural stalk) of the posterior lobe arises in the median
eminence.
(c) The pars nervosa retains its neural connection with the ventral diencephalon.
(2) Dominant features of the posterior lobe are the neurosecretory neurons that form
the magnocellular neurosecretory system. These unmyelinated nerve tracts arise from
the supraoptic and paraventricular nuclei within the ventral diencephalon and descend
through the infundibulum and neural stalk to terminate in the posterior lobe. The posterior lobe is a storage site for hormones and, therefore, is not correctly termed an
endocrine gland.
2. Histology
a. Cells. Two major cell types are found in equal numbers in the anterior lobe.
(1) Chromophils (granular secretory cells) exist in two forms.
(a) Acidophils (eosinophils) account for about 80% of the chromophils and are the
cellular source of prolactin and GH.
(b) Basophils comprise about 20% of the chromophils and are the source of TSH,
ACTH, LH, FSH, and J3-LPH.

(2) Chromophobes (agranular cells) are not precursors of the chromophils and are now
known to have an active secretory function. Most of these cells likely are degranulated
secretory cells
b. Neurons in the anterior lobe are almost exclusively postganglionic sympathetic fibers that
innervate blood vessels.
c. Nerve fibers of the neurohypophysial system terminate mostly in the pars nervosa. Interspersed between these neurosecretory fibers are numerous glial cells called pituicytes,
whose function, other than structural support, remains unknown.
3. Vascular supply (see also iii Bland Figure 7-3). A basic tenet of the neurovascular hypothesis
is that the concentration of the hypophysiotropic hormones is greater in hypophysial portal
blood than at any other site in the vasculature.
a. In humans, the capillaries at the base of the hypothalamus are formed directly from branches
of the superior hypophysial arteries, which arise from the internal carotid arteries. There are
few vascular anastomoses between the hypothalamic artery and the superior hypophysial
artery. The crucial regulatory connection between the hypothalamus and the anterior lobe
is via the hypophysial portal vessels.
b. The intermediate lobe is not perfused directly by the hypophysial portal system but is
regulated by bioaminergic secretomotor fibers originating in the hypothalamus.
c. The blood supply to the posterior lobe is largely separate from that of the anterior lobe. The
blood supply to the median eminence is greater than that to the entire pituitary gland.

C. Hormones of the posterior lobe: ADH and oxytocin (see Table 7-4). The physiologic aspects
of ADH are described in section Viii of Chapter 4. The physiologic aspects of oxytocin are
described below.
1. Synthesis and storage
a. Like ADH, oxytocin is a nonapeptide* that is synthesized within the cell bodies of the
peptidergic neurons of the magnocellular neurosecretory system.
b. This polypeptide is synthesized mainly in the paraventricular nuclei of the hypothalamus
and, like ADH, is stored in the posterior lobe of the pituitary gland.
2. Stimuli for release. Oxytocin secretion is brought about by stimulation of cholinergic nerve
fibers.
a. Stimulation of the tactile receptors in the areolar region of the female breast during suckling
activates somesthetic neural pathways, which transmit this signal to the hypothalamus. This
leads to the reflex secretion of oxytocin into the bloodstream and to milk release following
a latent period of 30-60 seconds. This reflex is called the milk let-down or milk ejection
reflex.
(1) Oxytocin causes milk release in lactating women by contraction of the myoepithelial
cells, which cover the stromal surface of the epithelium of the alveoli, ducts, and
cisternae of the mammary gland.
(2) Oxytocin secretion can be conditioned so that the physical stimulation of the nipple no
longer is required. Thus, lactating women can experience milk release in response to
the sight and sound of a baby.
(3) While oxytocin aids in the process, its presence is not absolutely required for successful
nursing in humans.
b. Oxytocin secretion also can occur in response to genital tract stimulation, such as that which
occurs during coitus and parturition.
c. Oxytocin is produced in men and also is released during genital tract stimulation. The role
of this neurohypophysial hormone in men is unknown.
3. Inhibition of release
a. Milk let-down can be inhibited by emotional stress and psychic factors such as fright.
b. Excitation of adrenergic fibers to the hypothalamus inhibits peptide release. Activation of
the sympathetic neurons with the concomitant release of norepinephrine and epinephrine
inhibits oxytocin secretion.
c. Ethanol inhibits endogenous oxytocin release, resulting in reduced myometrial contractility.
d. Enkephalins also inhibit oxytocin release.

*If the two cysteine residues are counted together as a single cystine residue, ADH and oxytocin are correctly
classified as octapeptides.

4. Physiologic effects
a. Oxytocin stimulates contraction of the smooth muscle (myoepithelium) of the lactating
mammary gland.
b. It also stimulates contraction of the smooth muscle of the uterus (myometrium).
(1) The sensitivity of the myometrium to exogenous oxytocin during pregnancy increases
as pregnancy advances.
(2) Oxytocin plays a role in labor and has been shown to be a useful therapeutic agent in
the induction of labor.
D. Hormones of the anterior lobe. The principal hormones of the anterior lobe of the pituitary
gland can be classified conceptually into two groups: hormones that stimulate other endocrine
glands to secrete hormones and hormones that have a direct effect on nonendocrine target tissues.
Only the latter group is described here, using GH and prolactin as examples.
1. GH also is known as human growth hormone (HGH), somatotropic hormone (STH), somatotropin, and somatocrinin.
a. Synthesis, chemistry, and general characteristics
(1) GH is synthesized by the acidophils of the anterior lobe and is stored in very large
amounts in the human pituitary gland. GH represents approximately 4%-10% of the
wet weight of the pituitary gland, which is equivalent to 5-15 mg.
(2) GH is a single, unbranched polypeptide chain containing 191 amino acid residues. It
has been synthesized in bacteria using recombinant DNA techniques.
(3) Humans exhibit a species specificity for GH, and only human and monkey GH preparations have biologic activity in humans.
(4) Like all other pituitary hormones, GH is secreted episodically in periods of 20-30
minutes. The large diurnal fluctuations represent integrations of many small secretory
episodes. A regular nocturnal peak in GH secretion occurs 1-2 hours after the onset of
deep sleep, which correlates with stage 3 or stage 4 slow-wave sleep.
(5) The plasma GH concentration in the growing child is not significantly higher than that
in the adult whose growth has ceased.
b. Control of secretion. The release of GH is primarily under the control of two hypophysiotropic hormones.
(1) Stimuli for release. Somatotropin releasing factor (SRF) is the putative releasing hormone for GH. SRF has not been identified or chemically synthesized. However, several
pharmacologic, physiologic, and psychic agents are known to stimulate GH release.
(a) Release of GH is mediated by monoaminergic and serotoninergic pathways; thus,
a-adrenergic, dopaminergic, and serotoninergic agonists as well as ,a-adrenergic
antagonists all stimulate GH release in humans.
(b) Bromocriptine (a dopamine agonist), enkephalins, endorphins (,a-endorphin), and
opiates stimulate GH secretion.
(c) Insulin-induced hypoglycemia is a potent stimulus of GH secretion as are pharmacologic doses of glucagon and vasopressin.
(d) Physiologic stimuli include hypoglycemia, increased plasma concentrations of
amino acids (arginine, leucine, lysine, tryptophan, and 5-hydroxytryptophan), and
decreased free fatty acid concentrations. In addition, estrogens stimulate GH synthesis and secretion.
(e) GH secretion is stimulated by moderate to vigorous exercise, emotional stress, and
stress due to fever, surgery, anesthesia, trauma, pyrogen administration, and repeated venipuncture. Fasting leads to elevated GH secretion after 2 or 3 days.
(2) Inhibitors of secretion
(a) GH can inhibit its own secretion via a short-feedback loop mechanism that operates
between the anterior lobe and the median eminence. Somatotropin-inhibiting hormone (SIH; somatostatin) inhibits the synthesis and release of GH.
(i) Somatostatin is a tetradecapeptide (14 amino acid residues) that has been chemically synthesized.
(ii) It is a product of the parvicellular neurosecretory neurons that terminate in the
median eminence and produce hypophysiotropic hormones.
(iii) Somatostatin also is found in other parts of the brain, in the gastrointestinal tract,
and in the delta cells of the pancreatic islets.
(iv) In addition to inhibiting GH secretion, somatostatin blocks the secretion of insulin,
glucagon, and gastrin and inhibits the intestinal absorption of glucose. These
effects of somatostatin produce a state of hypoglycemia.

(b) The secretion of GH in response to the aforementioned stimuli often is blunted in


obese individuals.
(c) Glucocorticoids decrease GH secretion, but their predominant effect is the interference with the metabolic actions of GH.
(d) A decline in GH secretion is observed in late pregnancy, despite the presence of
high estrogen levels.
(i) Impaired glucose tolerance is common, and clinical diabetes occurs frequently,
despite the above normal insulin secretion in response to a glucose load during
pregnancy.
(ii) Pregnancy regularly antagonizes the action of insulin and increases the pancreatic
secretory capacity of both normal and diabetic individuals.
(iii) The development of gestational diabetes probably is due to a greater degree of
insulin antagonism caused by normal plasma concentrations of human placental
lactogen (HPL).
c. Physiologic and metabolic effects
(1) Stimulation of growth of bone, cartilage, and connective tissue
(a) The effects of GH on skeletal growth are mediated by a family of polypeptides
called somatomedins [also termed insulin-like growth factors (lGF), which are
synthesized mainly in the liver. (Thyroid hormone and insulin also are necessary for
normal osteogenesis.)
(b) Somatomedin may be produced in non hepatic tissue as well, in that somatomedin
activity has been found in the serum, kidney, and muscle tissue.
(i) Receptors for somatomedin exist in chondrocytes, hepatocytes, adipocytes, and
muscle cells.
.
(ii) Somatomedin has insulin-like effects on tissues, including lipogenesis in adipose
tissue, increased glucose oxidation in fat, and increased glucose and amino acid
transport by muscle.
(c) GH, through somatomedin, stimulates proliferation of chondrocytes and the appearance of osteoblasts. The increase in the thickness of the epiphysial (cartilaginous)
end-plate accounts for the increase in linear skeletal growth.
(d) After epiphysial fusion, bone length can no longer be increased by GH, but bone
thickening can occur through periosteal growth. It is this growth that accounts for
the changes seen in hypersomatotropism (acromegaly).
(e) These reactions are the biochemical correlates of protein synthesis in general body
growth and also account for the hyperplasia and hypertrophy associated with increased tissue mass.
(2) Protein metabolism
(a) GH has predominantly anabolic effects on skeletal and cardiac muscle, where it
stimulates the synthesis of protein, RNA, and DNA.
(b) GH reduces circulating levels of amino acids and urea (i.e., it promotes nitrogen
retention), which accounts for the term positive nitrogen balance. Urinary urea
concentration also is decreased.
(c) GH promotes amino acid transport and incorporation into proteins.
(3) Fat metabolism
(a) GH has an overall catabolic effect on adipose tissue. It stimulates the mobilization
of fatty acids from adipose tissue, leading to a decreased triglyceride content of fatty
tissue and increased plasma levels of free fatty acids and glycerol.
(b) GH increases hepatic oxidation of fatty acids to the ketone bodies, acetoacetate
and /J-hydroxybutyrate.
(4) Carbohydrate metabolism. GH is a diabetogenic hormone. Because of its anti-insulin
effect, GH has the tendency to cause hyperglycemia.
(a) GH can produce an insulin-resistant diabetes mellitus primarily because of its lipolytic effect.
(i) Free fatty acids can antagonize the effect of insulin to promote glucose uptake
by skeletal muscle and adipose tissue.
(ii) Free fatty acids can stimulate gluconeogenesis.
(iii) Excess acetyl coenzyme A (acetyl-CoA) production favors gluconeogenesis, because pyruvate carboxylase requires acetyl-CoA to form oxaloacetate from pyruvate. Oxaloacetate is the rate-limiting factor in gluconeogenesis.
(iv) Acetyl-CoA also inhibits glycolysis by inhibition of pyruvate kinase.

(v) Free fatty acids stimulate hepatic glucose synthesis mainly via the stimulation of

fructose diphosphatase. At the same time, pyruvate kinase and phosphofructokinase both are inhibited by free fatty acids, which block glycolysis and favor
gluconeogenesis. Citrate also blocks glycolysis at the phosphofructokinase step.
(b) GH induces an elevation in basal plasma insulin levels.
(c) Because of its anti-insulin effect, GH inhibits glucose transport in adipose tissue.
Since adipose tissue requires glucose for triglyceride synthesis, GH antagonizes
insulin-stimulated lipogenesis.
(5) Mineral metabolism. GH promotes renal reabsorption of Ca2+, phosphate, and Na+.
d. Endocrinopathies
(1) Disorders associated with increased GH
(a) Growth retardation can occur when GH levels are increased and somatomedin
levels are depressed (e.g., in kwashiokror). (In the African pygmy, who is resistant
to the action of GH, both GH and somatomedin levels are normal. This condition
is due to a decrease in GH receptors.)
(b) Overproduction of GH during adolescence results in giantism, which is characterized by excessive growth of the long bones. Patients may grow to heights of as
much as 8 feet.
(c) Excessive GH secretion during adulthood, after the epiphysial plates of long bones
have fused, causes growth in those areas where cartilage persists. This leads to
acromegaly, a condition characterized by coarse facial features, underbite, prominent brow, enlarged hands and feet, and soft tissue hypertrophy (e.g., cardiomegaly,
hepatosplenomegaly, and renomegaly).
(2) Disorders associated with decreased GH
(a) Decreased GH secretion in immature persons leads to stunted growth, or dwarfism,
which is accompanied by sexual immaturity, hypothyroidism, and adrenal insufficiency.
(b) GH deficiency may be part of an overall lack of anterior pituitary hormones (panhypopituitarism) or due to an isolated genetic deficiency. Selective GH deficiency is
rare in adults; clinical manifestations may include impaired hair growth and a tendency toward fasting hypoglycemia.
(3) Treatment
(a) The treatment of choice for hypersomatotropism is selective surgical extirpation of
the pituitary adenoma without damage to other pituitary functions. Bromocriptine
is effective in suppressing, but not normalizing, GH levels in most acromegalic
patients. This substance tends to stimulate GH secretion in normal individuals.
(b) Disorders associated with GH deficiency can be treated with HGH.
2. Prolactin also is known as lactogenic hormone, mammotropic hormone, and galactopoietic
hormone.
a. Synthesis, chemistry, and general characteristics
(1) Prolactin is synthesized in the pituitary acidophils.
(2) Human prolactin is a single peptide chain containing 198 amino acid residues.
(3) It does not regulate the function of a secondary endocrine gland in humans.
b. Control of secretion. Two hypothalamic neurosecretory substances have been implicated
in prolactin secretion.
(1) Stimuli for release. Prolactin releasing factor (PRF) is the putative releasing hormone
for prolactin. PRF has not been identified or chemically synthesized; however, one of
these releasing factors is TRH, which causes the release of TSH.
(a) Prolactin secretion increases about 1 hour after the onset of sleep, and this increase
continues throughout the sleep period. The nocturnal peak occurs later than that
for GH.
(b) Prolactin secretion is enhanced by exercise and by stresses such as surgery under
general anesthesia, myocardial infarction, and repeated venipuncture.
(c) Plasma prolactin levels begin to increase by the eighth week of pregnancy and
usually reach peak concentrations by the thirty-eighth week.
(d) Nursing and breast stimulation are known to stimulate prolactin release. Oxytocin
does not stimulate prolactin secretion.
(e) Serum prolactin levels are elevated in those patients with primary hypothyroidism
who are believed to have high TRH levels in the hypophysial portal circulation.

(f) Dopamine antagonists (phenothiazine and tranquilizers), adrenergic blockers, and

serotonin agonists stimulate prolactin secretion.


(g) Pituitary stalk section and lesions that interfere with the portal circulation to the
pituitary gland also cause prolactin release.
(2) Inhibitors of release. Normally, the control of prolactin secretion is under constant
inhibition via prolactin inhibiting factor (PI F).
(a) Dopamine is secreted into the hypophysial portal vessels and may be the PIF. It is
physiologically the most important PIF.
(b) Serotonin antagonists and dopamine agonists (bromocriptine) block the secretion of
prolactin. Bromocriptine administered during the postpartum period reduces prolactin secretion to nonlactating levels and terminates lactation.
c. Physiologic effects
(1) Prolactin does not have an important role in maintaining the secretory function of the
corpus luteum and, therefore, is not a gonadotropic hormone in women.
(2) Prolactin plays an important role in the development of the mammary gland and in milk
synthesis.
(a) During pregnancy, the mammary duct gives rise to lobules of alveoli, which are the
secretory structures of this tissue. This differentiation requires prolactin, estrogens,
and progestogens. Once the lobuloalveolar system is developed, the role of prolactin
and corticosteroids in milk production, although essential, becomes minimal. GH
and thyroid hormone enhance milk secretion.
(b) Immediately following pregnancy, prolactin stimulates galactosyltransferase activity,
leading to the synthesis of lactose.
(c) In women, high serum levels of prolactin are associated with suppressed LH secretion and anovulation, which account for an absence of menses (amenorrhea) during
postpartum lactation.
(i) With continued nursing, FSH levels rise, but LH levels remain low.
(ii) In the early postpartum period, both the FSH and LH levels are low.
d. Endocrinopathies
(1) Hyperprolactinemia is not a rare condition but frequently is undiagnosed because
galactorrhea occurs in only about 30% of cases.
(a) In women, elevated serum prolactin manifests as infertility and amenorrhea.
(b) In men, hyperprolactinemia is a cause of impotence and decreased libido.
(2) Treatment of prolactin hypersecretion includes administration of bromocriptine, which
lowers prolactin levels and usually restores normal gonadal function.

VII. ADRENAL MEDULLA


A. Embryology
1. The neural crest gives rise to neuroblasts, which eventually give rise to the autonomic postganglionic neurons, the adrenal medulla, and the spinal ganglia.
2. The adrenal medulla consists of chromaffin cells (pheochromocytes), which are neuroectodermal derivatives and the functional analogs of the sympathetic postganglionic fibers of the
autonomic nervous system.
3. In early fetal life, the adrenal medulla contains only norepinephrine.
B. Morphology
1. Gross anatomy
a. The adrenal medulla represents essentially an enlarged and specialized sympathetic ganglion
and is called a neuroendocrine transducer because a neural signal to this organ evokes
hormonal secretion.
b. The adrenal medulla is the only autonomic neuroeffector organ without a two-neuron motor
innervation. It is innervated by long sympathetic preganglionic, cholinergic neurons that
form synaptic connections with the chromaffin cells.
c. Small clumps of chromaffin cells also can be found outside the adrenal medulla, along the
aorta and the chain of sympathetic ganglia.

2. Histology
a. Cells. There are two types of adrenomedullary chromaffin cells. Individual cells contain
either norepinephrine or epinephrine, which is stored largely in subcellular particles called
chromaffin granules. These granules are osmiophilic, electron-dense, membrane-bound secretory vesicles.
(1) Approximately 80% of the chromaffin granules in the human adrenal medulla synthesize
epinephrine (adrenalin). The remaining 20% synthesize norepinephrine (noradrenalin).
(2) The chromaffin granules contain catecholamines, protein, lipids, and adenine nucleotides (mainly ATP).
(a) One of the proteins localized in the particulate fraction is the enzyme, dopamine-,Bhydroxylase.
(b) Soluble acidic proteins found in the granules are called chromogranins.

b. Neurons
(1) The preganglionic sympathetic fibers that innervate the adrenal medulla traverse the

splanchnic nerve, which contains myelinated (type B) secretomotor fibers emanating


mainly from lower thoracic segments (T5 to T9) of the ipsilateral intermediolateral gray
column of the spinal cord.
(2) The cell bodies of the chromaffin cells do not have axons.
3. Vascular supply (Figure 7-6)
a. Arterial blood to the adrenal gland reaches the outer capsule from branches of the renal
and phrenic arteries, with a less important arterial input directly from the aorta. The adrenal
medulla is perfused by blood vessels in two ways.
(1) A type of portal circulation exists in the adrenal gland where the cortex and medulla
are in juxtaposition. From the capillary plexus on the outer adrenal capsule most of the
blood enters venous sinuses, which drain into and supply the medullary tissue. Thus,
most of the blood perfusing the adrenal medulla is derived from the portal system and
is partly deoxygenated.
(2) There also exists a direct arterial blood supply to the medulla via the medullary arteries,
which traverse the cortex.
b. Venous blood drains via a single central vein, composed almost entirely of bundles of
longitudinal smooth muscle fibers, which passes along the longitudinal axis of the gland.

C. Adrenomedullary hormones: monoamines


1. The adrenal medulla synthesizes and secretes biogenic amines. These dihydroxylated phenolic
amines, or catecholamines, are epinephrine and norepinephrine. Most of the met-en kephalin
in the circulation also originates in the adrenal medulla. Enkephalins are pentapeptides that
function as neurotransmitters or neuromodulators, which normally are localized in neuronal
processes and terminals (see V C).
a. Epinephrine is produced almost exclusively in the adrenal medulla, with smaller amounts
synthesized in the brain. Essentially all circulating epinephrine is derived from the adrenal
medulla.
b. Norepinephrine is widely distributed in neural tissues, including the adrenal medulla, sympathetic postganglionic fibers, and eNS. In the brain, the concentration of norepinephrine

Figure 7-6. The adrenal portal vascular system constitutes a functional


connection between the cortex and
medulla and has a high cortisol concentration. NE = norepinephrine;
E =epinephrine; ACTH = adrenocorticotropic hormone. (Adapted from
Pohorecky LA, Wurtman RJ: Adrenocortical control of epinephrine synthesis. Pharmacal Rev 23(1):1-35,
ACTH

1971.)

is the highest in the hypothalamus. The norepinephrine content of a tissue reflects the extent
or density of its sympathetic innervation. Norepinephrine has been demonstrated in almost
all tissues except the placenta, which is devoid of nerve fibers.
2. Bilaterally adrenalectomized human patients excrete practically no epinephrine in the urine.
Urinary levels of norepinephrine remain within normal limits, however, indicating that the
norepinephrine comes from extra-adrenal sources (i.e., the terminals of the postganglionic
sympathetic fibers and the brain).

D. Control of catecholamine synthesis


1. The biosynthetic pathway originates with L-tyrosine, which is derived from the diet or from
the hepatic hydroxylation of L-phenylalanine. Tyrosine is hydroxylated in the cytoplasm by
tyrosine hydroxylase to L-dopa (3,4-dihydroxyphenylalanine).
2. Dopa is converted in the cytosol to dopamine (3,4-dihydroxyphenylethylamine) by a nonspecific aromatic L-amino acid decarboxylase.
3. Dopamine enters the chromaffin granule, where it is converted to L-norepinephrine by
dopamine-J3-hydroxylase, which exists exclusively in the granule.
a. Norepinephrine is the end product in approximately 20% of chromaffin cells.
b. In about 80% of chromaffin cells, norepinephrine diffuses back into the chromaffin cytoplasm. There, it is N-methylated by phenylethanolamine-N-methyltransferase (PNMT) using
S-adenosylmethionine as a methyl donor to form L-epinephrine.
(1) PNMT is selectively localized in the adrenal medulla, the only site where it exists in
significant concentrations.
(2) PNMT activity is induced by very high local concentrations of glucocorticoids, which
are found only in the adrenal portal blood draining the adrenal cortex.
E. Control of catecholamine secretion
1. General considerations
a. ACh provides the major physiologic stimulus for the secretion of the adrenomedullary
hormones. In addition, angiotensin II, histamine, and bradykinin stimulate catecholamine
secretion.
(1) Catecholamine release is stimulated by ACh from the preganglionic sympathetic nerve
endings innervating the chromaffin cells.
(2) The final common effector pathway activating the adrenal medulla is the cholinergic
preganglionic fibers in the greater splanchnic nerve.
(3) ACh causes the depolarization of the chromaffin cells followed by the release of catecholamines by exocytosis. Ca2+ influx secondary to membrane depolarization is the
central event in stimulus-secretion coupling.
b. Cortisol and ACTH. Because catecholamine synthesis is dependent on cortisol, the functional integrity of the adrenal medulla indirectly depends on a functional pituitary gland for
ACTH secretion and a functional median eminence for corticotropin releasing hormone
(CRH) secretion. CRH is a hypophysiotropic hormone produced by the parvicellular nuclei
of the ventral diencephalon.
2. Physiologic and psychological stimuli for release. The adrenal medulla constitutes the
neuroeffector of the sympathoadrenomedullary axis that is activated during states of emergency.
This response to stress is called the fight-or-flight reaction. Among the conditions in which
the sympathetic nervous system is activated are fear, anxiety, pain, trauma, hemorrhage and
fluid loss, asphyxia and hypoxia, changes in blood pH, extreme cold or heat, severe exercise,
hypoglycemia, and hypotension. During hypoglycemia, the adrenal medulla is activated selectively. In humans, epinephrine and norepinephrine appear to be released independently by
specific stimuli.
a. Anger and active aggressive states or situations are associated with increased norepinephrine
secretion.
b. States of anxiety are associated with increased epinephrine secretion. In addition, epinephrine release is increased by tense but passive emotional displays or threatening situations of
an unpredictable nature.
c. Angiotensin II potentiates the release of catecholamines
d. Plasma concentrations of epinephrine vary according to physiologic or pathologic state as
follows:

(1) Basal level: 25-50 pg/ml (6 x 10- 10 mol/U


(2) Hypoglycemia: 230 pg/ml
(3) Diabetic ketoacidosis: 500 pg/ml
(4) Severe hypoglycemia: 1500 pg/ml

3. Regulation of adrenergic receptors. A reciprocal relationship exists between catecholamine


concentration and the number and function of adrenergic receptors.
a. A sustained decrease in catecholamine secretion is associated with an increased number of
adrenergic receptors in target cells and an increased responsiveness to catecholamines.
Conversely, a chronic increase in catecholamine secretion is associated with a decreased
number of adrenergic receptors in target cells and a decreased responsiveness to catecholamines.
b. This relationship may account in part for the phenomenon of denervation hypersensitivity,
which is observed in sympathetic neuroeffectors following autonomic fiber denervation.
F. Metabolism and inactivation of circulating catecholamines

1. General considerations. The plasma half-life of epinephrine and norepinephrine is 10 seconds


and 15 seconds, respectively. The biologic effects of circulating catecholamines are terminated
rapidly by both nonenzymatic and enzymatic mechanisms.
a. Neuronal uptake. Sympathetic nerve endings have the capacity to take up amines actively
from the circulation. This uptake of circulating catecholamines leads to nonenzymatic inactivation by intraneuronal storage and to enzymatic inactivation by a mitochondrial enzyme
called monoamine oxidase (MAO).
b. Extraneuronal uptake. The formation of catecholamine metabolites locally in innervated
tissues and systemically in the liver, kidney, lung, and gut implies catecholamine uptake by
a variety of cells.
c. Inactivation. Circulating epinephrine and norepinephrine are metabolized predominantly
in the liver and kidney.
2. Metabolic pathways for catecholamine inactivation
a. MAO is found in very high concentrations in the mitochondria of the liver, kidney, stomach,
and intestine. MAO catalyzes the oxidative deamination of a number of biogenic amines,
including the intraneuronal and circulating catecholamines.
(1) The combined actions of MAO and aldehyde oxidase on epinephrine and norepinephrine produce 3,4-dihydroxymandelic acid by oxidative deamination.
(2) The combined actions of MAO and aldehyde oxidase on the meta-O-methylated metabolites of epinephrine and norepinephrine (metanephrine and normetanephrine, respectively) produce 3-methoxy-4-hydroxymandelic acid (vanillylmandelic acid or VMA) by
oxidative deamination.
b. Catechol-O-methyltransferase (COMT) is found in the soluble fraction of tissue homogenates with the highest levels in liver and kidney. COMT is considered mainly as an extraneuronal enzyme, but it is also found in postsynaptic membranes. COMT metabolizes circulating
catecholamines in the kidney and liver and metabolizes locally released norepinephrine in
the effector tissue.
(1) COMT requires S-adenosylmethionine as a methyl donor.
(2) The action of COMT produces normetanephrine from norepinephrine, metanephrine
from epinephrine, and VMA from 3,4-dihydroxymandelic acid by 3-0-methylation.
3. Significance of catecholamine metabolites
a. Only 2%-3% of the catecholamines are excreted directly into the urine, mostly in conjugation with sulfuric or glucuronic acid. Most of the catecholamines produced daily are excreted
as the deaminated metabolites, VMA and 3-methoxy-4-hydroxyphenylglycol (MOPC). Only
a small fraction is excreted unchanged or as metanephrines.
b. Under normal circumstances, epinephrine accounts for a very small proportion of urinary
VMA and MOPC. Because the majority is derived from norepinephrine, urinary VMA and
MOPC reflect the activity of the nerve terminals of the sympathetic nervous system rather
than that of the adrenal medulla.
c. The excretion of unchanged epinephrine or plasma epinephrine provides a better index of
the physiologic activity of the sympathoadrenomedullary system than does the excretion of
catecholamine metabolites, since the latter reflects, to a considerable extent, norepinephrine
that is metabolized within nerve endings and the brain and never released at adrenergic
synapses in the active form.

G. Physiologic actions of catecholamines (Table 7-5). The effects of adrenomedullary stimulation


and sympathetic nerve stimulation generally are similar. In some tissues, however, epinephrine
and norepinephrine produce different effects due to the existence of two types of adrenergic
receptors, alpha (a) and beta ({3) receptors, which have different sensitivities for the various
catecholamines and, therefore, produce different responses. Epinephrine is the single most active
endogenous amine on both a and /3 receptors.
1. The a-adrenergic receptors are sensitive to both epinephrine and norepinephrine. These
receptors are associated with most of the excitatory functions of the body and with at least
one inhibitory function (i.e., inhibition of intestinal motility).
2. The /3-adrenergic receptors respond to epinephrine and, in general, are relatively insensitive
to norepinephrine. These receptors are associated with most of the inhibitory functions of the
body and with one important excitatory function (i.e., excitation of the myocardium).
H. Biochemical effects of catecholamines. Norepinephrine has little direct effect on carbohydrate
metabolism; however, both norepinephrine and epinephrine can inhibit glucose-induced secretion
of insulin from the beta cells of the pancreatic islets of Langerhans.

Table 7-5. Some Physiologic Effects of Catecholamines and Types of Adrenergic Receptors
Effector Organ

Receptor
Type

Eye
Radial muscle
Ciliary muscle

Response
Contraction (mydriasis)
Relaxation for far vision

Heart
Sinoatrial node

Increase in heart rate (increase in rate of diastolic


depolarization and decrease in duration of phase 4 of
sinoatrial nodal action potential)

Atrioventricular node

Increase in conduction velocity and shortening of


functional refractory period

Atria
Ventricles

Increase in contractility
Increase in contractility and irritability

Blood vessels

Constriction (arterioles and veins)


Dilation (predominates in skeletal muscle)*

Bronchial muscle

Relaxation (bronchodilation)

Gastrointestinal tract
Stomach
Intestine
Sphincters

/3

a,/3
a

Urinary bladder
Detrusor muscle
Trigone and sphincter
Skin
Pilomotor muscles
Sweat glands

Relaxation
Contraction

a
a

Uterus
Liver

Decrease in motility
Decrease in motility
Contraction

Piloerection
Selective stimulation (adrenergic sweating)*
Contraction
Relaxation

Glycogenolysis

Muscle

Glycogenolysis

Pancreatic islets

Inhibition of insulin secretion


Stimulation of insulin secretion

Adapted from Morgan HE: Function of the adrenal glands. In Best and Taylor's Physiological Basis of Medical Practice,
10th edition. Edited by Brobeck JR. Baltimore, Williams & Wilkins, 1979.
*Mediated via sympathetic cholinergic fibers.

1. Carbohydrate metabolism. Because hepatic stores of glycogen are limited (about 100 g) and
decrease only transiently after epinephrine activation, lactate derived from muscle glycogen
(300 g) is the major precursor for hepatic gluconeogenesis, the process that sustains hepatic
glucose formation and secretion. Gluconeogenesis mainly accounts for the hyperglycemic
action of epinephrine in normal physiologic states. In pathologic states (e.g., pheochromocytoma), the diabetogenic action of catecholamines is caused by the inhibition of insulin secretion
and the gluconeogenic effect of these hormones (usually norepinephrine), which are secreted
in excessive amounts. Since propranolol attenuates hyperlactacidemia and hyperglycemia, this
implies that epinephrine-induced glycogenolysis in muscle and in the liver is mediated by the
{3 and a receptors, respectively (see Table 7-5).

a. Glycogenolysis in the liver


(1) Epinephrine stimulates glycogenolysis in the liver via the Ca2+-activated glycogen phos-

phorylase and the inhibition of glycogen synthetase. Glucose-6-phosphatase, found


mainly in the liver and in lesser amounts in the kidney, forms free glucose, which
increases blood glucose.
(2) Since glucagon stimulates and insulin suppresses hepatic glycogenolysis, the effects
of epinephrine on insulin secretion (suppression) and glucagon secretion (stimulation)
reinforce the breakdown of glycogen and the increase in hepatic glucose secretion.
(3) Epinephrine also increases the hepatic production of glucose from lactate, amino acids,
and glycerol, all of which are gluconeogenic substances.

b. Glycogenolysis in muscle
(1) Epinephrine stimulates glycogenolysis in muscle by a {3-adrenergic receptor mechanism

involving the stimulation of adenyl cyclase and cAMP-induced stimulation of glycogen


phosphorylase. Concomitantly, glycogen synthetase activity is reduced.
(2) Muscle lacks glucose-6-phosphatase, and epinephrine-induced glycogenolysis in muscle
does not directly increase blood glucose. The glucose-6-phosphate is metabolized
to lactate or pyruvate, which is converted to glucose by the liver.
(3) The ultimate physiologic effect of epinephrine-stimulated glycogenolysis in muscle is
increased hepatic glucose secretion (hyperglycemia) via the hepatic conversion of muscle lactate to glucose.
c. Hyperglycemic effects of epinephrine on the liver are important only in conjunction with
the effects of epinephrine on glucagon and insulin secretion together with its glycogenolytic
effect on muscle in acute emergency situations. Epinephrine in physiologic concentrations
does not have a direct glycogenolytic effect in the liver.
(1) Much higher amounts of epinephrine than glucagon are required to cause hyperglycemia. However, epinephrine has a more pronounced hyperglycemic effect than glucagon
for the following important reasons.
(a) Epinephrine inhibits insulin secretion, while glucagon stimulates insulin secretion;
therefore, the hyperglycemic effect of glucagon is attenuated by insulin.
(b) Epinephrine stimulates glycogenolysis in muscle, thereby providing lactate for hepatic gluconeogenesis.
(c) Epinephrine stimulates glucagon secretion, which amplifies its hyperglycemic effect.
(d) Epinephrine stimulates ACTH secretion, which then stimulates cortisol secretion.
Cortisol also is a potent gluconeogenic hormone via the hepatic conversion of
alanine to glucose.
(e) Circulating catecholamines inhibit muscle glucose uptake, which is in contrast to
the effect of glucagon.
(2) Epinephrine indirectly inhibits insulin-mediated facilitated diffusion of glucose by muscle
and adipose tissue via its blockade of insulin secretion.
(3) Catecholamines also directly inhibit glucose uptake by the suppression of glucose transporter proteins in the cell membranes of skeletal and cardiac muscle cells and adipocytes.

2. Fat metabolism. In humans, the major site of lipogenesis from glucose is the liver.
a. A man of average size has fat stores that contain about 15 kg of triglyceride, some of which
can be mobilized as free fatty acids.
b. Epinephrine stimulates lipolysis by activating triglyceride lipase, which is called the intracellular hormone-sensitive lipase. The activation of this enzyme is via the {3-adrenergic receptor
(i.e., cAMP).

c. Mobilization of free fatty acids from stores in adipose tissue supplies a substrate for ketogenesis in the liver. Acetoacetate and J3-hydroxybutyrate are transported from the liver to the
peripheral tissues, where they are quantitatively important as energy sources.
(1) Cardiac muscle and the renal cortex use fatty acids and acetoacetate in preference to
glucose, whereas resting skeletal muscle uses fatty acids as the major source of energy.
(2) During extreme conditions, such as starvation and diabetes, the brain adapts to the use
of ketoacids. Ketoacids also are oxidized by skeletal muscle during starvation.
3. Gluconeogenesis refers to the formation of glucose from noncarbohydrate sources. Gluconeogenesis occurs in the liver and the kidney.
a. Gluconeogenic substances include pyruvate, lactate, glycerol, odd-chain fatty acids, and
amino acids. However, the major source of endogenous glucose production is protein, with
a smaller fraction available from the glycerol contained in fat. All of the constituent amino
acids in protein tissue, with the exception of leucine, can be converted to glucose.
b. The conversion of even-chain fatty acids is not possible in the mammalian liver because of
the absence of the enzymes necessary for the de novo synthesis of the four-carbon dicarboxylic acids from acetyl-CoA.
I. Endocrinopathies
1. Hyposecretion of catecholamines, such as occurs during tuberculosis and malignant destruction of the adrenal glands or following adrenalectomy, probably produces no symptoms or
other clinical features.
a. Catecholamine production from the sympathetic nerve endings appears to satisfy the normal
biologic requirements, because the adrenal medulla is not necessary for life.
b. The functional integrity of the adrenal medulla can be determined experimentally by the
administration of 2-deoxY-D-glucose. This non metabolizable carbohydrate induces intracellular glycopenia and extracellular hyperglycemia.
c. Spontaneous deficiency of epinephrine is unknown as a disease state, and adrenalectomized
patients do not require epinephrine replacement therapy.
2. Hypersecretion of catecholamines from chromaffin cell tumors (pheochromocytomas) produces demonstrable clinical features.
a. Pheochromocytoma patients have sustained or paroxysmal hypertension.
b. The hypersecretion of catecholamines is associated with severe headache, sweating (cold
or adrenergic sweating), palpitations, chest pain, extreme anxiety with a sense of impending
death, pallor of the skin caused by vasoconstriction, and blurred vision.
c. Most pheochromocytomas contain predominantly norepinephrine, and most affected patients secrete predominantly norepinephrine into the bloodstream. Evidence of epinephrine
hypersecretion increases the likelihood that the tumor origin is in the adrenal medulla.
However, an extra-adrenal site should not be excluded.
(1) If epinephrine is secreted primarily, the heart rate is increased.
(2) If norepinephrine is the predominant hormone, the pulse rate decreases reflexly in
response to marked hypertension.
d. Urinary excretion of catecholamines, metanephrines, and VMA is increased.
3. Clinical tests
a. The adrenolytic test involves the administration of an a-blocker (phentolamine) to observe
the effect of systemic blood pressure. A dramatic fall in blood pressure is pathognomonic
of pheochromocytoma because this procedure is without significant effect in normotensive
subjects.
b. The provocative test involves the administration of histamine, glucagon, or tyramine, which
will cause a rise in blood pressure.
4. Treatment requires surgical removal of the tumor.

VIII. ADRENAL CORTEX. This section describes the physiologic and biochemical aspects of glucocorticoids, the most important of which is cortisol (also known as hydrocortisone). The biologic characteristics of mineralocorticoids (specifically, aldosterone) are described in section IX of Chapter 4.

A. Embryology. Morphologically and physiologically, the fetal adrenal gland differs strikingly from
that of the adult. However, through all stages of life, the function of the adrenal cortex is dependent
on ACTH.

1. The adrenal cortex is a mesodermal derivative. It is axiomatic that all endocrine glands derived
from mesoderm synthesize and secrete steroid hormones.

2. The outer neocortex, which is the progenitor of the adult adrenal cortex, comprises about
15% of the total volume of this organ, and the inner fetal zone (inner zone or fetal cortex)
constitutes about 85%.
3. The adrenal gland is larger at birth than it is during adulthood. This is due to the fact that the
fetal zone undergoes rapid involution during the first few months of extrauterine life and
completely disappears by 3-12 months postpartum.

4. Near term, the fetal cortices of the fetal adrenal glands secrete 100-200 mg of steroids daily
in the form of sulfoconjugates, the principal one of which is the biologically weak androgen,
dehydroepiandrosterone (DHEA) sulfate. This 17-ketosteroid is an androgen containing 19
carbon atoms.

B. Morphology
1. Gross anatomy
a. The adrenal glands are paired structures situated above the kidneys.
b. Normally, each gland weighs about 5 g, of which the cortex constitutes approximately 80%.
2. Histology and function. The adrenal cortex consists of three distinct layers or zones of cells.
a. The outermost layer, the zona glomerulosa, is the site of aldosterone and corticosterone
synthesis. Aldosterone is the principal mineralocorticoid of the human adrenal cortex.

b. The wider, middle zone is the zona fasciculata, and the innermost layer is the zona
reticularis. The two inner zones of the adrenal cortex should be considered a functional
unit, where mainly cortisol (and some corticosterone) and DHEA are synthesized.
C. Adrenocortical hormones: corticosteroids (Tables 7-6, 7-7)
1. Secretion
a. The human adrenal cortex secretes two glucocorticoids (cortisol and corticosterone*), one
mineralocorticoid (aldosterone), biosynthetic precursors of three end products (progesterone, ll-deoxycorticosterone, and l1-deoxycortisoll, and androgenic substances (DHEA
and its sulfate ester).
b. The normal human adrenal cortex does not secrete physiologically effective amounts of
testosterone or estrogenic substances.

2. Transport
a. Under physiologic circumstances, about 75% of the plasma cortisol is bound to cortisolbinding globulin (CBG; transcortin), which is an a-globulin. (In addition to binding cortisol,
Table 7-6. Average 8:00

A.M.

Plasma Concentration and Secretion Rate of Corticosteroids in Adults

Corticosteroid
Cortisol
Corticosterone
ll-Deoxycortisol
Deoxycorticosterone
Aldosterone
18-Hydroxycorticosterone
Dehydroepiandrosterone (DHEA)
DHEA sulfate

Plasma Concentration
(lLg/dl)
13

Secretion Rate
(mg/day)
15

0.16
0.07
0.009
0.009
0.5
115

0.40
0.20
0.15
0.10
15
15

Reprinted from Genuth SM: The adrenal glands. In Physiology. Edited by Berne RM, Levy MN. SI. Louis, CV Mosby,
1983, p 1046.

*At physiologic concentrations, corticosterone has glucocorticoid activity.

Table 7-7. Blood Production Rates of Adrenal Androgens


Androgenic Steroid

Plasma
Concentration (lLg/dl)

Blood Production
Rate (mg/day)

0.8
0.034

7.0
0.34

0.06
0.14

1.4
3.4

Testosterone
Men
Women"
Androstenedione
Men
Women

Reprinted from Mulrow PJ: The adrenals. In Physiology and Biophysics. Edited by Ruch TC, Patton HD. Philadelphia,
WB Saunders, 1973, p 229.
'In women, about 50% of the blood testosterone is derived from androstenedione.

transcortin has a high binding affinity for progesterone, deoxycorticosterone, corticosterone,


and some synthetic analogs.> About 15% of the plasma cortisol is bound to plasma albumin,
and about 10% is unbound and represents the physiologically active steroid.
b. The 90% of the plasma cortisol bound to plasma protein represents the metabolically
inactive pool, which serves as a reservoir for free hormone.
3. Corticosteroidogenesis (Figure 7-7)
a. Uptake of cholesterol. Free cholesterol is the preferred precursor of the corticosteroids,
although the adrenal cortex can form cholesterol from acetyl-CoA. Most stored adrenal
cholesterol is esterified with fatty acids; it is the cholesterol ester content that is reduced
following ACTH stimulation of the fasiculata-reticularis complex.
b. Side-chain cleavage of cholesterol. The rate-limiting step in corticosteroidogenesis is the
mitochondrial conversion of cholesterol to pregnenolone by 20,22-desmolase.
c. Pregnenolone: the common precursor of all steroid hormones. Conversion of pregnenolone to progesterone requires two enzymes-3,8-hydroxysteroid dehydrogenase and
,:ls-isomerase---which are found in the endoplasmic reticulum (microsomes).
d. Hydroxylation reactions follow sequentially after the formation of pregnenolone and progesterone and require NADPH 2 and molecular 02'
(1) The hydroxylation reactions in the biosynthesis of aldosterone occur sequentially at the
C-21, C- 11, and C- 18 positions.
1 - - - - - 1 .. .. ..
1

Ovary

I:

Testis :

1 _ _ _ _ _ 1 ............ .

Cholesterol

I
.......... J........... ................ .

.... :~~l~~: .... .. ~~l~~: ....

1!;,5 Pathway: 1!;,5 - Pregnenolone ---...17a- HydrOxypregnenolone--+1 Dehydroepiandrosterone

L
1!;,4Pathway:

Progesterone _

11-Deoxycorticosterone

1
1
1I

Corticosterone

17a- Hydroxyprogesterone---+ : Androstenedione :......: Testosterone

11-Deoxycortisol

E~I!:.0~~ J

1_

..--.....

: ~~r~dloll

ICortisol

18-Hydroxycorticosterone

I Aldosterone

Figure 7-7. Summary of steroidogenesis in the adrenal cortex, testis, and ovary. Notice that there is no interconversion
between mineralocorticoids and glucocorticoids.

e.

f.

g.

h.

i.

(2) The hydroxylation reactions in the biosynthesis of cortisol occur sequentially at the
C-17, C-21, and C-11 positions.
Sex steroid pathways. Both pregnenolone and progesterone are substrates for the microsomal enzyme, 17a-hydroxylase, which is found not only in the adrenal cortex but also in
the testis and ovary.
(1) Pregnenolone is converted to 17a-hydroxypregnenolone, which can either continue
along the AS-pathway to the synthesis of androgens or enter the glucocorticoid pathway.
(2) Progesterone is converted to 17a-hydroxyprogesterone, which can either proceed along
the A 4-pathway to the synthesis of androgens or enter the mineralocorticoid pathway.
(3) The zona glomerulosa lacks 17a-hydroxylase and does not have the capacity to synthesize 17a-hydroxyprogesterone. For this reason, the zona glomerulosa cannot synthesize
cortisol.
(a) The conversion of pregnenolone to DHEA is in the AS-pathway.
(b) The conversion of progesterone to androstenedione is in the A4-pathway.
Glucocorticoid/mineralocorticoid pathways. Both progesterone and 17a-hydroxyprogesterone are substrates for the microsomal enzyme, 21-hydroxylase.
(1) Progesterone is converted to 11-deoxycorticosterone, which is in the mineralocorticoid
pathway.
(2) 17a-Hydroxyprogesterone is converted to 11-deoxycortisol, which is in the glucocorticoid pathway.
(3) 11-Deoxycorticosterone and 11-deoxycortisol are not precursors for the synthesis of
each other.
Diverging pathways. Both 11 -deoxycorticosterone and 11-deoxycortisol are acted upon
by mitochondrial 1113-hydroxylase.
(1) 11-Deoxycorticosterone forms corticosterone in the mineralocorticoid pathway.
(2) 11-Deoxycortisol is converted to cortisol, the principal glucocorticoid of the human
adrenal cortex.
(3) Corticosterone differs from cortisol only in that the former lacks a 17a-hydroxyl group.
Although this structural difference is small, it accounts for a very large difference in the
biologic activity of these two hormones.
Aldosterone synthesis. Some of the corticosterone serves as the- substrate for another
mitochondrial enzyme, 18-hydroxylase.
(1) This reaction leads to the formation of 18-hydroxycorticosterone.
(2) 18-Hydroxycorticosterone is converted to aldosterone by the mitochondrial enzyme,
18-hydroxysteroid dehydrogenase, which is found only in the zona,glomerulosa.
Synthesis and secretion of sex steroids
(1) Androgens. The adrenal cortex secretes four androgenic hormones: androstenedione,
testosterone, DHEA, and DHEA sulfate. Quantitatively, the most important sex steroids
produced by the human adrenal cortex are DHEA and DHEA sulfate. Except for testosterone, adrenocortical androgens are relatively weak and serve as precursors for hepatic
conversion to testosterone.
(a) DH EA is derived from 17a-hydroxypregnenolone by the action of 17,20-desmolase.
(b) DHEA is mainly conjugated with sulfuric acid and, as such, is bound to plasma
protein. While in this bound form, DHEA is not readily excreted but circulates in
higher concentrations than any other steroid.
(c) DHEA is the principal precursor of urinary 17-ketosteroids; however, the most
abundant urinary 17-ketosteroids are androsterone and etiocholanolone.
(d) DHEA and DHEA sulfate have androgenic activity by virtue of their peripheral
conversion to testosterone. DHEA sulfate is active as a minor precursor of other
19-carbon steroids formed in the gonads and placenta, which are sites of sulfatase
activity.
(i) Normal excretion rates for 17-ketosteroids are 5-14 mg/ day in women and 820 mg/day in men.
(ii) Normally, adrenocortical precursors represent the bulk of the urinary 17-ketosteroid pool, with a smaller contribution from the gonads.
(iii) The main androgen secreted by premenopausal women is androstenedione,
about 60% of which is of adrenocortical origin.

(2) Estrogens. The human adrenal cortex can synthesize minute amounts of estrogen.
The adrenal cortex makes its major contribution to the body's estrogen (estrone) pool
indirectly by supplying androstenedione together with DH EA and its sulfate as substrates

for conversion to estrogens by subcutaneous fat, hair follicles, mammary adipose tissue,
and other tissues.

4. Metabolism of corticosteroids
a. General considerations
(1) Corticosteroid inactivation occurs by:

(a) Enzymatic reduction of the 114 -5 double bond in the A-ring to form dihydrocortisol
(b) Enzymatic reduction of the ketonic oxygen substituent at the C-3 position to form

tetrahydrocortisol
(c) Conjugation with glucuronic acid to form a water-soluble metabolite that is readily
excreted by the kidney
(2) The major urinary metabolite of cortisol is tetrahydrocortisol glucuronide.
b. Hepatic conversion. The liver is the major extra-adrenal site of corticosteroid metabolism.
(1) Cortisol can be enzymatically converted to cortisone by 11,B-hydroxysteroid dehydrogenase and excreted as tetrahydrocortisone glucuronide.
(2) The ketonic oxygen substituent on the C-20 position of cortisol and other steroids can
be enzymatically converted to a hydroxyl group. These steroids can be subjected to
A-ring reduction, conjugated, and excreted as glucuronides.
(a) Cortisol is converted to cortol glucuronide.
(b) Cortisone is converted to cortolone glucuronide.
(c) Progesterone forms pregnanediol glucuronide.
(d) 17a-Hydroxyprogesterone forms pregnanetriol glucuronide.
(3) Steroids that contain a 17a-hydroxyl group are ketogenic. Cortisol can be enzymatically
converted to a 17-ketosteroid by 17,20-desmolase. About 5% of cortisol appears in
the urine as a 17-ketosteroid.
(4) Aldosterone also can undergo A-ring reduction and conjugation to form tetrahydroal-

dosterone glucuronide.
c. Conversion in other extra-adrenal tissues. Muscle, skin, fibroblasts, intestine, and lymphocytes also can carry out oxidation-reduction reactions at the C-3, C-11, C-17, and C-20
positions of the corticosteroid molecule.
D. Control of adrenocortical function (see Figure 7-4). Physiologic control of the rate of cortisol
secretion occurs via a double negative feedback loop, a mechanism that is characteristic of most
neuroendocrine control systems.

1. Regulation of ACTH secretion


a. The parvicellular peptidergic neurons of the hypothalamus release CRH, which stimulates
the secretion of ACTH from the anterior lobe of the pituitary gland via the hypophysial
portal system. CRH is a polypeptide consisting of 41 amino acid residues.
b. CRH secretion from the tuberoinfundibular neurons is regulated by neurotransmitters secreted by the monoaminergic neurons that innervate the peptidergic neurons.
(1) Hypothalamic secretion of CRH is stimulated by cholinergic neurons. Serotonin also is
a stimulatory signal to CRH neurons.
(2) Adrenergic neuron activity inhibits release of CRH. Gamma-aminobutyric acid
(GABA) also is a known inhibitor of CRH secretion.

2. Negative feedback action of corticosteroids


a. Of the endogenous corticosteroids, only cortisol has ACTH-suppressing activity. The synthetic glucocorticoid, dexamethasone, is a potent inhibitor of ACTH secretion and, therefore, of endogenous glucocorticoid secretion.
b. The negative feedback of cortisol is exerted at the level of the pituitary gland and the ventral
diencephalon.
(1) If free cortisol levels are supraphysiologic, ACTH secretion is suppressed and the adrenal
cortex ceases its secretory activity and undergoes disuse atrophy.
(2) Conversely, if plasma free cortisol levels are subnormal, the anterior lobe is released
from inhibition by cortisol, ACTH secretion rises, and the adrenal cortex secretes more
cortisol and becomes hypertrophic.
3. Hypophysial-adrenocortical rhythm. Normally, blood ACTH levels are higher in the morning
than in the evening. This accounts for the diurnal rhythms in cortisol secretion, plasma cortisol
concentration, and 17 -hydroxycorticosteroid excretion (see III A 1).

4. Hypophysial-adrenocortical response to stress. The normal hypothalamic-hypophysialadrenocortical control system can be overridden by a variety of challenges, which collectively
are referred to as stress.
a. Among the stresses shown to induce increased activity in this system are severe trauma,
pyrogens, hypoglycemia, histamine injection, electroconvulsive shock, acute anxiety, burns,
hemorrhage, exercise, infections, chemical intoxication, pain, surgery, psychological stress,
and cold exposure.
b. In stress conditions, ACTH secretion is stimulated despite the fact that systemic levels of
cortisol are much higher than those required to inhibit ACTH secretion completely in
unstressed conditions.

E. Physiologic effects of glucocorticoids. Of the naturally occurring steroids, only cortisol, cortisone, corticosterone, and ll-dehydrocorticosterone have appreciable glucocorticoid activity. Full
recovery from hypothalamic-hypophysial-adrenocortical suppression may require as long as 1
year following cessation of all steroid therapy.

1. Anti-inflammatory effects. Glucocorticoids inhibit inflammatory and allergic reactions in


several ways.
a. They stabilize lysosomal membranes, thereby inhibiting the release of proteolytic enzymes.
b. They decrease capillary permeability, thereby inhibiting diapedesis of leukocytes.
c. Glucocorticoids reduce the number of circulating lymphocytes, monocytes, eosinophils,
and basophils.
(1) The decreased number of these formed elements in blood is due primarily to a redistribution of the cells from the vascular compartment into the lymphoid tissue (e.g., spleen,
lymph nodes, bone marrow). Cellular lysis is not a major mechanism for decreasing the
number of these cells in the human circulation.
(2) The decrease in circulating basophils accounts for the fall in blood histamine levels and
the abrogation of the allergic response.
(a) The migration of inflammatory cells from capillaries is decreased.
(b) Glucocorticoids lessen the formation of edema and, thereby, reduce the swelling
of inflammatory tissue.
(3) Glucocorticoids cause an increase in the number of circulating neutrophils due to the
accelerated release from bone marrow and a reduced migration from the circulation.
Steroids also inhibit the ability of neutrophils to marginate to the vessel wall.
d. Glucocorticoids cause involution of the lymph nodes, thymus, and spleen, which leads to
decreased antibody production.
(1) This Iymphocytopenic effect aids in the prevention or reduction of the immune response
by an organ transplant recipient.
(2) Since recipients pretreated with large doses of glucocorticoids are susceptible to intercurrent infections, antibiotics are a necessary adjunct to the steroid therapy.
(3) Antibody production is not suppressed in humans at conventional steroid doses; however, chronic administration of high doses of glucocorticoids leads to an impairment of
host defense mechanisms.
e. Glucocorticoids lead to an increase in the total blood count because of the increased
numbers of neutrophils, erythrocytes, and platelets. The increase in circulating erythrocytes
(polycythemia) is due to the stimulation of hematopoiesis.

2. Renal effects
a. Glucocorticoids restore glomerular filtration rate and renal plasma flow to normal following
adrenalectomy. Mineralocorticoids do not have these effects.
b. Glucocorticoids facilitate free-water excretion (clearance) and uric acid excretion.

3. Gastric effects. Cortisol increases gastric flow and gastric acid secretion, while it decreases
gastric mucosal cell proliferation. The latter two effects lead to peptic ulceration following
chronic cortisol treatment.

4. Psychoneural effects of glucocorticoids have been noted following chronic high-dose therapy
with these steroids. Patients may become initially euphoric and then psychotic, paranoid, and
depressed.

5. Antigrowth effects
a. Large doses of cortisol have been shown to antagonize the effect of active vitamin D
metabolites on the absorption of Ca2+ from the gut, to inhibit mitosis of fibroblasts, and to

cause degradation of collagen. All of these effects lead to osteoporosis, which is a reduction
in bone mass per unit volume with a normal ratio of mineral-to-organic matrix.
b. Glucocorticoids delay wound healing because of the reduction of fibroblast proliferation.
Connective tissue is reduced in quantity and strength.
c. Chronic supraphysiologic doses of glucocorticoids suppress GH secretion and inhibit somatic
growth.
d. Although glucocorticoids increase the ability of muscle to perform work, large doses lead
to muscle atrophy and muscular weakness.
6. Vascular effect. Cortisol in pharmacologic doses enhances the vasopressor effect of norepinephrine. In the absence of cortisol, the vasopressor action of catecholamines is diminished,
and hypotension ensues. Thus, corticosteroids have a role in the maintenance of normal
arterial systemic blood pressure and volume through their support of vascular responsiveness to
vasoactive substances. (Cortisol enhances catecholamine synthesis via its activation of the
epinephrine-forming enzyme, PNMT.l
7. Stress adaptation. Glucocorticoids allow mammals to adapt to various stresses in order to
maintain homeostasis.
a. Resistance to stress is not increased by the administration of glucocorticoids.
b. Stress is associated with the activation of the hypothalamic-hypophysial-adrenal axis.

F. Metabolic effects of glucocorticoids


1. Carbohydrate metabolism. Cortisol is a carbohydrate-sparing hormone and, therefore, exerts
an anti-insulin effect, which leads to hyperglycemia and insulin-resistance.
a. Glucocorticoids maintain blood glucose and the glycogen content of the liver, kidney, and
muscle by promoting the conversion of amino acids to carbohydrates and the storage of
carbohydrate as hepatic glycogen.
b. Cortisol is hyperglycemic principally because of its gluconeogenic activity, which is related
to its protein catabolic effect on extrahepatic tissues, especially muscle.
(1) The proteolytic effect of glucocorticoids results in the mobilization of amino acids from
muscle and in an increase in plasma amino acid concentration.
(2) Alanine is quantitatively the major gluconeogenic amino acid precursor in the liver. Like
acetyl-CoA, alanine inhibits pyruvate kinase activity.
c. Cortisol also exerts an anti-insulin effect by blocking glucose transport in muscle and adipose
tissue. This effect accounts for the phenomenon of glucose intolerance or an eventual
"steroid diabetes."
d. Glucocorticoids augment the activity of key gluconeogenic enzymes by the induction of
hepatic enzyme synthesis.
(1) The gluconeogenic pathway has three steps that differ from those in the glycolytic
pathways as a result of their thermodynamic irreversibility. These enzymes and their
substrates are as follows.
(a) Glucose-6-phosphate -+ glucose (glucose-6-phosphatasel
(b) Fructose 1,6-diphosphate -+ fructose-6-phosphate (fructose 1,6-diphosphatasel
(c) Conversion of pyruvate to phosphoenolpyruvate requires two steps:
(i) Pyruvate -+ oxaloacetate (pyruvate carboxylase)
(ii) Oxaloacetate -+ phosphoenolpyruvate (phosphoenolpyruvate carboxykinasel
(2) Glucocorticoids are associated with activation of glycogen synthetase by glucose-6phosphate and of pyruvate carboxylase by acetyl-CoA. They also are associated with
indirect inhibition of glycolysis by free fatty acids, resulting in increased glucogenesis
and glycogenesis.
e. Cortisol also indirectly inhibits the activities of glycolytic enzymes, which accounts for its
anti-insulin effect. Enzymes that are blocked by the effect of glucocorticoids include:
(1) Glucokinase
(2) Phosphofructokinase
(3) Pyruvate kinase
f. Glucocorticoids mobilize fatty acids from adipose tissue to the liver, where metabolism of
free fatty acids may lead to products that inhibit glycolytic enzymes and favor gluconeogenesis.
(1) The glycerol released from the fat cell with the fatty acids also serves as a secondary
substrate for gluconeogenesis.

(2) The cortisol-inhibited glycolysis in peripheral tissue probably is indirectly blocked via
the inhibition of the key glycolytic enzyme, phosphofructokinase, by the elevated concentration of plasma free fatty acids.
g. Glucocorticoids are associated with compensatory hyperinsulinemia following hyperglycemia.
2. Protein metabolism. The most important gluconeogenic substrates are amino acids derived
from proteolysis in skeletal muscle.
a. Cortisol enhances the release of amino acids from proteins in skeletal muscle and other
extrahepatic tissues, including the protein matrix of bone.
(1) The amino acids released, especially the glucogenic amino acid alanine, are transported
to the liver and converted to glucose.
(2) Increased glucose production by cortisol via gluconeogenesis is associated with increased urea production via the conversion of amino nitrogen to urea. This effect
accounts for the increased urinary nitrogen excretion.
(3) The proteolysis in skeletal muscle brings about a negative nitrogen balance.
b. The amino acids taken up by the liver are used not only to form glucose or glycogen but
also to build new protein. This protein anabolic effect at the level of the liver is an important
exception to the overall protein catabolic effect of cortisol.
c. Equally important is the ability of glucocorticoids to inhibit the de novo synthesis of protein,
probably at the translational level. This is called an antianabolic effect of cortisol.
3. Fat metabolism. Glucocorticoids are lipolytic hormones. Their lipolytic effect is in part due to
the potentiation of the lipolytic actions of other hormones, such as GH, catecholamines,
glucagon, and thyroid hormone.
a. Glucocorticoids favor the mobilization of fatty acids from adipose tissue to the liver, where
the metabolism of fatty acids inhibits glycolytic enzymes and promotes gluconeogenesis. As
a result of increased fatty acid oxidation, glucocorticoids may lead to ketosis, especially in
the context of diabetes mellitus.
(1) The major site of stimulation is the gluconeogenic enzyme, fructose 1,6-diphosphatase,
which is activated by fatty acids.
(2) At the same time, pyruvate kinase and phosphofructokinase are inhibited by fatty acids.
Thus, glycolysis is inhibited while gluconeogenesis proceeds.
b. Glucocorticoids also indirectly stimulate lipolysis by blocking peripheral glucose uptake and
utilization. They inhibit re-esterification of fatty acids within adipocytes by inhibiting the use
of glucose.
c. Fatty acid synthesis is inhibited in the liver by cortisol, an effect not observed in adipose
tissue. The overall effect of glucocorticoids on fat is to induce a redistribution of fat together
with an increase in total body fat (i.e., truncal obesity). The increase in body weight is not
due to a growth effect that is the accretion of protein. (Recall that cortisol is an antigrowth
hormone.)
(1) There is a characteristic centripetal distribution of fat (i.e., an accumulation of fat in the
central axis of the body).
(a) The deposition of fat in the facies is called "moonface."
(b) The deposition of fat in the suprascapular region is referred to as "buffalo hump."
(c) Excessive fat distribution leads to a pendulous abdomen.
(2) Glucocorticoid-induced obesity reflects increased food intake rather than a change in
the rate of lipid metabolism.
d. Chronic excessive amounts of cortisol lead to hyperlipidemia and hypercholesterolemia.
Glucocorticoids increase appetite and, thereby, playa role in obesity.

IX. TESTIS
A. Embryology
1. Internal genitalia (Figure 7-8). The internal reproductive tract in males and females is derived
from one of two pairs of genital ducts. The wolffian (mesonephric) ducts give rise to the male
internal genitalia, and the mullerian ducts give rise to the female internal genitalia.
a. In the male, the internal genitalia (also called accessory sex organs) consist of the following
ducts and glands for the conveyance of sperm:
(1) Seminiferous tubules

iNDiFFERENT STAGE

+---Gonad
Mesonephros
.~f---

Mullerian duct

.rJ-:---- Woiffian duct


~,;---Urogenital

sinus

~:----

Epididymis

'" ooIlIIdt-- Testis


Fallopian
tube

Vas deferens

Uterus-----'~-

.- .-

"''''-"""'T--Seminal vesicle
Vagina----~o{

l' "

CWII'2

::~
f----Prostate gland

'1i;r>

FEMALE

MALE

Figure 7-8. Sex differentiation of the gonad and the internal genitalia. Up to 6 weeks gestation, the gonad is a
bipotential structure (indifferent stage), and the urogenital tract in both sexes consists of two pairs of genital ducts
(i.e., the wolffian and mLilierian ducts) and a mesonephros, all of which terminate in the urogenital sinus. in the
female, the gonad develops into an ovary and the mLilierian ducts become organized into the fallopian tubes, uterus,
and upper vagina, while the wolffian ducts remain vestigial. in the male, a testis develops and the wolffian ducts
differentiate into the epididymis, vas deferens, and seminal vesicle, while the mLilierian ducts regress. (Reprinted from
Wilson JD: Embryology of the genital tract. in Campbell's Urology, 4th edition. Edited by Harrison JH, et al. Philadelphia, WB Saunders, 1977, p 1473.)

(2) Rete testis*


(3) Ductuli efferentes
(4) Epididymis
(5) Vas deferens
(6) Ejaculatory duct
(7) Seminal vesicles
(8) Prostate gland
(9) Bulbourethral glands (Cowper's glands)
b. The wolffjan ducts are the excretory ducts of the mesonephric kidney and are attached to
the primordial gonad; the mullerian duct is formed parallel to the wolffian duct and is not
contiguous with the primitive gonad.
c. At 9-10 weeks gestation, following the appearance of the Leydig (interstitial) cells in the
fetal testis, the two wolffjan ducts begin to differentiate and give rise to the following male
internal genitalia:
(1) Epididymis
(2) Vas deferens
(3) Seminal vesicles
(4) Ejaculatory duct

'The duct system distal to the rete testis is known as the excurrent (excretory) duct system.

d. It is the primary function of androgen to induce the formation of the male accessory sex
organs during fetal life. Specifically, testicular androgen is required for the differentiation of
the male genital (wolffian) duct system.
e. Genetic sex determines gonadal sex, and gonadal sex determines phenotypic sex. The
control over the formation of the male phenotype requires the action of the following three
hormones.
(1) Mullerian duct inhibiting factor is produced by the Sertoli cells of the fetal testis and
induces the regression of the mullerian duct system. Mullerian duct inhibiting factor
functions as a paracrine secretion that diffuses to the paired mullerian ducts.
(2) Testosterone is secreted by the fetal testis and promotes growth and differentiation of
the wolffian duct system and the accessory sex organs.
(3) Testosterone also is the precursor for dihydrotestosterone, which is necessary for the
growth and differentiation of the male external genitalia (see IX A 2 b).
2. External genitalia. The external genitalia also begin to differentiate between 9 and 10 weeks
gestation.
a. In contrast to the internal genitalia, the external genitalia and urethra in both sexes develop
from common anlagen, which are the urogenital sinus, the genital sinus, the genital tubercle,
the genital swelling, and the genital (urethral) folds. The anlagen of the male external genitalia
give rise to the following structures.
(1) The urogenital sinus forms the male urethra and prostate gland.
(2) The genital tubercle grows into the glans penis.
(3) The genital swelling forms the scrotum.
(4) The urethral folds enlarge to form the outer two-thirds of the penile urethra and corpora
spongiosa.
b. The growth and development of the male external genitalia require dihydrotestosterone,
which is formed from the conversion of fetal testicular testosterone within the urogenital
sinus and lower urogenital tract.
3. Testis (see Figure 7-8). The primitive gonads develop midabdominally in association with the
mesonephric ridges.
a. Through 5 weeks gestation, the gonads are undifferentiated and consist of a medulla, a
cortex, and primordial germ cells. These germ cells are embedded in a layer of cortical
epithelium surrounding a core of medullary mesenchymal tissue.
b. At 6 weeks, the seminiferous tubules begin to form from the medulla. The cortical region of
the primitive gonad undergoes regression. (The female gonad develops from the cortical
region.) The primordial germ cells (gonocytes) arise outside the gonads and migrate from
the endodermal yolk sac epithelium to the urogenital ridge. During migration, the gonocytes
undergo continuous mitosis.
c. At 7 weeks gestation, the Sertoli (sustentacular) cells begin to form and secrete the H-Y
(histocompatibility-Y) antigen, which is under control of the Y chromosome. The Sertoli
cells are derivatives of the mesenchymal tissue of the urogenital ridge.
d. At 8-9 weeks of fetal life, the Leydig cells are formed and secrete testosterone in response
to chorionic gonadotropin, which is secreted by the placenta. The Leydig cells also are
mesenchymal derivatives, which appear in the connective tissue surrounding the seminiferous cords.
e. At 9 weeks gestation, definitive testes are present.
f. At 7-9 months gestation, the testes normally descend through the inguinal rings into the
scrotum.
B. Morphology
1. Gross anatomy
a. The adult testis is an ovoid gland, which is approximately 4-5 cm long and 2.5-3.0 cm
wide and ranges in weight from 10 to 45 g.
b. The testes normally are situated in the scrotum, where they are maintained at a temperature
that is about 2 C lower than normal body temperature. The lower temperature is necessary
for normal spermatogenesis.
c. Each gland is surrounded by a fibrous connective tissue membrane called the tunica albuginea, which, in turn, is surrounded by a serous membrane called the tunica vaginalis.
d. By puberty, the testes usually have developed sufficiently to perform the functions of
spermatogenesis and steroidogenesis. Generally, puberty begins between the ages of 12 and

14 years. In the United States, 95% of normal boys show signs of puberty by the age of 16
years.
2. Functional histology of testicular parenchyma. The testicular parenchyma consists mainly
of the seminiferous tubules and Leydig cells, which comprise about 90% and 10% of the
testicular volume, respectively. These two compartments are separated by boundary tissue,
which includes a basement membrane.
a. Tubular tissue. The seminiferous tubules have a total collective length of approximately
800 m (a half mile)!
(1) The spermatogenic tubules are organized into coiled loops, each of which begins and
terminates in a single duct called the tubulus rectus.
(2) The tubuli recti join to form the rete testis and eventually drain via the ductuli efferentes into the epididymis. The epididymis is the primary storage and final maturation
site for spermatozoa.
(3) From the epididymis, the spermatozoa are transmitted via the vas deferens and ejaculatory duct into the penile urethra.
b. Tubular cells. The basic cellular components of the tubules are the germinal cells and the
nongerminal (Sertoli) cells. The spermatogonia and the Sertoli cells are the only tubular
cells that lie on the tubular membrane called the basal lamina. The other germinal cells
(spermatocytes) are found between the Sertoli cells.
(1) Spermatogonia are nonmotile stem cells that divide by mitosis to form two cellular
pools: a pool of additional stem cells, which undergo continual renewal by mitosis,
and a pool of type A spermatogonia, which enter the maturation process called
spermatogenesis. Each spermatogonial cell (type A) gives rise to 64 sperm cells.
(a) Spermatogenesis requires about 64 days in man, and the transport of sperm cells
through the epididymis to the ejaculatory duct requires an additional 12-21 days.
There are three phases of spermatogenesis.
(i) The type A spermatogonia divide by mitosis to form primary spermatocytes,
which are diploid cells.
(ii) The primary spermatocytes continue the gametogenic process, which leads to
the formation of secondary spermatocytes and then, through two meiotic
divisions, to the formation of spermatids haploid, which also are haploid cells.
(iii) The third phase of spermatogenesis produces mature spermatozoa via the metamorphosis of spermatids. This process, called spermiogenesis, is characterized
by the absence of cell division.
(b) Because testosterone and FSH act directly on the seminiferous epithelium, these
hormones are required for spermatogenesis.
(2) Sertoli cells are nonmotile and, in the mature testis, nonproliferating tubular cells that
lie on the basal lamina.
(a) Structural features. The Sertoli cells extend through the entire thickness of the
germinal epithelium from the basement membrane to the lumen. The tight junctions
between the bases of the Sertoli cells serve two functions.
(i) They divide the seminiferous tubular epithelium into two functional pools: a
basal compartment containing the spermatogonia and an adluminal compartment
containing the spermatogonia and spermatids.
(ii) They form an effective permeability barrier within the seminiferous epithelium,
which is defined in man as the blood-testis barrier that limits the transport of
many substances from the blood to the seminiferous tubular lumen. This barrier
maintains germ cells in an immunologically privileged location, since mature
sperm cells are very immunogenic when introduced into the systemic circulation.

(b) Functions
(i) Sertoli cells provide mechanical support for the maturing gametes.

(ii) They also have a role in spermatogenesis, which may be attributable to their
high glycogen concentration being a potential energy source.

(iii) In the fetal testis, the Sertoli cells secrete H-Y antigen, which is a product of the
testis-organizing genes and is the cell surface glycoprotein responsible for the
induction of testicular organogenesis. H-Y antigen has been found in all cell
membranes from normal XY males except the cell membranes of immature germ
cells.
(iv) The Sertoli cells secrete mullerian duct inhibiting factor, the glycoprotein that
causes regression of the mullerian duct system.

(v) With the onset of fetal testicular differentiation induced by H-Y antigen and the
incorporation of primitive germ cells into the seminiferous tubules, the Sertoli
cells secrete a meiosis-inhibiting factor, which suppresses germ cell proliferation
and differentiation beyond the spermatogonial stage.
(vi) Sertoli cells participate in the release of spermatids by enveloping the residual
lobules of spermatid cytoplasm. Thus, the residual bodies are not cast off into
the lumen but are retained within the epithelium throughout the spermiation
process.
(vii) Sertoli cells secrete a watery, solute-rich (K+ and HC03 -) fluid into the seminiferous lumen. These cells actively pump ions into the intercellular spaces to create
a standing osmotic gradient that moves water from the Sertoli cell base to the
free surface of the lumen. This fluid movement provides a driving force for
conveying sperm from the testis to the epididymis, where most of this isosmotic
fluid is reabsorbed.
(viii) Sertoli cells synthesize estradiol from androgenic precursors.
(ix) Sertoli cells also produce and secrete an androgen-binding protein and inhibin
(see IX D 1 a).
c. Leydig cells are located between the seminiferous tubules and produce androgenic steroids.
These cells also are called interstitial cells.
(1) Leydig cells are extensive at birth but virtually disappear within the first 6 months of
postnatal life. Their reappearance marks the onset of puberty.
(2) At puberty, the fibroblast-like cells of the testis serve as stem cells that differentiate into
Leydig cells.

C. Hormones of the testis: steroids


1. Secretion and transport
a. Testosterone is the major hormone produced by the Leydig cells of the testis. Like all
naturally occurring androgens, testosterone consists of 19 carbon atoms.
(1) Testosterone is not stored in the testis. Cholesterol esters, the major precursor for
testosterone biosynthesis, are stored in the lipid droplets in the Leydig cells.
(2) A normal man secretes 4-9 mg of testosterone daily. More than 97% of secreted
testosterone is bound to plasma proteins; 68% is bound to albumin, and 30% is bound
to testosterone-binding globulin (also called sex hormone-binding globulin, because it
binds estradiol as weill. A very small percentage of the plasma testosterone is unbound.
b. Androstenedione also is secreted by the testis at a rate of about 2.5 mg/day and is an
important steroid precursor for blood estrogens in men.
(1) Many nonendocrine tissues (e.g., brain, skin, fat, liver) have the cytochrome P-4S0dependent aromatase, which converts qndrogens to estrogens.
(2) Major portions of blood estradiol and estrone in normal men are derived from blood
testosterone and androstenedione, respectively.
(3) In addition, the Sertoli and Leydig cells of the testis secrete small amounts of estradiol.
c. Dihydrotestosterone is synthesized by the testis, probably due to the action of Sa-reductase from the Sertoli cells on testosterone secreted by the Leydig cells.
(1) Only 20% of plasma dihydrotestosterone is synthesized in the testis. The remainder is
derived from the peripheral conversion of testosterone, which serves as a prohormone
in the skin and male reproductive tract (prostate gland and seminal vesicles).
(2) Dihydrotestosterone has more than twice the biologic activity of testosterone.
2. Testicular steroidogenesis (see Figure 7-7)
a. The key step in steroidogenesis is the conversion of cholesterol to pregnenolone. This
reaction is catalyzed by 20,22-desmolase, the rate-limiting enzyme for steroid biosynthesis
in all steroid-producing tissues.
(1) LH* activates 20,22-desmolase and, therefore, is the pituitary gonadotropic hormone
that regulates testosterone synthesis by the Leydig cells.
(2) In the developing male fetus, the stimulus for testosterone synthesis is chorionic gonadotropin, which is the placental hormone secreted in the highest amounts during the first
trimester.
b. The Leydig cells contain 17a-hydroxylase, which hydroxylates pregnenolone at position 17.

'In the male, LH is known as interstitial cell-stimulating hormone (ICSH).

(1) Some pregnenolone may be converted via 3-hydroxysteroid dehydrogenase/ ,:ls-isomerase to progesterone, which can be hydroxylated at the C- 17 position prior to its conversion to androstenedione along the ,:l4-pathway.
(2) In the human testis, the ,:ls-pathway is the preferential pathway for the synthesis of
testosterone, which is a ,:l4-steroid.
c. Androstenedione is the common final precursor in the synthesis of testosterone.

3. Metabolism
a. Dihydrotestosterone formation. Testosterone can serve as a pro hormone and be metabolized by Sa-reductase to the more active androgen, dihydrotestosterone. The activity of
Sa-reductase is high in the skin, prostate gland, seminal vesicles, epididymis, and liver. This
enzyme also is present in testicular tissue.
(1) Androgen target tissues are thought to be the principal sites of dihydrotestosterone
formation. The nuclear membrane and microsomes of androgen-sensitive tissues are
the physiologically important sites for conversion of testosterone to dihydrotestosterone.
Therefore, dihydrotestosterone not only is secreted by the testis into the circulation but
also is synthesized from testosterone that has entered the cells of androgen-dependent
tissues.
(2) The formation of dihydrotestosterone is an irreversible reduction reaction; therefore,
dihydrotestosterone cannot serve as a proestrogen.
(3) The anlagen of the prostate gland and external genitalia can form dihydrotestosterone
prior to the onset of virilization. Dihydrotestosterone is formed from testosterone prior
to the secretion of significant amounts of testosterone from the testis. The wolffian duct
derivatives can form dihydrotestosterone after the onset of androgen secretion and after
differentiation of the male genital system is far advanced.
(4) Dihydrotestosterone can be metabolized further to 17-ketosteroids and polar derivatives
found in the urine.
b. Estradiol and estrone formation. Testosterone and androstenedione can be converted to
estradiol and estrone, respectively, by the action of aromatase. Thus, the estrogens in the
male are derived from direct secretion by the testes and from peripheral conversion of
circulating androstenedione and testosterone.
(1) Aromatization of circulating androgens is the major pathway for estrogen formation in
the male.
(2) Aromatases are membrane-bound enzymes found in the brain, skin, liver, mammary
tissues, and most significantly, the adipose tissue.
c. 17-Ketosteroid formation. Testosterone can be metabolized to less active metabolites that
are conjugated in the liver and excreted into the urine as 17-ketosteroids.
(1) Androsterone and etiocholanolone are the major urinary metabolites of testosterone.
Testosterone glucuronide and 5a-androstanediol glucuronides are among the other
androgenic metabolites measured in urine.
(a) Testosterone glucuronide originates mainly in the liver from testosterone, androstenedione, and dihydrotestosterone. The measurement of plasma testosterone is
the mainstay for assessing Leydig cell function.
(b) Sa-Androstanediol glucuronides arise from the testosterone metabolites in both the
liver and the skin. Because an increased Sa-reductase activity in the skin and other
extrahepatic tissues is produced by increased androgen secretion, the measurement
of urinary Sa-androstanediol glucuronides has been recommended as an index of
clinical androgenicity.
(2) The excretory rate for urinary 17-ketosteroids in normal men is 1S-20 mg/day.
(a) Of this amount, 20%-40% are of testicular origin. The remainder are adrenocortical
secretions, the major one of which is DHEA.
(b) Because the urinary 17-ketosteroid pool reflects mainly adrenocortical activity, a
measurement of the 17-ketosteroid excretion is not a good index of testicular
function.
D. Hormonal control of testicular function (see Figure 7-2)
1. Hypothalamic-hypophysial-seminiferous tubular axis
a. LH and FSH are required for spermatogenesis. Since the effects of LH are mediated by
testosterone, testosterone and FSH are two hormones that act directly on Sertoli cells to
promote gametogenesis.

(1) Exogenous testosterone alone does not promote spermatogenesis in men lacking Leydig
cells. Spermatogenesis requires that a high concentration of testosterone be produced
locally by LH action on Leydig cells.
(2) Sertoli cells synthesize androgen-binding protein by an FSH-dependent process. This
protein binds testosterone and dihydrotestosterone and functions as a local androgenic
pool to support spermatogenesis.
(3) Sertoli cells also synthesize inhibin in response to FSH secretion. This protein inhibits
FSH secretion by direct negative feedback on the pituitary gland. Inhibin is not known
to suppress secretion of FSH releasing hormone (FSH-RH), a decapeptide produced by
the parvicellular peptidergic neurons. The selective rise in plasma FSH levels in individuals with damaged seminiferous tubules is due to a reduced secretion of inhibin.
b. Since normal spermatogenesis occurs in men with a Sa-reductase deficiency, dihydrotestosterone is not required for normal sperm development.
c. Plasma physiologic levels of androgens have little effect on the inhibition of FSH secretion.
d. Testosterone administration has little effect on FSH secretion, and very large doses are
required to suppress FSH in the male.
2. Hypothalamic-hypophysial-Leydig cell axis
a. The rate of testosterone synthesis and secretion by Leydig cells is stimulated primarily by
LH. The secretion of testosterone, in turn, inhibits LH secretion. It is unbound testosterone
that suppresses LH secretion. The likely major target of negative feedback is the hypothalamus.
b. Both testosterone and estradiol can inhibit LH secretion; however, since dihydrotestosterone
can also suppress LH, androgen conversion to estrogen is not a prerequisite for this inhibitory
action on the hypothalamus and pituitary gland.
c. Some neurosecretory neurons of the hypothalamus secrete a releasing hormone into the
hypophysial portal system, which then conveys it to the anterior lobe of the pituitary gland.
(1) This decapeptide, called gonadotropin-releasing hormone (Gn-RH), stimulates the
pituitary basophils.
(2) Gn-RH also is called LH releasing hormone (LH-RH) and FSH-RH because it elicits the
secretion of both LH and FSH.
E. Physiologic effects of androgens (Table 7-8)
1. Reproductive function. Androgens are essential for the control of spermatogenesis, the maintenance of the secondary sex characteristics, and the functional competence of the accessory
sex organs.
a. The accessory sex organs consist of excretory ducts and glands that transmit spermatozoa
and that secrete seminal fluid necessary for the survival and motility of spermatozoa after
ejaculation. The major accessory sex organs are the prostate gland and seminal vesicles.
b. The secondary sex characteristics are the physiologic characteristics of masculinity (e.g.,
growth of facial hair, recession of hair at the temples, enlargement of the larynx, thickening
of the vocal cords).
c. Seminal plasma, the fluid in which spermatozoa normally are ejaculated, originates almost
entirely from the prostate gland and seminal vesicles.
(1) The volume of the human ejaculate (semen) is 2-S ml, most of which is contributed by
the seminal vesicles.
(a) The prostate gland is the origin of citric acid, acid phosphatase, zinc, and spermine.

Table 7-8. Major Actions of Androgenic Hormones


Life Stage

Testosterone

Dihydrotestosterone

Fetal period

Development of epididymis, vas


deferens, and seminal vesicles

Development of penis, penile urethra, scrotum, and prostate gland

Puberty

Growth of penis, seminal vesicles,


musculature, skeleton, and larynx

Growth of scrotum, prostate gland,


pubic hair, and sebaceous glands

Adulthood

Spermatogenesis

Prostatic secretions

Reprinted from Genuth SM: The reproductive glands. In Physiology, 2nd edition. Edited by Berne RM, Levy MN. St.
Louis, CV Mosby, 1988, p 999.

(b) The seminal vesicles are the source of prostaglandins, fructose, ascorbic acid, and
phosphorylcholine. Metabolism of fructose provides energy for sperm motility.
(2) Sperm represents less than 10% of the ejaculate volume. The ejaculate contains 100300 million spermatozoa!
(a) Euspermia is defined as greater than 20 million sperm/ml of ejaculate.
(b) Oligospermia is defined as 5-20 million sperm/ml of ejaculate.
(c) Azoospermia is defined as 5 million sperm/ml of ejaculate.
2. Biologic effects. Androgens stimulate cell division as well as tissue growth and maturation and
are classified as protein anabolic hormones. (This anabolic effect in muscle is referred to as the
myotropic effect of androgens.) Only testosterone and dihydrotestosterone have significant
biologic activity. The 17-ketosteroids, androstenedione, DHEA, and etiocholanolone, are weak
androgens but important metabolites of testosterone.
a. In the adolescent, androgens produce linear growth, muscular development, and retention
of nitrogen, potassium, and phosphorus. Testosterone also accelerates epiphysial fusion of
the long bones. The skeletal development during puberty, particularly of the shoulder girdle,
is pronounced.
b. Testosterone stimulates differentiation of the wolffian duct system into the epididymis,
vas deferens, and seminal vesicles; dihydrotestosterone stimulates organogenesis of the
urogenital sinus and tubercle into the prostate gland, penis, urethra, and scrotum.
c. Androgens produce a low-pitched voice; stimulate growth of chest, axillary, and facial hair;
and cause temporal hair recession.
d. Androgens are responsible for libido.
e. Testosterone is a requisite for normal spermatogenesis.

x.

OVARY
A. Embryology

1. Internal genitalia (see Figure 7-8). The primordia of both male and female genital ducts, which
are derived from the mesonephros, are present in the fetus at 7 weeks gestation. The paired
mullerian ducts form parallel to the paired wolffian ducts. In the female fetus, the upper ends
of the mullerian ducts are the anlagen of the fallopian tubes (oviducts), whereas the lower ends
join to form the uterus, cervix, and upper end of the vagina.
a. The uterus and fallopian tubes, which develop from the mullerian ducts, do not require the
presence of an ovary.
b. In the absence of a fetal testis, mullerian duct inhibiting factor and testosterone are not
secreted by the fetal Sertoli cells and Leydig cells, respectively. Moreover, dihydrotestosterone is not formed from testosterone. Without the presence of these three hormones, at lO11 weeks gestation, the mullerian ducts begin to differentiate and the wolffian ducts undergo
regression.
c. This process of female genital duct development is completed at 18-20 weeks gestation.
2. External genitalia
a. The external genitalia of both sexes begin to differentiate at 9-10 weeks gestation. In the
female, this process proceeds without any known hormonal influence.
b. The external genitalia and urethra in both sexes develop from common anlagen, which are
the urogenital sinus and the genital tubercle, folds, and swelling.
(1) The urogenital sinus gives rise to the lower portion of the vagina and to the urethra.
(2) The genital tubercle is the origin of the clitoris.
(3) The genital swelling is the primordium of the labia majora.
(4) The genital folds develop into the labia minora.
3. Ovary. In the absence of the H-Y antigen, the gonadal primordium develops into an ovary,
provided that germ cells are present. Ovarian development occurs several weeks later than
does testicular differentiation.
a. At 8 weeks, when the testicular secretion of testosterone begins and before the ovarian
differentiation is completed, the fetal "ovary" has the capacity to synthesize estradiol.
(1) It is unlikely that the fetal ovary contributes significantly to the circulating estrogens in
the fetus. The fetus is exposed to estrogen (estriol) of placental origin. The site of
estradiol synthesis by the primordial ovary is not known.
(2) The ovary has no role in sex differentiation of the female genital tract.

b. Also at about 8 weeks gestation, the cortex of the primitive gonad undergoes active mitosis,
and epithelial cells infiltrate the gonad as a syncytium of tubules and cords. Primordial germ
cells are carried along with this inward migration.
(1) Proliferation of the cortex ceases at about 6 months.
(2) The rete ovarii secretes a meiosis-inducing factor.
(3) The germ cells begin to proliferate to form oogonia. This mitotic process is maximal
between 8 and 20 weeks, after which it diminishes, ceases, and never is resumed.
c. Unlike the fetal testis, the fetal ovary begins gametogenesis. Oogenesis, the formation of
primary oocytes from oogonia, begins at 15 weeks and reaches a peak between 20 and 28
weeks gestation.
(1) The primary oocytes enter a prolonged prophase (diplotene stage) of the first meiotic
division and remain in this state until ovulation occurs between 10 and 45 years later!
(2) The diploid primary oocytes become enveloped by a single layer of flat granulosa
cells and in this form are called primordial follicles. The formation of primordial follicles
reaches a peak between 20 and 25 weeks.
d. Also between 20 and 25 weeks, the gonad has acquired the morphologic appearance of an
ovary. During this period, the plasma concentration of pituitary FSH reaches a peak and the
first primary follicles appear [see X E 3 a (1 )].
B. Oogenesis
1. Fetal oogenesis. The period of oogonial proliferation results in a peak population of about 67 million germ cells in the two ovaries at 5 months gestation. Included in this group of cells are
oogonia, oocytes in various stages of prophase, and degenerating germ cells. This total number
of germ cells decreases to 2 million at term. The number of primordial follicles present in the
ovary at birth rapidly diminishes thereafter.
2. Postnatal oogenesis. By 6 months postpartum, all of the oogonia have been converted to
primary oocytes. By the onset of puberty, the number of primary oocytes has decreased to
about 400,000.
3. Prepubertal oogenesis. Between birth and puberty, the primary oocyte is surrounded by the
zona pellucida and six to nine layers of granulosa cells. These follicles are in varying stages of
development.
4. Pubertal oogenesis. In contrast to the male who produces spermatogonia and primary spermatocytes continuously throughout life, the female cannot form oogonia beyond 28 weeks gestation and must function with a declining pool of oocytes.
a. Meiosis in the female results in the formation of one viable oocyte (ootid). In contrast, each
primary spermatogonium in the male ultimately gives rise to 64 spermatozoa.
b. Oogenesis in the female begins in utero in response to meiosis-stimulating factor, while in
the male spermatogenesis is arrested at the spermatogonial stage in response to meiosisinhibiting factor.
c. Just prior to ovulation, the first polar body is extruded from the primary oocyte, which
completes the first meiotic division, and forms a secondary oocyte.
(1) This haploid cell immediately begins the second meiotic division but remains in metaphase.
(2) Extrusion of the second polar body (polocyte) does not occur until the mature ovum
(ootid) is fertilized by a sperm cell. Fertilization normally occurs in the ampulla of the
fallopian tube.

C. Morphology
1. Gross anatomy
a. The ovaries are ovoid glands with a combined weight of 10-20 g during the reproductive
years.
b. The ovaries are anchored to the broad ligament by the mesovarium.
2. Functional histology of the ovary and uterus
a. Ovary
(1) Structural divisions include the cortex (which is lined by the germinal epithelium and
contains all of the oocytes), the inner medulla, and the hilus (i.e., the point where the
ovary attaches to the mesentery).

(2) Functional subunits include the follicle and oocyte (each consisting of theca cells
and granulosa cells) and the corpus luteum.
(3) Gametogenesis in the female denotes folliculogenesis, which leads to the formation
of a mature ovum.
(a) During the preovulatory phase, the functional unit of the ovary is the follicle.
(b) During the postovulatory phase, the functional unit of the ovary is the corpus luteum.
(4) Steroidogenesis in the ovary is the synthesis and secretion of estradiol and progesterone. Although steroidogenesis occurs in three morphologic units (i.e., the follicle, corpus
luteum, and stroma), only the follicle and corpus luteum are major steroid-producing
units.
(a) Theca interna cells produce androstenedione and testosterone, which diffuse into
the granulosa cells.
(b) Granulosa cells synthesize estradiol and estrone from androgenic precursors produced by the theca interna cells.
b. Uterus
(1) Layers. The uterus consists of two major tissue layers.
(a) The outer layer, the myometrium, is a thick layer of smooth muscle.
(b) The inner layer of the uterus is the endometrium. At the height of its development,
during the luteal phase, the endometrium is approximately 5 mm thick. On the basis
of blood supply, the endometrium can be divided into two major layers.
(i) The stratum basalis is the abluminal (deeper) layer of the endometrium. This
layer functions as the regenerative layer in the growth and differentiation of
endometrial tissue that is sloughed during menses.
(ii) The stratum functionalis is the adluminal (superficial) layer of the endometrium,
which is shed during menses.
(2) Endometrial blood supply
(a) The stratum basalis receives its vascular supply from the basal (straight) arteries,
which arise from the uterine radial arteries.
(b) The stratum functional is is perfused by the spiral (coiled) arteries, which also
emanate from the radial arteries.
(c) Thus, the arcuate arteries give off radial arteries, and the radial arteries bifurcate to
form the basal and coiled arteries.

D. Hormones of the ovary: steroids. Ovarian hormones include two phenolic steroids-estradiol
(C-18) and estrone (C-18)-and the progestogen, progesterone (C-21).

1. Secretion and transport (Tables 7-9,7-10)


a. Estrogens. Over 70% of circulating estrogens are bound to sex steroid-binding globulin,
and 25% are bound to plasma albumin.
(1) Estradiol is the principal and biologically most active estrogen secreted by the ovary.
Ovarian estradiol accounts for more than 90% of the circulating estradiol.
(2) Estrone, a weak ovarian estrogen, also is formed by the peripheral conversion of
androstenedione.
(a) In premenopausal women, most of the circulating estrone is derived from estradiol
by conversion via 17-hydroxysteroid dehydrogenase.
(b) In postmenopausal women, estrone is the dominant plasma estrogen and is derived
via the prohormone pathway. Specifically, estrone is derived from the conversion
of adrenocortical androstenedione in peripheral tissues (mainly liver). In obese
women, there is a significant peripheral conversion of androstenedione to estrone
by adipose tissue. This extraovarian synthesis of estrogen is implicated in the higher
incidence of endometrial carcinoma in obese women.
(3) Estriol, the weakest of all of the naturally occurring estrogens, is synthesized by the
placenta and the liver but not the ovary. In nongravid women, estriol is formed in the
liver as a conversion product of estradiol and estrone.
b. Progesterone is not bound to sex hormone-binding globulin. The progesterones are bound
primarily to CBG (transcortin) and albumin.

Table 7-9. Types of Steroids and their Systemic Concentrations and Rates of Synthesis in Women

Steroid

Plasma
Concentration
(ng/dl)

Production
Rate (,...g/day)

Estrogens (C-18)
Estradiol
Early follicular phase
Late follicular phase
Middle luteal phase

6
50
20

80
700
300

Estrone
Early follicular phase
Late follicular phase
Middle luteal phase

5
20
10

100
500
250

30
200
200

600
4000
4000

100
1000

2000
25,000

40
20
150
500

250
50
3000
8000

Progestogens (C-21)
17-Hydroxyprogesterone
Early follicular phase
Late follicular phase
Middle luteal phase
Progesterone
Follicular phase
Luteal phase
Testosterone
Dihydrotestosterone
Androstenedione
Dehydroepiandrosterone

Reprinted from Lipsett MB: Steroid hormones. In Reproductive Endocrinology: Physiology, Pathophysiology and
Management. Edited by Yen SSC, Jaffe RB. Philadelphia, WB Saunders, 1978, p 84.

Table 7-10. Serum FSH and LH Concentrations During the Life Cycle of the Normal Female'
FSHt

lHt

Prepubertal period
(5-11 years)

4.5

3.9

Puberty
(11-13 years)

6.8

8.2

Reproductive period
Follicular phase
Midcycle
Luteal phase

8.3
19.3
6.9

12.8
83.5
11.6

Postmenopausal period

96.0

66.0

Stage of Life

Adapted from Ontjes DA, Walton j, Ney RL: The anterior pituitary gland. In Metabolic Control of Disease, 8th edition.
Edited by Bondy PK, Rosenberg LE. Philadelphia, WB Saunders, 1980, p 1192.
Mean values without standard deviations.
lConcentrations expressed in milli-International units.

2. Ovarian steroidogenesis (see Figure 7-7). The two pathways for biosynthesis of ovarian
steroids have in common the conversion of cholesterol to pregnenolone, a reaction stimulated
by LH and FSH via 20,22-desmolase.
a. One pathway proceeds by way of the ilS-pathway, which involves the synthesis of 17ahydroxypregnenolone and DHEA via 17a-hydroxylase and 17,20-lyase, respectively.
(1) DH EA, a 17 -ketosteroid,' is converted to another androgenic 17-ketosteroid,' androstenedione, by 313-hydroxysteroid dehydrogenase and ils-reductase.

'These 1 7 -ketosteroids are steroids that consist of 19 carbon atoms.

(2) Androstenedione can be reduced by 17-hydroxysteroid dehydrogenase to testosterone,


which is a reversible reaction.
(3) Testosterone is a precursor of estradiol via an aromatase reaction.
b. The other pathway proceeds via the conversion of pregnenolone to progesterone by 3J3-hydroxysteroid dehydrogenase and Jls-isomerase. Progesterone is the initial compound in the
Jl4-pathway.
(1) Progesterone is converted to 17a-hydroxyprogesterone by 17a-hydroxylase.
(2) 17a-Hydroxyprogesterone is another precursor for androstenedione via another 17,
20-lyase step.
(3) Androstenedione and testosterone are interconvertible with the enzyme 17-hydroxysteroid dehydrogenase.
(4) Androstenedione and testosterone are converted to estrone and estradiol, respectively,
by the action of aromatase. These two 18-carbon steroids (estrogens) also are interconvertible with 17-hydroxysteroid dehydrogenase.
3. Metabolism of ovarian steroids. The liver is the major site of steroid metabolism.

a. Catabolism of progestogens
(1) Progesterone is converted to pregnanediol.
(2) 17a-Hydroxyprogesterone is catabolized to pregnanetriol.

b. Catabolism of estrogens
(1) Large quantities of both estradiol and estrone are hydroxylated (primarily in the liver)

at the C-l 6 position to form estriol.


(2) Another major catabolic route for estrogens is hydroxylation at the C-2 and C-4 positions, which yields the catecholestrogens.
c. Estrogens are excreted in the urine in the form of soluble conjugates.
(1) Estriol, catecholestradiol, and catecholestrone are excreted primarily as glucuronidates.
(2) Estrone is excreted primarily as a sulfate conjugate.

E. Ovarian function
1. Menstruation
a. Menarche refers to the onset of menstruation, which normally occurs between the ages of
12 and 14 years. Prior to menarche, minimal amounts of estrogen are produced by the
peripheral conversion of androgens.

b. Menstrual cycle
(1) Duration. Although a duration of 25-30 days is considered typical, a cycle length of
28 days is the exception rather than the rule in adult women. In early adolescence, the
cycle is characterized by irregular menses and anovulation.
(2) Temporal reference points (Figure 7-9). The menstrual cycle conventionally begins
with the first day of menstruation, when the endometrial lining is shed along with blood
and uterine secretions. Days of the menstrual cycle are measured using two different
reference points.
(a) One system designates the first day of menses as day 0 and the last day as day 28.
(b) The other system designates the day of the LH peak (ovulation) as day 0, with
preovulatory days indicated with a minus sign and postovulatory days indicated
with a plus sign.
c. Menopause refers to the cessation of menses, which typically occurs at about age 50
following a gradual decrease in frequency. Menopause is due to a primary hypogonadism
(i.e., cessation of ovarian steroid secretion) and is associated with an increased gonadotropin
(predominantly FSH) secretion.

2. Ovarian cycle
a. Preovulatory phase. This phase is marked by follicular growth and maturation and by
endometrial proliferation. The preovulatory phase generally lasts 8-9 days but can be
quite variable (10-25 days). During this phase, the stratum basalis regenerates a stratum
functional is, and, by the end of this phase, one follicle (rarely more) has reached the final
stage of growth.
(1) A primary follicle begins as an oocyte surrounded by a single layer of cuboidal epithelial
cells called granulosa cells. The primary follicle becomes multi laminar by the mitosis of
the granulosa cells, which occurs with maturation.
(2) Upon cavitation of the granulosa, an antrum is formed, which is filled with liquor
folliculi secreted by the granulosa cells. The developing follicle now is called the
secondary follicle (vesicular follicle or, more commonly, graafian follicle).

36'8~
36.5
36.2
L-__________________________

60
LH
(mlu/ml)

40

20

20

FSH
(mlu/ml)

10

10
Prog
(ng/ml)

5
0

E2
(pg/ml)

'l
100

2
17a-OH Prog
(ng/ml)
Days from
LH peak:
Days from start
of cycle (menses):

+-M---+

Ovary:
Endometrium:

Postovulatory
Proliferative

Secretory

Figure 7-9. Hormonal (ovarian and


pituitary), uterine (endometrial), and
basal body temperature (887) correlates of the normal menstrual cycle.
Mean plasma concentrations ( SEM)
of LH, FSH, progesterone (Prog), estradiol (E2 ) (day + 1) and 17ahydroxyprogesterone (17a-OHProg)
are shown as a function of time. Ovulation occurs on day 15 (day + 1) following the LH surge, which occurs at
midcycle on day 14 (day 0). M =
menses, 0 = ovulation. (Adapted
from Thorneycroft lA, et al: The relation of serum 17-hydroxyprogesterone and estradiol 1 7-f3 levels during
the human menstrual cycle. Am) Obstet Cyneco/111 :947-951, 1971.)

(a) The granulosa cells secrete a protective shell, the zona pellucida, which surrounds
the oocyte.

(b) The stroma gives rise to a bilaminar theca, which surrounds the granulosa cells but
is separated from them by a basal lamina (lamina propria).
(i) The theca interna is a well-vascularized layer consisting of steroid-secreting cells
that lie on the basal lamina. The blood vessels do not penetrate this membrane
and, therefore, the granulosa cells are avascular until after ovulation.
(ii) The theca ederna is peripheral to the theca and is composed mainly of fibrous
connective tissue. It is less vascular than the theca interna.
(3) Just prior to ovulation, the primary oocyte of the secondary follicle completes the first
meiotic division, which began prior to birth, and forms a secondary oocyte with a
haploid nucleus and the first polar body. The second meiotic division takes place in the
ampulla of the oviduct and occurs only if fertilization occurs.
b. Ovulation. The secondary oocyte is released from the secondary follicle by a process called
ovulation, which usually occurs on day 15 of the average cycle. (In reference to the LH
peak, this midcycle event occurs on day 1, that is, one day following the LH surge.)
c. Postovulatory phase. The next 13-14 days constitute the postovulatory phase, during
which the endometrium is prepared for the possible implantation of the fertilized ovum,
which is a blastocyst when it arrives in the uterine cavity. This phase is relatively constant
in duration. As a result, the day of ovulation can be estimated by subtracting 14 days from
the total length of the menstrual cycle.
(1) If fertilization does not occur, implantation also does not occur because the hormonal
maintenance of the endometrial growth and differentiation is withdrawn.
(2) Without conception, ischemia and necrosis of the luminal endometrium result after 14
days, and the ensuing menses marks the beginning of another menstrual cycle.
(3) If conception occurs, the functional lifespan of the corpus luteum is extended, and it
continues to secrete estradiol and progesterone at increasing rates during the first 6-8
weeks gestation.

F. Neuroendocrine control of ovarian function (see Figure 7-4). The ovarian cycle is associated
with the secretion of ovarian steroids (estradiol and progesterone), which, in turn, are regulated
by FSH and LH from the pituitary gland. A single hypothalamic hormone, Gn-RH, differentially
regulates the secretion of FSH and LH.

1. Ovarian cycle
a. Preovulatory phase. Under the influence of FSH and LH, the primary follicle begins to
develop. The combined effects of LH on the theca" cells to produce androgenic proestrogens
and of FSH on the granulosa cells to aromatize these androgens to estrogens (estradiol)
result in a slowly increasing blood estradiol concentration. During the preovulatory phase,
the dominant gonadotropic hormone is FSH, and the dominant steroid is estradiol. Therefore,
the preovulatory phase of the ovarian cycle also is referred to as the follicular phase and
the estrogenic phase.
(1) Plasma estradiol concentration reaches a peak about 24 hours prior to the surge in LH
secretion, or about 48 hours prior to ovulation. The peak in plasma estradiol concentration occurs on day 13 (day -1). t
(a) As the plasma estradiol concentration increases, it exerts a negative feedback on
the hypothalamic-hypophysial complex, resulting in a gradual decline in FSH. LH
levels rise slightly through the follicular phase.
(b) Inhibin secretion by the granulosa cells also exerts a negative feedback effect on
FSH secretion.
(2) The peak in plasma estradiol concentration exerts a positive feedback effect on the
hypothalamic-hypophysial axis, causing a reflex release of Gn-RH and a concomitant
surge in pituitary LH secretion 24 hours later, on day 14 (day 0).
(a) A lesser increase in plasma FSH secretion occurs on day 14 (day 0).
(b) Increased plasma estradiol levels inhibit FSH secretion both directly and indirectly
at the level of the pituitary gland and ventral diencephalon, respectively.

'For the remainder of this chapter, the term theca denotes theca interna.
'Also throughout this chapter, the cycle days are numbered with reference to the onset of menses and with reference
to the LH peak (in parentheses).

(3) The follicular phase usually lasts 14 days, but any variability in the duration of the
menstrual cycle usually is attributable to variability in the length of the follicular phase.
It should be noted that the follicular phase begins with the first day of menses, while
the proliferative phase of the endometrium begins with the last day of menses.
(4) There is a fall in the plasma estradiol level following the estradiol peak. Note in Figure
7-9 that this fall in plasma estradiol precedes ovulation.
(5) During the preovulatory phase, the plasma progesterone concentration remains low.
(a) The bulk of this progesterone is derived from the peripheral conversion of adrenal
progestogens; however, large amounts of progesterone exist within the follicular
antrum.
(b) The principal progestin secreted by the granulosa cells during the late follicular
phase is 17a-hydroxyprogesterone.
b. Ovulatory phase. Ovulation occurs on day 15 (day + 1) in response to the surge of LH
secretion that occurred 24 hours earlier.
(1) Ovulation refers to the extrusion of a haploid secondary oocyte into the peritoneal
cavity. The oocyte enters the oviduct (fallopian tube) where fertilization occurs.
(2) The rupture of the secondary follicle by the midcycle surge of LH leads to the formation
of a new endocrine tissue (i.e., the corpus luteum), which involves proliferation, vascularization, and luteinization of the theca and granulosa cells. LH, therefore, is called the
luteotropic hormone of the menstrual cycle.
c. Postovulatory phase. LH also maintains the functional status of the corpus luteum during
the postovulatory phase. The hormone secreted in the greatest amounts by the corpus
luteum during this phase is progesterone. For these reasons, the postovulatory phase also is
called the luteal phase and the progestational phase.
(1) The decline in estrogen secretion prior to ovulation and at the time of ovulation removes
the positive feedback effect of estradiol on gonadotropin secretion.
(2) Both FSH and LH levels fall after their midcycle peaks but remain sufficiently high to
stimulate the newly formed lutein-theca cells and lutein-granulosa cells to secrete estradiol, estrone, and progesterone.
(3) About 6 days after ovulation, on day 21 (day + 7), the plasma concentrations of progesterone and 17a-hydroxyprogesterone peak coincidently with the second peak of plasma
estradiol concentration. Note in Figure 7-9 that this second peak is lower and broader
than the estradiol peak that occurs during the preovulatory phase.
(4) The plasma concentrations of progesterone during the entire ovarian cycle are higher
than those of estradiol. It is imperative to note that the units of concentration for
progesterone (ng/ml) are 1000 times greater than for estradiol (pg/ml).
(5) The effect of the raised plasma estradiol and progesterone levels is a negative feedback
on FSH and LH, respectively. Progesterone acts as an antiestrogen at this time because
it inhibits LH secretion when the second estradiol peak occurs.
(6) LH levels continue to decline during the luteal phase, while FSH levels begin to rise
progressively during the late luteal phase.
(7) The total amount of estradiol secreted during the follicular phase is comparable to that
secreted during the luteal phase. This can be appreciated by comparing the areas under
the estradiol curve during these two phases, using day 15 (day + 1) as the dividing line
between the follicular and luteal phases.
(8) Unless conception occurs and is followed by implantation of the blastocyst, the corpus
luteum undergoes involution following the reduction in gonadotropin secretion.
(a) The declines in estradiol and progesterone secretion remove the negative feedback
effect on the hypothalamic-hypophysial complex.
(b) The corpus luteum regresses after about 12 days of steroid hormone secretion.
(9) Progesterone is associated with a 0.2-0.5 C rise in basal body temperature, which
occurs immediately following ovulation and which persists during most of the luteal
phase (see Figure 7-9).
(a) The basal body temperature dips during the follicular phase.
(b) This temperature increment is used clinically as an index of ovulation.
2. Endometrial cycle
a. Hormonal effects on the myometrium. Estradiol and progesterone are antagonistic with
respect to their effects on the myometrium: estrogens promote uterine motility, and progestogens inhibit myometrial contractility.

b. Hormonal effects on the endometrium. Estradiol and progesterone are synergistic with
respect to their effects on the endometrium during the proliferative and secretory phases.
(1) The proliferative (preovulatory) phase of the menstrual cycle refers to the endometrial
changes that occur in response to estradiol.
(a) Estrogens stimulate mitosis of the stratum basalis, also correctly termed the stratum
basale, which regenerates the stratum functional is.
(b) Estrogens stimulate angiogenesis (neovascularization) in the stratum functionalis as
well as stimulate the growth of secretory glands. The blood vessels become the
spiral arteries that perfuse the stratum functionalis, also correctly termed the stratum
functionale. The glands contain glycogen but are non secretory at this time.
(c) The cervical epithelium secretes a watery mucus in response to estrogen stimulation.
(2) The secretory (postovulatory) phase is characterized by secretion of large amounts
of both progesterone and estradiol by the corpus luteum. The endometrium during this
secretory phase is hyperemic and has a "lace curtain" or "swiss cheese" appearance.
(a) Progesterone promotes differentiation of the endometrium, including elongation and
coiling of the mucous glands (which secrete a thick viscous fluid containing glycogen)
and spiraling of the blood vessels.
(b) Unless fertilization occurs, hormone secretion by the hypothalamic-hypophysial
complex and ovarian steroid secretion decline on about day 25 (day + 11).
(i) Menses, beginning on day 0 (day -14) of the following cycle, starts with vasoconstriction of the spiral arteries, which causes ischemia and necrosis.
(ii) The necrotic tissue releases vasodilator substances, causing vasodilation. The
necrotic walls of the spiral arteries rupture, causing hemorrhage and shedding of
cells over a period of 4-5 days.

G. Physiologic effects of ovarian steroids


1. Estrogens have important protein anabolic effects. Estrogens are responsible for the growth
and development of the fallopian tubes, uterus, vagina, and external genitalia as well as the
maintenance of these organs in adulthood. These steroids also promote cellular proliferation in
the mucosal linings of these structures.
a. Endometrium. Estrogens stimulate the regeneration of the stratum functionalis during the
proliferative phase of the endometrial cycle.
(1) The water content and blood flow to the endometrium are increased markedly.
(2) The spiral arteries of the stratum functionalis are especially sensitive to estrogens and
grow rapidly under their influence.
b. Myometrium. Estrogens increase the amount of contractile proteins (i.e., actin and myosin)
in the myometrium and, thereby, increase spontaneous muscular contractions. Estrogens
also sensitize the myometrium to the action of oxytocin, which promotes uterine contractility.
c. Cervix. Under the influence of estrogens, the uterine cervix secretes an abundance of thin,
watery mucus.
(1) This fluid can be drawn into very long threads when placed between two glass slides.
This is a clinical index of estrogen activity called spinnbarkheit.
(2) Cervical mucus also demonstrates the phenomenon of crystallization when it is dried
on a glass slide. The characteristic ferning pattern is due to the accumulation of sodium
chloride. This phenomenon also is used diagnostically as an index of endogenous
estrogen secretion.
d. Breast. Estrogens promote the development of the tubular duct system of the mammary
gland. Estrogens are synergistic with progesterone in stimulating the growth of the lobuloalveolar portions of this gland.
e. Bone. Estrogens, like androgens, exert a dual effect on skeletal growth in that they cause
an increase in osteoblastic activity, which results in a growth spurt at puberty.
(1) Estrogens hasten bone maturation and promote the closure of the epiphysial plates in
the long bones more effectively than does testosterone. Therefore, the female skeleton
usually is shorter than the male skeleton.
(2) Estrogens are responsible for the oval or roundish shape of the female pelvic inlet. This
inlet in the male is spade-shaped.
(3) Estrogens, to a lesser degree than testosterone, promote the deposition of bone matrix
by causing Ca2+ and HPOl- retention. In large amounts, estrogens also promote
retention of Na+ and water.

f. Liver. Estrogens stimulate the hepatic synthesis of the transport globulins, including
thyroxine-binding globulin and transcortin.
(1) This results in increased plasma concentrations of thyroxine and cortisol but unchanged
amounts of free thyroxine.
(2) Pregnant women often are in a state of mild hyperadrenocorticism because the elevated
placental progesterone competes with cortisol for binding sites on transcortin, thus
increasing plasma free cortisol.

2. Progesterone
a. Endometrium. The endometrium, which proliferates under the influence of estrogens,

b.

c.
d.
e.

becomes a secretory structure under the influence of progesterone.


(1) The endometrial glands become elongated and coiled and secrete a glycogen-rich fluid.
(2) Progesterone accounts for the differentiation of the stratum functional is.
Cervix. Under the influence of progesterone, the mucus secreted by the cervical glands is
reduced in volume and becomes thick and viscid. This consistency of cervical mucus
together with the absence of "ferning" provide presumptive evidence that ovulation and
luteinization have occurred.
Myometrium. Progesterone decreases the frequency and amplitude of myometrial contractions.
Breast. This steroid also promotes lobuloalveolar growth in the mammary gland.
Kidney. Progesterone promotes renal excretion of Na + .

XI. ENDOCRINE PLACENTA


A. Placenta formation
1. Timetable of early placental function (days measured from ovulation). The gestational period,
measured from the time of conception (ovulation) to parturition, is 38 weeks (266 days).
a. Day 0: Fertilization occurs in the distal portion of the oviduct, or ampulla.
(1) Fertilization triggers the final stage of the second meiotic division of the oocyte. The
second polar body is extruded from the oocyte, and a haploid number of chromosomes
are present in the female pronucleus.
(2) The lifespan of an unfertilized ovum is less than 20 hours following ovulation; sperm
cells are viable for about 24 hours after ejaculation.
b. Day +3 or +4: The morula enters the uterine cavity.
c. Day +5 or +6: The morula forms a cavity, the blastocoele, which is transformed into a

blastocyst.
d. Day + 7: The blastocyst is implanted into the endometrium. By the end of 1 week, a primitive
uteroplacental circulation begins to develop.

e. Day +21: The placenta is fully functional.


2. Early placental formation
a. At the time of implantation, or nidation, the blastocyst consists of two cellular masses
(Figure 7-10).
(1) The inner cell mass, the embryoblast, will form the embryo and, eventually, the fetus.
(2) The outer rim of cells, the trophoblast, forms the attachment to the endometrium and
gives rise to the fetal membranes.
b. The endometrium, under the influence of progesterone secreted by the corpus luteum, is
transformed into a decidua, which is the maternal portion of the placenta that surrounds
the conceptus.
3. Placental development. The trophoblast, which is entirely fetal in origin, develops into the
placenta. The trophoblast forms two cell layers (see Figure 7-10).
a. The inner layer forms the cytotrophoblast, which is on the fetal side of the blastocyst and
is the progenitor of the syncytiotrophoblast.
b. The outer layer forms the syncytiotrophoblast.

B. Hormones of the placenta (Figures 7-11, 7-12, 7-13). The fetus, placenta, and mother are
interdependent and constitute a functional unit called the feto-placento-maternal unit. The
pregnant woman at or near term produces 15-20 mg/ day of estradiol, 50-100 mg/ day of estriol,
250-300 mg/day of progesterone, 1-2 mg/day of aldosterone, and 3-8 mg/day of deoxycorticosterone (DOC). By itself, the placenta is an incomplete steroid-producing organ.

Endometrial epithelium

QlII(#I:~!-Endometrial

gland

Yolk sac -P-=.L...;;ff-f;':H-fb!-''l\-7!-

Embryoblast

_]~~~~~~~IIII~I~lij~=~ij4tcytotroPhOblast
Amniotic cavity

y--...,-..-.J""".~....=r ,Jr"N"'-'-"".um-H-.i'-----"----""::;:::'~- Syncytiotrophoblast

~~t:;~rjt?:P I/!,;-~::;":::':+:~.,L-

Maternal blood vessels

Figure 7-10. Structures formed by an implanting conceptus within the uterine endometrium. (Reprinted from Johnson
KE: Human Developmental Anatomy. Baltimore, Williams & Wilkins, 1988, p 52.)

1. HCG is a polypeptide containing 236 amino acid residues, making it the largest active peptide
hormone produced in humans.
a. Synthesis and secretion. The syncytiotrophoblast is the source of HCG, which is secreted
soon after fertilization.
b. Plasma concentration
(1) HCG reaches a plasma peak between 60 and 90 days gestation. This peak is 200 times
greater than the LH peak at the height of the ovulatory surge.
(2) HCG is detectable in maternal blood as early as 6-8 days after conception, which forms
the basis for the immunologic pregnancy test.
(3) HCG measurement in maternal blood is a useful index of the functional status of the
trophoblast.
2. Progesterone (see Figures 7-11, 7-12, 7-13)
a. Synthesis and secretion
(1) Placental progesterone is derived mainly from maternal cholesterol; the fetus does not
contribute significantly to placental progesterone formation.
(2) This C-21 steroid is synthesized by the trophoblast. Most (85%) of the progesterone
formed in the trophoblast is secreted into the maternal compartment.
b. Plasma concentration of placental progesterone rises steadily throughout gestation, reaching a maximal plateau at 36-40 weeks. There is no significant drop in plasma progesterone
concentration prior to labor.

--. -+= Multiple intermediate steps


FETAL ADRENAL CORTEX

I Cholesterol

-+ -+ DHEAS

FETAL LIVER

IDHEAS-+ 16a-OH DHEAS I


PLACENTA
Cholesterol -+ -+ Progesterone

16a-m DHEAS -+-+ E3

,r

12

-+ -+

DHEAS

Figure 7-11. Pathways of steroid hormone biosynthesis in the feto-placental unit. DHEAS = dehydroepiandrosterone
sulfate; 16a-OH DHEAS = 16a-hydroxydehydroepiandrosterone sulfate; E2 = estradiol; EJ = estriol. (Adapted from
Wilson JD, Foster DW: Williams Testbook of Endocrinology, 7th edition. Philadelphia, WB Saunders, 1985, p 423.)

= Multiple intermediate steps

FETUS

PLACENTA

MOTHER

Adrenal cortex

Adrenal cortex

~__D_H_EA__~I ~4--------------

DHEAS

DHEAS

!
AD

!
E, " - E2

116a - OH DHEAS

16a- OH DHEA
~
~

E3
Cholesterol

1
1Pregnenolone
Adrenal cortex

17a-Hydroxy~rogesterone _

proges}erone

Cortisol

Aldosterone

-I

1
Progesterone

I..

4 --------------

Liver, diet
Cholesterol

I
I

Figure 7-12. The primary pathways of estrogen and progesterone synthesis in the human feto-materno-placental
unit. DHEA(S) = dehydroepiandrosterone (sulfate); T6a-OH DHEA(S) = 16a-hydroxydehydroepiandrosterone (sulfate); AD = androstenedione; E, = estrone; E2 = estradiol; EJ = estriol.

Third trimester

Second trimester

First trimester

Progestogens /i

/,

/,

/,

/. Estrogens
/.
/.
/.

/
/
/

/
/

.....
:....

I
I
I

.........~:.:=.:=--~- -

I....,

i..L---

I
I

............................... ~..... . ... ... ....

2
Weeks gestation calculated from
first day of menses

~CG

. ..... .

Parturition

Figure 7-13. The patterns of maternal plasma hormone concentrations that occur during pregnancy. HCC = human
chorionic gonadotropin. (Adapted from Laycock j, Wise P: Essentials of Endocrinology, 2nd edition. New York,
Oxford University Press, 1983, p 155.)

(1) Both 17 a-hydroxyprogesterone and progesterone from the corpus luteum reach a peak

3-4 weeks postconception in response to HCG secretion.


(2) At 6-8 weeks postconception, progesterone reaches a nadir, while 17a-hydroxyprogesterone continues to decline. The 17a-hydroxyprogesterone levels reflect corpus
luteal secretion. The secondary rise in plasma progesterone reflects placental (trophoblast) secretion.
(3) An intact materno-placental circulation will produce essentially normal progesterone
levels, even in the event of fetal death.
c. Metabolism. The principal urinary metabolite of progesterone is pregnanediol, which is an
estimate of placental function.
d. Conversion to fetal corticoids
(1) The placenta produces pregnenolone from maternal cholesterol, but it lacks the enzymes
necessary for androgen synthesis (Le., 17a-hydroxylase and 17,20-desmolase).
(2) Pregnenolone synthesized by the placenta is oxidized to progesterone by 3,B-hydroxysteroid dehydrogenase/ b. 5 -isomerase.
(3) Placental progesterone circulates to the fetal adrenal cortex, where it is hydroxylated
at positions C-17, C-21, and C-ll to form aldosterone and cortisol. Thus, in early
pregnancy, the fetus requires placental progesterone to synthesize corticoids, because
the fetal zone of the adrenal cortex has a relative block in the 3,B-hydroxysteroid
dehydrogenase / b. 5 -isomerase system.
(4) Beyond 10 weeks gestation, the fetal adrenal cortex no longer depends on placental
progesterone for synthesis of aldosterone and cortisol.
3. Estrogens (see Figures 7-11, 7-12, 7-13). Quantitatively, estriol is the major estrogen of human
pregnancy, with smaller amounts of estradiol and estrone produced.
a. Synthesis and secretion
(1) These C-18 steroids are synthesized in the placental trophoblasts.
(2) Estriol is produced primarily from androgenic precursors formed in the fetal zone of the
adrenal cortex and the liver. The principal adrenal steroid is DHEA, which is a ketoste-

roid. DHEA is sulfoconjugated (sulfurylated) by sulfokinase* in the fetal adrenal to


DHEAS.
(a) DH EAS is the predominant precursor for estradiol and estrone synthesis after DHEAS
is deconjugated by placental sulfatase. These two estrogens contribute to the estriol
pool by conversion in the maternal liver.
(b) DHEAS is converted in the fetal liver to 16a-hydroxydehydroepiandrosterone sulfate by 16a-hydroxylase. This enzyme is not found in the placenta.
b. Plasma concentration. Like progesterone, the plasma estriol concentration rises steadily
throughout gestation, reaching a maximal plateau at 36-40 weeks. The secretory curve for
estriol parallels that for progesterone and correlates well with the fetal growth curve.
(1) Plasma estriol concentrations reflect the functional status of the feto-placental unit.
Falling levels indicate impending fetal death.
(2) Even with fetal death, plasma progesterone levels can remain within normal limits.

C. Physiologic effects of placental hormones


1. HCG is classified as an anterior pituitary-like hormone with biologic actions that mimic those
of LH (i.e., it can be used to induce ovulation).
a. HCG is a second luteotropic hormone, in that it maintains the function of the corpus luteum
until the feto-placental unit is autonomous in terms of hormone synthesis (about 6-7 weeks
postconception).
b. HCG converts the corpus luteum of menstruation into the corpus luteum of pregnancy,
thereby extending the functional lifespan of the corpus luteum.
c. HCG stimulates the corpus luteum of early pregnancy to secrete 17a-hydroxyprogesterone
and lesser amounts of progesterone, which reach a peak 3-4 weeks postconception. Blood
17a-hydroxyprogesterone level is an excellent indicator of corpus luteal function during
early pregnancy, because the placenta lacks significant 17a-hydroxylase activity.
d. HCG stimulates the fetal testis to secrete testosterone at a time prior to fetal pituitary LH
secretion.
e. HCG may serve as a tropic agent for the fetal zone of the adrenal cortex, which secretes
DHEA.
2. Progesterone
a. Progesterone inhibits uterine motility by hyperpolarization of the uterine myometrium.
b. It converts the secretory endometrium of the luteal phase of the menstrual cycle to the
decidua during pregnancy. Progesterone maintains the decidua.
c. Synergistic action of progesterone and estrogen is required to induce development of the
lobuloalveolar compartment, which prepares the breasts for lactation. Progesterone acts
primarily on the lobuloalveolar compartment.
d. Progesterone has an immunosuppressive role in protecting the fetus.
e. Progesterone contributes to the growth and development of the fetus (e.g., by acting as a
precursor for corticoid synthesis by the fetal adrenal cortex).
f. Progesterone promotes renal excretion of Na+, which antagonizes the effects of increased
aldosterone levels found in pregnancy.
3. Estrogens. The estrogenic effects of pregnancy are primarily due to estradiol, the most potent
of the estrogens.
a. Estrogens mediate the growth and development of the maternal reproductive organs.
(1) The gravid uterus increases 18-fold (about 1700%) in weight, beginning as a 60 g organ
in the nongravid state.
(2) During the gestational period, the uterus lengthens from 7 cm to 30 cm.
(3) Uterine volume at term is 500-1000 times greater than that before pregnancy.
(4) The increase in uterine size during pregnancy occurs by stretching and hypertrophy of
the myometrium.
b. Estrogens stimulate hepatic synthesis of thyroxine-binding globulin, steroid hormone-binding
globulin, and angiotensinogen as well as renal renin secretion. The latter two effects lead to
increased angiotensin II synthesis.
c. Estrogens stimulate development of the lactiferous ductal system in the mammary gland.

'Fetal sulfokinase conjugates metabolites of pregnenolone, progesterone derivatives, and the androgens (DHEA)
found in high levels in the feto-placental unit.

d. Just before term, the estrogen-to-progesterone ratio increases and the uterus is dominated
by estrogen.

XII. ENDOCRINE PANCREAS


A. Histology and function of the islets of Langerhans (Figure 7-14). The endocrine pancreas
consists of islet of Langerhans, which form less than 2% of the pancreatic tissue.

1. Cells. Four cell types have been identified.


a. Alpha cells make up about 25% of the islet cells and are the source of glucagon, which
consists of 29 amino acid residues.
b. Beta cells constitute about 60% of the islet cells and are associated with insulin synthesis.
This polypeptide consists of 51 amino acid residues.
c. Delta cells comprise about 10% of the islet cells and are the source of somatostatin, which
is a tetradecapeptide.
d. Pancreatic polypeptide cells comprise approximately 5% of the islet cells and synthesize
a polypeptide that contains 36 amino acid residues.
2. Neurotransmitters and epinephrine. Unmyelinated postganglionic sympathetic and parasympathetic nerve fibers terminate close to the three cell types (alpha, beta, and delta cells) and
modulate pancreatic endocrine function via the secretion of neurotransmitters.
a. ACh secretion causes insulin release only when glucose levels are elevated. ACh appears
to inhibit somatostatin release.
b. Norepinephrine secretion due to sympathetic stimulation via activation of the a-receptor
leads to inhibition of insulin release. Norepinephrine stimulates somatostatin release.
c. Epinephrine. Despite the dual a- and ,a-adrenergic receptor system in beta cells, the
a-adrenergic action of epinephrine predominates, so that insulin secretion in inhibited.
(Insulin release is mediated by a ,a-adrenergic receptor.)
3. Control of secretions. The alpha, beta, and delta cells constitute a functional syncytium, which
forms a paracrine control system for the coordinated secretion of pancreatic polypeptides (see
Figure 7-14L
a. Insulin inhibits alpha cell (glucagon) secretion, which increases peripheral glucose uptake
and opposes glucagon-mediated glucose production.
b. Glucagon stimulates beta cell (insulin) secretion and delta cell (somatostatin) secretion,
which increases hepatic glucose production and opposes hepatic glucose storage.
c. Somatostatin inhibits alpha cell (glucagon) and beta cell (insulin) secretion, which produces
hypoglycemia and inhibition of intestinal glucose absorption. The lowering of blood glucose
levels by somatostatin in diabetic patients probably is due both to inhibition of glucagon
secretion and to reduced intestinal absorption of glucose.
d. Pancreatic polypeptide inhibits insulin and somatostatin secretion via a direct pancreatic
effect.
B. Biosynthetic organization of the beta cell

1. Human proinsulin is a single-chain polypeptide of 86 amino acid residues, with a molecular


weight of approximately 9000 daltons.
a. Intracellular proteolytic cleavage of proinsulin forms insulin and C-peptide.

Figure 7-14. The paracrine system of the pancreatic islet


cells. The pattern of islet cell hormone secretion represents an integrated response by all of the islet cells to
humoral, neural, and paracrine regulation. Plus signs indi-

cate stimulation, and minus signs indicate inhibition. (Reprinted from Tepperman J: Endocrine function of the pancreas. In Metabolic and Endocrine Physiology, 4th
edition. Chicago, Year Book, 1980, p 233.)

b. The conversion of proinsulin to insulin is not fully completed, and about 5% of the secretory
product of the beta cell is proinsulin.
c. The biologic activity of proinsulin is about 5%-10% that of insulin.
d. The plasma half-life of proinsulin is 15 minutes.
2. C-peptide is the connecting peptide remaining after cleavage of proinsulin to insulin. In humans,
it consists of 31 amino acid residues.
a. Beta cell secretory products consist of equimolar amounts of insulin and C-peptide; therefore,
circulating C-peptide concentrations reflect beta cell activity.
b. The normal fasting concentration of C-peptide in peripheral blood is approximately 1.03.5 ng/ml.
c. C-peptide has no detectable biologic activity.
d. The plasma half-life of C-peptide is 30 minutes.
3. Insulin is stored in the beta cell granules as a crystalline hexamer complex with two atoms of
zinc per hexamer. In plasma, insulin is transported as a monomer.
a. Insulin has a molecular weight of about 6000 daltons and contains 51 amino acid residues.
b. Insulin secretion requires the presence of extracellular Ca2+. Inside the beta cell, Ca2+ binds
to a Ca 2 +-binding protein called calmodulin.
c. The plasma half-life of insulin is 5 minutes.

C. Control of insulin secretion


1. Carbohydrates
a. Monosaccharides that can be metabolized (e.g., hexose, triose) are more potent stimuli of
insulin secretion than carbohydrates that cannot be metabolized (e.g., mannose, 2-deoxy-ogiucose).
b. rhe principal stimulus for insulin release is glucose. As the blood glucose level rises above
4.5 mmol/L (80 mg/dll, it stimulates the release and synthesis of insulin.
c. Substances that inhibit glucose metabolism (e.g., 2-deoxy-o-glucose, o-mannoheptulose)
interfere with insulin secretion.
d. The reduction of glucose to sorbitol may contribute to insulin secretion.
e. Glucose also stimulates somatostatin release.
2. Gastrointestinal hormones
a. The plasma concentration of insulin is higher after oral administration of glucose than after
it has been administered intravenously, even though the arterial blood glucose concentration
remains lower. This augmented release of insulin following an oral glucose dose is due to
the secretion of gastrointestinal hormones, including:
(1) Gastric inhibitory peptide (GIP), which appears to be the principal gastrointestinal potentiator of insulin release
(2) Gastrin
(3) Secretin
(4) CCK
b. Gastrointestinal hormones also augment somatostatin release.
3. Amino acids
a. Amino acids vary in their ability to stimulate beta cells. Among the essential amino acids,
in decreasing order of effectiveness, are arginine, lysine, and phenylalanine.
b. The stimulation of insulin secretion by oral administration of amino acids exceeds that of
intravenously administered amino acids. Protein-stimulated secretion of CCK, gastrin, or
both may mediate this effect.
c. The analogs of leucine and arginine that cannot be metabolized also stimulate insulin
secretion.
4. Fatty acids and ketone bodies are not known to have an important role in the regulation of
insulin secretion in humans. The ingestion of medium-chain triglycerides causes a small increment in insulin levels.
5. Islet hormones. Glucagon stimulates insulin secretion and somatostatin inhibits insulin secretion. Somatostatin inhibits gastrin and secretin secretion, glucose absorption, and gastrointestinal
motility (Table 7-11).
6. Other hormones
a. GH induces an elevation in basal insulin levels, which precedes a change in blood glucose
levels, suggesting a direct beta-cytotropic effect.

Table 7-11. Physiologic Effects of Somatostatin


Action

Site
Anterior pituitary

Inhibits secretion of growth hormone and thyrotropin

Pancreas

Inhibits secretion of insulin, glucagon, and pancreatic polypeptide

Gastrointestinal tract

Inhibits secretion of gut hormones (gastrin, secretin, VIP, cholecystokinin), gastric acid, and pepsin; decreases blood flow, motility,
and carbohydrate absorption; increases water and electrolyte absorption

VIP = vasoactive intestinal peptide.


b. Hyperinsulinemia also has been observed with exogenous and endogenous increments of
corticosteroids, estrogens, progestogens, and PTH. Since blood glucose concentrations are
not reduced with these hormones, it is inferred that these hormones have an anti-insulin
effect.
7. Obesity. Hyperinsulinemia is observed in obese patients. An increase in body weight in the
absence of a disproportionate increase in body fat does not affect insulin levels.
8. Ions. Both K+ and Ca2+ are necessary for normal insulin and glucagon responses to glucose.
Therefore, hypokalemia leads to glucose intolerance.
9. Cyclic nucleotides. cAMP is a releaser of insulin.
D. Physiologic actions of insulin (Figure 7-15). Target cells with insulin receptors have been demonstrated in liver, muscle, adipose tissue, lymphocytes, monocytes, and granulocytes.
1. Carbohydrate metabolism

a. Liver
(1) The liver is freely permeable to glucose, and glucose transport can occur without insulin
by simple diffusion.
(a) Insulin acts on the liver to promote glucose uptake and to inhibit enzymatic processes
involved in glucose production and release (glycogenolysis).
(b) Because the hepatocyte is permeable to glucose, uptake of glucose in the liver is
not rate-limiting.
(2) A control point in glucose metabolism occurs when metabolism is initiated by the
phosphorylation of glucose to glucose-6-phosphate, which is catalyzed by hexokinase
and glucokinase.

Metabolic
fuels

Carbohydrate

Adipose cell

-+ Glucokinase
-+ Glycogen synthetase

.. Glucose transport
.. Glycerol synthesis

Phosphorylase

t Gluconeogenesis

Fat

.. lipogenesis

Protein

t Proteolysis

+T';.'ye,,;.,,'
.. Fatty acid
t Lipolysis

Muscle
.. Glucose transport
.. Glycolysis
.. Glycogen synthesis

'00'0 " )

Sy-;;:S
.. Amino acid uptake
.. Protein synthesis

Figure 7-15. The major target sites and metabolic actions of insulin. Insulin is primarily involved in the regulation ot
metabolic processes, the principal manifestation of which is the control of plasma glucose concentration. (Reprinted
from Felig P: Pathophysiology of diabetes mellitus. Med Clin North Am 55:821-834, 1971.)

(a) Hexokinase is saturated at normal plasma glucose concentrations and is not regulated by insulin.
(b) Glucokinase is only half-saturated at blood glucose concentrations between 90
and 100 mg/dl (5-10 mmol/U. Thus, the activity of this enzyme is insulin- and
glucose-dependent.
(3) The phosphorylation of fructose-6-phosphate by phosphofructokinase is enhanced by
insulin. A decrease in phosphofructokinase activity favors the reversal of glycolysis.
(4) Insulin diminishes hepatic glucose output by activating glycogen synthetase and by
inhibiting gluconeogenesis. The key intermediary reaction in gluconeogenesis is between
pyruvate and. phosphoenolpyruvate, which requires the enzymes pyruvate carboxylase
and phosphoenolpyruvate carboxykinase. The latter enzyme is inhibited in the presence
of glucose and insulin.
'
b. Muscle. The insulin-dependent facilitated diffusion mechanism for glucose is found in
skeletal and cardiac muscle.
(1) Glucose transport across muscle cell membranes requires insulin.
(2) Insulin activates glycogen synthetase and phosphofructokinase, which cause glycogen
synthesis and glucose utilization, respectively.
(3) It should be emphasized that glucose uptake in exercising muscle is not dependent on
increased insulin secretion. In resting muscle, glucose is a relatively unimportant fuel,
with the oxidation of fatty acids supplying most of the energy.
c. Adipose tissue. The insulin-dependent facilitated diffusion mechanism for glucose is
found also in adipose tissue.
(1) Insulin acts primarily to stimulate glucose transport.
(2) It activates glycogen synthetase and phosphofructokinase.
(3) The major end products of glucose metabolism in fat cells are fatty acids and a-glycerophosphate. The fat cell depends on glucose as a precursor of a-glycerophosphate, which
is important in fat storage because it esterifies with fatty acids to form triglycerides.

2. Fat metabolism. Insulin is a lipogenic as well as an anti lipolytic hormone.


a. Liver
(1) When insulin and carbohydrate are available, the human liver is quantitatively a more
important site of fat synthesis than is adipose tissue.
(2) In the absence of insulin, the liver does not actively synthesize fatty acids, but it is capable
of esterifying fatty acids with glycerol, which is phosphorylated by glycerokinase.
(a) Glycerol must be phosphorylated before it can be used in the synthesis of fat.
(b) In the absence of glycolytic breakdown of glucose to a-glycerophosphate, glycerokinase permits the esterification of fatty acids.
(3) In the absence of insulin, there is an increase in fat oxidation and the production of
ketone bodies. Insulin exerts a potent antiketogenic effect.
(4) Insulin promotes the synthesis and release of lipotropin lipase, which is an extracellular
enzyme that hydrolyzes both chylomicron and very-low-density lipoprotein (VLDU
triglyceride.
(a) Lipoprotein lipase catalyzes the hydrolysis of circulating lipoprotein triglyceride to
fatty acids and glycerol.
(b) Lipoprotein lipase is the key enzyme in the removal of lipoprotein triglyceride and
thereby is important in the formation of both light and heavy lipoprotein (LDL and
HDU.
(c) Lipoprotein lipase is active at the luminal surface of the capillary endothelial cell
and under normal conditions is completely absent from the circulation.
(d) The highest lipoprotein lipase activity is found in the heart, but its distribution
includes adipose tissue, lactating mammary gland (and milk), lung, skeletal muscle,
aorta, corpus luteum, brain, and placenta.
b. Adipose tissue
(1) Insulin deficiency also decreases the formation of fatty acids in adipose tissue.
(2) The major effect of insulin-stimulated glucose uptake in human fat cells is to provide
a-glycerophosphate for esterification of free fatty acids. The absence of a-glycerophosphate formation from glycolysis during insulin deficiency prevents the esterification of
free fatty acids, which are constantly released from triglycerides in the adipocytes.
(3) The lipolytic effect in the absence of insulin is due to an increase in the hormone-sensitive
lipase known as triglyceride lipase, the activity of which is normally inhibited by insulin.

3. Amino acid and protein metabolism. Insulin is an important protein anabolic hormone, and
it is necessary for the assimilation of a protein meal. The protein anabolic effect of insulin is
not dependent on increased glucose transport.
a. In diabetic patients, the muscle uptake of amino acids is reduced and elevated postprandial
blood levels are observed.
b. During severe insulin deficiency, hyperaminoacidemia involving branched-chain amino
acids (i.e., valine, leucine, and isoleucine) is present.
c. Insulin increases uptake of most amino acids into muscle and increases the incorporation
of amino acids into protein.
d. Insulin increases body protein stores by four mechanisms:
(1) Increased tissue uptake of amino acids
(2) Increased protein synthesis
(3) Decreased protein catabolism
(4) Decreased oxidation of amino acids

4. Electrolyte metabolism
a. Insulin lowers serum K+ concentration. This hypokalemic action of insulin is due to stimulation of K+ uptake by muscle and hepatic tissue.
b. Diabetic patients have a proclivity toward developing hyperkalemia in the absence of
acidosis.
c. Insulin has an anti natriuretic effect.

5. Membrane polarization
a. Insulin decreases membrane permeability to both Na+ and K+, but it decreases Na+ permeability to a greater extent, causing hyperpolarization of mammalian muscle.
b. The membrane hyperpolarization produced by insulin is the cause of the net shift of K+
from the extracellular to the intracellular space, and not the result of the shift.

6. Integration of insulin action: a summary (Figure 7-16)


a. Insulin is a very effective hypoglycemic hormone for two major reasons.
(1) It promotes both hepatic and muscle glycogen deposition.
(2) It enhances glucose utilization (glycolysis).
b. In terms of glucose transport, the major insulin-independent tissues are brain, erythrocytes,
liver, and epithelial cells of the kidney and intestine.
c. Insulin is the primary anabolic hormone in the body for the following reasons.
(1) Inhibition of hepatic gluconeogenesis decreases the hepatic requirement for amino
acids.
(2) The protein anabolic effect of insulin reduces the output of amino acids from muscle,
thereby decreasing the availability of glucogenic amino acids for gluconeogenesis.
(3) Glucose uptake by muscle is stimulated, providing an energy source to spare fatty acids,
the release of which is inhibited by the antilipolytic action of insulin.
(4) Fat accumulation is enhanced by increased hepatic lipogenesis.
(5) The antilipolytic action of insulin (inhibition of hepatic oxidation of fatty acids) is due to
the formation of a-glycerophosphate from glucose in the fat cell.
(6) The antilipolytic action of insulin at the level of the adipose cell reinforces the insulinmediated inhibition of hepatic ketogenesis and gluconeogenesis by depriving the liver
of precursor substrates for ketogenesis and an energy source (fatty acids) and cofactors
(acetyl-CoA) necessary for gluconeogenesis.
E. Control of glucagon secretion

1. Metabolic fuels
a. Hypoglycemia stimulates and hyperglycemia inhibits glucagon secretion.
b. Amino acids (e.g., arginine, alanine) also are stimuli for glucagon release.

_-Fatty acids---....;..
~Triglyceride

,'" t

Ehergy...

\.

Glucose----...,...
- Glycogen

.... --.Amino acids --....;...


~ Protein

Figure 7-16. Insulin exerts integrated and synergistic actions in the promotion of the storage of body fuels. It
enhances the storage of fat and protein, and it promotes
both the storage and utilization of carbohydrates. Solid
arrows denote stimulation, and dashed arrows indicate
inhibition. (Reprinted from Felig P: Disorders of carbohydrate metabolism. In Metabolic Control and Dis~ase, 8th
edition. Edited by Bondy PK and Rosenberg LE. Philadelphia, WB Saunders, 1980, p 294.)

c. Decreasing circulatory levels of fatty acids are associated with glucagon release.
2. Gastrointestinal hormones
a. CCK, gastrin, secretin, and GIP stimulate glucagon secretion.
b. The potentiation of glucagon secretion by the ingestion of a protein meal is probably
mediated via CCK secretion.
3. Fatty acids inhibit glucagon release.
F. Physiologic actions of glucagon. The major site of action of glucagon is the liver.
1. Carbohydrate metabolism
a. Glucagon has a hyperglycemic action, resulting primarily from stimulation of hepatic glycogenolysis. It should be emphasized, however, that the hyperglycemic effect of glucagon
does not involve inhibition of the peripheral utilization of glucose.
b. Glucagon is an important gluconeogenic hormone.
c. The hyperglycemic action of epinephrine is amplified by its stimulation of glucagon secretion
and its inhibition of insulin secretion.
d. Suppression of glucagon secretion by glucose is not essential for normal glucose tolerance
as long as insulin is available.
2. Fat metabolism
a. Glucagon is a lipolytic hormone because of its activation of glucagon-sensitive lipase (triglyceride lipase) in adipose tissue by cAMP.
b. Glucagon causes an elevation in the plasma level of fatty acids and glycerol.
(1) Glycerol is utilized as a glyconeogenic substrate in the liver.
(2) The oxidation of fatty acids as an energy substrate accounts for the glucose-sparing
effect of glucagon.
(3) Glucagon is essential for the ketogenesis brought about by the oxidation of fatty acids.
In the absence of insulin, glucagon can accelerate ketogenesis, which leads to metabolic
acidosis.
3. Protein metabolism
a. Glucagon has a net proteolytic effect in the liver.
b. This peptide is gluconeogenic, an effect that leads to increased amino acid oxidation and
urea formation.
c. In addition to its protein catabolic effect, glucagon has an antianabolic effect-inhibition of
protein synthesis.

XIII. THYROID GLAND


A. Histology and function
1. Thyroid components. The functional unit of the thyroid gland is the follicle (acinus) surrounded by a rich capillary plexus.
a. The follicular (acinar) epithelium consists of a single layer of cuboidal cells.
(1) The cell height of the follicular epithelium varies with the degree of stimulation of TSH.
(2) The glandular epithelium varies with the degree of stimulation, becoming columnar
when active and flat when inactive.
b. The lumen of the follicle is filled with a clear, amber proteinaceous fluid called colloid,
which is the major constituent of the thyroid mass.
c. Microvilli extend into the colloid from the apical (adluminal) border, which is the site of
the iodination reaction. The initial phase of thyroid hormone secretion (i.e., resorption of
the colloid by endocytosis) also occurs at the apical border.
d. The parafollicular (C) cells, which secrete calcitonin, do not border on the follicular
lumen.
2. Thyroid functions
a. Hormone secretion. The thyroid gland secretes two hormones, which are iodothyronines
and, therefore, derivatives of the amino acid tyrosine.
(1) The major secretory product of the thyroid gland is 3,5,3',5'-tetraiodothyronine (thyroxine), which is abbreviated as T 4 to denote the four iodide atoms. The other thyroid

hormone is 3,5,3'-triiodothyronine, which is abbreviated as T3 . T3 is secreted in small


amounts.
(2) Only these two thyronines have biologic activity.
(a) The molar activity ratio of T3 to T4 is 3-5: 1
(b) The secretory ratio of T4 to T3 is 10-20: 1
(c) The plasma concentration ratio of free T4 to free T3 is 2:1. Most of the T3 in the
plasma is derived from monodeiodination of T4 by the action of monodeiodinase
(5'-deiodinase) found in peripheral tissue.
(3) Reverse 3,3',5'-triiodothyronine (rT 3 ) is a biologically inactive thyronine also formed
by peripheral conversion catalyzed by 5-deiodinase.
b. Related functions. The thyroid cell performs two parallel functions in the synthesis of
thyroid hormone. *
(1) It synthesizes a protein substrate called thyroglobulin.
(a) This glycoprotein serves as a matrix in which thyroid hormone is formed.
(b) Thyroglobulin also is the storage form of thyroid hormone.
(c) Each thyroglobulin molecule contains approximately 120 tyrosyl residues.
(2) The thyroid cell accumulates inorganic iodide from the plasma.

B. Distribution of thyroid iodide


1. Iodide intake
a. In the United States, the daily dietary iodine intake is about 500 ""g.
b. About 1 mg of iodide is required per week (or 150 ""g/day) to maintain euthyroidism.
c. The thyroid gland stores enough thyroid hormone to maintain a euthyroid state for 3 months
without hormone synthesis.

2. Thyroid iodide
a. The thyroid gland contains 5-7 mg of iodide.
(1) Of the total thyroid iodide, 95% is in the extracellular space (i.e., stored in the colloid

as thyroglobulin).
(a) Two-thirds of the total iodide content in the colloid is in the form of biologically

inactive iodotyrosines.
(b) One-third of the colloid iodide content is in the form of biologically active thyronines (i.e., T4 and T3).
(c) The molar storage ratio of T4 to T3 is 9: 1, and the molar storage ratio of iodotyrosines

to iodothyronines is 2:1.
(2) The remaining 5% of the total thyroid iodide is in the intracellular space of the follicular

epithelium.

b. The thyroid gland contains the body's largest iodide pool.


C. Hormone transport: extracellular binding proteins. Extracellular binding proteins for thyroid
hormone are in the plasma, while the storage form of thyroid hormone (thyroglobulin) is in the
follicular lumen (colloid).

1. Thyroxine-binding proteins. Virtually all (99.95%) of T4 is bound to plasma proteins, leaving


about 0.05% unbound (free). This portion of unbound thyroxine represents the biologically
active hormone. Thyroxine is mainly associated with two of the three binding proteins.
a. Thyroxine-binding globulin (TBG) binds about 75% of the plasma T4. In normal individuals,
less than half of the available binding sites on TBG are saturated with T4.
b. Thyroxine-binding prealbumin (TBPA) binds about 15%-20% of the circulating T4.
c. About 9% of the T4 is bound to albumin.

2. Triiodothyronine-binding proteins
a. Almost all (99.5%) of T3 is transported bound to TBG.
b. Very little T3 is bound to albumin, and practically none is bound to TBPA.
c. About 0.5% of the T3 is unbound. The lower affinity of T3 for the plasma binding proteins
and, thus, the higher concentration of unbound T3 contributes to the greater biologic activity
of T3

The term "thyroid hormone" denotes thyroxine (T.) and triiodothyronine (T 3 ).

D. Biosynthesis and release of thyroid hormone. The thyroid gland accumulates or "traps" iodide
by an active transport mechanism that operates against a concentration and an electric gradient.
The normal thyroid iodide-to-plasma iodide concentration ratio is 25-40:1.

1. Synthesis. All of the biosynthetic steps are stimulated by TSH.


a. Iodide uptake (Figure 7-17)
(1) Active iodide uptake occurs at the basal membrane of the follicular cell and is not an
essential step in thyroid hormone synthesis.
(2) Iodide diffuses along an electric gradient into the lumen, where the luminal iodide-tofollicular cell iodide concentration ratio is 5:1.
(3) Radioactive iodide uptake by the thyroid gland is a useful therapeutic index of the
functional status of the thyroid gland. A 24-hour uptake normally ranges between 10%
and 35% of the administered dose.

= Urine
=Thyroid

= Plasma

75

"0
CD

iii

~ 50
.S:

rf.

25

Euthyroid

!I

Ii
___ - - - - - - - - - i'
--. \ """,It
i / \. . _
0_..1

O~-r_r~-,----,_------------~

12

24

Time (h)

~ 75
"0

15

~Cl

Hyperthyroid

50

.S:

'0
E

~ 25

;~---------------------

CD

c...

' ....... _ . _ . _ . _ _ _ . _ - _ . -

O~,-,-~~~=T~~--------T

75

CD

4 6

/\
i \
. "-

Ci5

Hypothyroid

_---

"

~ 50

.......

;<

.S:

'0

~,'"

~ 25

,,"

_---

............

,//

c...

24

12
Time (h)

_-----

...............
'-._._

O~~~~==~========~
2

12
Time (h)

24

Figure 7-17. Radioactive iodine uptake by individuals


on a relatively low-iodine diet. Percentages are plotted
against time after an oral dose of radioactive iodine for
plasma, urine, and the thyroid gland. In hyperthyroidism,
plasma radioactivity falls rapidly, then rises again as a
result of release of labeled T4 and T3 from the thyroid
gland. (Adapted from Ingbar SH, Woeber KA: Textbook
of Endocrinology, 4th edition. Edited by Williams RH.
Philadelphia, WB Saunders, 1968, p 144.)

b. Oxidation of iodide is mediated by a peroxidase and forms active iodide, which may be in
the form of iodinium ion (1+), a free radical of iodine (10 3-), or iodine (12).

c. Iodination of active iodide denotes the addition of iodide to the tyrosyl residues of thyroglobulin. The substrate for iodination is thyroglobulin.
(1) Iodination leads to the formation of the iodotyrosines within the preformed thyroglobulin
molecule rather than in free amino acids that are then incorporated into protein. The
hormonally inactive substrates formed by the iodination of thyroglobulin are called
mono- and diiodotyrosine.
(2) Iodination occurs by the catalytic action of thyroid peroxidase.
d. Coupling (condensation) of iodotyrosines occurs and forms biologically active thyronines
(T 3 and T4).
(1) The iodothyronines are formed at the apical border of the follicular cell and are held in
peptide linkage with thyroglobulin (as are the tyrosines).
(2) T4 synthesis requires the fusion of two diiodotyrosine molecules, and T3 synthesis
requires the condensation of a monoiodotyrosine molecule with a diiodotyrosine molecule.
(3) Thyroid peroxidase also mediates the coupling reaction.

2. Release
a. Secretion begins with endocytosis of the colloid at the apical border. This brings colloid
"droplets" into contact with protease-containing Iysosomes.
b. The release of hormones involves the following reactions:
(1) Hydrolysis of thyroglobulin by the thyroid protease and by peptidases, which liberate
free amino acids
(2) Secretion of iodothyronines into the blood and deiodination of the iodotyrosines, which
form a second iodide pool that can be recycled into hormone synthesis

E. Metabolism and excretion of thyroid hormone


1. T4 is the iodothyronine found in highest concentration in plasma and is the only one that arises
solely by direct secretion from the thyroid gland.
2. Most of the T3 present in plasma is derived from the peripheral conversion of T4 by monodeiodination via 5'-deiodinase.
a. The extrathyroid deiodination of T4 accounts for over 80% of the circulating T3'
b. The liver and kidney deiodinate T4 to form T3.

3. Thyroid hormone is metabolized by deiodination, deamination, and by conjugation with glucuronic acid. The conjugate then is secreted via the bile duct into the intestine.

4. In normal individuals, T4 and T3 are excreted mainly in the feces, with a small amount appearing
in the urine.

F. Control of thyroid function (see Figure 7-4)


1. Hypothalamic-hypophysial-thyroid axis. TSH secretion is influenced by four factors: TRH
secretion from the median eminence, the blood level of unbound T4, the blood level of unbound
T3 generated by the peripheral conversion of T4 to T3, and the peripheral conversion of T4 to
T3 within the pituitary gland.
a. TRH is a tripeptide synthesized by the parvicellular peptidergic neurons in the hypothalamus.
(1) TRH is transported to the median eminence, where it is stored. From there, TRH is
released into the hypophysial portal system and is carried to the anterior lobe of the
pituitary gland.
(2) TRH stimulates some of the basophils (thyrotrophs) to secrete TSH.
b. TSH stimulates the thyroid follicle to secrete thyroid hormone, most of which is bound to
plasma protein carriers.
(1) TSH stimulates the series of chemical reactions that lead to the synthesis of the iodothyronines.
(2) It is the circulating free T3 and T4 concentrations that influence (regulate) TSH release
by exerting a negative feedback effect at the level of the anterior lobe and probably at
the level of the hypothalamus. Both free T3 and free T4 are effective inhibitors of TSH

secretion when their plasma concentrations are increased.


(3) The pituitary gland also converts T4 to T3 by monodeiodinase, and this intrapituitary T3

plays a major role in the negative feedback that occurs at the pituitary level.

c. Other regulators
(1) Estrogens enhance TSH secretion.
(2) Large doses of iodide inhibit thyroid hormone release and, thereby, cause decreases in
serum T4 and T3 concentrations and an increase in TSH secretion.
(3) Somatostatin inhibits TSH secretion and the response to TRH.
(4) Dihydroxyphenylethylamine (dopamine), dopa, and bromocriptine decrease the basal
secretion of TSH.
2. Thyroid autoregulation
a. Thyroid function also is regulated by an intrinsic control system that maintains the constancy
of thyroid hormone stores.
(1) The high concentrations of intrathyroidal inorganic iodide lead to the inhibition of
thyroid release.
(2) High concentrations of organic iodide (thyroid hormone) lead to a decrease in iodide
uptake.
b. Both of these effects reduce the fluctuation in thyroid hormone secretion when an acute
change occurs in the availability of a requisite substrate (e.g., iodide).

3. Goiter
a. Any enlargement of the thyroid gland is called a goiter, and antithyroid substances that
cause thyroid enlargement are called goitrogens.
(1) Goitrogens are substances that block the synthesis of thyroid hormone.
(2) A goiter does not define the functional state of the thyroid gland.
b. If the goitrogen reduces thyroid hormone synthesis to subn'ormal levels, TSH secretion is
enhanced.
Co Goitrogens lead to the increased synthesis of endogenous TSH, which is responsible for the
formation of a hypertropic thyroid gland (goiter).
d. Goitrogenic agents include:
(1) Perchlorate, thiocyanate, and pertechnetate, which are monovalent anions that block
iodide trapping
(2) Thionamides (propylthiouracil and methimazole), which block the coupling of iodotyrosines
(3) Iodide deficiency
(4) Excess iodide
G. Physiologic effects of thyroid hormone
1. Thyroid hormone increases the basal metabolic rate (BMR) of most cells in the body. The
normal BMR for adult euthyroid males is 35-40 kcal/m 2 body surface/hr. Normal BMR is 6%10% lower in euthyroid females.
a. Exceptions to this effect occur in the gonads, brain, lymph nodes, thymus, lung, spleen,
dermis, and some accessory sex organs.
b. A correlate of the increase in BMR is an increase in the size and the number of mitochondria
together with an increase in the enzymes that regulate oxidative phosphorylation.
c. The increase in BMR also is associated with an increase in Na+-K+-ATPase (Na+ pump)
activity. The fluxes of Na+ (efflux) and K+ (influx) are estimated to require 10%-30% of
the total energy consumed by cells.
d. The increase in BMR accounts for the thermogenic effect of thyroid hormone.
2. Thyroid hormone is essential for normal bone growth and maturation as well as for the
maturation of neurologic tissue, especially the brain.
a. In hypothyroidism, there is a marked decrease in the myelination and arborization of neurons
in the brain.
b. If hypothyroidism is untreated, mental retardation occurs.

3. Thyroid hormone is necessary for normal lactation.


H. Metabolic effects of thyroid hormone
1. Carbohydrate metabolism
a. In physiologic amounts, thyroid hormone potentiates the action of insulin and promotes
glycogenesis and glucose utilization.
b. In pharmacologic amounts, thyroid hormone is a hyperglycemic agent.

(1) Thyroid hormone potentiates the glycogenolytic effect of epinephrine, causing glycogen

depletion.
(2) Thyroid hormone is gluconeogenic in that it increases the availability of precursors
(lactate and glycerol).
(3) In large doses, thyroid hormone promotes intestinal glucose absorption.

2. Protein metabolism
a. In physiologic amounts, thyroid hormone has a potent protein anabolic effect.
b. In large doses, thyroid hormone has a protein catabolic effect.

3. Fat metabolism
a. Thyroid hormone stimulates all aspects of lipid metabolism, including synthesis, mobilization, and utilization. On a net basis, the lipolytic effect is greater than the lipogenic effect.
b. There is a general inverse relationship between thyroid hormone levels and plasma lipids.
(1) Elevated thyroid hormone levels are associated with decreases in blood triglycerides,
phospholipids, and cholesterol.
(2) High levels of thyroid hormone also are associated with increases in plasma free fatty
acids and glycerol.
4. Vitamin metabolism. The metabolism of fat-soluble vitamins is affected by thyroid hormone.
For example, thyroid hormone is required for the synthesis of vitamin A from carotene and the
conversion of vitamin A to retinene.
a. In hypothyroid states, the serum carotene is elevated, and the skin becomes yellow.
b. This skin condition differs from that observed in jaundice in that the sclera of the eye is not
yellow.

XIV. PARATHYROID HORMONE, CALCITONIN, AND VITAMIN D

A. Role of Ca2+ in physiologic processes


1. Hemostasis. Ca2+ is necessary for the activation of clotting enzymes in plasma.
2. Ca2+ controls membrane excitation, and Ca 2 + influx occurs during the excitatory process of
nerve and muscle.
a. Excitable membranes contain specific Ca2+ channels.
b. Ca H entry does not require an active transport process, since the concentration gradient
across the membrane is larger for Ca 2 + than for any other ion.
(1) Ca H concentration ([Ca 2 +]) in the intracellular fluid (lCF) is about 10- 7 moll L.
(2) [Ca H ] in the extracellular fluid (ECF) is about 10- 1 mol/L (the actual value is 2.5 x
10- 1 mol/L).
(3) The [Ca 2 +] gradient from outside to inside the cell is on the order of 10,000 to 1!
3. Ca H

is bound to cell surfaces and has a role in the stabilization of the membrane and

intercellular adhesion.
4. CaH is necessary for muscle contraction (excitation-contraction coupling).
5. Ca H is essential in all excitation-secretion processes, such as the release of hormone by
endocrine cells and the release of other products by exocrine cells. It is also essential for

neurotransmitter release.
6. CaH is necessary for the production of milk and the formation of bone and teeth.

B. Ca2+ distribution
1. Skeletal storage. More than 99% of the total body CaH is stored in the skeleton.
a. The skeleton of a 70-kg individual contains about 1000 g of CaH compared to about 1 g
in the extracellular pool.
b. The skeleton also serves as a storage depot for phosphorus and contains about 80% of the
total body phosphorus.
c. Bone serves as a third-line defense in acid-base regulation by virtue of its CO/-, HCO l - ,
and PO/- content.
2. Plasma. The plasma concentration of total (ionized and nonionized) CaH is about 10 mg/dl,
which is equivalent to 5 mEq/L or 2.5 mmol/L.

a. Ca H is present in the plasma as:


(1) Ionized or free (45%)
(2) Complexed with HPO/-, HC03-, or citrate ion (10%)
(3) Bound to protein [primarily to albumin] (45%)
b. The sum of the ionized and complexed Ca H constitutes the diffusible fraction (55%) of
Ca H . The protein-bound form constitutes the nondiffusible fraction (45%).
c. PTH, calcitonin, and vitamin D regulate the serum-ionized Ca H concentration.

3. Ca2+ pools. Total body CaH can be conceptualized as two major "pools."
a. The larger Ca H pool, which contains over 99% of the total Ca H , consists of stable (mature)
bone. This represents the Ca H pool that is not readily exchangeable, and it is not available
for rapid mobilization.
b. The smaller Ca H pool, which contains less than 1% of the total body Ca H , consists of
labile (young) bone. This Ca H pool is readily exchangeable because it is in physicochemical
equilibrium with the ECF. The pool consists of calcium phosphate salts and provides an
immediate reserve for sudden decreases in blood [Ca H ].

C. Bone chemistry
1. Bone Ca H is found in the form of hydroxyapatite crystals. The empirical chemical formula
for this substance is CalO (P0 4 )6(OH)2 or [(Ca 3P0 4 )2]3 Ca(OH)2. Fluoride ion can replace the
OH- group and form fluoroapatite, or [(Ca 3P04 )2]3 . CaF 2.
a. The calcium:phosphorus ratio in bone is about 1.7:1.
b. A large surface area is provided by the microcrystalline structure of bone; it is estimated to
be 100 acres in humans!

2. Dry, fat-free bone consists of two-thirds mineral (inorganic) and one-third organic matrix.
a. Over 90% of the organic matrix is collagen.
b. The inorganic crystalline structure of bone imparts to it an elastic modulus similar to that of
concrete.
D. Bone development. Bone is both a tissue and an organ. It consists of cells and an extracellular
matrix containing organic and inorganic components. In its early development, bone exists as
osteoid, an organic, un mineralized matrix surrounding the bone cells that deposited it.
1. Mesoderm is the embryonic germ layer that gives rise to cartilage, bone, and muscle.

2. Neural crest cells also can differentiate as bone cells, including:


a. Cells that deposit dentin in teeth
b. Cartilage and bone of the head
E. Ca H regulation: overview. Ca H regulation involves three tissues (bone, intestine, and kidney),
three hormones (PTH, calcitonin, and activated vitamin D 3), and three cell types (osteoblasts,
osteocytes, and osteoclastsl.
1. Hormonal control of Ca 2 + metabolism. The pituitary gland does not playa major role in
regulating the cells that produce PTH, calcitonin, and activated vitamin D 3.
a. PTH is a polypeptide containing 84 amino acid residues. PTH is secreted by chief cells of
the four parathyroid glands.
(1) PTH is the hypercalcemic hormone of the body; it exerts its effects on the bone,
intestine, and kidney.
(2) PTH regulates only the plasma [Ca H ]. An inverse linear relationship exists between
plasma [Ca H ] and PTH secretion (Figure 7-18).
(a) When plasma [Ca H ] falls, PTH secretion increases.
(b) As plasma [Ca 2+] increases, PTH secretion decreases.
b. Calcitonin is a 32 amino acid residue polypeptide secreted by the parafollicular (C) cells of
the thyroid gland.
(1) Calcitonin is the hypocalcemic hormone of the body; it exerts a biologic effect on the
bone, intestine, and kidney.
(2) A positive linear relationship exists between plasma [Ca 2+] and calcitonin secretion (see
Figure 7-18).
(a) As plasma [Ca H ] increases, calcitonin secretion increases.
(b) When plasma [Ca H ] decreases, calcitonin secretion decreases.
(3) Calcitonin release is stimulated by pentagastrin.

c:

c:

'E

CD

'E

CD

c:

c:

J:

'c

c:

.s

Ii:

'0

2.5

5.0

7.5

10

Plasma total Ca2 concentration (mEq/L)

15

Figure 7-18. Relationship between


plasma total Ca2+ concentration and
blood levels of PTH and calcitonin.
(Reprinted from Tepperman J; Hormonal regulation of calcium homeostasis. In Metabolic and Endocrine
Physiology, 4th edition. Chicago,
Year Book, 1980, p 297.)

c. Vitamin D] (cholecalciferol) is a secosteroid containing 27 carbon atoms, which makes it


the largest steroid hormone. The term "vitamin D" denotes both vitamins D2 and D3. In
humans, the storage, transport, metabolism, and potency of vitamin D2 and vitamin D3 are
identical.
(1) Active metabolites. Only the active metabolites of vitamin D exert biologic activity.
(a) Calcidiol (25-hydroxyvitamin D 3 ; 25-hydroxycholecalciferol) is the major blood
form of vitamin D. This active metabolite is two to five times more effective than
vitamin D3 in preventing rickets.
(b) Calcitriol (1 ,25-dihydroxyvitamin D 3 ; 1,25-dihydroxycholecalciferol) is another active metabolite of vitamin D. On a molar basis, it is 100 times more potent than
calcidiol.
(2) Synthesis of active vitamin D]
(a) In the epidermis, the previtamin 7-dehydrocholesterol is transformed into the lipidsoluble vitamin D 3 , by nonenzymatic photoactivation upon exposure to the sun's
ultraviolet rays. Photoactivation converts the 4-ring sterol into a 3-ring sterol.
(i) Exposure of the skin to sunlight for 15-20 min/day or the irradiation of food has
an antirachitic effect.
(ii) The plant sterol, ergosterol, is transformed into vitamin D2 (ergocalciferol) by
irradiation and has been the main source of vitamin D that is added to foods
(e.g., milk).
(b) In the liver, vitamin D3 is converted to calcidiol by 25-hydroxylase.
(c) In cells of the proximal convoluted tubule, calcidiol is converted to calcitriol by the
action of 1a-hydroxylase. The activity of this enzyme is enhanced by PTH.
2. Cellular aspects of bone metabolism (Figure 7-19)
a. Osteoblasts are highly differentiated cells that are non mitotic in their differentiated state.
They are the bone-forming cells and are located on the bone-forming surface.
(1) Osteoblasts synthesize and secrete collagen.
(2) They contain abundant alkaline phosphatase activity.
(3) They are derived from bone marrow mesenchyme.
b. Osteocytes are osteoblasts that have become buried in bone matrix and are the most
numerous of the bone cells in mature bone.
(1) Each cell is surrounded by its own lacuna, but an extensive canalicular system connects
osteocytes and surface osteoblasts, forming a functional syncytium.
(2) In the osteocytic form, these cells no longer synthesize collagen.
(3) Osteocytes have an osteolytic activity, which is stimulated by PTH. The "osteocytic
osteolysis" in the bone matrix provides for the rapid movement of Ca2+ from bone into
the ECF space.
c. Osteoclasts are large, multinucleated cells containing numerous Iysosomes. They mediate
bone resorption at bone surfaces.
(1) These cells contain acid phosphatase,
(2) Osteoclasts are stimulated by PTH and form significant amounts of lactic and hyaluronic
acids.

Canaliculus-~;;:===~Fr=9~S:;~!~

Osteoblast

--++1---1'1--- Bone

Osteoclast

Figure 7-19. Anatomic relationships


among the three types of bone cells.
The canalicular system provides the
structure for a functional syncytium
between the osteocytes and osteoblasts. These intercellular connections between these two cell types are
disrupted at sites where osteoblasts
are found. (Reprinted from Avioli LV,
Raisz LG: Bone metabolism and disease. In Metabolic Control and Disease, 8th edition. Philadelphia, WB
Saunders, 1980, p 1715.)

(3) Osteoclasts might cause bone dissolution via an increased local concentration of H+,
which solubilizes bone mineral and increases the activity of enzymes that degrade
matrix.
(4) They are derived from circulating monocytes.
F. Physiologic actions of PTH (Figure 7-20)
1. Osseous tissue
a. PTH action on bone is increased mobilization of Ca2+ and phosphate (i.e., bone dissolution)
from the non readily exchangeable Ca2+ pool.
(1) The long-term effects of PTH on bone (i.e., the release of Ca2+ from bone) may be
related to its effects on bone remodeling, which involves bone resorption and accretion.
(2) PTH also is known to stimulate bone synthesis.
(3) The effects of PTH on osteogenesis can be both anabolic and catabolic in terms of
collagen metabolism.
b. PTH has three important effects on bone that account for its overall osteolytic activity.
(1) It stimulates osteoclastic and osteocytic activity.
(2) It stimulates the fusion of progenitor cells to form the multinucleated osteoclastic cells.
(3) It causes a transient suppression of osteoblastic activity.
c. Bone forms from cartilage, which serves as a template for cortical bone by periosteal
apposition and for trabecular bone by endochondral ossification.
Decreased plasma [Ca2+]

Ca2+ reabsorption

Ca2 + reabsorption

H2 PO.- excretion

H PO!- reabsorption

Ca2 + absorption
PO 4 3- absorption

Figure 7-20. Physiologic actions of


PTH and calcitriol on major target
organs.

(1) Some bones, particularly those in the skull, are formed without a cartilage anlage by
intramembranous bone formation.
(2) In adults, hematopoietic tissue is more abundant in trabecular bone.
d. cAMP is a mediator of bone resorption since PTH.stimulates adenyl cyclase in bone cells.
e. With the dissolution of stable bone, hydroxyproline is excreted in the urine. This forms the
basis for assessing collagen metabolism and thereby the relative rate of bone resorption.
2. Intestinal tissue
a. Ca2+ and phosphate are absorbed in the intestine by both active and passive transport, but
most of the intestinal absorption of Ca2+ occurs via facilitated diffusion.
b. PTH alone does not directly affect the intestinal absorption of Ca2+. PTH and calcitriol act
synergistically to absorb Ca2+ and phosphate.
(1) Intestinal absorption of Ca2+ does reflect parathyroid status, in that hypoparathyroid
states are associated with low absorption and hyperparathyroid states are associated
with high absorption.
(2) The increased intestinal absorption of Ca2+ promoted by PTH is mediated indirectly
through the increased synthesis of calcitriol.
(3) Calcitriol acts on the intestine to promote the transport of Ca2+ and phosphate.
3. Renal tissue
a. PTH increases the renal threshold for Ca2+ by promoting the active reabsorption of Ca2+
by the distal nephron. PTH inhibits the proximal tubular reabsorption of Ca2+. Thus, PTH
increases the tubular maximum (Tm) for Ca2+.
b. PTH inhibits phosphate reabsorption in the proximal tubules [i.e., it lowers the renal threshold for HP04-, which leads to a phosphate diuresis (phosphaturia)]. Thus, PTH decreases
the Tm for phosphate.
c. Both increased PTH secretion and phosphate depletion stimulate the formation of calcitriol
via the activation of 1a-hydroxylase.
d. The phosphaturic effect of PTH may be mediated by cAMP, because PTH activates adenyl
cyclase in the renal cortex.
e. PTH also increases the urinary excretion of Na+, K+, and HC0 3 - and decreases the
excretion of NH4 + and H+. These effects account for the metabolic acidosis that occurs in
hyperparathyroid states.

4. Summary. The unopposed effects of PTH on bone, intestine, and kidney include:
a. Hypercalcemia
b. Hypophosphatemia
c. Hypocalciuria, initially due to increased Ca2+ reabsorption (however, in chronic hyperparathyroid states, the hypercalcemia exceeds the renal threshold for Ca2+, and hypercalciuria
is observed)
d. Hyperphosphaturia
G. Physiologic actions of calcitonin
1. Osseous tissue
a. Calcitonin inhibits osteoclastic activity, and the anti hypercalcemic effect of calcitonin is
due principally to the direct inhibition of bone resorption. This effect is not dependent on a
functioning kidney, intestine, or parathyroid gland.
b. Calcitonin also diminishes the osteolytic activity of osteoclasts and osteocytes.
c. Calcitonin activity is associated with an increase in alkaline phosphatase synthesis from the
osteoblasts.
2. Intestinal tissue
a. Calcitonin inhibits gastric motility and gastrin secretion; however, it stimulates intestinal
secretion.
b. Calcitonin inhibits the intestinal (jejunal) absorption of Ca2+ and PO/-.
3. Renal tissue
a. Calcitonin promotes the urinary excretion of phosphate, Ca2+, and Na + .
b. It also inhibits renal 1a-hydroxylase activity, which leads to a decrease in the synthesis of
calcitriol.

H. Physiologic actions of the biologically active calciferols. The actions of calcitriol raise the
plasma Ca2+ in concert with PTH. This vitamin hormone acts directly on the bone, small intestine,
and kidney.

1. Osseous tissue
a. Calcitriol, together with PTH, increases the mobilization of Ca2+ and phosphate from the
bone.
(1) Paradoxically, by raising serum Ca2+ and HPO/- levels, it also fosters bone deposition.
(2) This effect of calcitriol on Ca2+ and HPO/- mobilization is not observed with vitamin
0 3 (cholecalciferol) or vitamin 02 (ergocalciferol).
b. Ca2+-binding protein in bone can be activated by PTH and calcitriol.
c. The antirachitic action of vitamin 0 has traditionally been measured by an increase in bone
formation following vitamin 0 administration to vitamin O--depleted rats.
(1) It appears that the direct effect of the calciferols on bone is resorption.
(2) The antirachitic influence of the calciferols appears to be due to an indirect effect on
bone through the direct stimulating effect of the calciferols on the intestinal absorption
of Ca2+ and phosphate.
d. In summary, calcitriol acts synergistically with PTH to cause bone dissolution through the
proliferation of osteoclasts. In short, increased osteoclastic activity by PTH requires calcitriol.

2. Intestinal tissue
a. Calcitriol is the principal factor in increased Ca2+ absorption in intestinal tissue.
(1) Calcitriol is the mediator hormone for the intestinal actions of PTH and calcitonin.
(2) In turn, the action of calcitriol is mediated by the induction of a Ca 2 +-binding protein.
b. Calcitriol causes a lesser increase in intestinal HPO/- absorption.

3. Renal tissue
a. Calcitriol promotes the distal tubular reabsorption of Ca2+ .
b. It promotes proximal tubular reabsorption of HPO/-.
c. The renal effects of PTH and calcitriol are similar in that both promote Ca2+ reabsorption;
however, the renal effects of these two hormones on phosphate reabsorption are different.
(1) PTH promotes phosphate diuresis.
(2) Calcitriol promotes phosphate reabsorption; however, pharmacologic doses of calcitriol
do have phosphaturic effects.

STUDY QUESTIONS
Directions: Each of the numbered items or incomplete statements in this section is followed by answers
or by completions of the statement. Select the one lettered answer or completion that is best in each
case.
1. Which of the following peptides is synthesized
by neurosecretory neurons?
(A)
(B)
(C)
(D)
(E)

Epinephrine
Norepinephrine
Somatomedin
Somatostatin
Somatotropin

2. Which of the following adrenomedullary enzymes is correctly paired with its substrate?
(A) Phenylethanolamine-N-methyltransferase/
epinephrine
(B) Phenylalanine hydroxylase/tyrosine
(C) Dopa decarboxylase/phenylalanine
(D) Dopamine l3-hydroxylase/dihydroxyphenylethylamine
(E) Tyrosine hydroxylase/ norepinephrine
3. When is the second meiotic division of the developing ovarian follicle completed?
(A) At puberty
(B) Just prior to ovulation
(C) During the follicular phase of the menstrual
cycle
(D) Just after conception
(E) During fetal development
4. In terms of serum concentration, the major
postmenopausal steroid hormone and pituitary
tropic hormone are
(A)
(B)
(C)
(D)
(E)

estradiol and FSH


estradiol and LH
estrone and FSH
estrone and LH
estriol and LH

6. Glucose transport occurs by insulin-dependent


facilitated diffusion in which of the following
tissues?
(A) Cardiac muscle
(B) Intestinal epithelium
(C) Renal epithelium
(D) Brain
7. Because hepatic glycogen stores are limited and
decrease only temporarily after epinephrine secretion, muscle glycogenolysis is the major mechanism for providing gluconeogenic precursors for
hepatic glucogenesis. This gluconeogenic substance derived from muscle is
(A)
(B)
(C)
(D)
(E)

lactate
acetyl-CoA
glucose
glycerol
alanine

8. Progesterone secretion during the second and


third trimesters of pregnancy is a measure of the
functional status of the
(A)
(B)
(C)
(D)
(E)

materno-placental unit
corpus luteum
fetal liver
fetal adrenal gland
maternal ovary

9. The most biologically active iodothyronine secreted by the thyroid follicles is


(A)

T3

(B) T.
(C)

rT3

(D) thyroglobulin
(E) triiodothyroacetic acid

5. A decrease in cortisol secretion would lead to


(A) increased storage of glycogen in the liver
(B) decreased ACTH secretion
(C) decreased adrenomedullary synthesis of epinephrine
(D) increased plasma glucose concentration
(E) increased hepatic protein synthesis

10. The primary site of 1,25-dihydroxycholecalciferol formation from its immediate precursor
is the
(A) bone
(B) liver
(C) skin
(0) nephron
( E) bloodstream

11. Which of the following endocrine organs is


larger at birth than in adulthood?
(A) Hypophysis
(B) Thyroid gland
(C) Adrenal gland
(D) Parathyroid glands
( E) Endocri ne pancreas
1 2. The major steroid hormone secreted by the
inner zone of the fetal adrenal cortex is
(A)
(B)
(C)
(D)
(E)

cortisol
DHEA
progesterone
estriol
corticosterone

13. Prostaglandins found in the seminal fluid are


secretory products of the
(A) prostate gland
(B) Sertoli cells
(C) seminal vesicles
(D) Leydig cells
(E) epididymis
14. Which of the following is a secretory product
of the lutein-granulosa cells in nongravid women?
(A)
(B)
(C)
(D)
(E)

Androstenedione
Pregnenolone
Pregnanediol
Estriol
Estrone

15. The cells that contribute most to testicular volume are the
(A) interstitial cells
(B) tubular cells
(C) spermatocytes
(D) connective tissue cells
16. An increase in plasma PTH level would lead
to an increase in which of the following?
(A)
(B)
(C)
(D)
(E)

The number of active osteoblasts


Plasma inorganic phosphate concentration
Renal synthesis of calcitriol
Collagen synthesis
Renal proximal tubular reabsorption of Ca2+

17. Active vitamin D3 (calcitrioi) and PTH have


many similar effects. Which of the following physiologic effects is specific only for calcitriol?
(A)
(B)
(C)
(D)
(E)

Increased renal phosphate reabsorption


Increased renal Ca2+ reabsorption
Increased intestinal Ca2+ absorption
Increased plasma [Ca2+]
Decreased plasma [H PO/-]

18. All of the following statements regarding testosterone are true EXCEPT
(A)
(B)
(C)
(D)

it is produced by the fetal testis


it inhibits LH secretion from the pituitary gland
it is a proestrogen
it is inactivated after conversion to dihydrotestosterone
(E) it accelerates epiphysial closure of the long
bones

19. All of the following are biochemical effects of


cortisol EXCEPT
(A)
(B)
(C)
(D)
( E)

hepatic lipogenesis
hepatic gluconeogenesis
muscle proteolysis
hepatic protein anabolism
hepatic glycogenesis

20. Abnormally high glucocorticoid levels would


be associated with an increase in all of the following activities in the liver EXCEPT
(A)
(B)
(C)
(D)
(E)

gluconeogenesis
glycogenesis
glycogenolysis
glucose production
protein synthesis

21. Characteristics of estriol include all of the following EXCEPT


(A)
(B)
(C)
(D)

production by the liver


production by the placenta
secretion by theca interna cells of the ovary
quantitatively the major urinary metabolite of
the estrogens
(E) the least biologically active of the endogenous
estrogens

22. Thyroid peroxidase is required for all of the


following steps in thyroid hormone synthesis
EXCEPT
(A)
(B)
(C)
(0)
(E)

iodide uptake
oxidation of iodide
iodination of active iodide
coupling of iodotyrosines
synthesis of iodothyronines

23. Oxytocin is a neurosecretory hormone released from the pars nervosa. Oxytocin secretion
promotes all of the following actions EXCEPT
(A) myometrial contraction
(B) lactogenesis
(C) milk ejection
(0) myoepithelial cell contraction
24. Activation of the sympathetic nervous system
would lead to all of the following responses
EXCEPT
(A) inhibition of peristalsis
(B) contraction of the radial ocular muscle
(C) renin secretion
(0) insulin secretion
(E) vasodilation in skeletal muscle

27. All of the following are substrates for MAO


EXCEPT
(A) norepinephrine
(B) VMA
(C) epinephrine
(0) metanephrine
(E) normetanephrine
28. Estriol synthesis during gestation requires all of
the following organs EXCEPT the
(A)
(B)
(C)
(0)
(E)

fetal pituitary gland


fetal liver
neocortex of the fetal adrenal gland
placenta
trophoblast

29. All of the following are neuropeptide hormones EXCEPT


(A)
(B)
(C)
(0)
(E)

AOH
/3-endorphin
oxytocin
somatomedin
TRH

30. All of the following are stimuli for GH release


EXCEPT

25. Insulin exerts all of the following effects


EXCEPT
(A) hyperpolarization of skeletal muscle cells
(B) promotion of lipogenesis
(C) stimulation of glycogen synthase activity
(0) increase in secondary active transport of glucose into muscle cells
(E) increase in glucose transport in adipocytes
26. Progesterone serves several important functions during pregnancy. All of the following physiologic or biochemical effects require progesterone
EXCEPT
(A) stimulation of myometrial contraction
(B) promotion of differentiation and growth of the
lactiferous ducts
(C) inhibition of renal Na+ excretion
(0) formation of cortisol by the fetal adrenal cortex
in early pregnancy
(E) formation of aldosterone by the fetal adrenal
cortex in early pregnancy

(A) bromocriptine
(B) hypoglycemia
(C) stress
(0) obesity
(E) vigorous exercise
31. True statements about /3-endorphin include all
of the following EXCEPT
(A) it reacts with the same receptors that bind morphine
(B) it is synthesized by pituitary basophils
(C) it is synthesized from corticotropin
(0) it is a proteolytic cleavage product of POMC

32. A 24-year-old woman has regular menstrual


cycles of 21-23 days. Ovulation can be expected
to occur between cycle days
(A) 7 and 9
(B) 10and 12
(C) 13 and 15
(D) 16 and 18
(E) 19 and 21

33. A 17-year-old patient with a normal female


phenotype is referred to an endocrinology clinic
because of sparse pubic and axillary hair and
amenorrhea. Chromosomal analysis reveals a male
genotype. Further evaluation reveals intraabdominal testes and circulating testosterone and
estrogen concentrations that are characteristic of
a normal man. The physician concludes that the
patient has testicular feminization syndrome, a genetic end-organ insensitivity to androgen due to
the absence of androgen receptors. Which of the
following findings would be consistent with this
syndrome?
(A) Normal wolffian duct development
(B) Normal mullerian duct development
(C) Regression of the internal genitalia
(D) Beard growth following androgen treatment
(E) Normal fertility

Questions 34-37
The graph below shows plasma steroid hormone levels as a function of time during a normal ovarian
cycle in a 22-year-old woman. The woman becomes pregnant during this cycle.

G
I
I

200

I
I

I
I

10
m
C

e~

2E

"'--

mOl
OIC

o~

a:

0
-15

-10

-5

o
+5
Day of ovarian cycle

+10

Adapted from Goodman HM: Basic Medical Endocrinology. New York, Raven, 1988, p 306.

+15

34. Ovulation is indicated by which of the following lettered points on the graph?

36. Implantation of the blastocyst corresponds to


which of the following points on the graph?

(A) A

(A) C

(B) B

(B) D

(C) C

(C) E

(D) D

(D) F

(E) E

(E)G

35. The LH peak is indicated by which of the following lettered points on the graph?

37. The gradual increase in plasma concentrations


of estradiol and progesterone beginning at point G
indicate steroid secretion by the

(A) A

(A) placenta
(B) corpus luteum
(C) adrenal cortex
(D) theca interna
( E) ovarian follicle

(B) B

(C) C
(D) D
(E) E

Directions: Each question below contains four suggested answers of which one or more is correct.
Choose the answer.
A if 1, 2, and 3 are correct
B if 1 and 3 are correct
C if 2 and 4 are correct
D if 4 is correct

E if 1, 2, 3, and 4 are correct


38. Hormones that may cause negative nitrogen
balance include

40. Epinephrine is a potent hyperglycemic agent


due to its ability to

(1) glucagon

(1) stimulate glucagon secretion

(2) thyroid hormone (excess)

(2) stimulate ACTH secretion

(3) cortisol

(3) stimulate hepatic and muscle glycogenolysis

(4) GH

(4) inhibit insulin secretion

39. Hormones that participate in maintenance of


the corpus luteum include

41. Administration of exogenous thyroid hormone


would likely lead to

(1) LH

(1) negative feedback inhibition of TSH secretion

(2) FSH

(2) decreased secretion of T3

(3) HCG

(3) decreased iodide uptake by the thyroid gland


(4) increased O 2 consumption by the brain

(4) progesterone

Directions: Each group of items in this section consists of lettered options followed by a set of numbered
items. For each item, select the one lettered option that is most closely associated with it. Each lettered
option may be selected once, more than once, or not at all.
Questions 42-45

Questions 50-55

Match each statement concerning plasma catecholamines with the specific catecholamine(s).

Match each physiologic effect with the appropriate


adrenergic receptor(s).

(A) Epinephrine
(B) Norepinephrine
(C) Both
(D) Neither

(A)
( B)
(C)
(D)

42. Decreases markedly following bilateral adrenalectomy


43. Mainly derived from tissues other than the adrenal medulla
44. Enzymatically inactivated by COMT
45. Elevated in most pheochromocytoma patients

50.
51.
52.
53.
54.
55.

Questions 46-49
Match each hormone classification with the hormone(s) to which it applies.
(A)
(B)
(C)
(D)

46.
47.
48.
49.

Somatostatin
Insulin
Both
Neither

Hypoglycemic hormone
Polypeptide hormone
Neurosecretory hormone
Hypophysiotropic hormone

Alpha receptor
Beta receptor
Both
Neither

Increased insulin secretion


Increased pupillary diameter
Bronchodilation
Increased renin secretion
Decreased peristalsis
Relaxation of the detrusor muscle

ANSWERS AND EXPLANATIONS


1. The answer is D. [VI 0 1 b (2) (a) (ii)] Somatostatin is a hypophysiotropic hormone produced by
the parvicellular neurosecretory neurons that terminate in the median eminence. Somatostatin inhibits
secretion of growth hormone (GH; somatotropin), a pituitary tropic hormone. The catecholamines,
norepinephrine and epinephrine are mainly products of the sympathetic postganglionic neurons and the
adrenal medulla, respectively. Somatomedin is a hepatic hormone.
2. The answer is D. [VII 0] The substrate for dopamine J3-hydroxylase is dihydroxyphenylethylamine
(dopamine), which is converted to norepinephrine within the cytoplasmic granules of the adrenomedullary chromaffin cell or within the vesicle of the postganglionic sympathetic neuron. The other substrates
and their enzymes are: epinephrine or norepinephrine/catechol-O-methyltransferase (COMT) or monoamine oxidase (MAO), tyrosine/tyrosine hydroxylase, phenylalanine/phenylalanine hydroxylase, and
dihydroxyphenylalanine (dopa)/dopa decarboxylase.
3. The answer is D. [X B 4 c (2)] The second meiotic division of the oocyte is not completed until just
after fertilization. From the third month of embryonic life, the primary oocytes are arrested in the
diplotene phase of the early prophase of meiosis. Completion of this first division occurs just prior to
ovulation by the extrusion of the first polar body, resulting in the formation of a secondary oocyte with
a haploid number of chromosomes (22 autosomes and 1 sex chromosome). The onset of ovulatory
menstrual cycles may lag several months behind menses, which usually occurs between the ages of 12
and 16. The second meiotic division occurs just after conception, yielding a fertilized ovum (zygote) and
a second polar body.
4. The answer is C. [X 01 a (2) (b); Table 7-10] In terms of serum concentration, the major postmenopausal steroid and pituitary tropic hormones are estrone and follicle-stimulating hormone (FSH). Menopause, defined as the physiologic cessation of menses, results from a loss in the cyclic ovarian function
caused by the failure of the ovary to respond to gonadotropins. The decline in ovarian function causes
an increase in pituitary tropic hormones, with a striking increase in plasma FSH concentration relative
to the increase in plasma luteinizing hormone (LH) level. Some estrogens continue to be produced in
postmenopausal women by the extraovarian conversion of androstenedione of adrenal origin to estrone.
5. The answer is C. [VII 0 3 b (2)] Since cortisol activates the epinephrine-forming enzyme,
phenylethanolamine-N-methyltransferase, a decrease in cortisol secretion would lead to decreased
adrenomedullary synthesis of epinephrine. Another effect of cortisol is to elevate blood glucose through
gluconeogenesis and inhibition of peripheral glucose transport and utilization. Cortisol's proteolytic effect
in extrahepatic tissues leads to increased transport of amino acids to the liver, where they are used
for gluconeogenesis, glycogenesis, and protein synthesis. Hypocortisolism, therefore, would result in
decrements in plasma glucose concentration, hepatic glycogen, and hepatic protein. A decline in free
blood cortisol levels would also lead to an increase in adrenocorticotropic hormone (ACTH; corticotropin) secretion due to the reduced negative feedback effect of cortisol on the hypothalamic-pituitary
complex.
6. The answer is A. [XII 0 1 b] Glucose is transported by facilitated diffusion in erythrocytes, skeletal
and cardiac muscle, and adipocytes. Facilitated diffusion for glucose transport requires a protein carrier
molecule in the cell membrane. The insulin-dependent facilitated diffusion mechanism for glucose is
found in skeletal and cardiac muscle and in adipose tissue. Some tissues obtain their glucose requirements
by insulin-independent mechanisms. For example, CNS cells and hepatocytes transport glucose by simple
diffusion, whereas erythrocytes transport glucose by insulin-independent facilitated diffusion. In renal
and gastrointestinal epithelium, glucose transport occurs by a secondary active transport system that is
not insulin-dependent but is Na+ -dependent. Since Na+ and glucose are transported in the same directions, this active transport system is called a symport system.
7. The answer is A. [VII H 1 b (2)] After an overnight fast, about 75% of the hepatic glucose output is
derived from glycogen and about 25% from gluconeogenesis. If fasting is prolonged or combined with
exercise, the hepatic glycogen stores are depleted much more rapidly and the percentage contribution
from gluconeogenesis increases. The gluconeogenic substrates are lactate, glycerol, and amino acids.
The lactate is derived from incomplete oxidation of glucose by the action of epinephrine on muscle,
which comprises about 45% of the body mass. Thus, net hepatic glucose synthesis and secretion requires
lactate production from muscle glycogenolysis. The major gluconeogenic source of endogenous glucose
production by the action of cortisol is alanine, with a smaller fraction available from the glycerol

released from triglycerides hydrolyzed in adipose tissue. Acetyl-CoA is not a gluconeogenic substance in
mammalian liver. Thus, epinephrine causes hyperglycemia because it stimulates hepatic glycogenolysis
and gluconeogenesis and muscle glycogenolysis while it inhibits insulin secretion.
8. The answer is A. [XI 8 2 (b) (3)] The major source of progesterone during the second and third
trimesters of pregnancy is the trophoblast of the placenta. Maternal cholesterol is the principal source
of precursor substrate for the biosynthesis of progesterone. Thus, progesterone levels during pregnancy
are an index of the functional status of the materno-placental unit. Plasma progesterone levels during
pregnancy are of limited value as an indicator of placental function alone because of the variability of
normal levels.
9. The answer is A. [XIII C 2] Triiodothyronine (T 3) is the most biologically active iodothyronine secreted
by the thyroid follicles. In the secretory process, thyroglobulin (the major storage form of thyroid
hormone) is degraded to free amino acids, including tetraiodothyronine (T4), T 3, monoiodotyrosine, and
diiodotyrosine. Of these, only T4 and T3 are released into the bloodstream, in a ratio of about 20:1. T3
has three to five times the biologic activity of T4 and is considered the most biologically active form of
thyroid hormone. T 3 also can be formed from T4 by the action of 5' -deiodinase in peripheral tissues,
especially the liver and kidney. The other circulating iodothyronines, such as reverse triiodothyronine (rT 3),
tetraiodothyroacetic acid, and triiodothyroacetic acid have much less biologic activity than T3 and T4.

10. The answer is D. [XIV ETc (3)] Active vitamin D3 (1,25-dihydroxycholecalciferol; calcitriol) is
formed in the proximal convoluted tubule by conversion (via the action of 1a-hydroxylase) from its
immediate precursor, 25-hydroxycholecalciferol (calcidiol). Vitamin D activation begins in the skin,
where the previtamin, 7-dehydrocholesterol, is photoactivated by sunlight to lipid-soluble vitamin D 3.
Vitamin D3 then is converted in the liver to calcidiol, the major circulating form of vitamin D 3.

11. The answer is C. [VIII A 3]ln the fetus, the adrenal glands are much larger relative to body size
than in the adult. In absolute size, they are almost as large at term as the fetal kidneys and are as large
as the adult adrenal glands. The fetal zone, or inner zone, of the fetal adrenal cortex undergoes complete
involution 4-12 weeks postpartum. The outer zone, or neocortex, undergoes further differentiation into
the adult adrenal cortex.

12. The answer is B. [IX 8 3 a (2)] The chief product of the fetal zone of the adrenal cortex is the weak
androgen dehydroepiandrosterone (DHEA), which is secreted as the inactive sulfate ester. DHEA sulfate
is converted in the fetal liver to 16a-hydroxydehydroepiandrosterone sulfate, which, upon aromatization
in the placenta, becomes estriol (estriol is synthesized only by the placenta and adult liver). After the
first trimester, the inner zone of the fetal adrenal cortex also can secrete cortisol and aldosterone.
Progesterone and corticosterone are synthesized, but not secreted, by the fetal adrenal cortex.

13. The answer is C. [IX ETc (T) (b) ] Prostaglandins found in seminal fluid are secretory products of
the seminal vesicles. Chemical analysis of seminal fluid provides an indirect measure of testicular function,
because the male accessory sex organs (i.e., the seminal vesicles and prostate gland) are androgendependent. The seminal vesicles secrete fructose, prostaglandins, and ascorbate into the seminal plasma;
the prostate gland secretes acid phosphatase and citrate into the seminal fluid. Originally, the prostate
was believed to be the source of the prostaglandins.
14. The answer is E. [X FTc (2)] The secretory products of the ovary are estradiol and estrone, which
are produced by the granulosa cells of the unruptured follicle and the lutein-granulosa cells of the corpus
luteum. Luteinization of the granulosa cells, which depends on LH, involves the appearance of lipid
droplets in the cytoplasm. The estrogenic precursors produced by the theca interna (and lutein-theca
internal are androstenedione and testosterone, which are converted by the granulosa (and luteingranulosa) cells into estrone and estradiol, respectively. The lutein-granulosa cells also produce and
secrete progesterone. Thus, androstenedione and pregnenolone are produced, but not secreted, by the
ovary. Estriol is not an ovarian product but is synthesized in the liver from estradiol and estrone and in
the placenta by the conversion of imported androgens. Pregnanediol is the urinary metabolite of progesterone and is formed in the liver.

15. The answer is B. [IX 8 2] Ninety percent of the volume of the testis is composed of tubular tissue,
with the remaining 10% consisting mainly of nontubular, or interstitial, cells (also called Leydig cells).
The seminiferous tubular epithelium contains three cell types: spermatogonia, spermatocytes, and Sertoli
cells. Spermatogonia and spermatocytes are germinal cells, and the Sertoli cells are nongerminal cells.

16. The answer is C. [XIV F 3 c ] The major renal effect of parathyroid hormone (PTH) is stimulation
of proximal tubular 1a-hydroxylase, an enzyme that converts calcidiol to calcitriol. PTH affects renal
Ca H reabsorption in two ways: it reduces Ca H reabsorption in the proximal tubule and increases Ca H
reabsorption in the distal tubule. The net effect is an increase in tubular Ca H reabsorption. This renal
effect of PTH can be lessened by PTH's effect on bone. PTH increases plasma [Ca H ] by promoting
bone resorption, which causes an increase in the renal filtered load of Ca H and, thus, hypercalciuria.
Also, PTH inhibits the proximal reabsorption of phosphate, thereby favoring phosphate excretion. PTH
acts on bone by dissolving the nonreadily exchangeable calcium phosphate "pool" known as stable
bone. PTH activates the osteoclasts-cells that cause osteolysis by their high content of lysosomal
enzymes. PTH also stimulates osteocytes, which are bone-bound osteoblasts that mediate osteocytic
osteolysis.

17. The answer is A. [XIV H 3 c] PTH and active vitamin 0 3 (calcitriol) have several similar effects,
particularly in Ca 2 + regulation. However, these two hormones exert different effects on renal phosphate
handling: calcitriol promotes phosphate reabsorption, whereas PTH promotes phosphate diuresis. PTH
and calcitriol both promote bone resorption, which leads to an increase in serum [Ca H ]. Calcitriol
promotes intestinal absorption of CaH and phosphate. Although PTH has no direct intestinal effect on
Ca H or phosphate absorption, it does stimulate renal 1a-hydroxylase activity and subsequent calcitriol
formation; thus, PTH indirectly promotes intestinal absorption of Ca H and phosphate. In the kidney,
PTH promotes distal tubular Ca H reabsorption, inhibits proximal phosphate reabsorption, and activates
proximal 1a-hydroxylase. Calcitriol enhances renal tubular reabsorption of both Ca H and phosphate.
At pharmacologic doses, calcitriol mimics the effects of PTH (i.e., it promotes phosphate excretion).
18. The answer is D. [IX C 1 c]ln skin and target cells of the male reproductive tract, testosterone may
be reduced to the more potent androgen, dihydrotestosterone, in a one-way reaction by the enzyme
Sa-reductase. Testosterone also is metabolized to estradiol by the action of aromatase in various tissues,
including brain, breast, and adipose tissue.
During fetal organization, the testis is stimulated by placental human chorionic gonadotropin (HCG)
to produce testosterone. The characteristic adolescent "growth spurt" in the male results from an
interplay of testosterone and GH, which promote growth of the vertebrae, shoulder girdle, and long
bones. This growth is self-limited, as androgens also accelerate epiphysial closure. In adulthood, testosterone is secreted from the testis in response to LH secretion from the pituitary gland; testosterone acts as
a negative feedback regulator of LH secretion and, thus, its own secretion.
19. The answer is A. [VIII F 3 a-c] The overall metabolic effects of cortisol are the release of amino
acids from muscle and both the storage and release of glucose and fatty acids. Cortisol inhibits fatty acid
synthesis in the liver, and it increases blood glycerol and fatty acid concentrations in concert with other
hormones (norepinephrine, epinephrine, glucagon) to increase lipolysis in adipose tissues. Glycerol is an
excellent index of lipolysis, because, unlike free fatty acids, it is not reused by adipocytes in the resynthesis
of triglycerides. Rather, glycerol is used by the liver as a gluconeogenic substrate. Cortisol also promotes
synthesis of gluconeogenic enzymes needed to convert the amino acids released from muscle to carbohydrate synthesis in the liver. Glucocorticoids promote proteolysis and inhibit protein synthesis in most
tissues except the liver.
20. The answer is C. [VIII F 1 a] Glucocorticoids are potent hyperglycemic hormones that, at high
levels, lead to glucose intolerance. Glucocorticoids stimulate the liver to produce glucose and glycogen
due, in part, to the increased synthesis of key gluconeogenic enzymes. They also promote muscle
proteolysis, resulting in the release of amino acids, which (aside from leucine) are gluconeogenic. The
synthesis of hepatic gluconeogenic enzymes accounts for the protein anabolic effect of glucocorticoids
in liver. Hepatic glycogenesis occurs via the activation of glycogen synthase by glucocorticoids. The
increased hepatic glucose production and secretion involves gluconeogenesis, not glycogen degradation
(glycogenolysis).
21. The answer is C. [X 0 1 a (3); XI B 3] Estriol, the least biologically active natural estrogen, is
produced only in the liver and placenta. It is not an ovarian product but represents the predominant
urinary end product of estrogen metabolism. In pregnancy, the levels of urinary estriol increase 1000-fold,
because estriol is formed by the trophoblast of the placenta. The liver converts estrone and estradiol
of ovarian origin into estriol, whereas the trophoblast converts 16a-hydroxydehydroepiandrosterone
(16a-OH DHEA) sulfate of fetal adrenal origin into estriol.

22. The answer is A. [XIII 0 T] The biosynthetic pathway for thyroid hormone has four steps: active
uptake of inorganic iodide, oxidation of iodide to active iodide, iodination of the tyrosyl residues on the
thyroglobulin molecule, and coupling of iodotyrosines to form iodothyronines. Only the first step does
not require thyroid peroxidase. Secretion depends on a lysosomal protease.
23. The answer is B. [VI C 2, 4, 0 2 c] Prolactin, not oxytocin, causes milk synthesis (lactogenesis);
oxytocin evokes milk secretion (ejection). Oxytocin is an octapeptide produced mainly in the paraventricular nucleus of the ventral diencephalon. Since it is synthesized in the magnocellular neurosecretory
neurons, oxytocin is classified as a neuropeptide, or neurosecretory hormone. The unmyelinated axons
of the paraventricular nucleus contribute to the supraoptico-hypophysial tract, which terminates in the
pars nervosa. The pars nervosa is a release center and not the site of oxytocin synthesis. Oxytocin has
two important smooth muscle effects: It causes milk ejection via contraction of the myoepithelial cells
of the mammary gland, and it promotes myometrial (uterine) contraction.
24. The answer is D. [VII H T c (T J (aJ] Activation of the sympathetic nervous system involves stimulation
of both a- and ,a-adrenergic receptors by both norepinephrine and epinephrine. The a-adrenergic
response of the pancreatic islet cells is dominant during sympathetic stimulation; thus, insulin secretion
(a response mediated by ,a-adrenergic receptors) is suppressed. Other responses that are mediated by
,a-adrenergic receptors include renin secretion, vasodilation, and inhibition of intestinal motility. Responses mediated by a-adrenergic receptors include inhibition of intestinal motility, contraction of the
radial eye muscle (mydriasis), and vasoconstriction. Vasodilation in skeletal muscle is caused by epinephrine and by stimulation of sympathetic cholinergic nerves.
25. The answer is D. [XII 0 T b]lnsulin decreases cell membrane permeability to both Na+ and K+,
but it decreases Na+ permeability more, causing hyperpolarization of skeletal and cardiac muscle cells
and adipocytes but not liver or pancreatic cells. Insulin also promotes lipogenesis, protein anabolism,
glycogenesis, and glycolysis. Insulin promotes glucose uptake by muscle and adipose tissue but not by
the liver, and it promotes storage and utilization of glucose in all three tissues. Glucose storage by muscle
and liver is achieved by formation of glycogen via an increase in glycogen synthase activity. Glucose
transport across skeletal and cardiac muscle cells and adipocytes occurs by insulin-dependent facilitated
diffusion.
26. The answer is A. [XI B 2 d, C 2] By 8 weeks gestation, maternal cholesterol can be converted
to progesterone by the placental trophoblast, which becomes the major producer of progesterone.
Progesterone is necessary for maintenance of the decidual cells of the endometrium, inhibition of
myometrial contraction by the hyperpolarization of uterine smooth muscle cells, and formation of a
dense, viscous mucus that seals off the uterine cavity. Progesterone, in synergy with estrogen, promotes
the growth and branching of the lobuloalveolar ductal system of the mammary gland. In early pregnancy,
placental progesterone serves as a precursor for the synthesis of cortisol and aldosterone by the fetal
zone of the adrenal cortex under the stimulation of HCG. Beyond 10 weeks gestation, the outer zone
of the fetal adrenal cortex (neocortex) can produce these two corticoids under the stimulation of ACTH.
Progesterone also exhibits a natriuretic effect by antagonizing the action of aldosterone on the renal
tubule.
27. The answer is B. [VII F 2 a (1), (2J] Vanillylmandelic acid (VMA) is the major urinary end product
of catecholamine metabolism. Monoamine oxidase (MAO) is a mitochondrial enzyme that catalyzes the
oxidative deamination of the catecholamines--dopamine, norepinephrine, and epinephrine. MAO also
inactivates the indolamine, serotonin (5-hydroxytryptamine). Substrates for MAO also include normetanephrine and metanephrine.
28. The answer is C. [XI B 3 a] The neocortex of the fetal adrenal gland does not participate in estriol
synthesis during gestation. Estriol is a product of the combined activities of the fetal pituitary gland, the
fetal zone of the adrenal cortex, the fetal liver, and the trophoblast of the placenta. The fetal zone of the
adrenal gland is the inner zone that persists only during gestation; the neocortex is the outer zone of the
fetal adrenal gland, which is the progenitor of the adult adrenal cortex. In the first trimester, the fetal
zone is primarily stimulated by HCG, with ACTH (from the fetal pituitary) gaining prominence thereafter.
The fetal zone synthesizes DHEA and its sulfate (DHEAS), which are transported to the placenta after
16a-hydroxylation by the fetal liver. It is 16a-OH DHEAS that is the precursor of placental estriol.
29. The answer is D. [VI 0 T c (1) (aJ] The protein anabolic effects of GH are mediated through a
GH-dependent peptide known as somatomedin, which is synthesized in the liver. Important biologic

effects of somatomedin include mitogenesis of chondrocytes and bone cells, stimulation of lipogenesis
and muscle glycogenesis, and GH-like actions in cartilage (e.g., sulfation of chondroitin). ADH and
oxytocin are neuropeptides synthesized mainly in the neurosecretory neurons of the supraoptic and
paraventricular nuclei, respectively. Thyrotropin releasing hormone (TRH) is produced in the neurosecretory neurons of the arcuate nucleus. Endorphins are secretory products of basophils in the adenohypophysis and of brain neurons. Thus, endorphins also can be classified as neuropeptides.

30. The answer is D. [VI 0 1 b (2) (b)] Secretion of GH in response to various stimuli often is
blunted in obesity. Stimuli for GH secretion include norepinephrine, bromocriptine, hypoglycemia, stress,
vigorous exercises, insulin, and arginine. The long-term metabolic effects of GH include hyperglycemia
due to gluconeogenesis, glycogenolysis, and lipolysis, all of which explain its anti-insulin effects. GH has
a fat catabolic effect, which increases plasma free fatty acid levels. It should be noted that these free
fatty acids are not substrates for gluconeogenesis.
31. The answer is C. [V 8 " 2] The polypeptide known as J3-endorphin is formed from J3-lipotropin
(not from ACTH), which is a cleavage product of the pro hormone pro-opiomelanocortin (POMC). POMC
and its derivatives are synthesized in the basophils of the pars distalis of the anterior lobe of the pituitary
gland. Endorphins, J3-lipotropin, and ACTH (corticotropin) also are found in the brain. ACTH can be
cleaved to form a-melanocyte-stimulating hormone (a-MSH) and a corticotropin-like peptide called
CLIP. J3-Endorphin binds with morphine receptors and produces morphine-like responses, such as
respiratory depression, analgesia, and miosis.

32. The answer is A. [X E 3 c] In this woman with a menstrual cycle of 21-23 days, ovulation would
be expected to occur between day 7 and day 9, counting from the first day of menses. The normal
menstrual cycle has an average duration of 28 days. The luteal, or postovulatory, phase corresponds to
the secretory phase of the endometrium or the progestational phase of the corpus luteum. This phase of
the menstrual cycle usually is very constant, lasting about 14 days, while the duration of the follicular
phase can be highly variable. Because of the constancy of the postovulatory phase, the time of ovulation
can be estimated by subtracting 14 days from the cycle length.

33. The answer is C. [IX A 1 e (1), (2)] Patients with testicular feminization have a female phenotype
but a male genotype. They have intra-abdominal testes that produce testosterone and estrogen concentrations that are characteristic of normal men, but their tissues are totally unresponsive to androgens due
to a lack of androgen receptors and they do not produce sperm. The external genitalia are female because
the primordial tissue develops in the female pattern unless stimulated by androgen. The androgen
resistance syndromes are disorders in which miillerian duct regression and testosterone synthesis are
normal. Male development of the embryo requires miillerian duct inhibiting factor (to prevent the
development of female internal genitalia), testosterone (to mediate wolffian duct development), and
dihydrotestosterone (to stimulate development of the prostate and male external genitalia). This patient
lacks female internal genitalia (because the miillerian duct inhibiting factor exerts its normal action) and
lacks male wolffian duct development (because testosterone does not exert its normal stimulatory action).
Since both duct systems regress, neither male nor female internal genitalia develop. Secondary sex
characteristics, including breast development, appear at puberty in response to the unopposed action of
estrogen formed extragonadally from testosterone. This patient has male pseudohermaphroditism, in that
the female phenotype in this genetic male is not due to the presence of both testes and ovaries.

34-37. The answers are: 34-C, 35-8, 36-D, 37-8. [X E 3 b, c (3), F 1 b; XI A 1 d; Figure 7-9] In a
normal ovarian cycle, the plasma concentration of estradiol peaks (point A), triggering a sudden discharge
of pituitary gonadotropins (LH and FSH) 12-24 hours later. This preovulatory gonadotropin surge (point
B) is more pronounced for LH than for FSH. Ovulation (point q occurs 16-20 hours after the LH peak
and marks the formation of the corpus luteum. LH maintains the functional and morphologic integrity
of the corpus luteum for 14 days.
The ovum can be fertilized between 6 and 20 hours following ovulation (point 0). If fertilization
occurs, a morula is formed, which enters the uterine cavity on the third to fourth day postovulation
(point EJ. The morula is converted to the blastocyst during the fifth to sixth day postovulation, which is
implanted in the endometrial wall of the uterus on the seventh day (point F).
The corpus luteum is the principal source of estrogen (estradiol) and progesterone during the first 68 weeks gestation. The principal gonadotropic hormone during this period is HCG, which is secreted by
the syncytiotrophoblast of the placenta. This anterior pituitary-like hormone has mainly LH activity and
is the luteotropic hormone of pregnancy. The trophoblast takes over as the major source of progesterone

and estrogen (estriol) secretion by 8 weeks gestation. HCG also stimulates the fetal testis to produce
testosterone and the fetal adrenal gland to produce corticoids in early pregnancy.
38. The answer is A (1, 2, 3). [VI 0 1 c (2) (b)) GH, like insulin and normal levels of thyroid hormone,
is a potent protein anabolic hormone (Le., it promotes nitrogen retention and, thus, a positive nitrogen
balance). Negative nitrogen balance results when nitrogen losses exceed nitrogen intake, as occurs with
deficient dietary protein or in stressful conditions (e.g., tissue trauma, disease states, burns, surgery).
Utilization of amino acids in hepatic gluconeogenesis results in negative nitrogen balance. Glucocorticoids
(e.g., cortisol) promote muscle proteolysis and inhibit protein synthesis, leading to increased BUN and
enhanced nitrogen excretion. Alanine is the predominant amino acid released by muscle. Glucagon is a
catabolic hormone for protein in muscle, and it also is an important gluconeogenic hormone. At normal
levels, thyroid hormone stimulates protein synthesis and degradation. The positive effect of thyroid
hormone on body growth is derived largely from stimulation of protein synthesis. In excess amounts,
thyroid hormone causes accelerated protein catabolism, leading to increased nitrogen excretion.
39. The answer is 8 (1, 3). [X F 1 b (2); XI C 1) Hormones that maintain the functional and morphologic
integrity of the corpus luteum (called luteotropic hormones) include LH and HCG. LH is the luteotropic
hormone during the luteal phase of the menstrual cycle; it converts the ovulated follicle into a functioning
corpus luteum. HCG is the luteotropic hormone of pregnancy; it stimulates progesterone secretion during
the first 6-8 weeks gestation, and it converts the corpus luteum of menstruation to the corpus luteum of
pregnancy. FSH and progesterone are not luteotropic hormones.
40. The answer is E (all). [VII H 1 c) Epinephrine is a potent hyperglycemic hormone due mainly to its
effects on the liver and pancreas. In the liver, it promotes glycogenolysis and gluconeogenesis, and the
glucose-6-phosphate formed by glycogenolysis is hydrolyzed to glucose. Epinephrine-induced glycogenolysis in muscle leads to formation of lactic acid, which is converted to glucose in the liver, further
elevating blood glucose. In the pancreas, epinephrine stimulates glucagon secretion and inhibits insulin
secretion, both hyperglycemic effects. Epinephrine also stimulates ACTH secretion, which, in turn, leads
to secretion of cortisol, which is a major hyperglycemic hormone.
41. The answer is A (1, 2, 3). [XIII 0 " F 1 b (2), C 1) Normally, thyroid hormone (T3 and T4) is
regulated by thyroid-stimulating hormone (TSH), which, in turn, is controlled by TRH. The circulating
levels of T3 and T4 also influence TSH release by exerting negative feedback control at the adenohypophysial and, to a lesser extent, hypothalamic levels. TSH influences the structure and function of thyroid
follicular cells and regulates all phases of thyroid hormone synthesis, storage, and secretion. Administration of exogenous thyroid hormone leads to disuse atrophy of the thyroid gland via TSH suppression.
Thus, iodide trapping and endogenous thyroid hormone secretion are reduced when TSH is suppressed
by exogenous thyroid hormone. In most cells of the body, thyroid hormone stimulates the cell membrane
enzyme Na+-K+-ATPase, thereby increasing O 2 consumption. This effect is not exerted on cells of the
brain, lymph nodes, gonads, lungs, spleen, and dermis.
42-45. The answers are: 42-A, 43-8, 44-C, 45-8. [VII C 2, F 2 b, I 2 c) The adrenal medulla is a
modified sympathetic ganglion consisting of chromaffin cells (pheochromocytes) that are the functional
analogs of the postganglionic neurons. About 80% of the adrenomedullary secretion is epinephrine, and
20% is norepinephrine. The plasma concentration ratio of epinephrine to norepinephrine normally is
1:4, the inverse of the secretory ratio. All circulating epinephrine originates in the adrenal medulla;
therefore, bilaterally adrenalectomized patients have very low plasma epinephrine levels. In contrast,
plasma norepinephrine levels in these patients remain within normal limits, which demonstrates that the
major source of circulating norepinephrine is not the adrenal medulla.
Most circulating norepinephrine is derived from the postganglionic sympathetic neurons. Norepinephrine has been demonstrated in almost all tissues except the placenta, which lacks nerve fibers. The
norepinephrine content of a tissue or organ serves as an index of the density of its sympathetic autonomic
innervation.
Catecholamines are catabolized by two enzymes: catecholamine-O-methyltransferase (COMT) and
monoamine oxidase (MAO). Norepinephrine and epinephrine are metabolized extracellularly by COMT
to normetanephrine and metanephrine, respectively. COMT is localized within the cytosol of sympathetic
effector cells.
Tumors may arise within the sympathoadrenomedullary system. Adrenal chromaffin cell tumors,
known as pheochromocytomas, usually arise within the adrenal medulla, are benign, and secrete primarily excessive amounts of norepinephrine. However, pheochromocytoma patients may have signs of
both norepinephrine and epinephrine hypersecretion. Most patients have paroxysms of hypertension,

tachycardia, sweating, tremor, palpitations, and nervousness. Most lose weight and are hyperglycemic
because of the catecholamine-induced inhibition of insulin secretion.

46-49. The answers are: 46-C, 47-C, 48-A, 49-A. [VIOl b (2) (a)i XII Alb, c, 3 c, 6 a] Somatostatin
and insulin both are hypoglycemic hormones. Somatostatin decreases blood glucose levels by decreasing
hepatic glucose production. It also inhibits the secretion of GH, glucagon, and insulin and decreases
intestinal glucose absorption. Insulin exerts its hypoglycemic action by promoting glucose uptake by
muscle and adipose tissue, by increasing storage and utilization of glucose (mainly by liver, muscle, and
adipose tissue), and by suppressing hepatic production and release of glucose.
Somatostatin and insulin are polypeptides, consisting of 14 and 51 amino acid residues, respectively.
Insulin is synthesized in the pancreatic beta cells; somatostatin is synthesized in the pancreatic delta cells
as well as many other sites, including the hypothalamus, cerebrum, thymus, thyroid, gastric and intestinal
epithelium, skin, and heart. Insulin inhibits glucagon secretion, and somatostatin inhibits secretion of GH,
TSH, insulin, glucagon, pancreatic polypeptide, gut hormones, gastric acid, and pepsin.
Somatostatin is synthesized in the arcuate nucleus of the tuberoinfundibular neurosecretory neurons
that terminate in the median eminence. Somatostatin of hypothalamic origin functions as a neurosecretory
hormone. It also is a hypophysiotropic hormone that inhibits GH and thyrotropin secretion following its
release into the hypophysial portal system. Insulin is neither a neurosecretory hormone nor a hypophysiotropic hormone.

50-55. The answers are: 50-8, 51-A, 52-8, 53-8, 54-C, 55-8. [VII Ci Table 7-5] Insulin secretion is
elicited by stimulation of the ,a-adrenergic receptor on the surface of the pancreatic beta cell.
In order to increase pupillary diameter, the radial eye muscle must contract. Contraction of this intrinsic
ocular muscle occurs by stimulation of the a-adrenergic receptor. The radial ocular muscle is innervated
only by postganglionic sympathetic nerves. A decrease in pupillary diameter occurs by stimulation of
the parasympathetic innervation of the pupillary sphincter.
Bronchodilation is caused by stimulation of ,a-adrenergic receptors. There are no a-adrenergic receptors in bronchial smooth muscle. Bronchoconstriction occurs by stimulation of the parasympathetic
innervation of the bronchi.
Renin is synthesized by the juxtaglomerular (JG) cells of the afferent arteriole, which receive postganglionic sympathetic innervation. Thus, these autonomic nerves secrete norepinephrine, which stimulates
,a-adrenergic receptors on the surface of the JG cells.
Intestinal smooth muscle contains both a- and ,a-adrenergic receptors. Intestinal motility is inhibited
by stimulation of either receptor type. Sphincters in the gastrointestinal tract contain only a-adrenergic
receptors, which, when stimulated, lead to sphincter contraction.
The detrusor muscle of the urinary bladder, in contrast to the trigone and sphincter, contains only
,a-adrenergic receptors, which, when stimulated, evoke relaxation of this smooth muscle. Stimulation of
the a-adrenergic receptors of the bladder produces contraction of these smooth muscles. Thus, sympathetic stimulation of the bladder blocks micturition.

Introduction

One of the least attractive aspects of pursuing an education is the necessity of being
examined on what has been learned. Instructors do not like to prepare tests, and students do
not like to take them.
However, students are required to take many examinations during their learning careers,
and little if any time is spent acquainting them with the positive aspects of tests and with
systematic and successful methods for approaching them. Students perceive tests as punitive
and sometimes feel that they are merely opportunities for the instructor to discover what the
student has forgotten or has never learned. Students need to view tests as opportunities to
display their knowledge and to use them as tools for developing prescriptions for further study
and learning.
A brief history and discussion of the National Board of Medical Examiners (NBME) examinations (i.e., Parts I, II, and III and FLEX) are presented here, along with ideas concerning
psychological preparation for the examinations. Also presented are general considerations and
test-taking tips, as well as ways to use practice exams as educational tools. (The literature
provided by the various examination boards contains detailed information concerning the
construction and scoring of specific exams.)

National Board of Medical Examiners Examinations


Before the various NBME exams were developed, each state attempted to license physicians
through its own procedures. Differences between the quality and testing procedures of the
various state examinations resulted in the refusal of some states to recognize the licensure of
physicians licensed in other states. This made it difficult for physicians to move freely from
one state to another and produced an uneven quality of medical care in the United States.
To remedy this situation, the various state medical boards decided they would be better
served if an outside agency prepared standard exams to be given in all states, allowing each
state to meet its own needs and have a common standard by which to judge the educational
preparation of individuals applying for licensure.
One misconception concerning these outside agencies is that they are licensing authorities.
This is not the case; they are examination boards only. The individual states retain the power
to grant and revoke licenses. The examination boards are charged with designing and scoring
valid and reliable tests. They are primarily concerned with providing the states with feedback
on how examinees have performed and with making suggestions about the interpretation and
usefulness of scores. The states use this information as partial fulfillment of qualifications upon
which they grant licenses.
Students should remember that these exams are administered nationwide and, although the
general medical information is the same, educational methodologies and faculty areas of
The author of this introduction, Michael J. O'Donnell, holds the positions of Assistant Professor of Psychiatry and
Director of Biomedical Communications at the University of New Mexico School of Medicine, Albuquerque, New
Mexico.

expertise differ from institution to institution. It is unrealistic to expect that students will know
all the material presented in the exams; they may face questions on the exams in areas that
were only superficially covered in their classes. The testing authorities recognize this situation,
and their scoring procedures take it into account.

The Exams
The first exam was given in 1916. It was a combination of written, oral, and laboratory
tests, and it was administered over a 5-day period. Admission to the exam required proof of
completion of medical education and 1 year of internship.
In 1922, the examination was changed to a new format and was divided into three parts.
Part I, a 3-day essay exam, was given in the basic sciences after 2 years of medical school.
Part II, a 2-day exam, was administered shortly before or after graduation, and Part III was
taken at the end of the first postgraduate year. To pass both Part I and Part II, a score equaling
75% of the total points available in each was required.
In 1954, after a 3-year extensive study, the NBME adopted the multiple-choice format. To
pass, a statistically computed score of 75 was required, which allowed comparison of test
results from year to year. In 1971, this method was changed to one that held the mean
constant at a computed score of 500, with a predetermined deviation from the mean to
ascertain a passing or failing score. The 1971 changes permitted more sophisticated analysis of
test results and allowed schools to compare among individual students within their respective
institutions as well as among students nationwide. Feedback to students regarding performance
included the reporting of pass or failure along with scores in each of the areas tested.
During the 1980s, the ever-changing field of medicine made it necessary for the NBME to
examine once again its evaluation strategies. It was found necessary to develop questions in
multidisciplinary areas such as gerontology, health promotion, immunology, and cell and
molecular biology. In addition, it was decided that questions should test higher cognitive levels
and reasoning skills.
To meet the new goals, many changes have been made in both the form and content of
the examination. These changes include reduction in the number of questions to approximately
800 in Part I and Part II to allow students more time on each question, with total testing time
reduced on Part I from 13 to 12 hours and on Part II from 12.5 to 12 hours. The basic
science disciplines are no longer allotted the same number of questions, which permits flexible
weighing of the exam areas. Reporting of scores to schools includes total scores for individuals
and group mean scores for separate discipline areas. Only pass/fail designations and total
scores are reported to examinees. There is no longer a provision for the reporting of individual
subscores to either the examinees or medical schools. Finally, the question format used in the
new exams, now referred to as Comprehensive (Comp) I and II, is predominately multiplechoice, best-answer.

The New Format


New questions, designed specifically for Comp I, are constructed in an effort to test the
student's grasp of the sciences basic to medicine in an integrated fashion- the questions are
designed to be interdisCiplinary. Many of these items are presented as vignettes, or case
studies, followed by a series of multiple-choice, best-answer questions.
The scoring of this exam is altered. Whereas in the past the exams were scored on a normal
curve, the new exam has a predetermined standard, which must be met in order to pass. The
exam no longer concentrates on the trivial; therefore, it has been concluded that there is a

common base of information that all medical students should know in order to pass. It is
anticipated that a major shift in the pass/fail rate for the nation is unlikely. In the past, the
average student could only expect to feel comfortable with half the test and eventually would

complete approximately 67% of the questions correctly, to achieve a mean score of 500.
Although with the standard setting method it is likely that the mean score will change and
become higher, it is unlikely that the pass/fail rates will differ significantly from those in the
past. During the first testing in 1991, there will not be differential weighing of questions.
However, in the future, the NBME will be researching methods of weighing questions based
on both the time it takes to answer questions vis-a.-vis their difficulty and the perceived
importance of the information. In addition, the NBME is attempting to design a method of
delivering feedback to the student that will have considerable importance in discovering
weaknesses and pinpointing areas for further study in the event that a retake is necessary.
Since many of the proposed changes will be implemented for the first time in June 1991,
specific information regarding actual standards, question emphasis, pass/fail rates, and so forth
were unavailable at the time of publication. The publisher will update this section as information
becomes available and as we attempt to follow the evolution and changes that occur in the
area of physician evaluation.

Materials Needed for Test Preparation


In preparation for a test, many students collect far too much study material only to find that
they simply do not have the time to go through all of it. They are defeated before they begin
because either they leave areas unstudied, or they race through the material so quickly that
they cannot benefit from the activity.
It is generally more efficient for the student to use materials already at hand; that is, class
notes, one good outline to cover or strengthen areas not locally stressed and to quickly review
the whole topic, and one good text as a reference for looking up complex material needing
further explanation.
~Iso, m~!JY.-_~tl.l(lent~_~1!empt.~9_.!!l~Lze far too m~ch info.!..mation, rather than learning
and understanding less material and then relying on that learned information to determine the
answers to questiorisafThe-time-QT-ihe examination. Relying too heavily on memorized
material causes anxiety,ancrillemo;:e'anx,o'u's'students become during a test, the less learned
knowledge they are likely to use.

Positive Attitude

6. positiv~ttitude an~~ realistk:-illlPLQ!!i=.~~tial tQ S.1!f_Ce~s.il!L!est ~king. If concentration is placed on the negative aspects of tests or on the potential for failure, anxiety increases
and performance decreases. A negative attitude generally develops if the student concentrates
on "I must pass" ratheE...!!"lan Q!)_ "I can pass." "What if I fail?" becomes th!,: mai~
m9.!ivating the gudent to run from failure rather than toward success. This results from
placing too much emphasis onscores rather than understanding tFiafscores have only slight
relevance to future professional performance.
The score received is only one aspect of test performance. Test performance also indicates
the student's ability to use information during evaluation procedures and reveals how this
ability might be used in the future. For example, when a patient enters the physician's office
with a problem, the physician begins by asking questions, searching for clues, and seeking
diagnostic information. Hypotheses are then developed, which will include several potential
causes for the problem. Weighing the probabilities, the physician will begin to discard those
hypotheses with the least likelihood of being correct. Good differential diagnosis involves the
ability to deal with uncertainty, to reduce potential causes to the smallest number, and to use
all learned information in arriving at a conclusion.
The same thought process can and should be used in testing situations. It might be termed
paper-and-pencil differential diagnosis. In each question with five alternatives, of which
one is correct, there are four alternatives that are incorrect. If deductive reasoning is used, as

in solving a clinical problem, the choices can be viewed as having possibilities of being correct.
The elimination of wrong choices increases the odds that a student will be able to recognize
the correct choice. Even if the correct choice does not become evident, the probability of
guessing correctly increases. Just as differential diagnosis in a clinical setting can result in a
correct diagnosis, eliminating choices on a test can result in choosing the correct answer.
Answering questions based on what is incorrect is difficult for many students since they
have had nearly 20 years experience taking tests with the implied assertion that knowledge
can be displayed only by knowing what is correct. It must be remembered, however, that
students can display knowledge by knowing something is wrong, just as they can display it by
knowing something is right. Students should begin to think in the present as they expect

themselves to think in the future.


Paper-and-Pencil Differential Diagnosis
The technique used to arrive at the answer to the following question is an example of the
paper-and-pencil differential diagnosis approach.
A recently diagnosed case of hypothyroidism in a 45-year-old man may result in which of
the following conditions?
(A) Thyrotoxicosis
(B) Cretinism
(C) Myxedema
(0) Graves' disease

(E) Hashimoto's thyroiditis


It is presumed that all of the choices presented in the question are plausible and partially
correct. If the student begins by breaking the question into parts and trying to discover what
the question is attempting to measure, it will be possible to answer the question correctly by
using more than memorized charts concerning thyroid problems.
The question may be testing if the student knows the difference between
"hypo" and "hyper" conditions.
The answer choices may include thyroid problems that are not "hypothyroid" problems.
It is possible that one or more of the choices are "hypo" but are not
"thyroid" problems, that they are some other endocrine problems.
"Recently diagnosed in a 45-year-old man" indicates that the correct
answer is not a congenital childhood problem.
"May result in" as opposed to "resulting from" suggests that the choices
might include a problem that causes hypothyroidism rather than results
from hypothyroidism, as stated.
By applying this kind of reasoning, the student can see that choice A, thyroid toxicosis,
which is a disorder resulting from an overactive thyroid gland ("hyper"), must be eliminated.
Another piece of knowledge, that is, Graves' disease is thyroid toxicosis, eliminates choice D.
Choice B, cretinism, is indeed hypothyroidism, but is a childhood disorder. Therefore, B is
eliminated. Choice E is an inflammation of the thyroid gland-here the clue is the suffix "itis."
The reasoning is that thyroiditis, being an inflammation, may cause a thyroid problem, perhaps
even a hypothyroid problem, but there is no reason for the reverse to be true. Myxedema,
choice C, is the only choice left and the obvious correct answer.

Preparing for Board Examinations


1. Study for yourself. Although some of the material may seem irrelevant, the more you
learn now, the less you will have to learn later. Also, do not let the fear of the test rob you of
an i~porta~~..2L):2l:l~ ..~~~.~tio~:lf you study t~ learn, t~~.!(l~k. is .I~ss ~ist(l.s.tefuU~an}tudymliQl~ly_ to pass a test.
.

2.

Review all areas. You should not be selective by studying perceived weak areas and
ignoring perceived strong areas. This is probably the last time you will have the time and the
motivation to review all of the basic sciences.

3. Attempt to understand, not just memorize, the material. Ask yourself: To whom does
the material apply? Where does it apply? When does it apply? Understanding the connections
among these points allows for longer retention and aids in those situations when guessing
strategies may be needed.
4. Try to anticipate questions that might appear on the test. Ask yourself how you might
construct a question on a specific topic.

5. Give yourself a couple days of rest before the test. Studying up to the last moment will
increase your anxiety and cause potential confusion.

Taking Board Examinations


1. In the case of NBME exams, be sure to pace yourself to use the time optimally. Each
booklet is designed to take 2 hours. You should use all of your allotted time; if you finish too
early, you probably did so by moving too quickly through the test.

2. Read each question and all the alternatives carefully before you begin to make decisions. Remember the questions contain clues, as do the answer choices. As a physician, you
would not make a clinical decision without a complete examination of all the data: the same
holds true for answering test questions.
3. Read the directions for each question set carefully. You would be amazed at how many
students make mistakes in tests simply because they have not paid close attention to the
directions.
4. It is not advisable to leave blanks with the intention of coming back to answer the questions
later. Because of the way Board examinations are constructed, you probably will not pick up
any new information that will help you when you come back, and the chances of getting
numerically off on your answer sheet are greater than your chances of benefiting by skipping
around. If you feel that you must come back to a question, mark the best choice and place a
note in the margin. Generally speaking, it is best not to change answers once you have made
a decision. Your intuitive reaction and first response are correct more often than changes
made out of frustration or anxiety. Never turn in an answer sheet with blanks. Scores are
based on the number that you get correct; you are not penalized for incorrect choices.

5. Do not try to answer the questions on a stimulus-response basis. It generally will not
work. Use all of your learned knowledge.

~. Do not let anxiety d~.st~~"y_~~~~iden~~ ~~_~~~l?l~.p'~.r,E,;~_~.?_~~ntiou~9u


know enough to pass.
Use.. ' all that you have'Tearned.
-""C. _,.."."
- -'-' __
, ....,.
"~"~

''''''~~''~-''.

'''_M''~~.)''

,'\< ........... ,

7. Do not try to determine how well you are doing as you proceed. You will not be able
to make an objective assessment, and your anxiety will increase.

8. Do not expect a feeling of mastery or anything close to what you are accustomed to.
Remember, this is a nationally administered exam, not a mastery test.

9. Do not become frustrated or angry about what appear to be bad or difficult questions.
You simply do not know the answers; you cannot know everything.

Specific Test-Taking Strategies


Read the entire question carefully, regardless of format. Test questions have multiple parts.
Concentrate on picking out the pertinent key words that might help you begin to problem-solve.
Words such as "always," "never," "mostly," "primarily," and so forth play significant roles.
In all types of questions, distractors with terms such as "always" or "never" most often are
incorrect. Adjectives and adverbs can completely change the meaning of questions-pay close
attention to them. Also, medical prefixes and suffixes (e.g., "hypo-," "hyper-," "-ectomy,"
"-itis") are sometimes at the root of the question. The knowledge and application of everyday
English grammar often is the key to dissecting questions.

Multiple-Choice Questions
Read the question and the choices carefully to become familiar with the data as given.
Remember, in multiple-choice questions there is one correct answer and there are four
distractors, or incorrect answers. (Distractors are plausible and possibly correct or they would
not be called distractors.) They are generally correct for part of the question but not for the
entire question. Dissecting the question into parts aids in discerning these distractors.
If the correct answer is not immediately evident, begin eliminating the distractors. (Many
students feel that they must always start at option A and make a decision before they move
to B, thus forcing decisions they are not ready to make.) Your first decisions should be made
on those choices you feel the most confident about.
Compare the choices to each part of the question. To be wrong, a choice needs to be
incorrect for only part of the question. To be correct, it must be totally correct. If you
believe a choice is partially incorrect, tentatively eliminate that choice. Make notes next to
the choices regarding tentative decisions. One method is to place a minus sign next to the
choices you are certain are incorrect and a plus sign next to those that potentially are correct.
Finally, place a zero next to any choice you do not understand or need to come back to for
further inspection. Do not feel that you must make final decisions until you have examined all
choices carefully.
When you have eliminated as many choices as you can, decide which of those that are left
has the highest probability of being correct. Remember to use paper-and-pencil differential
diagnosis. Above all, be honest with yourself. If you do not know the answer, eliminate as
many choices as possible and choose reasonably.

Vignette-Based Questions
Vignette-based questions are nothing more than normal multiple-choice questions that use
the same case, or grouped information, for setting the problem. The NBME has been researching question types that would test the student's grasp of the integrated medical basic
sciences in a more cognitively complex fashion than can be accomplished with traditional
testing formats. These questions allow the testing of information that is more medically relevant
than memorized terminology.
It is important to realize that several questions, although grouped together and referring to
one situation or vignette, are independent questions; that is, they are able to stand alone. Your
inability to answer one question in a group should have no bearing on your ability to answer
other questions in that group.
These are multiple-choice questions, and just as with single best-answer questions, you
should use the paper-and-pencil differential diagnosis, as was described earlier.

Single Best-Answer-Matching Sets

Single best-answer-matching sets consist of a list of words or statements followed by several


numbered items or statements. Be sure to pay attention to whether the choices can be used
more than once, only once, or not at all. Consider each choice individually and carefully.
Begin with those with which you are the most familiar. It is important always to break the
statements and words into parts, as with all other question formats. If a choice is only partially
correct, then it is incorrect.

Guessing
Nothing takes the place of a firm knowledge base, but with little information to work with,
even after playing paper-and-pencil differential diagnosis, you may find it necessary to guess
at the correct answer. A few simple rules can help increase your guessing accuracy. Always
guess consistently if you have no idea what is correct; that is, after eliminating all that you
can, make the choice that agrees with your intuition or choose the option closest to the top
of the list that has not been eliminated as a potential answer.
When guessing at questions that present with choices in numerical form, you will often find
the choices listed in ascending or descending order. It is generally not wise to guess the first
or last alternative, since these are usually extreme values and are most likely incorrect.

Using the Comprehensive Exam to Learn


All too often, students do not take full advantage of practice exams. There is a tendency to
complete the exam, score it, look up the correct answers to those questions missed, and then
forget the entire thing.
In fact, great educational benefits can be derived if students would spend more time using
practice tests as learning tools. As mentioned earlier, incorrect choices in test questions are
plausible and partially correct or they would not fulfill their purpose as distractors. This means
that it is just as beneficial to look up the incorrect choices as the correct choices to discover
specifically why they are incorrect. In this way, it is possible to learn better test-taking skills
as the subtlety of question construction is uncovered.
Additionally, it is advisable to go back and attempt to restructure each question to see if all
the choices can be made correct by modifying the question. By doing this, four times as much
will be learned. By all means, look up the right answer and explanation. Then, focus on each
of the other choices and ask yourself under what conditions they might be correct. For
example, the entire thrust of the sample question concerning hypothyroidism could be altered
by changing the first few words to read:
"Hyperthyroidism recently discovered in ... "
"Hypothyroidism prenatally occurring in ... "
"Hypothyroidism resulting from ... "
This question can be used to learn and understand thyroid problems in general, not only to
memorize answers to specific questions.
In the practice exams that follow, every effort has been made to simulate the types of
questions and the degree of question difficulty in the NBME Part I Comprehensive exam.
While taking these exams, the student should attempt to create the testing conditions that
might be experienced during actual testing situations. Approximately 1 minute should be
allowed for each question, and the entire test should be finished before it is scored.

Summary
Ideally, examinations are designed to determine how much information students have
learned and how that information is used in the successful completion of the examination.
Students will be successful if these suggestions are followed:
Develop a positive attitude and maintain that attitude.
Be realistic in determining the amount of material you attempt to master
and in the score you hope to attain.
Read the directions for each type of question and the questions themselves closely and follow the directions carefully.
Guess intelligently and consistently when guessing strategies must be
used.
Bring the paper-and-pencil differential diagnosis approach to each question in the examination.
Use the test as an opportunity to display your knowledge and as a tool
for developing prescriptions for further study and learning.
National Board examinations are not easy. They may be almost impossible for those who
have unrealistic expectations or for those who allow misinformation concerning the exams to
produce anxiety out of proportion to the task at hand. They are manageable if they are
approached with a positive attitude and with consistent use of all the information that has
been learned.
Michael

J.

O'Donnell

QUESTIONS
Directions: Each of the numbered items or incomplete statements in this section is followed by answers
or by completions of the statement. Select the one lettered answer or completion that is best in each
case.
1. At the end of a normal expiration, a young
woman has an interpleural pressure of -5 cm H 20.
Without expiring further, she closes off her nose
and mouth and performs a Valsalva maneuver
against the closed airway. If the airway pressure is
+20 mm Hg during the expiratory maneuver, the
interpleural pressure would be

(A) -5 cm H 2 0
(B) 5 cm H 2 0
(C) 10 cm H 2 0
(0) 15 cm H 20
(E) 20 cm H 20
2. Which of the following is characteristic of myopic eyes?
(A) They are too short for the refractive power of
their lenses
(B) They have a focal plane behind the retina
(C) They have a far point that is greater than
normal
(0) They require a diverging lens to correct the
optical defects
(E) They have a greater than normal power of accommodation
3. Assuming a total body fluid volume of 40 L,
1 5 L of which are ECF with an osmolarity of 300
mOsm/L, what will be the equilibrium ICF osmolarity if 500 ml of a 0.15 mol/L NaCi solution are
infused intravenously?
(A)
(B)
(C)
(0)
(E)

164 mOsm/L
277 mOsm/L
286 mOsm/L
300 mOsm/L
324 mOsm/L

4. The following data were obtained from a 55year-old male patient during cardiac catheterization: O 2 consumption = 210 ml I min, O 2 content
of right ventricular blood = 11 ml/dl, O 2 content
of brachial artery blood = 18 ml/dl, and heart rate
= 75 bpm. These data indicate that which of the
following was true during the catheterization of this
patient?
(A) The tissues received 29 ml 02/dl blood flow
(B) Cardiac output was about 1470 ml/min
(C) Pulmonary venous O 2 content was about 145
ml/dl blood
(0) Right ventricular stroke volume averaged about
40 ml
(E) Cardiac output was dangerously high
5. The regression of the corpus luteum at the end
of the postovulatory phase is caused by
a decrease in FSH secretion
a decrease in LH secretion
an increase in HCG secretion
a reduced capacity of the corpus luteum to
synthesize steroids
(E) ovarian failure

(A)
(B)
(C)
(0)

6. The rate of gastric emptying is controlled primarily by reflexes that occur


(A)
(B)
(C)
(0)
(E)

during chewing
during swallowing
when chyme enters the stomach
when chyme enters the intestine
during the interdigestive period

7. Spironolactone, which is an aldosterone antagonist, is injected into the renal artery of a laboratory
animal. What are the effects on Na+ and K+ excretion, assuming that there is no change in GFR or
RBF by this drug?
Na+
(A)
(B)

K+
i
~

(C)
(0)

(E)

unchanged

8. Which of the following conditions would cause


an increase in aortic systolic pressure and a decrease in aortic pulse pressure?
(A)
(B)
(e)
(D)
(E)

Increased heart rate


Increased arterial compliance
Decreased peripheral resistance
Increased stroke volume
Increased elastic modulus

12. Acetazolamide is administered to a glaucoma


patient. Given that this drug inhibits carbonic anhydrase in the renal proximal tubule, which of the
following substances will be excreted at a lower
rate?
(A) Na+

(B) H 2 0
(e) He0 3 (D) NH4+

9. A 32-year-old male electrician consults an internist and complains of episodes of palpitations and
sweating that occur when he climbs a ladder at
work. He feels "washed out" after these attacks,
which he ascribes to "nerves." Subsequent examination by the physician leads to a diagnosis of
pheochromocytoma. Body fluid analysis of this patient is most likely to reveal a low plasma concentration of
(A)
(B)
(e)
(D)
(E)

free fatty acids


insulin
fasting glucose
lactate
pyruvate

10. The following data are collected from a 40year-old male patient:
airway resistance

diffusing capacity of
the lungs

arterial pH
minute ventilation

27 cm H 20/L/sec
(normal = 2-3 cm
H 20/L/sec)
10 ml/min/mm Hg
(normal = 30
ml/min/mm Hg)
7.32
6 L/min

These data indicate that this patient most likely has


(A) an arterial Peo 2 less than 40 mm Hg
(B) an arterial P0 2 less than 85 mm Hg
(e) a mean pulmonary artery pressure less than 15
mm Hg
(D) a total work of breathing less than normal
(E) an arterial [He0 3 -j/[H 2e0 3 j ratio greater than
20:1
11. Elimination of t~e terminal ileum will result in
malabsorption of
(A)
(B)
(e)
(D)

vitamin B12
fats
both
neither

tEl K+
13. A decrease in the total osmotic pressure of
arterial blood would lead to an increase in urine
volume by
(A) increasing the hydrostatic pressure inside the
glomerulus
(B) increasing the permeability of the glomerular
capillaries to water
(e) inhibiting ADH secretion
(D) stimulating the secretion of aldosterone
(E) directly inhibiting the reabsorption of water by
the collecting ducts
14. During moderate exercise, a patient has a cardiac index of 6.5 L/min/m2, a hemoglobin concentration of 12 g/dl, a venous P0 2 of 30 mm Hg, and
a venous O 2 saturation of 50 % Assuming 100%
hemoglobin saturation in arterial blood, what is this
patient's O 2 consumption?
(A)
(B)
(e)
(D)
(E)

150 ml/min/m2
275 ml/min/m 2
520 ml/min/m2
790 ml/min/m2
1030 ml/min/m2

15. A red blood cell is placed in a solution. The


cell initially shrinks and then returns to its original
volume. The red cell's reaction indicates that the
solution is
(A)
(B)
(e)
(D)
(E)

hyperosmotic
hypo-osmotic
hyperosmotic
hyperosmotic
hypo-osmotic

and
and
and
and
and

hypertonic
hypertonic
isotonic
hypotonic
isotonic

16. A 32-year-old woman is admitted to the hospital with suspected partially compensated respiratory acidosis. Which of the following sets of laboratory data would confirm this suspicion?

[HCO J - ]
(mEq/L)

Peo 2
(mm Hg)

pH

17
31
9.8
24
20

19
80
30
45
25

7.9
7.22
7.14
7.5
7.5

(A)
(B)
(C)
(0)
(E)

17. Following a massive hemorrhage during delivery, a 34-year-old woman experiences a failure to
lactate and to menstruate. Which of the following
is most likely to be associated with this clinical
picture?
(A) Elevated prolactin secretion
(B) Excessive urinary Na + excretion
(C) Excessive water excretion
(0) Increased sensitivity to insulin
(E) Elevated gonadotropin secretion

20. A delirious 5-year-old boy is brought to the


emergency room. His parents report that their son
was in good health until 2 hours beforehand, when
his mental state suddenly began to deteriorate.
They suspect he may have swallowed a bottle of
aspirin, because they discovered an empty bottle
before leaving for the hospital. Laboratory evaluation reveals the following arterial blood data: [H+]
= 18 nmol/L, [HC0 3 -] = 13 mmol/L, and Peo 2
= 10 mm Hg. This patient's history and blood data
are most likely associated with
(A) respiratory alkalosis with partial renal compensation
(B) metabolic alkalosis with partial respiratory
compensation
(C) a reduced [HC0 3 -]/dissolved CO 2 ratio
(0) a greater than normal CO 2 content
21. If acidosis and hypokalemia result from loss of
fluid from the GI tract, the fluid was most likely
drained from the
(A) stomach
(B) intestine
(C) gallbladder

(0) pancreas

18. If the mean electrical axis of a ventricular depolarization is-30, the resultant EKG should reveal
the largest positive QRS complex in lead
(A) I
(B) II

(C) aVR
(0) aVL
(E) aVF
19. Normally, most of the H + is excreted by the
kidneys in the form of
(A) HC0 3 (B) phosphate ion
(C) NH4+
(0) titratable acid
(E) ,B-hydroxybutyrate ion

(E) colon
22. Tapping the patella tendon initiates a reflex
contraction of the quadriceps muscle. When the
contractile force of the quadriceps muscle reaches
its peak, there is an increase in the activity of
(A) the la afferent fibers innervating the quadriceps
muscle
(B) the Ib afferent fibers innervating the quadriceps
muscle
(C) both
(0) neither
23. When the ACh receptors on the pacemaker
cells of the heart are activated, there is an increase
in the membrane conductance to
(A) K+
(B) Na+
(C) Ca H

(0) CI-

(E) K+ and Na+

24. Normal O 2 delivery to the tissues would be


cut in half by a 50% decrease in the normal value
of
(A) arterial P0 2
(B) minute ventilation
(C) hemoglobin concentration
(0) inspired P0 2
25. A 45-year-old woman with severe vomiting
caused by pyloric obstruction would be expected
to show
(A) hyperchloremia
(B) an increase in plasma [HCO l - ]
(C) an increase in alveolar ventilation
(0) acid urine
(E) a decrease in arterial Pco 2
26. In the formation of HCI by the parietal cells,
the transport of H + across the parietal cell is coupled with the transport of
(A) K+
(B) CI(C) HCOl

(0) H+
(E) Na+

29. Gastric digestion is most important for which


of the following substances?
(A) Fats
(B) Carbohydrates
(C) Proteins
(0) Vitamins
(E) Minerals
30. Resistance to blood flow through the kidney
can be determined by
(A) measuring the clearance of PAH
(B) measuring the hydrostatic pressure difference
between the renal artery and renal vein
(C) measuring the RBF
(0) dividing the arteriovenous hydrostatic pressure
difference by the RBF
(E) dividing the RBF by the arteriovenous hydrostatic pressure difference
31. The following blood data are collected from a
27-year-old male patient: pH = 7.50, [HCO l - ] =
38 mmol/L, and P0 2 = 80 mm Hg. Given these
findings, what is the expected Pco 2 for this patient?
(A) 30 mm Hg
(B) 40 mm Hg
(C) 50 mm Hg

27. Primary hyperkalemic periodic paralysis is an


inherited disease characterized by alterations in K+
homeostasis. The K+ defect leads to intermittent
periods of muscle weakness due to failure of action
potential propagation. This failure is due to
(A) inactivation of Na+ channels
(B) membrane hyperpolarization
(C) both
(0) neither
28. The normal sequence of phases of the menstrual cycle is
(A) menses, preovulatory, ovulatory, estrogenic
(B) preovulatory, ovulatory, progestational, menses
(C) ovulatory, progestational, menses, luteal
(0) progestational, menses, follicular, preovulatory
(E) menses, follicular, ovulatory, estrogenic

(0) 60 mm Hg
(E)

70 mm Hg

32. Electrically excitable channels are required for


(A) the propagation of the action potential in nerve
fibers
(B) the release of synaptic transmitter from presynaptic nerve terminals
(C) both
(0) neither
33. A 57-year-old male patient who is breathing
air at sea level has a respiratory exchange ratio of
1. Arterial blood gas analysis of this patient reveals
the following: P0 2 = 85 mm Hg, Pco 2 = 30 mm
Hg, and pH = 7.52. This patient's blood data indicate which of the following?
(A) His alveolar-to-arterial P0 2 difference exceeds
20 mm Hg
(B) His plasma [HCO l - ] is increased
(C) He has been hypoventilating
(0) He has metabolic alkalosis
(E) He has chronic obstructive lung disease

34. Given the following data: glomerular capillary


hydrostatic pressure = 47 mm Hg, glomerular capillary colloid osmotic pressure = 28 mm Hg, Bowman's space hydrostatic pressure = 10 mm Hg,
and Bowman's space oncotic pressure = 0 mm
Hg, what is the glomerular filtration pressure?
(A) 2 mm Hg
(B) 4 mm Hg
(C) 6 mm Hg
(0) 9 mm Hg
(E) 10 mm Hg
35. Following the intravenous administration of 1
L of a 150 mmol NaCI solution into a patient with
a blood loss, there will be
(A) a decrease in the plasma Na+ concentration
(B) an increase in the osmolarity of the ICF compartment
(C) an increase in the volume of the ICF compartment
(0) edema
(E) a decrease in the colloid osmotic pressure of
the plasma
36. A 45-year-old male patient is studied and
found to have a respiratory rate of 15 breaths/min,
a tidal volume of 0.5 L, and a dead space of 200
ml. The patient is asked to increase his respiratory
rate to 30 breaths/min, and his tidal volume is
measured at 350 ml. Assuming no change in dead
space, which of the following is true regarding alveolar Peo 2 ?
(A) Peo2 will increase because of the decreased
ventilation
(B) Peo 2 will decrease because of the increased
ventilation
(C) Peo 2 will not change because it is not affected
by respiration
(0) Peo 2 will not change because alveolar ventilation remains constant
(E) The arterial blood pH will decrease because of
the increased ventilation
37. Cortisol affects the biosynthesis of epinephrine
in the adrenal medulla by
(A) augmenting the conversion of dopamine to epinephrine
(B) activating the epinephrine-forming enzyme
(C) inhibiting the methylation of norepinephrine
(0) activating catechol-O-methyltransferase
(E) increasing the release of acetylcholine

38. A positive QRS complex in leads aVR and aVF


indicates
(A) no axis deviation
(B) right axis deviation
(C) left axis deviation
(0) left ventricular hypertrophy
(E) a mean electrical axis between

o and 90

39. The following arterial blood data are collected


from a 42-year-old female patient: [H+] = 49
nEq/L, Peo 2 = 30 mm Hg, and P02 = 95 mm Hg.
Given these findings, what is the expected arterial
[HC0 3 -] for this patient?
(A) 13.2 mEq/L
(B) 14.7 mEq/L
(C) 15.8 mEq/L
(0) 16.5 mEq/L
(E) 1 7.1 m Eq / L
40. Which of the following statements best characterizes the transpulmonary pressure at the base
of the lung of a person who is standing?
(A) It is independent of lung volume
(B) It is equal to the transpulmonary pressure at
the apex of the lung
(C) It may be negative if the lung is at residual
volume
(0) It causes the basal alveoli to be more dilated
than the apical alveoli
(E) It causes the bronchioles at the lung base to be
more dilated than those at the apex
41. Which of the following reflexes is most dependent on a vago-vagal reflex?
(A) Chewing
(B) Swallowing
(C) Receptive relaxation
(0) Gastric emptying
(E) Intestinal segmentation

42. A 23-year-old woman with diabetes mellitus


is admitted to the hospital. She is dehydrated and
hyperpneic. Laboratory examination reveals high
urinary concentrations of acetoacetic acid and glucose, blood pH of 7.39, plasma [He0 3 -] of 19.0
mmol/L, and plasma Peo 2 of 33 mm Hg. These
data are most suggestive of
(A) metabolic alkalosis with complete renal com-

47. During the first 6-8 weeks of pregnancy, progesterone is secreted mainly by the
(A) maternal adrenal glands
(B) maternal theca interna

(e) corpus luteum


(D) fetal adrenal gland
(E) decidua

pensation
(B) metabolic acidosis with complete respiratory

compensation
(e) respiratory acidosis with partial renal compensation
(D) respiratory alkalosis with partial renal compensation
43. Epinephrine inhibits glucose uptake by muscle
and adipose tissue. This inhibitory effect is attributed to
(A) glucagon secretion
(B) thyroid hormone secretion

(e) inhibition of insulin secretion


(D) inhibition of GH secretion
(E) inhibition of cortisol secretion
44. The pars nervosa is derived from which of the
following embryonic tissues?
(A) Oral ectoderm
(B) Neural ectoderm
(C) Neural crest
(D) Endoderm
( E) Mesoderm
45. A 32-year-old man can generate an inspiratory
pressure of -50 mm Hg intermittently for several
minutes. How deep can this man lie underwater
while breathing through a tube, if the tube offers
no significant resistance to air flow?
(A) 37 mm

48. Which of the following factors best explains


an increase in the filtration fraction?
(A) Increased ureteral pressure
(B) Increased efferent arteriolar resistance
(e) Increased plasma protein concentration
(D) Decreased glomerular capillary hydrostatic
pressure
(E) Decreased glomerular filtration area

49. The greatest amount of fat absorption occurs


in the
(A) stomach

(B) duodenum
(C) ileum
(D) colon
(E) rectum

50. A 27-year-old, anxious man is examined in the


emergency room, after which the following arterial
blood data are obtained:

Blood Chemistry

Blood Gas

[Na+] = 140 mEq/L


[K+] = 4 mEq/L
[He0 3 -] = 19 mEq/L
[CI-] = 109 mEq/L

pH

Peo 2
P0 2

7.6
20 mm Hg
98 mm Hg

From the above data, the most likely diagnosis is


(A) hypoxia

(E) 74 cm

(B) metabolic alkalosis


(e) metabolic acidosis with respiratory compensation
(D) obstructive lung disease
( E) respiratory alkalosis

46. Which of the following conditions is most


likely to cause acidosis with marked dehydration?

51. Epinephrine-forming enzyme activity is increased directly by

(A)
(B)
(e)
(D)
(E)

(A) hydrocortisone
(B) acetylcholine
(e) norepinephrine

(B) 37 cm
(C) 68 mm
(D) 68 cm

Severe diarrhea
Severe, persistent vomiting
Excessive sweating
Drinking sodium lactate solution
Complete water deprivation for 24 hours

(D) ACTH
( E) ll-deoxycortisol

52. Two patients are studied and the following


data are collected:

Patient

A
B

Respiratory
Rate
(breaths/min)

Tidal
Volume
(ml)

Dead
Space

20
10

200
400

150
150

(ml)

Which of the following statements about these two


patients is true?
(A) The alveolar ventilation in patient A is greater
than in patient B
(B) The alveolar ventilation in patient B is greater
than in patient A
(C) The alveolar ventilation in both patients is equal
(D) The dead space ventilation in both patients is
equal
53. A semicomatose 19-year-old woman is
brought to the emergency room with dry skin, hyperventilation, hypotension, and a rapid pulse rate.
The following blood data are obtained:
pH = 7.14
[Na+] = 140 mEq/L
[K+] = 4.5 mEq/L
[CI-] = 82 mEq/L
[HC0 3 - ] = 11 mEq/L
Pco 2 = 30 mm Hg
[glucose] = 180 mg/dl
From the above history and laboratory data, the
most likely diagnosis is

56. When the arterial P0 2 drops from 100 mm Hg


to 27 mm Hg with an arterial pH of 7.4 and a Pco 2
of 40 mm Hg, O 2 content in the blood decreases
by about
(A) 10%
(B) 25%
(C) 33%
(D) 50%
(E) 75%

57. A newborn genotypic male is found to have an


adrenogenital syndrome due to a 1 7a-hydroxylase
defect. Which of the following biochemical reactions in the biosynthesis of gonadal hormones is
decelerated in this case of congenital adrenal hyperplasia?
(A)
(B)
(C)
(D)
(E)

Pregnenolone .... 17a-hydroxypregnenolone


Cholesterol .... pregnenolone
Progesterone .... corticosterone
Deoxycorticosterone .... corticosterone
17a-Hydroxypregnenolone .... 17 a-hydroxyprogesterone

58. I3-Adrenergic receptors mediate all of the following responses EXCEPT


(A)
(B)
(C)
(D)
(E)

ciliary muscle contraction


increased myocardial contractility
vasodilation
insulin secretion
decreased intestinal motility

(A) metabolic alkalosis


(6) metabolic acidosis

(C) respiratory alkalosis

(D) respiratory acidosis


(E) hypoglycemia
54. Which of the following hormones is correctly
paired with its effect on renal electrolyte reabsorption?
(A)
(B)
(C)
(D)
(E)

Calcitriol / increased HPO/- reabsorption


Calcitonin / increased Ca2+ reabsorption
Aldosterone / increased K+ reabsorption
Progesterone / increased Na+ reabsorption
Calcitonin / increased HPO/- reabsorption

55. During the process of vitamin B12 absorption,


almost all of the ingested vitamin B12
(A) binds to intrinsic factor in the stomach
(6) is absorbed in the stomach

(C) both
(D) neither

59. All of the following substances can be converted to more biologically active forms after secretion EXCEPT
(A) angiotensinogen
(B) angiotensin I
(C) tetraiodothyronine
(D) testosterone
60. A large dose of insulin is administered intravenously to a normal 34-year-old female patient. This
is likely to cause an increase in all of the following
EXCEPT
(A) plasma epinephrine concentration
(6) plasma K+ concentration
(C) ACTH secretion
(D) GH secretion
(E) glucagon secretion

61. Findings consistent with metabolic acidosis include all of the following EXCEPT

67. Stimuli for aldosterone secretion include all of


the following EXCEPT

(A) increased excretion of titrable acid


(B) increased excretion of NH4 +
(C) decreased excretion of NaHC0 3
(0) decreased respiratory rate
( E) a negative base excess

(A) angiotensin II
(B) hyperkalemia
(C) hypovolemia
(0) corticotropin
(E) atrial natriuretic factor

62. All of the following are the renal responses to


severe hemorrhage EXCEPT

68. Cutting the vagus nerve has a major effect on


all of the following EXCEPT

(A) increased Na+ reabsorption by the distal


nephron
(B) decreased GFR
(C) increased permeability of the collecting duct to
water
(0) decreased filtration fraction
(E) increased renin secretion

(A) the rate of liquid emptying from the stomach


(B) receptive relaxation
(C) migrating motor complex
(0) primary esophageal peristalsis

63. Untreated type I diabetes mellitus is associated with all of the following biochemical changes
EXCEPT
(A) positive nitrogen balance
(B) ketonemia
(C) ketonuria
(0) low plasma C-peptide concentration
(E) glycosuria

64. All of the following would increase alveolar


P0 2 EXCEPT
(A) breathing gas containing 24% O 2
(B) breathing gas containing 28% O 2
(C) ascending a mountain
(0) hyperventilating

65. Hypophysectomy results in the functional decline of many endocrine organs. All of the following changes are likely to occur after removal of the
pituitary gland EXCEPT
(A) atrophy of the thyroid gland
(B) dwarfism if performed during early adolescence
(C) deficiency of aldosterone
(0) cessation of menstrual cycles
(E) impaired testosterone secretion
66. All of the following are substrates for COMT
EXCEPT
(A) epinephrine
(B) norepinephrine
(C) normetanephrine
(0) dihydroxymandelic acid

69. GH secretion is increased by all of the following factors EXCEPT


(A) insulin administration
(B) arginine administration

(C) somatostatin administration


(0) onset of sleep

( E) exercise

70. All of the following substances are required


for thyroxine biosynthesis EXCEPT
(A) active iodide
(B) diiodotyrosine
(C) monoiodotyrosine
(0) thyroglobulin

(E) thyroid peroxidase

71. Hormones with lipolytic activity include all of


the following EXCEPT
(A) glucagon
(B) epinephrine
(C) insulin
(0) cortisol
(E) GH
72. Expansion of the antrum causes an increase in
all of the following EXCEPT
(A) secretion of gastrin
(B) secretion of pancreatic enzymes
(C) secretion of gastric acid (HC!)
(0) gastric motility
( E) receptive relaxation

73. The neural crest gives rise to all of the following tissues and cells EXCEPT
(A) Schwann cells
(B) cartilage and bone of the skull
(C) the neural lobe of the pituitary gland
(0) melanocytes
(E) the adrenal medulla

74. In hyperaldosteronemia, all of the following


conditions are likely to be observed EXCEPT
(A)
(B)
(C)
(D)
(E)

decreased hematocrit
fall in plasma oncotic pressure
increased ECF volume
hyperkalemia
metabolic alkalosis

Questions 77-80
A patient presents with crushing chest pain, shortness of breath, and marked anxiety. A preliminary
diagnosis of acute myocardial infarction is made.
Physical examination reveals evidence of pulmonary edema, cardiomegaly, peripheral edema, and
pulmonary hypertension. Arterial blood gas analysis, on room air, reveals the following: Peo 2 = 40
mm Hg, P0 2 = 60 mm Hg, pH = 7.32, and
[HC0 3 -] = 20 mEq/L.

77. This patient's blood data are most indicative


of
(A)
(B)
(C)
(D)

a diffusion abnormality
hypoventilation
a ventilation:perfusion abnormality
left-to-right cardiac shunt

Questions 75-76
The following data were obtained from intracellular
recordings of three cardiac cells (bpm = beats per
minute).

Resting membrane
potential
Spontaneous
depolarization
Intrinsic rate of
depolarization

Cell A

Cell 8

Cell C

-60 mV

-60 mV

-80 mV

yes

yes

no

80 bpm

45 bpm

none

75. Cell A most likely is located in


(A)
(B)
(C)
(D)
(E)

the SA node
atrial muscle
the AV node
Purkinje fibers
ventricular muscle

76. Pacemaker activity is exhibited by


(A)
(B)
(C)
(D)
(E)

cell A
cell B
cell C
cells A and B
cells A, B, and C

78. The patient is admitted to the CCU, sedated,


and given 40% O 2 by respirator, which is set to
deliver a tidal volume of 0.6 L at a rate of 16
breaths/min. An inspiratory pressure of 20 mm
Hg is required to deliver the tidal volume. If the
patient's predicted dead space is 150 ml, then his
calculated alveolar ventilation is approximately
(A)
(B)
(C)
(D)
(E)

5 L/min
7 L/min
9 L/min
12 L/min
16 L/min

79. While ventilation continues with 40% O 2 , a


Swan-Ganz catheter is inserted into the patient's
pulmonary artery. A repeat arterial blood analysis
reveals a P0 2 of 120 mm Hg, a Peo 2 of 43 mm
Hg, and pH of 7.31. These findings indicate that
(A)
(B)
(C)
(D)

the pulmonary edema has cleared


gas exchange is completely normal
hemoglobin concentration has increased
the alveolar-to-arterial P0 2 difference is increased

80. This patient's respiratory compliance is


(A)
(B)
(C)
(D)
(E)

0.03 L/mm Hg
0.1 L/mm Hg
9.0 L/mm Hg
12.0 mm Hg/L
26.7 mm Hg/L

Questions 81-82
Lung compliance in a 32-year-old female patient is studied. Data collected under control and experimental conditions are listed in the following table.

Change in

Control
Experimental

Respiratory Rate
(breaths/min)

Tidal Volume
(ml)

Interpleural
Pressure during
Inspiration (em H 2 0)

15
25

600
600

4
10

81. This patient's lung compliance during control


and experimental conditions was

82. This patient can be characterized as having


frequency-dependent compliance, which indicates

(A) unchanged
(6) 40 ml/breath and 24 ml/breath, respectively
(C) 150 ml/cm H 2 0 and 60 ml/cm H 2 0, respectively
(D) 150 cm H 2 0/ml and 60 cm H 2 0/ml, respectively

(A)
(B)
(C)
(D)

abnormal surfactant function


obstructive lung disease
restrictive lung disease
pulmonary vascular disease

Questions 83-87
The diagram below represents an experimental record obtained from an anesthetized dog.

Heart rate

Mean arterial
pressure

Total peripheral
resistance

Time

83. The experimental intervention at time 1 most


likely represents

86. The decrease in heart rate at time 3 was most


likely caused by

(A) electrical stimulation of the lumbar sympathetic nerve roots


(B) electrical stimulation of the superior cervical
ganglion (cardiac sympathetic nerves)
(C) administration of a ,a-adrenergic blocking drug
(0) stimulation of the right vagus nerve
(E) administration of a cholinergic blocking drug

(A) the direct effect of the drug on the SA node


(B) the direct effect of the drug on the ventricular
muscle
(C) the occurrence of ventricular extrasystoles
(0) a reflex effect mediated by the chemoreceptors
(E) a reflex effect mediated by the baroreceptors

84. The experimental intervention at time 2 most


likely represents
(A) electrical stimulation of the sacral sympathetic
nerve roots
(B) electrical stimulation of the superior cervical
ganglion (cardiac sympathetic nerves)
(C) administration of a sympathetic blocking drug
(0) stimulation of the right vagus nerve
(E) administration of a cholinergic blocking drug
85. A drug was administered at time 3. This drug
is most likely classified as
(A)
(B)
(C)
(0)

a cholinergic blocking agent


an a-adrenergic agonist
a ,a-adrenergic agonist
a ,a-adrenergic blocking agent

(E) an a-adrenergic blocking agent

87. A nerve was stimulated at time 4. This nerve


most likely was the
(A) cardiac end of a cut vagus nerve
(B) superior cervical ganglion of the sympathetic
chain
(C) central end of a cut carotid sinus nerve
(0) peripheral end of a cut carotid sinus nerve
(E) postganglionic fibers of the lumbar sympathetic
ganglia

Questions 88-93

90. In loop ABEH, mitral valve closure occurs at

The following six questions refer to the ventricular


pressure-volume (PV) relationships depicted below. Loop ABEH represents a normal PV loop.

(A) point A
(B) point B
(e) point E
(0) point H
(E) point I

I
I
I
I

Cl 200

91. In loop ABEH, ventricular filling is indicated


by

.s
~

150

F
E

I
I

iii

I
I

::;
<Il

0
.;::

a. 100

>

(A) segment AB
(B) segment BE
(C) segment EH
(0) segment HA

I
I

:l

(/)
(/)

I
I

92. The slope of the line from point G to point A


represents

50

I
I
I

0
0

50

100

150

Ventricular volume (ml)

200

(A) diastolic ventricular compliance


(B) systolic ventricular compliance
(e) diastolic ventricular elastance
(0) systolic arterial elastance
(E) diastolic arterial compliance

88. Loop AeOI shows the effect of


(A) increased heart rate
(B) increased contractility
(e) increased end-diastolic volume
(0) increased afterload
( E) decreased preload
89. Loop ABFG shows the effect of
(A) increased heart rate
(B) increased contractility
(e) increased end-diastolic volume
(0) increased afterload
( E) decreased preload

93. The volume of blood in the ventricles at points


0, E, and F is referred to as ventricular
(A) stroke volume
(B) end-diastolic volume
(C) end-systolic volume
(0) diastolic reserve volume
(E) residual volume

Questions 94-97

Questions 98-100

The data below represent the volume of distribution of tritiated water, inulin, and Evan's blue dye
in a 60-kg man after allowing time for equilibration.

A normal 55-year-old man who lives at an altitude


of 11,500 feet is seen for an annual physical examination. Findings on examination include a hemoglobin concentration of 18 gldl, an arterial P0 2 of
27 mm Hg, and an arterial pH of 7040.

Space
Tritiated water
Inulin
Evan's blue

Volume (L)
35
8
3

94. With a hematocrit ratio of 0040, the man's


blood volume is
(A) 2 L
(B) 3 L
(C) 4 L

(0) 5 L
(E) 6 L
95. The interstitial fluid volume is
(A) 3 L
(B) 5 L
(C) 8 L
(0) 11 L
(E) 24 L

96. The ECF volume is


(A) 2 L
(B) 4 L

(C) 6 L
(0) 8 L
(E) 10 L
97. Assuming that the total body water constitutes
70% of the lean body mass, the amount of body
fat in this man is approximately
(A) 5 kg
(B) 10 kg
(C)15kg
(0) 20 kg
(E) 25 kg

98. Based on the above findings, this man's arterial


O 2 content (mild!) would be approximately
(A) 8
(B) 12

(C) 15
(0) 18
(E) 24

99. This man's pulmonary artery pressure is likely


to be
(A) normal
(B) lower than normal due to inhibition of chemoreceptors caused by the low Peo 2
(C) higher than normal due to increased cardiac
output
(0) higher than normal due to hypoxic pulmonary
vasoconstriction
100. The hemoglobin saturation in this man's venous blood is likely to be
(A) normal
(B) less than normal due to decreased arterial hemoglobin saturation
(C) greater than normal due to increased cardiac
output
(0) greater than normal due to increased hemoglobin concentration

101. The curves below represent the clearances


of various substances as a function of their plasma
concentrations. Curve E represents the clearance
curve for glucose. Following the administration of
a substance (phlorizin) that blocks epithelial transport of glucose, the clearance curve for glucose
would resemble which of the following curves?

c
g

:
CD

61~
200

<.l

c:

f!!

<11
CD

U
100

Plasma concentration (mg/dl)

Adapted from Bauman JW, Chinard FP: Renal Function:

Physiological and Medical Aspects. SI. Louis, Mosby,


1975, p 43.
(A) A

(B) B
(C) C
(D) D
(E) E

Questions 102-103
The left ventricular and aortic pressure tracings below were recorded during cardiac catheterization of
a 62-year-old patient who complains of chest pain and dizziness on exertion.

--

' - =Aortic pressure


= Left ventricular pressure

200
Ci
I

.s
~

::J
VJ
VJ

100

a..

Time (sec)

102. The left ventricular and aortic pressure tracings indicate that this patient has

103. The most likely physical sign of this patient's


condition is

(A)
(8)
(C)
(D)
(E)

(A)
(8)
(C)
(D)

pulmonary stenosis
aortic stenosis
mitral stenosis
aortic insufficiency
mitral insufficiency

a
a
a
a

systolic murmur
diastolic murmur
presystolic murmur
mid-diastolic murmur

Questions 104-106
A male patient undergoes lung volume studies using the helium dilution method. The initial fraction
of helium in the spirometer is 0.05, and the helium
fraction after equilibration with the lungs is 0.03.
The volume of gas in the spirometer is kept constant at 4 L during the procedure by the addition
of 02. According to a spirogram, this patient's VC
is 5 L and his ERV is 2 L.

(A) Point A
(B) Point B
(C) Point C
(0) Point 0
(E) Point E
108. The period of isovolumic ventricular contraction is represented by which segment of the curve?

104. What is this patient's FRC?

to B
to C
to 0
to E
(E) E to A

(A)
(B)
(C)
(0)

(A) 1.0 L

(B) 1.7 L
(C) 2.7 L
(0) 3.0 L
(E) 5.0 L

A
B
C
0

109. The graph below shows the diurnal variation


in the plasma concentration of which of the following hormones?

105. What is this patient's RV?


(A) 0.7
(B) 1.0
(C) 1.7
(0) 2.7
(E) 3.0

107. Mitral valve opening is represented by which


Point on the curve?

L
L

I
I

I
I

I
I

I
I

I
I

106. What is this patient's TLC?

I
I
I
I

(A) 1.7 L

I
I

(B) 2.7 L

(C) 3.0 L

(E) 5.7 L

I
lI

I
I

:i~'------24
I

hrs,------Time (hrs)

Questions 107-108
The following two questions refer to the pressurevolume loop below.

150

:::l

en
en
!!? Cl
Q.I
E
75
S()

..:

i-Sleep (8 hrs)-..
!.---Awake,---- :

(0) 5.0 L

!!?

.s

CI>

>

0
0

50

100

Ventricular volume (ml)

150

(A) Thyroxine
(B) Insulin
(C) Testosterone
(0) Cortisol
(E) Estradiol

:
I
I

Questions 110-112
The following three questions refer to the diagram below.

Tracing A

'-____;---~----------C------------~~,

EKG

110. What does tracing A represent?

112. The duration of interval C is decreased by

(A) A ventricular pressure pulse


(B) A ventricular volume curve
(C) An atrial pressure pulse
(0) An atrial volume pulse
(E) An aortic pressure pulse

(A) vagus stimulation


(B) baroreceptor stimulation
(C) arteriolar constriction
(0) increasing heart rate
(E) decreasing venous pressure

111. Point B occurs in what phase of the cardiac

'iycle ?
(A) Atrial contraction
(B) Isovolumic contraction
(C) Rapid ejection
(0) Reduced ejection
(E) Rapid filling

Questions 113-115

Questions 116-118

A 50-year-old male patient who has smoked two


packs of cigarettes a day for 35 years complains of
shortness of breath, chronic cough, and production
of yellowish, foul-smelling sputum. He has clubbing of his finger nails, and the nail beds and lips
are noted to be cyanotic. Arterial blood gas analysis
of this patient reveals the following: Paz = 55 mm
Hg, Peo z = 56 mm Hg, HCO l - = 35 mEq/L, and
pH = 7.4.

The following arterial blood data are obtained from


a patient who is cyanotic at rest: Paz = 60 mm Hg,
hemoglobin saturation = 85 0/0, Peo z = 40 mm Hg,
pH = 7.39, and hemoglobin concentration = 18
g/di. Arterial Paz rises to 295 mm Hg after the
patient breathes 100% Oz at sea level for 20 minutes. Cardiac catheterization reveals normal pressures and a Paz of 40 mm Hg in the right atrium,
right ventricle, and pulmonary artery while the patient breathes 100% Oz.

11 3. These blood gas data are most consistent


with
(A)
(B)
(C)
(D)
(E)

acute respiratory failure


inadequate alveolar ventilation
anatomic shunt
anemia
carbon monoxide poisoning

114. Which of the following pathophysiologic


phenomena most likely initiated this patient's syndrome?
(A) Decreased alveolar ventilation followed by increased work of breathing
(B) Bronchial narrowing due to inflammation and
edema followed by pulmonary hypertension
(c) Bronchial narrowing due to inflammation and
edema followed by increased work of
breathing
(D) Hypercapnia followed by pulmonary hypertension
(E) Pulmonary hypertension followed by hypercapnia
115. The acid-base status of this patient's blood is
best categorized as
(A)
(B)
(C)
(D)
(E)

respiratory acidosis, uncompensated


metabolic acidosis, compensated
respiratory acidosis, compensated
lactic acidosis secondary to hypoxia
respiratory alkalosis

116. The most likely cause of hypoxia in this patient is


(A) a ventilation:perfusion abnormality (physiologic shunt)
(B) a right-to-Ieft anatomic shunt
(C) a left-to-right anatomic shunt
(D) a diffusion defect
(E) hypoventilation
117. The fraction of the cardiac output that represents shunted blood is
(A) 0.2
(6) 0.3
(C) 0.4
(D) 0.5
(E) 0.6

118. The most useful data for locating the site of


the shunt would be
(A) Paz in the left ventricle and left atrium
(6) airway resistance
(C) right ventricular and pulmonary arterial pressures
(D) left ventricular pressures
(E) compliance of the lungs

Questions 119-121
A lightly anesthetized male patient who is breathing
spontaneously has a cardiac output of 6 L/min, a
heart rate of 75 beatslmin, an O 2 consumption of
240 mllmin, and a mixed venous P0 2 of 40 mm
Hg. He is ventilated for 10 minutes at his normal
tidal volume but at twice the normal frequency
with a gas mixture that is 20% O 2 and 80% N 2
The airway pressure at the end of inspiration is 5
em H 20. On cessation of artificial ventilation, the
patient does not breathe for 1 minute.

119. The most important factor responsible for the


temporary apnea in this patient is reduced activity
of the
(A) peripheral chemoreceptors due to the high P0 2
(B) peripheral chemoreceptors due to the low
Pco 2
(C) pulmonary stretch receptors that inhibit inspiration
(D) medullary chemoreceptors due to the low
Pco 2
(E) medullary chemoreceptors due to the high P0 2

120. During the artificial ventilation, the mixed venous P0 2 in this patient would be
(A) equal to 40 mm Hg because the metabolic rate
was unchanged
(B) greater than 40 mm Hg because the hyperventilation would cause vasoconstriction and less
gas exchange with the tissues
(C) greater than 40 mm Hg because of the significant increase in Pso of hemoglobin.
(D) less than 40 mm Hg because of the hypoxia
that would occur from breathing this gas
mixture
(E) less than 40 mm Hg because of a decrease in
cardiac output

121. During the period of spontaneous breathing,


this patient had an arteriovenous O 2 difference of
(A)
(B)
(C)
(D)

4 mlldl
5 mlldl
6 mlldl
40 mlldl
(E) 80 mlldl

Directions: Each group of items in this section consists of lettered options followed by a set of numbered
items. For each item, select the one lettered option that is most closely associated with it. Each lettered
option may be selected once, more than once, or not at all.
Questions 122-126
Match each disorder with its associated acid-base
state.
(A)
(B)
(C)
(D)

Metabolic acidosis
Respiratory acidosis
Both
Neither

122.
123.
124.
125.
126.

Hypoventilation
Increased [total CO 2 ]
Decreased [total CO 2 ]
Decreased Pco 2
Increased [HCO l -liS x Pco 2 ratio

Questions 127-132

Questions 133-136

Match each characteristic with the appropriate


substance.

Each of the mechanisms described below is used


to alter the force of contraction in certain types of
muscle. For each mechanism, select the muscle
type or types that alter their contractile force in the
manner described.

(A) Aldosterone
(B) AOH
(C) Atrial natriuretic factor
(0) Renin
(E) Angiotensin I

1 27. An octapeptide
128. Regulates plasma [Na +1
129. Controls body Na+ content
130. Secreted by the zona glomerulosa of the adrenal cortex
1 31. Substrate for dipeptidyl carboxypeptidase
132. Produced by modified smooth muscle cells

(A)
(B)
(C)
(0)
( E)

Cardiac muscle
Skeletal muscle
Smooth and cardiac muscle
Smooth and skeletal muscle
Smooth, cardiac, and skeletal muscle

133. Alteration in the amount of Ca 2 + released


from the sarcoplasmic reticulum
134. Recruitment of additional muscle fibers
135. Summation of contractions
136. Alteration in preload

Questions 137-140
For each patient described below, select the set of arterial blood values that coincides with that patient's
acid-base disorder.

Patient
(A)
(B)
(C)
(0)
(E)

137.
138.
139.
140.

Patient
Patient
Patient
Patient

with
with
with
with

Peo 2
(mm Hg)

[HC0 3 - ]
(mmol/L)

pH

[H+]
(nmol/L)

25
30
40
45
55

15.0
16.0
25.0
35.0
27.5

7.40
7.31
7.41
7.51
7.37

40.0
49.0
38.9
30.9
42.7

metabolic alkalosis and partial respiratory compensation


metabolic acidosis and partial respiratory compensation
highest CO 2 content
highest alveolar ventilation

Questions 141-143

Questions 148-150

For each of the following nutrients, choose the


substance that is most important for its absorption
from the intestine.

Match each hormone imbalance with its effect on


plasma ion concentrations.

(A)
(B)
(C)
(D)
(E)

Intrinsic factor
Ferritin
Bile salts
Vitamin D
Trypsin

141. Iron
142. Ca H
143. Cholesterol

Questions 144-147
Match each site of secretory activity described below with the appropriate lettered region of the
nephron.

)(

148. Excessive PTH secretion


149. Deficient PTH secretion
150. Vitamin D intoxication

Questions 151-154
Match each motor deficit listed below with the
component of the motor control system that is affected.
(A)
(B)
(C)
(D)

Motor cortex
Subthalamic nucleus
Posterior parietal lobe
Vestibular nucleus

151.
152.
153.
154.

Q)

0
()

144.
145.
146.
147.

(A) Low plasma inorganic [HPO/-]


(B) Low plasma [Ca H ]
(C) Both
(D) Neither

Primary site of H+ secretion


Site of K+ secretion
Primary site of NH4 + secretion
Site of PAH secretion

Spontaneous movements
Spasticity
Apraxia
Nystagmus

Questions 155-159
Match each person described below with the set of blood data that best coincides with that person's
condition.
Arterial P0 2
(mm Hg)

Arterial Peo 2
(mm Hg)

98
50
50
105
50

40
65
40
35
32

(A)

(B)
(C)
(0)
(E)

155.
156.
157.
158.
159.

A
A
A
A
A

O 2 Content
(ml/dl)
arterial
arterial
arterial
venous
arterial

=
=
=
=
=

Arterial
pH
7.40
7.32
7.38
7.45
7.45

10
15
16
18
10

normal 40-year-old man who has been mountain climbing for two days
30-year-old anemic woman
73-year-old man who is hypoventilating
45-year-old woman who is hyperventilating
56-year-old man with moderately severe obstructive lung disease

Questions 160-164
The following information was obtained from a healthy 24-year-old man who was studied in a renal
laboratory:
Inulin
Concentration (mg/ml)

Glucose
Concentration

Urine
Flow Rate

Hematocrit
Ratio

Urine = 150
Renal arterial plasma = 1.50
Renal venous plasma = 1 .20

Urine = 1 mg/ml
Plasma = 90 mg/dl

1.2 ml/min

0.40

Using the above data, match each of the following measurements of renal function with the appropriate
lettered value.
(A) 0.20
(B) 108 mg/min
(e) 120 ml/min
(0) 600 ml/min
(E) 1000 ml/min
160.
161.
162.
163.
164.

Glomerular filtration rate


Renal blood flow
Filtration fraction
Renal plasma flow
Filtered load of glucose

Questions 165-167

Questions 172-175

Match each cause of an acid-base disturbance with


the characteristic set of body fluid changes.

Match each of the clinical conditions listed below


with the digestive system malfunction most likely
to cause the condition.

pH

Plasma [He0 3 -]
(mEq/L)

7.27
7.31
7.40
7.40
7.55

37
16
15
24
22

Plasma
(A)
(B)
(C)
(D)
(E)

Urine pH
acid
acid
alkaline
acid
alkaline

165. Hyperventilation
166. Chronic respiratory tract obstruction
1 67. Diabetic ketoacidosis

Questions 168-171
Match each visual defect described below with the
condition that causes it.
(A) Cataracts
(B) Astigmatism
(C) Presbyopia
(D) Myopia
(E) Hyperopia

168. A progressive decrease in the power of accommodation


169. A progressive loss of lens transparency
170. An optical defect that requires a diverging
lens to see distant objects clearly
171. An optical defect that requires continuous
accommodation to see distant objects clearly

(A)
(B)
(C)
(D)

Blockage of the bile duct


Malabsorption of vitamin B12
Malabsorption of water
Malabsorption of carbohydrates

172.
173.
174.
175.

Circulatory collapse
Malnutrition and steatorrhea
Jaundice
Anemia

ANSWERS AND EXPLANATIONS


1. The answer is D. [Ch 3 II 0 1] The interpleural pressure during the expiratory maneuver would be
15 cm H 20. Before the Valsalva maneuver, the transmural pressure across the lungs (i.e., transpulmonary
pressure) is equal to the alveolar pressure minus interpleural pressure, or 0 - (-5) = 5 cm H 20. Since
lung volume remained constant, the transmural pressure must also remain constant during the Valsalva
maneuver, so that when the alveolar pressure increases to 20 mm Hg, the interpleural pressure must
equal 15 in order to maintain the difference of 5 cm H 20 between the inside and outside of the lungs.

2. The answer is D. [Ch 1 VIII A 5 a] In myopia, the focal plane of the unaccommodated eye is in front
of the retina because the axial length of the eye is too long for the refractive power of the eye. To be
seen clearly without accommodation, an object must be placed close to the eye (i.e., the far point is less
than normal). However, if a diverging lens is placed in front of the eye, distant objects can be brought
into focus on the retina.
3. The answer is D. [Ch 4 II A 1-3; Figures 4-3,4-4; Table 4-1] Assuming complete dissociation, this
0.15 mol/L solution of sodium chloride (NaCI) is equivalent to a solution of 300 mOsm/L (0.15 mol/L
of Na+ + 0.15 mol/L of CI-). Because this NaCi solution is osmotically balanced with body fluids, there
is no shift of water between the major fluid compartments. Thus, the extracellular fluid (ECF) volume
increases with no change in the osmolar concentration of the ECF or intracellular fluid (ICF) compartment.
4. The answer is D. [Ch 2 VIII 0 1] The data obtained during cardiac catheterization of this patient can
be used to calculate the cardiac output (Q) using the Fick equation, which states that
.

O 2 consumption
arterial O 2 content - mixed venous O 2 content
Mixed venous blood (from the right ventricle or pulmonary artery) must be used to calculate cardiac
output. For this calculation, as in all others, the units must be consistent. So, using milliliters, the cardiac
output is calculated as

210 ml/min
= -------------

0.18 ml 02/ml blood - 0.11 ml 02/ml blood

210/0.07

3000 ml/min

The normal cardiac output in a man is approximately 5 L/min (5000 ml/min); thus, a cardiac output of
3000 ml/min is not dangerously high. The amount of O 2 transferred to the tissues is given by the
difference between arterial and venous O 2 content and, thus, is 7 ml 02/dl blood, not 29 ml. The arterial
O 2 content is equal to or only slightly less than the pulmonary venous O 2 content (due to thebesian
venous flow into the left ventricle), meaning that the pulmonary venous O 2 content would not be 145
ml/d\. Finally, stroke volume equals cardiac output/heart rate; thus, the stroke volume in this patient is
3000/75 = 40 ml, which is less than the normal stroke volume of 70 ml and the likely cause of the
subnormal cardiac output in this patient.
5. The answer is D. [Ch 7 X FIe (8)] During the postovulatory phase of the menstrual cycle, the
thickened, secretory endometrium depends on the continued presence of estradiol and progesterone. If
fertilization does not occur, the corpus luteum remains functional for 13-14 days and then undergoes
regression (Iuteolysis). After the regressing corpus luteum loses its ability to produce adequate amounts
of estradiol and progesterone, the innermost layer (adluminal, or stratum functionale, layer) of the
endometrium becomes ischemic, degenerates, becomes necrotic, and is sloughed into the uterine cavity.
The loss of proliferated endometrium (stratum functionale) is accompanied by bleeding known as menstruation (menses). The discontinuation of oral contraceptives after 3 weeks also permits menstruation.
6. The answer is D. [Ch 6 11/ 8 6] Gastric motility is controlled primarily by enterogastric reflexes that
are elicited when chyme enters the small intestine. The reflexes are both neural and hormonal and act
to inhibit gastric contractions. They prevent food from entering the intestine too rapidly. Although
distension of the antrum will elicit neuronal and hormonal excitatory reflexes, these reflexes are overcome
by the inhibitory enterogastric reflexes.
7. The answer is C. [Ch 4 IX F 1] Spironolactone is a competitive aldosterone antagonist that interferes
with the aldosterone-stimulated Na+ reabsorption in the distal tubular cell and in the collecting duct.
Inhibition of Na+ reabsorption is associated with a marked decrease in urinary excretion of K+ and H+.

Spironolactone is effective as a diuretic in normal subjects or in patients on a low-Na+ diet, but not in
adrenalectomized patients.
8. The answer is A. [Ch 2 VII C 1] Of the conditions listed, only an increase in heart rate would cause
an increase in aortic systolic pressure and a decrease in aortic pulse pressure. This can be verified using
a derivation of Hook's law, which states that arterial pressure is determined by the interaction of mean
arterial volume (V), pulse pressure (dP), arterial uptake during systole (dV), and the elasticity of the arterial
system (a constant represented by E), or E = dP x V IdV. An increase in heart rate increases cardiac
output and raises V, which is proportional to mean aortic pressure. If stroke volume remains constant,
then dV will not change significantly; so that dP must decrease to maintain a valid equation. An increase
in arterial compliance would decrease the value of E (compliance is the reciprocal of elastance), which
would lower systolic pressure. A decrease in peripheral resistance would decrease systolic pressure and
increase pulse pressure. An increase in either stroke volume or the elastic modulus would increase
systolic pressure but also increase pulse pressure.
9. The answer is B. [Ch 7 VII H 1 c (1), I 2] Pheochromocytoma is an adrenomedullary tumor
characterized by the hypersecretion of catecholamines, usually norepinephrine. Catecholamines block
insulin release (hypoinsulinemia) and increase both gluconeogenesis and fatty acid mobilization, which
account for glucose intolerance, fasting hyperglycemia, and glycosuria. The breakdown of muscle glycogen leads to elevated plasma pyruvate or lactate levels. The metabolic features of pheochromocytoma
resemble hyperthyroidism and include tremor, weight loss, heat intolerance, and increased basal metabolic rate. The cardinal sign is hypertension, which may be persistent or paroxysmal. Patients usually
complain of attacks that may be precipitated by emotion or physical exercise, which consist of pounding
headaches, sweating, pallor, pain or "tightness" in the chest, apprehension, paresthesia, nausea, and
vomiting. In this patient, the attacks were provoked by mechanical pressure that was exerted on the
tumor by changes in body position.

10. The answer is B. [Ch 3 II F 1; IX C 1 c] This patient has obstructive lung disease-probably
emphysema-as indicated by the high airway resistance and the decreased diffusion capacity of the
lungs. The minute ventilation is decreased probably because of the high respiratory work load, which
would increase arterial Peo 2. The pulmonary artery pressure would be increased due to the destruction
of pulmonary capillaries secondary to the emphysema and to the pulmonary vasoconstriction caused
by the hypoxia and hypercapnia. Arterial pH would tend to be decreased due to the hypercapnia, and
the [HC0 3 -]/[H 2C0 3 ] ratio would be decreased in accordance with the Henderson-Hasselbalch equation [see Ch 5 II D].
11. The answer is C. [Ch 6 IV 03 a (1), F 5 c] Vitamin B12 is absorbed in the terminal ileum in
association with intrinsic factor. Bile salts are also absorbed in the terminal ileum. If bile salts cannot be
absorbed and recirculated, lipid absorption will be compromised because the liver is unable to synthesize
enough bile to absorb all the fats ingested during a meal.

12. The answer is D. [Ch 4 VI C 2 h, 4; Figure 5-3] The primary effect of carbonic anhydrase inhibitors
such as acetazolamide is to inhibit both H+ secretion and NaHC0 3 reabsorption, making the urine
alkaline. NH4 + excretion is reduced as a result of the diminished H+ secretion. Carbonic anhydrase
inhibitors restrict H+ secretion by inhibiting the intracellular hydration of CO 2 , a primary source of
intracellular H+. The decline in H+ secretion inhibits the Na+-H+ exchange at the luminal membrane
of the proximal tubule, which is the primary site of NaHC0 3 reabsorption. HC0 3 - reabsorption is also
inhibited because only a limited amount of HC0 3 - can be reabsorbed by the distal segment. The
elevated intraluminal HC0 3 - augments Na+ and K+ excretion and results in NaHC0 3 diuresis. Carbonic
anhydrase inhibitors also block the dehydration of H 2 C0 3 formed in the tubular lumen. Chronic doses
of such drugs can lead to hyperchloremic acidosis (metabolic acidosis).
13. The answer is C. [Ch 4 VIII B 1-2, C 1] A decrease in plasma osmolality (osmolarity) leads to a
decrease in antidiuretic hormone (ADH) secretion. This results in an increase in free-water clearance
and diuresis. These osmoreceptors are found in the vicinity of the supraoptic nucleus of the hypothalamus.
ADH augments the water permeability of the cortical collecting duct and the water and urea permeabilities of the medullary collecting duct. It increases renal water reabsorption, resulting in the excretion of
a small volume of hypertonic urine. The major stimuli for ADH are an increase in the plasma osmolality
and a decrease in the effective circulating blood volume.

14. The answer is C. [Ch 2 VIII 0 1] This patient's O 2 consumption is 520 ml/min/m2. O 2 consumption
can be determined using the Fick principle, an important concept that has wide applicability in physiology
and medicine. To solve for O 2 consumption (V0 2), the Fick principle is expressed as
V0 2 = Q x (Cao 2 - CV02)
where Q = blood flow, and Cao 2 - Cvo 2 = the arteriovenous O 2 difference. In this problem, Q is given,
and, since arterial hemoglobin saturation is assumed to be 100%, Cao2 equals O 2 capacity, which equals
hemoglobin x 1.34, or 16 mlldl (160 milL). Using these values in the above formula gives

v0

2 =

6.5 x (160 - 80)


520 mllmin

It is important to use similar units for blood flow and O 2 content. Also, converting O 2 content to mill
before calculating is recommended.

15. The answer is C. [Ch 1 I C 3 d-f] The initial shrinkage of the red blood cell results from a higher
osmotic pressure in the solution than within the cell. However, since the red blood cell eventually returns
to its initial volume, the solution must be isotonic. The particles producing the higher osmotic pressure
in the solution have reflection coefficients of less than 1 and, thus, are able to diffuse across the
membrane. Although these particles initially cause water to flow out of the cell, they eventually reach
diffusional equilibrium across the cell membrane and, thus, are unable to keep the water from returning
to the cell.

16. The answer is B. [Ch 5 VIII 03, E 1; Figures 5-13, 5-14] In respiratory acidosis, there is a
primary increase in plasma Peo 2. The renal compensation for respiratory acidosis is increased HC03reabsorption, which increases the plasma [HC0 3-]. With partial compensation, the pH would not return
to normal. Only one set of data (8) indicates hypercapnia with a decrease in pH. Note that the pH and
the [HC0 3-]IS x Peo 2 ratio are below normal in this patient, which is consistent with acidosis.

17. The answer is D. [Ch 7 VI 0 1 c (4) (b)] This woman's inability to lactate and to menstruate
following parturition stems from pituitary failure related to the massive bleeding she experienced during
delivery. Most likely, this woman has postpartum pituitary necrosis (Sheehan's syndrome) due to ischemia
of the hypophysial portal system, which supplies 90% of the blood to the anterior pituitary gland. The
resultant pituitary failure may be complete (panhypopituitarism) or partial. In this case, the acidophils
responsible for production of prolactin, follicle-stimulating hormone (FSH), and luteinizing hormone (lH)
were affected, resulting in this patient's inability to lactate and to menstruate. This patient also would
exhibit hypoglycemia due to low plasma growth hormone (GH) levels and to low plasma adrenocorticotropic hormone (ACTH) levels, which would lead to decreased secretion of cortisol (a potent hyperglycemic hormone) and decreased synthesis of epinephrine (a hyperglycemic hormone that depends on
cortisol). The decreased levels of these hyperglycemic hormones would cause insulin sensitivity.
Since aldosterone secretion does not depend mainly on ACTH secretion, this patient should be able
to regulate Na+ balance. She also should have normal ADH activity, since the supraoptic nuclei and
pars nervosa are not perfused by the hypophysial portal system. Thus, this patient should demonstrate
normal water balance.

18. The answer is D. [Ch 2 III A 5, 83 d] The largest deflection in the EKG occurs when the electrical
vector is parallel to the lead axis. lead aVl lies at _30 and, therefore, should exhibit the largest positive
deflection of any lead.

19. The answer is C. [Ch 4 VI C 4 c; Table 4-6] The kidney excretes H+ as NH4 + and titratable acid.
Titratable acid exists mainly in the form of H 2P0 4-. The titratable acid is excreted mainly as NaH 2P0 4,
which is also called acid phosphate, monosodium phosphate, or monobasic phosphate. The normal
kidney excretes almost twice as much acid combined with NH3 than it excretes titratable acid. The rate
of NH4 + excretion increases during metabolic acidosis.

20. The answer is A. [Ch 5 VIII 0 4, E 2; Figure 5-14] The variable that shows the greatest degree of
change in this patient is arterial Peo 2, which is decreased by 75% compared to a 46% decrease in
[HC0 3-]. These findings are consistent with respiratory alkalosis, which, in this case, is due to hyperventilation brought on by salicylate toxicity. The resultant hypocapnia reduces tubular H+ secretion, so that

HC0 3 - reabsorption is attenuated, causing a compensatory loss of urinary HC0 3 - . The arterial pH of
this patient can be calculated using the Henderson-Hasselbalch equation as
pH

[HC0 3 - ]
pK + l o g - - S X Peo 2

13 mmol/L
6.1 + l o g - - - 0.3 mmol/L

=
=

6.1 + log 43.3


6.1 + 1.6
7.7

Note that the increased ratio of [HC0 3 -]IS x Peo 2 is consistent with alkalotic states. This condition
must be differentiated from metabolic alkalosis, which is associated with increases in both arterial
[HC0 3 - ] and Peo 2 Metabolic acidosis is associated with decreases in [HC0 3 -], Peo 2 , and pH.

21. The answer is E. [Ch 6 VB 1, 3] Although acidosis can result from loss of fluid from both the
intestine and the colon, only the colon secretes K+. Thus, excessive fluid loss from the colon will result
in both acidosis and hypokalemia.
22. The answer is B. [Ch 1 XI B 1, 2] The Ib afferent fibers convey information from the Golgi tendon
organs, which are stimulated by an increase in muscle tension. Thus, when the muscle contracts in
response to a stretch reflex, Ib afferent activity increases. la afferent activity decreases during the
contraction because its receptor, the muscle spindle, is unloaded (i.e., becomes slack) during contraction.
23. The answer is A. [Ch 1 11/ B 2 b; Ch 2 " C 1 b, d] The pacemaker cells of the heart contain
muscarinic receptors, which are activated by acetylcholine (ACh). When the ACh receptor on these cells
is activated, K+ conductance is increased, which causes the membrane to hyperpolarize and slows
pacemaker activity.
24. The answer is C. [Ch 3 IV C 3] O 2 delivery to the tissues is the product of arterial O 2 content and
cardiac output, which normally equals 1 L/min. Of the factors listed, only a 50% reduction in normal
hemoglobin concentration would reduce O 2 delivery by half. This would produce a proportional change
in O 2 content and an equivalent change in the O 2 delivery. Due to the nonlinear relationship between
P0 2 and hemoglobin saturation, changing the Po 2 , ventilation rate, or inspired P0 2 would have a relatively
small effect on O 2 delivery.
25. The answer is B. [Ch 5 VII/ D 2; Figure 5-13; Table 5-8] The severe loss of gastric fluid in this
patient would produce metabolic alkalosis due to the loss of HCI (a noncarbonic acid). She also would
exhibit hypovolemia due to the fluid loss. The development of alkalosis is sensed by the chemoreceptors
controlling ventilation, resulting in hypoventilation, and an increase in the arterial Peo 2 (hypercapnia),
which reduces the pH toward normal. The kidneys would be expected to excrete the excess HC0 3 - ,
raising the urinary pH.
26. The answer is A. [Ch 6 11/ C 2 b] The formation of HCI by the parietal cells is a two-step process.
First, CI- is transported into the parietal cell canaliculi. The negative potential developed by the flow of
CI- allows K+ to flow into the canaliculi. The K+ is then actively transported out of the canaliculi in
exchange for H+. Since both K+ and H+ are transported against their concentration gradients, an active
transport system (H+-K+ ATPase) is required.
27. The answer is A. [Ch 1 "C 2, 4] During attacks of hyperkalemic periodic paralysis, large amounts
of K+ are released from skeletal muscle fibers, causing a rise in extracellular K+ concentration. When
the extracellular K+ concentration increases, the membrane potential depolarizes, causing inactivation
of the Na+ channels.
28. The answer is B. [Ch 7 X E 3 a-c, F 1; Figure 7-9] The menstrual cycle consists of four phases.
These phases (with synonymous terms) are: menses (4-5 days), the preovulatory phase (also called
follicular, estrogenic, or proliferative phase; 10-12 days), the ovulatory phase, and the postovulatory
phase (also called luteal, progestational, or secretory phase; 14 days).

29. The answer is C. [Ch 6 11/ C 4, E; IV E 2 b (1)] A significant amount of protein digestion occurs in
the stomach because of the action of HCI, which begins to break proteins apart, and pepsin, a protease
enzyme secreted by gastric chief cells. These protein digestion products then act as secretagogues that
stimulate the secretion of pancreatic proteases.

30. The answer is D. [Ch 4 V C 1 b; Table 4-5] The simplest flow (F) equation that summarizes the
relationship between hydrostatic pressure gradient (~P) and resistance (R) is
~P

F =R
The pressure gradient is the hydrostatic pressure difference between the renal artery and renal vein. The
resistance to blood flow can be determined by dividing this hydrostatic pressure gradient by the renal
blood flow (RBF) as
~P

R=F
~P

RBF

31. The answer is C. [Ch 5 /I 0 3 b (1), E 2 b] From the data given, this patient's arterial Peo2 is
determined to be 50 mm Hg (normal = 40 mm Hg). The Henderson equation is used to estimate Peo 2 ,
but [H+] must be determined first using the Henderson-Hasselbalch equation. Given a pH of 7.5, [H+]
is calculated as
[H+] = antilog (9 - pH)
antilog (9 - 7.5)
= antilog 1.5 = 31.6 nmol/ L

Substituting to solve for Peo 2 ,


Peo 2

[H+] [HC03 -]

= ------

24

(31.6 nmol/L) (38 mmol/U

24
=

50 mm Hg

This patient's increased [HC0 3 -] indicates a metabolic alkalosis. The elevated [HC0 3 -] suppresses
respiratory drive, leading to compensatory elevation of Peo 2 Note that unit analysis cannot be used in
this equation.

32. The answer is C. [Ch 1 /I C 5; 11/ A 3] The flow of Ca2+ into the presynaptic nerve terminal
stimulates the release of neurotransmitter from synaptic vesicles by exocytosis. Ca2+ enters the nerve
terminal down its electrochemical gradient through channels opened by membrane depolarization.
Propagation of an action potential is accomplished by generating new action potentials along the nerve
fiber. The generation of action potentials is dependent on electrically excitable Na + and K+ channels.

33. The answer is A. [Ch 3 11/ C 3; IX C 1; Table 3-2] This patient has an alveolar-to-arterial P02
difference of 35 mm Hg. To calculate this, the alveolar P0 2 must be determined using the alveolar gas
equation. Given an arterial Peo 2 of 30 mm Hg, the alveolar P0 2 is determined as
alveolar P0 2

=
=

(760 - 47) x .21 - 30


150 - 30 = 120 mm Hg

Thus, the alveolar-to-arterial P0 2 difference is 120 - 85 = 35 mm Hg. This patient's low arterial Peo 2
and high pH indicate that he has respiratory alkalosis; thus, he has been hyperventilating and does not
have metabolic alkalosis. Respiratory alkalosis causes a decrease in plasma [HC0 3 -] due to the slope
of the blood buffer line. Lastly, chronic obstructive lung disease leads to hypoxia and CO 2 retention, not
hyperventi la tion.
34. The answer is D. [Ch 4 V A 1 a] The oncotic and hydrostatic pressures within the capillary and
within the interstitium contribute to the regulation of fluid exchange between the plasma and the

interstitial fluid (or Bowman's space). To determine whether there is net reabsorption or filtration, it is
necessary to compare the magnitude and direction of these two different pressures as
Outward Forces (mm Hg)
Capillary hydrostatic pressure
Bowman's space oncotic pressure

Inward Forces (mm Hg)

47

Bowman's space hydrostatic pressure


Capillary oncotic pressure

47

10
28
38

Because the outwardly directed forces exceed the inwardly directed forces, there will be a net filtration
force of 9 mm Hg. Note that the oncotic pressure in Bowman's capsule is assigned a value of zero
because the fluid in this region is an ultrafiltrate of plasma.
35. The answer is E. [Ch 4 II C 3 a; Figure 4-4; Table 4-2] As a result of the hemorrhage, this patient
has undergone the loss of water and electrolytes in isotonic concentration leading to dehydration
(reduced extracellular fluid volume). In this patient there will be no net water movement across the cell
membranes because both NaCI and water are administered as 1 L of isotonic (isosmotic) NaCI. Since
the administered NaCi will remain initially in the extracellular space, the only effects will be alL increase
in the ECF volume and a dilution of the plasma protein concentration, the latter effect accounting for
the decrease in the plasma colloid osmotic pressure.
A 0.9% NaCi solution is isotonic because it maintains the normal red blood cell volume. Furthermore,
this 0.9% NaCi solution contains dissociated Na+ and CI-, each in a concentration of 150 mEq/L.
Because NaCI is an electrolyte, this same solution is equivalent to a concentration of 300 mOsm/L,
assuming 100% dissociation of NaCI. Thus, alSO mmol/L NaCi solution is isotonic and isosmotic to
human plasma. Edema is ruled out in this patient because it represents the retention of both water and
NaCi as an isotonic solution, resulting in isotonic overhydration. The infusion of a large volume of isotonic
saline at a rate that exceeds urinary excretion results in edema.
36. The answer is D. [Ch 3 11/ B 3 b] The alveolar Pca 2 is directly proportional to the rate at which
CO 2 is produced by metabolism and inversely proportional to alveolar ventilation. The increased minute
ventilation would not alter the metabolic rate significantly, so any changes in Pea 2 must be related to
alveolar ventilation. Alveolar ventilation (VA) is the difference between minute ventilation and dead space
ventilation. During control conditions, alveolar ventilation is (0.5 L x 15) - (0.2 L x 15) = 4.5 L/min,
which is unchanged by the alteration in respiratory pattern [30 x (0.35 L - 0.2 U]. An increase in alveolar
ventilation would cause a decrease in Pea 2 , which would lead to respiratory alkalosis (i.e., increased
arterial pH).
37. The answer is B. [Ch 7 VII 0 3 b (2)] Norepinephrine can be converted to epinephrine by
methylation with the epinephrine-forming enzyme, phenylethanolamine-N-methyltransferase (PNMT),
which is highly localized in the cytosol of the adrenomedullary chromaffin cells. The methyl group donor
in this reaction is S-adenosylmethionine. Very high local concentrations of cortisol from the adrenal
cortex reach the adrenomedullary chromaffin cells via the adrenal portal system. PNMT is inducible
by glucocorticoids. The secretion of adrenomedullary catecholamines is stimulated by ACh from the
preganglionic nerve endings, which innervate the chromaffin cells. Thus, the adrenal medulla is a
functional extension of the nervous system. The synthesis of epinephrine in the adrenal medulla depends
on PNMT, cortisol, corticotropin (ACTH), and corticotropin-releasing hormone.
38. The answer is B. [Ch 2 11/ C 2 a, b] A positive EKG complex recorded from a unipolar electrode
indicates that the electrical vector is directed toward the electrode. Positive QRS complexes in aVR and
aVF indicate that the vector must lie in the right axis deviation quadrant (i.e., between + 120 and + 180).
No axis deviation or a mean electrical axis between 0 and 90 indicates that the vector would lie in the
normal quadrant, whereas left axis deviation or left ventricular hypertrophy would have the vector in
the left axis deviation quadrant (i.e., between _30 and _90).
39. The answer is B. [Ch 5 II E 2 b] From the data given, this patient's arterial [HC0 3 -] is determined
to be 14.7 mmol/L (mEq/U. Arterial [HC0 3 - ] is easily estimated using the Henderson equation, which
is stated as
[HC0 3 -]

PC0 2

24 [H+]

where [HC0 3 -] is expressed in mmol/L, [H+] in nmol/L, and Pea 2 in mm Hg.

Substituting,

[ HCO -]

30
2449

14.7 mmol/L

The acid-base disturbance in this case is an almost completely compensated metabolic acidosis (pH
7.32).
40. The answer is C. [Ch 3 VI A T] The lung is a passive structure whose transmural pressure varies as
a function of volume and phase of respiration. At residual volume, the transpulmonary pressure may be
negative at the base of the lung, since interpleural pressure could be positive due to the hydrostatic
effects of lung weight and the decreased retractile force at low lung volume. The transpulmonary pressure
always is greater at the apex than at the base when standing because of the hydrostatic effects of gravity.
Thus, the airways and alveoli at the apex of the lung always are more distended than those at the base
of the lung of a person who is standing.

41. The answer is C. [Ch 6111 B 2] Receptive relaxation occurs when food enters the stomach. Although
a small amount of receptive relaxation occurs as part of the esophageal reflex, the relaxation of the orad
stomach necessary to accommodate the food entering it during a meal is brought about by a vago-vagal
reflex initiated by the presence of food in the stomach. Chewing is entirely dependent on a nonvagal
reflex involving stretch receptors and motor efferents in the jaw muscles. Swallowing is coordinated by
a swallowing center in the brain stem and does not involve vagal reflexes. Although gastric emptying is
modified to some extent by vago-vagal reflexes responding to gastric distension and the presence of
chyme in the intestine, local reflexes and hormones are primarily responsible for regulating the rate of
gastric emptying. Intestinal contractions are controlled almost entirely by local reflexes and the presence
of circulating hormones.
42. The answer is B. [Ch 5 VII B 2 C (2); VIII 0 T, E 3; Figure 5- T4] This diabetic patient has metabolic
acidosis due to ketoacidosis. The ketoacidosis is attributable mainly to the formation of J3-hydroxybutyric
acid from the partial oxidation of fatty acids. The high concentration of ketoacids in the form of anions
is responsible for the increased anion gap; however, it is H+ retention, not anion accumulation, that is
responsible for the acidosis. Osmotic diuresis accounts for this patient's dehydration, which is not only
water loss but also increased renal excretion of Na+, K+, CI-, and glucose. Na+ and K+ also are lost
when they are excreted in association with the excess quantities of organic anions. In this case,
the decline in [HC0 3 -] is the primary change, and the decreased Peo 2 (i.e., hyperventilation) is the
compensatory response. Complete compensation is evidenced by the normal pH of 7.39 and the normal
ratio of [HC0 3 -liS x Peo 2 . It is possible to have an acidosis with a normal blood pH, or [H+], because
secondary changes diminish the extent of the acid-base imbalance.
43. The answer is C. [Ch 7 VII H T c (2)] Epinephrine is a potent hyperglycemic agent for several
reasons. It stimulates a-adrenergic receptors on the pancreatic beta cell, inhibiting insulin secretion and,
therefore, subsequent facilitated transport of glucose by muscle and adipose tissue. Epinephrine also
promotes hepatic and muscle glycogenolysis by activating cyclic adenosine 3',5'-monophosphate
(cAMP)-dependent phosphorylase; glycogenolysis in muscle leads to an increase in the plasma level of
lactate, which provides the liver with an important gluconeogenic substrate. The lipolytic effect of
epinephrine mobilizes free fatty acids, which enhances gluconeogenesis. Additionally, catecholamines
directly inhibit peripheral glucose uptake, partly due to the suppression of glucose transporters.
44. The answer is B. [Ch 7 VI A 2] The pars nervosa is a functional component of the neurohypophysis
(also called the neural lobe of the pituitary gland), which is derived from neural ectoderm. The neurohypophysis consists of three anatomic components: the median eminence, the infundibular stem, and the
pars nervosa. Like the median eminence, the pars nervosa does not synthesize hormones but, rather,
functions as a release center for hormones. The anterior lobe of the pituitary gland, or adenohypophysis,
is derived from oral ectoderm. Endocrine tissues that arise from endoderm include the pancreas, thyroid
gland, and parathyroid glands. Those derived from mesoderm include the adrenal cortex and gonads.
The adrenal medulla originates from the neural crest, as do the parafollicular cells of the thyroid gland.
45. The answer is D. [Ch 3 II 0 3] The man can lie under 68 cm of water. To calculate this depth
requires converting the pressure given in mm Hg to a pressure expressed in cm H 2 0, or 50 mm Hg x
1.36 cm H 2 0/mm Hg = 68 cm H 2 0. This man's alveolar pressure is equal to atmospheric pressure,

while the pressure outside his chest wall is 68 cm H 2 0 higher. To expand his lungs, this man must
generate a pressure slightly more negative than -68 cm H 2 0. Under these conditions, the man is
undergoing negative pressure breathing, which, if prolonged, can lead to pulmonary edema.
46. The answer is A. [Ch 4 II C 2 a (2); Figure 4-4; Ch 5 VIII 0 1 a] Severe diarrhea causes volume
depletion and electrolyte loss, which defines a state of dehydration. It is important to remember that
volume depletion refers to effective circulating blood volume. Since intestinal secretions are rich in K+
and HC0 3 - , diarrhea causes K+ depletion and HC03 - loss, which lead to hypokalemia and metabolic
acidosis, respectively. Vomiting, excessive sweating, and water deprivation can lead to volume depletion
but not acidosis. In fact, vomiting causes loss of high concentrations of gastric H+ and CI-, which leads
to metabolic alkalosis and hypochloremia. Daily sweat production can exceed 10 L in subjects exercising
in a hot climate. Severe sweating is associated with a significant loss of K+, which may contribute to
heat stroke.
47. The answer is C. [Ch 7 XI B 2 b (2)] The corpus luteum is the initial source of plasma progesterone
and 17a-hydroxyprogesterone, which peak at 3-4 weeks postconception. At 6-8 weeks postconception,
the progesterone reaches its lowest point, while the 17a-hydroxyprogesterone continues to decline.
Since the placenta cannot synthesize 17a-hydroxyprogesterone, the secondary rise in progesterone
reflects placental (trophoblast) function, and the 17a-hydroxyprogesterone curve is a correlate of corpus
luteal function of pregnancy. The theca interna and adrenal glands do not secrete progesterone. The
decidua is not the source of any hormones but is a specialized region of the endometrium that develops
into the maternal component of the placenta.
48. The answer is B. [Ch 4 V C 3 b; Table 4-5] The filtration fraction is that fraction of the plasma
flowing through the kidneys that is filtered into Bowman's capsule. Normally about one-fifth of the
plasma entering the 2 million glomerular capillaries is filtered. Filtration fraction is the ratio of glomerular
filtration rate (GFR) [125 ml/min] to renal plasma flow (RPF) [625 ml/min]. Both GFR and RPF are
regulated in parallel at the afferent arteriole (e.g., constriction decreases both) and inversely at the efferent
arteriole (e.g., constriction augments GFR and reduces RPF). As a result, only changes in efferent arteriolar
resistance, not those in afferent arteriolar resistance, affect the ratio of GFR to the RPF. Since fluid
movement across the glomerulus is governed by Starling's forces, it is proportional to the permeability
and surface area of the filtering membrane and to the balance between the hydrostatic and oncotic
forces. Ureteral obstruction results in an increase in the hydrostatic pressure in Bowman's space, reducing
the hydrostatic pressure gradient and, therefore, the GFR and the filtration fraction. An increase in the
plasma oncotic pressure contributes to a decrease in GFR as do decreases in the glomerular capillary
hydrostatic pressure and the glomerular filtration area.
49. The answer is B. [Ch 6 IV F 3 b (4) (e)] Although fats can be absorbed all along the intestine, from
the point at which the bile duct enters the duodenum until the bile salts are absorbed from the terminal
ileum, almost all of the digested lipids are absorbed by the time the chyme reaches the midjejunum.
Little, if any, lipid absorption occurs in the ileum.
50. The answer is E. [Ch 5 VIII 0 4, E 2; Figures 5-13,5-14] The most likely diagnosis is respiratory
alkalosis due to anxiety-induced hyperventilation. The key determinants of the cause and compensation
of the acid-base disorder are the arterial Peo 2 and [HC0 3 -]. The greater reduction in Peo 2 than in
[HC0 3 -] indicates that the primary disturbance is respiratory alkalosis. That both the arterial [HC0 3 -]
and [H+] change in the same direction further supports the diagnosis of a respiratory acid-base imbalance.
Both obstructive and restrictive lung diseases are common causes of respiratory acidosis.

51. The answer is A. [Ch 7 VII 03 b (2)] Hydrocortisone (cortisol) directly increases epinephrineforming enzyme activity. Epinephrine is synthesized in the adrenal medulla and certain brain neurons
from norepinephrine by the action of the enzyme PNMT. The adrenal cortex and adrenal medulla are
related both anatomically and functionally. Venous blood from the sinusoids of the cortex enters the
adrenal portal system and perfuses the adrenal medulla before entering the systemic circulation. Therefore, the chromaffin cells are exposed to a high concentration of cortisol. ACTH indirectly activates the
epinephrine-forming enzyme, because it stimulates the secretion of cortisol.
52. The answer is B. [Ch 3 11/ B 3] The alveolar ventilation in patient B is greater than in patient A.
Alveolar ventilation is minute ventilation minus dead space ventilation or respiratory rate times tidal
volume minus dead space. Thus, patient B has an alveolar ventilation of lOx (400 - 150) = 2500 ml/

min, whereas patient A has an alveolar ventilation of 20 x (200 - 150) = 1000 ml/min. Dead space
ventilation in patient A is 3000 mllmin, whereas in patient 8 it is 1500 mil min.
53. The answer is B. [Ch 5 VII 8 2 c; VIII 0 1, E 3; Figures 5-13,5-14] The decreases in arterial pH,
[HC0 3-], and Peo 2 are consistent with metabolic acidosis. There also is a decreased [HC0 3-]/5 x Peo 2
ratio (i.e., 12.2) and a widened anion gap (i.e., 47 mEq/U. The primary disturbance is the marked
reduction in [HC0 3-], and the compensatory response is hyperventilation (as indicated by the hypocapnia). That the [HC0 3-] decreases and the [H+] increases is evidence of metabolic acid-base imbalance.
The hyperglycemia and dehydration (as evidenced by dry skin) support a diagnosis of diabetes mellitus.
54. The answer is A. [Ch 7 XIV H 3 b] Calcitriol promotes renal tubular reabsorption of both CaH
and HPO/-, leading to hypercalcemia and hyperphosphatemia. Calcitonin has the opposite effects,
promoting the excretion of both electrolytes and, thus, leading to hypocalcemia and hypophosphatemia.
Aldosterone favors increased reabsorption of Na + and increased excretion of K+, leading to hypokalemia
and alkalemia; the [Na+] usually remains within normal limits because of a commensurate increase in
water reabsorption. Progesterone has an anti-aldosterone-like effect, in that it promotes Na+ excretion.
55. The answer is D. [Ch 6 IV F 5 c] Although intrinsic factor is secreted by parietal cells in the stomach,
it is not able to bind vitamin B'2 in the stomach because the vitamin is bound to another protein, called
protein R. When the vitamin reaches the intestine, the R protein is removed, and intrinsic factor is able
to bind to the vitamin B12 The B'2-intrinsic factor complex is absorbed in the terminal ileum.
56. The answer is D. [Ch 3 IV C 1 a] A drop in arterial P0 2 from 100 mm Hg to 27 mm Hg would
decrease the O 2 content of the blood by about 50%. The key to answering this question is to recognize
that 27 mm Hg is the normal value for the PSG, which is the P0 2 at a hemoglobin saturation of 50%.
Since hemoglobin is nearly 100% saturated at a P0 2 of 100 mm Hg, then O 2 content must decrease by
50%.
57. The answer is A. [Ch 7 VIII C 3 e (1)-(3); Figure 7-7] A deficiency of 17a-hydroxylase leads to
reduction in the 17a-hydroxylation of pregnenolone and progesterone, resulting in hypogonadism and
elevated blood gonadotropin levels. This enzyme deficiency also leads to increased production of
ll-deoxycorticosterone (ll-DOC). This mineralocorticoid causes Na+ retention, extracellular volume
expansion, and hypertension--effects that suppress renin and aldosterone secretion. The 17ahydroxylase defect also affects the gonads, preventing testicular and adrenal androgen synthesis in males
and ovarian estrogen synthesis in females and, thus, resulting in a female phenotype regardless of
genotypic sex. These patients require not only cortisol to suppress ACTH secretion but also sex steroid
treatment consistent with the genotypic sex.
58. The answer is A. [Ch 7 VII G; Table 7-5] The ciliary muscle is innervated by both postganglionic
sympathetic and postganglionic parasympathetic fibers. Only J3-adrenergic, not a-adrenergic, receptors
are found in this smooth muscle. J3-Adrenergic stimulation causes relaxation of the ciliary muscle, which,
in turn, increases the tension on the lens, causing the lens to become thinner and adapted for far vision.
The parasympathetic innervation of the ciliary muscle causes it to contract, which, in turn, decreases
the tension on the lens, causing the lens to become thick and adapted for near vision (accommodation).
The heart contains only J3-adrenergic receptors, whereas vascular smooth muscle, pancreatic beta cells,
and intestinal smooth muscle contain both a- and J3-adrenergic receptors. Stimulation of either type of
receptor in the intestine causes inhibition of peristalsis.
59. The answer is A. [Ch 4 IX 0 2 e; Ch 7 IX C 3 a; XIII A 2 a, E 2] Angiotensin I, tetraiodothyronine
(T4 ), and testosterone all can be considered pro hormones. Angiotensinogen is a liver-derived globulin
that has no effect on aldosterone secretion. Angiotensinogen is converted by renin into angiotensin I, an
inactive decapeptide that is rapidly converted into the active octapeptide, angiotensin II, by angiotensinconverting enzyme (ACE), found mainly in pulmonary endothelial cells. Most of the circulating triiodothyronine (T 3) is formed extrathyroidally from T4 by 5'-deiodinase, an enzyme found mainly in the liver and
kidney. On a molar basis, T3 has 3-5 times the bioactivity of T4 Testosterone can be reduced to the
more potent androgen dihydrotestosterone (DHT) in an irreversible reaction catalyzed by Sa-reductase,
an enzyme found in the prostate, seminal vesicles, epididymis, and skin. DHT has twice the bioactivity
of testosterone.
60. The answer is B. [Ch 7 XII 0 4 a, 6 a, b] Insulin is stimulatory to K+ uptake by cells, and high
concentrations of exogenous insulin cause extracellular hypokalemia. This hypokalemic action of insulin

is due to the increased K+ uptake by muscle and liver. Although the primary stimulus to the alpha cell
is hypoglycemia, hypoglycemia-induced catecholamine release undoubtedly plays a role. Hypoglycemia
also is a potent stimulus for the release of GH (somatotropin) and ACTH (corticotropin). Among the
stimuli for ACTH release are pain, anxiety, pyrogens, and hypoglycemia.

61. The answer is D. [Ch 5 VIII 0 1 b] Metabolic acidosis is associated with a low arterial pH, a
reduced plasma [HC0 3-], and a compensatory increase in alveolar ventilation and renal excretion of
H +. Renal excretion of titrable acid is limited by the amount of filtered buffers (e.g., phosphate buffer)
and by the ability of the kidney to lower urinary pH. Thus, the net acid excretion by the kidney is largely
controlled by factors that regulate the urinary secretion of NH3 and the urinary excretion of N H4 +. There
is a concomitant, but limited, increase in titratable acid. Base excess is negative in metabolic acidosis.
62. The answer is D. [Ch 4 V C 3 b; IX 0 2 a (1) (a), C (1) (a), (b)] Since hemorrhage decreases both
GFR and RPF commensurately, there is no change in the filtration fraction, which is the ratio of GFR to
RPF. Severe hemorrhage results in a state of hypovolemia, which, in turn, is a major stimulus for the
secretions of both aldosterone and ADH. Bodily responses to hypovolemia take precedence over the
regulation of solute concentration. The decrease in blood volume initiates a neurocirculatory reflex arc
composed of afferent and efferent limbs. The afferent limb is activated by a reduction in plasma volume.
The reduced plasma volume leads to a decrease in central blood volume, decreased atrial and pulmonary
venous pressures, and a resultant reduction in the activity of low-pressure baroreceptors in these regions.
There also is a concomitant reduction in the firing rate of the high-pressure baroreceptors in the carotid
sinus and aortic arch. The decreased firing rates of these receptors cause an increase in ADH secretion.
Another afferent input originates in the low-pressure baroreceptors in the kidney. Stimulation of the
juxtaglomerular OG) cells by hypovolemia evokes the secretion of renin, which forms angiotensin I, a
decapeptide converted to angiotensin II by ACE in the pulmonary circulation. Angiotensin II is also a
stimulator of aldosterone secretion. Thus, aldosterone enhances Na+ reabsorption, and ADH enhances
water reabsorption by increasing the permeability of the collecting duct to water.

63. The answer is A. [Ch 7 XII 03] Untreated diabetes mellitus is associated with negative nitrogen
balance. The disease reflects a state of severe insulin deficiency combined with a decreased concentration
of plasma C-peptide. Since insulin and C-peptide are secreted in equimolar amounts, C-peptide directly
reflects pancreatic beta cell secretory activity and, therefore, endogenous insulin secretion.
The most characteristic feature of untreated diabetes mellitus is fasting hyperglycemia due, in part, to
the lack of insulin and, in larger part, to the unopposed action of the counter-regulatory (insulinantagonizing) hormones (glucagon, cortisol, epinephrine, GH). These four hormones cause the liver to
change from a glucose-utilizing to a glucose-producing organ, leading to hyperglycemia and glycosuria.
The hormones also increase the rate of lipolysis, which leads to a fatty acid-induced decrease in
glucose uptake. Cortisol and glucagon promote muscle proteolysis, leading to hyperaminoacidemia,
hyperaminoaciduria, elevated blood urea nitrogen (BUN), and negative nitrogen balance. This hormonal
imbalance switches liver fatty acid metabolism from oxidation, reesterification, or both to ketogenesis.
The sum of these effects is an increase in blood fatty acids, amino acids, and ketones due to the gradual
loss of muscle and adipose tissue. The ketosis leads to ketonemia and, eventually, ketonuria.

64. The answer is C. [Ch 3 III A, B 3 a] Ascending a mountain results in a decrease in P0 2 due to the
reduction in the barometric pressure. Air contains about 21 % O 2 ; therefore, administering supplemental
O 2 would increase the Po 2 Hyperventilation would reduce Peo 2 , resulting in a higher Po 2 , according
to the alveolar gas equation.
65. The answer is C. [Ch 4 IX 0 1 a (2) (a); Ch 7 VIOl d (2)] Aldosterone secretion is not significantly
depressed following hypophysectomy and patients can regulate Na+ and K+ balance. However, hypophysectomy without hormone replacement therapy is incompatible with human life. The pituitary gland is
essential for cellular differentiation, somatic growth, adaptation to stress, and reproduction. If the pituitary
gland is removed from young, rapidly growing animals, dwarfism occurs due to the absence of GH.
Atrophy of the gonads, thyroid gland, and adrenal cortex also occurs in the absence of pituitary tropic
hormones.

66. The answer is C. [Ch 7 VII F 2 b (2)] Catecholamine-O-methyltransferase (COMT) catalyzes the
metabolic inactivation of dopamine, norepinephrine, and epinephrine. Normetanephrine is a deaminated
catecholamine produced by the action of COMT on norepinephrine. Dihydroxymandelic acid also is a
substrate for COMT. COMT is found in the soluble fraction of cells, especially the liver and kidney, and
it is more important in the metabolic degradation of circulating catecholamines.

67. The answer is E. [Ch 4 IX 0 2 e; X C 1 d] Atrial natriuretic factor (ANF) is secreted by the atrial
myocytes in response to increased blood volume. AN F prevents angiotensin formation by inhibiting renin
release. The cardiac hormones inhibit angiotensin II and ACTH-two stimuli for aldosterone secretion.
Angiotensin II is the primary stimulus for aldosterone secretion. Angiotensin II synthesis is regulated by
renin release from the JG cells. The principal stimuli for renin secretion are decreased perfusion pressure
in the afferent arterioles, decreased [Na+] in the macula densa area of the nephron, and norepinephrine
secretion from the sympathetic neurons innervating the JG cells. All actions of renin are mediated through
the generation of angiotensin II. Other stimuli for aldosterone include corticotropin (ACTH), a high
plasma [K+], and a low plasma [Na+]. The stimulatory effect of ACTH on aldosterone secretion is
powerful but short-lived and is not a major factor in the control of aldosterone production.
68. The answer is C. [Ch 6 II Ole (3) (a); 11/ B 2, 6 a (1); IV B 2 c] The migrating motor complex
(MMC) is initiated by motilin (a hormone released from the endocrine cells in the small intestine) and is
not affected by vagotomy. Cutting the vagus nerve prevents receptive relaxation from occurring and thus
increases the rate of liquid emptying from the stomach. Primary esophageal peristalsis is coordinated by
the swallowing center and thus is prevented by vagotomy.
69. The answer is C. [Ch 7 VIOl b (2) (a)] Somatostatin is the GH-inhibiting hormone produced in
the arcuate nucleus of the tuberoinfundibular neural tract. Somatostatin is released from the median
eminence and enters the hypothalamic-hypophysial portal system that perfuses the adenohypophysis.
This neuropeptide also is produced in duodenum and the pancreatic delta cells. The plasma GH level is
elevated in response to any form of stress as well as to exercise and to deep sleep (stages III and IV).
Both insulin- and arginine-induced hypoglycemia are potent stimuli for GH secretion and can be used
as provocative tests of pituitary GH reserve.
70. The answer is C. [Ch 7 XII/ 0 1 a-d] Thyroid hormone biosynthesis occurs in four steps: (1) active
uptake of inorganic iodide (1-), (2) oxidation of inorganic iodide to active iodide (I +, known as iodinium),
(3) formation of iodotyrosines by the iodination of tyrosine residues within the matrix of thyroglobulin,
and (4) coupling (condensation) of iodotyrosines to form iodothyronines. All four steps require TSH;
steps 2, 3, and 4 are catalyzed by the membrane-bound enzyme thyroid peroxidase. Diiodotyrosine and
monoiodotyrosine have no biologic activity; however, the iodothyronines-T3 and T4 (thyroxine)-are
biologically active. Therefore, thyroxine synthesis requires TSH, active iodide, thyroglobulin, thyroid
peroxidase, and the coupling of two molecules of diiodotyrosine.
71. The answer is C. [Ch 7 XII 0 2] Insulin is a lipogenic as well as antilipolytic hormone. It decreases
the activity of the intracellular lipase, triglyceride lipase ("hormone-sensitive lipase"), by inhibiting the
formation of cAMP. Insulin activates the extracellular lipase, lipoprotein lipase, which is responsible for
the hydrolysis of plasma lipoproteins. The hydrolysis of triglycerides in the adipocyte delivers long-chain
fatty acids and glycerol into the circulation. Lipolysis in adipocytes is mediated by triglyceride lipase.
Catecholamines play a primary role in lipolysis, whereas glucagon, GH, cortisol, and ACTH have
lesser lipolytic effects. Cortisol and T3 modulate the sensitivity of adipocytes to the lipolytic effects of
catecholamines.
72. The answer is E. [Ch 6 11/ B 2 b, 6 a (1), b (1), C 1 b (2) (b), 2 c (1); IV C 3 b] Receptive relaxation
occurs when the proximal stomach (fundus and corpus) is stretched by the presence of food. Distension
of the antrum causes gastrin to be released from G cells. Gastrin enhances gastric acid (HC!) and
pancreatic enzyme secretion as well as gastric motility. In addition, expansion of the antrum initiates a
vago-vagal reflex that enhances antral contractions.

73. The answer is C. [Ch 7 VI A 2] The neural lobe of the pituitary gland, or neurohypophysis, is
derived from the neural tube (neural ectoderm). The neural crest gives rise to a wide variety of cells,
including neurons with perikarya outside the CNS, parafollicular cells of the thyroid gland, fibroblasts,
dentin-producing cells, vascular smooth muscle cells, melanocytes, and Schwann cells. Also derived
from the neural crest are the cartilage and bone of the skull and the adrenal medulla.
74. The answer is D. [Ch 4 IX B 1-3, C 1 a, b, E 1] Aldosterone acts on the connecting tubule and
collecting tubules to increase the reabsorption of Na+ and the secretion of K+ and H+. Thus, excess
amounts of this hormone lead to hypokalemia (kaliuresis), alkalemia (alkalosis), and hypertension. By
promoting Na + retention, excess aldosterone secretion leads to an increase in blood volume via an
increase in plasma volume. This effect, in turn, brings about a decline in hematocrit and in plasma oncotic
pressure.

amounts of this hormone lead to hypokalemia (kaliuresis), alkalemia (alkalosis), and hypertension. By
promoting Na+ retention, excess aldosterone secretion leads to an increase in blood volume via an
increase in plasma volume. This effect, in turn, brings about a decline in hematocrit and in plasma oncotic
pressure.
75-76. The answers are: 75-A, 76-0. [Ch 2 II B 3 a, c (5) (b)] Sinoatrial (SA) cells are characterized
by relatively low membrane potentials, spontaneous depolarization (which indicates pacemaker activity),
and a relatively fast rate of depolarization. Cell A meets all of these criteria and, thus, is most likely
derived from the SA node.
Pacemaker activity is synonymous with spontaneous depolarization, which is a decrease in the diastolic
membrane potential. The decrease in membrane potential is caused by the decline in K+ conductance;
less K+ leaves the cell during diastole while Na+ continues to leak into the cell. Thus, membrane potential
declines until threshold is reached, at which time an all-or-none action potential is generated. Pacemaker
cells lack the fast Na+ channels, so the rate of membrane depolarization during phase 0 of the cardiac
action potential is slow.
77-80. The answers are: 77-C, 78-8, 79-0, 80-A. [Ch 3 II D 1 c; /II B 3; VII B; Table 3-2; Ch 5 V/II
D 1] This patient's blood data are most consistent with a ventilation:perfusion abnormality. Although the
Pco 2 is normal, the P0 2 is decreased, as are pH and [HC0 3 -]. These latter two findings indicate a
metabolic acidosis that is most likely due to accumulation of lactic acid caused by the decreased P0 2
and cardiac output. Ventilation:perfusion abnormalities are the most common cause of clinical hypoxia.
A variety of factors affecting the lungs and cardiovascular system may cause these abnormalities.
Given a predicted dead space of 150 ml, this patient's calculated alveolar ventilation is 7.2 L/min.
Alveolar ventilation is the volume/min that is effective in gas exchange, which is calculated as follows:
(tidal volume - dead space) x respiratory rate. Thus, in this patient, the alveolar ventilation equals (0.6 0.15) x 16 = 7.2 L/min.
This patient's P0 2 value while breathing 40% O 2 (120 mm Hg) indicates an increase in the alveolar-toarterial P0 2 difference. Applying the alveolar gas equation, the alveolar P0 2 is calculated as
alveolar P0 2

43
(760 - 47) x 0.40 - 0.8

230 mm Hg

Thus, there is a difference of 110 mm Hg in alveolar-to-arterial P0 2 in this patient (230 - 120 = 110).
Normally, the alveolar-to-arterial P0 2 is 5-10 mm Hg. The large difference in this patient is due to
a physiologic shunt secondary to the ventilation:perfusion abnormality. Changes in the hemoglobin
concentration do not affect the arterial Po 2 , which depends solely on the amount of O 2 that is in physical
solution in the blood.
This patient's respiratory compliance is 0.03 Llmm Hg. The compliance of the respiratory system is
calculated as the change in volume (Le., tidal volume) divided by the change in distending pressure.
Since the respirator is delivering intermittent positive pressure (Le., 20 mm Hg in order to distend the
lungs to accept the delivered volume), the pressure in the lungs at the end of expiration is zero (Le.,
atmospheric). Thus, compliance of the respiratory system is 0.6 L/20 mm Hg = 0.03 Llmm Hg. Normal
respiratory compliance is about 0.1 Llmm Hg. The decreased compliance ("stiff lungs") in this patient
could be due to the pulmonary edema.
81-82. The answers are: 81-C, 82-8. [Ch 3 II C 4 b, D 2 c; VI B 1] Lung compliance is calculated as
the change in volume per unit change in distending pressure. Since alveolar pressure is zero at the
beginning and end of inspiration, the transmural (distending) pressure for the lung is zero minus the
interpleural pressure. Only the change in interpleural pressure between the beginning and end of inspiration is given. This difference divided into the tidal volume gives the lung compliance during dynamic
conditions, or 150 and 60 ml/cm H 2 0. Note that compliance is expressed as volume/pressure.
The change in this patient's lung compliance when she alters her respiratory rate is termed frequencydependent compliance (FOC). FOC occurs in the presence of high airway resistance, which causes some
acini not to fill completely at rapid rates of respiration, due to long time constants. Thus, FOC indicates
the presence of high airway resistance, which is synonymous with obstructive lung disease.
83-87. The answers are: 83-0, 84-8, 85-8, 86-1:, 87-8. [Ch 2 IX B 1-21 4 dl b] From the record, it i5
obvious that the major effect at time 1 was a slowing of the heart rate. Cardiac output (Q) must have
remained relatively constant, since there is little change in arterial pressure (P) and total peripheral
resistance (TPR) [Q = P/TPR]. Both electrical stimulation of the superior cervical ganglion and administra-

tion of a cholinergic blocking drug would cause an increased heart rate, whereas electrical stimulation
of the lumbar sympathetic nerve roots would result in a significant rise in resistance in the lower portion
of the body. The administration of a ,a-adrenergic blocking drug would reduce heart rate as well as
cardiac contractility, so that cardiac output would be significantly reduced, which it is not. Vagal
stimulation can cause marked cardiac slowing, but ventricular end-diastolic volume can increase, so that
stroke volume increases sufficiently to compensate for the reduced heart rate.
At time 2, there is obviously a rise in heart rate, arterial pressure, and total peripheral resistance. The
only factor that could cause this sequence of changes is stimulation of the sympathetic system, which
includes the heart.
An a-adrenergic agonist increases peripheral resistance, which raises arterial blood pressure. This
results in baroreceptor stimulation, causing a reflex slowing of the heart rate. An a-adrenergic blocking
drug has the opposite effect. Cholinergic blocking agents and ,a-adrenergic agonists increase heart rate.
A ,a-adrenergic blocking agent would reduce heart rate and contractility, resulting in a decreased cardiac
output and a reduced arterial pressure.
All adrenergic agonists would cause an increase in heart rate as a direct effect on the SA node,
isoproterenol having the greatest effect and norepinephrine the least. A drug such as epinephrine, which
has both a- and ,a-adrenergic properties, can increase contractility and peripheral resistance so that a
large increase in arterial pressure occurs, which can then affect the baroreceptors and result in a reflex
slowing of cardiac rate. The decrease in heart rate at time 3 was not caused by a reflex effect mediated
by the chemoreceptors, because the chemoreceptors, when activated, would lead to a rise in heart rate
as well as an increase in contractility. Ventricular extrasystoles have no effect on peripheral resistance.
Arterial pressure, peripheral resistance, and heart rate all rise after time 4, indicating a primary effect
on the heart, since the increased pressure would otherwise result in a reflexly produced decreased heart
rate and vasodilation. The only nerves likely to induce these effects are the superior cervical ganglia,
which are the sympathetic innervation to the heart. Postganglionic fibers of the lumbar sympathetic
ganglia would not affect the heart directly. Stimulating the peripheral end of a cut carotid sinus nerve
would have no effect on the cardiovascular system, whereas stimulating the central end of this nerve
would cause cardiac slowing and vasodilation rather than the results presented here. Vagal stimulation
would cause cardiac slowing.

88-93. The answers are: 88-D, 89-8, 90-A, 91-D, 92-C, 93-C. [Ch 2 VB 1; VI A 3 b, d, 4 b, c; Figure
2-19; Ch 3 " 0, E] The maximal pressure-volume (PV) point for loop ACOI lies on the same line as
normal, which indicates that there is no change in contractility. The increased ventricular pressure reflects
an increase in arterial resistance and results in the increased pressure against which the ventricle must
work. The end-diastolic volume is indicated by point A; all three loops start at the same end-diastolic
volume. Preload is a synonym for end-diastolic volume, so this choice is not correct.
Loop ABFG shows the effect of increased contractility. An increase in contractility causes the maximal
PV relationship to move to the left and assume a steeper slope. Thus, stroke volume is increased as a
result of these changes, and there is a smaller volume of blood that remains in the ventricles at the end
of systole.
Mitral valve closure occurs at the onset of systole (point A) and represents the beginning of the
isovolumic contraction period. During this interval, as the name implies, the ventricular volume remains
constant, and ventricular pressure rises until it exceeds the pressure in the aorta.
Ventricular filling occurs during diastole (indicated by segment HA) and is the increase in volume that
occurs in preparation for the next contraction. During steady state conditions, the ventricular filling will
be equal to the stroke volume that is ejected during the next systole. Changes in venous return, duration
of diastole, afterload, and ventricular contractility all can influence the ventricular filling and the stroke
volume in a complex fashion. The ventricular PV curve provides an excellent model for understanding
these relationships.
The slope of the line from point G to point A has the units of mm Hg/ml, which is compatible with
elastance. Compliance is the reciprocal of elastance and has units of volume/pressure. The line from
point G to point A occurs during diastole, and the slope is a measure of the diastolic elastance of the
ventricles.
Stroke volume is represented as the width of each PV loop (i.e., segments AI, AH, and AG). The
end-diastolic volume occurs at point A for the three loops. The diastolic reserve volume is the volume
of blood from the end-diastolic point to the maximal volume that the ventricle can hold-this volume
is not shown on the graph. The residual volume represents a minimal volume of blood that lies between
the trabeculae carneae and the papillary muscles. This volume is a part of the end-systolic volume and
can never be ejected from the ventricles.

94-97. The answers are: 94-0, 95-8, 96-0, 97-8. [Ch 4 "A 1 b, 2 a, B 1 a, 2 a-c; Figures 4-3, 4-4]
The man's plasma volume is equal to the Evan's blue space, which is 3 L. Since his plasma volume
represents 60% of his blood volume, the total blood volume is calculated by dividing the Evan's blue
space by 0.6, or
plasma volume (L)
blood volume (L) = - - - - - - (1-hematocrit)
3 L
blood volume = = 5 L
0.6

The interstitial fluid volume (lSFV) cannot be calculated directly by the dilution principle because there
is no substance that is confined to the interstitial fluid space. Since the interstitial fluid space constitutes
part of the ECF volume, it can be determined by subtracting the plasma volume from the ECF. The ECF
volume of this man is equal to the inulin space. Therefore, the ISFV is
ISFV = inulin space - Evan's blue space
=8L-3L
= 5 L
The ECF volume is determined directly by the dilution of such substances as inulin, mannitol, sucrose,
thiosulfate, radiosodium, and radiochloride. Thus, the volume of the ECF is equal to the inulin space,
which is given as 8 L.
The mathematical relationship between lean body mass (LBM) and total body water (TBW) is
TBW (L) = 0.7 LBM (kg)
Therefore,
TBW (L)
LBM (kg) = - - 0.7
Since the total body water is given as the tritiated water space, the calculation is
35 L
= 50 L
LBM (kg) = 0.7
But 50 L of water weighs 50 kg. Subtracting the 50 kg LBM from the 60 kg body weight leaves 10 kg
for the weight of the body fat.

98-100. The answers are: 98-8, 99-0, 100-8. [Ch 3 IV CIa, 3; VB 2; IX C, 02 b; Figure 3-31] This
man's arterial O 2 content is 12 ml/dl. To calculate this, it is important to recognize that the normal Pso
is 27 mm Hg, meaning that this man's arterial hemoglobin saturation must be 50%. One reason that this
man's hemoglobin saturation is low is his exposure to a low barometric pressure (PB) at an altitude of
11,500 feet. The PB at this altitude is about 490 mm Hg, providing an alveolar P0 2 of about 50 mm Hg.
The O 2 capacity is calculated from the hemoglobin concentration as follows: 18 g/dl x 1.34 ml/g =
24.12 ml I dI. Disregarding the small amount of O 2 that is dissolved in plasma, the arterial O 2 content
would be O 2 capacity x saturation, or 24.12 x 0.5 = 12 ml I dI.
This man's pulmonary artery pressure is likely to be higher than normal due to hypoxic pulmonary
vasoconstriction. At high altitudes there would be an increased resistance to flow through the pulmonary
vasculature. This increased resistance is the result of smooth muscle contraction in the walls of the
pulmonary blood vessels in response to an unknown mediator brought on by hypoxia. The cardiac
output in people who live for long periods at high altitudes is in the normal range, although it increases
upon initial exposure to such altitudes. Chemoreceptors have no significant effect on the pulmonary
circulation.
Venous hemoglobin saturation is decreased at high altitudes, because the hemoglobin starts in the
lungs somewhat unsaturated due to the low Po 2 Residents at high altitudes have an increased hemoglobin
concentration as a compensation for the low P0 2 in the tissues. If hemoglobin concentration rises
sufficiently, the O 2 content may be in the normal range, but the hemoglobin saturation would be reduced
secondary to the low Po 2 Although cardiac output is normal in people acclimated to high altitude, the
venous hemoglobin saturation is above normal in people with increased cardiac output. This is due to
less O 2 being removed from each unit of blood when O 2 delivery is increased by the higher cardiac
output.

101. The answer is C. [Ch 4 IV B; VI B 2; Figures 4-7, 4-11C] The Na+-dependent reabsorption of
glucose is blocked by phlorizin, a phenolic glycoside. Thus, this competitive inhibitor virtually blocks the

secondary active transport of glucose by the proximal tubule, which leads to glycosuria. Therefore,
following phlorizin administration, the clearance of glucose becomes equal to the clearance of inulin.
Curve C depicts the clearance curve for a filtered substance that is neither reabsorbed nor secreted, such
as inulin.

102-103. The answers are: 102-8, 103-A. [Ch 2 VI B 1] The gradient that occurs between the
ventricular and aortic systolic pressures is diagnostic of aortic stenosis. The normal aortic valve provides
a negligible resistance, and the aortic pressure is nearly identical to the ventricular pressure during the
phase of rapid ventricular ejection. A similar picture is seen if right ventricular and pulmonary pressures
are measured in the presence of pulmonary valve stenosis, but the pressures are proportionately reduced
because of the low resistance of the pulmonary circulation.
Semilunar valve stenosis represents an impediment to the ejection of blood from the ventricle and
results in an ejection-type murmur during systole. An ejection murmur is diamond-shaped (i.e., it is a
crescendo-decrescendo sound that has maximal intensity in midsystole, when the pressure gradient is
largest).
104-106. The answers are: 104-C, lOS-A, 106-E. [Ch 3 II H 1 d (1), 2 a] This patient's functional
residual capacity (FRC) is 2.7 l. Since the volume of gas in the spirometer is kept constant, the degree
of dilution produced by the lungs after equilibration must be determined by calculating the ratio of F,/
F2 , which equals 0.05 ...;- 0.03 = 1.67. Thus, the volume of the spirometer plus the lung volume is 1.67
times the volume of the spirometer, or 1.67 x 4 = 6.7 l. Subtracting the volume of the spirometer leaves
the lung volume at the start of the test (i.e., FRC), or 6.7 - 4 = 2.7 l.
This patient's residual volume (RV) is 0.7 l. Since FRC is the sum of RV and expiratory reserve volume
(ERV), RV is determined as: RV = FRC - ERV, or 2.7 - 2.0 = 0.7 l.
This patient's total lung capacity (TLC) is 5.7 l. TLC is the sum of vital capacity (VC) and RV, or 5 +
0.7 = 5.7 l.

107-10S. The answers are: 107-E, lOS-A. [Ch 2 Figure 2-19] Opening of the mitral valve signals the
onset of ventricular filling. This occurs when ventricular pressure and volume are at their lowest point
(i.e., point E on the pressure-volume loop).
Isovolumic contraction is the first period of ventricular systole. It begins (point A) when ventricular
pressure rises above atrial pressure and the mitral valve closes. Isovolumic contraction ends (point B)
when the semilunar valve opens and ventricular ejection begins. The onset of ejection is signaled by the
point where ventricular volume begins to decrease.
109. The answer is D. [Ch 7/1/ A 1]ln humans, there is a diurnal variation in the secretory patterns of
ACTH and cortisol (hydrocortisone) and in the excretion of 17-hydroxycorticoids. In individuals who
sleep regularly from about 11 :00 P.M. to 7:30 A.M., the peak of this circadian rhythm occurs between 6
A.M. and 8 A.M., while the nadir is observed between midnight and 2 A.M. Thus, the peak plasma level
of glucocorticoid is entrained to the activity cycle with maximal ACTH-cortisol secretion appearing about
1 hour after awakening. Changes in sleep periods or in longitude cause phase shifts in the ACTH-cortisol
secretory pattern; however, the rhythm itself persists. There is an abrogation of the pituitaryadrenocortical rhythm in patients with hypercortisolism.
110-112. The answers are: 110-8, lll-E, 112-D. [Ch 2 VI A 4 b, c, 6] Tracing A is a ventricular
volume curve. The ventricular volume curve is distinctive because of the decline in left ventricular
volume during systole and the rise during diastole. The timing of systole and diastole can be identified
from the curve's relationship with the EKG tracing.
Rapid filling begins with the opening of the AV valves early in diastole. Diastole normally begins shortly
after the end of the T wave and is indicated by the occurrence of the second heart sound (not shown).
Interval C should be readily recognized as diastole. The duration of diastole is largely determined by
heart rate, which alters the length of the cardiac cycle. The period of systole does shorten with an
increase in heart rate, but diastole is affected much more. At very high heart rates, stroke volume is
reduced because of such an abbreviation of diastole that ventricular filling time becomes inadequate.
Thus, it is important to terminate ventricular tachycardia because of the reduced cardiac output that
results from the rapid rate.
113-115. The answers are: 113-8, 114-C, 115-C. [Ch 3 II C 1; //I B 2 a (3),3 b; IX C 1 a; Ch 5 VII 0
7 a; VIII 03, E 7; Figure 5-74] The increased Pco 2 in this patient indicates that alveolar ventilation is
inadequate. Minute ventilation may be normal or increased while alveolar ventilation is reduced, because
the total dead space increases secondary to ventilation:perfusion abnormalities caused by the pulmonary

disease. (It is important to remember that total dead space equals alveolar dead space plus anatomic
dead space.) The normal arterial pH in this patient indicates that renal compensation has occurred.
Renal compensation may require 1-2 weeks, meaning that the abnormality is chronic, ruling out acute
respiratory failure. Anemia and carbon monoxide poisoning are not viable alternatives, since arterial P0 2
is normal in these two conditions. An anatomic shunt causes hypoxia and typically results in a lowered
Pe0 2 due to the increased ventilation that results from this type of hypoxia.
Smoking causes inflammation and edema in the airways (bronchitis), which can lead to infection of
the airways and consequent mucus production. The resultant airway narrowing increases airway resistance and the work of breathing. If respiratory work increases sufficiently, there will be respiratory muscle
fatigue and a decrease in alveolar ventilation, which results in hypercapnia and hypoxia. These latter
events cause pulmonary artery vasoconstriction and pulmonary hypertension.
This patient has respiratory acidosis, by definition, due to the hypercapnia. Respiratory acidosis is
compensated by renal retention of HC0 3 - (normal = 24 mEq/U. Metabolic or lactic acidosis is not
present because of the increased HC0 3 - levels. Respiratory alkalosis is produced by a decrease in Pe0 2 .
116-118. The answers are: 116-8, 117-E, 118-A. [Ch 3 VII B 2,3; IX C 1; Table 3-5] A right-to-Ieft
anatomic shunt causes mixed venous blood to flow from the right to the left ventricle without being
oxygenated, leading to hypoxia and an increased alveolar-arterial P0 2 difference. Ventilation:perfusion
imbalance also leads to hypoxia but is not the cause in this patient. With ventilation:perfusion imbalance,
arterial P0 2 should exceed 500 mm Hg when the patient breathes 100% O 2 at sea level, and alveolar
P0 2 should be 673 mm Hg (which can be calculated using the alveolar gas equation). A left-to-right
anatomic shunt does not cause hypoxia, since oxygenated blood (from the left side of the circulation)
enters the right ventricle or the pulmonary artery. Administration of 100% O 2 will completely correct
the hypoxia due to diffusion abnormalities or hypoventilation.
The fraction of the cardiac output that represents shunted blood can be calculated using the shunt
equation, which states that
Qs

(Ci0 2

Ca0 2 )

Qt

(Ci0 2

Cii0 2 )

where Qs = the shunted blood flow, Qt = cardiac output, Ci0 2 = pulmonary capillary O 2 content, and
Ca0 2 and Cii0 2 = arterial and venous O 2 content, respectively. The calculation usually is sufficiently
accurate only if hemoglobin O 2 is used and the dissolved O 2 is disregarded. In the pulmonary capillaries,
hemoglobin should be 100% saturated while the patient is breathing O 2 so that
Ci0 2

=
=
=

O 2 capacity
18 g hemoglobinldl x 1.34 ml 02/g
24.1 mlldl

Arterial O 2 content can be determined as


Ca0 2

=
=
=

Ci0 2 x hemoglobin saturation


24.1 mlldl x 0.85
20.5 mlldl

Venous O 2 content can be determined if it is remembered that the normal venous P0 2 also is 40 mm
Hg, which is equivalent to 75% hemoglobin saturation, giving
Cv0 2

24.1 x 0.75
18.1 mlldl

=
=

Thus,
Qs

24.1 - 20.5

Qt

24.1 - 18.1
3.6
= -

6.0

0.6

To determine the site of a right-to-Ieft shunt, the P0 2 must be measured in the left heart rather than
the right (i.e., the unsaturated venous blood is used to locate the point where P0 2 drops). Thus, if there
were an interatrial septal defect with right-to-Ieft flow, the P0 2 in the left atrium would be lower than
the P0 2 in the pulmonary veins. A ventricular septal defect would show a drop in P0 2 in the left ventricle
compared to that in the left atrium. If the P0 2 were low and equal in the pulmonary veins, left atrium,
and left ventricle, then the shunt would necessarily be within the lungs, which could indicate the presence
of an arteriovenous anastomosis.

119-121. The answers are: 119-0, 120-E, 121-A. [Ch 2 V/II A, 0 1; Ch 3/11 B 3 b, C 3 d; VI/I A 3;
IX C 2] The major drive for respiration comes from the medullary chemoreceptors, which respond to
local H + concentration, which, in turn, depends on the Peo 2 in the surrounding tissues. During the
period of increased ventilation, Peo 2 in the body is reduced, thus eliminating the respiratory drive until
CO 2 again reaches a threshold value at the end of apnea. The medullary chemoreceptors do not respond
to changes in P0 2 and, thus, are not the cause of the apnea.
During positive pressure ventilation, the intrathoracic pressure increases, which reduces the pressure
gradient between the peripheral tissues and the right heart. Thus, venous return and cardiac output
decline during positive pressure ventilation, and the tissues remove more O 2 from each unit of blood as
the blood flows through systemic capillaries. Consequently, during positive pressure ventilation venous
blood contains less O 2 than normal and the mixed venous P0 2 decreases. The mixed venous P0 2 would
not be less than 40 mm Hg because of the hypoxia that would occur from breathing this gas mixture,
which contains only 0.9% less O 2 than air; the hyperventilation would more than make up for the slight
reduction in Po 2 . Hyperventilation causes a decrease in Peo 2 and an increased pH, both of which would
reduce the Pso of hemoglobin, not increase it.
The arteriovenous O 2 content difference is calculated by dividing the O 2 consumption by the cardiac
output: 240 mllmin -:- 6 L/min = 40 ml 02/L of blood. Since the measurements in the question are
given in mlldl, the correct answer is 4 ml/dl.

122-126. The answers are: 122-8, 123-8, 124-A, 12S-A, 126-0. [Ch 5 VII A 1-5, 0; VII 0 1, 3]
Hypoventilation, or decreased alveolar ventilation, is the cause of respiratory acidosis. In metabolic
acidosis, the increased [H+] in the arterial blood is a stimulus for hyperventilation and a decline in Peo 2
The main components of total CO 2 content ([total CO 2]) are [HC0 3 - ] and dissolved CO 2; an increase
in either factor will increase the CO 2 content. In respiratory acidosis, [total CO 2 ] increases via an increase
in Peo 2 , and in metabolic alkalosis, [total CO 2 ] increases via an increase in [HC0 3 - ] . The compensatory
responses to respiratory acidosis and metabolic alkalosis increase the CO 2 content further by elevating
the [HC0 3 -] and S x Peo 2 , respectively.
[Total CO 2 ] is decreased when either [HC0 3 - ] or dissolved CO 2 is decreased. Thus, in respiratory
alkalosis, [total CO 2 ] decreases due to a decline in arterial Peo 2 , and in metabolic acidosis, [total CO 2 ]
decreases due to a decline in [HC0 3 -]. The renal compensation to respiratory alkalosis is increased
[HC0 3 -] excretion, which lowers [total CO 2 ] further. Similarly, the respiratory compensation to metabolic acidosis is hyperventilation, which also reduces the CO 2 content further.
The reduced [HC0 3 -] and increased [H+] of metabolic acidosis lead to a compensatory hyperventilation, which results in decreased arterial Peo 2 (hypocapnia). Respiratory acidosis is due to hypoventilation,
which results in increased arterial Peo 2 (hypercapnia).
The normal [HC0 3 -liS x Peo 2 ratio is 20:1. This ratio is decreased in acidotic states, by either the
increased Peo 2 of respiratory acidosis or the decreased [HC0 3 -] of metabolic acidosis. This ratio is
increased in alkalotic conditions, by either the decreased Peo 2 of respiratory alkalosis or the elevated
[HC0 3 -] of metabolic alkalosis.

127-132. The answers are: 127-8, 128-8, 129-A, 130-A, 131-E, 132-0. [Ch 4 VI/I A 2, C 1; IX A "
B " 02 a (1), e (3) (a)] ADH, also known as arginine vasopressin, is an octapeptide synthesized mainly
in the supraoptic nucleus of the ventral diencephalon. ADH also can be classified as a nonapeptide, if
the single cystine moiety is counted as two cysteine residues. ADH is stored in the pars nervosa, from
which it is secreted.
ADH regulates plasma osmolality, which normally is about 300 mOsm/kg. It promotes water reabsorption mainly from the tubular fluid in the renal collecting ducts by increasing the water permeability of
these cells. ADH allows humans to elaborate a small volume, hypertonic urine in order to conserve
water. Since ADH promotes free-water reabsorption, it determines the plasma [Na +].
Aldosterone, the most potent endogenous mineralocorticoid, acts primarily on the renal collecting
ducts to promote Na+ reabsorption and K+ and H+ excretion. It has similar effects on sweat, salivary,
and intestinal glands. Thus, aldosterone controls the Na+ content of the body. In turn, the Na+ content
determines the volume of the various fluid compartments. Aldosterone also increases Na+ reabsorption
by the connecting segment of the nephron. Aldosterone is synthesized in and secreted from the outermost
layer of the adrenal cortex, called the zona glomerulosa.
Dipeptidyl carboxypeptidase, also known as ACE or kininase II, is located mainly on the endothelial
surface of pulmonary capillaries. ACE catalyzes the conversion of angiotensin I (an inactive decapeptide)
to angiotensin II (an active octapeptide). In addition, ACE simultaneously inactivates the nonapeptide,
bradykinin. Thus, ACE leads to the increased formation of angiotensin II (a vasoconstrictor) and the
decreased formation of bradykinin (a vasodilator). Angiotensin II functions as a vasoconstrictor and
aldosterone-inhibiting hormone.

Renin is a proteolytic enzyme produced by the JG cells of the afferent arteriole. These cells are modified
smooth muscle cells that have acquired secretory function and, thus, are described as myoepithelial cells.
JG cells function as low-pressure baroreceptors that are stimulated by a decrease in renal perfusion
pressure caused by a decrease in systemic blood volume or pressure.

133-136. The answers are: 133-C, 134-0, 135-0, 136-E. [Ch 1 IV A 3, B 3, C 3] The amount of Ca2+
released from the sarcoplasmic reticulum of the smooth and cardiac muscle cells is normally varied to
control contractile force. In skeletal muscle, maximal amounts of Ca2+ enter the cell with each contraction. Skeletal and smooth muscles are able to recruit additional fibers when more force is required. The
heart must contract in a coordinated fashion so that all of its muscle fibers are recruited at the same
time. In addition, the heart must relax between contractions; thus, it cannot use repetitive stimulation as
a mechanism for increased force production. Smooth, cardiac, and skeletal muscles all are able to
influence the force of contraction by varying the initial length (preload) of their sarcomeres.

137-140. The answers are: 137-0, 138-B, 139-0, 140-A. [Ch 5 VII A 3, 5 c, 0 1 b, 2; VIII 0 1,2,
4, E 2, F] The blood data for patient 0 include high pH U[H+j), high [HC0 3 -], and a compensatory
increase in Peo 2-findings that coincide with partially compensated metabolic alkalosis. That [H+] and
[HC0 3 -] change in opposite directions indicates a primary metabolic disturbance, and the pH of 7.5
indicates alkalosis. The increment in Peo 2 shows that partial respiratory compensation has occurred.
The blood data for patient B include low pH (r[H+j), low [HC0 3 - ] , low CO 2 content, and low Peo 2 findings that coincide with a compensated metabolic acidosis. When metabolic acidosis is compensated
by a respiratory alkalosis, the pH tends to return to normal, the CO 2 content drops further, and the Peo 2
decreases.
Patient 0 has the highest CO 2 content (i.e., [HC0 3 -] + S x Peo 2 ). This patient has partially compensated metabolic alkalosis indicated by a 42% increase in [HC0 3 -] with only a 13% compensatory
increase in Peo 2 . The CO 2 content is equal to 36.35 mmol/L (35 mmol/L + 1.35 mmol/L).
The blood data for patient A include a low pH (r[H+j), Peo 2 , and a compensatory decrease in
[HC0 3 -]-findings that coincide with respiratory alkalosis, a condition caused by high alveolar ventilation. Note that the change in the alternate variable ([HC0 3 -j) is in the same direction as the change in
the primary variable (Peo 2 ). In this patient, the pH of 7.4 together with hypocapnia and decreased
[HC0 3 - ] indicates full compensation for a respiratory alkalosis (i.e., a [HC0 3 -liS x Peo 2 ratio of 20:1)
and thus the highest alveolar ventilation.

141-143. The answers are: 141-B, 142-0, 143-C. [Ch 6 IV F 3 b, 5 d-e]lron is absorbed from the
duodenum and proximal jejunum by a membrane-bound carrier protein on the luminal surface of the
enterocyte. Once inside the cell, iron combines with an iron-binding protein to form a complex called
ferritin. Before being extruded from the serosal surface of the cell, the iron dissociates from ferritin.
Ca2+ is absorbed from the duodenum by a membrane-bound carrier on the luminal surface of the
enterocyte that is formed in response to the presence of vitamin D. Once inside the enterocyte, the Ca2+
is extruded from the serosal surface of the cell by an active transport system.
Cholesterol must be dissolved in micelles before it can be absorbed. Micelles are small spherical
globules formed from bile salts. The polar, water-soluble end of the bile salt faces outward, and the
lipid-soluble tail portion of the bile salt faces inward. Cholesterol dissolves in the interior of the micelle,
and, when the micelle makes contact with the intestinal membrane, the cholesterol diffuses from the
micelle into the enterocyte.

144-147. The answers are: 144-B, 145-E, 146-B, 147-B. [Ch 4 VI C 3,4 a, c; Figure 4-9; Table 4-4B]
Most H+ secretion by the nephron occurs in the proximal tubule by Na+ -H+ exchange. This active H+
efflux is linked through a countertransport mechanism to Na+ influx across the luminal membrane. Most
of this secreted H+ is not excreted but is reabsorbed in the form of H 2 0. Most important, any secreted
H+ that combined with HC0 3 - in the lumen forms H 2 C0 3 , which is dehydrated into CO 2 and H 2 0,
both of which are reabsorbed. Thus, the secreted H+ that combines with HC03 - does not contribute
to the urinary excretion of acid.
K+ secretion occurs mainly in the cortical collecting tubule. The excreted K+ is derived mainly from
K+ secretion in this region of the nephron. Secretion of K+ involves active pumping across the peritubular
membrane followed by passive diffusion across the luminal membrane into the tubular lumen.
The cells of the proximal tubule are the major site of ammonia production. Addition of NH4 + to the
tubular fluid occurs primarily in the proximal tubule. Thus, the accumulation of ammonia in the lumen
involves both nonionie diffusion of NH3 and transport of NH4 +.
The proximal tubule is the major site for the active secretion of para-aminohippuric acid (PAH) in its
anionic form.

148-150. The answers are: 148-A, 149-8, 150-0. [Ch 7 XIV F 3, H] Parathyroid hormone (PTH) is
the hypercalcemic hormone of the body. Excessive amounts of PTH would produce hypercalcemia and
hypophosphatemia together with hypercalciuria and hyperphosphaturia. The major regulator of PTH
synthesis and secretion is serum ionized calcium concentration ([Ca 2 +]). PTH maintains the normal
plasma total calcium concentration at about 5 mEq/L by interacting with kidney, bone, and intestine.
PTH stimulates bone resorption, and it decreases the tubular maximum (Tm) for phosphate by decreasing
proximal tubular reabsorption of phosphate, resulting in phosphate diuresis. When PTH secretion is high,
the fraction of filtered phosphate that is reabsorbed may fall from the normal 80%-95% to 5%-20%.
PTH increases the Tm for Ca 2 + by increasing distal tubular reabsorption of Ca2+. Although PTH stimulates
renal Ca2+ reabsorption, the urinary Ca2+ is greater than normal in states of excess PTH because of the
increased filtered load of Ca2+.
There is an inverse relationship between PTH secretion and plasma [Ca2+]: when plasma [Ca2+] falls
PTH increases, and when plasma [Ca2+] rises PTH secretion is suppressed. Thus, a decrease in PTH
secretion is caused by an increase in plasma [Ca2+]. This increase in [Ca 2 +] is associated with a
decrease in [HPO/-]. Inorganic phosphate has no direct influence on PTH secretion; rather, it is the
phosphate-induced decrease in plasma [Ca 2 +] that stimulates PTH secretion. Conversely, a decrease in
plasma inorganic [HPO/-] increases plasma [Ca2+] and indirectly inhibits PTH secretion.
Overall, the physiologic action of active vitamin 0 3 (calcitrioi) is to increase extracellular [Ca2+] and
[HPO/-]. The effects of calcitriol are exerted primarily on the intestine and bone and, to a lesser extent,
the kidney. Calcitriol increases renal tubular reabsorption of Ca2+ and phosphate. It also is the principal
mediator of PTH-induced intestinal Ca2+ and phosphate absorption. Thus, vitamin 0 3 toxicity leads to
hypercalcemia directly by bone resorption and renal Ca2+ reabsorption and indirectly by mediating
PTH-induced intestinal Ca2+ absorption. This hypercalcemia suppresses PTH secretion, leading to increased plasma [HPO/-] and increased renal phosphate reabsorption. This is in contradistinction to
hyperparathyroidism, which causes an increase in [Ca 2 +], a decrease in [HPOl-], and an increase in
plasma PTH level. Thus, hyperparathyroidism must be distinguished from other causes of hypercalcemia.

151-154. The answers are: 151-8, 152-A, 153-C, 154-0. [Ch 7 XII A 1, C 1 b (2), 2 b, c, 02 b)
Lesions to the subthalamic nucleus produce ballismus, a movement disorder characterized by wild,
flinging movements of the arms and legs on the side of the body opposite to the lesion. Lesions within
the motor cortex interrupt axons that are inhibitory to brain stem nuclei, which are, in turn, excitatory
to alpha motoneurons innervating antigravity muscles. Loss of these inhibitory fibers (release from
inhibition) results in the overactivity of the antigravity muscles that characterizes spasticity. Lesions to
the posterior parietal lobe result in apraxia, which is the inability to carry out motor tasks voluntarily,
even though the muscle strength and coordination that is required to carry out the movement is intact.
Lesions to the vestibular nucleus damage the circuitry required for the vestibular ocular reflex, resulting
in nystagmus.

155-159. The answers are: 155-E, 156-A, 157-8, 158-0, 159-C. [Ch 3 /II 8 3 a; VII 03; V/II 8 1; IX
C " 3, 0 1 a] While mountain climbing, the arterial P0 2 would be reduced, with arterial Peo 2 reduced
by reflex hyperventilation. Since this is acute exposure to high altitude, arterial pH would be increased;
not enough time has passed for the kidneys to correct the blood pH. Only one set of blood data (E)
coincides with these conditions.
Although arterial P0 2 is normal in anemia, the arterial O 2 content is reduced in proportion to the
decrease in hemoglobin concentration. Since the chemoreceptors are not stimulated, ventilation does
not change and, thus, arterial Peo 2 and pH are normal. Only one set of blood data (A) coincides with
these conditions.
Hypoventilation is synonymous with hypercapnia and, thus, with an increase in arterial Peo 2 Only
one set of blood data (8) reflects this change.
During hyperventilation, arterial Peo 2 decreases and arterial P0 2 increases. Only one set of blood data
(0) coincides with these conditions.
Patients with chronic obstructive lung disease typically have a reduced arterial P0 2 due to the venti lation:perfusion abnormality that is present. In early stages of the disease, these patients have a normal-tolow Peo 2 and an arterial pH in the normal range, since the kidneys can maintain the correct HC0 3 - /
H 2 C0 3 ratio. Only one set of blood data (0 coincides with these conditions.

160-164. The answers are 160-C, 161-E, 162-A, 163-0, 164-8. [Ch 4 IV 8 1, C 1 a; V C; VI A 3,8
3 c (2); Figures 4-7, 4-10, 4-11; Table 4-5] The use of inulin clearance (C in ) to measure glomerular

filtration rate (GFR) is valid because all of the filtered inulin is excreted in the urine without being
reabsorbed or secreted by the renal tubules. Thus, GFR is equal to C;n as
U;n X V
GFR=C = - Pin

In

where U;n and P;n = the urinary and plasma inulin concentrations, respectively, and
volume/ minute. Substituting,
GFR = C;n =

V=

the urinary

150 mg/ml x 1.2 ml/min

---=-------1.5 mg/ml

180 mg/min
1.5 mg/ml
= 120 ml/min
Note that with a unit analysis, both mg/ml terms cancel out.
Renal plasma flow (RPF) is calculated from the clearance of para-aminohippuric acid (PAH); however,
there are no PAH data provided for this man. Thus, it is necessary to determine RPF from the filtration
fraction (FF), defined as the ratio of GFR to RPF, or
GFR
FF=RPF
FF can be calculated from the arterial and venous inulin concentrations as

A -V
on
In
A;n
where A;n and V;n are the arterial and venous inulin concentrations, respectively. Substituting,
1.50 mg/ml - 1.20 mg/ml
FF = ---=------'--1.50 mg/ml
fraction of inulin filtered

0.3
= -=

1.50

0.20

With FF and GFR determined, the RPF is easily determined as


FF

GFR
= --'

RPF'

GFR
120
RPF = - - = ml/min
FF
0.2

600 ml/min

Because FF = the ratio of GFR to RPF, FF also can be calculated using the extraction of a substance
such as inulin to determine RPF. If the urinary concentration of inulin is 150 mg/ml and the urine flow
rate is 1.2 ml/min, the urinary excretion rate of inulin is 180 mg/min. If, when the excretion rate was
measured, the inulin concentration was 1.50 mg/ml in renal arterial plasma and 1.20 mg/ml in renal
venous plasma, each milliliter of plasma traversing the kidneys must have contributed 0.30 mg to the
180 mg that was excreted. Hence, the RPF must have been
180 mg/min
- - - - - = 600 ml/min
0.3 mg/ml
Thus, with the RPF determined, FF can be calculated as
GFR
FF = RPF

120 ml/min
=

600 ml/min

= 0.2

Once RPF is determined, renal blood flow (RBF) can also be discerned. RBF is calculated by dividing
RPF by the term (1 - hematocrit), or
RBF =

RPF

600 ml/min

600 ml/min

1 - hematocrit

1 - 0.40

0.60

= 1000 ml/min

The quantity (or amount) of the substance filtered per unit time is termed the filtered load (FL), or
amount filtered. It is equal to the product of the GFR (or C;n) and the plasma concentration of that
substance, or

FL

= GFR

PG

= 120 ml/min x 0.9 mg/ml


= 108 mg/min

Note that the ml terms cancel out with a unit analysis and that it is necessary to express the plasma
glucose concentration (Pc) in mg/ml and not in the unit of mg/dl given in the original data.
165-167. The answers are: 165-C, 166-A, 167-B. [Ch 5 VII A 1,2, 0; VIII 0 1,3, 4; Figure 5-14]
Hyperventilation is defined as alveolar ventilation in excess of the body's need for CO 2 elimination,
which results in decreased arterial peo 2-the underlying factor in respiratory alkalosis. Although Peo 2 is
not given, it can be calculated using the mathematical relationship: [HC0 3 -]IS x Peo 2 = 20/1. Applying
this equation, it is clear that the values in set C indicate a decrease in Peo 2 , as

15

20

- - - = -,

SxPeo 2

or S x Peo 2

0.75 mmol/L

Since
S

Peo 2

0.75 mmol/L

and
S

0.03 mmol/L/mm Hg,

then
Peo 2 =

0.75 mmol/L
0.03 mmol/L/mm Hg

25 mm Hg

A Peo 2 of 25 mm Hg is significantly lower than normal (40 mm Hgl. In acute respiratory alkalosis, there
is a decrease in urinary H+ excretion and an increase in urinary HC03 - excretion. In this case, there
has been complete renal compensation, as evidenced by the return of the [HC0 3 -]IS x Peo 2 ratio to
normal (20:1).
In chronic respiratory tract obstruction (e.g., due to tracheal stenosis, foreign body, or tumor), alveolar
ventilation is insufficient to excrete CO 2 at a rate required by the body, which leads to increased arterial
Peo 2-the underlying factor in respiratory acidosis. The kidney increases H+ secretion, resulting in the
addition of HC0 3 - to the ECF. The values in set A coincide with these changes. In this case, there is
partial compensation of the respiratory acidosis as evidenced by the increase in [HC0 3 -]. In chronic
respiratory acidosis, the respiratory centers become less sensitive to hyperca;:>nia and acidosis and rely
on the associated hypoxemia as the primary drive to ventilation. Correction of the low P0 2 by the
administration of O 2 will diminish respiratory drive, resulting in hypoventilation, a further increase in
Peo 2 , and possibly CO 2 narcosis. For this reason, O 2 must be given with extreme caution to patients
with chronic hypercapnia.
Metabolic acidosis exhibits the characteristics of increased [H+] (decreased pH), a reduced [HC0 3 -J,
and a compensatory hyperventilation resulting in hypocapnia. The kidney responds to the increased H+
load by increasing the secretion and excretion of NH4 + and the excretion of titratable acid (H 2 P0 4-).
The values in set B coincide with these changes.
168-171. The answers are: 168-C, 169-A, 170-D, 171-E. [Ch 1 VIII A 2 b, 5] Presbyopia (impairment
of vision due to old age) is caused by a decrease in the elasticity of the lens. As a result, the eyes are
unable to accommodate for near vision. Another condition associated with aging is cataracts, in which
the lens becomes progressively less transparent. Myopia is caused by an overall refractive power that is
too great for the axial length of the eyeball. It causes distant objects to be focused in front of the retina
and can be corrected by a diverging lens. In hyperopia, the overall refractive power is too low for the
axial length of the eyeball, and so the eyes must continuously accommodate to see distant objects
dearly.
172-175. The answers are: 172-C, 173-A, 174-A, 175-B. [Ch 6 IV 03 c-d, 6 a (4), b, F 1 b (5),4 a
(T), 5 c (4) (b)] If water is not properly absorbed in the small intestine, rapid dehydration and consequent
circulatory collapse will result.
Any blockage of the bile duct prevents bile from being secreted and thus prevents the proper absorption
of fat. Malabsorption of fat results in malnutrition and steatorrhea. In addition, when bile is not secreted,
bilirubin cannot be secreted, and the resulting accumulation of bilirubin in the soft tissues leads to
jaundice.
Vitamin Bt2 is required for the normal production of red blood cells (erythropoiesis). Thus, failure to
absorb vitamin B12I caused either by lack of intrinsic factor or by intestinal disease, will lead to pernicious
anemia.
Carbohydrate malabsorption usually results in watery diarrhea and intestinal gas.

Index

Note: A 1 after a page number denotes a table; an (, a figure; an n, a footnote; a Q, a question; and an E, an explanation.

A
Absorption
in colon, 327
from gastrointestinal tract, 303
in small intestine, 315, 322-325
in stomach, 315
Accessory sex organs, 370-371, 376
see also Genitalia, internal
Accommodation, ocular, 54, 85Q, 91 E
ACE, see Angiotensin-converting
enzyme
Acetazolamide, 311, 432Q, 455E
Acetic acid, 257, 292Q, 299E
Acetoacetic acid, 257, 268, 269, 270,
292Q, 299E, 363
Acetyl coenzyme A (acetyl-CoA), and
glucose metabolism, 355-356
Acetylcholine (ACh), 18
as autonomic neurotransmitter, 22
and cardiac conduction, 102
degradation, 21
in gastric secretions, 312, 313(,
328Q, 329Q, 331E, 332E, 333E
and hormone release, 346, 347, 359,
491
and neurosecretory system, 346
and pancreatic secretions, 31 8
in peristalsis, 309, 328Q, 331 E
storage and release, 18-19, 20f
in synaptic transmission, 19-22, 83Q,
89E
synthesis, 18
Acetylcholine receptors, 18, 19(, 20,
22, 83Q, 89E
and pacemaker of heart, 433Q, 457E
Acetylcholinesterase (AChEase), 18,
19(,21
Achalasia, 307
Acid
Br0nsted definition 258
conjugate, 2581
elimination, 257-258
food as source, 257-258, 270
production, 257-258, 289Q, 295E
Acid-base balance, 257-288,
289Q-294Q, 295E-301 E
and aldosterone, 237, 275
organs controlling, 260
renal regulation, 270-276
respiratory regulation, 269-270
role of bone, 401
Acid-base disorders (imbalance),
276-285, 288, 289Q-294Q,
295E-301 E

causes, 286-287, 290Q, 291 Q,


297E-299E
compensatory responses 2771,
285-288, 289Q-291Q, 293Q,
294Q, 295E-296E, 298E-301 E
compensation, versus correction,

285
versus secondary changes, 285
respiratory versus renal, 285, 291 Q,
298E, 299E
in diabetes mellitus, 269, 2741, 276,
292Q, 294Q, 300E, 301 E
effects on cerebrospinal fluid, 260,
291Q, 298E
metabolic, 437Q, 450Q, 453Q, 462E,
471 E, 474E
and pH-[HCa, -] diagram, 284-285,
294Q, 300E-301 E
primary changes
compensatory mechanisms,
285-288, 291 Q, 298E, 300E
versus secondary changes, 277-278,
300E
respiratory, 436Q, 448Q, 453Q,
461 E, 468E-469E, 474E
respiratory versus metabolic, 277,
297E-299E
simple versus mixed, 278
Acid-base physiology, 257-288,
289Q-294Q, 295E-301 E
Acid-base status, evaluation of,
279-285, 293Q, 300E
graphics for, 282
Acidemia, see Acidosis
Acidophils, of anterior pituitary, 352
Acidosis, 276-278, 288, 289Q-293Q,
259E-299E, 436Q, 449Q, 461 E,
470E
circulatory effects, 171
definition, 276
in diabetes mellitus, 269, 2741, 276,
292Q, 294Q, 300E, 301 E, 436Q,
460E, 461 E
from gastrointestinal fluid loss, 433Q,
457E
hydrogen ion concentration in, 2601,
300E
hyperchloremic, 455E
hyperkalemic, 224, 225, 237
metabolic, 225, 237, 2771, 288,
289Q-293Q, 295E-301 E, 438Q,
449Q, 455E, 463E, 470E
ammonium excretion in, 276
and anion gap, 281, 296E
base excess in, 279

causes, 286, 291 Q, 298E, 300E


compensatory responses, 2771, 286,
288, 289Q-291Q, 293Q, 294Q,
295E, 297E-301 E
definition, 277
from diet, 270
glucagon in, 396
and parathyroid hormone, 405
and pH-[HCa, -] diagram, 284-285,
287
relation to carbon dioxide tension,
260, 289Q, 290Q, 295E, 298E
pH in, 2601, 289Q, 295E
respiratory, 2771, 288, 290Q-294Q,
297E-301 E, 433Q, 449Q, 456E,
470E
and anion gap, 281
causes, 286-287, 300E, 301 E
compensatory responses, 2771, 287,
288, 290Q, 294Q, 297E, 300E,
301E
definition, 277
on pH-[HCa, -] diagram, 284-285,
287, 294Q, 301 E
relation to carbon dioxide tension,
260
Acids
foodstuffs as source, 257-258, 270
Acinus, 172-173
Acromegaly, 355, 356
ACTH, see Adrenocorticotropic
hormone
Action potential, 7-18, 83Q, 84Q,
86Q, 87Q, 88E, 89E, 91 E, 92E,
434Q,458E
biphasic, 107
in cardiac muscle, 25, 35, 100-102
components, 8f, 10-11, 87Q, 92E
compound, 36-37
conduction velocity, 13-15, 83Q, 89E
of myocardial cells, 100, 101
through atrioventricular node, 103
definition, 7
in electrocardiography, 107
of myocardial cells, 100-102
phases, 8f, 10-11, 83Q, 87Q, 88E,
92E
see also specific phases (e.g.,
Depolarization)
in myocardial cells, 101-102
propagation along axon, 13-15, 83Q,
89E
and neuron type, 36-37
and receptor adaptation, 36
recording, 8-9, 87Q, 92E

in synaptic transmission, 19-20


Active transport, see under Transport
Acupuncture, and enkephalins, 351
Adenohypophysis (anterior pituitary)
see also Pituitary gland
anatomic relationships, 344f
blood supply, 344, 353
histology, 352-353
hormones of, 354-357
in hypothalamic-hypophysial axis,
345f
regulation by parvicellular
neurosecretory neurons, 347t; 348
Adenosine, and coronary circulation,
133
Adenosine triphosphate (A TP), in
muscle contraction, 31, 84Q, 89E,
90E
in cardiac muscle contraction, 105f
Adenyl cyclase, activation by hormone,
341
ADH, see Antidiuretic hormone
Adiadochokinesia, 77, 87Q, 91 E
Adipose tissue, insulin effects in, 394,
395, 409Q, 416E
Adrenal cortex, 363-370
embryology, 364
in fetus
effects of HCG on, 390
hormones of, 388t; 389, 408Q,
414E
histology, 364
hormones of, 364-370
see also Corticosteroids and 'specific

hormones
morphology, 364, 408Q, 414E
Adrenal hyperplasia, congenital, 437Q,
462E
Adrenal medulla, 357-363
blood supply, 358
embryology, 357
enkephalins in, 351
enzymes of, 407Q, 413E
histology, 358
hormones of, 358-359
see also Catecholamines;
Epinephrine; Norepinephrine
innervation, 347, 358
morphology, 357-358, 408Q, 414E
as neuroendocrine transducer, 347,
357
Adrenalectomy
and catecholamines, 363, 412Q,
419E
Adrenalin, see Epinephrine
Adrenergic agonists
and airway resistance, 159, 187Q,
193E
and heart, 441 Q, 465E
Adrenergic blocking agents
and heart, 441 Q, 465E
Adrenergic receptors, 24, 25, 83Q,
89E
alpha-adrenergic, 24, 25, 83Q, 89E,
3611
beta-adrenergic, 24, 25, 83Q, 89E,
3611
and airway resistance, 159
and renin release, 239
and cardiovascular function, 133,
140Q,144E
and catecholamines, 360, 361, 409Q,
416E
and insulin secretion, 391, 409Q,
412Q, 416E, 419E

physiologic effects, 3611, 412Q,


419E, 437Q, 462E
Adrenocorticotropic hormone (ACTH),
237
see also Hypophysial-adrenocortical
control system
and aldosterone, 237-238
and antidiuretic hormone, 235
in brain, 349
cleavage products, 349
and cortisol secretion, 367-368,
407Q,413E
and endorphins, 349-350
half-life, 339
precursor of, 337, 349
source, 335t
Adrenogenital syndrome, 437Q, 462E
Adrenolytic test, 363
Afterdischarge, reflex, 64
Afterload
skeletal muscle, 33-34, 86Q, 91 E
ventricular, 119, 120t; 145E
Air, composition, 163, 188Q, 194E
Airways
closure, 173-174
and airway diameter, 174
dynamic compression, 159, 160,
161 f, 187Q, 194E
resistance, 158, 159, 186Q, 187Q,
188Q, 192E, 193E, 194E, 195E,
432Q,455E
and airway diameter, 158, 159,
187Q,193E
effect of aging on, 187Q, 194E
and acini, 173
and respiratory tests, 180
and %fEV" 194E
Alanine, 369, 370
Albuminuria, 253E
Aldehyde oxidase, 360
Aldosterone, 131, 142Q, 146E, 205,
235-242, 252E, 364t, 431 Q,
450Q, 454E-455E, 470E-471 E
see also Hyperaldosteronemia,
Hypoaldosteronemia
and acid-base balance, 237, 252E,
275
antagonist, 241-242
control mechanisms, 237-241, 244Q,
250E
and extracellular fluid volume,
236-237
fetal, 388t; 389, 409Q, 416E
half-life, 340
and hydrogen ion secretion, 275
level in pregnancy, 386
metabolism, 236, 367
physiologic effects, 439Q, 464E-465E
and potassium ion, 236, 237, 238,
239, 252E, 327
and renal transport, 437Q, 462E
secretion, stimuli for, 438Q, 464E
and sodium ion, 236, 238, 239,
252E, 327
source, 335t
and sympathetic nervous system, 239
synthesis, 235, 246Q, 254E, 365, 366
effect of endorphins, 350
site of, 364
and water excretion, 236
Aldosterone-stimulating hormone, 240
Aldosteronism (hyperaldosteronism),
241,242
see also Hyperaldosteronemia
Alimentary canal, see Gastrointestinal
tract

Alkalemia, see Alkalosis


Alkalis, foodstuffs as source, 257, 270
Alkalosis, 276-278
hydrogen ion concentration in, 260t,
290Q,296E
hypokalemic, 224, 225, 237
and lactic acid formation, 268
metabolic, 225, 237, 277t, 288,
289Q-294Q, 295E-296E, 299E,
300
base excess in, 279
causes, 286, 294Q, 300E, 301 E
compensatory responses, 277t, 286,
287t; 288, 289Q, 293Q,
295E-296E, 300E
definition, 277
from diet, 270
and pH-[HCO, -) diagram, 284-285,
288, 294Q, 301 E
relation to carbon dioxide tension,
260
pH in, 260t, 289Q, 295E
respiratory, 277t, 288, 289Q-294Q,
295E-301 E, 433Q, 456E-457E
causes, 287, 300E, 301 E
compensatory responses, 277t, 287,
288, 290Q, 294Q, 295E, 296E,
300E, 301 E
definition, 277
and pH-[HCO, -) diagram, 284-285,
287, 294Q, 300E
relation to carbon dioxide tension,
260, 289Q, 295E
and control of respiration, 179, 184
All-or-none law (response), 11, 100
Allergy, glucocorticoids in, 368
Alpha-adrenergic receptors, see under
Adrenergic receptors
Alpha cells of pancreas, 391
Alpha motoneuron, 18, 19t; 37, 68,
69, 84Q, 90E
in withdrawal reflex, 64, 65
Altitude
and cardiovascular function, 184,
443Q,467E
and hypoxia, 181
and mountain sickness, 185
and nitrogen tension, 194E
and oxygen content, 168
and oxygen tension, 194E
and total lung capacity, 184
Alveolar dead space, 164, 175-176
ventilation of, 175
Alveolar gas (air) equation, 1651, 458E,
465E
Alveolar plateau, 173(, 174
Alveolar pressure, 155(, 186Q, 187Q,
192E, 193E
during respiration, 152, 186Q, 189Q,
192E, 195E
Alveolar to arterial oxygen gradient,
effect of exercise, 138
Alveolar ventilation, 164-165, 188Q,
189Q, 194E, 196E, 435Q, 437Q,
448Q, 450Q, 459E, 461 E-462E,
468E-469E, 471 E
see also Ventilation:perfusion ratio
calculation of, 439Q, 465E
Alveoli, and surface forces, 156-158,
186Q, 192E
Amenorrhea, prolactin and, 357
Amine hormones, 338t, 339
see also Catecholamines; Thyroid
hormones
Amine precursor uptake and
decarboxylation (APUD) cells, 348

Amino acids
absorption, 323, 328Q, 331 E
derivatives of, 339
effects of glucocorticoids on, 369,
370
effects of insulin on, 395
and glucagon secretion, 395
and insulin secretion, 392
malabsorption, 323
renal transport, 2101, 211, 212, 216(,
223, 255E
transport across cell membrane, 4
p-Aminohippurate, see
Para-aminohippurate
Ammonia, synthesis, 275
Ammonium ion

in body fluids, 2091


NH,/NH. + buffer, 275
production of, 275
renal transport, 2101, 2251, 226,
243Q, 250E, 433Q, 451Q, 456E,
471E
effect of acetazolamide on,
454E-455E
effect of parathyroid hormone on,
405
urinary excretion, 2741, 275-276,
289Q, 296E, 299E
Amylases
pancreatic, 317, 322
salivary, 303, 304, 305, 315, 322
Amylopectin, 322
Amylose, 322
Anaerobic threshold, 135, 137-138
Anal canal, 326
Anal sphincters, 326-327
Anatomic dead space, 164, 174
Anatomic shunt, 175, 1821, 183,
189Q, 190Q, 196E, 197E
Androgen-binding protein, 376
Androgen resistance syndromes, 410Q,
417E
Androgens
adrenal, 3651, 366
anabolic effect, 377
biologic effects, 377, 408Q, 410Q,
415E,417E
catabolism, 375
as estrogen precursors, 365(, 374,
375,379,381,383
physiologic effects, 376-377, 410Q,
417E
reproductive functions, 372, 376-377,
41OQ,417E
testicular, synthesis, 366, 374-375
Androstenedione, 3651, 366
as estrogen precursor, 365(, 374,
375, 379, 381
in female, 379, 3801
in ovarian steroidogenesis, 380-381
synthesis, 365(, 366, 375
as testosterone precursor, 365(, 375
Androsterone, 366, 375
Anemia, 453Q, 474E
and blood gases, 452Q, 472E
hypoxia from, 1821, 183f, 184,
190Q, 196E, 197E
Anesthesia, and respiration, 449Q,
470E
AN F, see Atrial natriuretic factor
Angina pectoris, 117
Angiotensin, 131
see a/50 Renin-angiotensin system
angiotensin I, 240, 252E, 450Q,
470E-471 E

angiotensin II, 240, 246Q, 252E,


254E
changes in receptor number, 341
effect of estrogens on, 390
physiologic effects, 240, 252E
angiotensin III, 240, 252E
synthesis, 239-240, 246Q, 254E
Angiotensin-converting enzyme (ACE),
131, 240, 252E, 470E
Angiotensinogen, 239-240, 252E
effect of estrogens on, 390
Anion gap, 279-281, 290Q, 296E,
462E
calculation of, 281, 296E
constituents, 280
definition, 279-280
Anions
in body fluids, 208, 2091
in serum, normal values, 280f
Anovulation, and prolactin, 357
ANP, see Atrial natriuretic factor
Anterior lobe of pituitary, see
Adenohypophysis
Antidiuresis, 230, 233
Antidiuretic hormone (ADH), 131,205,
234-235, 244Q, 246Q,
250E-251E, 253E-255E, 353,
450Q, 470E-471 E
control mechanisms, 234-235, 244Q,
246Q, 250E-251 E, 253E-254E
effect of plasma osmolality on, 432Q,
455E
half-life, 339
and hypernatremia, 246Q, 254E
and hyponatremia, 234, 237
and neurosecretory system, 345, 346,
347, 409Q, 417E
source, 3351
structure, 353
synthesis and storage, 353
in urine concentration and dilution,
227, 228, 229(, 230, 247Q, 255E
Antiport (countertransport), 211 (, 212
Antiporters, 5
Antral ring, 309f
Antrum, of stomach, 307(, 330Q,
333E, 438Q, 464E
Anus, 304f
Aorta, 93
Aortic body, in respiratory control,
178, 190Q, 196E
Aortic valve, 119, 141 Q, 143Q, 146E,
148E
insufficiency, 123, 124f
stenosis, 123, 124(, 445Q, 468E
AP, see Action potential
Apnea
after artificial ventilation, 449Q, 470E
central, 181
during sleep, 181, 190Q, 196E
reflex, 179
Apneustic breathing (apneusis), 178,
180f, 190Q, 196E
Apneustic center, 178, 190Q, 196E
Apoferritin, 325
Apraxia, 451Q, 472E
APUD (amine precursor uptake and
decarboxylation) cells, 348
Arginine vasopressin, see Antidiuretic
hormone
Aromatases, 375
ARP, see Refractory periods
Arrhythmias, see Cardiac arrhythmias
Arterial hypoxia, 182-183, 190Q, 197E
Arteries, 93-94

pressure-volume relationships, 93,


94(, 126
renal, 201-202
Arterioles, 94-95, 140Q, 144E
renal, 201, 202
Arteriosclerosis, and hypoxia, 183
Arteriovenous (A-V) shunts, 95, 140Q,
144E
Arteriovenous oxygen difference,
calculation of, 449Q, 470E
Artificial ventilation, 449Q, 470E
Ascending colon, 304f, 326f
Ascorbic acid, see Vitamin C
Aspirin toxicity
gastric, 31 5
metabolic 433Q, 456E-457E
Asthma, and airway resistance, 159,
187Q,194E
Astigmatism, 55, 453Q, 474E
Asystole, 103
Ataxia, 76, 87Q, 91 E
Atelectasis, 156, 157, 162, 183
Atherosclerosis, and myocardial
oxygen, 121
Athetosis, 76
Atmospheric pressure, 163
and alveolar pressure, 436Q,
460E-461 E
ATP, see Adenosine triphosphate
Atria, in cardiac cycle, 121-122,
141Q, 143Q, 146E, 148E
Atrial arrhythmias, 113- 115
Atrial natriuretic factor (ANF; atrial
natriuretic polypeptide; ANP),
131, 242, 335, 450Q, 470E-471E
Atriopeptigen, 242
Atrioventricular (A V) junctional (nodal)
rhythms, 115
Atrioventricular nodal block, 101, 103,
116
Atrioventricular nodal delay, 103
Atrioventricular node, 99, 102(, 103,
141Q,145E
as pacemaker, 102
Atrioventricular valve ring, 99
Atrioventricular valves, 118, 122,
143Q, 148E
insufficiency, 123
Audition, see Hearing
Auditory information, encoding, 50
Auditory neurons, central, 50-51
Auditory ossicles, 45-46, 84Q, 90E
Auditory receptors, 48
Auditory transduction, 48-50
Auerbach's (myenteric) plexus, 307
Autacoid, definition, 343
Autocrine hormones, 335, 336f
Autocrine Signaling (communication),
336(, 343
Automaticity of myocardial cells, 102
factors affecting, 102-103
Autonomic (vegetative) nervous system
and atrioventricular nerve, 103
and cardiovascular function, 132-133
and salivation, 305, 328Q, 331 E
and stomach, 308
synaptic transmission in, 22-25
Autonomic secretomotor neurons,

346-347
AV, see Atrioventricular
Axis deviation, in electrocardiography,
112
Axo-axonic synapse, 27-28, 87Q, 92E
Axon hillock, in synaptic transmission,

26f, 27, 84Q, 89E


Azoospermia, 377

B
Bachman's bundle (interatrial tract),
102('103, 141Q, 145E
Ballismus, 472E
Banding pattern of skeletal muscle,
28-29
Barometric pressure, 163, 164
and cardiovascular function, 443Q,
467E
Baroreceptor (moderator) reflex, 132
Baroreceptors, cardiovascular,

131-132, 142Q, 146E, 147E,


254E
Basal body temperature, 382(, 384
Basal ganglia, 75-76, 87Q, 92E
role in coordinated movement, 69,
70f, 87Q, 92E
Basal lamina, 18
Basal metabolic rate (BMR), 400
Base
see also Acid-base; Alkalis
Br0nsted definition, 258
conjugate, 258t, 279
Base excess (BE), 279
Basic electrical rhythm, see Slow

waves
Basilar membrane of inner ear, 47, 48f,
49f
Basophils, of anterior pituitary, 352
[BB], see Buffer base concentration
BE, see Base excess
Bed wetting, 80, 85Q, 90E
Belching (eructation), 307, 327
Bernoulli effect, 125(, 196E
Beta-adrenergic receptors, see under
Adrenergic receptors
Beta cells of pancreas, 391
adrenergic receptors, 391, 409Q,
416E
biosynthetic organization, 391-392
effect of glucagon on, 391
effect of somatostatin on, 391
Bicarbonate buffer systems, 264, 265,
289Q, 292Q, 295E, 299E
Bicarbonate/carbon dioxide buffer
system, 264
Bicarbonate ion
[HCO, -]IS X PCO, ratio, 261
in acid-base disorders, 3, 277-278,
287-288, 290Q-293Q,
297E-299E, 449Q, 470E
in acid-base abnormalities, 277-278,
289Q-294Q, 295E-298E, 433Q,
453Q, 456E-457E, 474E
in body fluids, 208, 209t
in blood, 259(, 291Q, 293Q, 294Q,
300E, 301 E, 433Q, 453Q, 456E,
474E
calculation of, 262-263, 297E,
435Q, 459E-460E
in cerebrospinal fluid, 259f
in gastric secretions, 311
in pancreatic secretions, 317, 318,
330Q, 332E, 333E
in urine, 276, 289Q, 291Q, 296E,
298E-299E
body's source of, 258
in bone, 264, 401
and carbon dioxide transport,
169-170, 195E
as total carbon dioxide, 270
in hydrogen ion generation, 267
1055 in stool, and noncarbonic acid,
258
in muscle, 264

relationship to carbon dioxide tension


and pH, 259
and renal chloride transport, 226
and renal sodium transport, 224,
271 f, 272f
renal transport of, 2101, 216(, 223,
225-226, 247Q, 248Q, 255E,
433Q,456E
effect of acetazolamide on,
454E-455E
effect of parathyroid hormone on,
405
role in acid-base balance, 270
tubular reabsorption, 271 f, 272-273,
276, 289Q, 296E, 299E
in spermiation, 374
synthesis in kidney, 27
Bicuspid (mitral) valve, 118
Big gastrin (G-34), 311
Bile, 316
cells forming, 318
composition, 319
function, 318
release, 318
storage, 318, 321
Bile acids, 319, 329Q, 332E
conversion to bile salts, 319
enterohepatic circulation, 320f
in lipid emulsification, 323
Bile ducts, 316, 320f
disorders of, 453Q, 474E
Bile pigments, 319
Bile salts, 319, 320, 329Q, 332E
absorption, 324, 328Q, 329Q, 330Q,
33IE, 332E, 333E
enterohepatic circulation, 319, 320f
in lipid absorption, 323-324
and micelles, 323, 324, 329Q, 332E
synthesis rate, 319
regulation, 320, 329Q, 332E
Biliary secretions, 318-321
bile-dependent versus -independent,
320
regulation of, 320, 329Q, 332E
Bilirubin, 319, 321
Biliverdin, 319
Bioaminergic neurons, 345(, 348
Biogenic amines, 358
deamination by monoamine oxidase,
360, 409Q, 417E
Biological clock, 78
BioI's breathing, 180
Biotin, 325
Bipolar cells of retina, 57, 59, 60f
Blastocoele, 386
Blastocyst, 383, 386, 410Q-411Q,
418E
Blood, peripheral pooling, 99
Blood-brain barrier, 96
in respiratory control, 178, 196E
Blood celis, effects of glucocorticoids
on, 368
Blood flow
see also Circulation; Renal blood flow
and hypoxia, 183
pressure-velocity relationships, 94(,
140Q,144E
pulmonary, relation to pressure,
170-171, 189Q, 195E, 196E
regulation, 94-95, 140Q, 143Q,
144E, 148E
resistance to, 125-126, 141 Q, 142Q,

146E, 147E
Blood gases, see Gases; see a/so
Ventilation:perfusion ratio

Blood pressure, 123, 140Q, 141Q,


144E-146E
arterial, 123, 126, 141Q, 143Q,
146E, 148E, 432Q, 455E
physiologic determinants, 126
regulation, 131-133, 218-219
effects of glucocorticoids on, 369
role of kidneys in, 199
venous, 127
in congestive heart failure, 129
relation to venous return, 127f
Blood-testis barrier, 373
Blood urea nitrogen (BUN), and serum
osmolality, 281-282
Blood volume
calculation of, 203, 443Q, 467E
in congestive heart failure, 129
regulation, 133
Body (corpus) of stomach, 307f
Body fat
calculation of, 202, 443Q, 467E
effects of glucocorticoids on, 370
Body fluids, 202-209
see also Water; Body water; and
specific kinds (e.g., Extracellular
fluid)
in biliary secretion, 320
compartments, 202-203, 206f
volume measurement, 203
composition, 208(, 209t, 244Q, 251 E
hydrogen ion concentration,
259-260, 290Q, 296E
ionic composition, 1I, 208, 2091,
245Q, 252E
gastrointestinal loss, 433Q, 457E
osmolality, disturbances of, 205-209,
244Q, 246Q, 252E, 254E
pH, 2S7, 259-260, 291Q, 298E
volume, disturbances of, 205-209,
244Q, 246Q, 252E, 254E
Body temperature, basal, 382(, 384
Body water
see also Water; Body fluids
and antidiuretic hormone, 234, 244Q,
250E-25IE
deficit, 205; see also Dehydration
distribution, 202, 203(, 245Q, 252E,
253E
total (TBW), 202, 203t; 246Q, 253E,
431 Q, 443Q, 454E, 467E
volume measurement, 204
Body weight, and insulin levels, 393
Bohr effect, 168
Bohr's method for measuring dead
space, 164
Bombesin (gastrin-releasing peptide,
GRP), 313t; 314, 330Q, 333E
Bone
in acid-base regulation, 264, 295E,
401
and calcium ion storage, 401, 402
cell types, 403-404
chemistry, 402
effect of estrogen on, 385
effect of testosterone on, 377, 385
effect of thyroid hormones on, 400
effect of calcitonin on, 405
effect of calcitriol on, 406
effect of parathyroid hormone on,
403, 404-405, 406, 408Q, 415E
embryology, 402
and phosphorus storage, 401, 402
structure, 402
Bowman's capsule, structure, 200,
201f

Boyle's law, and respiratory forces,


150,151
Bradycardia, 11 3
Bradykinin (kallidin-9), 96, 97, 240,
246Q,254E
Brain
see also Cerebral cortex
blood flow, 134, 143Q, 14BE
effect of thyroid hormones on, 400,
411Q,418E
Brain death, 80-81
Brain stem, 71-72
and cardiovascular function, 132
and respiratory control, 178, 190Q,
196E
and coordinated movement, 69, 70f
Breast
effect of estrogen on, 385, 390
effect of progesterone on, 385, 386,
390, 409Q, 416E
Breast-feeding
and amenorrhea, 357
and oxytocin release, 353, 354
and prolactin release, 356
Breathing patterns, 180-181, 190Q,
196E
see also Respiration
BnJ)nsted acid-base relationships, 258
Bromocriptine, 356, 357
Bronchiole, muscle contraction in, 25
Bronchitis, chronic
cause 164
and airway resistance, 159, 187Q,
194E
Brunner's glands, 321
Brush border of intestinal mucosa, 315
Buffer acids, 256
Buffer base concentration ([BB]), 279
Buffer systems of body, 263-267,
269Q, 292Q, 295E, 299E
distribution, 264-265
Buffering, isohydric, 266
Buffers
definition, 263
of blood, 263-264, 269Q, 295E
bicarbonate versus non bicarbonate,
2631, 264-265, 269Q, 295E, 299E
distribution, 264-265
of interstitial fluid, 265, 292Q, 299E
of intracellular fluids, 265, 289Q,
295E, 299E
of lymph, 265, 299E
of plasma, 264-265, 299E
in renal tubule, 226, 243Q, 250E
in tissues, 264, 292Q, 299E
Bulk flow, 7
in microcirculatory exchange, 96-96
BUN, see Blood urea nitrogen
Bundle branch blocks, 11 7
Bundle branches, 102f, 104, 140Q,
145E
Bundle of His, 99, 101, 102f, 104,
140Q, 141Q, 145E

c
C (parafollicular) cells of thyroid, 396
C-peptide, 392
Cable properties of cell membrane,
16-16
Calcidiol (calcifediol), see
2S-Hydroxyvitamin 0,
Calcilerols, see Vitamin 0
Calcitonin, 401, 402-405
cells secreting, 396, 402

and calcium absorption, 405


and phosphate absorption, 405
and renal transport, 437Q, 462E
effects of calcitriol on, 406
physiologic actions, 405
and plasma calcium level, 402, 403f,
406Q,415E
Calcitriol, see 1,25-0ihydroxyvitamin
OJ; Vitamin 0
Calcium bilirubinate gallstones, 321
Calcium-binding protein, 406
Calcium ion
see also Electrolytes; Ions
and action potential, 27-28
in heart, 25
in body fluids, 206, 2091, 402
in extracellular fluid, 401
in intracellular fluid, 401
in plasma, 401-402, 406Q, 415E
and calcitonin level, 402, 403f
and parathyroid hormone level,
402, 403f, 451Q, 472E
and vitamin 0 level, 451Q, 472E
in bone, 401, 402, 403
chemistry, 402
effect of calcitriol on, 406
effect of parathyroid hormone on,
404
daily intake, 325
distribution, 401-402
effects of growth hormone on, 356
effects on heart, 25, 34-35, 100,
101-102,103,104, 105f, 142Q,
147E
and glucagon secretion, 393
gradient across cell membrane, 401
and insulin secretion, 392, 393
intestinal absorption, 325, 451 Q,
471E
calcitonin and, 405
calcitriol and, 405, 406Q, 415E
effect of calcitriol on, 406
parathyroid hormone and, 405,
408Q,415E
in muscle contraction
in skeletal muscle, 29, 30, 31-33
in cardiac muscle, 34-35, 104, 10Sf,
142Q,147E
in smooth muscle, 22, 25, 35
in physiologic processes, 401
regulation of, 402-403
renal transport, 210t
effect of calcitonin on, 405, 408Q,
415E
effect of calcitriol on, 405
parathyroid hormone and, 405,
408Q,415E
and sarcoplasmic reticulum, 22, 25,
29, 62Q-84Q, 88E-90E
transport across cell membrane, 4-5
Calmodulin, 35, 392
cAMP, see Cyclic AMP
Capillaries, 95, 140Q, 143Q, 144E,
148E
peritubular, 201-202
Capillary hydrostatic pressure, 96, 97,
143Q, 148E
Carbamino compounds (carbamates),
269
Carbaminohemoglobin, 267
Carbohydrate
see also Starch
absorption, 322, 326Q, 331E
dietary intake, 322
digestion in small intestine, 322
digestion in stomach, 315

malabsorption, 322, 453Q, 474E


metabolism
effect of catecholamines, 361, 362,
363, 407Q, 413E
effect of glucagon, 396
effect of glucocorticoids, 369-370,
407Q, 406Q, 413E, 415E, 416E
effect of growth hormone, 355
effect of insulin, 393-394, 395,
409Q,416E
effect of thyroid hormones, 400-401
and insulin secretion, 392
Carbon dioxide
in acid-base abnormalities, 277-276,
291 Q-294Q, 296E-301 E, 432Q,
433Q, 439Q, 450Q, 456E, 465E,
471 E
form in blood, 269-270
body's source of, 257
in bone, 264
output, effect of exercise, 136
and pH, 283
production of, 267-268
pulmonary diffusion, 166, 169Q,
195E
in respiratory control, 176, 179,
190Q, 196E
solubility coefficients, 167t
total concentration, 277n, 291Q,
294Q, 296E, 299E, 301 E
in acid-base abnormalities, 277,
276, 291Q, 293Q, 294Q,
296E-301 E, 449Q, 470E
transport in blood, 169-170, 166Q,
169Q,195E
as weak acid, 257, 269Q, 295E
Carbon dioxide/bicarbonate buffer
system, 262-283
Carbon dioxide combining power, and
bicarbonate, 270
Carbon dioxide-forming acids, 270
Carbon dioxide partial pressure
(tension; Peo,)
in acid-base abnormalities, 30, 277,
276, 289Q-294Q, 295E-299E,
433Q,456E
alveolar, 165, 186Q, 169Q,
194E-197E, 432Q, 455E
of blood, 165, 168Q, 190Q, 195E,
197E, 259f, 260, 291 Q, 293Q,
294Q, 296E, 300E, 301 E, 432Q,
437Q, 439Q, 452Q, 455E, 462E,
465E, 472E
of cerebrospinal fluid, 259f, 260,
291Q,296E
in hypoxia, 162, 184
measurement of, 263, 290Q, 296E,
434Q,458E
relation to hydrogen ion
concentration, 260
relation to pH and bicarbonate ion,
259, 290Q, 296E
in respiratory control, 176, 179, 164,
190Q,196E
and ventilation:perfusion ratio,
175-176
versus combining power, 270
Carbon dioxide response test, 179-160
Carbon monoxide
combining with hemoglobin, 166-169
and respiratory control, 176
solubility coefficients, 167t
Carbonate ion, in bone, 264, 295E,

401
Carbonic acid, 257, 267-266, 270,
295E

buffering of, 268, 270, 289Q, 295E


and carbon dioxide transport, 169,
170t;195E
formation, 257
in gastric secretions, 311
Carbonic anhydrase
in gastric secretions, 311
inhibitors, 432Q, 455E
and renal bicarbonate transport, 224,
226,273
tissues containing, 261 n
Carboxyhemoglobin (HbCO), 168-169
and hypoxia, 183
Cardia of stomach, 307f
Cardiac, see also Heart; Myocardium
Cardiac arrhythmias, 113-116
atrial, 113-115
AV junctional (noda!), 115
sinus, 113
ventricular, 115-116
Cardiac cycle, 121-123, 141Q, 143Q,
146E-148E, 447Q, 468E
Cardiac dipole, in electrocardiography,
107
Cardiac function curves, 127t; 128
Cardiac output, 127-131, 140Q,
141Q, 145E, 146E, 431Q, 454E
in congestive heart failure, 129
effect of venoconstriction, 99
effect of exercise, 137
and Frank-Starling effect, 106, 142Q,
147E
and hypoxia, 182, 190Q, 197E
measurement of, 130
relation to arterial blood pressure,
126, 141Q, 146E
relation to venous blood pressure,
127f
Cardiac shock, 136
Cardiac standstill, 103
Cardiodynamics, 121-124
Cardiovascular function curves,
128-129
Cardiovascular system, 93-139,
140Q-143Q, 144E-148E
blood pressure-flow relationships, 94f
changes at birth, 135
regulation of, 131 -13 3
functions, 93
responses to hypoxia, 184
in thermoregulation, 93
Carotid arteries, 134
Carotid body, 178, 190Q, 196E
Carotid sinus, 134, 140Q, 145E
and arterial pressure, 131, 132
Carrier-mediated transport, 3-5, 82Q,
88E
in kidney, 212, 246Q, 254E
types of carriers, 5
Cataplexy, 80
Cataracts, 453Q, 474E
Catecholamines, 338t, 339, 358-359,
407Q, 409Q, 412Q, 413E, 416E,
417E,419E
see also Epinephrine; Norepinephrine;
Sympathomimetic drugs
and adrenergic receptors, 360, 361,
409Q,416E
biochemical effects, 361-363
and carbohydrate metabolism, 361,
362, 407Q, 414E
circulation, 339
diabetogenic effect, 362
and insulin, 361, 362, 432Q, 455E
effects of glucocorticoids on, 369
and gluconeogenesis, 362

half-life, 339
metabolism, 360, 409Q, 417E
metabolites in urine, 360
in pheochromocytoma, 363, 432Q,
455E
physiologic effects, 361, 409Q, 416E
and renin release, 239
secretion, 339, 359-360, 363, 412Q,
419E
source, 335t
storage, 339
structure, 339
synthesis, 339
and thyroid hormones, 341
Catecholestrogens, 381
Catechol-O-methyltransferase (COMT),
360, 412Q, 419E, 438Q,
463E-464E
Cations
in body fluids, 208, 209t
in serum, normal values, 280f
exchange, and renal sodium transport,
224
CBC, see Cortisol-binding globulin
CCK, see Cholecystokinin
Cell membrane, 1-2
and calcium ion, 401
electrical potentials, 7-18, 87Q, 92E
see also Action potential
electrical analog, 10f, 15
of heart, 100-102, 439Q, 465E
in electrocardiography, 107
nonpropagating, 16, 17
resting potential (resting membrane
potential; RMP), 7-18, 87Q, 92E
calculation of, 15- 16, 82Q, 88E
definition, 7, 9
factors affecting, 9
of myocardial cells, 100, 101,
439Q,465E
recording, 8-9
and slow waves (basic electrical
rhythm), 308-309
permeability, effect of hormones on,
341
polarization
effect of insulin on, 395
effect of potassium ion, 10
transport across, 2-7, 82Q, 88E
in kidney, 210-212
energy requirements, 212, 254E
transport systems, 210-211
Cell volume, and osmotic pressure,
5-6, 82Q, 88E
Cellular homeostasis, 1-7
Cellulose, 322
Central nervous system, 25-28
see also components (e.g., Brain;
Cerebral cortex; Spinal cord)
and cardiovascular function, 132
endorphins in, 349, 350
enkephalins in, 351
CephaliC phase of gastric secretion,
310,311, 328Q, 331E
Cephalic phase of pancreatic secretion,
318
Cerebellum, 76-77
in coordinated movement, 69, 70t;
87Q, 91 E
Cerebral cortex
see also Brain
in cardiovascular function, 132

in motor control, 7776


Cerebrocerebellum, 76, 77
Cerebrospinal fluid

in acid-base abnormalities, 260,


291Q,298E
bicarbonate ion in, 259f
buffering properties, 260, 292Q, 299E
carbon dioxide tension, 259t; 260,
291Q, 298E
hydrogen ion in, 259t, 260t
pH, 259t; 259t, 260t, 289Q-291Q,
295E, 296E, 298E
pressure, and cerebral circulation,
134
Cervical mucus
effect of estrogens on, 385
effect of progesterone on, 386
ferning pattern, 385
Chemoreceptors
in respiratory control, 178-179, 184,
190Q,196E
as trigger zone for vomiting, 310
Chest wall
receptors of, 179
in respiratory mechanics, 149, 153,
154, 155t; 186Q, 192E, 197E
Chewing (mastication), 303-304,
328Q,331E
Chewing reflex, 303, 435Q, 460E
Cheyne-Stokes respiration, 180
Chief (peptic) cells, 311
Chloride ion
and action potential, 27-28
in body fluids, 208, 209t
in gastric secretions, 311, 312f
intestinal absorption, 324, 327
and renal sodium transport, 224, 226,
273
renal transport of, 21 Ot, 211, 21 6t;
223, 226, 248Q, 255E
in urine concentration and dilution,
227-231
Chloride shift, and carbon dioxide
transport, 169-170, 189Q, 195E
Cholecalciferol, see Vitamin D J
Cholecystectomy, and bile secretion,
321
Cholecystokinin (CCK), 310,
328Q-330Q, 331 E-333E
and bile flow, 318, 320
CCK-8,318
CCK-33, 318
and slow waves, 317, 329Q, 332E
in gallbladder contraction, 321,
330Q, 333E
and pancreatic secretions, 318
Cholera enterotoxin, 322
Choleretics, 320
Cholesterol
in bile, 319
intestinal absorption, 451 Q, 471 E
and micelles, 323, 324, 329Q, 332E
plasma levels, and thyroid hormone
levels, 401
re-esterification, 324
in steroidogenesis, 340, 365, 374,
380
in fetal hormone synthesis, 388f
in placental hormone synthesis, 386,
388f, 389
Cholesterol esterase, 323
Cholesterol gallstones, 321
Cholesterol lipase, 317
Cholinergic blocking agents, effects on
heart, 441 Q, 465E

Cholinergic

neuron~

and airway resistance, 159


and pacemaker, 99

Cholinergic neurotransmitters, and


neurosecretory system, 346
Chorea, Huntington's, 76
Chorionic gonadotropin (HCG)
and corpus luteum, 390, 411 Q, 418E
and 17a-hydroxyprogesterone, 389
and progesterone, 389
in male fetus, 372, 374
physiologic effects, 390, 411 Q, 418E
in pregnancy, 386, 387, 390, 418E
as index of embryo status, 387
Chromaffin cells (pheochromocytes),
357, 358
tumors of (pheochromocytoma), 363,
412Q,419E
Chromaffin granules, 358
Chromatin, interaction with hormones,
342
Chromogranins, 358
Chromophils of anterior pituitary, 352
Chromophobes of anterior pituitary,
353
Chronic bronchitis, 164
see also Chronic obstructive
pulmonary disease
Chronic obstructive pulmonary (lung)
disease (COPD, COLD), 432Q,
452Q, 455E, 472E
and airway resistance, 159, 160,
187Q,194E
cardiovascular effects, 138
forced vital capacity in, 163f, 180
and frequency-dependent
compliance, 173
residual volume in, 163(, 197E
response to exercise, 138, 139f
total lung capacity in, 163f
Chronic respiratory tract obstruction,
453Q,474E
Chylomicrons, 324, 328Q, 331 E
Chyme, 308
and bile release, 31 8
in colon, 326
in gallbladder contraction, 321
in small intestine, 316, 318, 329Q,
332E
in stomach, 309
Chymotrypsinogens, 31 7
Chymotrypsins, 317
Cimetidine, 31 2
Circadian (diurnal) cycles, 78, 343,
446Q,468E
of ACTH, 367
of corticosteroids, 343, 367
of cortisol, 367
Circulation
see also Blood flow
changes at birth, 13 5
coronary, 133-134
pulmonary, 170-172, 188Q, 189Q,
195E
role of placenta, 135
Circulatory beds, 133-135
Circulatory-interstitial exchange, 96-98,
140Q,144E
Clearance, definition, 212
see also Renal clearance and specific
substances (e.g., Inulin clearance)
Clearance ratio, 214-215
Closing capacity, and lung volume,
191Q,197E
Coactivation of alpha and gamma
motoneurons, 68

Cochlea, 47-48
Collagen, role in respiratory mechanics,
156

Collecting duct of renal tubule, 200,


247Q, 254E, 255E
in urine concentration and dilution,
227, 228(, 229(, 230, 247Q,
254E, 255E
Colloid oncotic pressure, 7, 82Q, 88E
Colloid osmotic pressure
of plasma, 435Q, 459E
in renal transport, 217, 245Q, 253E,
435Q, 458E-459E
Colloids
in plasma, 253E, 254E
of thyroid, 396, 399
Colon, 304(, 320(, 326-327, 330Q,
333E
absorption in, 327
gas (flatus) in, 327
motility, 326
neural control, 326
secretion in, 327, 330Q, 333E, 433Q,
457E
Color perception, 58
Color blindness, 58
Coma, 80
Common bile duct, 316, 319, 320f
Compensation, see Acid-base
disorders, compensation
Compliance (distensibility)
arterial, 432Q, 442Q, 455E, 466E
of elastic arteries, 93
frequency-dependent, 173, 440Q,
465E
relation to elastance, 156, 192E
respiratory (lung), 154, 173, 186Q,
187Q, 192E-194E, 432Q, 439Q,
440Q, 455E, 465E
ventricular, 442Q, 466E
Conception
and corpus luteum, 383, 410Q-411Q,
418E
and ovarian cycle, 383, 41 OQ-411 Q,
418E
Conceptus, 386, 387f
Conducting zone (dead space), 164
Conduction velocity, see under Action
potential
Cones, of retina, 56, 58, 86Q, 91 E
Congenital adrenal hyperplasia, 437Q,
462E
Congestive heart failure
and cardiovascular function, 129,
142Q,146E
and hypoxia, 183
Conjugate acids, 258t
Conjugate bases, 258t, 279
Conn's syndrome, 241
Connective tissue, in respiratory
mechanics, 156
Consciousness, 78, 80-81
disturbances of, 80-81
Contractility, cardiac, 106, 120(,
142Q, 147E, 446Q, 468E
see also Muscle, contraction
relation to cardiac function, 128, 129
and stroke volume, 119
Convection, 210, 244Q, 251E
Convoluted tubules, see under Renal
tubules
COPD, see Chronic obstructive
pulmonary disease
Cor pulmonale, 118
Cornea, 52-53
Coronary bypass operations, 133
Coronary circulation, 133-134
Coronary insufficiency, 117-118,
141Q, 146E

and hypoxia, 183


Corpus (body) of stomach, 307f
Corpus luteum, 379, 384, 410Q-411Q,
418E
and conception, 383, 41 OQ-411 Q,
418E
and chorionic gonadotropin, 390,
411Q,418E
and luteinizing hormone, 384,
410Q-411Q,418E
in postovulatory phase, 384, 385
in pregnancy, hormones secreted,
389, 410Q-411Q, 418E
regression, 384, 431 Q, 454E
Cortical nephron, 200
Corticosteroid-binding globulin (CBG,
cortisol-binding globulin,
transcortin), 236, 364-365
Corticosteroids, 364-367
see also Cortisol; Glucocorticoids;
Mineralocorticoids
circadian cycle, 343, 367
and insulin levels, 393
metabolism, 367
in pregnancy, 386
synthesis, 365-367
Corticosterone, 364, 365(, 366
Corticotropin-like intermediate lobe
peptide (CLIP), 349
Corticotropin, see Adrenocorticotropic
hormone
Corticotropin-releasing hormone
(CRH), 237, 359, 367
Cortisol, 364t, 408Q, 415E
see also Glucocorticoids;
Hypophysial-adrenocortical
control system
and ACTH secretion, 367-368, 407Q,
413E
and adrenal medulla, 435Q, 459E
antianabolic effect, 370, 407Q,
411Q, 413E, 418E
and catecholamine synthesis, 359,
407Q, 413E, 435Q, 436Q, 459E,
461E
effect of estrogen, 386
fetal, 388(, 389, 408Q, 414E
half-life, 340
metabolism, 367
and placental progesterone, 386,
409Q,416E
source, 335t
synthesis, 365f, 366
site of, 364
transport, 364-365
versus corticosterone, 366
Cortisol-binding globulin (CBG,
transcortin, corticosteroid-binding
globulin), 236, 364-365
effect of estrogen on, 386
and placental progesterone, 386
Cotransport (symport), 211 f, 21 2,
246Q, 254E
Countercurrent exchangers, 227,
230-231, 246Q, 254E
Countercurrent multipliers, 227-230,
246Q,254E
Countertransport (anti port), 211 (, 212
Creatinine
clearance, 212(, 216-217, 243Q,
245Q, 247Q, 249E, 250E, 252E,
255E
as estimate of glomerular filtration
rate, 213, 216, 217
versus excretion, 21 6
plasma concentration, 21 6, 21 7

production, 199, 208f, 2091


CRH, see Corticotropin-releasing
hormone
Crib death, 181
Cross-bridges
in myocardium, 100f, 104, 105f
role in muscle contraction, 30-32, 33
in skeletal muscle, 28, 30-31
Crossed extensor reflex, 65
Crypts of Lieberkuhn, 321
Crystalloids in plasma, 253E, 254E
Cupula, vestibular, 73, 74f
Cushing reflex, 134
Cutaneous reflexes, 64-65, 84Q, 90E
Cutaneous sensation, 40-44
Cyanide, and hypoxia, 183, 190Q,
197E
Cyanosis, 182, 188Q, 195E, 290Q,
297E
Cyclic AMP (cyclic adenosine
3',5'-monophosphate, cAMP), and
hormones, 340f, 341-342, 393
Cytopempsis, 98
Cytotrophoblast, 386, 387f

D
Dalton's law, 163
Days of ovarian (menstrual) cycle, 381,
382f, 410Q, 417E
Dead space (conducting zone), 164
alveolar, 164, 175-176
anatomic, 164, 174
ventilation of, 164, 175, 188Q, 194E
Deafness, 51
Decerebrate rigidity, 72, 74
Decomposition of movement, 77
Defecation, 326-327
neural control, 327
Deglutition, see Swallowing
Dehydration, 205, 244Q, 252E, 436Q,
461E
effect on acid-base balance, 269
hyperosmotic, 205, 206f, 2061, 207
hyposmotic, 206f, 2061, 207
isosmotic, 205, 206f, 2061
signs, 207
7-Dehydrocholesterol, 403
Dehydroepiandrosterone (DHEA), 364,
365f, 366
in female, 3801
fetal, 389-390, 408Q, 414E
in ovarian steroidogenesis, 380

placental, 388f
Dehydroepiandrosterone sulfate
(DHEAS), 3641, 366
as estrogen precursor, 390
fetal, 364, 387f, 389-390, 408Q,
414E
Delta cells of pancreas, 391
Denervation hypersensitivity, 132, 360
Dense bars, of synapse, 18, 19f
Deoxycholic acid, 319
Deoxycorticosterone (DOC), 3641, 386
ll-Deoxycortisol, 3641
Deoxyhemoglobin, 167, 170
buffering role, 264, 266
and cyanosis, 182
Depolarization
of action potential (upstroke phase),
8f, 11, 12, 83Q, 86Q, 87Q, 89E,
91 E, 92E
at end-plate, 21
of auditory hair cells, 49-50

in electrocardiography, 107, 108f,


109f, 140Q, 141Q, 145E, 433Q,
456E
of myocardial cells, 100, 101, 104,
141Q, 145E, 439Q, 465E
of pacemaker cells, 102, 140Q, 145E
of ventricles, 104
of vestibular hair cells, 73
Descending colon, 326f
Desynchronized (fast-wave) sleep, 79
Dexamethasone, and ACTH secretion,
367
DH EA, see Dehydroepiandrosterone
DHEAS, see Dehydroepiandrosterone
sulfate
Diabetes mellitus, 436Q, 438Q, 460E,
461 E, 463E
acid-base abnormalities in, 269, 2741,
276, 292Q, 294Q, 300E, 301 E
hyperkalemia, 395
ketoacidosis, 2741, 276, 292Q,
294Q, 300E, 301 E, 436Q, 453Q,
460E,474E
amino acids in, 395
diuresis in, 225
fat metabolism in, 394
gestational, 355
and growth hormone, 355
and somatostatin, 391
Diaphragm, 149, 150f, 186Q, 192E
in respiratory control, 176
Diarrhea
and acid-base imbalance, 291 Q,
298E-299E
and bile salt losses, 320
and intestinal carbohydrates, 322
and intestinal fluid losses, 322
Diastasis, 122
Diastole, 99,122,123, 141Q, 143Q,
146E, 148E, 447Q, 468E
Diastolic pressure, 123, 142Q, 143Q,
147E, 148E
Diastolic reserve volume, 442Q, 466E
Dicrotic notch (incisura), 122, 141 Q,
146E
Diffusing capacity of lungs, 432Q,
455E
Diffusion across cell membrane, 2-3,
82Q,88E
in microcirculatory exchange, 96,
140Q,144E
in lung, 165-166, 189Q, 190Q, 195E,
197E
in hypoxia, 182
Diffusion capacity of lungs test, 165,
184, 190Q, 197E
Diffusion coefficient, 1651, 166, 189Q,
195E
Diffusion potential, 9, 101
Digestion, 303
saliva in, 305
in small intestine, 315, 322-325
in stomach, 315, 434Q, 458E
Digestive system, 303, 328Q-330Q,
331 E-333E

see also individual components


Digitalis, 105, 106
Dihydrocortisol, 367
Dihydrotestosterone
and development of female
phenotype, 377
and development of male phenotype,
372
effects in fetus, 3761, 377
effects in puberty, 3761
in female, 3801

physiologic effects, 3761


synthesis, 374, 375, 408Q, 415E
Dihydroxychenodeoxycholic acid, 319
1,25-Dihydroxycholecalciferol
(1,25-dihydroxyvitamin D 3 ,
calcitriol), 199, 325, 3381, 403,
407Q,414E
half-life, 340
and parathyroid hormone, 405, 406,
408Q,415E
and calcium absorption, 405
and phosphate absorption, 405
and calcitonin, 406
physiologic actions, 404f, 405-406,
408Q,415E
effect on bone, 406
effect on intestinal function, 406
effect on renal function, 406, 437Q,
462E
source, 335
Diiodotyrosine, 399
Dipeptidyl carboxypeptidase, see
Angiotensin-converting enzyme
Diphosphoglycerate (DPG), in hypoxia,
184
Direct energy utilizing transport, 3, 4f
Disaccharides, 322
Dissociation (ionization) constant, 259n
actual (K) versus apparent (K'), 261
Distal sodium-potassium (Na+ -K+)
exchange process, 236
Distal tubule, see under Renal tubules
Distensibility, see Compliance
Diuresis, 230, 233
in diabetes mellitus, 225
Diurnal, see Circadian
DNA, hormonal effects on, 342
DOC, see Deoxycorticosterone
Donnan equilibrium, 7, 8f, 86Q, 91 E
l-Dopa, and catecholamine synthesis,
359
Dopamine
and APUD cells, 348
and catecholamine synthesis, 359,
407Q,413E
and Parkinson's disease, 76
and prolactin inhibiting factor, 357
Dopaminergic neurons, and
neurosecretory system, 345
Dorsal respiratory group (DRG), 176,
177
Downstroke, see Repolarization
Dreaming, 79, 85Q, 90E
Drug dosage determinations, 217
Ductal cells of pancreas, 317, 318
Ductus arteriosus, 135
Duodenal ulcer, 330Q, 333E
Duodenum, 304f, 307f, 315, 320f,
330Q, 332E
in enterohepatic circulation, 319,
320f
Dwarfism, 356
Dynamic equilibrium (steady state),
343
Dynorphins, 349, 351
Dysmetria, 77, 87Q, 91 E
Dyspnea, 179, 188Q, 195E
Dystonia, 76

E
Ear, 45-48
external, 45
inner, 47-48
see also Vestibular system

middle, 45-46, 84Q, 85Q, 90E, 91 E


EC, see Excitation-contraction
ECF, see Extracellular fluid
ECG, see Electrocardiogram
Edema, 96-97
and aldosterone, 236, 237, 241
in congestive heart failure, 129
dependent, 99, 119, 290Q, 297E
pulmonary, 119, 439Q, 465E
in mountain sickness, 185
and zones of lung, 171
and renin release, 239
EEG, see Electroencephalogram
Effective filtration pressure (EFP), 217
Effective renal blood flow, 223
Effective renal plasma flow, 223
Effort-independent flow, 159-160,
187Q, 193E, 194E
Einthoven's triangle, 108, 109f
EKG, see Electrocardiogram
Elastance, 156, 187Q, 192E, 194E
arterial, 93f, 126, 442Q, 466E
ventricular, 442Q, 466E
Elastance forces, 152f
in emphysema, 154
Elastance work of breathing, 161
Elastic arteries, 93-94
Elastic constant, 126
Elastic modulus, 432Q, 455E
Elastic recoil
and airway dimensions, 160
of elastic arteries, 93
Electrical activity of cell membrane,
7-18, 87Q, 92E
see also Action potential; Resting
potential
in electrocardiography, 107
in heart, 100-102, 141 Q, 145E,
439Q,465E
Electrocardiogram
and cardiac cycle, 121
components (waves and intervals),
110-111, 140Q, 145E
Electrocardiography, 107-113, 140Q,
141Q, 145E, 146E
leads, 107-110
principles, 107, 433Q, 435Q, 456E,
459E
Electrogenic cotransport, 212
Electrolytes
see also Ions, Solutes; and individual
substances (e.g., Sodium ion)
in body fluids, 209t
in bile, 319, 320
in gastric secretions, 311, 312f,
434Q,457E
in pancreatic secretions, 317
in saliva, 305
in serum, 280f
intake, 199
intestinal absorption, 324
intestinal secretion, 322
renal excretion, 199
renal transport, 209-212, 223-226
effect of acetazolamide, 454E-455E
effect of hormones, 437Q, 462E
Electroneutral cotransport, 21 2
Electroneutral countertransport, 212
Electroneutrality law, 279-280
Electrotonic potentials, 16- 18
Embryoblast, 386, 387f
Emesis (vomiting), 310, 328Q, 331E

Emphysema, 455E
see also Chronic obstructive
pulmonary disease

and airway resistance, 159, 432Q,


455E
and compliance, 154
End-diastolic pressure, 122, 141 Q,
142Q, 145E, 147E
End-diastolic volume, 105, 119, 120f,
123, 141Q, 145E, 442Q, 466E
End-plate, see Motor end-plate
End-plate potential (EPP), 20, 21
End-systolic volume, 120f, 123, 442Q,
466E
Endocrine control, 343-348
Endocrine organs, 335
embryonic origins, 337, 340
neural control, 346-348
as transducers, 346, 347f
Endocrine pancreas, 317, 391-396,
408Q,414E
Endocrine placenta, 386-391, 407Q,
414E
Endocrine signaling, 336f
Endocrinopathies, 437Q, 462E
catecholamine-associated, 363,
412Q,419E
growth hormone-associated, 356
postpartum, 433Q, 456E
prolactin-associated, 356
Endolymph of vestibular system, 47,
73, 74f
Endometrial cycle, 382f, 384-385
proliferative phase, 382f, 384, 385
secretory phase, 382f, 385
Endometrium, 379
and decidua, 386
effect of estradiol, 385
effect of progesterone, 385, 386, 390
in ovarian (menstrual) cycle, 381,
382f, 383, 384, 385
and placenta, 386, 387f
Endoplasmic reticulum, smooth, 324
Endorphins, 349-350, 409Q, 417E
in brain, 350
chemistry, 349, 409Q, 417E
a-endorphin, 349
,B-endorphin, 349, 409Q, 417E
distribution, 350, 409Q, 41 7E
"y-endorphin, 349
physiologic effects, 350
in prohormone complex, 337
Enkephalins, 349, 350-352
in central nervous system, 351
chemistry, 350-351
distribution, 351
physiologic effects, 351
Enteric nervous system, 25

neurotransmitter in, 22, 25


and receptive relaxation, 308, 329Q,
332E
Entero-oxyntin, 313, 329Q, 332E
Enterocytes, 31 5
and chylomicrons, 324
and lipid absorption, 323, 324
Enterogastric reflexes, 309, 431 Q,
454E
Enterogastrones, 310
Enterohepatic circulation, 319, 320f
Enterokinase, 323
Enzymes
see also types (e.g., Lipases)
in carbohydrate metabolism
glucocorticoids and, 369
in gluconeogenic pathway, 369
in hormone actions, 341-342

Eosinophils, of anterior pituitary, 352


Epinephrine (adrenalin), 407Q, 413 E
see also Catecholamines;
Sympathomimetic drugs
and airway resistance, 159
and cortisol, 435Q, 436Q, 459E,
461E
and endocrine pancreas, 391
and fat metabolism, 362
and glucagon, 361 t, 362, 396, 411 Q,
418E
and glucose, 436Q, 460E
and glycogenolysis, 3611, 362, 407Q,
411Q, 414E, 418E
half-life, 360
and hyperglycemia, 362, 411 Q, 418E
and insulin secretion, 391, 396,
411Q,418E
and pheochromocytoma, 363, 412Q,
419E
plasma levels, 359-360
release, 347
stimuli for, 359
and renin release, 239
source, 335t, 358, 412Q, 419E
synthesis, 359
and thyroid hormones, 401
EPP, see End-plate potential
Equilibrium, dynamic (steady state),
343
Equilibrium potential, 9, lOt, 82Q, 88E
Equilibrium volume of lung, 153, 154,
156, 186Q, 192E, 194E
see also Functional residual capacity
of lung
Equivalent dipole, in
electrocardiography, 107
Ergocalciferol, see Vitamin D,
Ergosterol, 403
Eructation (belching), 307, 327
Erythrocytes as buffers, 263-264, 265,
292Q, 299E
Erythropoietin, 184, 199
Esophageal reflux, 306
Esophagus, 303, 304f, 306-307, 328Q,
331E
innervation, 306, 328Q, 331 E
sphincters, 306, 328Q, 331 E
Essential hypertension, 119, 136
Estradiol, 379, 380t
effect on endometrium, 385
effect on FSH, 384
effect on myometrium, 384
as estrone precursor, 379
fetal, 377
in ovarian (menstrual) cycle, 382f,
383, 384
placental, 388f
in pregnancy, 386, 388f, 389-390,
410Q-411Q,418E
source, 335t
synthesis, 365f, 379, 381
Estriol, 381, 408Q, 409Q, 416E, 417E
placental, 377, 388f, 409Q, 417E
fetal precursors, 388f, 389
in pregnancy, 386, 389-390
synthesis, 379, 388f, 408Q, 416E,
417E
Estrogens
adrenal, 366
catabolism, 381, 408Q, 416E
effect on cervical mucus, 385
effect on endometrium, 385

intestinal, 321

effect on myometrium, 384, 385, 390

pancreatic, 317-318, 330Q, 333E


salivary, 305

in fetus, 377, 408Q, 414E


and gonadotropin secretion, 384

and insulin levels, 393


in menstrual (ovarian) cycle, 382(,
383,384
physiologic effects, 385-386, 390-391
placental, 389-390
in pregnancy, 388(, 389-390, 391,
410Q-411Q,418E
secretion, clinical tests of, 385
synthesis, 365(, 366, 374, 375,
408Q,415E
and TSH, 400
transport, 379
urinary excretion, 381, 408Q, 416E
Estrone, 366-367, 379, 380t
in pregnancy, 388(, 389-390
synthesis, 365(, 379
Etiocholanolone, 366, 375
Euspermia, 377
Excitation-contraction (EO coupling,
29-30
and calcium ion, 401
and cardiac muscle, 34, 102,
104-105
and smooth muscle, 35
Excretion rate, urinary, 220, 221
Exercise, and cardiovascular function,
128,129('137-139, 140Q, 144E,
145E, 148E
Exocrine cells of pancreas, 31 7
Expiration, 150-151, 186Q-188Q,
191Q, 192E, 194E, 197E
Expiratory force, 151, 180
Expiratory muscles, 151, 186Q, 192E,
193E
weakness of, 151, 180, 194E, 197E
Expiratory reserve volume (ERV), 161,
162(, 188Q, 190Q, 196E, 446Q,
468E
Extensor reflex, crossed, 65
Extracellular fluid (ECF) compartment,
202-203, 244Q-246Q, 252E,
253E
composition, 1, 208(, 209t
osmolality, 206(, 245Q, 252E
volume, 206(, 431Q, 454E
effect of aldosterone, 236-237
measurement, 204-205, 246Q,
253E, 443Q, 467E
Extrafusal muscle fiber, 66, 67
Extrapyramidal system, 77, 78
Eye
movements in vestibular reflexes,
74-75
optical properties, 52-54, 85Q, 91 E
refractive media, 52-54
structure, 52-53

F
Facilitated diffusion, 3, 82Q, 88E
in kidney, 211, 244Q, 251E
Fainting (syncope), 99, 134, 135
Farsightedness, 54
Fast fibers
fast-twitch fatigable (FF), 70, 84Q,
90E
fatigue-resistant (FR), 71
of myocardial cells, 101
Fast-wave (rapid-eye-movement,
desynchronized, paradoxical,
dream) sleep, 79
Fat
see also Adipose tissue; Lipid; Obesity
absorption site, 432Q, 436Q, 455E,
461E

in body, calculation of, 443Q, 467E


daily intake, 323
digestion, 303, 323
by saliva, 305, 323
in stomach, 315, 323
malabsorption, 474E
metabolism
effect of hormones, 438Q, 464E
epinephrine, 362
glucagon, 396
glucocorticoids, 370, 408Q, 415E
growth hormone, 355, 356
insulin, 393(, 394, 395, 409Q,
416E
thyroid hormones, 401
in liver, 394
in stool, 324
Fat-free mass (lean body mass; LBM),
202
Fatty acids, 323, 324, 328Q, 329Q,
332E, 363
see a/so Free fatty acids
and glucagon, 396
and glucocorticoids, 370, 408Q,
415E
and insulin, 392, 394, 395
Feature detection, in visual cortex,
60-61
Feedback control of hormones, 344,
345f
in hypothalamic-hypophysialadrenocortical control system, 367
in hypothalamic-hypophysial-Leydig
cell system, 376
in hypothalamic-hypophysial-thyroid
axis, 399, 411 Q, 418E
in ovarian cycle, 383, 384
types of loops, 344, 345f
Female phenotype, 377, 410Q, 41 7E
Fenestrations between cells, 96
Ferric ion, 325
Ferritin, 325
Ferrous ion, 325
Fertilization, 386
and meiosis in oocyte, 386, 407Q,
413E
and ovarian cycle, 383, 41 OQ-411 Q,
418E
site of, 378, 384, 386
Feto-placento-maternal unit, 386, 387(,
407Q,414E
FEV" see Forced expiratory volume in
1 second
FF fibers, see under Fast fibers
Fibrillation
atrial, 114-115
ventricular, 116
Fibrosis, pulmonary, and compliance,
154, 173
Fick equation, 454E
Fick principle, 130, 432Q, 456E
and blood flow, 222-223
Fick's law of diffusion, 2, 82Q, 88E,
165t, 189Q, 195E
Fight-or-flight reaction, 359
Filaments of skeletal muscle, 28, 29(,
30-31
Filtered load, 210n, 213n, 220, 244Q,
245Q, 251 E, 253E
in renal titration curve, 221, 245Q,
253E
as test of renal function, 452Q,
472E-474E
Filtration, 5
see a/so specific type (e.g.,
Glomerular)

in microcirculatory exchange, 96, 97,


143Q, 148E
Filtration coefficient, of glomerular
membrane, 21 7
Filtration fraction, 219, 436Q, 461E
in tests of renal function, 452Q,
472E-474E
Fixed acid, 257, 292Q, 299E
see a/so Noncarbonic acid
effect on pH, 283
net excretion, 276, 289Q, 296E
Flatus (colonic gas), 327
Flavor, 61
Flexor reflex, 64-65
Flow, dynamics of, 124, 125f
see a/so Blood flow
Fluid, see Body fluid; Water; and
specific kinds (e.g., Extracellular
fluid)
Flutter, atrial, 114
Folic acid, 325
Follicle, see Graafian, Ovarian, Thyroid
follicles
Follicle-stimulating hormone (FSH)
effect of estradiol, 384
half-life, 339
in menopause, 381, 407Q, 413E
in ovarian (menstrual) cycle, 382(,
383,384, 410Q-411Q, 418E
in ovarian steroidogenesis, 380
postpartum, 357
serum levels in female, 380t
in spermatogenesis, 375-376
Foodstuffs, as source of acids and
alkalis, 257, 270
Foot processes (junctional feet) of
skeletal muscle, 29, 30
Foramen ovale, 135
Forced expiration, and airway
resistance, 159, 161 (, 187Q,
193E, 194E
see a/so Vital capacity
Forced expiratory volume in 1 second
(FEV,), 163, 180, 187Q, 191Q,
193E, 194E, 197E
Forced vital capacity, 163, 180, 187Q,
194E
in lung disease, 163f
Foreign matter in airways, 164
Fowler's method for measuring dead
space, 164, 174
FR fibers, see under Fast fibers
Frank-Starling effect/relationship
(Starling's law of the heart),
105-106,128, 141Q, 142Q,
145E, 147E
FRC, see Functional residual capacity
of lung; see a/so Equilibrium
volume of lung
Free fatty acids
glucocorticoids and, 369-370
and glucose metabolism, 355-356
insulin and, 394
mobilization, 362-363
and thyroid hormone levels, 401
Free-water clearance, 232
Free-water reabsorption, 233
Frequency-dependent compliance,
173, 440Q, 465E
Frontal lobe, role in motor control, 69
Fructose absorption, 322, 329Q, 332E
FSH, see Follicle-stimulating hormone
FSH releasing hormone (FSH-RH), 376
5ee iJ./50 Gonadotropin-releasing
hormone
Functional hyperemia, 94, 143Q, 148E

Functional residual capacity of lung


(FRC), 153, 162, 163, 180, 188Q,
191Q, 194E, 197E, 446Q, 468E
see a/50 Equilibrium volume of lung
Fundus of stomach, 307t; 330Q, 333E

G
G cells, 311, 333E
G protein, 22, 23t; 24, 25, 83Q, 89E
G6PD, see Glucose-6-phosphate
dehydrogenase
G-17 (little gastrin), 311
G-34 (big gastrin), 311
Galactopoietic hormone, see Prolactin
Galactose, absorption, 322
Gallbladder, 304t; 316, 320t; 321,
330Q, 332E, 333E
contraction (emptying), 321, 330Q,
333E
in enterohepatic circulation, 320f
functions, 321
Gallstones, 321
Gametogenesis
female, 378, 379
male, 373
Gamma-aminobutyric acid (GABA),
and CRH secretion, 367
Gamma efferent fiber, 66
Gamma loop in skeletal muscle
activity, 69
Gamma motoneuron, 37, 68, 69
Ganglion cell receptive fields, 59, 60f
Ganglion cells of retina, 57, 59, 60f
Ganglionic neurotransmission, 22
Gap junctions, 25, 26t; 96
in neurocrine communication,

342-343
Gas exchange, pulmonary, 163-166,
188Q-190Q, 192E, 194E, 196E,
197E
in hypoxia, 182
Gas law, general, 151, 1521
Gases
partial pressure (tension) of, 163
versus content, 167
solubility coefficients, 1671
transport in blood, 166-170
Gastric, see a/50 Stomach
Gastric emptying, 309, 330Q, 332E,
431 Q, 435Q, 454E, 460E
regulation of, 309-310, 328Q, 331 E
Gastric inhibitory peptide (GIP), effect
on insulin secretion, 392
Gastric motility, 308-310, 328Q, 331E,
431Q,454E
effect of calcitonin on, 405
Gastric mucosal barrier, 314-315
Gastric phase of gastric secretion,
310-311
Gastric phase of pancreatic secretion,
318
Gastric secretions, 310-314,
328Q-330Q, 331E-333E
function, 310
and iron absorption, 325
neural regulation, 310, 312, 313t;
314, 328Q, 331E
pH and hydrogen ion concentration,
2601
phases, 310-311, 328Q, 331 E
Gastric secretory cells, 311

effect on slow waves, 317, 329Q,


332E
in gastric motility, 309, 310, 328Q,
331E
and pancreatic secretions, 31 8
secretion, effect of calcitonin, 405
in signal transmission, 342
source, 335
Gastrin inhibitory peptide (GIP), 314,
332E
Gastrin-releasing peptide (GRP,
bombesin), 313t; 314, 330Q,
333E
Gastroenteropancreatic (GEP) system,
348
Gastrointestinal tract
accessory organs, 303, 304t; 316
endocrine cells of, 348
fluid loss from, 433Q, 457E
hormones of, 335
and endorphins, 350
and glucagon, 396
and insulin, 392
and somatostatin, 392, 3931
physiology of, 303-327, 328Q-330Q,
331 E-333E
structure of, 303, 304f
Gating, and membrane potential,
11-12, 13t; 83Q, 88E, 89E
General gas law, 151, 1521
Generator (receptor) potential, 37-38
Genital tract, embryology, 371 t; 377
Genitalia
external
embryology, 372, 377
female, 377
male, 372
internal
see a/50 Accessory sex organs
embryology, 370-372, 377
female, 377, 410Q, 417E
male, 370-371, 410Q, 417E
GEP, see Gastroenteropancreatic
system
GH, see Growth hormone
Gigantism, 356
GIP, see Gastric inhibitory peptide
Globus pallid us, 75
Glomerular filtration, 209, 217-218
factors affecting, 217, 245Q, 253E,
435Q, 458E-459E
and inulin clearance, 213-214, 215
versus systemic filtration, 217-218,
245Q, 253E
Glomerular filtration rate (GFR), 209,
217-220, 245Q, 252E, 253E
and aldosterone, 236
autoregulation, 218-219
and blood volume, 131
determinants of, 218-220, 245Q,
253Ez
measurement of
by creatinine clearance, 213, 21 6,
217
by inulin clearance, 213-214
as test of renal function, 452Q,
472E-474E
Glomerulus, structure, 200, 201 t;
247Q,255E
Glossopharyngeal nerve, in respiratory
control, 177, 178
Glucagon
cells producing, 317, 391

effect on fatty acids, 396


effect on glucose, 391, 396, 396
effect on glycerol, 396
effect on glycogenolysis, 396
effect on insulin secretion, 391, 392
effect on pancreatic cells, 391
effect on protein metabolism, 396,
411Q,418E
effect on slow waves, 317
effect on somatostatin secretion, 391
in ketogenesis, 396
physiologic actions, 396
secretion
and calcium ion, 393
regulation of, 395-396
effect of amino acids, 395
effect of epinephrine, 362, 396,
411Q,418E
effect of fatty acids, 396
effect of gastrointestinal hormones,
396
effect of glucose, 395, 396
effect of insulin, 391
effect of somatostatin, 391, 3931
and potassium ion, 393
source, 3351
Glucocorticoids, 364, 368
anti'inflammatory effects, 368
effect on growth hormone, 355
and insulin, 369, 370
metabolic effects, 369-370, 407Q,
408Q, 413E, 415E, 416E
physiologic effects, 368-369, 407Q,
408Q, 413E, 415E
source, 3351
and stress, 368, 369
synthesis, 365-367
Glucokinase
and glucose metabolism, 393-394
Gluconeogenesis, 363, 407Q, 414E
catecholamines and, 362
enzymes and, 369
glucocorticoids and, 369, 370, 408Q,
415E,416E
glycerol and, 396
insulin and, 394, 395
role of kidney, 199
thyroid hormones and, 401
Glucose
absorption, 322, 328Q, 329Q, 331E,
332E
blood level, and serum osmolality,
281-282
clearance, 212t; 221 t; 243Q, 245Q,
249E, 252E
effect of epinephrine, 436Q, 460E
effect of glucagon, 396
effect of insulin
in adipose tissue, 393t; 394, 409Q,
416E
in liver, 393-394, 395
in muscle, 393t; 394, 39
effect of pancreatic hormones, 391,
412Q,419E
effect of thyroid hormones, 400-401
filtered load, as test of renal function,
221 t; 222, 452Q, 472E-474E
and glucagon secretion, 395, 396
and insulin secretion, 392
and somatostatin secretion, 392
metabolism
and acetyl-CoA, 355-356
and free fatty acids, 355-356

Gastric ulcer, 315

effect on carbohydrate metabolism,

glucocorticoids and, 369-370,

Gastrin, 311, 312, 313, 314, 328Q,


330Q, 331E, 333E

396
effect on fat metabolism, 396

407Q, 413E, 416E


and growth hormone, 355-356

in liver, 393-394
renal clearance, 444Q, 467E-468E
renal transport, 2101, 211, 216(, 222,
223, 243Q, 245Q, 248Q, 249E,
253E, 255E, 444Q, 467E-468E
titration curve, 221 (,222, 245Q,
253E
transport across cell membrane, 3, 4,
82Q
insulin-dependent tissues, 394,
407Q,413E
insulin-independent tissues, 395,
413E
in kidney, 211 (,212, 245Q, 253E
Glucose-6-phosphate dehydrogenase
(G6PD), and methemoglobin, 169
Glycerol
effect of glucagon, 396
in fat synthesis, 394
in gluconeogenesis, 396
and thyroid hormone, 401
Glycine, renal transport, 248Q, 255E
Glycogen
effect of insulin, 395
effect of thyroid hormones, 400-401
metabolism, hormonal regulation, 342
Glycogenolysis
epinephrine and, 361t, 362, 407Q,
411Q, 414E, 418E
hepatic, effect of glucagon, 396
insulin and, 393
Glycosuria, 222, 249E
Gn-RH, see Gonadotropin-releasing
hormone
Goblet cells, intestinal, 321
Goiter, 400
Goitrogens, 400
Goldman-Hodgkin-Katz (GHK)
equation, 16
Golgi tendon organs, 66, 457E
Gonadal hormones, 3351
see also Androgens; Estrogens; and

specific hormones
synthesis, 437Q, 462E
Gonadotropin-releasing hormone
(Gn-RH), 376, 383
in ovarian cycle, 383
Gonadotropins
effect of estrogen, 384
in menopause, 381, 407Q, 413E
in ovarian (menstrual) cycle, 382(,
383,384, 410Q-411Q, 418E
Gonads, embryology
female, 372, 377-378
male, 372
sex differentiation, 371 f
Gonane (sterane), 339
Graafian (secondary, vesicular) follicle,
in ovarian cycle, 381, 383
Granulosa cells, 379
effect of FSH, 383
in ovarian cycle, 381, 383, 384
and progestin secretion, 384
in steroidogenesis, 379
Gravity
effect on venous system, 99
pulmonary effects, 170-171, 172,
189Q,I96E
Greater curvature of stomach, 307f
Group la afferent fiber, 66, 67(, 68,
69, 433Q, 457E
Group Ib afferent fiber, 66, 433Q,
457E

Group II fiber, 66
Growth
effect of androgens, 377, 408Q, 415E

effect of estrogen, 385


effect of thyroid hormones, 400
effect of glucocorticoids, 368-369
retardation, 356
stunted, 356
Growth hormone (GH, somatocrinin),
354-356, 409Q, 411Q, 417E,
418E
chemistry, 354
deficiency of, 356
effect of somatostatin, 3931
effect of glucocorticoids, 369
excessive, 356
and insulin, 355, 356, 392
metabolic effects, 355-356
physiologic effects, 355-356, 411Q,
418E
precursor form, 337
secretion, 354, 409Q, 417E
diurnal pattern, 354
factors affecting, 438Q, 464E
inhibition, 354-355
source, 3351
synthesis and storage, 354
Guanosine triphosphate (GTP) binding
protein, see G protein
Gustation, 61-62, 84Q, 90E

H
Hair cells
of organ of Corti, 48, 84Q, 90E
of vestibular system, 72-74
Haldane effect, 1 70
Hartnup disease, 323
Haustral shuttling, 326
Haustrations, 326, 329Q
HbO" HHbO" see Hemoglobin,
oxygenated
HCG, see Chorionic gonadotropin
Hearing, 44-51, 84Q, 85Q, 90E, 91 E
impaired, 50, 51
perception mechanisms, 48-51
Heart, 99-107, 140Q-143Q,
144E-148E
see also Cardiac; Myocardium
electrical activity, 100-102, 141 Q,
145E, 439Q, 465E
electrical conduction in, 23, 25,
102-104
disturbances in, 116-11 7
as pump, 118-121
rate
and blood pressure, 126, 141 Q,
146E, 432Q, 455E
disturbances in, see Cardiac
arrhythmias
and exercise, 137, 139f
factors affecting, 102-103, 441 Q,
465E-466E
in hypoxia, 182
and pulse pressure, 432Q, 455E
sounds, 122-123, 124(, 141 Q, 143Q,
146E, 147E
murmurs, 123-124, 445Q, 468E
valves, 118-119, 122, 141Q, 143Q,
146E, 147E-148E
disorders, 123-124, 445Q, 468E
Heart disease
arrhythmias, see Cardiac arrhythmias
conduction disturbances, 116-11 7
congestive heart failure, 106, 119,

14ZQ,146E
effect on cardiovascular function,
127(,129, 142Q, 146E

and hypoxia, 183


and venous return, 127f
response to exercise, 139
valvular disorders, 123-124, 445Q,
468E
Heartburn, 306
Hematocrit
and blood viscosity, 125
and blood volume, 203
Heme, 325
Hemiballismus, 75-76
Hemodynamics, 124-126
Hemoglobin, 167, 190Q, 197E
and bilirubin formation, 321
as buffer, 263-264, 265-267
and carbon monoxide, 168-1 69
deficiency, and hypoxia, 183, 190Q,
197E
oxygenated (HbO" HHbO,), 265,
269
protonated (HHb, HHbO,), 265, 2691
saturation, 167-1 68, 188Q, 190Q,
195E, 197E, 443Q, 467E
saturation curve, 167(, 168f
see also Oxygen dissociation curve;
Oxyhemoglobin dissociation curve
Hemoglobin 5, 169
Hemorrhage, 135
endocrine effects, 433Q, 456E
renal effects, 438Q, 463E
Hemostasis, calcium ion in, 401
Henderson equation, 262-263, 290Q,
296E, 297E, 458E, 459E-460E
Henderson-Hasselbalch equation,
260-262, 297E, 457E, 458E
Henry's law, 166-167, 188Q, 194E
Hepatic duc!, 320f
Hepatocytes, 318, 320f
Hering-Breuer reflex, 179
Hexaxial electrocardiography system,
109(, 110, 141Q, 146E
Hexokinase, and glucose metabolism,
393-394
HGH, see Growth hormone
HHb, see Hemoglobin, protonated
Hiccups, 176
Hirschsprung's disease, 326
His bundle, see Bundle of His
Histamine
and APUD cells, 348
in gastric secretions, 312, 313(,
328Q, 330Q, 331 E, 333E
and microcirculation, 96, 97, 143Q,
148E
Histotoxic hypoxia, 1821, 183(, 184,
197E
Homeostasis
cellular, 1-7
definition, 1
role of endocrine system, 343
Hook's law, 126, 455E
Hormones, 335-340
see also individual classes (e.g.,
Polypeptide hormones) and

specific hormones
activation, 337
autocrine, 335, 336f
and biochemical reactions, 336
bodily processes affected, 336
chemical classes, 336, 3381
chemistry, 337-340
circadian (diurnal) cycles, 446Q,
468E

cyclic AMP.mediated, 341-342


definition, 335
degradation, 337

feedback control, 344, 345f


in hypothalamic-hypophysialadrenocortical control system, 367
in hypothalamic-hypophysial-Leydig
cell system, 376
in hypothalamic-hypophysial-thyroid
axis, 399, 411Q, 418E
in ovarian cycle, 383, 384
functions, 335-336
and gastric motility, 310
half-life, 337
inactivation, 337
and information transfer, 336(,
341-343
and membrane permeability, 341
in neuroendocrine transducer
systems, 3461
paracrine, 335, 336f
plasma levels, 336
precursor forms, 337
protein-bound versus unbound, 337
receptors, 337, 340-341, 410Q, 417E
changes in number, 341
interactions with, 340-341
as regulators, 336
and renal electrolyte transport, 437Q,
462E
response time (latent period),
336-337
tissue sources, 335
HPO/-, H,P0 4 - , see Phosphates
Human chorionic gonadotropin (HCG),
see Chorionic gonadotropin
Human growth hormone, see Growth
hormone
Human placental lactogen (HPl), 355
Huntington's chorea, 76
H-Y (histocompatibility-Y) antigen, 372,
373,377
Hydration-dehydration:dissociationassociation reaction, 267
Hydrochloric acid
cells secreting, 311
gastric functions, 311
gastric secretion, 311-314, 328Q,
329Q, 330Q, 331 E, 332E, 333E
inhibitors, 312, 313f
mechanisms, 311, 312f
regulation of, 312-314, 329Q,
330Q, 332E, 333E
stimulators, 312, 313f
Hydrocortisone, see Cortisol
Hydrogen ion
see a/so pH
and anion gap, 281
chemistry, 258
concentration, 258-263
in acidosis, 276, 2771, 300E
in alkalosis, 276, 2771
in body fluids, 259-260, 290Q,
296E
calculation by Henderson equation,
263, 290Q, 296E
calculation from pH, 262, 290Q,
296E
conversion to pH, 262
compatible with life, 259
maintenance of, 258
relation to pH, 258-259
relation to carbon dioxide tension,
260
effect on bone, 404
generation and elimination, 267-269

renal transport, 21 at, 211, 22 5,


243Q, 250E, 433Q, 451Q, 456E,
471E

buffering, 226, 243Q, 250E


and renal sodium transport, 211 (,
224, 225, 271-272, 272-273
effect of aldosterone, 275
effect of parathyroid hormone, 405
effect of spironolactone, 454E-455E
source of, 257
urine/plasma gradient, 271
Hydronium ion (H,O+), 258
Hydrostatic pressure, 170- 171
in capillaries, 96, 97, 143Q, 148E
in renal transport, 210, 217, 218,
220, 245Q, 253E, 435Q,
458E-459E
.a-Hydroxybutyric acid, 268, 269, 270,
301 E, 363
source of, 257
25-Hydroxycholecalciferol, see
25-Hydroxyvitamin D,
17 -Hydroxycorticosteroids, circadian
pattern, 343, 367
18-Hydroxycorticosterone, 3641
16a-Hydroxydehydroepiandrosterone, placental, 388f
16a-Hydroxydehydroepiandrosterone
sulfate
fetal synthesis, 388(, 390
placental, 388f
17a-Hydroxypregnenolone, in ovarian
steroidogenesis, 380
17 -Hydroxyprogesterone, 3801
17a-Hydroxyprogesterone
fetal, 388(, 408Q, 414E
level in pregnancy, 389
in ovarian (menstrual) cycle, 382(,
384
in ovarian steroidogenesis, 381

Hydroxyproline, urinary, as index of


bone resorption, 405
25-Hydroxyvitamin D, (calcidiol,
calcifediol,
25-hydroxycholecalciferol), 325,
403
half-life, 340
source, 335
Hyperaldosteronemia, 224, 225, 237,
439Q, 464E-465E
Hyperaldosteronism, 241, 242
Hypercapnia (hypercarbia), 165, 277,
294Q, 301 E, 456E
and alveolar dead space, 176
and cerebral blood flow, 134
circulatory effects, 171
and hypoventilation, 182, 186Q,
188Q, 192E, 195E, 197E
and hypoxia, 182, 186Q, 192E, 197E
Hyperchloremic acidosis, 455E
and anion gap, 281, 296E
Hyperemia, 94, 95, 143Q, 148E
Hyperglycemia
cortisol and, 369, 370, 407Q, 413E,
416E
epinephrine and, 362, 411 Q, 418E
Hyperkalemia, 237, 238
Hyperkalemic acidosis, 224, 225, 237
Hyperkalemic periodic paralysis, 434Q,
457E
Hyperopia, 54, 453Q, 474E
Hyperosmotic, definition, 205, 244Q,
252E
Hyperosmotic dehydration, 205, 206(,
2061, 207
Hyperosmotic overhydration, 206(,

2061. 207
Hyperpigmentation from ACTH, 350
Hyperpnea, 182, 300E

Hyperpolarization
of action potential (undershoot
phase), 8(, 11, 12, 83Q, 89E
of auditory hair cells, 49-50
of sinoatrial node, 102
of vestibular hair cells, 73
Hyperprolactinemia, 357
Hypersomatotropism, 355, 356
Hypertension, 136-137, 143Q, 148E
and cardiac work, 119
causes, 136-137
essential, 119, 136
pulmonary, 119, 171, 184, 439Q,
465E
symptoms and consequences, 137
treatment, 137
types, 119
Hyperthyroidism, and catecholamine
effects, 341
Hyperventilation, 300E, 437Q, 453Q,
456E-457E, 462E, 474E
causes, 436Q, 461 E
effect on blood gases, 452Q, 472E
effect on body fluids, 453Q, 474E
in hypoxia, 184
Hypoaldosteronemia, 224, 237
Hypocapnia (hypocarbia), 277, 290Q,
291 Q, 294Q, 298E, 299E, 300E,
456E-457E
and cerebral blood flow, 134
Hypoglycemia, and epinephrine, 359,
360
Hypogonadism, primary, in
menopause, 381
Hypokalemia, 237
from gastrointestinal fluid loss, 433Q,
457E
and glucose intolerance, 393
Hypokalemic alkalosis, 224, 225, 237
Hypokinesia, in Parkinson's disease, 76
Hypokinetic (ischemic) hypoxia, 1821,
183
Hyponatremia, 237, 238
Hypophysectomy, endocrine effects,
438Q,463E
Hypophysial-adrenocortical control
system, 367
response to stress, 368
Hypophysial portal system, 343-344,
353, 456E
and hypothalamic-pituitary regulation,
343, 344(, 348, 353
Hypophysiotropic hormones, 343,
344, 345, 3461, 348, 412Q, 419E
and growth hormone release, 354
Hypophysis, see Pituitary gland;
Adenohypophysis;
Neurohypophysis
Hyposmotic, definition, 205, 244Q,
247Q, 252E, 254E
Hyposmotic dehydration, 206f, 2061,
207
Hyposmotic overhydration, 206(, 2061,
207
Hypotension, and renin release, 239
Hypothalamic hormones, 3351, 343,
348
Hypothalamic-hypophysial axes,
343-344, 348, 41 2Q, 419E
Hypothalamic-hypophysialadrenocortical axis, 237-238,
367-368
Hypothalamic-hypophysial-Leydig cell

axis, 376
Hypothalamic-hypophysialseminiferous tubular axis, 375-376

Hypothalamic-hypophysial-thyroid
axis, 399
and ovarian cycle, 383, 384, 385
Hypothalamic-hypophysial portal
system, 343-344
Hypothalamic-pituitary-target gland
model of feedback control, 344,

345f
Hypothalamus
anatomic relationships, 344f
and cardiovascular function, 132
and endorphins, 350
median eminence, 343-344
and neurohormones, 345
Hypothyroidism
mental retardation from, 400
skin discoloration in, 401
Hypoventilation, 301 E, 449Q, 470E
effects on blood gases, 452Q, 472E
and hypercapnia, 182, 186Q, 188Q,
192E, 195E, 197E
in hypoxia, 182, 186Q, 190Q, 192E,
196E, 197E
Hypovolemia
and antidiuretic hormone, 234-235
and renin release, 239
Hypovolemic shock, 135-136
Hypoxemia, 166, 190Q, 197E, 290Q,
297E
and ventilation:perfusion ratio, 175,
182-183, 190Q, 196E
Hypoxia, 181-185, 188Q, 190Q,
195E, 196E, 197E, 448Q, 469E
and alveolar dead space, 176
anemic, 182t, 183f, 184, 190Q,
196E, 197E
arterial, 182-183, 190Q, 197E
causes, 182-184, 190Q, 196E, 197E
circulatory effects, 171
histotoxic, 182t, 183f, 184, 197E
and hyperlacticemia, 268
hypokinetic (ischemic), 182t, 183
physiologic responses, 184-185
symptoms and signs, 181-182
types, 182t, 183f, 197E
Hypoxic pulmonary vasoconstriction,
171,184,185

ICF, see Intracellular fluid


Ileum, 304f, 315, 320f, 330Q, 333E
in enterohepatic circulation, 319
substances absorbed in, 432Q, 455E
Immune response, and glucocorticoids,
368
Immunologic pregnancy test, 387
Immunologically privileged site, of
germ cells, 373
Impedance matching, auditory, 46
Implantation, and ovarian cycle, 383,
418E
Incisura (dicrotic notch), 122, 141 Q,
146E
Indicator dilution method, 203-204
in measuring cardiac output, 130
Indirect energy utilizing transport, 3-5
Inflammation, effect of glucocorticoids,
368
Information transfer, by hormones,

336f, 341-343
Ingestion of food, 303
Inhibin, 376, 383
Inhibitory reflex..pathway, 65
Inotropism

negative, 106-107, 119


positive, 103, 106, 119, 147E
Inspiration, 149-150, 186Q, 191Q,
192E, 197E
Inspiratory capacity of lung (lC), 163,
188Q,194E
Inspiratory force, 150
Inspiratory muscles, 149, 150f, 186Q,
192E
Inspiratory reserve volume (lRV), 161,
162f, 188Q, 190Q, 196E
Insulin
and calcium ion, 392, 393
chemistry, 392
deficiency
see also Diabetes mellitus
and amino acid metabolism, 395
and fat metabolism, 394
half-life, 339
physiologic actions, 393-395, 407Q,
409Q, 413E, 416E, 437Q,
462E-463E
in amino acid metabolism, 395
in carbohydrate metabolism,
393-394, 409Q, 416E
on cell membrane polarization, 395
effect of glucocorticoids, 369, 370
effect of growth hormone, 355, 356
effect of pregnancy, 355
effect of thyroid hormones, 400
in fat metabolism, 393f, 394, 395,
409Q,416E
on glucagon secretion, 391
in gluconeogenesis, 394, 395
on glucose, 391, 395, 412Q, 419E
on glycogen, 393, 395
on pancreatic alpha cells, 391
in protein metabolism, 395
on slow waves, 317, 329Q, 332E
and potassium ion, 393, 395
precursor of, 337, 391-392
as primary anabolic hormone, 395
receptors for, 393
secretion
and beta-adrenergic receptors, 391,
409Q, 412Q, 416E, 419E
effect of acetylcholine, 391
effect of amino acids, 392
effect of carbohydrates, 391
effect of catecholamines, 361, 362,
391,396, 411Q, 418E, 432Q,
455E
effect of corticosteroids, 393
effect of cyclic AMP, 393
effect of estrogens, 393
effect of gastrointestinal hormones,
392
effect of glucagon, 391, 392
effect of glucose, 391
effect of growth hormone, 392
effect of pancreatic polypeptide, 391
effect of parathyroid hormone, 393
effect of progestogens, 393
effect of somatostatin, 391, 392,

393t
regulation of, 391, 392-393
autonomic, 346-347
and sodium ion, 395
source, 335t
cells producing, 391
cells secreting, 31 7
Insulin-like growth factors (lGF), see
Somatomedin
Intention tremor, 77
Interatrial tract (Bachman's bundle),
102(,103, 141Q, 145E

Intercellular (paracellular) pathway of


renal transport, 21 2
Intercostal nerve, in respiratory control,
177
Internodal tracts of heart, 103
Interpleural pressure, see Pleural space,
pressure in
Interpleural space, see Pleural space
Interpolated beat (ventricular), 115
Interstitial cell-stimulating hormone
(ICSH; LH), see Luteinizing
hormone
Interstitial cells, see Leydig cells
Interstitial fluid, 203
buffer systems of, 265, 292Q, 299E
composition, 209t, 244Q, 251E
volume measurement, 204, 443Q,
467E
Intestinal phase of gastric secretion,
311
Intestinal phase of pancreatic secretion,
318
Intestine
see also Small intestine; Duodenum;
Jejunum; Ileum; Colon
absorption from, 451Q, 471E
effect of calcitonin, 405
effect of calcitriol, 406
gas in, 322, 327
secretions, 321-322
segmentation, 316, 317, 326, 329Q,
332E, 435Q, 460E
Intoxicants (toxins), and osmolar gap,
281-282
Intracellular fluid (lCF) compartment,
203, 244Q, 252E, 431Q, 454E
buffer systems of, 265, 289Q, 295E,
299E
composition, 1, 208(, 209t, 244Q,
251E
osmolality, 206f
pH and hydrogen ion concentration,

259t, 260t
volume, 206f
measurement, 205
Intrafusal muscle fiber, 65(, 66, 67, 69
Intraocular pressure, 53
Intrinsic factor, 311, 314, 328Q,
330Q, 331 E, 333E
and vitamin B" absorption, 325,
437Q,462E
Intrinsic nervous system of stomach,
see Enteric nervous system
Inulin, 213, 246Q, 253E
renal transport, 216(, 245Q, 253E
Inulin clearance, 212f, 213-214, 221(,
245Q, 252E, 253E, 255E
as test of renal function, 452Q,
472E-474E
Inulin space, 253E
Iodide
dietary requirements, 397
distribution in thyroid gland, 397
and thyroid hormones, 398-399, 400,
409Q, 411Q, 416E, 418E
effect on TS H, 400
radioactive, thyroid uptake, 398
Iodination, of thyroglobulin, 396, 399
Iodine, dietary intake, 397
lodothyronines, 339, 396, 397, 407Q,
414E
lodotyrosines, 397
Ionization (dissociation) constant (K)

259n, 261
Ions

see a/50 Electrolytes; Solutes; and


specific ions (e.g., Sodium ion)
and blood-brain barrier, 178, 196E
in body fluids, 1I, 208, 2091, 244Q,
245Q, 251E, 252E
in pancreatic secretions, 31 7
in serum, 280f
and electrical activity of heart, 100,
101-102,103
and membrane potentials, 9-12, 15,
87Q, 92E
renal transport, 2101, 248Q, 255E,
451Q,471E
exchange mechanisms, 271-272
in synaptic transmission, 19-21, 83Q,
89E
transport across cell membrane, 2,
3-5,7
Iron
absorption, 325, 329Q, 330Q, 333E,
451Q,471E
versus intake, 325
ferric versus ferrous, 325
transport to plasma, 325
Irradiation, in reflex pathway, 64
Ischemic (hypokinetic) hypoxia, 1821,
183
Islets of Langerhans, 391-396
autonomic innervation, 346-347
cell types, 391
neurotransmitters of, 391
Isohydric buffering, 266
Isometric contraction, 31-33
Isometric portion of isotonic
contraction, 33-34
Isoproterenol, and renin release, 239
Isosmotic, definition, 205, 244Q, 252E
Isosmotic dehydration, 205, 206(, 2061
Isosmotic overhydration, 206(, 2061,
207
Isotonic contraction, 33-34, 86Q, 91 E
Isovolumic ventricular contraction,
120(, 446Q, 468E
Isovolumic ventricular relaxation, 120f

J receptors, in respiration, 179, 190Q,


196E
Jaundice, 321, 453Q, 474E
Jejunum, 304(, 315, 320f
Junctional arrhythmia (rhythm), 115
Junctional feet (foot processes), of
skeletal muscle, 29, 30
Junctional folds, 18, 19f
Juxtaglomerular apparatus, 238-239
Juxtaglomerular cells, 238
and renin release, 347
Juxtamedullary nephron, 201

K, K', see Dissociation constant


Kallidin-9 (bradykinin), 240, 246Q,
254E
Ketoacidosis, diabetic, 2741, 276,
292Q, 294Q, 300E, 301 E, 436Q,
453Q, 460E, 474E
Ketoacids, 363
Ketogenic steroids, 367
Ketone bodies, 268, 269, 270
effect on insulin secretion, 392
production (ketogenesis), 257
glucagon and, 396

insulin and, 394, 395


Ketonuria, 269
Ketosis, from glucocorticoids, 370
17-Ketosteroids, 366, 367, 375
in ovarian steroidogenesis, 380
Kidney
see a/50 Renal
blood vessels, 201-202
functions, 199, 227
in acid-base balance, 260, 270-276
structure, 200-202
Kinetic energy, 124, 125f
Kinin system, 240f
Kininase II (angiotensin-converting
enzyne), 131, 240, 252E, 470E
Kinocilium, of vestibular system, 73-74
Knee jerk (patellar) reflex, 65, 433Q,
457E
Kussmaul's respiration, 180f
Kwashiorkor, 356

L
Labyrinth, of ear, 45(, 47, 72
Lactase, 322
Lactate, source of, 257
Lactation
and amenorrhea, 357
and calcium ion, 401
effect of estrogen, 385, 390
effect of progesterone, 385, 386,
390, 409Q, 416E
effect of thyroid hormones, 400
and oXylocin release, 353, 354,
409Q,416E
Lacteals, 315
Lactic acid, 268, 269, 292Q, 299E
Lactogenic hormone, see Prolactin
Lactose, 322
intolerance, 322
Laplace's law, 95, 147E
and respiratory mechanics, 156, 157f
Large intestine, see Colon
Latch-bridges, in smooth muscle, 35
Lateral inhibition, and stimulus
localization, 39, 40f
LBM, see Lean body mass
Leads, electrocardiographic, 107-110
Lean body mass (fat-free mass; LBM),
202
Lecithins
in bile, 319
and gallstones, 321
in lipid emulsification, 323
Length-tension relationship of skeletal
muscle, 32
Lengthening reaction, 65, 66-67
Lens, and principles of image
formation, 51, 52(, 85Q, 91E
Lens of eye, 53, 85Q, 91 E
Leu-enkephalin, 349
precursors of, 350, 351
Leydig (interstitial) cells, 371, 372, 374,
408Q,414E
and development of female
phenotype, 377
in spermatogenesis, 376
LH, see Luteinizing hormone
LH releasing hormone (LH-RH), see
Gonadotropin releasing hormone
Licorice intoxication, 241
Limbus, of inner ear, 47, 48(, 49f

Lipases
in fat digestion, 304, 305, 315, 323
pancreatic, 317, 323, 330Q, 333E

Lipid-soluble materials, diffusion of, 2,


82Q,88E
Lipids
see a/50 Fat
absorption, 323-324, 328Q, 331 E
in cell membrane, 1, 2f
in circulation, 324, 328Q, 331 E
and thyroid hormone levels, 401
digestion, 323
emulsification, 323
malabsorption, 324, 330Q, 333E
Lipolysis, hormones inducing, 438Q,
464E
epinephrine, 362
glucocorticoids, 370
/3-Lipoprotein, in lipid absorption, 324
/3-Lipotropic hormone (/3-LPH), 349
")I-Lipotropic hormone (")I-LPH), 349
Lipotropins (lipotropic hormones) 349
in prohormone complex, 337
Lipotropin lipase, 394
Lithocholic acid, 319
Little gastrin (G-17), 311
Liver, 304(, 316, 319, 320f
fat metabolism in, 394
glucose metabolism in, 393-394
insulin effects in, 393, 394, 395
Load-velocity relationship, in isotonic
contraction, 34
Loading zone of oxygen-dissociation
curve, 167-168
Long-loop feedback, 344, 345f
Loop of Henle, 200, 201, 243Q,
247Q, 249E, 254E, 255E
in urine concentration and dilution,
227, 228-229, 230(, 243Q, 246Q,
247Q, 249E, 254E, 255E
and vasa recta, 202
Lung capacities, 161, 162-163, 186Q,
188Q, 191Q, 192E, 194E, 197E
Lungs
see a/50 Pulmonary; Respiratory
in acid-base balance, 260
compliance (distensibility), 154, 173,
186Q, 187Q, 192E-194E
diffusing capacity, 432Q, 455E
fluid in, and respiratory mechanics,
156,158f
gas content, 153, 188Q, 194E
gas flow in, 150-153, 186Q-188Q,
192E-194E
and airway resistance, 159-161,
187Q,193E
effort-independent, 159-160, 187Q,
193E, 194E
time constant (Rc), 172-173
volume, 154, 161-162, 186Q-189Q,
191 Q, 192E, 194E, 196E, 197E
and airway resistance, 187Q, 193E
and expirato"ry gas flow, 160
minimal, 154
and pulmonary vascular resistance,
171-172
regional, 172, 189Q, 196E
tidal,172
zones of, 170-171
Luteal phase of menstrual cycle, see
Ovarian cycle, postovulatory
phase
Lutein-granulosa cells, 384, 408Q,
415E
Lutein-theca cells, 384
Luteinizing hormone (LH)
and corpus luteum, 384, 411 Q, 418E
half-life, 339

in ovarian (menstrual) cycle, 381,


382(, 383, 384, 41OQ-411 Q,
418E
midcycle surge, 382(, 383, 384,
410Q-411Q,418E
in ovarian steroidogenesis, 380
and ovulation, 381, 383, 384
and prolactin, 357
and progesterone, 384
serum levels in female, 380t
in spermatogenesis, 375-376
in testosterone synthesis, 374, 376
Luteotropic hormone
of menstrual cycle, 384, 411 Q, 418E
of pregnancy, 390, 411 Q, 418E
Lymph, buffer systems of, 265, 299E
Lymphatic system, 98, 140Q, 144E
Lysophosphatides, 324

M
Macula densa cells, 239, 247Q, 254E
Magnesium ion, 208, 209t, 210t
Magnocellular neurosecretory neuron
system, 345-346, 352
anatomic relationships, 345, 352
in pituitary control, 346t, 347-348
Malabsorption, 432Q, 455E
and enterohepatic circulation,
319-320
Male phenotype, 372
Malignant hypertension, 136
Malnutrition, 453Q, 474E
Maltase, 322
Mammary gland, development, 357
Mammotropic hormone, see Prolactin
Mannitol, 247Q, 254E
MAO, see Monoamine oxidase
Mass movement, in colon, 326
Mastication (chewing), 303-304, 328Q,
331E
Maximal expiratory force (pressure),
151,180
Maximal voluntary ventilation test, 179
Maximum tubular reabsorptive capacity
(Tm, Tr), 220, 243Q, 250E
Maximum tubular secretory capacity
(Tm, Ts), 220, 243Q, 244Q,
250E-252E
Mean circulatory pressure (MCP), 127(,
128
Mean electrical axis (MEA), in
electrocardiography, 111-112
Mechanical sensation, 40-42
Median eminence of hypothalamus,
344, 352
blood supply, 343-344, 353
Medulla
of brain
in respiratory control, 176-178,
190Q,196E
in cardiovascular function, 132
of kidney
in urine concentration and dilution,
227, 228(, 229-230
Meiosis, in oocyte, 378, 383, 407Q,
413E
and fertilization, 386, 407Q, 413E
Meiosis-inhibiting factor, 378
Meiosis-stimulating factor, 378
Meissner's corpuscle, 41 (, 42
Meissner's (submucosal) plexus, 307
Melanocyte-stimulating hormones
(MSHs),349

Melanotropin, in prohormone complex,


337
Melatonin, source, 335
Membrane, see Cell membrane
Membrane potential, see Action
potential; Cell membrane,
electrical potentials
Menarche, 381
Menopause, 381, 407Q, 413E
Menses (menstruation), 381, 383, 385,
454E
Menstrual cycle, see Ovarian
(menstrual) cycle
Mental retardation, from
hypothyroidism, 400
Merkel's disk, 41 (, 42
Mesangial cells, 239
Mesoderm, and steroid hormones, 364
Mesonephric ducts, see Wolffian ducts
Messengers, first and second, 340(,
341-342
Metabolic acid-base imbalance, 437Q,
450Q, 453Q, 462E, 471 E, 474E
metabolic acidosis, see under
Acidosis
metabolic alkalosis, see under
Alkalosis
Metabolic rate, basal (BMR), 400
Metanephrine, 360, 409Q, 417E
Metarterioles, 95
Met-enkephalin, 349, 358
adrenal, 351
in central nervous system, 351
precursor of, 350
Methemoglobin, 169
and cyanosis, 182
and hypoxia, 183
and respiratory control, 178
3-MethoxY-4-hydroxyphenylglycol
(MOPG),360
Micelles
in bile, 319
in lipid absorption, 323-324, 328Q,
332E
structure, 323
Michaelis-Menten kinetics, 3
Microcirculation, 95-98, 140Q, 143Q,
144E, 148E
Microvilli, intestinal, 315
Migrating motor complex (MMC), 310,
316, 329Q, 332E, 333E
Milieu interieur, 203
Milk, secretion, 353, 357
see also Lactation
Milk ejection reflex, 353
Milk let-down reflex, 353
Mineral acids, body's response to, 269
Mineralocorticoids, 364
biosynthetic pathway, 365-367
source, 335t
Minute ventilation, 164, 188Q, 194E,
432Q,455E
effect of exercise, 138
Mitral (bicuspid) valve, 118, 442Q,
446Q, 466E, 468E
insufficiency, 124(
stenosis, 124f
MMC, see Migrating motor complex
Mobitz A V nodal blocks, 11 6
Monge's disease, 185
Monoamine oxidase (MAO), and
catecholamines, 360, 409Q, 417E
Monoaminergic neurons, 344(, 346
Monoamines, adrenomedullary,
358-359
Monoglycerides, absorption, 323-324

Monoiodotyrosine, 399
Monosaccharides, absorption, 322
Morphine, and enkephalins, 351
Morphinomimetic peptides, 348,
409Q, 417E
Motilin, 310, 328Q-330Q, 331E, 333E
effect on slow waves, 317, 329Q,
332E
Motoneurons, 37, 84Q, 90E
see also Alpha motoneuron
in spinal cord, 70
Motor areas of cerebral cortex, 77
Motor command, 69
Motor control system, 69-78, 84Q,
87Q, 90E, 92E
components, 69, 70(, 87Q, 92E,
451Q,472E
role of stretch reflex, 67-69
Motor deficits, 75-76, 76-77, 77-78,
451Q,472E
Motor end-plate, 18, 19(, 83Q, 89E
Motor unit, 70, 84Q, 90E
size principle, 71
Mountain climbing, effects on blood
gases, 452Q, 472E
see also Altitude
Mountain sickness, 185
Mouth, see Oral cavity
Movement
control of, see Motor control system
decomposition of, 77
disorders of, 75-76, 76-77, 77-78,
451Q,472E
Mucin, 305
Mucous cells of stomach, 311, 314,
315
Muller cells of retina, 57
MOiler maneuver, 150
Mullerian ducts, 370, 371, 377, 410Q,
417E
MUllerian duct inhibiting factor
and development of female
phenotype, 377
and development of male phenotype,
372
Muscarinic receptors, 23, 83Q, 89E
Muscles
see also Cardiac muscle; Skeletal
muscle; Smooth muscle
buffers in, 264, 265
cardiac, 99, 100(, 141 Q, 145E
contraction, 28-35, 84Q, 89E, 450Q,
471E
of cardiac muscle, 34-35, 104-105,
119
and cardiac cycle, 121-122, 143Q,
147E
of skeletal muscle, 30-34, 84Q, 89E
of smooth muscle, 35, 84Q, 89E
effect of androgens, 377
effect of exercise, 137
effect of glucocorticoids, 369, 408Q,
415E
effect of insulin, 394, 395, 409Q,
416E
of expiration, 151, 186Q, 192E, 193E
weakness of, 151, 180, 194E, 197E
of inspiration, 149, 150(' 186Q, 192E
fibers of, 65 f, 66
in motor unit, 70
reflexes, 65-67, 84Q, 90E
types, 28
Muscle spindle, 65-66
Muscle twitch, 31-32, 84Q, 90E
Muscular arteries, 93-94

Myelin, and propagation of action


potential, 14, 18, 36t; 37
Myenteric (Auerbach's) plexus, 307
Myocardial infarction, 117-118, 142Q,
147E, 183, 439Q, 465E
Myocardial ischemia, 117, 142Q, 147E
Myocardium, 34-35, 99, lOOt; 141Q,
145E
see also Heart; Cardiac
action potential in, 25, 35, 100-102
blood supply, 133, 141Q, 146E
cells, structure, lOOt; 104
oxygen consumption, 121, 133,
142Q,146E
oxygen supply, 121
Myofibrils
of myocardial cells, 104
of skeletal muscle, 28, 29f
Myometrium, 379
effect of estrogens, 384, 385, 390
effect of oxytocin, 354, 409Q, 416E
effect of progesterone, 384, 386,
390, 409Q, 416E
Myopia, 54, 431Q, 453Q, 454E, 474E

N
Na+ -K+ exchange process, see under
Sodium ion
Na+ -K+ pump, see under Sodium ion
Narcolepsy, 80
Nearsightedness, 54
a-Neo-endorphin, 349
precursor of, 351
Nephron, 209-212, 247Q, 254E,
451Q,471E
cortical, 200
juxtamedullary, 201
structure, 200, 247Q, 254E
Nernst potential, 9, lOt
Nerve fibers
group la afferent, 66, 67t; 68, 69,
433Q,457E
group Ib afferent, 66, 433Q, 457E
group II, 66
types, 36-37
relation to function, 37, 40, 433Q,
457E
Nerve terminals, presynaptic, 434Q,
458E
Nervous system, physiology of, 1-81,
82Q-87Q, 88E-92E
Neural control of endocrine tissue,
346-348
Neural crest, tissues arising from,
439Q,464E
Neurocrine communication, 342-343
Neuroendocrine transducers, 346, 347f
adrenal medulla as, 357
Neurohemal organ, 345
Neurohormones, 343, 345, 346t
Neurohypophysis (posterior pituitary)
blood supply, 343-344, 353
embryology, 352
hormones of, 353-354
morphology, 344t; 352
regulation, by neurosecretory
neurons, 346t, 347-348
Neurokinins, and airway dimensions,

159
Neurologic tissue, effect of thyroid
hormones on, 400

Neuromodulator, definition, 351


Neuromuscular junction (synapse), 18,
19f

Neuromuscular transmission, 18-22


Neurons
classification, 36-37
neurosecretory, 234
Neuropeptides
and airway dimensions, 159
of gastroenteropancreatic system, 348
Neurophysins, 234
Neurophysiology, 1-81, 82Q-87Q,
88E-92E
Neurosecretory mechanism of
hormone action, 343, 412Q, 419E
Neurosecretory neurons, 234,
344-346, 352, 407Q, 409Q,
413E, 416E, 417E
classification, 345, 346t
neuroendocrine functions, 345
in pituitary control, 346t, 347-348
Neurotransmitters, 18, 19t; 83Q, 84Q,
89E, 90E, 434Q, 458E
of autonomic nervous system, 22,
23t; 25
and calcium ion, 401
of central nervous system, 26
of endocrine pancreas, 391
of enteric nervous system, 22, 25
in neurocrine communication,
342-343
and neuromodulators, 351
release of, 19-20
Neurovascular hypothesis, 353
NH 3 ; NH4 +, see Ammonia;
Ammonium ion
Nicotinic acid, 325
Nicotinic receptors, 22, 83Q, 89E
Nidation, 386
Nightmares, 80
Nitrogen
balance
effect of growth hormone, 355,
411Q,418E
effect of glucocorticoids, 370,
411Q,418E
negative, 411 Q, 418E
positive, 355, 418E
partial pressure (tension; PN,), 163,
188Q,194E
solubility coefficients, 167t, 194E
Nociceptors, 43
Nonbicarbonate buffer systems, 263t,
264, 265, 295E, 299E
Noncarbonic acids, 257, 268-269,
292Q, 299E
buffering of, 269, 292Q, 299E
formation, 257-258
metabolically produced, 270
renal excretion, 271
source, 257, 268
Noncarbonic acidosis, 269
Nonvolatile acid, 257, 292Q, 299E
see also Noncarbonic acid
Noradrenergic neurons, effect on
pacemaker, 99
Noradrenergic neurotransmitters, and
neurosecretory system, 346
Norepinephrine (noradrenalin), 407Q,
413E
see also Catecholamines; Epinephrine;
Sympathomimetic drugs
as autonomic neurotransmitter, 22
distribution, 358-359, 412Q, 419E
effect on insulin, 391
effect on somatostatin, 391
and endocrine pancreas, 391
half-life, 360
metabolism, 25

and neurosecretory system, 346


and pheochromocytoma, 363, 412Q,
419E, 432Q, 455E
receptors, 24-25
and renin release, 239
secretion, 347
stimuli for, 359
source, 335t
synthesis, 359
Normetanephrine, 360, 409Q, 417E
Normocardia, 113
Nuclear bag fibers, 66, 67f
Nuclear chain fibers, 66
Nucleases, pancreatic, 317
Null point (transition zone) in
electrocardiography, 112
Nutrients
absorption in stomach, 31 5
intestinal absorption, 451Q, 471E
Nystagmus, 75, 451Q, 472E

o
Obesity, and insulin levels, 393
Obstructive airway disease, see
Chronic obstructive pulmonary
disease
Odorants, 63
Odors, distinguishing between, 63
Ohm's law, 15, 88E
and flow, 124
Olfaction, 62-64, 84Q, 90E
Olfactory mucosa, 63
Oligospermia, 377
Oncotic pressure, 7, 82Q, 88E
in microcirculatory exchange, 96, 97f,
98, 148E
in renal transport, 217, 245Q, 253E,
435Q, 458E-459E
Ondine's curse, 181
Oocyte, 379
and life stages, 378
meiosis in, 378, 383, 386, 407Q,
413E
in ovarian cycle, 381, 383, 384
primary versus secondary, 378, 383
Oogenesis, 378
and life stages, 378
Oogonia, 378
Ootid,378
Opiates
and enkephalins, 351-352
receptors for, binding of endorphins,
350
Opioid, definition, 348
Opioid peptides, 348-352
effect on pain, 350, 351
Optical defects, 453Q, 474E
Optics, physiology of, 51-55, 85Q, 91 E
Oral cavity, 303-305
in swallowing, 305
Oral contraceptives, and
renin-angiotensin system, 240-241
Organ of Corti, 48
movements, 48-49
Organic acids, in body fluids, 209t
Orthophosphates, see Phosphates
Osmolality,S, 6t; 82Q, 88E, 205,
206t; 206t, 244Q, 246Q, 247Q,
252E, 254E
of plasma (serum)
effect on ant,d,uret,c hormone, 455E
calculation of, 281-282
Osmolar gap, 281-282

Osmolarity,S, 431Q, 435Q, 454E,

Overshoot of action potential, 8(, 11,

83Q, 86Q, 89E, 91E

459E
Osmoreceptors, 455E
hypothalamic, and antidiuretic
hormone, 234, 250E-251 E
Osmosis, 5-7, 86Q, 91 E, 205, 212
Osmotic diuresis, 233
Osmotic flow equation, 6, 7, 82Q,

86Q, 88E, 91E


Osmotic gradients, in urine
concentration and dilution,

227-232
Osmotic pressure,S, 6(, 82Q, 86Q,

88E, 91 E, 432Q, 435Q, 456E,


459E
and antidiuretic hormone, 234,
250E-251 E
arterial, effect on urine volume,

432Q,455E
Ossicles, auditory, 45-46, 84Q, 90E
Osteoblasts, 403, 404(, 408Q, 415E
Osteoclasts, 403-404
Osteocytes, 403, 404f
Osteogenesis, effect of parathyroid
hormone, 404
Osteoid, 402
Osteolysis
effect of calcitonin, 405
effect of parathyroid hormone, 404
Osteoporosis, from glucocorticoids,

Ovulation
date of, 382(, 383, 384
definition, 384
identification methods
body temperature, 384
no ferning, 386
in ovarian (menstrual) cycle, 381,

382(, 383, 384, 41 OQ-411 Q,


417E,418E
Ovum, unfertilized, lifespan, 386
Oxygen
alveolar to arterial gradient, effect of
exercise, 138
consumption
calculation of, 432Q, 456E
effect of exercise, 138
myocardial, 121, 133, 142Q, 146E
content in blood, 168
arterial, 431Q, 437Q, 443Q, 452Q,

454E, 462E, 467E, 472E


arterial versus venous, 449Q, 470E
in hypoxia, 183f
pulmonary venous, 431 Q, 454E
delivery, 182, 190Q, 197E, 434Q,

457E
in hypoxia, 184
partial pressure (tension; Po,), 164,

194E
alveolar, 164, 165t, 186Q, 189Q,

369
Otoconia of inner ear, 74
Otolith organs, 72
Ovarian follicles, 379
in ovarian cycle, 381, 383
primary, 378
neuroendocrine control, 383
versus secondary, 381
primordial, 378
secondary (vesicular, graafian)
in ovarian cycle, 381, 383
rupture of, 384
Ovarian (menstrual) cycle, 381-383,

410Q-411Q, 417E, 418E


luteotropic hormone of, 384, 411 Q,
418E
neuroendocrine control, 383-385
ovulation in, 381, 382(, 383, 384,
410Q-411Q, 417E, 418E
phases of. 434Q, 457E
postovulatory (secretory,
progestational), 382(, 383, 384,

385, 431Q, 454E


preovulatory (follicular, proliferative),

381, 382(, 383


duration, 381, 384
hormones in, 383
neuroendocrine control, 383-384
onset, 384
Ovarian steroids
physiologic effects, 385-386
synthesis, 365(, 379, 380-381
Ovary, 377-386
embryology, 377-378
function, 381-383
functional units, 379
histology, 378-379
hormones of, 379-381
morphology, 378-379
in ovarian (menstrual) cycle, 382f
Overhydration, 205, 207, 244Q, 252E
effect on acid-base balance, 269
hyperosmotic, 2061, 206t, 207
hyposmotic, 206(, 206t, 207
isosmotic, 206f, 206t, 207

190Q, 192E, 195E-197E, 434Q,


438Q, 458E, 463E
arterial, 182, 190Q, 197E, 432Q,
437Q, 439Q, 452Q, 455E, 462E,
465E,472E
in hypoxia, 181, 182-183, 184
in respiratory control, 178, 184
venous, 184, 190Q, 197E
and ventilation:perfusion ratio,
175-176, 177f
pulmonary diffusion, 166, 189Q,

195E
and respiratory control, 178, 184
solubility coefficients, 167t
transport in blood, 167-169
Oxygen-dissociation curve, 167-168
see a/50 Oxyhemoglobin dissociation
curve; Hemoglobin
Oxyhemoglobin, 167, 170
buffering role, 264, 266
Oxyhemoglobin dissociation curve
see a/50 Oxygen-dissociation curve;
Hemoglobin
in hypoxia, 183f
Oxyntic glands, 311
Oxyntic (parietal) cells, 311, 328Q,

331 E, 333E
Oxytocin, 353-354
half-life, 339
in labor, 354, 409Q, 416E
and neurosecretory system, 345, 346,

347, 353, 409Q, 416E, 417E


physiologic effects, 354, 409Q, 416E
secretion, 353
source, 335t
structure, 353
synthesis and storage, 353

ectopic, 102
electronic, 103
factors affecting, 99
for slow waves
in small intestine, 317
in stomach, 308
Pacemaker potential, 102
Pacinian corpuscle, 40-41
PAH, see Para-aminohippurate
Pain, 43-44
effect of endorphins, 350
effect of enkephalins, 351
projected, 44
receptors, 43
referred, 44
reflexes, 44, 64-65
relation to temperature perception,

42,43
Palpitation, cardiac, 115
Pancreas, 304(, 31 6
cell types, 31 7
endocrine, 391-396, 408Q, 414E
see a/50 Islets of Langerhans
hormones of, 335t, 391-396, 408Q,

414E
secretions, 317-318, 330Q, 332E,

333E
neural control, 318
phases, 318
vagus nerve in, 318, 328Q, 331E
Pancreatic lipase, 317, 323
Pancreatic polypeptide
cells producing, 317, 391, 412Q,

419E
effect of somatostatin, 393t
effect on insulin secretion, 391
effect on somatostatin secretion, 391

Pancreozymin, source, 335


Panhypopituitarism, 356, 438Q, 456E,

463E
Para-aminohippurate (PAH)
clearance, 212(, 221 (, 243Q, 245Q,

249E, 250E, 252E, 253E


and renal plasma flow, 222-223
renal transport, 216(, 222-223, 245Q,

248Q, 253E, 255E, 451Q, 471E


Para-aminohippurate titration curve,

221(,222
Paracellular (intercellular) pathway of
renal transport, 212
Paracrine control system of pancreatic
islets, 391
Paracrine hormones, 335, 336f
Paracrine signaling (communication),

336(, 342
and APUD cells, 348
Paradoxical (fast-wave) sleep, 79
Parafollicular (C) cells of thyroid, 396
Parasympathetic nervous system
see a/50 Vagus nerve
and cardiovascular function, 133,

140Q,144E
postganglionic fibers, 22
in salivation, 305
in stomach, 308
Parathyroid hormone (PTH), 335t, 401,

402-405
and calcium absorption, 325
cells secreting, 402
effect of calcitriol on, 406
physiologic actions, 404-405, 408Q,

415E
PAC, see Premature atrial contraction
Pacemaker
of heart, 102, 140Q, 145E, 433Q,

439Q, 457E, 465E

effect on bone, 404-405, 406,

408Q,415E
effect on calcitriol, 405, 408Q, 415E
effect on calcium absorption, 405

effect on calcium-binding protein in


bone, 406
effect on electrolyte excretion, 405,
408Q,415E
effect on insulin levels, 393
effect on osteoclasts, 403
effect on osteocytes, 403
effect on phosphate absorption, 405
effect on renal function, 405, 408Q,
415E
and plasma calcium level, 402, 403(,
408Q, 415E, 451Q, 472E
and plasma phosphate level, 451 Q,
472E
precursor forms, 337
in vitamin D synthesis, 403
Parietal lobe, in motor control, 69
Parietal (oxyntic) cells, 311, 328Q,
331E, 333E, 434Q, 457E
Parkinson's disease, 76
Paroxysmal atrial tachycardia (PAT),
113-114
Pars nervosa, 436Q, 460E
Partial pressure (tension) of gases, 163,
190Q, 197E
versus content, in liquid, 167
Parvicellular neurosecretory system,
345-346
in pituitary control, 347(, 348
Passive transport, 2-3
in kidney, 211
PAT, see Paroxysmal atrial tachycardia
Patellar (knee-jerk) reflex, 65, 433Q,
457E
Patent ductus arteriosus, 124f, 135
Peo" see Carbon dioxide, partial
pressure (tension)
Pew, see Transthoracic pressure
Pelvic inlet, shape, 385
Pepsin
formation from pepsinogen, 314
in protein digestion, 303, 315, 322
Pepsinogen, 314
cells secreting, 311, 314
Peptic (chief) cells, 311
Peptidases
intestinal, 323
pancreatic, 31 7
Peptide hormones, 337, 3381, 339
see also Polypeptides; Polypeptide
hormones
Peptidergic neurons
anatomic relationships, 344f
in neural control of pituitary, 344,
345(, 348
and neurosecretory system, 345, 3461
Percent FEY" 163
relation to FEY" 194E
Perilymph of vestibular system, 47
Periodic paralysis, hyperkalemic,
434Q,457E
Peripheral resistance, 125-126, 142Q,
147E
effect on arterial blood pressure, 126,
141Q, 146E, 432Q, 455E
Peristalsis, 303, 305
in colon, 326
in esophagus, 306, 328Q, 331 E
in stomach, 308-309, 328Q, 331 E
gastroduodenal, 307
and migrating motor complex, 310,
333E
in small intestine, 316, 329Q, 332E
Peri tubular capillaries, 201-202
Permeability, and Fick's law of
diffusion, 2

Pernicious anemia, 325


Perspiration, 199
pH, 258-263
see also Hydrogen ion, concentration
in acid-base abnormalities, 277, 278,
289Q, 290Q, 293Q, 295E-298E,
300E
in acidosis, 276, 2771, 289Q, 290Q,
293Q, 295E, 297E, 298E, 300E
in alkalosis, 276, 2771, 289Q, 293Q,
295E, 300E
of body fluids, 257, 259-260, 291 Q,
298E
of blood, 259-260, 290Q, 291Q,
293Q, 294Q, 297E, 298E, 300E,
301 E
arterial, 432Q, 437Q, 439Q,
452Q, 455E, 462E, 465E, 472E
in respiratory acidosis, 433Q, 456E
arterial versus venous, 260
of plasma, 259-260
in acid-base disorders, 453Q, 474E
of gastric secretions, 311
of urine, in acid-base disorders,
453Q,474E
calculation
by Henderson-Hasselbalch equation,
262
from hydrogen ion concentration,
262
conversion to hydrogen ion
concentration, 262-263, 290Q,
296E
compatible with life, 259
effect of carbon dioxide, 283
effect of fixed acid or base, 283
effect on gastric secretions, 313,

328Q, 329Q, 331 E, 332E


relation to hydrogen ion, 258-259
relation to carbon dioxide tension and
bicarbonate ion, 259, 290Q, 298E
in respiratory control, 178- 179,
190Q, 196E
pH-[HCO, -] diagram, 282-285, 294Q,
300E-301 E
in acid-base compensatory responses,
287-288
construction, 282-284
interpretation, 284-285, 294Q,
300E-301 E
uses, 282
Phantom limb sensation, 44
Pharynx, 303, 305-306
in swallowing, 305-306
Phenolic derivatives, 339
Phenotype, 372, 377, 410Q, 417E
Phenylethanolamine-Nmethyltransferase (PNMT), 359
Pheochromocytes, see Chromaffin cells
Pheochromocytoma, 363, 412Q,
419E, 432Q, 455E
catecholamines in, 432Q, 455E
Phosphates
in body fluids, 208, 2091
in plasma
and parathyroid hormone level,
451Q,472E
and vitamin D level, 451 Q, 472E
in bone, 401
effect of calcitriol, 406
effect of parathyroid hormone, 404
as buffers, 2631, 265, 274-275
effect of growth hormone, 356
intestinal absorption
effect of calcitonin, 405
effect of calcitriol, 405, 406-

effect of parathyroid hormone, 405


depletion, effect on calcitriol, 405
renal transport, 2101, 223, 2251,
243Q, 250E, 433Q, 456E
effect of calcitonin, 405, 408Q,
415E
effect of calcitriol, 405
effect of parathyroid hormone, 405,
408Q,415E
and titratable acid, 273-275, 299E
Phosphodiesterases, 342
Phospholipase, 317, 323
Phospholipids
in bile, 319
in plasma, and thyroid hormone
levels, 401
Phosphoprotein phosphatases, 342
Phosphoric acid, 268, 269
source of, 257
Phosphorus, storage in bone, 401, 402
Phrenic nerve, in respiratory control,
176,177
Physiologic shunt, 175, 176, 182-183,
197E
Pickwickian syndrome, 181
Pineal gland, 347
Pituicytes, 353
- Pituitary gland, 352-357
see also Adenohypophysis;
Neurohypophysis
blood supply, 343-344
embryology, 352, 436Q, 460E
endorphins in, 349, 350
enkephalins in, 351
failure of, 433Q, 456E
hormones, 3351, 352-357
control by hypophysiotropic
hormones, 343, 344, 345(, 348
effect of endorphins, 350
morphology, 344(, 352, 408Q, 414E
neural control
hypothalamic-hypophysial, 343-344,
345(,348
neurosecretory neurons, 344-346,
347-348
removal, endocrine effects, 438Q,
463E
pK', definition, 259n, 261
Pl, see Transpulmonary pressure
Placenta, 386-391, 407Q, 414E
and circulation, 135
formation, 386
hormones of, 386-390, 407Q,
410Q-411Q, 414E, 418E
physiologic effects, 390-391
synthesis, 388f
estrogen, 389-390
progesterone, 387, 388(, 389,
409Q,416E
Plasma
albumin in, and oncotic pressure,
253E
.
buffer systems, 264-265, 299E
calcium ion in, 401-402, 403(, 408Q,
415E
clearance mechanisms, and inulin
clearance, 213-214
colloids in, 253E, 254E
composition of, 2091, 244Q, 251 E
crystalloids in, 253E, 254E
hydrogen ion concentration, 2591,
260, 293Q, 300E
ionic concentration, 260
osmolality, effect on antidiuretic
hormone, 455E
pH, 259-260

proteins in, and osmotic pressure, 7,

82Q,88E
renin in, 240-241
volume, 202-203
measurement of, 204-205
role of albumin, 253E
Plethysmograph, 162
Pleural space (interpleural space)
fluid in, 149, 153
gas in, 149
pressure in (PPL; interpleural pressure),
153, 154, 155(, 186Q, 187Q,
189Q, 192E, 193E, 195E, 196E,
431 Q, 435Q, 454E, 460E
and effort-independent flow, 159,
160(, 187Q, 194E
variations in, 172, 189Q, 195E,
196E
PN" see Nitrogen, partial pressure
(tension)
Pneumotaxic center, 178, 190Q, 196E
Pneumothorax, 149
Po" see Oxygen, partial pressure
(tension)
Poiseuille's law, 125-126, 141Q, 146E
Polar bodies, 386
in oogenesis, 378
in ovarian cycle, 383
Polkissen, 239
Polocyte, 378
Polycythemia, 184
Polypeptides, absorption, 323
Polypeptide hormones
see also Peptide hormones
half-life, 339
secretion, 339, 412Q, 419E
size, 337
storage, 337
Polysynaptic reflex, 64
POMC, see Pro-opiomelanocortin
Pons, in respiratory control, 178,
190Q, 196E
Portal circulation, 316, 320f
adrenal, 358
enterohepatic, 319
hypothalamic-pituitary, 343-344, 353
Positive end expiratory pressure (PEEP),
effect on cardiovascular system,
99
Positive pressure ventilation, 449Q,
470E
Postcapillary venules, 95, 140Q, 144E,
148E
Posterior lobe of pituitary, see
Neurohypophysis
Postovulatory phase of menstrual
cycle, see Ovarian cycle,
postovulatory phase
Postpartum endocrinopathy, 433Q,
456E
Postsynaptic response, in central
nervous system, 27
Potassium ion
see also Electrolytes; Ions; Solutes;
Sodium-potassium pump
and action potential, 11-12, 13(,
87Q,92E
and aldosterone, 236, 237, 238, 239
in body fluids, 208, 2091
in gastric secretions, 311, 31 2f
in pancreatic secretions, 31 7
plasma concentration, 260
colonic secretion, 327, 330Q, 333E
and electrical activity of heart, 100,
101-102,103
and glucagon secretion, 393

and hearing, 49-50


in hyperkalemic periodic paralysis,
434Q,457E
and insulin secretion, 393, 395
and renal bicarbonate transport, 271 (,
272(, 273
and renal sodium transport, 224
renal transport, 2101, 211, 223, 225,
244Q, 247Q, 251E, 254E, 451Q,
471E
effect of acetazolamide, 431 Q,
454E-455E
effect of parathyroid hormone, 405
effect of spironolactone, 431 Q,
454E-455E
and renin release, 239
in spermiation, 374
and spironolactone, 241, 242
Potential energy, 124, 125f
Potentials, electrical, see under Cell
membrane; see also Action
potential
PPL, see Pleural space, pressure in
Pre-atriopeptigen, 242
Precapillary sphincter, 95
Pregnancy
diabetes in, 355
effect on growth hormone, 355
effect on insulin, 355
estrogens in, 388(, 389-390
hyperadrenocorticism in, 386
luteotropic hormone of, 390, 411 Q,
418E
progesterone in, 436Q, 461E
prolactin in, 356, 357
Pregnanediol, 381, 389
Pregnanetriol, 381
Pregnenolone
in ovarian steroidogenesis, 380, 381
placental, 388(, 389
in steroidogenesis, 365-366, 374-375
Preload, in muscle contraction
in cardiac muscle, 105, 106, 141 Q,
145E
in skeletal muscle, 32
Premature atrial contraction (PAC), 114
Premature junctional beats, 115
Premature ventricular contraction
(PVC), 114(, 115
Preovulatory phase of menstrual cycle,
see Ovarian cycle, preovulatory
phase
Preprohormones, 337
Presbyopia, 55, 453Q, 474E
Pressoreceptors, 254E
see also Baroreceptors
Pressure
partial, of gases, 163, 190Q, 197E
versus content in liquid, 167
perception of, 41
see also Baroreceptors
Pressure gradient, 124, 125f
Pressure pulse, 123
Presynaptic inhibition, 27-28, 87Q,
92E
Presynaptic nerve terminals, 434Q,
458E
Primary active transport, 3, 4f
in kidney, 210-211, 244Q, 251E
Primordial follicles, 378
Pro-opiomelanocortin (POMC), 337,
349, 409Q, 417E
Proenkephalins, 350-351
Progestational phase of menstrual
cycle, see Ovarian cycle,
postovulatory phase

Progesterone, 379, 3801, 407Q-409Q,


414E,416E
effect on luteinizing hormone, 384
fetal, 388(, 408Q, 414E
metabolite, 389
in ovarian (menstrual) cycle, 382(,
384
plasma level, versus estradiol levels,
384
in pregnancy, 386, 387, 389, 390,
391, 407Q, 409Q-411Q, 414E,
416E, 418E, 436Q, 461E
effect on cortisol, 386, 409Q, 416E
physiologic effects, 390, 409Q,
411Q, 416E, 418E
on basal body temperature, 384
on cervical mucus, 386
on endometrium, 385, 386, 390
on myometrium, 384, 386, 390,
409Q,416E
on renal transport, 437Q, 462E
placental, 387, 388(, 389, 407Q,
409Q, 414E, 416E
conversion to fetal corticoids, 388(,
389
in steroidogenesis, 365-366, 375, 381
synthesis, 365
transport, 379
Progestogens, 379, 3801
catabolism, 381
effect on insulin, 393
level in pregnancy, 389(, 407Q,
409Q-411Q, 414E, 416E, 418E
Prohormones, 337, 437Q, 462E
Proinsulin, 391-392
Projected pain, 44
Projectile vomiting, 310
Prolactin, 356-357
half-life, 339
physiologic effects, 357
precursor form, 337
secretion, 356-357
inhibition of, 357
source, 3351
Prolactin-inhibiting factor (PI F), 345,
357
Prolactin releasing factor (PRF), 356
Proliferative phase of menstrual cycle,
see Ovarian cycle, preovulatory
phase
Proportionality constant, 261
Propranolol, and renin release, 239
Prostate gland, 376, 408Q, 414E
Proteases, pancreatic, 317, 323
Proteins
absorption, 323
in body fluids, 208, 2091, 244Q,
251E
in gastric secretions, 312-313
in saliva, 305
in cell membrane, 1-2, 3f
dietary requirement, 322
digestion, 303, 322-323, 328Q, 331E
in small intestine, 323
in stomach, 315, 322-323, 328Q,
331E
effect of androgens, 377
effect of estrogens, 385
effect of glucocorticoids, 369, 370,
407Q, 408Q, 413E, 415E, 416E
effect of insulin, 393f
intestinal, sources, 322
metabolism
effect of glucagon, 396, 411 Q, 418E
effect of insulin, 395

effect of thyroid hormones, 401,


411Q,418E
of myocardial cells, lOOt; 104, 105f
in plasma, osmotic pressure, 7, 82Q,
88E
renal transport, 210t
of skeletal muscle filaments, 28,
30-31, 84Q, 89E
of smooth muscle filaments, 35, 84Q,
89E
in stool, 323
synthesis, hormones in, 342
growth hormone, 355
Protein hormones
half-life, 339
secretion, 339
size, 337
storage, 337
Protein kinases, 341-342
cyclic AMP-dependent, 341-342
in neurotransmission, 22, 25
Proximal tubule, see under Renal
tubules
PRS, see Transrespiratory pressure
Pseudohermaphroditism, 410Q, 417E
Psyche, effects of glucocorticoids on,
368
PTH, see Parathyroid hormone
PTM, see Transmural pressure
Ptyalin, 305
Puberty, in males, age at, 372-373
Pulmonary, see also Lung; Respiration;
Respiratory
Pulmonary artery pressure, 170, 190Q,
197E, 432Q, 443Q, 455E, 467E
increased, consequences, 184
Pulmonary blood flow, relation to
pressure, 170-171, 189Q, 195E,
196E
see also Ventilation:perfusion ratio
Pulmonary circulation, 170-172, 188Q,
189Q, 195E
changes at birth, 135
Pulmonary edema, 119, 439Q, 465E
see also lungs, fluid in
in mountain sickness, 185
and zones of lung, 171
Pulmonary fibrosis, and compliance,
154, 173
Pulmonary gas exchange, 163-166,
188Q-190Q, 192 E, 194E, 196E,
197E
in hypoxia, 182
Pulmonary hypertension, 119, 171,
184, 439Q, 465E
Pulmonary surfactant, 156-157, 186Q,
187Q, 192E, 193E
Pulmonary vascular resistance,
171-172, 189Q, 195E
Pulmonary vasoconstriction, hypoxic,
171,184,185
Pulmonary venous oxygen content,
431Q,454E
Pulmonary wedge pressure, 136
Pulmonic valve, 119, 141Q, 146E
stenosis, 123
Pulse pressure, 123, 126, 141Q, 146E
factors affecting, 432Q, 455E
Purkinje fibers, 102t; 104, 140Q, 145E
and action potential, 101
as pacemaker, 102
PVC, see Premature ventricular
contraction
Pygmy, and growth hormone, 356
Pyloric glands, 311, 333E

Pyloric obstruction, effect on


electrolytes, 434Q, 457E
Pyloric sphincter, 307
in gastric emptying, 309
Pylorus, 307t; 333E
Pyramidal system (pyramidal tract), 77,
87Q,92E
role in coordinated movement, 70t;
87Q,92E
Pyridoxine, 325

R
R protein, in vitamin B" absorption,
325
Radial traction, in airways, 160, 187Q,
193E, 194E
Ranitidine, 312
Rapid-eye-movement (fast-wave) sleep,
79
RBF, see Renal blood flow
RC, see lungs, time constant
Reactive hyperemia, 95
Receptive relaxation, 435Q, 460E
gastric, 308, 329Q, 332E
Receptors
for acetylcholine, 18, 19t; 20, 22,
83Q,89E
and pacemaker of heart, 433Q,
457E
adrenergic
alpha-adrenergic, 24, 25, 83Q, 89E
and cardiovascular function, 133,
140Q,144E
phYSiologic effects, 361 t, 412Q,
419E
beta-adrenergic, 24, 25, 83Q, 89E
and cardiovascular function, 133,
140Q,144E
and insulin secretion, 391, 409Q,
416E
physiologic effects, 3611, 41 2Q,
419E, 437Q, 462E
and renin release, 239
and beta cells of pancreas, 391,
409Q,416E
and catecholamines, 360, 361,
409Q,416E
for norepinephrine, 24-25
in cell membrane, 2
as effector molecules, 341
for hormones, 337, 340-341, 410Q,
417E
changes in number, 341
for growth hormone, and African
pygmy, 356
for insulin, 393
for somatomedin, 355
muscarinic, 23, 83Q, 89E
nicotinic, 22, 83Q, 89E
for opiates
binding of endorphins, 350
for plasma osmolarity, 455E
in respiratory reflexes, 179
sensory, 35-36
for hearing, 48
for lengthening reaction, 66
for olfaction, 63-64
for pain, 43
for pain reflex, 64
for pressure, 254E
see also Baroreceptors
for taste, 61, 84Q, 90E
as transducers, 35, 37-38
for vestibular system, 72-74

for vision, 56, 86Q, 91 E


electrophysiology, 58-59, 60f
photochemistry, 57-58, 86Q, 91 E
for stretch, 254E
see also Baroreceptors
in blood vessel walls, 131
for stretch reflex, 65
volume, 254E
see also Baroreceptors
Receptor (generator) potential, 37-38
Reciprocal innervation, 65, 84Q, 85Q,
90E
Rectosphincteric reflex, 327
Rectum, 326-327
contractions in, 326
neural control, 327
Reentry, cardiac, 113
Referred pain, 44
Reflection coefficient, 6-7, 82Q, 86Q,
88E, 91 E, 456E
Reflexes
chewing, 303, 435Q, 460E
crossed extensor, 65
Cushing, 134
cutaneous, 64-65, 84Q, 90E
definition, 64
dependent on vago-vagal reflex,
435Q,460E
enterogastric, 309, 431 Q, 454E
flexor, 64-65
in gastric emptying, 309
knee-jerk (patellar), 433Q, 457E
milk let-down (milk ejection), 353
multi synaptic (polysynaptic), 64
originating in muscle, 65-67, 84Q,
90E
pain, 44, 64-65
rectosphincteric, 327
respiratory, 179
salivary, 305
spinal, 64-69, 84Q, 90E
and spinal shock, 72
stretch, 65-66, 84Q, 90E, 433Q,
457E
role in control of movement, 67-69
swallowing, 435Q, 460E
vago-vagal, 435Q, 460E
vestibular, 74-75
withdrawal, 64-65, 84Q, 90E
Refractive errors, 54-55
Refractive media of eye, 52-54
Refractory periods of cell membrane,
12-13, 83Q, 86Q, 87Q, 89E, 91 E,
92E
in myocardial cells, 101
Reissner's membrane, 47
Releasing hormones, source, 335t
REM (rapid-eye-movement, fast-wave)
sleep, 79
Renal, see also Kidney
Renal arteriole, and glomerular
filtration rate, 219-220
Renal blood flow
autoregulation, 218-219
effective, 223
and glomerular filtration rate, 218-220
and para-aminohippurate clearance,
222-223
measurement of resistance to, 434Q,
458E
as test of renal function, 452Q,
472E-474E
versus renal plasma flow, 223
Renal clearance, 212-217, 243Q,
245Q, 249E, 252E
calculation of, 213-215, 243Q, 249E

clearance ratio, 214-215


Renal compensation, 285-286, 291Q,
294Q, 298E, 299E, 301 E
Renal epithelial transport, 210-212
Renal erythropoietic factor, see
Erythropoietin
Renal function, 209-212, 243Q-248Q,
249E-255E
effect of calcitonin, 405, 408Q, 415E
effect of calcitriol, 406
effect of glucocorticoids, 368, 370
effect of parathyroid hormone, 405,
408Q,415E
tests of, 452Q, 472E-474E
creatinine clearance, 217
filtered glucose load, 452Q,
472E-474E
glomerular filtration rate, 217,
472E-474E
glucose transport maximum, 222,
245Q,253E
inulin clearance, 213-214
para-aminohippurate transport
maximum, 222
renal blood flow, 452Q, 472E-474E
renal plasma flow, 452Q,
472E-474E
tubular clearance ratio, 214-215
tubular fluid/plasma concentration
ratio, 215, 216f
Renal nerves, and renin release, 239
Renal physiology, 199-242,
243Q-248Q, 249E-255E
Renal plasma flow
effect of aldosterone, 236
effective versus true, 223
as test of renal function, 452Q,
472E-474E
versus renal blood flow, 223
Renal threshold concentration, 220,
221 (, 244Q, 251E-252E
Renal titration curve, 221, 245Q, 253E
Renal tubules, 200, 247Q, 255E
collecting duct, 200, 247Q, 254E,
255E
in urine concentration and dilution,
227, 228(, 229(, 230, 247Q,
254E, 255E
distal convoluted tubule, 200
function, 210t
sodium-potassium (Na+ -K+)
exchange process, 236
in urine concentration and dilution,
227, 228(, 229(, 230
maximum tubular reabsorptive
capacity (Tm, Tr), 220, 243Q,
250E
maximum tubular secretory capacity
(Tm, Ts), 220, 243Q, 244Q,
250E-252E
proximal convoluted tubule, 200,
201 (, 243Q, 247Q, 249E, 255E
electrolyte transport in, 211 (, 432Q,
455E
function, 21 Ot
transport systems, 211 f
in urine concentration and dilution,
227, 228(, 229f, 230(, 247Q,
255E
reabsorption in, 209, 210t, 220-227,
243Q, 247Q, 250E, 254E-255E
and inulin clearance, 214, 215
secretion in, 209, 21 Ot, 220-227,
243Q, 247Q, 249E, 250E,
254E-255E
and inulin clearance, 214, 215

structure, 200, 201 (, 247Q, 254E,


255E
transport in, 209, 210t, 220-227,
247Q, 248Q, 254E-255E
hormonal effects on, 437Q, 462E
tubular clearance ratio, 214-215
tubular fluid/plasma concentration
ratio, 215, 216(, 245Q, 247Q,
248Q, 252E, 254E, 255E
tubular fluid/ plasma osmolality ratio,
229f
tubular transport maximum (Tm), 220,
243Q, 245Q, 250E, 253E
Renin, 199, 244Q, 245Q, 250E, 252E,
450Q, 470E-471 E
and angiotensin synthesis, 239-240,
245Q, 246Q, 252E, 254E
as hormone, 347
and juxtaglomerular cells, 238, 244Q,
250E
in plasma, 240-241
secretion, factors affecting, 239,
244Q,250E
effect of estrogens, 390
Renin-angiotensin-aldosterone system,
131, 142Q, 146E
Renin-angiotensin system, 240f, 245Q,
246Q, 252E, 254E
and aldosterone, 238, 244Q, 245Q,
250E, 252E
Repolarization
of action potential (downstroke
phase), 8(, 11, 12, 83Q, 86Q,
89E, 91 E, 92E
in electrocardiography, 107, 108f
of myocardial cells, 101-102
Reproductive tract
embryology, 370-372, 377-378
Residual volume
of lung (RV), 153f, 161-162, 163f,
186Q, 188Q, 190Q, 191Q, 192E,
196E, 197E, 446Q, 468E
in lung disease, 163f
of ventricle, 442Q, 466E
Resistance
to blood flow, 125, 141 Q, 142Q,
146E, 147E
and arterial blood pressure, 126,
142Q,147E
in kidney, measurement of, 434Q,
458E
peripheral
effect on arterial blood pressure,
432Q,455E
total peripheral, 125-126
Resistance forces, 152(, 186Q, 187Q,
192E, 194E
in respiratory mechanics, 158-160,
161(, 187Q, 193E
Resistance work of breathing, 161
Respiration
in acid-base balance, 269-270
effect of exercise, 138
external versus internal, 149
patterns of, 180-181, 190Q, 196E
regulation of, 176-181, 190Q, 196E
dysfunction of, 180-181
in hypoxia, 184
and venous return, 99
Respiratory, see also Pulmonary; Lung
Respiratory acid-base imbalance,
436Q, 448Q, 453Q, 461 E,
468E-469E, 474E
Respiratory acidosis, see under
Acidosis

Respiratory alkalOSiS, see under


Alkalosis
Respiratory centers in brain, 176-178,
190Q, 196E
Respiratory compensation, 179,
285-286, 291Q, 294Q, 298E,
299E, 301 E
Respiratory cycle, 151-153, 191 Q,
197E
as buffer system, 269
Respiratory distress syndrome, 157,
186Q, 187Q, 192E, 193E
Respiratory drive, 449Q, 470E
in chronic hypoxia, 184, 190Q, 197E
evaluation of, 179- 180
Respiratory exchange ratio, effect of
exercise, 138
Respiratory failure, from respiratory
muscle fatigue, 161
Respiratory function tests, 179-180,
191Q, 197E
in hypoxia, 182t
Respiratory (lung) compliance, 432Q,
439Q, 440Q, 455E, 465E
Respiratory mechanics, 149-163
Respiratory membrane, 165-166
Respiratory quotient, 266
Respiratory reflexes, 179
Respiratory responses to gases, as test,
179-180
Respiratory system, 149-185,
186Q-191Q,192E-197E
see a/so Lungs
functions, 149
physiology, 149-185, 186Q-191 Q,
192E-197E
pressure-volume relationships,
154-156, 162(, 186Q, 192E,
193E, 196E
Respiratory tract obstruction, chronic,
453Q,474E
Rest (spontaneous) tremor, 76, 77,
87Q,91E
Resting potential (resting membrane
potential; RMP), see under Cell
membrane
Restrictive lung disease (RLD), 163(,
154, 187Q, 193E
Retching, 310
Reticular activating system (RAS), 80,
85Q,90E
in respiratory control, 177
Reticular formation, 71-72
and cardiovascular function, 132
Retina, 55-59
cell types, 55-57
circuitry, 59, 60f
electrophysiology, 58-59, 60f
structure, 55f
Retropulsion, gastric, 308, 309
Reverberating circuits of reflex
pathway, 64
Reversal potential, 21
Reverse triiodothyronine, 397
Rhodopsin, 57-58
Riboflavin, 325
Rigidity, 74
alpha, 74
decerebrate, 72, 74
gamma, 74
lead-pipe (cogwheel), 76
Rigor mortis, molecular cause, 31
RMP, see resting potential
Rods of retina, 56, 86Q, 91 E
RPF, see Renal plasma flow
RRP, see Refractory periods

Ruffini's corpuscle, 41
RV, see Residual volume, of lung

s
S, see Solubility constant
S fibers, see Slow-twitch fibers
SA, see Sinoatrial node
Saccule, 45t; 72
Salicylate toxicity, 433Q, 456E-457E
Saliva (salivary secretions), 303,
304-305
composition, 305
functions, 305
Salivary glands, 304-305
Salivary reflexes, 305
Salivation, neural control, 305, 328Q,
331E
Saltatory conduction, 14
Sarcomeres, 19t; 28, 29f
of myocardial cells, lOOt; 104, 145E
Sarcoplasmic reticulum (SR)
and calcium ion, 22, 25, 82Q-84Q,
88E-90E
of myocardial cells, lOOt; 104
of skeletal muscle, 28, 29
Saturation kinetics, 3
Scuba diving, 188Q, 194E, 195E
SEC, see Series elastic component
Secondary active transport, 3-5, 211 t;
212, 244Q, 251E, 255E, 272
Secondary (graafian, vesicular) follicle,
in ovarian cycle, 381, 383
Secondary sex characteristics, male,
and androgens, 376, 377, 410Q,
417E
Secosteroids, 339
Secretagogues
for gastrin, 314, 330Q, 333E
for pancreatic proteases, 323
Secretin, 310, 314, 328Q-330Q,
331 E-333E
and biliary secretion, 320, 329Q,
332E
effect on slow waves, 317, 329Q,
332E
and pancreatic secretion, 318, 329Q,
332E
source, 335
Secretomotor neurons, and innervation
of endocrine tissue, 346-347
Secretory phase of menstrual cycle, see
Ovarian cycle, postovulatory
phase
Segmentation contractions
intestinal, 316, 317, 329Q, 332E,
435Q,460E
in colon, 326
and slow waves, 316-317
Semen, 376-377
Semicircular canals, 45t; 72-74
Semilunar valves, 119, 122, 141 Q,
143Q, 146E, 147E
Seminal fluid, 376, 408Q, 414E
Seminal vesicles, 376, 377, 408Q,
414E
Seminiferous (spermatogenic) tubules,
373
Sensible sweat, 199
Sensory adaptation, 38
Sensory coding, 38

Sensory mechanisms, 35-40, 84Q, 90E


Septic shock, 136
Series elastic component (SEC), 33-34
Serotonin, and APUD cells, 348

Sertoli (sustentacular) cells, 372, 408Q,


414E
and female phenotype, 377
functions, 373-374, 408Q, 414E
morphology, 373
in spermatogenesis, 373-374,
375-376
Sex
genetic versus gonadal versus
phenotypic, 372, 410Q, 417E
male secondary characteristics, and
androgens, 376, 377, 410Q, 417E
Sex steroids, 366, 374-375, 379-381
Sex steroid-binding globulin, 379
Sheehan's syndrome, 456E
Shock, 135-136
Short-loop feedback, 344, 345f
Shunt equation, 469E
Shunts, of blood, 175, 176, 182-183,
189Q, 190Q, 196E, 197E, 448Q,
469E
SIADH, see Syndrome of inappropriate
antidiuretic hormone secretion
Sick sinus syndrome, 116
Sickle cell anemia, hemoglobin in, 169
Sigmoid colon, 326f
SIH (somatotropin-inhibiting hormone),
see Somatostatin
Simple diffusion, 2-3, 82Q, 88E
in kidney, 210, 244Q, 251 E
Single-breath nitrogen test, 173-174
Sinoatrial (SA) node, 102t; 140Q,
141Q, 145E
as pacemaker, 102
SA nodal block, 11 6
Sinus arrest, 103
Sinus arrhythmia, 113
Sinus rhythm, 113
Sinus tachycardia, 113
Skeletal muscle, 28-34, 84Q, 89E
blood flow in, 134-135, 140Q, 144E,
148E
contraction, 30-34, 84Q, 89E
isometric, 31-33
isotonic, 33-34, 86Q, 91 E
mechanical properties, 31-34
as pump, 99
structure, 28-29
Sleep, 78-81, 85Q, 90E
and antidiuretic hormone release, 235
assessment of, 78-79
disorders of, 80, 85Q, 90E
fast-wave (rapid-eye-movement,
desynchronized, paradoxical,
dream), 79
slow-wave, 78-79, 85Q, 90E
stages, 79, 85Q, 90E
states, 78-79
Sleep apnea, 181, 190Q, 196E
Sleep centers, 80, 90E
Sleep cycle, 79-80
Sleepwalking, 80
Slow channels, of myocardial cells,
100
Slow fibers, of myocardial cells,
100-101
Slow-twitch (S) fibers, 70-71, 84Q, 90E
Slow vital capacity, 162
Slow waves (basic electrical rhythm),
308-309
in sleep, 78-79, 85Q, 90E
in small intestine, 316-317

in stomach, 308
Small intestine, 315-325
digestion and absorption in, 315,
322-325

and enterohepatic circulation, 319


motility, 316-317, 329Q, 330Q,
332E, 333E
structure, 304t; 315-316
Smell, 62-64, 84Q, 90E
Smoking, consequences of, 448Q,
468E-469E
Smooth endoplasmic reticulum, 324
Smooth muscle, 28, 35, 84Q, 89E
Snoring, 181, 196E
Sodium channels, of myocardial cells,
101
Sodium ion
see a/50 Ions; Electrolytes; Solutes
and action potential, 11-13, 83Q,
87Q, 88E, 89E, 92E
in amino acid and polypeptide
absorption, 323, 328Q, 329Q,
331 E, 332E
in body fluids, 208, 2091, 246Q,
254E
in carbohydrate absorption, 322,
328Q, 329Q, 331E, 332E
as carrier in transport, 212, 244Q,
251 E, 255E
colonic absorption of, 327
effect of aldosterone, 236, 238, 239
ver~us antidiuretic hormone effects,
236
effect of antidiuretic hormone, 234,
236, 244Q, 250E-251 E
effect of estrogen, 385
effect of growth hormone, 356
effect of insulin, 395
effect of progesterone, 386, 390,
409Q,416E
effect of spironolactone, 241, 431 Q,
454E-455E
and electrical activity of heart, 100,
101-102
electrogenic cotransport, 212
electro neutral cotransport, 21 2
electro neutral countertransport, 212
and extracellular fluid volume, 205
and hypernatremia, 246Q, 254E
and hyponatremia, 237
intestinal absorption of, 324
in pancreatic secretions, 31 7
plasma concentration, 260
and renal bicarbonate transport, 224,
271 t; 272-273
and renal chloride transport, 226
renal transport of, 210-212, 216t;
223, 224-225, 244Q, 246Q,
248Q, 251 E, 254E, 255E,
271-272, 291Q, 298E
effect of acetazolamide, 431 Q,
454E-455E
effect of calcitonin, 405
effect of parathyroid hormone, 405
reabsorption, 211t; 224-225, 246Q,
254E
and renin release, 239
role in vision, 59
in urine concentration and dilution,
227-231
Sodium-calcium exchange, and cardiac
muscle, 34
Sodium-coupled transport, 212, 244Q,
251E,255E
Sodium-hydrogen antiporter

pancreatiC, 31 7
renal, 272
Sodium-potassium (Na + -K+) exchange
process, 236

Sodium-potassium pump [Na+ -K+


pump; sodium-potassium
adenosine triphosphatase
(Na+ -K+ -ATPase); Na+ -K+ -ATPase
pump], 3, 4(, 82Q, 88E
and Donnan equilibrium, 7
in renal transport, 210-211, 212, 225,
251E
and membrane potential, 10- 11,
11-12
thyroid hormones and, 400
Sodium chloride
and extracellular fluid volume, 205,
244Q, 252E
intestinal absorption, 324
in urine concentration and dilution,
226, 227-231, 255E
Solubility constant (5), 261
Solutes
see a/50 Electrolytes; Ions; and
specific substances (e.g., Sodium
ion)
in body fluids, 208, 209t, 244Q,
245Q, 251 E, 252E
concentration in nephron, 215, 216f,
247Q, 254E
renal clearance, 212-217, 243Q,
245Q, 249E, 252E, 253E
renal excretion, 199, 243Q, 249E
renal transport, 210t, 212, 214(, 215,
216(, 223-226, 243Q, 247Q,
248Q, 249E, 254E-255E
Somatocrinin, see Growth hormone
Somatomedin, 355, 407Q, 409Q,
413E,417E
distribution, 355
and growth hormone, 355
and growth retardation, 356
physiologic effects, 355
source, 335
Somatostatin (somatotropin-inhibiting
hormone, SIH), 354, 407Q, 413E
and acetylcholine, 391
cells secreting, 317, 391, 412Q, 419E
distribution, 354
effect on insulin secretion, 391, 392,
393t
effect on pancreatic cells, 391
effect on thyroid-stimulating hormone,
400
and gastrointestinal hormones, 392,
393t
and glucagon, 391, 393t
and glucose, 391, 392, 412Q, 419E
in diabetes mellitus, 391
and norepinephrine, 391
and pancreatic polypeptide, 391,
393t
in gastric secretions, 312, 313, 314,
328Q, 329Q, 331 E-333E
physiologic effects, 354, 392, 393t
in signal transmission, 342
Somatotropin, 407Q, 413E
Somatotropin-inhibiting hormone, see
Somatostatin
Somatotropin releasing factor, 354
Somatotropin (somatotropic hormone),
see Growth hormone
Somnambulism, 80
Sound, localization of, 51
Sound frequency, perception of, 50
Sound intensity, perception of, 50

Sperm, 377
lifespan, 386
Spermatocytes, 373
Spermatogenesis, 373, 375-376
versus oogenesis, 378
Spermatogonia, 373, 378
Spermatozoa, 377, 378
Spermiogenesis, 373
Sphincters
anal, 326-327
esophageal, 306, 328Q, 331 E
ofOddi, 316, 320(, 321
pyloric, 307, 309
Spinal cord, 70-71
in coordinated movement, 69, 70f
Spinal reflexes, 64-69, 84Q, 90E
Spinal shock, 72
Spinnbarkheit, 385
Spinocerebellum, 76, 77
Spirogram, 162f
Spirometer, 151, 191Q, 197E
Spironolactone, 241-242
effect on renal potassium, 431 Q,
454E-455E
effect on renal sodium, 431 Q,
454E-455E
Spontaneous (rest) tremor, 76, 77,
87Q,91E
SR, see Sarcoplasmic reticulum
Standing
effect on transpulmonary pressure,
435Q,460E
effect on venous system, 99, 131,
140Q,145E
Starch, digestion, 303
by saliva, 305
Starches, 322
Starling hypothesis, 96, 97(, 148E
Starling's law of the heart
(Frank-Starling effect/relationship),
105-106,128, 141Q, 142Q,
145E, 147E
Steady state, 343
Steatorrhea, 319-320, 453Q, 474E
Sterane (gonane), 339
Stereocilia
of organ of Corti, 48-50
of vestibular system, 73-74
"Steroid diabetes," 3 69
effect of estrogens on, 390
Steroid hormones, 338t, 339-340
see a/50 specific hormones
half-life, 340
interaction with chromatin, 342
ketogenic, 367
and mesoderm, 364
ovarian, 379-381
parent compound, 339
secretion, 340
storage, 340
structure, 339
synthesis, see Steroidogenesis
testicular, 374-375
Steroid-binding globulins (steroid
hormone-binding globulin), 339
effect of estrogens on, 390
Steroidogenesis
biosynthetic pathway, 365f
ovarian, 365(, 379, 380-381
testicular, 365(, 374-375
Stevens power law function, 38
5TH, see Growth hormone

transduction by receptor, 37-38


Stokes-Adams syndrome, 116, 132
Stomach, 304(, 307-315
see a/50 Gastric
digestion and absorption in, 315
effect of glucocorticoids, 368
innervation, 307-308
musculature, 307
structure, 307
Stratum basalis (basal e) of
endometrium, 379
effect of estrogens, 385
in ovarian cycle, 381
Stratum functionalis (functionale) of
endometrium, 379
effect of estrogens, 385
effect of progesterone, 386
in ovarian cycle, 381
Stress
cardiovascular responses to, 135-139
effect on hypophysial-adrenocortical
control system, 368
glucocorticoids and, 368, 369
Stretch receptors, 254E
see a/50 Baroreceptors
Stretch reflex, 65-66, 84Q, 90E, 433Q,
457E
role in control of movement, 67-69
Striated muscle, 28
Striatum, 75, 76, 87Q, 92E
Stroke volume of heart, 119-121, 123,
431Q, 432Q, 442Q, 454E, 455E,
466E
and arterial blood pressure, 126
and contractility, 120f
effect of exercise, 137
effect of increased afterload, 120f
Stupor, 80
Submucosal (Meissner's) plexus, 307
Substance P
and airway dimensions, 159
and signal transmission, 342
Substantia nigra, 75, 76, 87Q, 92E
Subthalamic nucleus, 75-76
Sucrase, 322
Sucrose, 322
Sudden infant death syndrome (5105),
181
Sulfur, in body fluids, 209t
Sulfuric acid, 268, 269
source, 257
Summation
in muscle contraction, 33
synaptic, 26, 27
Supraventricular tachycardia, 113
Surface forces, in respiratory
mechanics, 156, 157(, 158f
Surface potential, in
electrocardiography, 107
Surface tension, in respiratory
mechanics, 156, 157f, 158(,
186Q, 187Q, 192E, 193E
Surfactant, pulmonary, 156-157, 186Q,
187Q, 192E, 193E
Sustentacular cells, see Sertoli cells
Swallowing (deglutition), 303, 305-307,
328Q, 331 E
disorders of, 306-307
phases of, 305-306
Swallowing center, 305, 306, 328Q,
331E
Swallowing reflex, 435Q, 460E

Sound waves

Stimulus

Sweat, sensible, 199

physics of, 44-45


propagation of, 48
Spasticity, 72, 74, 451Q, 472E

perception of intensity, 38, 39f


perception of location, 38-39
perception of quality, 39-40

Sympathetic nervous system


and aldosterone secretion, 239
effect on coronary Circulation, 134

effect on heart, 103, 104, 106, 127f,


132-133, 441 Q, 465E
effect on kidney, 219
effect on pacemaker, 99
effect on vascular function, 127f, 128
and norepinephrine content of tissue,
358, 409Q, 416E
postganglionic fibers, 24-25
and renin release, 239
in salivation, 305, 331 E
in stomach, 308
Sympathoadrenomedullary axis, 347f,
359, 409Q, 416E
index of activity, 360
Sympathomimetic drugs
see a/50 Catecholamines; Epinephrine;
Norepinephrine
effect on coronary circulation, 134
effect on heart, 103, 104, 106, 127f
effect on vascular function, 127f, 128
Symport (cotransport), 211 f, 212,
246Q,254E
Symporters, 5
Synapse, 18, 19f, 83Q, 89E
see a/50 Neuromuscular junction
axo-axonic, 27-28, 87Q, 92E
Synaptic junction, see Neuromuscular
junction
Synaptic neurotransmitter, 434Q, 458E
Synaptic transmission, 18-28, 83Q,
84Q, 89E, 90E
in autonomic nervous system, 22-25
in central nervous system, 25-26
Syncope (fainting), 99, 134, 135
Syncytia of heart, 99
Syncytiotrophoblast, 386, 387f
Systemic filtration, 217-218, 253E
Systemic filtration rate, 218
Systemic hypertension, 119
Systole, 99, 122, 123, 146E, 447Q,
468E
Systolic pressure, 123, 148E
factors affecting, 432Q, 455E

T
T" see Triiodothyronine
T., see Thyroxine
T tubules
of myocardial cells, 100f, 104
of skeletal muscle, 29
Tachycardia, 113, 140Q, 145E, 182
atrial, 113-114
in hypoxia, 184
junctional, 115
sinus, 113
supraventricular, 113
ventricular, 115-116, 140Q, 145E
Tachypnea, 179, 182
Taenia coli, 326f
Taste, 61-62, 84Q, 90E
Taste buds, 61, 62
TBG, see Thyroxine-binding globulin
TBPA, see Thyroxine-binding
prealbumin
TBW, see Total body water
Te, see Terminal cisternae
Tectorial membrane of inner ear, 48,
49f
Temperature
of body, basal, 382f, 384
perception of, 42-43
Tension
of gases, 163
of muscle, 31

Terminal cisternae (TC)


of myocardial cells, 100f, 104
of skeletal muscle, 29, 30
Testicular feminization syndrome,
410Q,417E
Testicular parenchyma, histology,
373-374, 408Q, 415E
Testicular steroidogenesis, 365f,
374-375
Testis, 370-377
embryology, 370-372
in fetus, effects of HCG on, 390
hormones of, 374-375
morphology, 372-374, 408Q, 415E
Testosterone, 365t, 366, 374, 408Q,
415E
as estrogen precursor, 365f, 375,
408Q,415E
in female, 379, 380t
and female phenotype, 377, 410Q,
417E
and male phenotype, 372
in ovarian sterOidogenesis, 381
physiologic effects, 376t, 408Q, 415E
in fetus, 376t, 377, 408Q, 415E
in puberty, 376t, 377
in spermatogenesis, 375-376
as precursor of estradiol, 365f, 375,
381
source, 335t
synthesis, 365f, 374-375
Tetanus, 33, 84Q, 90E
Tetrahydrocortisol, 367
Tetraiodothyronine, see Thyroxine
Theca cells (theca interna cells), 379
in steroidogenesis, 379
Theca externa, in ovarian cycle, 383
Theca interna
effect of LH on, 383
in ovarian cycle, 383
Thermodilution, to measure cardiac
output, 13 0- 131
Thermoreceptors, 42
Thiamine, 325
Threshold concentration, 220, 221 f,
244Q, 251 E-252E
Threshold of action potential, 8f, 10,
83Q, 86Q, 87Q, 88E, 91 E, 92E
Threshold potential, 100
Thyroglobulin, 339, 397
site of iodination, 396
Thyroid follicle, 396
Thyroid gland, 396-401, 407Q, 408Q,
414E
control of, 399
autoregulation, 400
functions, 396-397
histology, 396
iodide content, 397
test of, 398
Thyroid hormones, 335t, 338t, 339,
396-401, 407Q-409Q, 411 Q,
414E, 416E, 418E
binding proteins for, 397
and catecholamines, 341
chemistry, 396-397
circulation, 339
effect on basal metabolic rate (BMR),
400
effect on epinephrine, 401
effect on glucose, 400-401
effect on insulin, 400
effect on iodide uptake, 399, 411Q,
418E
effect on thyroid-stimulating hormone,
399, 411Q, 418E

excretion, 399
half-life, 339
interaction with chromatin, 342
metabolic effects, 400-401, 411 Q,
418E
metabolism, 399
physiologic effects, 400, 411 Q, 418E
and plasma lipid levels, 401
secretion, 339, 399
storage, 339
structure, 339
synthesis, 339, 398-399, 409Q, 416E,
438Q,464E
role of thyroid gland, 397
transport, 397
Thyroid peroxidase, in thyroid
hormone synthesis, 399, 409Q,
416E
Thyroid-stimulating hormone (TSH)
effect of goitrogens on, 400
effect on thyroid follicle, 396
functions, 399
secretion, control of, 399-400, 411 Q,
418E
Thyronines, 397
Thyrotropin, effect of somatostatin,
393t
Thyrotropin releasing hormone (TRH),
399, 409Q, 41 7E
and prolactin, 356
Thyroxine
chemistry, 396-397
effect of estrogen, 386
source, 335t
synthesis, 438Q, 464E
as triiodothyronine precursor, 397,
399
Thyroxine-binding globulin (TBG), 339,
397
effect of estrogen on, 386, 390
Thyroxine-binding prealbumin (TBPA),
397
Thyroxine-binding proteins, 397
Tidal volume (VT) of lung, 161, 162f,
172, 188Q-190Q, 194E, 196E
in breathing patterns, 180f, 187Q,
193E
Tight junctions, 96
of Sertoli cells, 373
Timed vital capacity, see Forced
expiratory volume in 1 second
(FEV,)
Tinnitus, 51
Tissue buffers, 264, 292Q, 299E
Tissue hypoxia, and hyperiacticemia,
268
Titratable acid, 243Q, 250E, 433Q,
456E
definition, 273
excretion, 226, 273-275, 289Q,
291 Q, 296E, 298E-299E
TlC, see Total lung capacity
Tm, see Tubular transport maximum;

Maximum tubular reabsorptive


capacity; Maximum tubular
secretory capacity
Tongue, 304
in swallowing, 305
Tonicity, 6, 432Q, 435Q, 456E, 459E
Tonotopic organization, 50

Total body water (TBW), 202, 203(,


246Q, 253E, 431Q, 443Q, 454E,
467E
volume measurement, 204

Total lung capacity (TlC), 153t; 162,


186Q, 188Q, 191 Q, 192E, 197E,
446Q,468E
in chronic hypoxia, 184
in lung disease, 163f
Touch, perception of, 41-42
Toxins (intoxicants), and osmolar gap,
281-282
Tr, see Maximum tubular reabsorptive
capacity
Transcellular fluid, 203, 243Q, 244Q,
249E, 251 E
Transcellular pathway of renal
transport, 212, 244Q, 251 E
Transcortin, see Cortisol-binding
globulin
T ransepithelial transport, 21 2
Transference equation, and membrane
potential, 15, 16, 82Q, 88E
Transferrin, 325
Transition zone (null point), in
electrocardiography, 11 2
Transmembrane proteins, 2, 3f
Transmural pressure (PTM), 154, 155t;
156, 186Q, 187Q, 192E, 193E,
196E
Transport
active, 3-5, 82Q, 88E, 220
primary, in kidney, 210-211, 244Q,
251E
secondary, 211 (, 212, 244Q, 251 E,
255E, 272
carrier-mediated, 3-5, 82Q, 88E
in kidney, 212
circulatory-interstitial, 95, 96-98,
140Q,144E
electrogenic, 212
elecironeutral, 21 2
energy requirements, 212, 254E
facilitated, 3, 82Q, 88E
passive, 2-3
in kidney, 211
Transport globulins, effect of estrogen
on, 386, 390
Transport maximum (Tm), see Tubular
transport maximum
Transpulmonary pressure (Pl), 154,
155(, 159, 172, 431 Q, 435Q,
454E,460E
Transrespiratory pressure (PRS),
154-156, 186Q, 192E, 197E
Transthoracic pressure (Pew), 154,
155f
Transverse colon, 326f
Tremor
intention, 77
in Parkinson's disease, 76, 77
spontaneous (rest), 76, 77, 87Q, 91 E
TRH, see Thyrotropin releasing
hormone
Triaxial electrocardiography system,
109(,110
Tricuspid valve, 11 8
Triglyceride metabolism, 394
T riglycerides
cleavage, 323
reconstitution, 324
and thyroid hormone levels, 401
Trihydroxycholic acid, 319
Triiodothyronine
chemistry, 397
formation from thyroxine, 397, 399
in pituitary, 399
source, 3351
Triiodothyronine-binding proteins, 397
Trophoblast, 386

estrogen secretion by, 389


progesterone secretion by, 389
Tropic hormones, source, 3351
True renal plasma flow, 223
Trypsin, 303, 317, 323
Trypsin inhibitor, 318
Trypsinogen, 317, 323
Ts, see Maximum tubular secretory
capacity
TSH, see Thyroid-stimulating hormone
Tubular clearance ratio, 214-215
Tubular fluid/plasma concentration
ratio, 215, 216(, 245Q, 247Q,
248Q, 252E, 254E, 255E
Tubular fluid/plasma osmolality ratio,
in urine concentration and
dilution, 229f
Tubular transport maximum (Tm), 220,
243Q, 245Q, 250E, 253E
Tubule, see Renal tubules; T tubules
Tunica albuginea, 372
Tunica vaginal is, 372
Two-point threshold, 39, 40f
Tyrosine, and catecholamine synthesis,
359

u
Ulcer, duodenal, 330Q, 333E
Ultrashort-loop feedback, 344, 345f
Undershoot (hyperpolarization) phase
of action potential, 8(, 11, 12,
83Q,89E
Uniporters, 5
Unloading, in muscle contraction, 68,
457E
Unloading zone of oxygen-dissociation
curve, 167(, 168
Upright posture
effect on transpulmonary pressure,
435Q,460E
effect on venous system, 99, 131,
140Q,145E
Upstroke (depolarization) phase of
action potential, 8(, 11, 12, 83Q,
86Q, 87Q, 89E, 91 E, 92E
Urate, 243Q, 249E
Urea
clearance, 212(, 245Q, 252E
production, 199, 208(, 2091
renal transport, 2101, 228(, 243Q,
247Q, 250E, 254E, 255E
in urine concentration and dilution,

227, 228(, 231-232, 247Q, 255E


Uric acid, 199, 220, 243Q, 249E,
250E
source, 257
Urinary 17-ketosteroids, 366, 367, 375
Urinary acid, excretion, 2741
Urinary metabolites, as index of
testicular function, 375
Urine
acidification, 271
composition of, 208(, 2091
hydrogen ion concentration, 2601
concentration and dilution, 227-233,
246Q, 247Q, 254E, 255E
anatomic participants, 227, 246Q,
247Q, 254E, 255E
mechanisms of, 227
pH,2601
in a~id-base disorders, 453Q, 474E
Urine/plasma concentration ratio, see
Tubular fluid/plasma concentration
ratio

Urine/plasma osmolality ratio, see


Tubular fluid/plasma osmolality
ratio

Urobilin, 319
Urogenital tract, embryology, 371 (,
377
Uterine motility
effect of estrogens, 384
effect of progesterone, 384, 390,
409Q,416E
Uterus, morphology, 379
Utricle, 45(, 72, 73, 74f

v
Vago-vagal reflex, 435Q, 460E
Vagus nerve, 438Q, 464E
and airway resistance, 159, 187Q,
193E
and atrioventricular node, 103
in esophageal peristalsis, 306
in gallbladder contraction, 321
in gastric motility, 308
in gastric secretions, 310, 312, 313(,
314, 328Q, 331E
and heart rate, 102, 132
and insulin release, 346
in pancreatic secretion, 318, 328Q,
331E
in receptive relaxation, 308, 329Q,
332E
in respiratory control, 177, 178,
190Q, 196E
and salivation, 305, 328Q, 331E
Val salva maneuver, 151, 431 Q, 454E
effect on cardiovascular system, 99,
140Q,145E
Valves
arteriolar, 94
of heart, 118-119, 122, 141Q, 143Q,
146E, 147E-148E
disorders, 123-124, 445Q, 468E
venous, 99
Valvulae conniventes, 315
van't Hoff equation,S
Vanillylmandelic acid (VMA), 360,
409Q,417E
Varicose veins, 99
Vasa recta, 202
in urine concentration and dilution,
227, 230-231
Vascular cuffing, 171
Vascular function curves, 127-128
Vasoactive intestinal peptide (VIP)
and airway dimensions, 159
source, 335
Vasoconstriction, hypoxic pulmonary,
171,184,185
Vasodilation, effect on cardiovascular
function, 128, 129f
Vasomotion, 95
Vasomotor tone, and sympathetic
nervous system, 132
Vasopressin, see Antidiuretic hormone
Vasovagal reflex, 135
VC, see Vital capacity
Vectors, in electrocardiography, 107,
111-113
Vectorcardiographic loops, 112-113
Vegetative nervous system, see
Autonomic nervous system

Veins, 98-99
renal, 202
Venoconstriction, 99

Venous oxygen content, versus arterial


content, 449Q, 470E
Venous oxygen tension, 184, 190Q,
197E
Venous pressure, see under Blood
pressure
Venous return, 127-131
relation to venous pressure, 127f
Venous valves, 99
Ventilation
alveolar, 164-165, 188Q, 189Q,
194E, 196E, 435Q, 437Q, 450Q,
459E, 461E-462E, 471E
see also Ventilation:perfusion ratio
artificial, 449Q, 470E
distribution in lung, 172-174, 189Q,
196E
minute, 164
Ventilation:perfusion ratio ('VA/O),
174-176,177(, 187Q, 189Q,
190Q, 194E-197E
abnormalities, 439Q, 448Q, 465E,
468E-469E
and hypoxemia, 175, 182-183,
190Q,196E
Ventilatory patterns, 180-181, 190Q,
196E
Ventral respiratory group (VRG), 176,
177-178
Ventricles
in cardiac cycle, 122-123, 141 Q,
143Q, 146E, 148E
changes after birth, 135
conduction pathway, 103, 104
effect of venoconstriction, 99
as pumps, 119, 120(, 141Q, 145E
Ventricular compliance, 442Q, 466E
Ventricular contraction, isovolumic,
120(, 446Q, 468E
Ventricular elastance, 442Q, 466E
Ventricular end-diastolic pressure, 122,
141Q, 142Q, 145E, 147E
Ventricular end-diastolic volume, 105,
119,120('123, 141Q, 145E,
442Q,466E
Ventricular end-systolic volume, 123,
442Q,466E
Ventricular fibrillation, 116
Ventricular filling, 120f, 442Q, 446Q,
466E,468E
Ventricular hypertrophy, 118, 119
Ventricular pressure-volume
relationships, 119, 120f, 143Q,
148E, 442Q, 446Q, 466E, 468E
Ventricular residual volume, 442Q,
466E
Ventricular tachycardia, 115-116,
140Q, 145E
Ventricular volume curve, 447Q, 468E
Venules, 95, 140Q, 144E
Vesicular follicle (secondary, graafian),
in ovarian cycle, 381, 383
Vestibular nuclei, 74
Vestibular receptors, 72-74
Vestibular reflexes, 74-75
Vestibular system, 72-75
fluids of, 47, 73
Vestibulocerebellum, 76-77

Vestibulospinal tracts, 74
Vibration, perception of, 40-41
Villi, intestinal, 315
VIP, see Vasoactive intestinal peptide
Vision, 51-61, 84Q, 85Q, 90E, 91E
neural pathways, 60
photochemistry, 57-58, 86Q, 91E
Visual cortex, 60-61
Visual defects, 453Q, 474E
Visual image, formation of, 51, 52f,
85Q,91E
Visual receptor cells, 56, 86Q, 91 E
Vital capacity (VO, 162-163, 180,
186Q, 190Q, 191Q, 192E, 193E,
196E, 197E, 446Q, 468E
see also Forced expiration
timed, see Forced expiratory volume
in 1 second (FEV,)
Vital capacity maneuver, 162{, 163{,
187Q,193E
Vitamins
fat-soluble
absorption, 323-324, 325, 328Q,
331E
effects of thyroid hormones on, 401
and micelles, 323-324, 325
water-soluble, absorption, 325, 329Q,
332E
Vitamin A
and thyroid hormones, 401
and vision, 58
Vitamin B12
absorption, 325, 432Q, 437Q, 455E,
462E
malabsorption, 453Q, 474E
Vitamin C
absorption, 325, 329Q, 332E
and iron absorption, 325
Vitamin 0, 401, 402, 403, 406
see also Caicitriol;
1,25-Dihydroxyvitamin 0,;
25-Hydroxyvitamin 0,
absorption, 325
activation by kidney, 199
active metabolites, 403
antirachitic action, measurement of,
406
and calcium absorption, 325
nomenclature, 403
and plasma calcium level, 402, 408Q,
415E, 451Q, 472E
and plasma phosphate level, 451 Q,
472E
Vitamin 0, (calciferol, ergocalciferol,
activated ergosterol), 403
versus calcitriol, in effects on bone,
406
Vitamin 0, (cholecalciferol), 403
source, 335
synthesis, 403, 407Q, 414E
versus caicitriol, in effects on bone,
406
Vitamin hormones, 339
Volatile acid, 257, 292Q, 299E
see also Carbonic acid
Volume contraction, 205, 206t, 207
see also Dehydration
Volume distribution, 246Q, 253E

Volume expansion, 207


see also Overhydration
Volume receptors, 254E
see also Baroreceptors
Vomiting center, 310
Vomiting (emesis), 310, 328Q, 331 E
effect on electrolytes, 434Q, 457E

w
Wakefulness, 78, 80
Water
see also Fluid; Body fluid; Body water
in bile, 320, 321
regulation of, 320
effect of estrogen on, 385
gastric transport, 3 15
insensible loss, 199
intake, 199
intestinal absorption, 324
in colon, 327, 433Q, 457E
malabsorption of, 453Q, 474E
intestinal secretion, 322
in pancreatic secretions, 31 7
renal conservation, measurement of,
232{,233
renal excretion, 199
effect of antidiuretic hormone, 235
effect of aldosterone, 236
measurement of, 232-233
renal transport, 21 Ot, 211 {, 223, 226
effect of acetazolamide, 454E-455E
and renal sodium transport, 224
transport across membranes, 226
across cell membrane, 212
in urine concentration and dilution,
227-232, 247Q, 255E
Water diuresis, 233
Water-soluble materials, diffusion of,
2-3
Wenckebach block, 11 6
Withdrawal reflex, 64-65, 84Q, 90E
Wolff-Parkinson-White syndrome, 103,
116
Wolffian (mesonephric) ducts, 370,
371,377, 410Q, 417E
in female, 377
testosterone and, 372, 377
Wound healing, effect of
glucocorticoids, 369

z
Z line of skeletal muscle, 28, 29f
Zero gravity, pulmonary effects, 172
Zero isopotential point, in
vectorcardiography, 112
Zero potential, in electrocardiography,
107, 108f, 109f
Zona fasciculata, of adrenal cortex,
364
Zona glomerulosa, of adrenal cortex,
364
Zona pellucid a, 383
Zona reticularis, of adrenal cortex, 364
Zymogens, pancreatic, 31 7

Вам также может понравиться